Consti Cases Finals Part 3

You might also like

Download as docx, pdf, or txt
Download as docx, pdf, or txt
You are on page 1of 304

EN BANC alleged fund irregularities in the Armed Forces of the

Philippines. 1cräläwvirtualibräry
G.R. No. 136760. July 29, 2003
On the same date, Senator Vicente C. Sotto III also filed Resolution
No. 160, directing the appropriate senate committee to conduct an
THE SENATE BLUE RIBBON COMMITTEE, represented by its
inquiry, in aid of legislation, into the alleged mismanagement of the
Chairman, SENATOR AQUILINO Q. PIMENTEL, JR., petitioner,
funds and investment portfolio of the Armed Forces Retirement and
vs. HON. JOSE B. MAJADUCON, Presiding Judge of Branch 23,
Separation Benefits System (AFP-RSBS) xxx. 2cräläwvirtualibräry
Regional Trial Court of General Santos City, and ATTY. NILO J.
FLAVIANO,respondents.
The Senate President referred the two resolutions to the
Committee on Accountability of Public Officers and Investigations
[G.R. No. 138378. July 29, 2003
(Blue Ribbon Committee) and the Committee on National Defense
and Security.
AQUILINO Q. PIMENTEL, JR., Petitioner, vs. THE HONORABLE JOSE
S. MAJADUCON, in his capacity as Presiding Judge of Branch 23,
During the public hearings conducted by the Senate Blue Ribbon
Regional Trial Court, General Santos City, respondent.
Committee (hereafter called the Committee), it appeared that the
AFP-RSBS purchased a lot in General Santos City, designated as Lot
DECISION X, MR-1160, for P10,500.00 per square meter from private
respondent Atty. Nilo J. Flaviano. However, the deed of sale filed
YNARES-SANTIAGO, J.: with the Register of Deeds indicated that the purchase price of the
lot was only P3,000.00 per square meter. 

For resolution are two consolidated petitions: (a) G.R. No.


136760, for certiorari, prohibition, mandamus and preliminary The Committee thereafter caused the service of a subpoena to
injunction, assailing the resolution dated November 11, 1998 of respondent Atty. Flaviano, directing him to appear and testify
Judge Jose S. Majaducon of the Regional Trial Court of General before it. Respondent refused to appear at the hearing. Instead, he
Santos City, Branch 23, which denied the Senate Blue Ribbon filed a petition for prohibition and preliminary injunction with
Committees motion to dismiss the petition for prohibition, prayer for temporary restraining order with the Regional Trial Court
injunction with writ of preliminary injunction filed by private of General Santos City, Branch 23, which was docketed as SP Civil
respondent Atty. Nilo J. Flaviano; and (b) G.R. No. 138378, for Case No. 496. 
review of the resolution dated April 15, 1999 of respondent Judge
Majaducon declaring petitioner Senator Aquilino Q. Pimentel, Jr. On October 21, 1998, the trial court issued a Temporary
guilty of indirect contempt of court. Restraining Order directing the Committee to CEASE and DESIST
from proceeding with the inquiry in P.S. 160 particularly in General
The antecedent facts are as follows:  Santos City and/or anywhere in Region XI or Manila on matters
affecting the patenting/titling and sale of Lot X, MR-1160-D to AFP-
RSBS, and from issuing subpoenas to witnesses from Region XI,
G.R. No. 136760: particularly from General Santos City, pending the hearing of the
petition for prohibition and injunction. 3cräläwvirtualibräry
On August 28, 1998, Senator Blas F. Ople filed Senate Resolution
No. 157 directing the Committee on National Defense and Security On November 5, 1998, the Committee filed a motion to dismiss the
to conduct an inquiry, in aid of legislation, into the charges of then petition on the grounds of (a) lack of jurisdiction, and (b) failure to
Defense Secretary Orlando Mercado that a group of active and state a valid cause of action. It further argued that the issuance of
retired military officers were organizing a coup detat to prevent the the Temporary Restraining Order was invalid for violating the rule
administration of then President Joseph Estrada from probing against ex-parte issuance thereof; and that the same was not
enforceable beyond the territorial jurisdiction of the trial court. 
On November 11, 1998, the trial court denied petitioners motion to gross ignorance of the rules and procedures when he issued the
dismiss and granted the writ of preliminary injunction, thus: temporary restraining order and the writ of preliminary injunction
because, under the principle of separation of powers, courts cannot
interfere with the exercise by the legislature of its authority to
WHEREFORE, PREMISES CONSIDERED, the motion to dismiss is
conduct investigations in aid of legislation. 6cräläwvirtualibräry
DENIED, and the WRIT OF PRELIMINARY INJUNCTION is hereby
issued against respondent. It is enjoined from enforcing its
subpoenas to petitioner in Region XI to appear and testify before it Reacting to the aforesaid news report, respondent Judge
in any of its inquiry or investigation anywhere in the Philippines Majaducon motu proprio initiated a charge for indirect contempt of
regarding the acquisition by the AFP-RSBS of Lot X, MR-1160-D, court against Senator Aquilino Q. Pimentel, Jr., news reporter
located in General Santos City. The bond of petitioner filed on Perseus Echeminada, Philippine Star publisher Maximo Soliven,
October 21, 1998, for P500,000.00 for the TRO also serves as his editor-in-chief Ramon J. Farolan, and executive editor Bobby G.
bond in this injunction. dela Cruz, which was docketed as Special Civil Case No. 496. Judge
Majaducon averred that the news report created in the minds of the
reader the impression that he violated the separation of powers
SO ORDERED.4cräläwvirtualibräry
clause of the Constitution and that he was guilty of gross ignorance
of the rules and procedures.
Hence, the instant petition for certiorari which was docketed as
G.R. No. 136760, alleging that respondent Judge Majaducon
After the respondents submitted their respective answers, a
committed grave abuse of discretion and/or acted without or in
decision was rendered on April 15, 1999 finding petitioner Pimentel
excess of jurisdiction when he:
guilty of indirect contempt.

I. DENIED PETITIONERS MOTION TO DISMISS THE PETITION


Hence, the instant petition based on the following grounds:
FOR PROHIBITION AND PRELIMINARY INJUNCTION
FILED BY PRIVATE RESPONDENT, ATTY. NILO J.
FLAVIANO, AGAINST THE PETITIONER IN SP. CIVIL I. THE EXPRESSION GROSS IGNORANCE OF THE RULES OF
CASE NO. 496. PROCEDURE OR GROSS IGNORANCE OF THE LAW IN
REFERENCE TO THE RESPONDENTS EX-PARTE
ISSUANCE OF INJUNCTIVE RELIEF IS NOT PEJORATIVE
II. ISSUED (1) A TEMPORARY RESTRAINING ORDER EX-PARTE
AS TO CONSTITUTE A GROUND FOR INDIRECT
FOR A PERIOD OF TWENTY (20) DAYS AGAINST THE
CONTEMPT.
PETITIONER ON OCTOBER 21, 1998, AND (2) A WRIT
OF PRELIMINARY INJUNCTION ON NOVEMBER 11, 1998
ENJOINING THE PETITIONER FROM ENFORCING ITS II. THIS HONORABLE COURT ITSELF USES GROSS IGNORANCE
SUBPOENAS TO PRIVATE RESPONENT IN REGION XI. OF THE LAW AND OTHER EXPRESSIONS OF SIMILAR
FORCEFUL IMPORT IN DESCRIBING GROSS AND
PALPABLE ERRORS OF JUDGES.
III. APPLIED THE RULING OF BENGZON VS. SENATE BLUE
RIBBON IN GRANTING INJUNCTIVE RELIEF TO
PRIVATE RESPONDENT.5cräläwvirtualibräry III. BY UPHOLDING HIS CONTEMPT CHARGE AGAINST THE
PETITIONER, THE RESPONDENT JUDGE HAS, IN EFFECT,
PREEMPTED THIS HONORABLE COURT IN RESOLVING
G.R. No. 138378:
THE ISSUES RAISED AGAINST HIM IN G.R. NO. 136760.

On January 13, 1999, the newspaper, The Philippine Star published


IV. THE PUBLICATION BY PHILIPPINE STAR OF THE BLUE
a news report on the filing by the Committee with this Court of the
RIBBON PETITION IN G.R. NO. 136760, OR EXCERPTS
petition for certiorariwhich was docketed as G.R. No. 136760. The
THEREOF WAS A LEGITIMATE EXERCISE OF FREEDOM
news report quoted portions of the petition filed by the Committee,
OF EXPRESSION AND OF THE PRESS.
alleging that Regional Trial Court Judge Majaducon was guilty of
The two petitions, namely, G.R. No. 136760 and G.R. No. 138378, The principle of separation of powers essentially means that
were ordered consolidated on December 11, 2000. legislation belongs to Congress, execution to the Executive, and
settlement of legal controversies to the Judiciary. Each is prevented
from invading the domain of the others. 11 When the Senate Blue
The issues for resolution in these joint petitions are: (a) whether or
Ribbon Committee served subpoena on respondent Flaviano to
not respondent Judge Jose Majaducon committed grave abuse of
appear and testify before it in connection with its investigation of
discretion when he dismissed petitioners motion to dismiss the
the alleged misuse and mismanagement of the AFP-RSBS funds, it
petition for prohibition and issued the writ of preliminary
did so pursuant to its authority to conduct inquiries in aid of
injunction; and (b) whether or not respondent Judge erred in
legislation. This is clearly provided in Article VI, Section 21 of the
convicting petitioner Pimentel of indirect contempt of court.
Constitution, thus:

On the first issue, petitioner Committee contends that courts have


The Senate or the House of Representatives or any of its respective
no jurisdiction to restrain Congress from performing its
committees may conduct inquiries in aid of legislation in
constitutionally vested function to conduct investigations in aid of
accordance with its duly published rules of procedure. The rights of
legislation, following the principle of separation of powers.
persons appearing in or affected by such inquiries shall be
Moreover, the petition filed by respondent Flaviano before the trial
respected.
court failed to state a cause of action considering that the
legislative inquiry did not deal with the issuance of the patent and
title to Lot X, MR-1160-D in the name of AFP-RSBS, which is well Hence, the Regional Trial Court of General Santos City, or any court
within the courts jurisdiction, but with the anomaly in the purchase for that matter, had no authority to prohibit the Committee from
thereof, which falls squarely within the ambit of Senate Resolutions requiring respondent to appear and testify before it.
Nos. 1577 and 160.8cräläwvirtualibräry
The ruling in Bengzon, cited by respondent, does not apply in this
On the other hand, respondent Flaviano contends that the trial case. We agree with petitioner Committee that the factual
court may properly intervene into investigations by Congress circumstances therein are different from those in the case at bar.
pursuant to the power of judicial review vested in it by the In Bengzon, no intended legislation was involved and the subject
Constitution. He avers that he has a valid cause of action to file the matter of the inquiry was more within the province of the courts
petition for prohibition considering that the Committees rather than of the legislature. More specifically, the investigation in
investigation will delve into the validity of the patenting and titling the said case was an offshoot of the privilege speech of then
of Lot X, MR-1160-D which, as admitted by petitioner, falls within Senator Enrile, who urged the Senate to look into a possible
the competence of judicial courts. In fact, the validity of the violation of the Anti-Graft and Corrupt Practices Act by the relatives
purchase by AFP-RSBS of the subject lot is already the subject of a of then President Corazon Aquino, particularly Mr. Ricardo Lopa, in
pending action before the Regional Trial Court of General Santos connection with the alleged sale of 36 to 39 corporations belonging
City and the Ombudsman of Mindanao. Finally, he cites the case to Benjamin Romualdez. On the other hand, there was in this case a
of Bengzon v. Senate Blue Ribbon Committee,9 and argues that clear legislative purpose, as stated in Senate Resolution No. 160,
preliminary injunction may issue in cases pending before and the appropriate Senate Committee was directed to look into
administrative bodies such as the Ombudsman or the Office of the the reported misuse and mismanagement of the AFP-RSBS funds,
Prosecutor as long as the right to self-incrimination guaranteed by with the intention of enacting appropriate legislation to protect the
the Bill of Rights is in danger. Furthermore, an information against rights and interests of the officers and members of the Armed
him has been filed with the Sandiganbayan. Forces of the Philippines. Further, in Bengzon, the validity of the
sale of Romualdezs corporations was pending with the
Sandiganbayan when the Senate Blue Ribbon Committee decided to
We find for petitioner. There is grave abuse of discretion when the
conduct its investigation. In short, the issue had already been pre-
respondent acts in a capricious, whimsical, arbitrary or despotic
empted by the court.
manner in the exercise of his judgment, as when the assailed order
is bereft of any factual and legal justification. 10 In this case, the
assailed resolution of respondent Judge Majaducon was issued In the instant case, the complaint against respondent Flaviano
without legal basis. regarding the anomaly in the sale of Lot X, MR-1160 was still
pending before the Office of the Ombudsman when the Committee Rule 71, Section 3(d) of the 1997 Rules of Civil Procedure provides:
served subpoena on him. In other words, no court had acquired
jurisdiction over the matter. Thus, there was as yet no
Section 3. Indirect contempt to be punished after charge and
encroachment by the legislature into the exclusive jurisdiction of
hearing. After a charge in writing has been filed, and an
another branch of the government. Clearly, there was no basis for
opportunity given to the respondent to comment thereon within
the respondent Judge to apply the ruling in Bengzon. Hence, the
such period as may be fixed by the court and to be heard by himself
denial of petitioners motion to dismiss the petition for prohibition
or counsel, a person guilty of any of the following acts may be
amounted to grave abuse of discretion.
punished for indirect contempt:

In G.R. No. 138378, Petitioner, Senator Aquilino Pimentel, Jr.,


x x x 
contends that respondent judge erred in finding him, as
representative of the Committee, guilty of indirect contempt of
court under Rule 71, Section 3(d) of the 1997 Rules of Civil d) Any improper conduct tending, directly or indirectly, to impede,
Procedure. According to Pimentel, the phrase gross ignorance of obstruct, or degrade the administration of justice; x x x.
the rules of law and procedure, which the Committee used in the
petition, is not depreciatory, but merely a description of normal After deliberating on the parties arguments, we find that petitioner
usage in petitions where the acts of lower courts are challenged Pimentel is not guilty of improper conduct which obstructs or
before higher judicial bodies. In fact, this Court often uses the degrades the administration of justice.
phrase in its decisions to describe judges who commit gross and
palpable mistakes in their interpretation and application of the law.
Petitioner further maintains that when the Committee used the Verily, it does not appear that Pimentel caused the publication in
phrase, it did so without malice. Rather, it was only to stress the the Philippine Star of the fact of filing of the petition for certiorari
unfamiliarity of or disregard by the respondent Judge of a basic by the Committee and the reproduction of excerpts thereof. He had
rule of procedure, and to buttress its arguments in support of its no right to choose which news articles will see print in the
petition for certiorari. newspaper. Rather, it is the publisher thereof which decides which
news events will be reported in the broadsheet. In doing so, it is
allowed the widest latitude of choice as to what items should see
Petitioner Pimentel also contends that he had no participation in the light of day so long as they are relevant to a matter of public
the publication in the Philippine Star of excerpts from the interest, pursuant to its right of press freedom. 12cräläwvirtualibräry
Committees petition for certiorari. Even assuming arguendo that it
was within his control, he pointed out that he could not have
prevented the editors and writers of the newspaper from Respondent Judges allegation that petitioner made it appear that
publishing the same, lest he violate their constitutional right of free an administrative complaint was filed against him is without basis.
expression. Indeed, the report by the Philippine Star of the filing of From a careful perusal of the records, it appears that while the
the petition and the reproduction of its contents was a legitimate Committee prayed for the imposition of administrative sanctions
exercise of press freedom. against respondent Judge Majaducon for gross ignorance of the
law, no formal administrative complaint was instituted separately
from the petition for certiorari.
Respondent Judge counters that Pimentel was guilty of indirect
contempt of court, first,  for causing the publication of the
Committees petition in the Philippine Star notwithstanding that the Finally, the statement that respondent Judge was grossly ignorant
same was sub judice; second, for making derogatory remarks in of the rules of law and procedure does not constitute improper
the petition itself which affected the honor and integrity of the conduct that tends to impede, obstruct or degrade the
respondent judge and degraded the administration of justice; administration of justice. As correctly argued by petitioner, the
and third, for making it appear that an administrative complaint phrase gross ignorance of the rules of law and procedure is
was filed against respondent Judge for gross ignorance of the law. ordinarily found in administrative complaints and is a necessary
These, he said, constituted malicious and false report which description to support a petition which seeks the annulment of an
obstructed the administration of justice. order of a judge wherein basic legal principles are disregarded.
In Spouses Bacar v. Judge De Guzman, Jr.,13 it was held that when
the law is so elementary, not to know it or to act as if a judge does
not know it, constitutes gross ignorance of the law. In this case,
there was no showing that petitioner Pimentel, as representative of
the Committee, used the phrase to malign the trial court. Rather, it
was used to express what he believed as a violation of the basic
principle of separation of powers.

In this connection, it bears stressing that the power to declare a


person in contempt of court must be exercised on the preservative,
not vindictive principle, and on the corrective and not retaliatory
idea of punishment.14 This was aptly expressed in the case
of Nazareno v. Barnes:15cräläwvirtualibräry

A judge, as a public servant, should not be so thin-skinned or


sensitive as to feel hurt or offended if a citizen expresses an honest
opinion about him which may not altogether be flattering to him.
After all, what matters is that a judge performs his duties in
accordance with the dictates of his conscience and the light that
God has given him. A judge should never allow himself to be moved
by pride, prejudice, passion, or pettiness in the performance of his
duties. He should always bear in mind that the power of the court
to punish for contempt should be exercised for purposes that are
impersonal, because that power is intended as a safeguard not for
the judges as persons but for the functions that they exercise.

WHEREFORE, in view of the foregoing, the petitions docketed as


G.R. Nos. 136760 and 138378 are GRANTED. The resolution of the
Regional Trial Court of General Santos City, Branch 23, in Special
Civil Case No. 496 dated November 11, 1998, which denied the
Senate Blue Ribbon Committees motion to dismiss, is REVERSED
and SET ASIDE. The Writ of Preliminary Injunction issued by the
trial court on November 11, 1998 is DISSOLVED. The resolution
dated April 15, 1999, which declared Senator Aquilino Q. Pimentel,
Jr. guilty of indirect contempt of court, is REVERSED and SET
ASIDE. The petition for indirect contempt is ordered DISMISSED.

SO ORDERED.
[G.R. NO. 167173 : December 27, 2007] petitioners to appear and testify in the inquiry being conducted pursuant to
P.S. Resolution No. 166.
STANDARD CHARTERED BANK (Philippine Branch), PAUL SIMON
MORRIS, SUNDARA RAMESH, OWEN BELMAN, SANJAY AGGARWAL, The facts are as follows:
RAJAMANI CHANDRASHEKAR, MARIVEL GONZALES, MA. ELLEN
VICTOR, CHONA G. REYES, ZENAIDA IGLESIAS, RAMONA BERNAD,
On February 1, 2005, Senator Juan Ponce Enrile, Vice Chairperson of
MICHAELANGELO AGUILAR, and FERNAND
respondent, delivered a privilege speech entitled "Arrogance of
TANSINGCO, Petitioners, v. SENATE COMMITTEE ON BANKS,
Wealth"1 before the Senate based on a letter from Atty. Mark R. Bocobo
FINANCIAL INSTITUTIONS AND CURRENCIES, as represented by its
denouncing SCB-Philippines for selling unregistered foreign securities in
Chairperson, HON. EDGARDO J. ANGARA, Respondent.
violation of the Securities Regulation Code (R.A. No. 8799) and urging the
Senate to immediately conduct an inquiry, in aid of legislation, to prevent
DECISION the occurrence of a similar fraudulent activity in the future. Upon motion of
Senator Francis Pangilinan, the speech was referred to respondent. Prior to
the privilege speech, Senator Enrile had introduced P.S. Resolution No.
NACHURA, J.:
166,2 to wit:

Before us is a Petition for Prohibition (With Prayer for Issuance of


RESOLUTION
Temporary Restraining Order and/or Injunction) dated and filed on March
11, 2005 by petitioners against respondent Senate Committee on Banks,
Financial Institutions and Currencies, as represented by its Chairperson DIRECTING THE COMMITTEE ON BANKS, FINANCIAL INSTITUTIONS
Edgardo J. Angara (respondent). AND CURRENCIES, TO CONDUCT AN INQUIRY, IN AID OF
LEGISLATION, INTO THE ILLEGAL SALE OF UNREGISTERED AND
HIGH-RISK SECURITIES BY STANDARD CHARTERED BANK, WHICH
Petitioner Standard Chartered Bank (SCB)-Philippines is an institution
RESULTED IN BILLIONS OF PESOS OF LOSSES TO THE INVESTING
incorporated in England with limited liability and is licensed to engage in
PUBLIC
banking, trust, and other related operations in the Philippines. Petitioners
Paul Simon Morris, Sundara Ramesh, Owen Belman, Sanjay Aggarwal,
Rajamani Chandrashekar, Marivel Gonzales, Ma. Ellen Victor, Chona G. WHEREAS, Republic Act No. 7721, otherwise known as the "Law
Reyes, Zenaida Iglesias, Ramona Bernad, Michaelangelo Aguilar, and Liberalizing the Entry and Scope of Operations of Foreign Banks in the
Fernand Tansingco are the Chief Executive Officer, Chief Operations Officer, Philippines," was approved on May 18, 1994 to promote greater
Country Head of Consumer Banking, General Manager for Credit Card and participation of foreign banks in the Philippine Banking Industry that will
Personal Loans, Chief Financial Officer, Legal and Compliance Officer, stimulate economic growth and serve as a channel for the flow of funds
former Trust and Investment Services Head, Country Tax Officer, Head of into the economy;
Corporate Affairs, Head of Banking Services, Head of Client Relationships,
and the Head of Global Markets of SCB-Philippines, respectively.
WHEREAS, to promote greater competition in the Philippine Banking
Respondent, on the other hand, is one of the permanent committees of the
Industry, foreign banks were accorded the same privileges, allowed to
Senate of the Philippines.
perform the same functions and subjected to the same limitations under
relevant banking laws imposed upon domestic banks;
The petition seeks the issuance of a temporary restraining order (TRO) to
enjoin respondent from (1) proceeding with its inquiry pursuant to
WHEREAS, Standard Chartered Bank was among the foreign banks granted
Philippine Senate (P.S.) Resolution No. 166; (2) compelling petitioners who
the privilege to do business in our country under Republic Act No. 7721;
are officers of petitioner SCB-Philippines to attend and testify before any
further hearing to be conducted by respondent, particularly that set on
March 15, 2005; and (3) enforcing any hold-departure order (HDO) and/or WHEREAS, there are complaints against Standard Chartered Bank whose
putting the petitioners on the Watch List. It also prays that judgment be actions have reportedly defrauded hundreds of Filipino investors of billions
rendered (1) annulling the subpoenae ad testificandum and duces tecum of pesos through the sale of unregistered securities in the form of high-risk
issued to petitioners, and (2) prohibiting the respondent from compelling mutual funds falsely advertised and marketed as safe investment havens;
WHEREAS, there are reports that Standard Chartered Bank clearly knew WHEREAS, existing laws including the Securities Regulation Code seem to
that its actions were violative of Philippine banking and securities laws but be inadequate in preventing the sale of unregistered securities and in
cleverly disguised its illegal acts through the use of pro-forma agreements effectively enforcing the registration rules intended to protect the investing
containing waivers of liability in favor of the bank; public from fraudulent practices;

WHEREAS, there are reports that in the early stages of conducting these WHEREAS, the regulatory intervention by the SEC and BSP likewise
questionable activities, the Bangko Sentral ng Pilipinas warned and appears inadequate in preventing the conduct of proscribed activities in a
eventually fined Standard Chartered Bank a measly P30,000 for violating manner that would protect the investing public;
Philippine banking laws;
WHEREAS, there is a need for remedial legislation to address the situation,
WHEREAS, the particular operations of Standard Chartered Bank may having in mind the imposition of proportionate penalties to offending
constitute "conducting business in an unsafe and unsound manner," entities and their directors, officers and representatives among other
punishable under Section 37 of Republic Act No. 7653 and should have additional regulatory measures;
drawn the higher penalty of revocation of its quasi-banking license;
Now, therefore, BE IT RESOLVED, AS IT IS HEREBY RESOLVED, to direct
WHEREAS, Republic Act No. 8791 or the "General Banking Act of 2000" the Committee on Banks, Currencies, and Financial Institutions, to conduct
deems a particular act or omission as conducting business in an unsafe and an inquiry, in aid of legislation, into the reported sale of unregistered and
unsound manner as follows: high-risk securities by Standard Chartered Bank which resulted in billions
of losses to the investing public.
"Section 56.2 The act or omission has resulted or may result in material
loss or damage or abnormal risk to the institution's depositors, creditors, Acting on the referral, respondent, through its Chairperson, Senator
investors, stockholders or to the Bangko Sentral or to the public in Edgardo J. Angara, set the initial hearing on February 28, 2005 to
general." investigate, in aid of legislation, the subject matter of the speech and
resolution filed by Senator Enrile.
WHEREAS, the sale of unregistered securities is also a clear violation of
Republic Act No. 8799 or "The Securities Regulation Code of 2000" which Respondent invited petitioners, among others, to attend the hearing,
states: requesting them to submit their written position paper. Petitioners, through
counsel, submitted to respondent a letter3 dated February 24, 2005
presenting their position, particularly stressing that there were cases
"Section 8.1 Securities shall not be sold or offered for sale or distribution
pending in court allegedly involving the same issues subject of the
within the Philippines, without a registration statement duly filed with and
legislative inquiry, thereby posing a challenge to the jurisdiction of
approved by the Commission. Prior to such sale, information on the
respondent to continue with the inquiry.
securities, in such form and with such substance as the Commission may
prescribe, shall be made available to each prospective purchaser."
On February 28, 2005, respondent commenced the investigation. Senator
Enrile inquired who among those invited as resource persons were present
WHEREAS, the Securities and Exchange Commission (SEC) reportedly
and who were absent. Thereafter, Senator Enrile moved that subpoenae be
issued a Cease-and-Desist Order (CDO) against Standard Chartered Bank
issued to those who did not attend the hearing and that the Senate request
for the sale of these unregistered securities but the case was reportedly
the Department of Justice, through the Bureau of Immigration and
settled administratively and dismissed after Standard Chartered Bank paid
Deportation, to issue an HDO against them and/or include them in the
a fine of P7 Million;
Bureau's Watch List. Senator Juan Flavier seconded the motion and the
motion was approved.
WHEREAS, the SEC reportedly made an official finding that Standard
Chartered Bank actively engaged in promoting and marketing the so-called
Respondent then proceeded with the investigation proper. Towards the end
"Global Third Party Mutual Funds" to the investing public and even set
of the hearing, petitioners, through counsel, made an Opening
revenue quotas for the sale of these funds;
Statement4 that brought to the attention of respondent the lack of proper VIOLATION OF PETITIONERS' RIGHT AGAINST SELF-INCRIMINATION AND
authorization from affected clients for the bank to make disclosures of their RIGHT TO PURSUE AND DEFEND THEIR CAUSE IN COURT RATHER THAN
accounts and the lack of copies of the accusing documents mentioned in ENGAGE IN TRIAL BY PUBLICITY - A CLEAR VIOLATION OF DUE PROCESS,
Senator Enrile's privilege speech, and reiterated that there were pending RIGHT TO PRIVACY AND TO TRAVEL.
court cases regarding the alleged sale in the Philippines by SCB-Philippines
of unregistered foreign securities.
IV.

The February 28, 2005 hearing was adjourned without the setting of the
THE COMMITTEE ACTED IN GRAVE ABUSE OF DISCRETION AMOUNTING
next hearing date. However, petitioners were later served by respondent
TO LACK OF JURISDICTION BY DISREGARDING ITS OWN RULES.5
with subpoenae ad testificandum and duces tecum to compel them to
attend and testify at the hearing set on March 15, 2005. Hence, this
petition. Petitioners argue that respondent has no jurisdiction to conduct the inquiry
because its subject matter is the very same subject matter of the following
cases, to wit:
The grounds relied upon by petitioners are as follows:

(a) CA-G.R. SP No. 85078, entitled "Manuel V. Baviera v. Hon. Esperanza


I.
P. Rosario, et al., pending before the 9th Division of the Court of Appeals.
In the petition, Mr. Baviera seeks to annul and set aside the dismissal by
THE COMMITTEE ACTED WITHOUT JURISDICTION AND/OR ACTED WITH the Department of Justice of his complaint against Standard Chartered
GRAVE ABUSE OF DISCRETION AMOUNTING TO LACK OF JURISDICTION IN Bank and its officers accusing them of SELLING UNREGISTERED
CONDUCTING AN INVESTIGATION, PURPORTEDLY IN AID OF FOREIGN SECURITIES IN VIOLATION OF P.D. NO. 1869
LEGISLATION, BUT IN REALITY PROBING INTO THE ISSUE OF WHETHER (SYNDICATED ESTAFA) AND ARTICLE 315 OF THE REVISED PENAL
THE STANDARD CHARTERED BANK HAD SOLD UNREGISTERED FOREIGN CODE.
SECURITIES IN THE PHILIPPINES. SAID ISSUE HAS LONG BEEN THE
SUBJECT OF CRIMINAL AND CIVIL ACTIONS NOW PENDING BEFORE THE
(b) CA-G.R. SP No. 86200, entitled "Manuel V. Baviera v. Hon. Rafael
COURT OF APPEALS, REGIONAL TRIAL COURT OF PASIG CITY,
Buenaventura, et al.", pending before the 15th Division of the Court of
METROPOLITAN TRIAL COURT OF MAKATI CITY AND THE PROSECUTOR'S
Appeals. In the petition, Mr. Baviera seeks to annul and set aside the
OFFICE OF MAKATI CITY.
termination for lack of probable cause by the Anti-Money Laundering
Council ("AMLC") of the investigation of Standard Chartered Bank for
II. money laundering activities BY SELLING UNREGISTERED FOREIGN
SECURITIES.
THE COMMITTEE ACTED IN GRAVE ABUSE OF DISCRETION AMOUNTING
TO LACK OF JURISDICTION BY CONDUCTING AN INVESTIGATION, (c) CA-G.R. SP No. 87328, entitled "Manuel V. Baviera v. Hon. Esperanza
PURPORTEDLY "IN AID OF LEGISLATION," BUT IN REALITY IN "AID OF Paglinawan Rozario, et al.," pending before the 16th Division of the Court
COLLECTION" BY A HANDFUL OF TWO (2) CLIENTS OF STANDARD of Appeals. The petition seeks to annul and set aside the dismissal by the
CHARTERED BANK OF LOSSES WHICH WERE FOR THEIR ACCOUNT AND Department of Justice of Mr. Baviera's complaint accusing SCB and its
RISK. AT ANY RATE, SUCH COLLECTION IS WITHIN THE PROVINCE OF THE officers of violation of the Securities Regulation Code by SELLING
COURT RATHER THAN OF THE LEGISLATURE. UNREGISTERED FOREIGN SECURITIES.

III. (d) Civil Case No. 70173, entitled "Mr. Noel G. Sanchez, et al. v. Standard
Chartered Bank," pending before Branch 155 of the Regional Trial Court of
Pasig City. Plaintiff seeks damages and recovery of their investment
THE COMMITTEE ACTED WITHOUT JURISDICTION AND/OR ACTED WITH
accusing the bank of SELLING UNREGISTERED FOREIGN SECURITIES.
GRAVE ABUSE OF DISCRETION AMOUNTING TO LACK OF JURISDICTION IN
COMPELLING PETITIONERS, SOME OF WHOM ARE RESPONDENTS IN THE
PENDING CRIMINAL AND CIVIL ACTIONS BROUGHT BY SAID CLIENTS, IN
(e) Criminal Case No. 332034, entitled "People of the Philippines v. Manuel Central to the Court's ruling in Bengzon - - that the Senate Blue Ribbon
V. Baviera,"pending before Branch 64 of the Metropolitan Trial Court of Committee was without any constitutional mooring to conduct the
Makati City. Petitioner Morris is the private complainant in this information legislative investigation - - was the Court's determination that the intended
for extortion or blackmail against Mr. Baviera for demanding the payment inquiry was not in aid of legislation. The Court found that the speech of
of US$2 Million with the threat to EXPOSE THE BANK'S "LARGE SCALE Senator Enrile, which sought such investigation contained no suggestion of
SCAM" CONSISTING [OF] ILLEGAL SELLING OF UNREGISTERED any contemplated legislation; it merely called upon the Senate to look into
FOREIGN SECURITIES BY THE BANK, before various government possible violations of Section 5, Republic Act No. 3019. Thus, the Court
offices, such as the Department of Justice, the BIR, Bangko Sentral ng held that the requested probe failed to comply with a fundamental
Pilipinas, Regional Trial Courts, and both houses of Congress. requirement of Section 21, Article VI of the Constitution, which states:

(f) Criminal Case No. 331395, entitled "People of the Philippines v. Manuel The Senate or the House of Representatives or any of its respective
V. Baviera,"pending before Branch 64 of the Metropolitan Trial Court of committees may conduct inquiries in aid of legislation in accordance with
Makati City. Petitioners Victor and Chona Reyes are the private its duly published rules of procedure. The rights of persons appearing in or
complainants in this information for perjury committed by Mr. Baviera in affected by such inquiries shall be respected.
securing a hold departure order against the petitioners herein from the
Department of Justice for their alleged involvement in syndicated estafa
Accordingly, we stopped the Senate Blue Ribbon Committee from
and swindling BY SELLING UNREGISTERED FOREIGN SECURITIES.
proceeding with the legislative investigation in that case.

(g) I.S. No. 2004-B-2279-80, entitled "Aurelio Litonjua III and Aurelio
Unfortunately for the petitioners, this distinguishing factual milieu in
Litonjua, Jr. v. Antonette de los Reyes, et al.," pending before the Office of
Bengzon does not obtain in the instant case. P.S. Resolution No. 166 is
the Prosecutor, Makati City. This is a criminal complaint accusing SCB and
explicit on the subject and nature of the inquiry to be (and already being)
its officers of estafa for SELLING UNREGISTERED FOREIGN SECURITIES.6
conducted by the respondent Committee, as found in the last three
Whereas clauses thereof, viz.:
Citing Bengzon, Jr. v. Senate Blue Ribbon Committee,7 the petitioners claim
that since the issue of whether or not SCB-Philippines illegally sold
WHEREAS, existing laws including the Securities Regulation Code seem to
unregistered foreign securities is already preempted by the courts that took
be inadequate in preventing the sale of unregistered securities and in
cognizance of the foregoing cases, the respondent, by this investigation,
effectively enforcing the registration rules intended to protect the investing
would encroach upon the judicial powers vested solely in these courts.
public from fraudulent practices;

The argument is misplaced. Bengzon does not apply squarely to petitioners'


WHEREAS, the regulatory intervention by the SEC and BSP likewise
case.
appears inadequate in preventing the conduct of proscribed activities in a
manner that would protect the investing public;
It is true that in Bengzon, the Court declared that the issue to be
investigated was one over which jurisdiction had already been acquired by
WHEREAS, there is a need for remedial legislation to address the situation,
the Sandiganbayan, and to allow the [Senate Blue Ribbon] Committee to
having in mind the imposition of proportionate penalties to offending
investigate the matter would create the possibility of conflicting judgments;
entities and their directors, officers and representatives among other
and that the inquiry into the same justiciable controversy would be an
additional regulatory measures; (emphasis supplied)
encroachment on the exclusive domain of judicial jurisdiction that had set
in much earlier.
The unmistakable objective of the investigation, as set forth in the said
resolution, exposes the error in petitioners' allegation that the inquiry, as
To the extent that, in the case at bench, there are a number of cases
initiated in a privilege speech by the very same Senator Enrile, was simply
already pending in various courts and administrative bodies involving the
"to denounce the illegal practice committed by a foreign bank in selling
petitioners, relative to the alleged sale of unregistered foreign securities,
unregistered foreign securities x x x." This fallacy is made more glaring
there is a resemblance between this case and Bengzon. However, the
when we consider that, at the conclusion of his privilege speech, Senator
similarity ends there.
Enrile urged the Senate "to immediately conduct an inquiry, in aid of illegal activities of SCB-Philippines, with the end view of preventing the
legislation, so as to prevent the occurrence of a similar fraudulent activity future occurrence of any similar fraudulent activity by the banks in
in the future." general.9 Baviera, on the other hand, was not a "complainant" but merely a
witness in the investigation, invited to testify on the alleged illegal sale of
unregistered foreign securities by SCB-Philippines, being one of the
Indeed, the mere filing of a criminal or an administrative complaint before
supposed victims thereof.
a court or a quasi-judicial body should not automatically bar the conduct of
legislative investigation. Otherwise, it would be extremely easy to subvert
any intended inquiry by Congress through the convenient ploy of instituting The Court further notes that when it denied petitioners' prayer for the
a criminal or an administrative complaint. Surely, the exercise of sovereign issuance of a TRO to restrain the hearing set on March 15,
legislative authority, of which the power of legislative inquiry is an essential 2005,10 respondent proceeded with the investigation. On the said date,
component, cannot be made subordinate to a criminal or an administrative outraged by petitioners' imputation that it was conducting the investigation
investigation. "in aid of collection," respondent held petitioners, together with their
counsel, Atty. Reynaldo Geronimo, in contempt and ordered their detention
for six hours.
As succinctly stated in the landmark case Arnault v. Nazareno8' 

Petitioners filed a Motion for Partial Reconsideration of this Court's


[T]he power of inquiry - with process to enforce it - is an essential and
Resolution dated March 14, 2005 only with respect to the denial of the
appropriate auxiliary to the legislative function. A legislative body cannot
prayer for the issuance of a TRO and/or writ of preliminary injunction,
legislate wisely or effectively in the absence of information respecting the
alleging that their being held in contempt was without legal basis, as the
conditions which the legislation is intended to affect or change; and where
phrase "in aid of collection" partakes of an absolutely privileged allegation
the legislative body does not itself possess the requisite information -
in the petition.
which is not infrequently true - recourse must be had to others who
possess it.
We do not agree. The Court has already expounded on the essence of the
contempt power of Congress and its committees in this wise' 
Neither can the petitioners claim that they were singled out by the
respondent Committee. The Court notes that among those invited as
resource persons were officials of the Securities and Exchange Commission The principle that Congress or any of its bodies has the power to punish
(SEC) and the Bangko Sentral ng Pilipinas (BSP). These officials were recalcitrant witnesses is founded upon reason and policy. Said power must
subjected to the same critical scrutiny by the respondent relative to their be considered implied or incidental to the exercise of legislative power.
separate findings on the illegal sale of unregistered foreign securities by How could a legislative body obtain the knowledge and information on
SCB-Philippines. It is obvious that the objective of the investigation was which to base intended legislation if it cannot require and compel the
the quest for remedies, in terms of legislation, to prevent the recurrence of disclosure of such knowledge and information, if it is impotent to punish a
the allegedly fraudulent activity. defiance of its power and authority? When the framers of the Constitution
adopted the principle of separation of powers, making each branch
supreme within the realm of its respective authority, it must have intended
Still, petitioners insist that the inquiry conducted by respondent was, in
each department's authority to be full and complete, independently of each
fact, "in aid of collection." They claim that Atty. Bocobo and Manuel
other's authority or power. And how could the authority and power become
Baviera, the latter a party to the pending court cases cited by petitioners,
complete if for every act of refusal, every act of defiance, every act of
were only seeking a friendly forum so that they could recover their
contumacy against it, the legislative body must resort to the judicial
investments from SCB-Philippines; and that the respondent has allowed
department for the appropriate remedy, because it is impotent by itself to
itself to be used as the conveniently available vehicle to effect this purpose.
punish or deal therewith, with affronts committed against its authority or
dignity.11
However, as correctly pointed out by respondent in its Comment on the
petition, Atty. Bocobo did not file a complaint before the Senate for the
The exercise by Congress or by any of its committees of the power to
purpose of recovering his investment. On the contrary, and as confirmed
punish contempt is based on the principle of self-preservation. As the
during the initial hearing on February 28, 2005, his letter-complaint
branch of the government vested with the legislative power, independently
humbly requested the Senate to conduct an inquiry into the purportedly
of the judicial branch, it can assert its authority and punish contumacious They were summoned by respondent merely as resource persons, or as
acts against it. Such power is sui generis, as it attaches not to the witnesses, in a legislative inquiry. As distinguished by this Court' 
discharge of legislative functions per se, but to the sovereign character of
the legislature as one of the three independent and coordinate branches of
[An] accused occupies a different tier of protection from an ordinary
government.12
witness. Whereas an ordinary witness may be compelled to take the
witness stand and claim the privilege as each question requiring an
In this case, petitioners' imputation that the investigation was "in aid of incriminating answer is shot at him, an accused may altogether refuse to
collection" is a direct challenge against the authority of the Senate take the witness stand and refuse to answer any and all questions.17
Committee, as it ascribes ill motive to the latter. In this light, we find the
contempt citation against the petitioners reasonable and justified.
Concededly, this right of the accused against self-incrimination is extended
to respondents in administrative investigations that partake of the nature
Furthermore, it is axiomatic that the power of legislative investigation of or are analogous to criminal proceedings. The privilege has consistently
includes the power to compel the attendance of witnesses. Corollary to the been held to extend to all proceedings sanctioned by law; and to all cases
power to compel the attendance of witnesses is the power to ensure that in which punishment is sought to be visited upon a witness, whether a
said witnesses would be available to testify in the legislative investigation. party or not.18
In the case at bench, considering that most of the officers of SCB-
Philippines are not Filipino nationals who may easily evade the compulsive
However, in this case, petitioners neither stand as accused in a criminal
character of respondent's summons by leaving the country, it was
case nor will they be subjected by the respondent to any penalty by reason
reasonable for the respondent to request the assistance of the Bureau of
of their testimonies. Hence, they cannot altogether decline appearing
Immigration and Deportation to prevent said witnesses from evading the
before respondent, although they may invoke the privilege when a question
inquiry and defeating its purpose. In any event, no HDO was issued by a
calling for an incriminating answer is propounded.19
court. The BID instead included them only in the Watch List, which had the
effect of merely delaying petitioners' intended travel abroad for five (5)
days, provided no HDO is issued against them.13 Petitioners' argument, that the investigation before respondent may result
in a recommendation for their prosecution by the appropriate government
agencies, such as the Department of Justice or the Office of the
With respect to the right of privacy which petitioners claim respondent has
Ombudsman, does not persuade.
violated, suffice it to state that privacy is not an absolute right. While it is
true that Section 21, Article VI of the Constitution, guarantees respect for
the rights of persons affected by the legislative investigation, not every As held in Sinclair v. United States20 - - 
invocation of the right to privacy should be allowed to thwart a legitimate
congressional inquiry. In Sabio v. Gordon,14 we have held that the right of It may be conceded that Congress is without authority to compel
the people to access information on matters of public concern generally disclosures for the purpose of aiding the prosecution of pending suits; but
prevails over the right to privacy of ordinary financial transactions. In that the authority of that body, directly or through its Committees, to require
case, we declared that the right to privacy is not absolute where there is an pertinent disclosures in aid of its own constitutional power is not abridged
overriding compelling state interest. Employing the rational basis because the information sought to be elicited may also be of use in such
relationship test, as laid down in Morfe v. Mutuc,15 there is no infringement suits. x x x It is plain that investigation of the matters involved in suits
of the individual's right to privacy as the requirement to disclosure brought or to be commenced under the Senate resolution directing the
information is for a valid purpose, in this case, to ensure that the institution of suits for the cancellation of the leases might directly aid in
government agencies involved in regulating banking transactions respect of legislative action.
adequately protect the public who invest in foreign securities. Suffice it to
state that this purpose constitutes a reason compelling enough to proceed
with the assailed legislative investigation.16 The prosecution of offenders by the prosecutorial agencies and the trial
before the courts is for the punishment of persons who transgress the law.
The intent of legislative inquiries, on the other hand, is to arrive at a policy
As regards the issue of self-incrimination, the petitioners, officers of SCB- determination, which may or may not be enacted into law.
Philippines, are not being indicted as accused in a criminal proceeding.
Except only when it exercises the power to punish for contempt, the
respondent, as with the other Committees of the Senate or of the House of
Representatives, cannot penalize violators even if there is overwhelming
evidence of criminal culpability. Other than proposing or initiating
amendatory or remedial legislation, respondent can only recommend
measures to address or remedy whatever irregularities may be unearthed
during the investigation, although it may include in its Report a
recommendation for the criminal indictment of persons who may appear
liable. At best, the recommendation, along with the evidence, contained in
such a Report would be persuasive, but it is still up to the prosecutorial
agencies and the courts to determine the liabilities of the offender.

Finally, petitioners sought anew, in their Manifestation and Motion21 dated


June 21, 2006, the issuance by this Court of a TRO and/or writ of
preliminary injunction to prevent respondent from submitting its
Committee Report No. 75 to the Senate in plenary for approval. However,
16 days prior to the filing of the Manifestation and Motion, or on June 5,
2006, respondent had already submitted the report to the Senate in
plenary. While there is no showing that the said report has been approved
by the Senate, the subject of the Manifestation and Motion has inescapably
become moot and academic.

WHEREFORE, the Petition for Prohibition is DENIED for lack of merit. The
Manifestation and Motion dated June 21, 2006 is, likewise, DENIED for
being moot and academic. 

SO ORDERED.
said Burt in the Buenavista Estate. The second sum of P500,000 was all
paid to the same Ernest H. Burt through his other attorney-in-fact, the
North Manila Development Co., Inc., also represented by Jean L. Arnault,
G.R. No. L-3820 July 18, 1950
for the alleged interest of the said Burt in the Tambobong
Estate.chanroblesvirtualawlibrary chanrobles virtual law library
JEAN L. ARNAULT, Petitioner, vs. LEON NAZARENO, Sergeant-at-
arms, Philippine Senate, and EUSTAQUIO BALAGTAS, Director of
The original owner of the Buenavista Estate was the San Juan de Dios
Prisons, Respondents.
Hospital. The Philippine Government held a 25-year lease contract on said
estate, with an option to purchase it for P3,000,000 within the same period
J.C. Orendain, Augusto Revilla, and Eduardo Arboleda for petitioner. of 25 years counted from January 1, 1939. The occupation Republic of the
Office of the Solicitor General Felix Bautista Angelo, Lorenzo Sumulong, Philippines purported to exercise that option by tendering to the owner the
Lorenzo Ta�ada, and Vicente J. Francisco for respondents. sum of P3,000,000 and, upon its rejection, by depositing it in court on June
21, 1944, together with the accrued rentals amounting to P3224,000.
OZAETA, J.: Since 1939 the Government has remained in possession of the
estate.chanroblesvirtualawlibrary chanrobles virtual law library

This is an original petition for habeas corpus to relieve the petitioner from


his confinement in the New Bilibid Prison to which he has been committed On June 29, 1946, the San Juan de Dios Hospital sold the Buenavista
by virtue of a resolution adopted by the Senate on May 15, 1950, which Estate for P5,000,000 to Ernest H. Burt, who made a down payment of
reads as follows: P10,000 only and agreed to pay P5000,000 within one year and the
remainder in annual installments of P500,000 each, with the stipulation
that failure on his part to make any of said payments would cause the
Whereas, Jean L. Arnault refused to reveal the name of the person to forfeiture of his down payment of P10,000 and would entitle the Hospital to
whom he gave the P440,000, as well as answer other pertinent questions rescind to sale to him. Aside from the down payment of P10,000, Burt has
related to the said amount; Now, therefore, be made no other payment on account of the purchase price of said
it.chanroblesvirtualawlibrary chanrobles virtual law library estate.chanroblesvirtualawlibrary chanrobles virtual law library

Resolved, that for his refusal to reveal the name of the person to whom he The original owner of the Tambobong Estate was the Philippine Trust
gave the P440,000 Jean L. Arnault be committed to the custody of the Company. On May 14, 1946, the Philippine Trust Company sold estate for
Sergeant-at-Arms and imprisoned in the New Bilibid Prison, Muntinlupa, the sum of P1,200,000 to Ernest H. Burt, who paid P10,000 down and
Rizal, until discharged by further order of the Senate or by the special promise to pay P90,000 within nine months and the balance of P1,100,000
committee created by Senate Resolution No. 8, such discharge to be in ten successive installments of P110,000 each. The nine-month period
ordered when he shall have purged the contempt by revealing to the within which to pay the first installment of P90,000 expired on February
Senate or to the said special committee the name of the person to whom 14, 1947, without Burt's having paid the said or any other amount then or
he gave the P440,000, as well as answer other pertinent questions in afterwards. On September 4, 1947, the Philippine Trust Company sold,
connection therewith. conveyed, and delivered the Tambobong Estate to the Rural Progress
Administration by an absolute deed of sale in consideration of the sum of
The facts that gave rise to the adoption of said resolution, insofar as P750,000. On February 5, 1948, the Rural Progress Administration made,
pertinent here, may be briefly stated as follows:chanrobles virtual law under article 1504 of the Civil Code, a notarial demand upon Burt for the
library resolution and cancellation of his contract of purchase with the Philippine
Trust Company due to his failure to pay the installment of P90,000 within
the period of nine months. Subsequently the Court of First Instance of Rizal
In the latter part of October, 1949, the Philippine Government, through the ordered the cancellation of Burt's certificate of title and the issuance of a
Rural Progress Administration, bought two estates known as Buenavista new one in the name of the Rural Progress Administration, from which
and Tambobong for the sums of P4,500,000 and P500,000, respectively. Of order he appealed to the Supreme Court. 1 chanrobles virtual law library
the first sum, P1,000,000 was paid to Ernest H. Burt, a nonresident
American, thru his attorney-in-fact in the Philippines, the Associated
Estates, Inc., represented by Jean L. Arnault, for alleged interest of the
It was in the face of the antecedents sketched in the last three preceding to investigate the Buenavista and Tambobong Estate deals. It shall be the
paragraphs that the Philippine Government, through the Secretary of duty of the said Committee to determine whether the said purchase was
Justice as Chairman of the Board of Directors of the Rural Progress honest, valid, and proper and whether the price involved in the deal was
Administration and as Chairman of the Board of Directors of the Philippine fair and just, the parties responsible therefor, and any other facts the
National Bank, from which the money was borrowed, accomplished the Committee may deem proper in the premises. Said Committee shall have
purchase of the two estates in the latter part of October, 1949, as stated at the power to conduct public hearings; issue subpoena or subpoena duces
the outset.chanroblesvirtualawlibrary chanrobles virtual law library tecumto compel the attendance of witnesses or the production of
documents before it; and may require any official or employee of any
bureau, office, branch, subdivision, agency, or instrumentality of the
On February 27, 1950, the Senate adopted its Resolution No. 8, which
Government to assist or otherwise cooperate with the Special Committee in
reads as follows:
the performance of its functions and duties. Said Committee shall submit
its report of findings and recommendations within two weeks from the
RESOLUTION CREATING A SPECIAL COMMITTEE TO INVESTIGATE THE adoption of this Resolution.
BUENAVISTA AND THE TAMBOBONG ESTATES
DEAL.chanroblesvirtualawlibrary chanrobles virtual law library
The special committee created by the above resolution called and
examined various witnesses, among the most important of whom was the
WHEREAS, it is reported that the Philippine government, through the Rural herein petitioner, Jean L. Arnault. An intriguing question which the
Progress Administration, has bought the Buenavista and the Tambobong committee sought to resolve was that involved in the apparent
Estates for the aggregate sum of five million pesos;chanrobles virtual law unnecessariness and irregularity of the Government's paying to Burt the
library total sum of P1,500,000 for his alleged interest of only P20,000 in the two
estates, which he seemed to have forfeited anyway long before October,
WHEREAS, it is reported that under the decision of the Supreme Court 1949. The committee sought to determine who were responsible for and
dated October 31, 1949, the Buenavista Estate could have been bought for who benefited from the transaction at the expense of the
three million pesos by virtue of a contract entered into between the San Government.chanroblesvirtualawlibrary chanrobles virtual law library
Juan de Dios Hospital and Philippine Government in 1939;chanrobles
virtual law library Arnault testified that two checks payable to Burt aggregating P1,500,000
were delivered to him on the afternoon of October 29, 1949; that on the
WHEREAS, it is even alleged that the Philippine Government did not have same date he opened a new account in the name of Ernest H. Burt with the
to purchase the Buenavista Estate because the occupation government had Philippine National Bank in which he deposited the two checks aggregating
made tender of payment in the amount of three million pesos, Japanese P1,500,000; and that on the same occasion he draw on said account two
currency, which fact is believed sufficient to vest title of Ownership in the checks; one for P500,000, which he transferred to the account of the
Republic of the Philippines pursuant to decisions of the Supreme Court Associated Agencies, Inc., with the Philippine National Bank, and another
sustaining the validity of payments made in Japanese military notes during for P440,000 payable to cash, which he himself cashed. It was the desire of
the occupation;chanrobles virtual law library the committee to determine the ultimate recipient of this sum of P440,000
that gave rise to the present case.chanroblesvirtualawlibrary chanrobles
virtual law library
WHEREAS, it is reported that the Philippine Government did not have to
pay a single centavo for the Tambobong Estate as it was already practically
owned by virtue of a deed of sale from the Philippine Trust Company dated At first the petitioner claimed before the Committee:
September 3, 194, for seven hundred and fifty thousand pesos, and by
virtue of the recission of the contract through which Ernest H. Burt had an Mr. ARNAULT (reading from a note). Mr. Chairman, for questions involving
interest in the estate; Now, therefore, be the disposition of funds, I take the position that the transactions were
it.chanroblesvirtualawlibrary chanrobles virtual law library legal, that no laws were being violated, and that all requisites had been
complied with. Here also I acted in a purely functional capacity of
RESOLVED, That a Special Committee, be, as it hereby is, created, representative. I beg to be excused from making answer which might later
composed of five members to be appointed by the President of the Senate be used against me. I have been assured that it is my constitutional right
to refuse to incriminate myself, and I am certain that the Honorable
Members of this Committee, who, I understand, are lawyers, will see the The CHAIRMAN. The other check of P440,000 which you also made on
justness of my position. October 29, 1949, is payable to cash; and upon cashing this P440,000 on
October 29, 1949, what did you do with that amount?chanrobles virtual law
library
At as subsequent session of the committee (March 16) Senator De Vera, a
member of the committee, interrogated him as follows:
Mr. ARNAULT. I turned it over to a certain
person.chanroblesvirtualawlibrary chanrobles virtual law library
Senator DE VERA. Now these transactions, according to your own
typewritten statement, were legal?chanrobles virtual law library
The CHAIRMAN. The whole amount of P440,000? chanrobles virtual law
library
Mr. ARNAULT. I believe so.chanroblesvirtualawlibrary chanrobles virtual law
library
Mr. ARNAULT. Yes.chanroblesvirtualawlibrary chanrobles virtual law library
Senator DE VERA. And the disposition of that fund involved, according to
your own statement, did not violate any law?chanrobles virtual law library The CHAIRMAN. Who was that certain person to whom you delivered these
P440,000 which you cashed on October 29, 1949?chanrobles virtual law
library
Mr. ARNAULT. I believe so.

Mr. ARNAULT. I don't remember the name; he was a representative of


x x x           x x x           x x xchanrobles virtual law library
Burt.chanroblesvirtualawlibrary chanrobles virtual law library

Senator DE VERA. So that if the funds were disposed of in such a manner


The CHAIRMAN. That representative of Burt to whom you delivered the
that no laws were violated, how is it that when you were asked by the
P440,000 was a Filipino?chanrobles virtual law library
Committee to tell what steps you took to have this money delivered to
Burt, you refused to answer the questions, saying that it would incriminate
you?chanrobles virtual law library Mr. ARNAULT. I don't know.chanroblesvirtualawlibrary chanrobles virtual
law library
Mr. ARNAULT. Because it violates the rights of a citizen to privacy in his
dealings with other people. The CHAIRMAN. You do not remember the name of that representative of
Burt to whom you delivered this big amount of P440,000?chanrobles virtual
law library
x x x           x x x           x x xchanrobles virtual law library

Mr. ARNAULT. I am not sure; I do not remember the


Senator DE VERA. Are you afraid to state how the money was disposed of
name.chanroblesvirtualawlibrary chanrobles virtual law library
because you would be incriminated, or you would be incriminating
somebody? chanrobles virtual law library
The CHAIRMAN. That certain person who represented Burt to whom you
delivered the big amount on October 29, 1949, gave you a receipt for the
Mr. ARNAULT. I am not afraid; I simply stand on the privilege to dispose of
amount?chanrobles virtual law library
the money that has been paid to me as a result of a legal transaction
without having to account for any use of it.
Mr. ARNAULT. No.chanroblesvirtualawlibrary chanrobles virtual law library
But when in the same session the chairman of the committee, Senator
Sumulong, interrogated the petitioner, the latter testified as follows: The CHAIRMAN. Neither did you ask a receipt? chanrobles virtual law
library
Mr. ARNAULT. I didn't ask.chanroblesvirtualawlibrary chanrobles virtual law The CHAIRMAN. When was that instruction given to you by Burt?chanrobles
library virtual law library

The CHAIRMAN. And why did you give that certain person, representative Mr. ARNAULT. Long time ago.chanroblesvirtualawlibrary chanrobles virtual
of Burt, this big amount of P440,000 which forms part of the P1-� million law library
paid to Burt?chanrobles virtual law library
The CHAIRMAN. In what year did Burt give you that verbal instruction;
Mr. ARNAULT. Because I have instructions to that when Burt was still here in the Philippines?chanrobles virtual law library
effect.chanroblesvirtualawlibrary chanrobles virtual law library
Mr. ARNAULT. Yes.chanroblesvirtualawlibrary chanrobles virtual law library
The CHAIRMAN. Who gave you the instruction?chanrobles virtual law
library
The CHAIRMAN. But at that time Burt already knew that he would receive
the money?chanrobles virtual law library
Mr. ARNAULT. Burt.chanroblesvirtualawlibrary chanrobles virtual law library
Mr. ARNAULT. No.chanroblesvirtualawlibrary chanrobles virtual law library
The CHAIRMAN. Where is the instruction; was that in writing?chanrobles
virtual law library
The CHAIRMAN. In what year was that when Burt while he was here in the
Philippines gave you the verbal instruction?chanrobles virtual law library
Mr. ARNAULT. No.chanroblesvirtualawlibrary chanrobles virtual law library
Mr. ARNAULT. In 1946.chanroblesvirtualawlibrary chanrobles virtual law
The CHAIRMAN. By cable?chanrobles virtual law library library

Mr. ARNAULT. No.chanroblesvirtualawlibrary chanrobles virtual law library The CHAIRMAN. And what has that certain person done for Burt to merit
receiving these P440,000?chanrobles virtual law library
The CHAIRMAN. In what form did you receive that instruction?chanrobles
virtual law library Mr. ARNAULT. I absolutely do not
know.chanroblesvirtualawlibrary chanrobles virtual law library
Mr. ARNAULT. Verbal instruction.chanroblesvirtualawlibrary chanrobles
virtual law library The CHAIRMAN. You do not know?chanrobles virtual law library

The CHAIRMAN. When did you receive this verbal instruction from Burt to Mr. ARNAULT. I do not know.chanroblesvirtualawlibrary chanrobles virtual
deliver these P440,000 to a certain person whose name you do not like to law library
reveal?chanrobles virtual law library
The CHAIRMAN. Burt did not tell you when he gave you the verbal
Mr. ARNAULT. I have instruction to comply with the request of the instruction why that certain person should receive these P440,000?
person.chanroblesvirtualawlibrary chanrobles virtual law library chanrobles virtual law library

The CHAIRMAN. Now, you said that instruction given to you by Burt was Mr. ARNAULT. He did not tell me.chanroblesvirtualawlibrary chanrobles
verbal?chanrobles virtual law library virtual law library

Mr. ARNAULT. Yes.chanroblesvirtualawlibrary chanrobles virtual law library


The CHAIRMAN. And Burt also authorized you to give this big amount to Mr. ARNAULT. I would say Spanish
that certain person without receipt?chanrobles virtual law library name.chanroblesvirtualawlibrary chanrobles virtual law library

Mr. ARNAULT. He told me that a certain person would represent him and The CHAIRMAN. And how about his Christian name; is it also a Spanish
where could I meet him.chanroblesvirtualawlibrary chanrobles virtual law name?chanrobles virtual law library
library
Mr. ARNAULT. I am not sure; I think the initial is
The CHAIRMAN. Did Burt know already that certain person as early as J.chanroblesvirtualawlibrary chanrobles virtual law library
1946?chanrobles virtual law library
The CHAIRMAN. Did he have a middle name?chanrobles virtual law library
Mr. ARNAULT. I presume much before
that.chanroblesvirtualawlibrary chanrobles virtual law library
Mr. ARNAULT. I never knew it.chanroblesvirtualawlibrary chanrobles virtual
law library
The CHAIRMAN. Did that certain person have any intervention in the
prosecution of the two cases involving the Buenavista and Tambobong
The CHAIRMAN. And how about his family name which according to your
estates?chanrobles virtual law library
recollection is Spanish; can you remember the first letter with which that
family name begins?chanrobles virtual law library
Mr. ARNAULT. Not that I know of.chanroblesvirtualawlibrary chanrobles
virtual law library
Mr. ARNAULT. S, D or F.chanroblesvirtualawlibrary chanrobles virtual law
library
The CHAIRMAN. Is that certain person related to any high government
official?chanrobles virtual law library
The CHAIRMAN. And what was the last letter of the family name?
chanrobles virtual law library
Mr. ARNAULT. No, I do not know.chanroblesvirtualawlibrary chanrobles
virtual law library
Mr. ARNAULT. I do not know.chanroblesvirtualawlibrary chanrobles virtual
law library
The CHAIRMAN. Why can you not tell us the name of that certain person?
chanrobles virtual law library
The CHAIRMAN. Have you seen that person again after you have delivered
this P440,000?chanrobles virtual law library
Mr. ARNAULT. Because I am not sure of his name; I cannot remember the
name.chanroblesvirtualawlibrary chanrobles virtual law library
Mr. ARNAULT. Yes.chanroblesvirtualawlibrary chanrobles virtual law library

The CHAIRMAN. When gave that certain person that P440,000 on October
The CHAIRMAN. Several times?chanrobles virtual law library
29, 1949, you knew already that person?chanrobles virtual law library

Mr. ARNAULT. Two or three times.chanroblesvirtualawlibrarychanrobles


Mr. ARNAULT. Yes, I have seen him several
virtual law library
times.chanroblesvirtualawlibrary chanrobles virtual law library

The CHAIRMAN. Here in Manila?chanrobles virtual law library


The CHAIRMAN. And the name of that certain person is a Filipino name?
chanrobles virtual law library
Mr. ARNAULT. Yes.chanroblesvirtualawlibrary chanrobles virtual law library
The CHAIRMAN. And in spite of the fact that you met that person two or The CHAIRMAN. How tall is that certain person?chanrobles virtual law
three times, you never were able to find out what was his name? library
chanrobles virtual law library
Mr. ARNAULT. Between 5-2 and 5-6.
Mr. ARNAULT. If I knew, I would [have] taken it down. Mr. Peralta knows
my name; of course, we have not done business. Lots of people in Manila
On May 15, 1950, the petitioner was haled before the bar of the Senate,
know me, but they don't know my name, and I don't know them. They
which approved and read to him the following resolution:
sa{ I am "chiflado" because I don't know their
names.chanroblesvirtualawlibrary chanrobles virtual law library
Be it resolved by the Senate of the Philippines in Session
assembled: chanrobles virtual law library
The CHAIRMAN. That certain person is a male or female?chanrobles virtual
law library
That Jean L. Arnault, now at the bar of the Senate, be arraigned for
contempt consisting of contumacious acts committed by him during the
Mr. ARNAULT. He is a male.chanroblesvirtualawlibrary chanrobles virtual
investigation conducted by the Special Committee created by Senate
law library
Resolution No. 8 to probe the Tambobong and Buenavista estates deal of
October 21, 1949, and that the President of the Senate propounded to him
The CHAIRMAN. You are sure that he is a male at least?chanrobles virtual the following interrogatories:chanrobles virtual law library
law library
1. What excuse have you for persistently refusing to reveal the name of
Mr. ARNAULT. Let us say 38 or 40 years, more or the person to whom you gave the P440,000 on October 29, 1949, a person
less.chanroblesvirtualawlibrary chanrobles virtual law library whose name it is impossible for you not to remember not only because of
the big amount of money you gave to him without receipt, but also by your
own statements you knew him as early as 1946 when General Ernest H.
The CHAIRMAN. Can you give us, more or less, a description of that certain
Burt was still in the Philippines, you made two other deliveries of money to
person? What is his complexion: light, dark or light brown?chanrobles
him without receipt, and the last time you saw him was in December 1949?
virtual law library

Thereupon petitioner's attorney, Mr. Orendain, submitted for him a written


Mr. ARNAULT. He is like the gentleman there (pointing to Senator Cabili),
answer alleging that the questions were incriminatory in nature and
but smaller. He walks very straight, with military
begging leave to be allowed to stand on his constitutional right not to be
bearing.chanroblesvirtualawlibrary chanrobles virtual law library
compelled to be a witness against himself. Not satisfied with that written
answer Senator Sumulong, over the objection of counsel for the petitioner,
The CHAIRMAN. Do you know the residence of that certain person to whom propounded to the latter the following question:
you gave the P440,000?chanrobles virtual law library
Sen. SUMULONG. During the investigation, when the Committee asked you
Mr. ARNAULT. No.chanroblesvirtualawlibrary chanrobles virtual law library for the name of that person to whom you gave the P440,000, you said that
you can [could] not remember his name. That was the reason then for
The CHAIRMAN. During these frequent times that you met that certain refusing to reveal the name of the person. Now, in the answer that you
person, you never came to know his residence?chanrobles virtual law have just cited, you are refusing to reveal the name of that person to
library whom you gave the P440,000 on the ground that your answer will be self-
incriminating. Now, do I understand from you that you are abandoning
your former claim that you cannot remember the name of that person, and
Mr. ARNAULT. No, because he was coming to the that your reason now for your refusal to reveal the name of that person is
office.chanroblesvirtualawlibrary chanrobles virtual law library that your answer might be self-incriminating? In other words, the question
is this: What is your real reason for refusing to reveal the name of that
person to whom you gave the P440,000: that you do not remember his Mr. ARNAULT. I do not remember .chanroblesvirtualawlibrarychanrobles
name or that your answer would be self-incriminating? virtual law library

xxx     xxx     xxx Sen. SUMULONG. Now, if you do not remember the name of that person,
how can you say that your answer might be incriminating? If you do not
remember his name, you cannot answer the question; so how could your
Mr. ORENDAIN. Mr. President, we are begging for the rules of procedure
answer be self-incriminating? What do you say to that?chanrobles virtual
that the accused should not be required to testify unless he so
law library
desires.chanroblesvirtualawlibrary chanrobles virtual law library

Mr. ARNAULT. This is too complicated for me to explain. Please, I do not


The PRESIDENT. It is the duty of the respondent to answer the question.
see how to answer those questions. That is why I asked for a lawyer, so he
The question is very clear. It does not incriminate him.
can help me. I have no means of knowing what the situation is about. I
have been in jail 13 days without communication with the outside. How
xxx     xxx     xxx could I answer the question? I have no knowledge of legal procedure or
rule, of which I am completely ignorant.
Mr. ARNAULT. I stand by every statement that I have made before the
Senate Committee on the first, second, and third hearings to which I was xxx     xxx     xxx
made in my letter to this Senate of May 2, 1950, in which I gave all the
reasons that were in my powers to give, as requested. I cannot change
Sen. SUMULONG. Mr. President, I ask that the question be
anything in those statements that I made because they represent the best
answered.chanroblesvirtualawlibrary chanrobles virtual law library
that I can do , to the best of my
ability.chanroblesvirtualawlibrary chanrobles virtual law library
The PRESIDENT. The witness is ordered to answer the question. It is very
clear. It does not incriminate the witness.
The PRESIDENT. You are not answering the question. The answer has
nothing to do with the question.chanroblesvirtualawlibrary chanrobles
virtual law library xxx     xxx     xxx

Sen. SUMULONG. I would like to remind you , Mr. Arnault, that the reason Mr. ARNAULT. I do not remember. I stand on my constitutional rights. I
that you gave during the investigation for not revealing the name of the beg to be excused from making further answer,
person to whom you gave the P440,000 is not the same reason that you please.chanroblesvirtualawlibrary chanrobles virtual law library
are now alleging because during the investigation you told us: "I do not
remember his name." But, now, you are now saying: "My answer might
Sen. SUMULONG. In that mimeographed letter that you sent addressed to
incriminate me." What is your real position?chanrobles virtual law library
the President of the Senate, dated May 2, 1950, you stated there that you
cannot reveal the name of the person to whom you gave the P440,000
Mr. ARNAULT. I have just stated that I stand by my statements that I because if he is a public official you might render yourself liable for
made at the first, second, and third hearings. I said that I wanted to be prosecution for bribery, and that if he is a private individual you might
excused from answering the question. I beg to be excused from making render yourself liable for prosecution for slander. Why did you make those
any answer that might be incriminating in nature. However, in this answer, statements when you cannot even tell us whether that person to whom you
if the detail of not remembering the name of the person has not been gave the P440,000 is a public official or a private individual ? We are giving
included, it is an oversight.chanroblesvirtualawlibrary chanrobles virtual you this chance to convince the Senate that all these allegations of yours
law library that your answers might incriminate you are given by you honestly or you
are just trying to make a pretext for not revealing the information desired
by the Senate.chanroblesvirtualawlibrary chanrobles virtual law library
Sen. SUMULONG. Mr. Arnault, will you kindly answer a simple question: Do
you remember or not the name of the person to whom you gave the
P440,000?chanrobles virtual law library
The PRESIDENT. You are ordered to answer the The PRESIDENT. How is it that you do not remember events that happened
question.chanroblesvirtualawlibrary chanrobles virtual law library a short time ago and, on the other hand, you remember events that
occurred during your childhood?chanrobles virtual law library
Mr. ARNAULT. I do not even understand the question. (The question is
restated and explained.)chanrobles virtual law library Mr. ARNAULT. I cannot explain.

Mr. ARNAULT. That letter of May 2, was prepared by a lawyer for me and The Senate then deliberated and adopted the resolution of May 15
signed it. That is all I can say how I stand about this letter. I have no hereinabove quoted whereby the petitioner was committed to the custody
knowledge myself enough to write such a letter, so I had to secure the help of the Sergeant-at-Arms and imprisoned until "he shall have purged the
of a lawyer to help me in my period of distress. contempt by revealing to the Senate or to the aforesaid Special Committee
the name of the person to whom he gave the P440,000, as well as answer
other pertinent questions in connection therewith."chanrobles virtual law
In that same session of the Senate before which the petitioner was called
library
to show cause why he should not be adjudged guilty of contempt of the
Senate, Senator Sumulong propounded to the petitioner questions tending
to elicit information from him as to the identity of the person to whom he The Senate also adopted on the same date another resolution (No. 16) , to
delivered the P440,000; but the petitioner refused to reveal it by saying wit:
that he did not remember. The President of the Senate then propounded to
him various questions concerning his past activities dating as far back as
That the Special Committee created by Senate Resolution No. 8 be
when witness was seven years of age and ending as recently as the post
empowered and directed to continue its investigation of the Tambobong
liberation period, all of which questions the witness answered satisfactorily.
and Buenavista Estates deal of October 21, 1949, more particularly to
In view thereof, the President of the Senate also made an attempt to illicit
continue the examination of Jean L. Arnault regarding the name of the
the desired information from the witness, as follows:
person to whom he gave the P440,000 and other matters related
therewith.
The PRESIDENT. Now I am convinced that you have a good memory.
Answer: Did you deliver the P440,000 as a gift, or of any consideration?
The first session of the Second Congress was adjourned at midnight on
chanrobles virtual law library
May 18, 1950.chanroblesvirtualawlibrary chanrobles virtual law library

Mr. ARNAULT. I have said that I had instructions to deliver it to that


The case was argued twice before us. We have given its earnest and
person, that is all.chanroblesvirtualawlibrary chanrobles virtual law library
prolonged consideration because it is the first of its kind to arise since the
Constitution of the Republic of the Philippines was adopted. For the first
The PRESIDENT. Was it the first time you saw that person?chanrobles time this Court is called upon to define the power of either House of
virtual law library Congress to punish a person not a member for contempt; and we are fully
conscious that our pronouncements here will set an important precedent
for the future guidance of all
Mr. ARNAULT. I saw him various times, I have already
concerned.chanroblesvirtualawlibrary chanrobles virtual law library
said.chanroblesvirtualawlibrary chanrobles virtual law library

Before discussing the specific issues raised by the parties, we deem it


The PRESIDENT. In spite of that, you do not have the least remembrance
necessary to lay down the general principles of law which form the
of the name of that person?chanrobles virtual law library
background of those issues.chanroblesvirtualawlibrary chanrobles virtual
law library
Mr. ARNAULT. I cannot remember.chanroblesvirtualawlibrarychanrobles
virtual law library
Patterned after the American system, our Constitution vests the powers of
the Government in three independent but coordinate Departments -
Legislative, Executive, and Judicial. The legislative power is vested in the
Congress, which consists of the Senate and the House of Representatives. it may enter is also wider. It would be difficult to define any limits by which
(Section 1, Article VI.) Each house may determine the rules of its the subject matter of its inquiry can be bounded. It is not necessary to do
proceedings, punish its Members for disorderly behavior, and, with the so in this case. Suffice it to say that it must be coextensive with the range
concurrence of two-thirds of all its Members, expel a Member. (Section 10, of the legislative power.chanroblesvirtualawlibrary chanrobles virtual law
Article VI.) The judicial power is vested in the Supreme Court and in such library
inferior courts as may be established by law. (Section 1, Article VIII.) Like
the Constitution of the United States, ours does not contain an express
In the present case the jurisdiction of the Senate, thru the Special
provision empowering either of the two Houses of Congress to punish
Committee created by it, to investigate the Buenavista and Tambobong
nonmembers for contempt. It may also be noted that whereas in the
Estates deal is not challenged by the petitioner; and we entertain no doubt
United States the legislative power is shared by and between the Congress
as to the Senate's authority to do so and as to the validity of Resolution
of the United States, on the one hand, and the respective legislatures of
No. 8 hereinabove quoted. The transaction involved a questionable and
the different States, on the other - the powers not delegated to the United
allegedly unnecessary and irregular expenditure of no less than P5,000,000
States by the Constitution nor prohibited by it to States being reserved to
of public funds, of which Congress is the constitutional guardian. It also
the States, respectively, or to the people - in the Philippines, the legislative
involved government agencies created by Congress to regulate or even
power is vested in the Congress of the Philippines alone. It may therefore
abolish. As a result of the yet uncompleted investigation, the investigating
be said that the Congress of the Philippines has a wider range of legislative
committee has recommended and the Senate approved three bills (1)
field than the Congress of the United States or any State Legislature. Our
prohibiting the Secretary of Justice or any other department head from
form of Government being patterned after the American system - the
discharging functions and exercising powers other than those attached to
framers of our Constitution having drawn largely from American institutions
his own office, without ]previous congressional authorization; (2)
and practices - we can, in this case, properly draw also from American
prohibiting brothers and near relatives of any President of the Philippines
precedents in interpreting analogous provisions of our Constitution, as we
from intervening directly or indirectly and in whatever capacity in
have done in other cases in the past. Although there is no provision in the
transactions in which the Government is a party, more particularly where
Constitution expressly investing either House of Congress with power to
the decision lies in the hands of executive or administrative officers who
make investigations and exact testimony to the end that it may exercise its
are appointees of the President; and (3) providing that purchases of the
legislative functions as to be implied. In other words, the power of inquiry -
Rural Progress Administration of big landed estates at a price of P100,000
with process to enforce it - is an essential and appropriate auxiliary to the
or more, shall not become effective without previous congressional
legislative function. A legislative body cannot legislate wisely or effectively
confirmation. 2 chanrobles virtual law library
in the absence of information respecting the conditions which the
legislation is intended to effect or change; and where the legislative body
does not itself possess the requisite information - which is not infrequently We shall now consider and pass upon each of the questions raised by the
true - recourse must be had to others who do possess it. Experience has petitioner in support of his contention that his commitment is
shown that mere requests for such information are often unavailing, and unlawful.chanroblesvirtualawlibrary chanrobles virtual law library
also that information which is volunteered is not always accurate or
complete; so some means of compulsion is essential to obtain what is First He contends that the Senate has no power to punish him for contempt
needed. (McGrain vs. Daugherty, 273 U.S., 135; 71 L. ed., 580; 50 A.L R., for refusing to reveal the name of the person to whom he gave the
1.) The fact that the Constitution expressly gives to Congress the power to P440,000, because such information is immaterial to, and will not serve,
punish its Members for disorderly behavior, does not by necessary any intended or purported legislation and his refusal to answer the
implication exclude the power to punish for contempt any other person. question has not embarrassed, obstructed, or impeded the legislative
(Anderson vs.Dunn, 6, Wheaton, 204; 5 L. ed., 242.) But no person can be process. It is argued that since the investigating committee has already
punished for contumacy as a witness before either House, unless his rendered its report and has made all its recommendations as to what
testimony is required in a matter into which that House has jurisdiction to legislative measures should be taken pursuant to its findings, there is no
inquire. (Kilbourn vs. Thompson, 26 L. ed., necessity to force the petitioner to give the information desired other than
377.).chanroblesvirtualawlibrary chanrobles virtual law library that mentioned in its report, to wit: "In justice to Judge Quirino and to
Secretary Nepomuceno, this atmosphere of suspicion that now pervades
Since, as we have noted, the Congress of the Philippines has a wider range the public mind must be dissipated, and it can only be done if appropriate
of legislative field than either the Congress of the United States or a State steps are taken by the Senate to compel Arnault to stop pretending that he
Legislature, we think it is correct to say that the field of inquiry into which cannot remember the name of the person to whom he gave the P440,000
and answer the questions which will definitely establish the identity of that with a legitimate object if it is capable of being so construed, and we have
person . . ." Senator Sumulong, Chairman of the Committee, who appeared no right to assume that the contrary was intended." (People ex
and argued the case for the respondents, denied that that was the only rel. McDonald vs. Keeler, 99 N.Y., 463; 52 Am. Rep., 49; 2 N.E., 615,
purpose of the Senate in seeking the information from the witness. He said quoted with approval by the Supreme Court of the United States in the said
that the investigation had not been completed, because, due to the case of McGrain vs.Daugherty, it is necessary deduction from the decision
contumacy of the witness, his committee had not yet determined the in Re Chapman, 41 L. ed., 1154, that where the questions are not
parties responsible for the anomalous transaction as required by Resolution pertinent to the matter under inquiry a witness rightfully may refuse to
No. 8; that, by Resolution No. 16, his committee was empowered and answer. So we are of the opinion that where the alleged immateriality of
directed to continue its investigation, more particularly to continue its the information sought by the legislative body from a witness is relied upon
examination of the witness regarding the name of the person to whom he to contest its jurisdiction, the court is in duty bound to pass upon the
gave the P440,000 and other matters related therewith; that the bills contention. The fact that the legislative body has jurisdiction or the power
recommended by his committee had not been approved by the House and to make the inquiry would not preclude judicial intervention to correct a
might not be approved pending the completion of the investigation; and clear abuse of discretion in the exercise of that
that those bills were not necessarily all the measures that Congress might power.chanroblesvirtualawlibrary chanrobles virtual law library
deem it necessary to pass after the investigation is
finished.chanroblesvirtualawlibrary chanrobles virtual law library
Applying the criterion laid down in the last two preceding paragraphs to the
resolution of the issue under consideration, we find that the question for
Once an inquiry is admitted or established to be within the jurisdiction of a the refusal to answer which the petitioner was held in contempt by the
legislative body to make, we think the investigating committee has the Senate is pertinent to the matter under inquiry. In fact, this is not and
power to require a witness to answer any question pertinent to that cannot be disputed. Senate Resolution No. 8, the validity of which is not
inquiry, subject of course to his constitutional right against self- challenged by the petitioner, requires the Special Committee, among other
incrimination. The inquiry, to be within the jurisdiction of the legislative things, to determine the parties responsible for the Buenavista and
body to make, must be material or necessary to the exercise of a power in Tambobong estates deal, and it is obvious that the name of the person to
it vested by the Constitution, such as to legislate, or to expel a Member; whom the witness gave the P440,000 involved in said deal is pertinent to
and every question which the investigator is empowered to coerce a that determination - it is in fact the very thing sought to be determined.
witness to answer must be material or pertinent to the subject of the The contention is not that the question is impertinent to the subject of the
inquiry or investigation. So a witness may not be coerced to answer a inquiry but that it has no relation or materiality to any proposed legislation.
question that obviously has no relation to the subject of the inquiry. But We have already indicated that it is not necessary for the legislative body
from this it does not follow that every question that may be propounded to to show that every question propounded to a witness is material to any
a witness must be material to any proposed or possible legislation. In other proposed or possible legislation; what is required is that is that it be
words, the materiality of the question must be determined by its direct pertinent to the matter under inquiry.chanroblesvirtualawlibrary chanrobles
relation to any proposed or possible legislation. The reason is, that the virtual law library
necessity or lack of necessity for legislative action and the form and
character of the action itself are determined by the sum total of the
It is said that the Senate has already approved the three bills
information to be gathered as a result of the investigation, and not by a
recommended by the Committee as a result of the uncompleted
fraction of such information elicited from a single
investigation and that there is no need for it to know the name of the
question.chanroblesvirtualawlibrary chanrobles virtual law library
person to whom the witness gave the P440,000. But aside from the fact
that those bills have not yet been approved by the lower house and by the
In this connection, it is suggested by counsel for the respondents that the President and that they may be withdrawn or modified if after the inquiry is
power of the Court is limited to determining whether the legislative body completed they should be found unnecessary or inadequate, there is
has jurisdiction to institute the inquiry or investigation; that once that nothing to prevent the Congress from approving other measures it may
jurisdiction is conceded, this Court cannot control the exercise of that deem necessary after completing the investigation. We are not called upon,
jurisdiction; and it is insinuated, that the ruling of the Senate on the nor is it within our province, to determine or imagine what those measures
materiality of the question propounded to the witness is not subject to may be. And our inability to do so is no reason for overruling the question
review by this Court under the principle of the separation of powers. We propounded by the Senate to the
have to qualify this proposition. As was said by the Court of Appeals of New witness.chanroblesvirtualawlibrary chanrobles virtual law library
York: "We are bound to presume that the action of the legislative body was
The case of Re Chapman , 166 U.S., 661; 41 L. ed., 1154, is in point here. there is no suggestion that the judiciary has instituted an inquiry to
The inquiry there in question was conducted under a resolution of the determine the parties responsible for the deal. Under the circumstances of
Senate and related to charges, published in the press, that senators were the case, it appearing that the questioned transaction was affected by the
yielding to corrupt influences in considering a tariff bill then before the head of the Department of Justice himself, it is not reasonable to expect
Senate and were speculating in stocks the value of which would be affected that the Fiscal or the Court of First Instance of Manila will take the initiative
by pending amendments to the bill. Chapman, a member of a firm of stock to investigate and prosecute the parties responsible for the deal until and
brokers dealing in the stock of the American Sugar Refining Company, unless the Senate shall determined those parties are and shall taken such
appeared before the committee in response to a subpoena and asked, measures as may be within its competence to take the redress the wrong
among others, the following questions: that may have been committed against the people as a result of the
transaction. As we have said, the transaction involved no less than
P5,000,000 of public funds. That certainly is a matter of a public concern
Had the firm, during the month of March, 1894, bought or sold any stock
which it is the duty of the constitutional guardian of the treasury to
or securities, known as sugar stocks, for or in the interest, directly or
investigate.chanroblesvirtualawlibrary chanrobles virtual law library
indirectly, of any United Senate senator?chanrobles virtual law library

If the subject of investigation before the committee is within the range of


Was the said firm at that time carrying any sugar stock for the benefit of,
legitimate legislative inquiry and the proposed testimony of the witness
or in the interest, directly or indirectly, of any United Senate senator?
called relates to that subject, obedience, to its process may be enforced by
the committee by imprisonment. (Sullivan vs. Hill, 73 W. Va., 49; 79 S.E.,
He refused to answer the questions and was prosecuted under an Act of 670; 40 Ann. Cas. [1916 B.], 1115.)chanrobles virtual law library
Congress for contempt of the Senate. Upon being convicted and sent to jail
he petitioned the Supreme Court of the United States for a writ of habeas
The decision in the case of Kilbourn vs.Thompson, 26 L. ed., 377, relied
corpus. One of the questions decided by the Supreme Court of the United
upon by the petitioner, is not applicable here. In that case the inquiry
States in that case was whether the committee had the right to compel the
instituted by the House of Representatives of the United States related to a
witness to answer said questions, and the Court held that the committee
private real-estate pool or partnership in the District of Columbia. Jay Cook
did have such right, saying:
and Company had had an interest in the pool but become bankrupts, and
their estate was in course of administration in a federal bankruptcy court in
The questions were undoubtedlypertinent to the subject-matter of the Pennsylvania. The United States was one of their creditors. The trustee in
inquiry. The resolution directed the committee to inquire whether any the bankruptcy proceeding had effected a settlement of the bankrupts'
senator has been, or is, speculating in what are known as sugar stocks interest in the pool, and of course his action was subject to examination
during the consideration of the tariff bill now before the Senate." What the and approval or disapproval by the bankruptcy court. Some of the
Senate might or might not do upon the facts when ascertained, we cannot creditors, including the United States, were dissatisfied with the
say, nor are we called upon to inquire whether such ventures might be settlement. The resolution of the House directed the Committee "to inquire
defensible, as contended in argument, but is plain that negative answers into the nature and history of said real-estate pool and the character of
would have cleared that body of what the Senate regarded as offensive said settlement, with the amount of property involve, in which Jay Cooke
imputations, while affirmative answers might have led to further action on and Co. were interested, and the amount paid or to be paid in said
the part of the Senate within its constitutional powers. (Emphasis settlement, with power to send for persons and papers, and report to this
supplied.) House." The Supreme Court of the United States, speaking thru Mr. Justice
Miller, pointed out that the resolution contained no suggestion of
It may be contended that the determination of the parties responsible for contemplated legislation; that the matter was one in respect of which no
the deal is incumbent upon the judicial rather than upon the legislative valid legislation could be had; that the bankrupts' estate and the trustee's
branch. But we think there is no basis in fact or in law for such assumption. settlement were still pending in the bankruptcy court; and that the United
The petitioner has not challenged the validity of Senate Resolution No. 8, States and other creditors were free to press their claims in that
and that resolution expressly requires the committee to determine the proceeding. And on these grounds the court held that in undertaking the
parties responsible for the deal. We are bound to presume that the Senate investigation "the House of Representatives not only exceeded the limit of
has acted in the due performance of its constitutional function in instituting its own authority, but assumed a power which could only be properly
the inquiry, if the act is capable of being so construed. On the other hand, exercised by another branch of the government, because the power was in
its nature clearly judicial." The principles announced and applied in that
case are: that neither House of Congress possesses a "general power of that the facts connected with one such default revealed the possibility of
making inquiry into the private affairs of the citizen"; that the power other and greater maladministration, such considerations had not been put
actually possessed is limited to inquires relating to matters of which the before the Court. Nor had it been acquainted with the every-day nature of
particular House has jurisdiction, and in respect of which it rightfully may the particular investigation and the powers there exerted by the House,
take other action; that if the inquiry relates to a matter wherein relief or powers whose exercise was customary and familiar in legislative practice.
redress could be had only by judicial proceeding, it is not within the range Instead of assuming the character of an extraordinary judicial proceeding,
of this power , but must be left to the court, conformably to the the inquiry, place in its proper background, should have been regarded as
constitutional separation of government a normal and customary part of the legislative process. Detailed
powers.chanroblesvirtualawlibrary chanrobles virtual law library definiteness of legislative purpose was thus made the demand of the court
in Killbourn vs. Thompson. But investigators cannot foretell the results that
may be achieved. The power of Congress to exercise control over a real-
That case differs from the present case in two important respects: (1)
estate pool is not a matter for abstract speculation but one to be
There the court found that the subject of the inquiry, which related to a
determined only after an exhaustive examination of the problem.
private real-estate pool or partnership, was not within the jurisdiction of
Relationship, and not their possibilities, determine the extent of
either House of Congress; while here if it is not disputed that the subject of
congressional power. Constitutionality depends upon such disclosures.
the inquiry, which relates to a transaction involving a questionable
Their presence, whether determinative of legislative or judicial power,
expenditure by the Government of P5,000,000 of public funds, is within the
cannot be relegated to guesswork. Neither Congress nor the Court can
jurisdiction of the Senate, (2) There the claim of the Government as a
predict, prior to the event, the result of the investigation."chanrobles
creditor of Jay Cooke and Company, which had had an interest in the pool,
virtual law library
was pending adjudication by the court; while here the interposition of the
judicial power on the subject of the inquiry cannot be expected, as we have
pointed out above, until after the Senate shall have determined who the The other case relied upon by the petitioner is Marshall vs. Gordon, 243
parties responsible are and shall have taken such measures as may be U.S., 521; 61. ed., 881. The question there was whether the House of
within its competence to take to redress the wrong that may have been Representatives exceeded its power in punishing, as for contempt of its
committed against the people as a result of the authority, the District Attorney of the Southern District of New York, who
transaction.chanroblesvirtualawlibrary chanrobles virtual law library had written, published, and sent to the chairman of one of its committees
an ill-tempered and irritating letter respecting the action and purposes of
the committee in interfering with the investigation by the grand jury of
It is interesting to note that the decision in the case of
alleged illegal activities of a member of the House of Representatives.
Killbourn vs. Thompson has evoked strong criticisms from legal scholars.
Power to make inquires and obtain evidence by compulsory process was
(See Potts, Power of Legislative Bodies to Punish for Contempt [1926], 74
not involved. The court recognized distinctly that the House of
U. Pa. L. Rev., 692-699; James L. Land is, Constitutional Limitations on the
Representatives had implied power to punish a person not a member for
Congressional Power of Investigation [1926], 40 Harvard L. Rev., 153, 154,
contempt, but held that its action in this instance was without
214-220.) We quoted the following from Professor Land is' criticism: "Mr.
constitutional justification. The decision was put on the ground that the
Justice Miller saw the case purely as an attempt by the House to secure to
letter, while offensive and vexatious, was not calculated or likely to affect
the Government certain priority rights as creditor of the bankrupt concern.
the House in any of its proceedings or in the exercise of any of its
To him it assumed the character of a lawsuit between the Government and
functions. This brief statement of the facts and the issues decided in that
Jay Cooke and Co., with the Government, acting through the House,
case is sufficient to show the inapplicability thereof to the present case.
attempting to override the orderliness of established procedure and
There the contempt involved consisted in the district attorney's writing to
thereby prefer a creditors' bill not before the courts but before Congress.
the chairman of the committee an offensive and vexatious letter, while
That bankruptcy proceedings had already been instituted against Jay Cooke
here the contempt involved consists in the refusal of the witness to answer
and Co., in a federal court gave added impetus to such a conception. The
questions pertinent to the subject of an inquiry which the Senate has the
House was seeking to oust a court of prior acquired jurisdiction by an
power and jurisdiction to make . But in that case, it was recognized that
extraordinary and unwarranted assumption of "judicial power"! The broader
the House of Representatives has implied power to punish a person not a
aspect of the investigation had not been disclosed to the Court. That Jay
member of contempt. In that respect the case is applicable here in favor of
Cooke and Co.'s indebtedness and the particular funds in question were
the Senate's (and not of the Petitioner's )
only part of the great administrative problem connected with the use and
contention.chanroblesvirtualawlibrary chanrobles virtual law library
disposition of public monies, that the particular failure was of consequence
mainly in relation to the security demanded for all government deposits,
Second. It is next contended for the petitioner that the Senate lacks And the essential nature of the power also makes clear the cogency and
authority to commit him for contempt for a term beyond its period of application of the two limitations which were expressly pointed out in
legislative session, which ended on May 18, 1950. This contention is based Anderson vs. Dunn, supra, that is, that the power even when applied to
on the opinion of Mr. Justice Malcolm, concurred in by Justices Street and subjects which justified its exercise is limited to imprisonment and such
Villa-Real, in the case of Lopez vs. De los Reyes (1930), 55 Phil., 170. In imprisonment may not be extended beyond the session of the body in
that case it appears that on October 23, 1929, Candido Lopez assaulted a which the contempt occurred.
member of the House of Representatives while the latter was going to the
hall of the House of Representatives to attend the session which was then
Interpreting the above quotations, Chief Justice Avance�a held:
about to begin, as a result of which assault said representative was unable
to attend the sessions on that day and those of the two days next following
by reason of the threats which Candido Lopez made against him. By the From this doctrine it follows, in my judgement, that the imposition of the
resolution of the House adopted November 6, 1929, Lopez was declared penalty is limited to the existence of the legislative body, which ceases to
guilty of contempt of the House of Representatives and ordered punished function upon its final periodical dissolution. The doctrine refers to its
by confinement in Bilibid Prison for a period of twenty-four hours. That existence and not to any particular session thereof. This must be so,
resolution was not complied with because the session of the House of inasmuch as the basis of the power to impose such penalty is the right
Representatives adjourned at midnight on November 8, 1929, and was which the Legislature has to self-preservation, and which right is
reiterated at the next session on September 16, 1930. Lopez was enforceable during the existence of the legislative body. Many causes might
subsequently arrested, whereupon he applied for the writ of habeas be conceived to constitute contempt to the Legislature, which would
corpus in the Court of First Instance of Manila, which denied the continue to be a menace to its preservation during the existence of the
application. Upon appeal to the Supreme Court, six justices voted to grant legislative body against which contempt was
the writ: Justice Malcolm, Street, and Villa-real, on the ground that the committed.chanroblesvirtualawlibrary chanrobles virtual law library
term of imprisonment meted out to the petitioner could not legally be
extended beyond the session of the body in which the contempt occurred; If the basis of the power of the legislature to punish for contempt exists
and Justices Johns, Villamor, and Ostrand, on the ground that the while the legislative body exercising it is in session, then that power and
Philippine Legislature had no power to punish for contempt because it was the exercise thereof must perforce continue until the final adjournment and
a creature merely of an Act of the Congress of the United States and not of the election of its successor.
a Constitution adopted by the people. Chief Justice Avance�a, Justice
Johnson, and Justice Romualdez wrote separate opinions, concurring with
Justice Malcolm, Street, and Villa-Real, that the Legislature had inherent Mr. Justice Johnson's more elaborate opinion, supported by quotations
power to punish for contempt but dissenting from the opinion that the from Cooley's Constitutional Limitations and from Jefferson's Manual, is to
order of commitment could only be executed during the particular session the same effect. Mr. Justice Romualdez said: "In my opinion, where as in
in which the act of contempt was the case before us, the members composing the legislative body against
committed.chanroblesvirtualawlibrary chanrobles virtual law library which the contempt was committed have not yet completed their three-
year term, the House may take action against the petitioner
herein."chanrobles virtual law library
Thus, on the question under consideration, the Court was equally divided
and no decisive pronouncement was made. The opinion of Mr. Justice
Malcolm is based mainly on the following passage in the case of We note that the quotations from Anderson vs. Dunn and
Anderson vs.Dunn, supra: Marshall vs. Gordon relied upon by Justice Malcolm are obiter dicta.
Anderson vs. Dunn was an action of trespass against the Sergeant-at-Arms
of the House of Representatives of the United States for assault and
And although the legislative power continues perpetual, the legislative body battery and false imprisonment. The plaintiff had been arrested for
ceases to exist on the moment of its adjournment or periodical dissolution. contempt of the House, brought before the bar of the House, and
It follows that imprisonment must terminate with that adjournment. reprimanded by the Speaker, and then discharged from custody. The
question as to the duration of the penalty was not involved in that case.
as well as on the following quotation from Marshall vs. Gordon, supra: The question there was "whether the House of Representatives can take
cognizance of contempt committed against themselves, under any
circumstances." The court there held that the House of Representatives
had the power to punish for contempt, and affirmed the judgment of the any portion of itself in any parliamentary function beyond the end of the
lower court in favor of the defendant. In Marshall vs. Gordon, the question session without the consent of the other two branches. When done, it is by
presented was whether the House had the power under the Constitution to a bill constituting them commissioners for the particular purpose." But the
deal with the conduct of the district attorney in writing a vexatious letter as context shows that the reference is to the two houses of Parliament when
a contempt of its authority, and to inflict punishment upon the writer for adjourned by prorogation or dissolution by the King. The rule may be the
such contempt as a matter of legislative power. The court held that the same with the House of Representatives whose members are all elected for
House had no such power because the writing of the letter did not obstruct the period of a single Congress: but it cannot well be the same with the
the performance of legislative duty and did not endanger the preservation Senate, which is a continuing body whose members are elected for a term
of the power of the House to carry out its legislative authority. Upon that of six years and so divided into classes that the seats of one third only
ground alone, and not because the House had adjourned, the court ordered become vacant at the end of each Congress, two thirds always continuing
the discharge of the petitioner from into the next Congress, save as vacancies may occur through death or
custody.chanroblesvirtualawlibrary chanrobles virtual law library resignation.chanroblesvirtualawlibrary chanrobles virtual law library

The case where the question was squarely decided is Mr. Hinds in his collection of precedents, says: "The Senate, as a
McGrain vs. Daugherty, supra. There it appears that the Senate had continuing body, may continue its committees through the recess following
adopted a resolution authorizing and directing a select committee of five the expiration of a Congress;" and, after quoting the above statement from
senators to investigate various charges of misfeasance and nonfeasance in Jefferson's Manual, he says: "The Senate, however being a continuing
the Department of Justice after Attorney General Harry M. Daugherty body, gives authority to its committees during the recess after the
became its supervising head. In the course of the investigation the expiration of a Congress." So far as we are advised the select committee
committee caused to be served on Mally S. Daugherty, brother of Harry M. having this investigation in charge has neither made a final report nor been
Daugherty and president of the Midland National Bank of Washington Court discharged; nor has been continued by an affirmative order. Apparently its
House, Ohio, a subpoena commanding him to appear before it for the activities have been suspended pending the decision of this case. But, be
purpose of giving testimony relating to the subject under consideration. this as it may, it is certain that the committee may be continued or revived
The witness failed to appear without offering any excuse for his failure. The now by motion to that effect, and if, continued or revived, will have all its
committee reported the matter to the Senate and the latter adopted a original powers. This being so, and the Senate being a continuing body, the
resolution, "That the President of the Senate pro tempore issue his warrant case cannot be said to have become moot in the ordinary sense. The
commanding the Sergeant-at-Arms or his deputy to take into custody the situation is measurably like that in Southern P. Terminal Co. vs. Interstate
body of the said M.S. Daugherty wherever found, and to bring the said Commerce Commission, 219 U. S., 498, 514-516; 55 L. ed., 310, 315,
M.S. Daugherty before the bar of the Senate, then and there to answer 316; 31 Sup. Ct. Rep., 279, where it was held that a suit to enjoin the
such questions pertinent to the matter under inquiry as the Senate may enforcement of an order of the Interstate Commerce Commission did not
order the President of the Senate pro tempore to propound; and to keep become moot through the expiration of the order where it was capable of
the said M.S. Daugherty in custody to await the further order of the repetition by the Commission and was a matter of public interest. Our
Senate." Upon being arrested, the witness petitioned the federal court in judgment may yet be carried into effect and the investigation proceeded
Cincinnati for a writ of habeas corpus. The federal court granted the writ with from the point at which it apparently was interrupted by reason of
and discharged the witness on the ground that the Senate, in directing the the habeas corpus proceedings. In these circumstances we think a
investigation and in ordering the arrest, exceeded its power under the judgment should be rendered as was done in the case
Constitution. Upon appeal to the Supreme Court of the United States, one cited.chanroblesvirtualawlibrary chanrobles virtual law library
of the contentions of the witness was that the case ha become moot
because the investigation was ordered and the committee was appointed
What has been said requires that the final order in the District Court
during the Sixty-eighth Congress, which expired on March 4, 1926. In
discharging the witness from custody be reversed.
overruling the contention, the court said:

Like the Senate of the United States , the Senate of the Philippines is a
. . . The resolution ordering the investigation in terms limited the
continuing body whose members are elected for a term of six years and so
committee's authority to the period of the Sixty-eighth Congress; but this
divided that the seats of only one-third become vacant every two years,
apparently was changed by a later and amendatory resolution authorizing
two-thirds always continuing into the next Congress save as vacancies may
the committee to sit at such times and places as it might deem advisable
occur thru death or resignation. Members of the House of Representatives
or necessary. It is said in Jefferson's Manual: "Neither House can continue
are all elected for a term of four years; so that the term of every Congress performing the particular legislative function involved. To hold that it may
is four years. The Second Congress of the Philippines was constituted on punish the witness for contempt only during the session in which
December 30, 1949, and will expire on December 30, 1953. The resolution investigation was begun, would be to recognize the right of the Senate to
of the Senate committing the Petitioner was adopted during the first perform its function but at the same time to deny to it an essential and
session of the Second Congress, which began on the fourth Monday of appropriate means for its performance. Aside from this, if we should hold
January and ended in May 18, 1950.chanroblesvirtualawlibrary chanrobles that the power to punish for contempt terminates upon the adjournment of
virtual law library the session, the Senate would have to resume the investigation at the next
and succeeding sessions and repeat the contempt proceedings against the
witness until the investigation is completed-an absurd, unnecessary, and
Had said resolution of commitment been adopted by the House of
vexatious procedure, which should be
Representatives, we think it could be enforced until the final adjournment
avoided.chanroblesvirtualawlibrary chanrobles virtual law library
of the last session of the Second Congress in 1953. We find no sound
reason to limit the power of the legislative body to punish for contempt to
the end of every session and not to the end of the last session terminating As against the foregoing conclusion it is argued for the petitioner that the
the existence of that body. The very reason for the exercise of the power to power may be abusively and oppressively exerted by the Senate which
punish for contempt is to enable the legislative body to perform its might keep the witness in prison for life. But we must assume that the
constitutional function without impediment or obstruction. Legislative Senate will not be disposed to exert the power beyond its proper bounds.
functions may be and in practice are performed during recess by duly And if, contrary to this assumption, proper limitations are disregarded, the
constituted committees charged with the duty of performing investigations portals of this Court are always open to those whose rights might thus be
or conducting hearing relative to any proposed legislation. To deny to such transgressed.chanroblesvirtualawlibrary chanrobles virtual law library
committees the power of inquiry with process to enforce it would be to
defeat the very purpose for which that the power is recognized in the
Third. Lastly, the petitioner invokes the privilege against self-incrimination.
legislative body as an essential and appropriate auxiliary to is legislative
He contends that he would incriminate himself if he should reveal the name
function. It is but logical to say that the power of self-preservation is
of the person to whom he gave the P440,000 if that person be a public
coexistent with the life to be
official be (witness) might be accused of bribery, and if that person be a
preserved.chanroblesvirtualawlibrary chanrobles virtual law library
private individual the latter might accuse him of oral
defamation.chanroblesvirtualawlibrary chanrobles virtual law library
But the resolution of commitment here in question was adopted by the
Senate, which is a continuing body and which does not cease exist upon
The ground upon which the witness' claim is based is too shaky, in firm,
the periodical dissolution of the Congress or of the House of
and slippery to afford him safety. At first he told the Committee that the
Representatives. There is no limit as to time to the Senate's power to
transactions were legal, that no laws were violated, and that all requisites
punish for contempt in cases where that power may constitutionally be
had been replied with; but at the time he begged to be excused from
exerted as in the present case.chanroblesvirtualawlibrary chanrobles virtual
making answers "which might later be used against me." A little later he
law library
explained that although the transactions were legal he refused to answer
questions concerning them "because it violates the right of a citizen to
Mere reflection upon the situation at hand convinces us of the soundness of privacy in his dealings with other people . . . I simply stand on my privilege
this proposition. The Senate has ordered an investigation of the Buenavista to dispose of the money that has been paid to me as a result of a legal
and Tambobong estates deal, which we have found it is within its transaction without having to account for the use of it." But after being
competence to make. That investigation has not been completed because apparently convinced by the Committee that his position was untenable,
of the refusal of the petitioner as a witness to answer certain questions the witness testified that, without securing any receipt, he turned over the
pertinent to the subject of the inquiry. The Senate has empowered the P440,000 to a certain person, a representative of Burt, in compliance with
committee to continue the investigation during the recess. By refusing to Burt's verbal instruction made in 1946; that as far as he know, that certain
answer the questions, the witness has obstructed the performance by the person had nothing to do with the negotiations for the settlement of the
Senate of its legislative function, and the Senate has the power to remove Buenavista and Tambobong cases; that he had seen that person several
the obstruction by compelling the witness to answer the questions thru times before he gave him the P440,000 on October 29, 1949, and that
restraint of his liberty until he shall have answered them. That power since then he had seen him again two or three times, the last time being in
subsists as long as the Senate, which is a continuing body, persists in December, 1949, in Manila; that the person was a male, 39 to 40 years of
age, between 5 feet, 2 inches and 5 feet, 6 inches in height. Butt the Generally, the question whether testimony is privileged is for the
witness would not reveal the name of that person on these pretexts: " I determination of the Court. At least, it is not enough for the witness to say
don't remember the name; he was a representative of Burt." "I am not that the answer will incriminate him. as he is not the sole judge of his
sure; I don't remember the name."chanrobles virtual law library liability. The danger of self-incrimination must appear reasonable and real
to the court, from all the circumstances, and from the whole case, as well
as from his general conception of the relations of the witness. Upon the
We are satisfied that those answers of the witness to the important
facts thus developed, it is the province of the court to determine whether a
question, what is the name of that person to whom you gave the
direct answer to a question may criminate or not. . . . The fact that the
P440,000? were obviously false. His insistent claim before the bar of the
testimony of a witness may tend to show that he has violated the law is not
Senate that if he should reveal the name he would incriminate himself,
sufficient to entitle him to claim the protection of the constitutional
necessarily implied that he knew the name. Moreover, it is unbelievable
provision against self-incrimination, unless he is at the same time liable to
that he gave the P440,000 to a person to him
prosecution and punishment for such violation. The witness cannot assert
unknown.chanroblesvirtualawlibrary chanrobles virtual law library
his privilege by reason of some fanciful excuse, for protection against an
imaginary danger, or to secure immunity to a third person. ( 3
"Testimony which is obviously false or evasive is equivalent to a refusal to Wharton's Criminal Evidence, 11th ed., secs. 1135,1136.)chanrobles virtual
testify and is punishable as contempt, assuming that a refusal to testify law library
would be so punishable." (12 Am. Jur., sec. 15, Contempt, pp. 399-400.)
In the case of Mason vs. U.S., 61 L. ed., 1198, it appears that Mason was
It is the province of the trial judge to determine from all the facts and
called to testify before a grand jury engaged in investigating a charge of
circumstances of the case whether the witness is justified in refusing to
gambling against six other men. After stating that he was sitting at a table
answer. (People vs. Gonzo, 23 N.E. [2d], 210 [Ill. App., 1939].) A witness
with said men when they were arrested, he refused to answer two
is not relieved from answering merely on his own declaration that an
questions, claiming so to do might tend to incriminate him: (1) "Was there
answer might incriminate him, but rather it is for the trial judge to decide
a game of cards being played on this particular evening at the table at
that question. (Mason vs. U.S., 244 U. S., 362; 61 L. ed., 1193, 1200.)
which you are sitting?" (2) "Was there a game of cards being played at
another table at this time?" The foreman of the grand jury reported the
matter to the judge, who ruled "that each and all of said questions are As against witness's inconsistent and unjustified claim to a constitutional
proper and that the answers thereto would not tend to incriminate the right, is his clear duty as a citizen to give frank, sincere, and truthful
witness." Mason was again called and refused to answer the first question testimony before a competent authority. The state has the right to exact
propounded to him, but, half yielding to frustration, he said in response to fulfillment of a citizen's obligation, consistent of course with his right under
the second question: "I don't know." In affirming the conviction for the Constitution. The witness in this case has been vociferous and militant
contempt, the Supreme Court of the United States among other things in claiming constitutional rights and privileges but patently recreant to his
said: duties and obligations to the Government which protects those rights under
the law. When a specific right and a specific obligation conflict with each
other, and one is doubtful or uncertain while the other is clear and
In the present case, the witness certainly were not relieved from answering
imperative, the former must give way to the latter. The right to life is one
merely because they declared that so to do might incriminate them. The
of the most sacred that the citizen may claim, and yet the state may
wisdom of the rule in this regard is well illustrated by the enforced answer,
deprive him of it if he violates his corresponding obligation to respect the
"I don't know ," given by Mason to the second question, after he had
life of others. As Mr. Justice Johnson said in Anderson vs. Dunn: "The
refused to reply under a claim of constitutional privilege.
wretch beneath the gallows may repine at the fate which awaits him, and
yet it is not certain that the laws under which he suffers were made for the
Since according to the witness himself the transaction was legal, and that security." Paraphrasing and applying that pronouncement here, the
he gave the P440,000 to a representative of Burt in compliance with the petitioner may not relish the restraint of his liberty pending the fulfillment
latter's verbal instruction, we find no basis upon which to sustain his claim by him of his duty, but it is no less certain that the laws under which his
that to reveal the name of that person might incriminate him. There is no liberty is restrained were made for his
conflict of authorities on the applicable rule, to wit:  welfare.chanroblesvirtualawlibrary chanrobles virtual law library
From all the foregoing, it follows that the petition must be denied, and it is [G.R. NO. 169667 : April 20, 2006]
so ordered, with costs.chanroblesvirtualawlibrary chanrobles virtual law
library
ALTERNATIVE LAW GROUPS, INC. (ALG), Petitioner, v. HON.
EDUARDO R. ERMITA, in his capacity as Executive
Paras, Pablo, Bengzon, Montemayor, and Reyes, JJ., concur. Secretary, Respondent.

[G.R. NO. 169834 : April 20, 2006]

PDP - LABAN, Petitioner, v. EXECUTIVE SECRETARY EDUARDO R.


ERMITA, Respondent.

[G.R. NO. 169777* : April 20, 2006] [G.R. NO. 171246 : April 20, 2006]

SENATE OF THE PHILIPPINES, represented by FRANKLIN M. JOSE ANSELMO I. CADIZ, FELICIANO M. BAUTISTA, ROMULO R.
DRILON, in his capacity as Senate President, JUAN M. FLAVIER, in RIVERA, JOSE AMOR AMORANDO, ALICIA A. RISOS-VIDAL,
his capacity as Senate President Pro Tempore, FRANCIS N. FILEMON C. ABELITA III, MANUEL P. LEGASPI, J. B. JOVY C.
PANGILINAN, in his capacity as Majority Leader, AQUILINO Q. BERNABE, BERNARD L. DAGCUTA, ROGELIO V. GARCIA, and the
PIMENTEL, JR., in his capacity as Minority Leader, SENATORS INTEGRATED BAR FOR THE PHILIPPINES, Petitioners, v. HON.
RODOLFO G. BIAZON, "COMPANERA" PIA S. CAYETANO, JINGGOY EXECUTIVE SECRETARY EDUARDO R. ERMITA, Respondent.
EJERCITO ESTRADA, LUISA "LOI" EJERCITO ESTRADA, JUAN PONCE
ENRILE, RICHARD J. GORDON, PANFILO M. LACSON, ALFREDO
S.LIM, M. A. MADRIGAL, SERGIO OSMENA III, RALPH G. RECTO, and DECISION
MAR ROXAS, Petitioners, v.EDUARDO R. ERMITA, in his capacity as
Executive Secretary and alter-ego of President Gloria Macapagal- CARPIO MORALES, J.:
Arroyo, and anyone acting in his stead and in behalf of the
President of the Philippines, Respondents.
A transparent government is one of the hallmarks of a truly republican
state. Even in the early history of republican thought, however, it has been
[G.R. NO. 169659 : April 20, 2006] recognized that the head of government may keep certain information
confidential in pursuit of the public interest. Explaining the reason for
BAYAN MUNA represented by DR. REYNALDO LESACA, JR., Rep. vesting executive power in only one magistrate, a distinguished delegate to
SATUR OCAMPO, Rep. CRISPIN BELTRAN, Rep. RAFAEL MARIANO, the U.S. Constitutional Convention said: "Decision, activity, secrecy, and
Rep. LIZA MAZA, Rep. TEODORO CASINO, Rep. JOEL VIRADOR, dispatch will generally characterize the proceedings of one man, in a much
COURAGE represented by FERDINAND GAITE, and COUNSELS FOR more eminent degree than the proceedings of any greater number; and in
THE DEFENSE OF LIBERTIES (CODAL) represented by ATTY. proportion as the number is increased, these qualities will be diminished."1
REMEDIOS BALBIN, Petitioners, v.EDUARDO ERMITA, in his capacity
as Executive Secretary and alter-ego of President Gloria History has been witness, however, to the fact that the power to withhold
Macapagal-Arroyo, Respondent. information lends itself to abuse, hence, the necessity to guard it zealously.

[G.R. NO. 169660 : April 20, 2006] The present consolidated petitions for certiorari and prohibition proffer that
the President has abused such power by issuing Executive Order No. 464
FRANCISCO I. CHAVEZ, Petitioner, v.EDUARDO R. ERMITA, in his (E.O. 464) last September 28, 2005. They thus pray for its declaration as
capacity as Executive Secretary, AVELINO J. CRUZ, JR., in his null and void for being unconstitutional. 
capacity as Secretary of Defense, and GENEROSO S. SENGA, in his
capacity as AFP Chief of Staff, Respondents.
In resolving the controversy, this Court shall proceed with the recognition Security to Conduct an Inquiry, in Aid of Legislation, and in the National
that the issuance under review has come from a co-equal branch of Interest, on the Role of the Military in the So-called "Gloriagate Scandal";
government, which thus entitles it to a strong presumption of and (5) Senate Resolution No. 295 filed by Senator Biazon - Resolution
constitutionality. Once the challenged order is found to be indeed violative Directing the Committee on National Defense and Security to Conduct an
of the Constitution, it is duty-bound to declare it so. For the Constitution, Inquiry, in Aid of Legislation, on the Wire-Tapping of the President of the
being the highest expression of the sovereign will of the Filipino people, Philippines. 
must prevail over any issuance of the government that contravenes its
mandates. 
Also invited to the above-said hearing scheduled on September 28 2005
was the AFP Chief of Staff, General Generoso S. Senga who, by
In the exercise of its legislative power, the Senate of the Philippines, letter3 dated September 27, 2005, requested for its postponement "due to
through its various Senate Committees, conducts inquiries or investigations a pressing operational situation that demands [his utmost personal
in aid of legislation which call for, inter alia, the attendance of officials and attention" while "some of the invited AFP officers are currently attending to
employees of the executive department, bureaus, and offices including other urgent operational matters."
those employed in Government Owned and Controlled Corporations, the
Armed Forces of the Philippines (AFP), and the Philippine National Police
On September 28, 2005, Senate President Franklin M. Drilon received from
(PNP). 
Executive Secretary Eduardo R. Ermita a letter4 dated September 27, 2005
"respectfully request[ing] for the postponement of the hearing [regarding
On September 21 to 23, 2005, the Committee of the Senate as a whole the NorthRail project] to which various officials of the Executive
issued invitations to various officials of the Executive Department for them Department have been invited" in order to "afford said officials ample time
to appear on September 29, 2005 as resource speakers in a public hearing and opportunity to study and prepare for the various issues so that they
on the railway project of the North Luzon Railways Corporation with the may better enlighten the Senate Committee on its investigation." 
China National Machinery and Equipment Group (hereinafter North Rail
Project). The public hearing was sparked by a privilege speech of Senator
Senate President Drilon, however, wrote5Executive Secretary Ermita that
Juan Ponce Enrile urging the Senate to investigate the alleged overpricing
the Senators "are unable to accede to [his request]" as it "was sent
and other unlawful provisions of the contract covering the North Rail
belatedly" and "[a]ll preparations and arrangements as well as notices to
Project.
all resource persons were completed [the previous] week."

The Senate Committee on National Defense and Security likewise issued


Senate President Drilon likewise received on September 28, 2005 a
invitations2dated September 22, 2005 to the following officials of the AFP:
letter6 from the President of the North Luzon Railways Corporation Jose L.
the Commanding General of the Philippine Army, Lt. Gen. Hermogenes C.
Cortes, Jr. requesting that the hearing on the NorthRail project be
Esperon; Inspector General of the AFP Vice Admiral Mateo M. Mayuga;
postponed or cancelled until a copy of the report of the UP Law Center on
Deputy Chief of Staff for Intelligence of the AFP Rear Admiral Tirso R.
the contract agreements relative to the project had been secured.
Danga; Chief of the Intelligence Service of the AFP Brig. Gen. Marlu Q.
Quevedo; Assistant Superintendent of the Philippine Military Academy
(PMA) Brig. Gen. Francisco V. Gudani; and Assistant Commandant, Corps On September 28, 2005, the President issued E.O. 464, "Ensuring
of Cadets of the PMA, Col. Alexander F. Balutan, for them to attend as Observance of the Principle of Separation of Powers, Adherence to the Rule
resource persons in a public hearing scheduled on September 28, 2005 on on Executive Privilege and Respect for the Rights of Public Officials
the following: (1) Privilege Speech of Senator Aquilino Q. Pimentel Jr., Appearing in Legislative Inquiries in Aid of Legislation Under the
delivered on June 6, 2005 entitled "Bunye has Provided Smoking Gun or Constitution, and For Other Purposes,"7 which, pursuant to Section 6
has Opened a Can of Worms that Show Massive Electoral Fraud in the thereof, took effect immediately. The salient provisions of the Order are as
Presidential Election of May 2005"; (2) Privilege Speech of Senator Jinggoy follows:
E. Estrada delivered on July 26, 2005 entitled "The Philippines as the Wire-
Tapping Capital of the World"; (3) Privilege Speech of Senator Rodolfo SECTION 1. Appearance by Heads of Departments Before Congress. - In
Biazon delivered on August 1, 2005 entitled "Clear and Present Danger"; accordance with Article VI, Section 22 of the Constitution and to implement
(4) Senate Resolution No. 285 filed by Senator Maria Ana Consuelo the Constitutional provisions on the separation of powers between co-equal
Madrigal - Resolution Directing the Committee on National Defense and branches of the government, all heads of departments of the Executive
Branch of the government shall secure the consent of the President prior to Senior officials of executive departments who in the judgment of the
appearing before either House of Congress. department heads are covered by the executive privilege; 

When the security of the State or the public interest so requires and the Generals and flag officers of the Armed Forces of the Philippines and such
President so states in writing, the appearance shall only be conducted in other officers who in the judgment of the Chief of Staff are covered by the
executive session.  executive privilege; 

SECTION. 2. Nature, Scope and Coverage of Executive Privilege.'  Philippine National Police (PNP) officers with rank of chief superintendent or
higher and such other officers who in the judgment of the Chief of the PNP
are covered by the executive privilege; 
(a) Nature and Scope. - The rule of confidentiality based on executive
privilege is fundamental to the operation of government and rooted in the
separation of powers under the Constitution (Almonte v. Vasquez, G.R. No. Senior national security officials who in the judgment of the National
95367, 23 May 1995). Further, Republic Act No. 6713 or the Code of Security Adviser are covered by the executive privilege; andcralawlibrary
Conduct and Ethical Standards for Public Officials and Employees provides
that Public Officials and Employees shall not use or divulge confidential or
Such other officers as may be determined by the President. 
classified information officially known to them by reason of their office and
not made available to the public to prejudice the public interest.
SECTION 3. Appearance of Other Public Officials Before Congress. - All
public officials enumerated in Section 2 (b) hereof shall secure prior
Executive privilege covers all confidential or classified information between
consent of the President prior to appearing before either House of Congress
the President and the public officers covered by this executive order,
to ensure the observance of the principle of separation of powers,
including:
adherence to the rule on executive privilege and respect for the rights of
public officials appearing in inquiries in aid of legislation. (Emphasis and
Conversations and correspondence between the President and the public underscoring supplied)ςrαlαωlιbrαrÿ
official covered by this executive order (Almonte v. Vasquez G.R. No.
95367, 23 May 1995; Chavez v. Public Estates Authority, G.R. No. 133250,
Also on September 28, 2005, Senate President Drilon received from
9 July 2002); 
Executive Secretary Ermita a copy of E.O. 464, and another
letter8 informing him "that officials of the Executive Department invited to
Military, diplomatic and other national security matters which in the appear at the meeting [regarding the NorthRail project] will not be able to
interest of national security should not be divulged (Almonte v. Vasquez, attend the same without the consent of the President, pursuant to [E.O.
G.R. No. 95367, 23 May 1995; Chavez v. Presidential Commission on Good 464]" and that "said officials have not secured the required consent from
Government, G.R. No. 130716, 9 December 1998).  the President." On even date which was also the scheduled date of the
hearing on the alleged wiretapping, Gen. Senga sent a letter9 to Senator
Biazon, Chairperson of the Committee on National Defense and Security,
Information between inter-government agencies prior to the conclusion of
informing him "that per instruction of [President Arroyo], thru the
treaties and executive agreements (Chavez v. Presidential Commission on
Secretary of National Defense, no officer of the [AFP] is authorized to
Good Government, G.R. No. 130716, 9 December 1998); 
appear before any Senate or Congressional hearings without seeking a
written approval from the President" and "that no approval has been
Discussion in close-door Cabinet meetings (Chavez v. Presidential granted by the President to any AFP officer to appear before the public
Commission on Good Government, G.R. No. 130716, 9 December 1998);  hearing of the Senate Committee on National Defense and Security
scheduled [on] 28 September 2005."
Matters affecting national security and public order (Chavez v. Public
Estates Authority, G.R. No. 133250, 9 July 2002).  Despite the communications received from Executive Secretary Ermita and
Gen. Senga, the investigation scheduled by the Committee on National
(b) Who are covered. - The following are covered by this executive order:
Defense and Security pushed through, with only Col. Balutan and Brig. E.O. 464 should they be summoned by Congress; and CODAL alleges that
Gen. Gudani among all the AFP officials invited attending. its members have a sworn duty to uphold the rule of law, and their rights
to information and to transparent governance are threatened by the
imposition of E.O. 464.
For defying President Arroyo's order barring military personnel from
testifying before legislative inquiries without her approval, Brig. Gen.
Gudani and Col. Balutan were relieved from their military posts and were In G.R. No. 169660, petitioner Francisco I. Chavez, claiming that his
made to face court martial proceedings.  constitutional rights as a citizen, taxpayer and law practitioner, are affected
by the enforcement of E.O. 464, prays in his petition that E.O. 464 be
declared null and void for being unconstitutional. 
As to the NorthRail project hearing scheduled on September 29, 2005,
Executive Secretary Ermita, citing E.O. 464, sent letter of regrets, in
response to the invitations sent to the following government officials: Light In G.R. No. 169667, petitioner Alternative Law Groups, Inc.12 (ALG),
Railway Transit Authority Administrator Melquiades Robles, Metro Rail alleging that as a coalition of 17 legal resource non-governmental
Transit Authority Administrator Roberto Lastimoso, Department of Justice organizations engaged in developmental lawyering and work with the poor
(DOJ) Chief State Counsel Ricardo V. Perez, then Presidential Legal Counsel and marginalized sectors in different parts of the country, and as an
Merceditas Gutierrez, Department of Transportation and Communication organization of citizens of the Philippines and a part of the general public, it
(DOTC) Undersecretary Guiling Mamonding, DOTC Secretary Leandro has legal standing to institute the petition to enforce its constitutional right
Mendoza, Philippine National Railways General Manager Jose Serase II, to information on matters of public concern, a right which was denied to
Monetary Board Member Juanita Amatong, Bases Conversion Development the public by E.O. 464,13 prays, that said order be declared null and void
Authority Chairperson Gen. Narciso Abaya and Secretary Romulo L. for being unconstitutional and that respondent Executive Secretary Ermita
Neri.10 NorthRail President Cortes sent personal regrets likewise citing E.O. be ordered to cease from implementing it. 
464.11
On October 11, 2005, Petitioner Senate of the Philippines, alleging that it
On October 3, 2005, three petitions, docketed as G.R. NOS. 169659, has a vital interest in the resolution of the issue of the validity of E.O. 464
169660, and 169667, for certiorari and prohibition, were filed before this for it stands to suffer imminent and material injury, as it has already
Court challenging the constitutionality of E.O. 464.  sustained the same with its continued enforcement since it directly
interferes with and impedes the valid exercise of the Senate's powers and
functions and conceals information of great public interest and concern,
In G.R. No. 169659, petitioners party-list Bayan Muna, House of
filed its petition for certiorari and prohibition, docketed as G.R. No. 169777
Representatives Members Satur Ocampo, Crispin Beltran, Rafael Mariano,
and prays that E.O. 464 be declared unconstitutional. 
Liza Maza, Joel Virador and Teodoro Casino, Courage, an organization of
government employees, and Counsels for the Defense of Liberties
(CODAL), a group of lawyers dedicated to the promotion of justice, On October 14, 2005, PDP-Laban, a registered political party with members
democracy and peace, all claiming to have standing to file the suit because duly elected into the Philippine Senate and House of Representatives, filed
of the transcendental importance of the issues they posed, pray, in their a similar petition for certiorari and prohibition, docketed as G.R. No.
petition that E.O. 464 be declared null and void for being unconstitutional; 169834, alleging that it is affected by the challenged E.O. 464 because it
that respondent Executive Secretary Ermita, in his capacity as Executive hampers its legislative agenda to be implemented through its members in
Secretary and alter-ego of President Arroyo, be prohibited from imposing, Congress, particularly in the conduct of inquiries in aid of legislation and
and threatening to impose sanctions on officials who appear before transcendental issues need to be resolved to avert a constitutional crisis
Congress due to congressional summons. Additionally, petitioners claim between the executive and legislative branches of the government.
that E.O. 464 infringes on their rights and impedes them from fulfilling
their respective obligations. Thus, Bayan Muna alleges that E.O. 464
Meanwhile, by letter14 dated February 6, 2006, Senator Biazon reiterated
infringes on its right as a political party entitled to participate in
his invitation to Gen. Senga for him and other military officers to attend
governance; Satur Ocampo, et al. allege that E.O. 464 infringes on their
the hearing on the alleged wiretapping scheduled on February 10, 2005.
rights and duties as members of Congress to conduct investigation in aid of
Gen. Senga replied, however, by letter15dated February 8, 2006, that
legislation and conduct oversight functions in the implementation of laws;
"[p]ursuant to Executive Order No. 464, th[e] Headquarters requested for
Courage alleges that the tenure of its members in public office is
a clearance from the President to allow [them] to appear before the public
predicated on, and threatened by, their submission to the requirements of
hearing" and that "they will attend once [their] request is approved by the calls for judicial review was not taken up; instead, the parties were
President." As none of those invited appeared, the hearing on February 10, instructed to discuss it in their respective memoranda. 
2006 was cancelled.16
After the conclusion of the oral arguments, the parties were directed to
In another investigation conducted jointly by the Senate Committee on submit their respective memoranda, paying particular attention to the
Agriculture and Food and the Blue Ribbon Committee on the alleged following propositions: (1) that E.O. 464 is, on its face, unconstitutional;
mismanagement and use of the fertilizer fund under the Ginintuang and (2) assuming that it is not, it is unconstitutional as applied in four
Masaganang Ani program of the Department of Agriculture (DA), several instances, namely: (a) the so called Fertilizer scam; (b) the NorthRail
Cabinet officials were invited to the hearings scheduled on October 5 and investigation (c) the Wiretapping activity of the ISAFP; and (d) the
26, November 24 and December 12, 2005 but most of them failed to investigation on the Venable contract.22
attend, DA Undersecretary Belinda Gonzales, DA Assistant Secretary Felix
Jose Montes, Fertilizer and Pesticide Authority Executive Director Norlito R.
Petitioners in G.R. No. 16966023 and G.R. No. 16977724 filed their
Gicana,17 and those from the Department of Budget and
memoranda on March 7, 2006, while those in G.R. No. 16966725 and G.R.
Management18 having invoked E.O. 464.
No. 16983426 filed theirs the next day or on March 8, 2006. Petitioners in
G.R. No. 171246 did not file any memorandum.
In the budget hearings set by the Senate on February 8 and 13, 2006,
Press Secretary and Presidential Spokesperson Ignacio R. Bunye,19 DOJ
Petitioners Bayan Muna et al. in G.R. No. 169659, after their motion for
Secretary Raul M. Gonzalez20and Department of Interior and Local
extension to file memorandum27 was granted, subsequently filed a
Government Undersecretary Marius P. Corpus21 communicated their
manifestation28 dated March 14, 2006 that it would no longer file its
inability to attend due to lack of appropriate clearance from the President
memorandum in the interest of having the issues resolved soonest,
pursuant to E.O. 464. During the February 13, 2005 budget hearing,
prompting this Court to issue a Resolution reprimanding them.29
however, Secretary Bunye was allowed to attend by Executive Secretary
Ermita.
Petitioners submit that E.O. 464 violates the following constitutional
provisions:
On February 13, 2006, Jose Anselmo I. Cadiz and the incumbent members
of the Board of Governors of the Integrated Bar of the Philippines, as
taxpayers, and the Integrated Bar of the Philippines as the official Art. VI, Sec. 2130
organization of all Philippine lawyers, all invoking their constitutional right
to be informed on matters of public interest, filed their petition Art. VI, Sec. 2231
for certiorari and prohibition, docketed as G.R. No. 171246, and pray that
E.O. 464 be declared null and void.
Art. VI, Sec. 132

All the petitions pray for the issuance of a Temporary Restraining Order
enjoining respondents from implementing, enforcing, and observing E.O. Art. XI, Sec. 133
464.
Art. III, Sec. 734
In the oral arguments on the petitions conducted on February 21, 2006,
the following substantive issues were ventilated: (1) whether respondents Art. III, Sec. 435
committed grave abuse of discretion in implementing E.O. 464 prior to its
publication in the Official Gazette or in a newspaper of general circulation;
and (2) whether E.O. 464 violates the following provisions of the Art. XIII, Sec. 16 36
Constitution: Art. II, Sec. 28, Art. III, Sec. 4, Art. III, Sec. 7, Art. IV. Sec.
1, Art. VI, Sec. 21, Art. VI, Sec. 22, Art. XI, Sec. 1, and Art. XIII, Sec. 16. Art. II, Sec. 2837
The procedural issue of whether there is an actual case or controversy that
Respondents Executive Secretary Ermita et al., on the other hand, pray in oversight functions. They maintain that Representatives Ocampo et al.
their consolidated memorandum38 on March 13, 2006 for the dismissal of have not shown any specific prerogative, power, and privilege of the House
the petitions for lack of merit.  of Representatives which had been effectively impaired by E.O. 464, there
being no mention of any investigation called by the House of
Representatives or any of its committees which was aborted due to the
The Court synthesizes the issues to be resolved as follows:
implementation of E.O. 464. 

1. Whether E.O. 464 contravenes the power of inquiry vested in Congress; 


As for Bayan Muna's alleged interest as a party-list representing the
marginalized and underrepresented, and that of the other petitioner groups
2. Whether E.O. 464 violates the right of the people to information on and individuals who profess to have standing as advocates and defenders
matters of public concern; andcralawlibrary of the Constitution, respondents contend that such interest falls short of
that required to confer standing on them as parties "injured-in-fact."40
3. Whether respondents have committed grave abuse of discretion when
they implemented E.O. 464 prior to its publication in a newspaper of Respecting petitioner Chavez, respondents contend that Chavez may not
general circulation. claim an interest as a taxpayer for the implementation of E.O. 464 does not
involve the exercise of taxing or spending power.41
Essential requisites for judicial review
With regard to the petition filed by the Senate, respondents argue that in
Before proceeding to resolve the issue of the constitutionality of E.O. 464, the absence of a personal or direct injury by reason of the issuance of E.O.
ascertainment of whether the requisites for a valid exercise of the Court's 464, the Senate and its individual members are not the proper parties to
power of judicial review are present is in order. assail the constitutionality of E.O. 464. 

Like almost all powers conferred by the Constitution, the power of judicial Invoking this Court's ruling in National Economic Protectionism Association
review is subject to limitations, to wit: (1) there must be an actual case or v. Ongpin42 and Valmonte v. Philippine Charity Sweepstakes
controversy calling for the exercise of judicial power; (2) the person Office,43 respondents assert that to be considered a proper party, one must
challenging the act must have standing to challenge the validity of the have a personal and substantial interest in the case, such that he has
subject act or issuance; otherwise stated, he must have a personal and sustained or will sustain direct injury due to the enforcement of E.O. 464.44
substantial interest in the case such that he has sustained, or will sustain,
direct injury as a result of its enforcement; (3) the question of That the Senate of the Philippines has a fundamental right essential not
constitutionality must be raised at the earliest opportunity; and (4) the only for intelligent public decision-making in a democratic system, but
issue of constitutionality must be the very lis mota of the case.39 more especially for sound legislation45 is not disputed. E.O. 464, however,
allegedly stifles the ability of the members of Congress to access
Except with respect to the requisites of standing and existence of an actual information that is crucial to law-making.46 Verily, the Senate, including its
case or controversy where the disagreement between the parties lies, individual members, has a substantial and direct interest over the outcome
discussion of the rest of the requisites shall be omitted. of the controversy and is the proper party to assail the constitutionality of
E.O. 464. Indeed, legislators have standing to maintain inviolate the
prerogative, powers and privileges vested by the Constitution in their office
Standing and are allowed to sue to question the validity of any official action which
they claim infringes their prerogatives as legislators.47
Respondents, through the Solicitor General, assert that the allegations in
G.R. NOS. 169659, 169660 and 169667 make it clear that they, adverting In the same vein, party-list representatives Satur Ocampo (Bayan Muna),
to the non-appearance of several officials of the executive department in Teodoro Casino (Bayan Muna), Joel Virador (Bayan Muna), Crispin Beltran
the investigations called by the different committees of the Senate, were (Anakpawis), Rafael Mariano (Anakpawis), and Liza Maza (Gabriela) are
brought to vindicate the constitutional duty of the Senate or its different allowed to sue to question the constitutionality of E.O. 464, the absence of
committees to conduct inquiry in aid of legislation or in the exercise of its any claim that an investigation called by the House of Representatives or
any of its committees was aborted due to the implementation of E.O. 464 public funds or assets are involved and petitioners in G.R. NOS. 169777
notwithstanding, it being sufficient that a claim is made that E.O. 464 and 169659 have direct and specific interests in the resolution of the
infringes on their constitutional rights and duties as members of Congress controversy, petitioner PDP-Laban is bereft of standing to file its petition.
to conduct investigation in aid of legislation and conduct oversight Its allegation that E.O. 464 hampers its legislative agenda is vague and
functions in the implementation of laws.  uncertain, and at best is only a "generalized interest" which it shares with
the rest of the political parties. Concrete injury, whether actual or
threatened, is that indispensable element of a dispute which serves in part
The national political party, Bayan Muna, likewise meets the standing
to cast it in a form traditionally capable of judicial resolution.55 In fine, PDP-
requirement as it obtained three seats in the House of Representatives in
Laban's alleged interest as a political party does not suffice to clothe it with
the 2004 elections and is, therefore, entitled to participate in the legislative
legal standing.
process consonant with the declared policy underlying the party list system
of affording citizens belonging to marginalized and underrepresented
sectors, organizations and parties who lack well-defined political Actual Case or Controversy
constituencies to contribute to the formulation and enactment of legislation
that will benefit the nation.48
Petitioners assert that an actual case exists, they citing the absence of the
executive officials invited by the Senate to its hearings after the issuance of
As Bayan Muna and Representatives Ocampo et al. have the standing to E.O. 464, particularly those on the NorthRail project and the wiretapping
file their petitions, passing on the standing of their co-petitioners Courage controversy. 
and Codal is rendered unnecessary.49
Respondents counter that there is no case or controversy, there being no
In filing their respective petitions, Chavez, the ALG which claims to be an showing that President Arroyo has actually withheld her consent or
organization of citizens, and the incumbent members of the IBP Board of prohibited the appearance of the invited officials.56 These officials, they
Governors and the IBP in behalf of its lawyer members,50 invoke their claim, merely communicated to the Senate that they have not yet secured
constitutional right to information on matters of public concern, asserting the consent of the President, not that the President prohibited their
that the right to information, curtailed and violated by E.O. 464, is attendance.57 Specifically with regard to the AFP officers who did not attend
essential to the effective exercise of other constitutional rights51 and to the the hearing on September 28, 2005, respondents claim that the instruction
maintenance of the balance of power among the three branches of the not to attend without the President's consent was based on its role as
government through the principle of checks and balances.52 Commander-in-Chief of the Armed Forces, not on E.O. 464.

It is well-settled that when suing as a citizen, the interest of the petitioner Respondents thus conclude that the petitions merely rest on an unfounded
in assailing the constitutionality of laws, presidential decrees, orders, and apprehension that the President will abuse its power of preventing the
other regulations, must be direct and personal. In Franciso v. House of appearance of officials before Congress, and that such apprehension is not
Representatives,53 this Court held that when the proceeding involves the sufficient for challenging the validity of E.O. 464. 
assertion of a public right, the mere fact that he is a citizen satisfies the
requirement of personal interest.
The Court finds respondents' assertion that the President has not withheld
her consent or prohibited the appearance of the officials concerned
As for petitioner PDP-Laban, it asseverates that it is clothed with legal immaterial in determining the existence of an actual case or controversy
standing in view of the transcendental issues raised in its petition which insofar as E.O. 464 is concerned. For E.O. 464 does not require either a
this Court needs to resolve in order to avert a constitutional crisis. For it to deliberate withholding of consent or an express prohibition issuing from the
be accorded standing on the ground of transcendental importance, President in order to bar officials from appearing before Congress. 
however, it must establish (1) the character of the funds (that it is public)
or other assets involved in the case, (2) the presence of a clear case of
As the implementation of the challenged order has already resulted in the
disregard of a constitutional or statutory prohibition by the public
absence of officials invited to the hearings of petitioner Senate of the
respondent agency or instrumentality of the government, and (3) the lack
Philippines, it would make no sense to wait for any further event before
of any party with a more direct and specific interest in raising the questions
considering the present case ripe for adjudication. Indeed, it would be
being raised.54 The first and last determinants not being present as no
sheer abandonment of duty if this Court would now refrain from passing on effectively, such power is so far incidental to the legislative function as to
the constitutionality of E.O. 464.  be implied. In other words, the power of inquiry - with process to enforce it
- is an essential and appropriate auxiliary to the legislative function. A
legislative body cannot legislate wisely or effectively in the absence of
Constitutionality of E.O. 464 
information respecting the conditions which the legislation is intended to
affect or change; and where the legislative body does not itself possess the
E.O. 464, to the extent that it bars the appearance of executive officials requisite information - which is not infrequently true - recourse must be
before Congress, deprives Congress of the information in the possession of had to others who do possess it. Experience has shown that mere requests
these officials. To resolve the question of whether such withholding of for such information are often unavailing, and also that information which
information violates the Constitution, consideration of the general power of is volunteered is not always accurate or complete; so some means of
Congress to obtain information, otherwise known as the power of inquiry, compulsion is essential to obtain what is needed.59 . . . (Emphasis and
is in order. underscoring supplied)ςrαlαωlιbrαrÿ

The power of inquiry That this power of inquiry is broad enough to cover officials of the
executive branch may be deduced from the same case. The power of
The Congress power of inquiry is expressly recognized in Section 21 of inquiry, the Court therein ruled, is co-extensive with the power to
Article VI of the Constitution which reads: legislate.60 The matters which may be a proper subject of legislation and
those which may be a proper subject of investigation are one. It follows
that the operation of government, being a legitimate subject for legislation,
SECTION 21. The Senate or the House of Representatives or any of its is a proper subject for investigation. 
respective committees may conduct inquiries in aid of legislation in
accordance with its duly published rules of procedure. The rights of persons
appearing in or affected by such inquiries shall be respected. (Underscoring Thus, the Court found that the Senate investigation of the government
supplied)ςrαlαωlιbrαrÿ transaction involved in Arnault was a proper exercise of the power of
inquiry. Besides being related to the expenditure of public funds of which
Congress is the guardian, the transaction, the Court held, "also involved
This provision is worded exactly as Section 8 of Article VIII of the 1973 government agencies created by Congress and officers whose positions it is
Constitution except that, in the latter, it vests the power of inquiry in the within the power of Congress to regulate or even abolish." 
unicameral legislature established therein - the Batasang Pambansa - and
its committees. 
Since Congress has authority to inquire into the operations of the executive
branch, it would be incongruous to hold that the power of inquiry does not
The 1935 Constitution did not contain a similar provision. Nonetheless, in extend to executive officials who are the most familiar with and informed
Arnault v. Nazareno,58 a case decided in 1950 under that Constitution, the on executive operations. 
Court already recognized that the power of inquiry is inherent in the power
to legislate.
As discussed in Arnault, the power of inquiry, "with process to enforce it,"
is grounded on the necessity of information in the legislative process. If the
Arnault involved a Senate investigation of the reportedly anomalous information possessed by executive officials on the operation of their
purchase of the Buenavista and Tambobong Estates by the Rural Progress offices is necessary for wise legislation on that subject, by parity of
Administration. Arnault, who was considered a leading witness in the reasoning, Congress has the right to that information and the power to
controversy, was called to testify thereon by the Senate. On account of his compel the disclosure thereof.
refusal to answer the questions of the senators on an important point, he
was, by resolution of the Senate, detained for contempt. Upholding the
Senate's power to punish Arnault for contempt, this Court held: As evidenced by the American experience during the so-called "McCarthy
era," however, the right of Congress to conduct inquiries in aid of
legislation is, in theory, no less susceptible to abuse than executive or
Although there is no provision in the Constitution expressly investing either judicial power. It may thus be subjected to judicial review pursuant to the
House of Congress with power to make investigations and exact testimony Court's certioraripowers under Section 1, Article VIII of the Constitution.
to the end that it may exercise its legislative functions advisedly and
For one, as noted in Bengzon v. Senate Blue Ribbon Committee,61 the Schwartz defines executive privilege as "the power of the Government to
inquiry itself might not properly be in aid of legislation, and thus beyond withhold information from the public, the courts, and the
the constitutional power of Congress. Such inquiry could not usurp judicial Congress."64 Similarly, Rozell defines it as "the right of the President and
functions. Parenthetically, one possible way for Congress to avoid such a high-level executive branch officers to withhold information from Congress,
result as occurred in Bengzon is to indicate in its invitations to the public the courts, and ultimately the public."65
officials concerned, or to any person for that matter, the possible needed
statute which prompted the need for the inquiry. Given such statement in
Executive privilege is, nonetheless, not a clear or unitary concept.66 It has
its invitations, along with the usual indication of the subject of inquiry and
encompassed claims of varying kinds.67 Tribe, in fact, comments that while
the questions relative to and in furtherance thereof, there would be less
it is customary to employ the phrase "executive privilege," it may be more
room for speculation on the part of the person invited on whether the
accurate to speak of executive privileges "since presidential refusals to
inquiry is in aid of legislation. 
furnish information may be actuated by any of at least three distinct kinds
of considerations, and may be asserted, with differing degrees of success,
Section 21, Article VI likewise establishes crucial safeguards that proscribe in the context of either judicial or legislative investigations." 
the legislative power of inquiry. The provision requires that the inquiry be
done in accordance with the Senate or House's duly published rules of
One variety of the privilege, Tribe explains, is the state secrets privilege
procedure, necessarily implying the constitutional infirmity of an inquiry
invoked by U.S. Presidents, beginning with Washington, on the ground that
conducted without duly published rules of procedure. Section 21 also
the information is of such nature that its disclosure would subvert crucial
mandates that the rights of persons appearing in or affected by such
military or diplomatic objectives. Another variety is the informer's privilege,
inquiries be respected, an imposition that obligates Congress to adhere to
or the privilege of the Government not to disclose the identity of persons
the guarantees in the Bill of Rights.
who furnish information of violations of law to officers charged with the
enforcement of that law. Finally, a generic privilege for internal
These abuses are, of course, remediable before the courts, upon the proper deliberations has been said to attach to intragovernmental documents
suit filed by the persons affected, even if they belong to the executive reflecting advisory opinions, recommendations and deliberations
branch. Nonetheless, there may be exceptional circumstances, none comprising part of a process by which governmental decisions and policies
appearing to obtain at present, wherein a clear pattern of abuse of the are formulated.68
legislative power of inquiry might be established, resulting in palpable
violations of the rights guaranteed to members of the executive
Tribe's comment is supported by the ruling in In re Sealed Case, thus:
department under the Bill of Rights. In such instances, depending on the
particulars of each case, attempts by the Executive Branch to forestall
these abuses may be accorded judicial sanction. Since the beginnings of our nation, executive officials have claimed a
variety of privileges to resist disclosure of information the confidentiality of
which they felt was crucial to fulfillment of the unique role and
Even where the inquiry is in aid of legislation, there are still recognized
responsibilities of the executive branch of our government. Courts ruled
exemptions to the power of inquiry, which exemptions fall under the rubric
early that the executive had a right to withhold documents that might
of "executive privilege." Since this term figures prominently in the
reveal military or state secrets. The courts have also granted the executive
challenged order, it being mentioned in its provisions, its preambular
a right to withhold the identity of government informers in some
clauses,62 and in its very title, a discussion of executive privilege is crucial
circumstances and a qualified right to withhold information related to
for determining the constitutionality of E.O. 464. 
pending investigations. x x x"69 (Emphasis and underscoring
supplied)ςrαlαωlιbrαrÿ
Executive privilege
The entry in Black's Law Dictionary on "executive privilege" is similarly
The phrase "executive privilege" is not new in this jurisdiction. It has been instructive regarding the scope of the doctrine.
used even prior to the promulgation of the 1986 Constitution.63 Being of
American origin, it is best understood in light of how it has been defined
This privilege, based on the constitutional doctrine of separation of powers,
and used in the legal literature of the United States.
exempts the executive from disclosure requirements applicable to the
ordinary citizen or organization where such exemption is necessary to the
discharge of highly important executive responsibilities involved in In this jurisdiction, the doctrine of executive privilege was recognized by
maintaining governmental operations, and extends not only to military and this Court in Almonte v. Vasquez.77 Almonte used the term in reference to
diplomatic secrets but also to documents integral to an appropriate the same privilege subject of Nixon. It quoted the following portion of the
exercise of the executive' domestic decisional and policy making functions, Nixon decision which explains the basis for the privilege: 
that is, those documents reflecting the frank expression necessary in intra-
governmental advisory and deliberative communications.70 (Emphasis and
"The expectation of a President to the confidentiality of his conversations
underscoring supplied)ςrαlαωlιbrαrÿ
and correspondences, like the claim of confidentiality of judicial
deliberations, for example, has all the values to which we accord deference
That a type of information is recognized as privileged does not, however, for the privacy of all citizens and, added to those values, is the necessity
necessarily mean that it would be considered privileged in all instances. For for protection of the public interest in candid, objective, and even blunt or
in determining the validity of a claim of privilege, the question that must be harsh opinions in Presidential decision-making. A President and those who
asked is not only whether the requested information falls within one of the assist him must be free to explore alternatives in the process of shaping
traditional privileges, but also whether that privilege should be honored in policies and making decisions and to do so in a way many would be
a given procedural setting.71 unwilling to express except privately. These are the considerations
justifying a presumptive privilege for Presidential communications. The
privilege is fundamental to the operation of government and inextricably
The leading case on executive privilege in the United States is U.S. v.
rooted in the separation of powers under the Constitution x x x "
Nixon, 72 decided in 1974. In issue in that case was the validity of President
(Emphasis and underscoring supplied)ςrαlαωlιbrαrÿ
Nixon's claim of executive privilege against a subpoena issued by a district
court requiring the production of certain tapes and documents relating to
the Watergate investigations. The claim of privilege was based on the Almonte involved a subpoena duces tecum issued by the Ombudsman
President's general interest in the confidentiality of his conversations and against the therein petitioners. It did not involve, as expressly stated in the
correspondence. The U.S. Court held that while there is no explicit decision, the right of the people to information.78 Nonetheless, the Court
reference to a privilege of confidentiality in the U.S. Constitution, it is recognized that there are certain types of information which the
constitutionally based to the extent that it relates to the effective discharge government may withhold from the public, thus acknowledging, in
of a President's powers. The Court, nonetheless, rejected the President's substance if not in name, that executive privilege may be claimed against
claim of privilege, ruling that the privilege must be balanced against the citizens' demands for information.
public interest in the fair administration of criminal justice. Notably, the
Court was careful to clarify that it was not there addressing the issue of
In Chavez v. PCGG,79 the Court held that this jurisdiction recognizes the
claims of privilege in a civil litigation or against congressional demands for
common law holding that there is a "governmental privilege against public
information. 
disclosure with respect to state secrets regarding military, diplomatic and
other national security matters."80 The same case held that closed-door
Cases in the U.S. which involve claims of executive privilege against Cabinet meetings are also a recognized limitation on the right to
Congress are rare.73 Despite frequent assertion of the privilege to deny information. 
information to Congress, beginning with President Washington's refusal to
turn over treaty negotiation records to the House of Representatives, the
Similarly, in Chavez v. Public Estates Authority,81 the Court ruled that the
U.S. Supreme Court has never adjudicated the issue.74However, the U.S.
right to information does not extend to matters recognized as "privileged
Court of Appeals for the District of Columbia Circuit, in a case decided
information under the separation of powers,"82 by which the Court meant
earlier in the same year as Nixon, recognized the President's privilege over
Presidential conversations, correspondences, and discussions in closed-
his conversations against a congressional subpoena.75 Anticipating the
door Cabinet meetings. It also held that information on military and
balancing approach adopted by the U.S. Supreme Court in Nixon, the Court
diplomatic secrets and those affecting national security, and information on
of Appeals weighed the public interest protected by the claim of privilege
investigations of crimes by law enforcement agencies before the
against the interest that would be served by disclosure to the Committee.
prosecution of the accused were exempted from the right to information. 
Ruling that the balance favored the President, the Court declined to enforce
the subpoena.76
From the above discussion on the meaning and scope of executive
privilege, both in the United States and in this jurisdiction, a clear principle
emerges. Executive privilege, whether asserted against Congress, the the power of either House of Congress to "conduct inquiries in aid of
courts, or the public, is recognized only in relation to certain types of legislation." As the following excerpt of the deliberations of the
information of a sensitive character. While executive privilege is a Constitutional Commission shows, the framers were aware that these two
constitutional concept, a claim thereof may be valid or not depending on provisions involved distinct functions of Congress.
the ground invoked to justify it and the context in which it is made.
Noticeably absent is any recognition that executive officials are exempt
MR. MAAMBONG. x x x When we amended Section 20 [now Section 22 on
from the duty to disclose information by the mere fact of being executive
the Question Hour] yesterday, I noticed that members of the Cabinet
officials. Indeed, the extraordinary character of the exemptions indicates
cannot be compelled anymore to appear before the House of
that the presumption inclines heavily against executive secrecy and in
Representatives or before the Senate. I have a particular problem in this
favor of disclosure. 
regard, Madam President, because in our experience in the Regular
Batasang Pambansa - as the Gentleman himself has experienced in the
Validity of Section 1 interim Batasang Pambansa - one of the most competent inputs that we
can put in our committee deliberations, either in aid of legislation or in
congressional investigations, is the testimonies of Cabinet ministers. We
Section 1 is similar to Section 3 in that both require the officials covered by
usually invite them, but if they do not come and it is a congressional
them to secure the consent of the President prior to appearing before
investigation, we usually issue subpoenas.
Congress. There are significant differences between the two provisions,
however, which constrain this Court to discuss the validity of these
provisions separately.  I want to be clarified on a statement made by Commissioner Suarez when
he said that the fact that the Cabinet ministers may refuse to come to the
House of Representatives or the Senate [when requested under Section
Section 1 specifically applies to department heads. It does not, unlike
22] does not mean that they need not come when they are invited or
Section 3, require a prior determination by any official whether they are
subpoenaed by the committee of either House when it comes to inquiries in
covered by E.O. 464. The President herself has, through the challenged
aid of legislation or congressional investigation. According to Commissioner
order, made the determination that they are. Further, unlike also Section
Suarez, that is allowed and their presence can be had under Section 21.
3, the coverage of department heads under Section 1 is not made to
Does the gentleman confirm this, Madam President?cralawlibrary
depend on the department heads' possession of any information which
might be covered by executive privilege. In fact, in marked contrast to
Section 3 vis - à-vis Section 2, there is no reference to executive privilege MR. DAVIDE. We confirm that, Madam President, because Section 20 refers
at all. Rather, the required prior consent under Section 1 is grounded on only to what was originally the Question Hour, whereas, Section 21 would
Article VI, Section 22 of the Constitution on what has been referred to as refer specifically to inquiries in aid of legislation, under which anybody for
the question hour.  that matter, may be summoned and if he refuses, he can be held in
contempt of the House.83 (Emphasis and underscoring
supplied)ςrαlαωlιbrαrÿ
SECTION 22. The heads of departments may upon their own initiative, with
the consent of the President, or upon the request of either House, as the
rules of each House shall provide, appear before and be heard by such A distinction was thus made between inquiries in aid of legislation and the
House on any matter pertaining to their departments. Written questions question hour. While attendance was meant to be discretionary in the
shall be submitted to the President of the Senate or the Speaker of the question hour, it was compulsory in inquiries in aid of legislation. The
House of Representatives at least three days before their scheduled reference to Commissioner Suarez bears noting, he being one of the
appearance. Interpellations shall not be limited to written questions, but proponents of the amendment to make the appearance of department
may cover matters related thereto. When the security of the State or the heads discretionary in the question hour. 
public interest so requires and the President so states in writing, the
appearance shall be conducted in executive session. 
So clearly was this distinction conveyed to the members of the Commission
that the Committee on Style, precisely in recognition of this distinction,
Determining the validity of Section 1 thus requires an examination of the later moved the provision on question hour from its original position as
meaning of Section 22 of Article VI. Section 22 which provides for the Section 20 in the original draft down to Section 31, far from the provision
question hour must be interpreted vis - à-vis Section 21 which provides for
on inquiries in aid of legislation. This gave rise to the following exchange assumption that these provisions pertained to two different functions of the
during the deliberations:  legislature. Both Commissioners understood that the power to conduct
inquiries in aid of legislation is different from the power to conduct inquiries
during the question hour. Commissioner Davide's only concern was that the
MR. GUINGONA. [speaking in his capacity as Chairman of the Committee
two provisions on these distinct powers be placed closely together, they
on Style] We now go, Mr. Presiding Officer, to the Article on Legislative and
being complementary to each other. Neither Commissioner considered
may I request the chairperson of the Legislative Department,
them as identical functions of Congress. 
Commissioner Davide, to give his reaction.

The foregoing opinion was not the two Commissioners' alone. From the
THE PRESIDING OFFICER (Mr. Jamir). Commissioner Davide is
above-quoted exchange, Commissioner Maambong's committee - the
recognized.ςηαñrοblεš νιr†υαl lαω lιbrαrÿ
Committee on Style - shared the view that the two provisions reflected
distinct functions of Congress. Commissioner Davide, on the other hand,
MR. DAVIDE. Thank you, Mr. Presiding Officer. I have only one reaction to was speaking in his capacity as Chairman of the Committee on the
the Question Hour. I propose that instead of putting it as Section 31, it Legislative Department. His views may thus be presumed as representing
should follow Legislative Inquiries. that of his Committee. 

THE PRESIDING OFFICER. What does the committee say?cralawlibrary In the context of a parliamentary system of government, the "question
hour" has a definite meaning. It is a period of confrontation initiated by
MR. GUINGONA. I ask Commissioner Maambong to reply, Mr. Presiding Parliament to hold the Prime Minister and the other ministers accountable
Officer. for their acts and the operation of the government,85 corresponding to what
is known in Britain as the question period. There was a specific provision
for a question hour in the 1973 Constitution86 which made the appearance
MR. MAAMBONG. Actually, we considered that previously when we of ministers mandatory. The same perfectly conformed to the
sequenced this but we reasoned that in Section 21, which is Legislative parliamentary system established by that Constitution, where the ministers
Inquiry, it is actually a power of Congress in terms of its own lawmaking; are also members of the legislature and are directly accountable to it. 
whereas, a Question Hour is not actually a power in terms of its own
lawmaking power because in Legislative Inquiry, it is in aid of legislation.
And so we put Question Hour as Section 31. I hope Commissioner Davide An essential feature of the parliamentary system of government is the
will consider this. immediate accountability of the Prime Minister and the Cabinet to the
National Assembly. They shall be responsible to the National Assembly for
the program of government and shall determine the guidelines of national
MR. DAVIDE. The Question Hour is closely related with the legislative policy. Unlike in the presidential system where the tenure of office of all
power, and it is precisely as a complement to or a supplement of the elected officials cannot be terminated before their term expired, the Prime
Legislative Inquiry. The appearance of the members of Cabinet would be Minister and the Cabinet remain in office only as long as they enjoy the
very, very essential not only in the application of check and balance but confidence of the National Assembly. The moment this confidence is lost
also, in effect, in aid of legislation. the Prime Minister and the Cabinet may be changed.87

MR. MAAMBONG. After conferring with the committee, we find merit in the The framers of the 1987 Constitution removed the mandatory nature of
suggestion of Commissioner Davide. In other words, we are accepting that such appearance during the question hour in the present Constitution so as
and so this Section 31 would now become Section 22. Would it be, to conform more fully to a system of separation of powers.88To that extent,
Commissioner Davide?cralawlibrary the question hour, as it is presently understood in this jurisdiction, departs
from the question period of the parliamentary system. That department
MR. DAVIDE. Yes.84 (Emphasis and underscoring supplied)ςrαlαωlιbrαrÿ heads may not be required to appear in a question hour does not,
however, mean that the legislature is rendered powerless to elicit
information from them in all circumstances. In fact, in light of the absence
Consistent with their statements earlier in the deliberations, of a mandatory question period, the need to enforce Congress' right to
Commissioners Davide and Maambong proceeded from the same
executive information in the performance of its legislative function becomes Ultimately, the power of Congress to compel the appearance of executive
more imperative. As Schwartz observes: officials under Section 21 and the lack of it under Section 22 find their
basis in the principle of separation of powers. While the executive branch is
a co-equal branch of the legislature, it cannot frustrate the power of
Indeed, if the separation of powers has anything to tell us on the subject
Congress to legislate by refusing to comply with its demands for
under discussion, it is that the Congress has the right to obtain information
information. 
from any source - even from officials of departments and agencies in the
executive branch. In the United States there is, unlike the situation which
prevails in a parliamentary system such as that in Britain, a clear When Congress exercises its power of inquiry, the only way for department
separation between the legislative and executive branches. It is this very heads to exempt themselves therefrom is by a valid claim of privilege.
separation that makes the congressional right to obtain information from They are not exempt by the mere fact that they are department heads.
the executive so essential, if the functions of the Congress as the elected Only one executive official may be exempted from this power - the
representatives of the people are adequately to be carried out. The President on whom executive power is vested, hence, beyond the reach of
absence of close rapport between the legislative and executive branches in Congress except through the power of impeachment. It is based on her
this country, comparable to those which exist under a parliamentary being the highest official of the executive branch, and the due respect
system, and the nonexistence in the Congress of an institution such as the accorded to a co-equal branch of government which is sanctioned by a
British question period have perforce made reliance by the Congress upon long-standing custom. 
its right to obtain information from the executive essential, if it is
intelligently to perform its legislative tasks. Unless the Congress possesses
By the same token, members of the Supreme Court are also exempt from
the right to obtain executive information, its power of oversight of
this power of inquiry. Unlike the Presidency, judicial power is vested in a
administration in a system such as ours becomes a power devoid of most
collegial body; hence, each member thereof is exempt on the basis not
of its practical content, since it depends for its effectiveness solely upon
only of separation of powers but also on the fiscal autonomy and the
information parceled out ex gratia by the executive.89(Emphasis and
constitutional independence of the judiciary. This point is not in dispute, as
underscoring supplied)ςrαlαωlιbrαrÿ
even counsel for the Senate, Sen. Joker Arroyo, admitted it during the oral
argument upon interpellation of the Chief Justice.
Sections 21 and 22, therefore, while closely related and complementary to
each other, should not be considered as pertaining to the same power of
Having established the proper interpretation of Section 22, Article VI of the
Congress. One specifically relates to the power to conduct inquiries in aid
Constitution, the Court now proceeds to pass on the constitutionality of
of legislation, the aim of which is to elicit information that may be used for
Section 1 of E.O. 464. 
legislation, while the other pertains to the power to conduct a question
hour, the objective of which is to obtain information in pursuit of Congress'
oversight function.  Section 1, in view of its specific reference to Section 22 of Article VI of the
Constitution and the absence of any reference to inquiries in aid of
legislation, must be construed as limited in its application to appearances
When Congress merely seeks to be informed on how department heads are
of department heads in the question hour contemplated in the provision of
implementing the statutes which it has issued, its right to such information
said Section 22 of Article VI. The reading is dictated by the basic rule of
is not as imperative as that of the President to whom, as Chief Executive,
construction that issuances must be interpreted, as much as possible, in a
such department heads must give a report of their performance as a
way that will render it constitutional. 
matter of duty. In such instances, Section 22, in keeping with the
separation of powers, states that Congress may only request their
appearance. Nonetheless, when the inquiry in which Congress requires The requirement then to secure presidential consent under Section 1,
their appearance is "in aid of legislation" under Section 21, the appearance limited as it is only to appearances in the question hour, is valid on its face.
is mandatory for the same reasons stated in Arnault.90 For under Section 22, Article VI of the Constitution, the appearance of
department heads in the question hour is discretionary on their part. 
In fine, the oversight function of Congress may be facilitated by
compulsory process only to the extent that it is performed in pursuit of Section 1 cannot, however, be applied to appearances of department heads
legislation. This is consistent with the intent discerned from the in inquiries in aid of legislation. Congress is not bound in such instances to
deliberations of the Constitutional Commission.  respect the refusal of the department head to appear in such inquiry,
unless a valid claim of privilege is subsequently made, either by the Thus, underlying this requirement of prior consent is the determination by
President herself or by the Executive Secretary. a head of office, authorized by the President under E.O. 464, or by the
President herself, that such official is in possession of information that is
covered by executive privilege. This determination then becomes the basis
Validity of Sections 2 and 3
for the official's not showing up in the legislative investigation.

Section 3 of E.O. 464 requires all the public officials enumerated in Section
In view thereof, whenever an official invokes E.O. 464 to justify his failure
2(b) to secure the consent of the President prior to appearing before either
to be present, such invocation must be construed as a declaration to
house of Congress. The enumeration is broad. It covers all senior officials
Congress that the President, or a head of office authorized by the
of executive departments, all officers of the AFP and the PNP, and all senior
President, has determined that the requested information is privileged, and
national security officials who, in the judgment of the heads of offices
that the President has not reversed such determination. Such declaration,
designated in the same section (i.e. department heads, Chief of Staff of the
however, even without mentioning the term "executive privilege," amounts
AFP, Chief of the PNP, and the National Security Adviser), are "covered by
to an implied claim that the information is being withheld by the executive
the executive privilege." 
branch, by authority of the President, on the basis of executive privilege.
Verily, there is an implied claim of privilege.
The enumeration also includes such other officers as may be determined by
the President. Given the title of Section 2 - "Nature, Scope and Coverage of
The letter dated September 28, 2005 of respondent Executive Secretary
Executive Privilege" ', it is evident that under the rule of ejusdem generis,
Ermita to Senate President Drilon illustrates the implied nature of the claim
the determination by the President under this provision is intended to be
of privilege authorized by E.O. 464. It reads:
based on a similar finding of coverage under executive privilege. 

In connection with the inquiry to be conducted by the Committee of the


En passant, the Court notes that Section 2(b) of E.O. 464 virtually states
Whole regarding the Northrail Project of the North Luzon Railways
that executive privilege actually covers persons. Such is a misuse of the
Corporation on 29 September 2005 at 10:00 a.m., please be informed that
doctrine. Executive privilege, as discussed above, is properly invoked in
officials of the Executive Department invited to appear at the meeting will
relation to specific categories of information and not to categories of
not be able to attend the same without the consent of the President,
persons. 
pursuant to Executive Order No. 464 (s. 2005), entitled "Ensuring
Observance Of The Principle Of Separation Of Powers, Adherence To The
In light, however, of Sec 2(a) of E.O. 464 which deals with the nature, Rule On Executive Privilege And Respect For The Rights Of Public Officials
scope and coverage of executive privilege, the reference to persons being Appearing In Legislative Inquiries In Aid Of Legislation Under The
"covered by the executive privilege" may be read as an abbreviated way of Constitution, And For Other Purposes". Said officials have not secured the
saying that the person is in possession of information which is, in the required consent from the President. (Underscoring supplied)ςrαlαωlιbrαrÿ
judgment of the head of office concerned, privileged as defined in Section
2(a). The Court shall thus proceed on the assumption that this is the
The letter does not explicitly invoke executive privilege or that the matter
intention of the challenged order. 
on which these officials are being requested to be resource persons falls
under the recognized grounds of the privilege to justify their absence. Nor
Upon a determination by the designated head of office or by the President does it expressly state that in view of the lack of consent from the
that an official is "covered by the executive privilege," such official is President under E.O. 464, they cannot attend the hearing.
subjected to the requirement that he first secure the consent of the
President prior to appearing before Congress. This requirement effectively
Significant premises in this letter, however, are left unstated, deliberately
bars the appearance of the official concerned unless the same is permitted
or not. The letter assumes that the invited officials are covered by E.O.
by the President. The proviso allowing the President to give its consent
464. As explained earlier, however, to be covered by the order means that
means nothing more than that the President may reverse a prohibition
a determination has been made, by the designated head of office or the
which already exists by virtue of E.O. 464. 
President, that the invited official possesses information that is covered by
executive privilege. Thus, although it is not stated in the letter that such
determination has been made, the same must be deemed implied.
Respecting the statement that the invited officials have not secured the Congress is left to speculate as to which among them is being referred to
consent of the President, it only means that the President has not reversed by the executive. The enumeration is not even intended to be
the standing prohibition against their appearance before Congress.  comprehensive, but a mere statement of what is included in the phrase
"confidential or classified information between the President and the public
officers covered by this executive order." 
Inevitably, Executive Secretary Ermita's letter leads to the conclusion that
the executive branch, either through the President or the heads of offices
authorized under E.O. 464, has made a determination that the information Certainly, Congress has the right to know why the executive considers the
required by the Senate is privileged, and that, at the time of writing, there requested information privileged. It does not suffice to merely declare that
has been no contrary pronouncement from the President. In fine, an the President, or an authorized head of office, has determined that it is so,
implied claim of privilege has been made by the executive. and that the President has not overturned that determination. Such
declaration leaves Congress in the dark on how the requested information
could be classified as privileged. That the message is couched in terms
While there is no Philippine case that directly addresses the issue of
that, on first impression, do not seem like a claim of privilege only makes it
whether executive privilege may be invoked against Congress, it is
more pernicious. It threatens to make Congress doubly blind to the
gathered from Chavez v. PEA that certain information in the possession of
question of why the executive branch is not providing it with the
the executive may validly be claimed as privileged even against Congress.
information that it has requested. 
Thus, the case holds:

A claim of privilege, being a claim of exemption from an obligation to


There is no claim by PEA that the information demanded by petitioner is
disclose information, must, therefore, be clearly asserted. As U.S. v.
privileged information rooted in the separation of powers. The information
Reynolds teaches:
does not cover Presidential conversations, correspondences, or discussions
during closed-door Cabinet meetings which, like internal-deliberations of
the Supreme Court and other collegiate courts, or executive sessions of The privilege belongs to the government and must be asserted by it; it can
either house of Congress, are recognized as confidential. This kind of neither be claimed nor waived by a private party. It is not to be lightly
information cannot be pried open by a co-equal branch of government. A invoked. There must be a formal claim of privilege, lodged by the head of
frank exchange of exploratory ideas and assessments, free from the glare the department which has control over the matter, after actual personal
of publicity and pressure by interested parties, is essential to protect the consideration by that officer. The court itself must determine whether the
independence of decision-making of those tasked to exercise Presidential, circumstances are appropriate for the claim of privilege, and yet do so
Legislative and Judicial power. This is not the situation in the instant without forcing a disclosure of the very thing the privilege is designed to
case.91(Emphasis and underscoring supplied)ςrαlαωlιbrαrÿ protect.92(Underscoring supplied)ςrαlαωlιbrαrÿ

Section 3 of E.O. 464, therefore, cannot be dismissed outright as invalid by Absent then a statement of the specific basis of a claim of executive
the mere fact that it sanctions claims of executive privilege. This Court privilege, there is no way of determining whether it falls under one of the
must look further and assess the claim of privilege authorized by the Order traditional privileges, or whether, given the circumstances in which it is
to determine whether it is valid.  made, it should be respected.93 These, in substance, were the same criteria
in assessing the claim of privilege asserted against the Ombudsman in
Almonte v. Vasquez94 and, more in point, against a committee of the
While the validity of claims of privilege must be assessed on a case to case
Senate in Senate Select Committee on Presidential Campaign Activities v.
basis, examining the ground invoked therefor and the particular
Nixon.95
circumstances surrounding it, there is, in an implied claim of privilege, a
defect that renders it invalid per se. By its very nature, and as
demonstrated by the letter of respondent Executive Secretary quoted A.O. Smith v. Federal Trade Commission is enlightening: 
above, the implied claim authorized by Section 3 of E.O. 464 is not
accompanied by any specific allegation of the basis thereof (e.g., whether
[T]he lack of specificity renders an assessment of the potential harm
the information demanded involves military or diplomatic secrets, closed-
resulting from disclosure impossible, thereby preventing the Court from
door Cabinet meetings, etc.). While Section 2(a) enumerates the types of
balancing such harm against plaintiffs' needs to determine whether to
information that are covered by the privilege under the challenged order,
override any claims of privilege.96(Underscoring supplied)ςrαlαωlιbrαrÿ
And so is U.S. v. Article of Drug:97 the records of the association, a decent respect for the House of
Representatives, by whose authority the subpoenas issued, would have
required that (he) state (his) reasons for noncompliance upon the return of
On the present state of the record, this Court is not called upon to perform
the writ. Such a statement would have given the Subcommittee an
this balancing operation. In stating its objection to claimant's
opportunity to avoid the blocking of its inquiry by taking other appropriate
interrogatories, government asserts, and nothing more, that the
steps to obtain the records. 'To deny the Committee the opportunity to
disclosures sought by claimant would inhibit the free expression of opinion
consider the objection or remedy is in itself a contempt of its authority and
that non-disclosure is designed to protect. The government has not shown
an obstruction of its processes. His failure to make any such statement was
- nor even alleged - that those who evaluated claimant's product were
"a patent evasion of the duty of one summoned to produce papers before a
involved in internal policymaking, generally, or in this particular instance.
congressional committee[, and] cannot be condoned." (Emphasis and
Privilege cannot be set up by an unsupported claim. The facts upon which
underscoring supplied; citations omitted)
the privilege is based must be established. To find these interrogatories
objectionable, this Court would have to assume that the evaluation and
classification of claimant's products was a matter of internal policy Upon the other hand, Congress must not require the executive to state the
formulation, an assumption in which this Court is unwilling to indulge sua reasons for the claim with such particularity as to compel disclosure of the
sponte.98 (Emphasis and underscoring supplied)ςrαlαωlιbrαrÿ information which the privilege is meant to protect.103 A useful analogy in
determining the requisite degree of particularity would be the privilege
against self-incrimination. Thus, Hoffman v. U.S.104declares: 
Mobil Oil Corp. v. Department of Energy99similarly emphasizes that "an
agency must provide 'precise and certain' reasons for preserving the
confidentiality of requested information."  The witness is not exonerated from answering merely because he declares
that in so doing he would incriminate himself - his say-so does not of itself
establish the hazard of incrimination. It is for the court to say whether his
Black v. Sheraton Corp. of America100amplifies, thus:
silence is justified, and to require him to answer if 'it clearly appears to the
court that he is mistaken.' However, if the witness, upon interposing his
A formal and proper claim of executive privilege requires a specific claim, were required to prove the hazard in the sense in which a claim is
designation and description of the documents within its scope as well as usually required to be established in court, he would be compelled to
precise and certain reasons for preserving their confidentiality. Without this surrender the very protection which the privilege is designed to guarantee.
specificity, it is impossible for a court to analyze the claim short of To sustain the privilege, it need only be evident from the implications of
disclosure of the very thing sought to be protected. As the affidavit now the question, in the setting in which it is asked, that a responsive answer
stands, the Court has little more than its sua sponte speculation with which to the question or an explanation of why it cannot be answered might be
to weigh the applicability of the claim. An improperly asserted claim of dangerous because injurious disclosure could result." x x x (Emphasis and
privilege is no claim of privilege. Therefore, despite the fact that a claim underscoring supplied)ςrαlαωlιbrαrÿ
was made by the proper executive as Reynolds requires, the Court can not
recognize the claim in the instant case because it is legally insufficient to
The claim of privilege under Section 3 of E.O. 464 in relation to Section
allow the Court to make a just and reasonable determination as to its
2(b) is thus invalid per se. It is not asserted. It is merely implied. Instead
applicability. To recognize such a broad claim in which the Defendant has
of providing precise and certain reasons for the claim, it merely invokes
given no precise or compelling reasons to shield these documents from
E.O. 464, coupled with an announcement that the President has not given
outside scrutiny, would make a farce of the whole procedure.101 (Emphasis
her consent. It is woefully insufficient for Congress to determine whether
and underscoring supplied)ςrαlαωlιbrαrÿ
the withholding of information is justified under the circumstances of each
case. It severely frustrates the power of inquiry of Congress. 
Due respect for a co-equal branch of government, moreover, demands no
less than a claim of privilege clearly stating the grounds therefor. Apropos
In fine, Section 3 and Section 2(b) of E.O. 464 must be invalidated. 
is the following ruling in McPhaul v. U.S:102

No infirmity, however, can be imputed to Section 2(a) as it merely provides


We think the Court's decision in United States v. Bryan, 339 U.S. 323, 70
guidelines, binding only on the heads of office mentioned in Section 2(b),
S. Ct. 724, is highly relevant to these questions. For it is as true here as it
on what is covered by executive privilege. It does not purport to be
was there, that 'if (petitioner) had legitimate reasons for failing to produce
conclusive on the other branches of government. It may thus be construed It follows, therefore, that when an official is being summoned by Congress
as a mere expression of opinion by the President regarding the nature and on a matter which, in his own judgment, might be covered by executive
scope of executive privilege.  privilege, he must be afforded reasonable time to inform the President or
the Executive Secretary of the possible need for invoking the privilege. This
is necessary in order to provide the President or the Executive Secretary
Petitioners, however, assert as another ground for invalidating the
with fair opportunity to consider whether the matter indeed calls for a claim
challenged order the alleged unlawful delegation of authority to the heads
of executive privilege. If, after the lapse of that reasonable time, neither
of offices in Section 2(b). Petitioner Senate of the Philippines, in particular,
the President nor the Executive Secretary invokes the privilege, Congress is
cites the case of the United States where, so it claims, only the President
no longer bound to respect the failure of the official to appear before
can assert executive privilege to withhold information from Congress. 
Congress and may then opt to avail of the necessary legal means to
compel his appearance.
Section 2(b) in relation to Section 3 virtually provides that, once the head
of office determines that a certain information is privileged, such
The Court notes that one of the expressed purposes for requiring officials
determination is presumed to bear the President's authority and has the
to secure the consent of the President under Section 3 of E.O. 464 is to
effect of prohibiting the official from appearing before Congress, subject
ensure "respect for the rights of public officials appearing in inquiries in aid
only to the express pronouncement of the President that it is allowing the
of legislation." That such rights must indeed be respected by Congress is
appearance of such official. These provisions thus allow the President to
an echo from Article VI Section 21 of the Constitution mandating that
authorize claims of privilege by mere silence.
"[t]he rights of persons appearing in or affected by such inquiries shall be
respected." 
Such presumptive authorization, however, is contrary to the exceptional
nature of the privilege. Executive privilege, as already discussed, is
In light of the above discussion of Section 3, it is clear that it is essentially
recognized with respect to information the confidential nature of which is
an authorization for implied claims of executive privilege, for which reason
crucial to the fulfillment of the unique role and responsibilities of the
it must be invalidated. That such authorization is partly motivated by the
executive branch,105 or in those instances where exemption from disclosure
need to ensure respect for such officials does not change the infirm nature
is necessary to the discharge of highly important executive
of the authorization itself. 
responsibilities.106 The doctrine of executive privilege is thus premised on
the fact that certain informations must, as a matter of necessity, be kept
confidential in pursuit of the public interest. The privilege being, by Right to Information
definition, an exemption from the obligation to disclose information, in this
case to Congress, the necessity must be of such high degree as to
E.O 464 is concerned only with the demands of Congress for the
outweigh the public interest in enforcing that obligation in a particular
appearance of executive officials in the hearings conducted by it, and not
case. 
with the demands of citizens for information pursuant to their right to
information on matters of public concern. Petitioners are not amiss in
In light of this highly exceptional nature of the privilege, the Court finds it claiming, however, that what is involved in the present controversy is not
essential to limit to the President the power to invoke the privilege. She merely the legislative power of inquiry, but the right of the people to
may of course authorize the Executive Secretary to invoke the privilege on information. 
her behalf, in which case the Executive Secretary must state that the
authority is "By order of the President," which means that he personally
There are, it bears noting, clear distinctions between the right of Congress
consulted with her. The privilege being an extraordinary power, it must be
to information which underlies the power of inquiry and the right of the
wielded only by the highest official in the executive hierarchy. In other
people to information on matters of public concern. For one, the demand of
words, the President may not authorize her subordinates to exercise such
a citizen for the production of documents pursuant to his right to
power. There is even less reason to uphold such authorization in the
information does not have the same obligatory force as a subpoena duces
instant case where the authorization is not explicit but by mere silence.
tecum issued by Congress. Neither does the right to information grant a
Section 3, in relation to Section 2(b), is further invalid on this score.
citizen the power to exact testimony from government officials. These
powers belong only to Congress and not to an individual citizen. 
Thus, while Congress is composed of representatives elected by the people, Although the above statement was made in reference to statutes, logic
it does not follow, except in a highly qualified sense, that in every exercise dictates that the challenged order must be covered by the publication
of its power of inquiry, the people are exercising their right to information.  requirement. As explained above, E.O. 464 has a direct effect on the right
of the people to information on matters of public concern. It is, therefore, a
matter of public interest which members of the body politic may question
To the extent that investigations in aid of legislation are generally
before this Court. Due process thus requires that the people should have
conducted in public, however, any executive issuance tending to unduly
been apprised of this issuance before it was implemented. 
limit disclosures of information in such investigations necessarily deprives
the people of information which, being presumed to be in aid of legislation,
is presumed to be a matter of public concern. The citizens are thereby Conclusion
denied access to information which they can use in formulating their own
opinions on the matter before Congress - opinions which they can then
Congress undoubtedly has a right to information from the executive branch
communicate to their representatives and other government officials
whenever it is sought in aid of legislation. If the executive branch withholds
through the various legal means allowed by their freedom of expression.
such information on the ground that it is privileged, it must so assert it and
Thus holds Valmonte v. Belmonte: 
state the reason therefor and why it must be respected.

It is in the interest of the State that the channels for free political
The infirm provisions of E.O. 464, however, allow the executive branch to
discussion be maintained to the end that the government may perceive and
evade congressional requests for information without need of clearly
be responsive to the people's will. Yet, this open dialogue can be effective
asserting a right to do so and/or proffering its reasons therefor. By the
only to the extent that the citizenry is informed and thus able to formulate
mere expedient of invoking said provisions, the power of Congress to
its will intelligently. Only when the participants in the discussion are aware
conduct inquiries in aid of legislation is frustrated. That is impermissible.
of the issues and have access to information relating thereto can such bear
For 
fruit.107 (Emphasis and underscoring supplied)ςrαlαωlιbrαrÿ

[w]hat republican theory did accomplish was to reverse the old


The impairment of the right of the people to information as a consequence
presumption in favor of secrecy, based on the divine right of kings and
of E.O. 464 is, therefore, in the sense explained above, just as direct as its
nobles, and replace it with a presumption in favor of publicity, based on the
violation of the legislature's power of inquiry. 
doctrine of popular sovereignty. (Underscoring supplied)109

Implementation of E.O. 464 prior to its publication


Resort to any means then by which officials of the executive branch could
refuse to divulge information cannot be presumed valid. Otherwise, we
While E.O. 464 applies only to officials of the executive branch, it does not shall not have merely nullified the power of our legislature to inquire into
follow that the same is exempt from the need for publication. On the need the operations of government, but we shall have given up something of
for publishing even those statutes that do not directly apply to people in much greater value - our right as a people to take part in government.
general, Tañada v. Tuvera states:
WHEREFORE, the petitions are PARTLY GRANTED. Sections 2(b) and 3 of
The term "laws" should refer to all laws and not only to those of general Executive Order No. 464 (series of 2005), "Ensuring Observance of the
application, for strictly speaking all laws relate to the people in general Principle of Separation of Powers, Adherence to the Rule on Executive 
albeit there are some that do not apply to them directly. An example is a
law granting citizenship to a particular individual, like a relative of
Privilege and Respect for the Rights of Public Officials Appearing in
President Marcos who was decreed instant naturalization. It surely cannot
Legislative Inquiries in Aid of Legislation Under the Constitution, and For
be said that such a law does not affect the public although it
Other Purposes," are declared VOID. Sections 1 and 2(a) are, however,
unquestionably does not apply directly to all the people. The subject of
VALID. 
such law is a matter of public interest which any member of the body
politic may question in the political forums or, if he is a proper party, even
in courts of justice.108(Emphasis and underscoring supplied)ςrαlαωlιbrαrÿ SO ORDERED.
commander-in-chief are the cornerstones of a professional military in the
firm cusp of civilian control. These values of obedience and deference
expected of military officers are content-neutral, beyond the sway of the
officer's own sense of what is prudent or rash, or more elementally, of right
or wrong. A self-righteous military invites itself as the scoundrel's activist
solution to the "ills" of participatory democracy.

Petitioners seek the annulment of a directive from President Gloria


Macapagal-Arroyo1enjoining them and other military officers from testifying
before Congress without the President's consent. Petitioners also pray for
injunctive relief against a pending preliminary investigation against them,
in preparation for possible court-martial proceedings, initiated within the
military justice system in connection with petitioners' violation of the
aforementioned directive.

The Court is cognizant that petitioners, in their defense, invoke weighty


constitutional principles that center on fundamental freedoms enshrined in
the Bill of Rights. Although these concerns will not be

addressed to the satisfaction of petitioners, the Court recognizes these


values as of paramount importance to our civil society, even if not
determinative of the resolution of this petition. Had the relevant issue
before us been the right of the Senate to compel the testimony of
G.R. NO. 170165 : August 15, 2006] petitioners, the constitutional questions raised by them would have come
to fore. Such a scenario could have very well been presented to the Court
in such manner, without the petitioners having had to violate a direct order
B/GEN. (RET.) FRANCISCO V. GUDANI AND LT. COL. ALEXANDER F. from their commanding officer. Instead, the Court has to resolve whether
BALUTAN Petitioners, v. LT./GEN. GENEROSO S. SENGA CORONA, AS petitioners may be subjected to military discipline on account of their
CHIEF OF STAFF OF THE CARPIO-MORALES, ARMED FORCES OF THE defiance of a direct order of the AFP Chief of Staff.
CALLEJO, SR., PHILIPPINES, COL. GILBERTO AZCUNA, JOSE C. ROA
AS THE PRE-TRIAL TINGA, INVESTIGATING OFFICER, THE CHICO-
NAZARIO, PROVOST MARSHALL GENERAL GARCIA, and OF THE The solicited writs of certiorari and prohibition do not avail; the petition
ARMED FORCES OF THE PHILIPPINES AND THE GENERAL COURT- must be denied.
MARTIAL, Respondents.
I.
DECISION
The petitioners are high-ranking officers of the Armed Forces of the
TINGA, J.: Philippines (AFP). Both petitioners, Brigadier General Francisco Gudani
(Gen. Gudani) and Lieutenant Colonel Alexander Balutan (Col. Balutan),
belonged to the Philippine Marines. At the time of the subject incidents,
A most dangerous general proposition is foisted on the Court - that soldiers both Gen. Gudani and Col. Balutan were assigned to the Philippine Military
who defy orders of their superior officers are exempt Academy (PMA) in Baguio City, the former as the PMA Assistant
Superintendent, and the latter as the Assistant Commandant of Cadets.2
from the strictures of military law and discipline if such defiance is
predicated on an act otherwise valid under civilian law. Obedience and
deference to the military chain of command and the President as
On 22 September 2005, Senator Rodolfo Biazon (Sen. Biazon) invited PER INSTRUCTION OF HER EXCELLENCY PGMA, NO AFP PERSONNEL SHALL
several senior officers of the AFP to appear at a public hearing before the APPEAR BEFORE ANY CONGRESSIONAL OR SENATE HEARING WITHOUT
Senate Committee on National Defense and Security (Senate Committee) HER APPROVAL. INFORM BGEN FRANCISCO F GUDANI AFP AND LTC
scheduled on 28 September 2005. The hearing was scheduled after topics ALEXANDER BALUTAN PA (GSC) ACCORDINGLY.7
concerning the conduct of the 2004 elections emerged in the public eye,
particularly allegations of massive cheating and the surfacing of copies of
The following day, Gen. Senga sent another letter to Sen. Biazon, this time
an audio excerpt purportedly of a phone conversation between President
informing the senator that "no approval has been granted by the President
Gloria Macapagal Arroyo and an official of the Commission on Elections
to any AFP officer to appear" before the hearing scheduled on that day.
(COMELEC) widely reputed as then COMELEC Commissioner Virgilio
Nonetheless, both Gen. Gudani and Col. Balutan were present as the
Garcillano. At the time of the 2004 elections, Gen. Gudani had been
hearing started, and they both testified as to the conduct of the 2004
designated as commander, and Col. Balutan a member, of "Joint Task
elections.
Force Ranao" by the AFP Southern Command. "Joint Task Force Ranao"
was tasked with the maintenance of peace and order during the 2004
elections in the provinces of Lanao del Norte and Lanao del Sur.3 ` The Office of the Solicitor General (OSG), representing the respondents
before this Court, has offered additional information surrounding the
testimony of Gen. Gudani and Col. Balutan. The OSG manifests that the
Gen. Gudani, Col. Balutan, and AFP Chief of Staff Lieutenant General
couriers of the AFP Command Center had attempted to deliver the radio
Generoso Senga (Gen. Senga) were among the several AFP officers who
message to Gen. Gudani's residence in a subdivision in Parañaque City late
received a letter invitation from Sen. Biazon to attend the 28 September
in the night of 27 September 2005, but they were not permitted entry by
2005 hearing. On 23 September 2005, Gen. Senga replied through a letter
the subdivision guards. The next day, 28 September 2005, shortly before
to Sen. Biazon that he would be unable to attend the hearing due to a
the start of the hearing, a copy of Gen. Senga's letter to Sen. Biazon sent
previous commitment in Brunei, but he nonetheless "directed other officers
earlier that day was handed at the Senate by Commodore Amable B.
from the AFP who were invited to attend the hearing."4
Tolentino of the AFP Office for Legislative Affairs to Gen. Gudani, who
replied that he already had a copy. Further, Gen. Senga called Commodore
On 26 September 2005, the Office of the Chief of Staff of the AFP issued a Tolentino on the latter's cell phone and asked to talk to Gen. Gudani, but
Memorandum addressed to the Superintendent of the PMA Gen. Cristolito Gen. Gudani refused. In response, Gen. Senga instructed Commodore
P. Baloing (Gen. Baloing). It was signed by Lt. Col. Hernando DCA Iriberri Tolentino to inform Gen. Gudani that "it was an order," yet Gen. Gudani
in behalf of Gen. Senga.5 Noting that Gen. Gudani and Col. Balutan had still refused to take Gen. Senga's call.8
been invited to attend the Senate Committee hearing on 28 September
2005, the Memorandum directed the two officers to attend the
A few hours after Gen. Gudani and Col. Balutan had concluded their
hearing.6 Conformably, Gen. Gudani and Col. Balutan filed their respective
testimony, the office of Gen. Senga issued a statement which noted that
requests for travel authority addressed to the PMA Superintendent.
the two had appeared before the Senate Committee "in spite of the fact
that a guidance has been given that a Presidential approval should be
On 27 September 2005, Gen. Senga wrote a letter to Sen. Biazon, sought prior to such an appearance;" that such directive was "in keeping
requesting the postponement of the hearing scheduled for the following with the time[-]honored principle of the Chain of Command;" and that the
day, since the AFP Chief of Staff was himself unable to attend said hearing, two officers "disobeyed a legal order, in violation of A[rticles of] W[ar] 65
and that some of the invited officers also could not attend as they were (Willfully Disobeying Superior Officer), hence they will be subjected to
"attending to other urgent operational matters." By this time, both Gen. General Court Martial proceedings x x x" Both Gen. Gudani and Col.
Gudani and Col. Balutan had already departed Baguio for Manila to attend Balutan were likewise relieved of their assignments then.9
the hearing.
On the very day of the hearing, 28 September 2005, President Gloria-
Then on the evening of 27 September 2005, at around 10:10 p.m., a Macapagal-Arroyo issued Executive Order No. 464 (E.O. 464). The OSG
message was transmitted to the PMA Superintendent from the office of notes that the E.O. "enjoined officials of the executive department
Gen. Senga, stating as follows: including the military establishment from appearing in any legislative
inquiry without her approval."10This Court subsequently ruled on the
constitutionality of the said executive order in Senate v. Ermita.11 The
relevance of E.O. 464 and Senate to the present petition shall be discussed unconstitutional, it still was tantamount to "the crime of obstruction of
forthwith. justice." Petitioners further argue that there was no law prohibiting them
from testifying before the Senate, and in fact, they were appearing in
obeisance to the authority of Congress to conduct inquiries in aid of
In the meantime, on 30 September 2005, petitioners were directed by
legislation.
General Senga, through Col. Henry A. Galarpe of the AFP Provost Marshal
General, to appear before the Office of the Provost Marshal General
(OPMG) on 3 October 2005 for investigation. During their appearance Finally, it is stressed in the petition that Gen. Gudani was no longer subject
before Col. Galarpe, both petitioners invoked their right to remain to military jurisdiction on account of his compulsory retirement on 4
silent.12 The following day, Gen. Gudani was compulsorily retired from October 2005. It is pointed out that Article 2, Title I of the Articles of War
military service, having reached the age of 56.13 defines persons subject to military law as "all officers and soldiers in the
active service" of the AFP.
In an Investigation Report dated 6 October 2005, the OPMG recommended
that petitioners be charged with violation of Article of War 65, on willfully II.
disobeying a superior officer, in relation to Article of War 97, on conduct
prejudicial to the good order and military discipline.14 As recommended, the
We first proceed to define the proper litigable issues. Notably, the guilt or
case was referred to a Pre-Trial Investigation Officer (PTIO) preparatory to
innocence of petitioners in violating Articles 65 and 97 of the Articles of
trial by the General Court Martial (GCM).15 Consequently, on 24 October
War is not an issue before this Court, especially considering that per
2005, petitioners were separately served with Orders respectively
records, petitioners have not yet been subjected to court martial
addressed to them and signed by respondent Col. Gilbert Jose C. Roa, the
proceedings. Owing to the absence of such proceedings, the correct inquiry
Pre-Trial Investigating Officer of the PTIO. The Orders directed petitioners
should be limited to whether respondents could properly initiate such
to appear in person before Col. Roa at the Pre-Trial Investigation of the
proceedings preparatory to a formal court-martial, such as the
Charges for violation of Articles 6516and 9717 of Commonwealth Act No.
aforementioned preliminary investigation, on the basis of petitioners' acts
408,18and to submit their counter-affidavits and affidavits of witnesses at
surrounding their testimony before the Senate on 28 September 2005. Yet
the Office of the Judge Advocate General.19 The Orders were accompanied
this Court, consistent with the principle that it is not a trier of facts at first
by respective charge sheets against petitioners, accusing them of violating
instance,21 is averse to making any authoritative findings of fact, for that
Articles of War 65 and 97.
function is first for the court-martial court to fulfill.

It was from these premises that the present petition for certiorari and


Thus, we limit ourselves to those facts that are not controverted before the
prohibition was filed, particularly seeking that (1) the order of President
Court, having been commonly alleged by petitioners and the OSG (for
Arroyo coursed through Gen. Senga preventing petitioners from testifying
respondents). Petitioners were called by the Senate Committee to testify in
before Congress without her prior approval be declared unconstitutional;
its 28 September 2005 hearing. Petitioners attended such hearing and
(2) the charges stated in the charge sheets against petitioners be quashed;
testified before the Committee, despite the fact that the day before, there
and (3) Gen. Senga, Col. Galarpe, Col. Roa, and their successors-in-
was an order from Gen. Senga (which in turn was sourced "per instruction"
interest or persons acting for and on their behalf or orders, be permanently
from President Arroyo) prohibiting them from testifying without the prior
enjoined from proceeding against petitioners, as a consequence of their
approval of the President. Petitioners do not precisely admit before this
having testified before the Senate on 28 September 2005.20
Court that they had learned of such order prior to their testimony, although
the OSG asserts that at the very least, Gen. Gudani already knew of such
Petitioners characterize the directive from President Arroyo requiring her order before he testified.22 Yet while this fact may be ultimately material in
prior approval before any AFP personnel appear before Congress as a "gag the court-martial proceedings, it is not determinative of this petition, which
order," which violates the principle of separation of powers in government as stated earlier, does not proffer as an issue whether petitioners are guilty
as it interferes with the investigation of the Senate Committee conducted in of violating the Articles of War.
aid of legislation. They also equate the "gag order" with culpable violation
of the Constitution, particularly in relation to the public's constitutional
What the Court has to consider though is whether the violation of the
right to information and transparency in matters of public concern.
aforementioned order of Gen. Senga, which emanated from the President,
Plaintively, petitioners claim that "the Filipino people have every right to
could lead to any investigation for court-martial of petitioners. It has to be
hear the [petitioners'] testimonies," and even if the "gag order" were
acknowledged as a general principle23 that AFP personnel of whatever rank prior consent before appearing before Congress. The pertinent factors in
are liable under military law for violating a direct order of an officer considering that question are markedly outside of those which did become
superior in rank. Whether petitioners did violate such an order is not for relevant in adjudicating the issues raised in Senate. It is in this petition
the Court to decide, but it will be necessary to assume, for the purposes of that those factors come into play.
this petition, that petitioners did so.
At this point, we wish to dispose of another peripheral issue before we
III. strike at the heart of the matter. General Gudani argues that he can no
longer fall within the jurisdiction of the court-martial, considering his
retirement last 4 October 2005. He cites Article 2, Title I of Commonwealth
Preliminarily, we must discuss the effect of E.O. 464 and the Court's ruling
Act No. 408, which defines persons subject to military law as, among
in Senate on the present petition. Notably, it is not alleged that
others, "all officers and soldiers in the active service of the [AFP]," and
petitioners were in any way called to task for violating E.O. 464,
points out that he is no longer in the active service.
but instead, they were charged for violating the direct order of Gen.
Senga not to appear before the Senate Committee, an order that
stands independent of the executive order. Distinctions are called for, This point was settled against Gen. Gudani's position in Abadilla v.
since Section 2(b) of E.O. 464 listed "generals and flag officers of the Ramos,27 where the Court declared that an officer whose name was
Armed Forces of the Philippines and such other officers who in the dropped from the roll of officers cannot be considered to be outside the
judgment of the Chief of Staff are covered by the executive privilege," as jurisdiction of military authorities when military justice proceedings were
among those public officials required in Section 3 of E.O. 464 "to secure initiated against him before the termination of his service. Once jurisdiction
prior consent of the President prior to appearing before either House of has been acquired over the officer, it continues until his case is terminated.
Congress." The Court in Senate declared both Section 2(b) and Section 3 Thus, the Court held:
void,24 and the impression may have been left following Senate that it
settled as doctrine, that the President is prohibited from requiring military
The military authorities had jurisdiction over the person of Colonel Abadilla
personnel from attending congressional hearings without having first
at the time of the alleged offenses. This jurisdiction having been vested in
secured prior presidential consent. That impression is wrong.
the military authorities, it is retained up to the end of the proceedings
against Colonel Abadilla. Well-settled is the rule that jurisdiction once
Senate turned on the nature of executive privilege, a presidential acquired is not lost upon the instance of the parties but continues until the
prerogative which is encumbered by significant limitations. Insofar as E.O. case is terminated.28
464 compelled officials of the executive branch to seek prior presidential
approval before appearing before Congress, the notion of executive control
Citing Colonel Winthrop's treatise on Military Law, the Court further stated:
also comes into consideration.25 However, the ability of the President to
require a military official to secure prior consent before appearing before
Congress pertains to a wholly different and independent specie of We have gone through the treatise of Colonel Winthrop and We find the
presidential authority the commander-in-chief powers of the President. By following passage which goes against the contention of the petitioners, viz' 
tradition and jurisprudence, the commander-in-chief powers of the
President are not encumbered by the same degree of restriction as that 3. Offenders in general - Attaching of jurisdiction. It has further been held,
which may attach to executive privilege or executive control. and is now settled law, in regard to military offenders in general, that if the
military jurisdiction has once duly attached to them previous to the date of
During the deliberations in Senate, the Court was very well aware of the the termination of their legal period of service, they may be brought to trial
pendency of this petition as well as the issues raised herein. The decision by court-martial after that date, their discharge being meanwhile withheld.
in Senate was rendered with the comfort that the nullification of portions of This principle has mostly been applied to cases where the offense was
E.O. 464 would bear no impact on the present petition since petitioners committed just prior to the end of the term. In such cases the interests of
herein were not called to task for violating the executive order. Moreover, discipline clearly forbid that the offender should go unpunished. It is held
the Court was then cognizant that Senate and this case would ultimately therefore that if before the day on which his service legally
hinge on disparate legal issues. Relevantly, Senate purposely did not touch terminates and his right to a discharge is complete, proceedings
upon or rule on the faculty of the President, under the aegis of the with a view to trial are commenced against him - as by arrest or
commander-in-chief powers26 to require military officials from securing the service of charges, - the military jurisdiction will fully attach
and once attached may be continued by a trial by court-martial war.33 Congress is also empowered to revoke a proclamation of martial law
ordered and held after the end of the term of the enlistment of the or the suspension of the writ of habeas corpus.34The approval of the
accused x x x 29 Commission on Appointments is also required before the President can
promote military officers from the rank of colonel or naval
captain.35Otherwise, on the particulars of civilian dominance and
Thus, military jurisdiction has fully attached to Gen. Gudani inasmuch as
administration over the military, the Constitution is silent, except for the
both the acts complained of and the initiation of the proceedings against
commander-in-chief clause which is fertile in meaning and
him occurred before he compulsorily retired on 4 October 2005. We see no
reason to unsettle the Abadilladoctrine. The OSG also points out that under
Section 28 of Presidential Decree No. 1638, as amended, "[a]n officer or implication as to whatever inherent martial authority the President may
enlisted man carried in the retired list [of the Armed Forces of the possess.36
Philippines] shall be subject to the Articles of War x x x"30 To this citation,
petitioners do not offer any response, and in fact have excluded the matter
The commander-in-chief provision in the Constitution is denominated as
of Gen. Gudani's retirement as an issue in their subsequent memorandum.
Section 18, Article VII, which begins with the simple declaration that "[t]he
President shall be the Commander-in-Chief of all armed forces of the
IV. Philippines x x x"37 Outside explicit constitutional limitations, such as those
found in Section 5, Article XVI, the commander-in-chief clause vests on the
President, as commander-in-chief, absolute authority over the persons and
We now turn to the central issues.
actions of the members of the armed forces. Such authority includes the
ability of the President to restrict the travel, movement and speech of
Petitioners wish to see annulled the "gag order" that required them to military officers, activities which may otherwise be sanctioned under
secure presidential consent prior to their appearance before the Senate, civilian law.
claiming that it violates the constitutional right to information and
transparency in matters of public concern; or if not, is tantamount at least
Reference to Kapunan, Jr. v. De Villa38 is useful in this regard. Lt. Col.
to the criminal acts of obstruction of justice and grave coercion. However,
Kapunan was ordered confined under "house arrest" by then Chief of Staff
the proper perspective from which to consider this issue entails the
(later President) Gen. Fidel Ramos. Kapunan was also ordered, as a
examination of the basis and authority of the President to issue such an
condition for his house arrest, that he may not issue any press statements
order in the first place to members of the AFP and the determination of
or give any press conference during his period of detention. The Court
whether such an order is subject to any limitations.
unanimously upheld such restrictions, noting:

The vitality of the tenet that the President is the commander-in-chief of the
[T]he Court is of the view that such is justified by the requirements of
Armed Forces is most crucial to the democratic way of life, to civilian
military discipline. It cannot be gainsaid that certain liberties of
supremacy over the military, and to the general stability of our
persons in the military service, including the freedom of speech,
representative system of government. The Constitution reposes final
may be circumscribed by rules of military discipline. Thus, to a
authority, control and supervision of the AFP to the President, a civilian
certain degree, individual rights may be curtailed, because the
who is not a member of the armed forces, and whose duties as
effectiveness of the military in fulfilling its duties under the law
commander-in-chief represent only a part of the organic duties imposed
depends to a large extent on the maintenance of discipline within
upon the office, the other functions being clearly civil in nature.31Civilian
its ranks. Hence, lawful orders must be followed without question
supremacy over the military also countermands the notion that the military
and rules must be faithfully complied with, irrespective of a
may bypass civilian authorities, such as civil courts, on matters such as
soldier's personal views on the matter. It is from this viewpoint that
conducting warrantless searches and seizures.32
the restrictions imposed on petitioner Kapunan, an officer in the AFP, have
to be considered.39
Pursuant to the maintenance of civilian supremacy over the military, the
Constitution has allocated specific roles to the legislative and executive
Any good soldier, or indeed any ROTC cadet, can attest to the fact that the
branches of government in relation to military affairs. Military
military way of life circumscribes several of the cherished freedoms of
appropriations, as with all other appropriations, are determined by
civilian life. It is part and parcel of the military package. Those who cannot
Congress, as is the power to declare the existence of a state of
abide by these limitations normally do not pursue a military career and Further traditional restrictions on members of the armed forces are those
instead find satisfaction in other fields; and in fact many of those imposed on free speech and mobility.ςηαñrοblεš νιr†υαl lαω lιbrαrÿ
discharged from the service are inspired in their later careers precisely by
their rebellion against the regimentation of military life. Inability or
Kapunan is ample precedent in justifying that a soldier may be restrained
unwillingness to cope with military discipline is not a stain on character, for
by a superior officer from speaking out on certain matters. As a general
the military mode is a highly idiosyncratic path which persons are not
rule, the discretion of a military officer to restrain the speech of a soldier
generally conscripted into, but volunteer themselves to be part of. But for
under his/her command will be accorded deference, with minimal regard if
those who do make the choice to be a soldier, significant concessions to
at all to the reason for such restraint. It is integral to military discipline that
personal freedoms are expected. After all, if need be, the men and women
the soldier's speech be with the consent and approval of the military
of the armed forces may be commanded upon to die for country, even
commander.
against their personal inclinations.

The necessity of upholding the ability to restrain speech becomes even


It may be so that military culture is a remnant of a less democratic era, yet
more imperative if the soldier desires to speak freely on political matters.
it has been fully integrated into the democratic system of governance. The
The Constitution requires that "[t]he armed forces shall be insulated from
constitutional role of the armed forces is as protector of the people and of
partisan politics," and that '[n]o member of the military shall engage
the State.40 Towards this end, the military must insist upon a respect for
directly or indirectly in any partisan political activity, except to
duty and a discipline without counterpart in civilian life.41 The laws and
vote."47 Certainly, no constitutional provision or military indoctrination will
traditions governing that discipline have a long history; but they are
eliminate a soldier's ability to form a personal political opinion, yet it is vital
founded on unique military exigencies as powerful now as in the past.42In
that such opinions be kept out of the public eye. For one, political belief is a
the end, it must be borne in mind that the armed forces has a distinct
potential source of discord among people, and a military torn by political
subculture with unique needs, a specialized society separate from civilian
strife is incapable of fulfilling its constitutional function as protectors of the
society.43 In the elegant prose of the eminent British military historian,
people and of the State. For another, it is ruinous to military discipline to
John Keegan:
foment an atmosphere that promotes an active dislike of or dissent against
the President, the commander-in-chief of the armed forces. Soldiers are
[Warriors who fight wars have] values and skills [which] are not those of constitutionally obliged to obey a President they may dislike or distrust.
politicians and diplomats. They are those of a world apart, a very ancient This fundamental principle averts the country from going the way of
world, which exists in parallel with the everyday world but does not belong banana republics.
to it. Both worlds change over time, and the warrior world adopts in step to
the civilian. It follows it, however, at a distance. The distance can never be
Parenthetically, it must be said that the Court is well aware that our
closed, for the culture of the warrior can never be that of civilization
country's recent past is marked by regime changes wherein active military
itself'.44
dissent from the chain of command formed a key, though not exclusive,
element. The Court is not blind to history, yet it is a judge not of history
Critical to military discipline is obeisance to the military chain of command. but of the Constitution. The Constitution, and indeed our modern
Willful disobedience of a superior officer is punishable by court-martial democratic order, frown in no uncertain terms on a politicized military,
under Article 65 of the Articles of War.45 "An individual soldier is not free to informed as they are on the trauma of absolute martial rule. Our history
ignore the lawful orders or duties assigned by his immediate superiors. For might imply that a political military is part of the natural order, but this
there would be an end of all discipline if the seaman and marines on board view cannot be affirmed by the legal order. The evolutionary path of our
a ship of war [or soldiers deployed in the field], on a distant service, were young democracy necessitates a reorientation from this view, reliant as our
permitted to act upon their own opinion of their rights [or their opinion of socio-political culture has become on it. At the same time, evolution
the mandates a similar demand that our system of governance be more
responsive to the needs and aspirations of the citizenry, so as to avoid an
environment vulnerable to a military apparatus able at will to exert an
President's intent], and to throw off the authority of the
undue influence in our polity.
commander whenever they supposed it to be unlawfully
exercised."46
Of possibly less gravitas, but of equal importance, is the principle that
mobility of travel is another necessary restriction on members of the
military. A soldier cannot leave his/her post without the consent of the of her power as commander-in-chief, and that as a consequence a military
commanding officer. The reasons are self-evident. The commanding officer officer who defies such injunction is liable under military justice. At the
has to be aware at all times of the location of the troops under command, same time, we also hold that any chamber of Congress which seeks the
so as to be able to appropriately respond to any exigencies. For the same appearance before it of a military officer against the consent of the
reason, commanding officers have to be able to restrict the movement or President has adequate remedies under law to compel such attendance.
travel of their soldiers, if in their judgment, their presence at place of call Any military official whom Congress summons to testify before it may be
of duty is necessary. At times, this may lead to unsentimental, painful compelled to do so by the President. If the President is not so inclined, the
consequences, such as a soldier being denied permission to witness the President may be commanded by judicial order to compel the attendance of
birth of his first-born, or to attend the funeral of a parent. Yet again, the military officer. Final judicial orders have the force of the law of the
military life calls for considerable personal sacrifices during the period of land which the President has the duty to faithfully execute.50
conscription, wherein the higher duty is not to self but to country.
Explication of these principles is in order.
Indeed, the military practice is to require a soldier to obtain permission
from the commanding officer before he/she may leave his destination. A
As earlier noted, we ruled in Senate that the President may not issue a
soldier who goes from the properly appointed place of duty or absents from
blanket requirement of prior consent on executive officials summoned by
his/her command, guard, quarters, station, or camp without proper leave is
the legislature to attend a congressional hearing. In doing so, the Court
subject to punishment by court-martial.48 It is even clear from the record
recognized the considerable limitations on executive privilege, and affirmed
that petitioners had actually requested for travel authority from the PMA in
that the privilege must be formally invoked on specified
Baguio City to Manila, to attend the Senate Hearing.49 Even petitioners are
grounds. However, the ability of the President to prevent military
well aware that it was necessary for them to obtain permission from their
officers from testifying before Congress does not turn on executive
superiors before they could travel to Manila to attend the Senate Hearing.
privilege, but on the Chief Executive's power as commander-in-
chief to control the actions and speech of members of the armed
It is clear that the basic position of petitioners impinges on these forces. The President's prerogatives as commander-in-chief are not
fundamental principles we have discussed. They seek to be exempted from hampered by the same limitations as in executive privilege.
military justice for having traveled to the Senate to testify before the
Senate Committee against the express orders of Gen. Senga, the AFP Chief
Our ruling that the President could, as a general rule, require military
of Staff. If petitioners' position is affirmed, a considerable exception would
officers to seek presidential approval before appearing before Congress is
be carved from the unimpeachable right of military officers to restrict the
based foremost on the notion that a contrary rule unduly diminishes the
speech and movement of their juniors. The ruinous consequences to the
prerogatives of the President as commander-in-chief. Congress holds
chain of command and military discipline simply cannot warrant the
significant control over the armed forces in matters such as budget
Court's imprimaturon petitioner's position.
appropriations and the approval of higher-rank promotions,51 yet it is on
the President that the Constitution vests the title as commander-in-chief
V. and all the prerogatives and functions appertaining to the position. Again,
the exigencies of military discipline and the chain of command mandate
that the President's ability to control the individual members of the armed
Still, it would be highly myopic on our part to resolve the issue solely on
forces be accorded the utmost respect. Where a military officer is torn
generalities surrounding military discipline. After all, petitioners seek to
between obeying the President and obeying the Senate, the Court will
impress on us that their acts are justified as they were responding to an
without hesitation affirm that the officer has to choose the President. After
invitation from the Philippine Senate, a component of the legislative branch
all, the Constitution prescribes that it is the President, and not the Senate,
of government. At the same time, the order for them not to testify
who is the commander-in-chief of the armed forces.52
ultimately came from the President, the head of the executive branch of
government and the commander-in-chief of the armed forces.
At the same time, the refusal of the President to allow members of the
military to appear before Congress is still subject to judicial relief. The
Thus, we have to consider the question: may the President prevent a
Constitution itself recognizes as one of the legislature's functions is the
member of the armed forces from testifying before a legislative inquiry?
conduct of inquiries in aid of legislation.53Inasmuch as it is ill-advised for
We hold that the President has constitutional authority to do so, by virtue
Congress to interfere with the President's power as commander-in-chief, it
is similarly detrimental for the President to unduly interfere with Congress's As discussed in Arnault, the power of inquiry, "with process to enforce it,"
right to conduct legislative inquiries. The impasse did not come to pass in is grounded on the necessity of information in the legislative process. If the
this petition, since petitioners testified anyway despite the presidential information possessed by executive officials on the operation of their
prohibition. Yet the Court is aware that with its pronouncement today that offices is necessary for wise legislation on that subject, by parity of
the President has the right to require prior consent from members of the reasoning, Congress has the right to that information and the power to
armed forces, the clash may soon loom or actualize. compel the disclosure thereof.

We believe and hold that our constitutional and legal order sanctions a As evidenced by the American experience during the so-called "McCarthy
modality by which members of the military may be compelled to attend era", however, the right of Congress to conduct inquirites in aid of
legislative inquiries even if the President desires otherwise, a modality legislation is, in theory, no less susceptible to abuse than executive or
which does not offend the Chief Executive's prerogatives as commander-in- judicial power. It may thus be subjected to judicial review pursuant to the
chief. The remedy lies with the courts. Court's certioraripowers under Section 1, Article VIII of the Constitution.

The fact that the executive branch is an equal, coordinate branch of For one, as noted in Bengzon v. Senate Blue Ribbon Committee, the inquiry
government to the legislative creates a wrinkle to any basic rule that itself might not properly be in aid of legislation, and thus beyond the
persons summoned to testify before Congress must do so. There is constitutional power of Congress. Such inquiry could not usurp judicial
considerable interplay between the legislative and executive branches, functions. Parenthetically, one possible way for Congress to avoid such
informed by due deference and respect as to their various constitutional result as occurred in Bengzon is to indicate in its invitations to the public
functions. Reciprocal courtesy idealizes this relationship; hence, it is only officials concerned, or to any person for that matter, the possible needed
as a last resort that one branch seeks to compel the other to a particular statute which prompted the need for the inquiry. Given such statement in
mode of behavior. The judiciary, the third coordinate branch of its invitations, along with the usual indication of the subject of inquiry and
government, does not enjoy a similar dynamic with either the legislative or the questions relative to and in furtherance thereof, there would be less
executive branches. Whatever weakness inheres on judicial power due to room for speculation on the part of the person invited on whether the
its inability to originate national policies and legislation, such is balanced by inquiry is in aid of legislation.
the fact that it is the branch empowered by the Constitution to compel
obeisance to its rulings by the other branches of government.
Section 21, Article VI likewise establishes critical safeguards that proscribe
the legislative power of inquiry. The provision requires that the inquiry be
As evidenced by Arnault v. Nazareno54 and Bengzon v. Senate Blue Ribbon done in accordance with the Senate or House's duly published rules of
Committee,55among others, the Court has not shirked from reviewing the procedure, necessarily implying the constitutional infirmity of an inquiry
exercise by Congress of its power of legislative inquiry.56 Arnaultrecognized conducted without duly published rules of procedure. Section 21 also
that the legislative power of inquiry and the process to enforce it, "is an mandates that the rights of persons appearing in or affected by such
essential and appropriate auxiliary to the legislative function."57 On the inquiries be respected, an imposition that obligates Congress to adhere to
other hand, Bengzon acknowledged that the power of both houses of the guarantees in the Bill of Rights.
Congress to conduct inquiries in aid of legislation is not "absolute or
unlimited", and its exercise is circumscribed by Section 21, Article VI of the
These abuses are, of course, remediable before the courts, upon the proper
Constitution.58 From these premises, the Court enjoined the Senate Blue
suit filed by the persons affected, even if they belong to the executive
Ribbon Committee from requiring the petitioners in Bengzon from testifying
branch. Nonetheless, there may be exceptional circumstances' wherein a
and producing evidence before the committee, holding that the inquiry in
clear pattern of abuse of the legislative power of inquiry might be
question did not involve any intended legislation.
established, resulting in palpable violations of the rights guaranteed to
members of the executive department under the Bill of Rights. In such
Senate affirmed both the Arnault and Bengzon rulings. It elucidated on the instances, depending on the particulars of each case, attempts by the
constitutional scope and limitations on the constitutional power of Executive Branch to forestall these abuses may be accorded judicial
congressional inquiry. Thus: sanction59 .
In Senate, the Court ruled that the President could not impose a blanket appearing before the legislature to testify, the Chief Executive is
prohibition barring executive officials from testifying before Congress nonetheless obliged to comply with the final orders of the courts. 
without the President's consent notwithstanding the invocation of executive
privilege to justify such prohibition. The Court did not rule that the power
Petitioners have presented several issues relating to the tenability or
to conduct legislative inquiry ipso facto superseded the claim of executive
wisdom of the President's order on them and other military officers not to
privilege, acknowledging instead that the viability of executive privilege
testify before Congress without the President's consent. Yet these issues
stood on a case to case basis. Should neither branch yield to the other
ultimately detract from the main point - that they testified before the
branch's assertion, the constitutional recourse is to the courts, as the final
Senate despite an order from their commanding officer and their
arbiter if the dispute. It is only the courts that can compel, with
commander-in-chief for them not to do so,61in contravention of the
conclusiveness, attendance or non-attendance in legislative inquiries.
traditions of military discipline which we affirm
today.ςηαñrοblεš νιr†υαl lαω lιbrαrÿ
Following these principles, it is clear that if the President or the Chief of
Staff refuses to allow a member of the AFP to appear before Congress, the
The issues raised by petitioners could have very well been raised and
legislative body seeking such testimony may seek judicial relief to compel
properly adjudicated if the proper procedure was observed. Petitioners
the attendance. Such judicial action should be directed at the heads of the
could have been appropriately allowed to testify before the Senate without
executive branch or the armed forces, the persons who wield authority and
having to countermand their Commander-in-chief and superior officer
control over the actions of the officers concerned. The legislative purpose
under the setup we have prescribed.
of such testimony, as well as any defenses against the same - whether
grounded on executive privilege, national security or similar concerns -
would be accorded due judicial evaluation. All the constitutional We consider the other issues raised by petitioners unnecessary to the
considerations pertinent to either branch of government may be raised, resolution of this petition.
assessed, and ultimately weighed against each other. And once the courts
speak with finality, both branches of government have no option but to Petitioners may have been of the honest belief that they were defying a
comply with the decision of the courts, whether the effect of the decision is direct order of their Commander-in-Chief and Commanding General in
to their liking or disfavor. obeisance to a paramount idea formed within their consciences, which
could not be lightly ignored. Still, the Court, in turn, is guided by the
Courts are empowered, under the constitutional principle of judicial review, superlative principle that is the Constitution, the embodiment of the
to arbitrate disputes between the legislative and executive branches of national conscience. The Constitution simply does not permit the infraction
government on the proper constitutional parameters of power.60This is the which petitioners have allegedly committed, and moreover, provides for an
fair and workable solution implicit in the constitutional allocation of powers orderly manner by which the same result could have been achieved without
among the three branches of government. The judicial filter helps assure offending constitutional principles.
that the particularities of each case would ultimately govern, rather than
any overarching principle unduly inclined towards one branch of WHEREFORE, the petition is DENIED. No pronouncement as to costs. 
government at the expense of the other. The procedure may not move as
expeditiously as some may desire, yet it ensures thorough deliberation of
all relevant and cognizable issues before one branch is compelled to yield SO ORDERED.
to the other. Moreover, judicial review does not preclude the legislative and
executive branches from negotiating a mutually acceptable solution to the
impasse. After all, the two branches, exercising as they do functions and
responsibilities that are political in nature, are free to smooth over the
thorns in their relationship with a salve of their own choosing.

And if emphasis be needed, if the courts so rule, the duty falls on


the shoulders of the President, as commander-in-chief, to authorize
the appearance of the military officers before Congress. Even if the
President has earlier disagreed with the notion of officers
G.R. No. 174318             October 17, 2006

PRESIDENTIAL COMMISSION ON GOOD GOVERNMENT (PCGG) and


CAMILO L. SABIO, Chairman, NARCISO S. NARIO, RICARDO M.
ABCEDE, TERESO L. JAVIER and NICASIO A. CONTI, Commissioners,
MANUEL ANDAL and JULIO JALANDONI, PCGG nominees to
Philcomsat Holdings Corporation, petitioners, 
vs.
RICHARD GORDON, in his capacity as Chairman, and MEMBERS OF
THE COMMITTEE ON GOVERNMENT CORPORATIONS AND PUBLIC
ENTERPRISES, MEMBERS OF THE COMMITTEE ON PUBLIC
SERVICES, SENATOR JUAN PONCE-ENRILE, in his capacity as
member of both said Committees, MANUEL VILLAR, Senate
President, THE SENATE SERGEANT-AT-ARMS, and SENATE OF THE
PHILIPPINES, respondents.

x --------------------------------------------------------------------------- x

G.R. No. 174177             October 17, 2006

PHILCOMSAT HOLDINGS CORPORATIONS, PHILIP G. BRODETT,


LUIS K. LOKIN, JR., ROBERTO V. SAN JOSE, DELFIN P. ANGCAO,
ROBERTO L. ABAD, ALMA KRISTINA ALOBBA, and JOHNNY
TAN, petitioners, 
vs.
SENATE COMMITTEE ON GOVERNMENT CORPORATIONS and PUBLIC
ENTERPRISES, its MEMBERS and CHAIRMAN, the HONORABLE
SENATOR RICHARD GORDON and SENATE COMMITTEE ON PUBLIC
G.R. No. 174340             October 17, 2006
SERVICES, its Members and Chairman, the HONORABLE SENATOR
JOKER P. ARROYO, respondents.
IN THE MATTER OF THE PETITION FOR ISSUANCE OF WRIT OF
HABEAS CORPUS OF CAMILO L. SABIO,petitioner,
J. ERMIN ERNEST LOUIE R. MIGUEL, petitioner-relator, 
vs.
HONORABLE SENATOR RICHARD GORDON, in his capacity as
Chairman, and the HONORABLE MEMBERS OF THE COMMITTEE ON
DECISION
GOVERNMENT CORPORATIONS AND PUBLIC ENTERPRISES and THE
COMMITTEE ON PUBLIC SERVICES of the Senate, HONORABLE
SENATOR JUAN PONCE-ENRILE, in his official capacity as Member,
HONORABLE MANUEL VILLAR, Senate President, SENATE
SERGEANT-AT-ARMS, and the SENATE OF THE
PHILIPPINES, respondents.
SANDOVAL-GUTIERREZ, J.:
x --------------------------------------------------------------------------- x
Two decades ago, on February 28, 1986, former President Corazon C.
Aquino installed her regime by issuing Executive Order (E.O.) No.
1,1 creating the Presidential Commission on Good Government (PCGG). She precipitately released P265 million and granted P125 million loan
entrusted upon this Commission the herculean task of recovering the ill- to a relative of an executive committee member; to date there
gotten wealth accumulated by the deposed President Ferdinand E. Marcos, have been no payments given, subjecting the company to an
his family, relatives, subordinates and close associates.2 Section 4 (b) of estimated interest income loss of P11.25 million in 2004;
E.O. No. 1 provides that: "No member or staff of the Commission shall
be required to testify or produce evidence in any judicial,
WHEREAS, there is an urgent need to protect the interest of the
legislative or administrative proceeding concerning matters within
Republic of the Philippines in the PHC, PHILCOMSAT, and POTC
its official cognizance." Apparently, the purpose is to ensure PCGG's
from any anomalous transaction, and to conserve or salvage any
unhampered performance of its task.3
remaining value of the government's equity position in these
corporations from any abuses of power done by their respective
Today, the constitutionality of Section 4(b) is being questioned on the board of directors;
ground that it tramples upon the Senate's power to conduct legislative
inquiry under Article VI, Section 21 of the 1987 Constitution, which reads:
WHEREFORE, be it resolved that the proper Senate
Committee shall conduct an inquiry in aid of legislation, on
The Senate or the House of Representatives or any of its the anomalous losses incurred by the Philippine Overseas
respective committees may conduct inquiries in aid of legislation Telecommunications Corporation (POTC), Philippine
in accordance with its duly published rules of procedure. The Communications Satellite Corporation (PHILCOMSAT), and
rights of persons appearing in or affected by such inquiries shall Philcomsat Holdings Corporations (PHC) due to the alleged
be respected. improprieties in the operations by their respective board of
directors. 
The facts are undisputed.
Adopted.
On February 20, 2006, Senator Miriam Defensor Santiago introduced
Philippine Senate Resolution No. 455 (Senate Res. No. 455),4 "directing an (Sgd) MIRIAM DEFENSOR SANTIAGO
inquiry in aid of legislation on the anomalous losses incurred by the
Philippines Overseas Telecommunications Corporation (POTC), Philippine
On the same date, February 20, 2006, Senate Res. No. 455 was submitted
Communications Satellite Corporation (PHILCOMSAT), and PHILCOMSAT
to the Senate and referred to the Committee on Accountability of Public
Holdings Corporation (PHC) due to the alleged improprieties in their
Officers and Investigations and Committee on Public Services. However, on
operations by their respective Board of Directors." 
March 28, 2006, upon motion of Senator Francis N. Pangilinan, it was
transferred to the Committee on Government Corporations and Public
The pertinent portions of the Resolution read: Enterprises.5

WHEREAS, in the last quarter of 2005, the representation and On May 8, 2006, Chief of Staff Rio C. Inocencio, under the authority of
entertainment expense of the PHC skyrocketed to P4.3 million, as Senator Richard J. Gordon, wrote Chairman Camilo L. Sabio of the PCGG,
compared to the previous year's mere P106 thousand; one of the herein petitioners, inviting him to be one of the resource
persons in the public meeting jointly conducted by the Committee on
Government Corporations and Public Enterprises and Committee on Public
WHEREAS, some board members established wholly owned PHC
Services. The purpose of the public meeting was to deliberate on Senate
subsidiary called Telecommunications Center, Inc. (TCI), where
Res. No. 455.6
PHC funds are allegedly siphoned; in 18 months, over P73 million
had been allegedly advanced to TCI without any accountability
report given to PHC and PHILCOMSAT; On May 9, 2006, Chairman Sabio declined the invitation because of prior
commitment.7 At the same time, he invoked Section 4(b) of E.O. No.
1 earlier quoted.
WHEREAS, the Philippine Star, in its 12 February 2002 issue
reported that the executive committee of Philcomsat has
On August 10, 2006, Senator Gordon issued a Subpoena Ad No member or staff of the Commission shall be
Testificandum,8 approved by Senate President Manuel Villar, requiring required to testify or produce evidence in any
Chairman Sabio and PCGG Commissioners Ricardo Abcede, Nicasio judicial legislative or administrative proceeding
Conti, Tereso Javier and Narciso Nario to appear in the public hearing concerning matters within its official cognizance.
scheduled on August 23, 2006 and testify on what they know relative to
the matters specified in Senate Res. No. 455. Similar subpoenae were
With all due respect, Section 4(b) of E.O. No. 1 constitutes a
issued against the directors and officers of Philcomsat Holdings
limitation on the power of legislative inquiry, and a recognition by
Corporation, namely: Benito V. Araneta, Philip J. Brodett, Enrique L. Locsin,
the State of the need to provide protection to the PCGG in order to
Manuel D. Andal, Roberto L. Abad, Luis K. Lokin, Jr., Julio J. Jalandoni,
ensure the unhampered performance of its duties under its
Roberto V. San Jose, Delfin P. Angcao, Alma Kristina Alloba and Johnny
charter. E.O. No. 1 is a law, Section 4(b) of which had not been
Tan.9
amended, repealed or revised in any way.

Again, Chairman Sabio refused to appear. In his letter to Senator Gordon


To say the least, it would require both Houses of Congress and
dated August 18, 2006, he reiterated his earlier position, invoking Section
Presidential fiat to amend or repeal the provision in controversy.
4(b) of E.O. No. 1. On the other hand, the directors and officers of
Until then, it stands to be respected as part of the legal system in
Philcomsat Holdings Corporation relied on the position paper they
this jurisdiction. (As held in People v. Veneracion, G.R. Nos.
previously filed, which raised issues on the propriety of legislative inquiry. 
119987-88, October 12, 1995: Obedience to the rule of law forms
the bedrock of our system of justice. If judges, under the guise of
Thereafter, Chief of Staff Ma. Carissa O. Coscolluela, under the authority of religious or political beliefs were allowed to roam unrestricted
Senator Gordon, sent another notice10to Chairman Sabio requiring him to beyond boundaries within which they are required by law to
appear and testify on the same subject matter set on September 6, 2006. exercise the duties of their office, then law becomes meaningless.
The notice was issued "under the same authority of the Subpoena Ad A government of laws, not of men excludes the exercise of broad
Testificandum previously served upon (him) last 16 August 2006." discretionary powers by those acting under its authority. Under
this system, judges are guided by the Rule of Law, and ought to
'protect and enforce it without fear or favor,' 4 [Act of Athens
Once more, Chairman Sabio did not comply with the notice. He sent a
(1955)] resist encroachments by governments, political parties, or
letter11 dated September 4, 2006 to Senator Gordon reiterating his reason
even the interference of their own personal beliefs.)
for declining to appear in the public hearing.

xxxxxx
This prompted Senator Gordon to issue an Order dated September 7, 2006
requiring Chairman Sabio and Commissioners Abcede, Conti, Javier and
Nario to show cause why they should not be cited in contempt of the Relevantly, Chairman Sabio's letter to Sen. Gordon dated August
Senate. On September 11, 2006, they submitted to the Senate their 19, 2006 pointed out that the anomalous transactions referred to
Compliance and Explanation,12 which partly reads: in the P.S. Resolution No. 455 are subject of pending cases before
the regular courts, the Sandiganbayan and the Supreme Court
(Pending cases include: a. Samuel Divina v. Manuel Nieto, Jr., et
Doubtless, there are laudable intentions of the subject
al., CA-G.R. No. 89102; b. Philippine Communications Satellite
inquiry in aid of legislation. But the rule of law requires that
Corporation v. Manuel Nieto, et al.; c. Philippine Communications
even the best intentions must be carried out within the
Satellite Corporation v. Manuel D. Andal, Civil Case No. 06-095,
parameters of the Constitution and the law. Verily, laudable
RTC, Branch 61, Makati City; d. Philippine Communications
purposes must be carried out by legal methods. (Brillantes, Jr., et
Satellite Corporation v. PHILCOMSAT Holdings Corporation, et al.,
al. v. Commission on Elections, En Banc [G.R. No. 163193, June
Civil Case No. 04-1049) for which reason they may not be able to
15, 2004])
testify thereon under the principle of sub judice. The laudable
objectives of the PCGG's functions, recognized in several cases
On this score, Section 4(b) of E.O. No. 1 should not be ignored as decided by the Supreme Court, of the PCGG will be put to naught
it explicitly provides: if its recovery efforts will be unduly impeded by a legislative
investigation of cases that are already pending before the Chairman Sabio, Commissioners Abcede, Conti, Nario, and Javier, and the
Sandiganbayan and trial courts. PCGG's nominees to Philcomsat Holdings Corporation, Manuel Andal and
Julio Jalandoni, likewise filed a petition for certiorari and prohibition against
the same respondents, and also against Senate President Manuel Villar,
In Bengzon v. Senate Blue Ribbon Committee, (203 SCRA 767,
Senator Juan Ponce Enrile, the Sergeant-at-Arms, and the entire Senate.
784 [1991]) the Honorable Supreme Court held:
The case was docketed as G.R. No. 174318.

"…[T]he issues sought to be investigated by the


Meanwhile, Philcomsat Holdings Corporation and its officers and directors,
respondent Committee is one over which jurisdiction had
namely: Philip G. Brodett, Luis K. Lokin, Jr., Roberto V. San Jose, Delfin P.
been acquired by the Sandiganbayan. In short, the issue
Angcao, Roberto L. Abad, Alma Kristina Alobba and Johnny Tan filed a
has been pre-empted by that court. To allow the
petition for certiorari and prohibition against the Senate Committees on
respondent Committee to conduct its own investigation of
Government Corporations and Public Enterprisesand Public Services, their
an issue already before the Sandigabayan would not only
Chairmen, Senators Gordon and Arroyo, and Members. The case was
pose the possibility of conflicting judgments between a
docketed as G.R. No. 174177.
legislative committee and a judicial tribunal, but if the
Committee's judgment were to be reached before that of
the Sandiganbayan, the possibility of its influence being In G.R. No. 174340 (for habeas corpus) and G.R. No. 174318 (for certiorari
made to bear on the ultimate judgment of the and prohibition) Chairman Sabio, Commissioners Abcede, Conti, Nario, and
Sandiganbayan can not be discounted. Javier; and the PCGG's nominees Andal and Jalandoni alleged: first,
respondent Senate Committees disregarded Section 4(b) of E.O. No. 1
without any justifiable reason; second, the inquiries conducted by
xxxxxx
respondent Senate Committees are not in aid of legislation; third, the
inquiries were conducted in the absence of duly published Senate Rules of
IT IS IN VIEW OF THE FOREGOING CONSIDERATIONS that Procedure Governing Inquiries in Aid of Legislation; and fourth, respondent
the Commission decided not to attend the Senate inquiry to testify Senate Committees are not vested with the power of contempt. 
and produce evidence thereat. 
In G.R. No. 174177, petitioners Philcomsat Holdings Corporation and its
Unconvinced with the above Compliance and Explanation, the Committee directors and officers alleged: first, respondent Senate Committees have no
on Government Corporations and Public Enterprises and the Committee on jurisdiction over the subject matter stated in Senate Res. No. 455; second,
Public Services issued an Order13 directing Major General Jose Balajadia the same inquiry is not in accordance with the Senate's Rules of Procedure
(Ret.), Senate Sergeant-At-Arms, to place Chairman Sabio and his Governing Inquiries in Aid of Legislation; third, the subpoenae against the
Commissioners under arrest for contempt of the Senate. The Order bears individual petitioners are void for having been issued without
the approval of Senate President Villar and the majority of the authority; fourth, the conduct of legislative inquiry pursuant to Senate Res.
Committees' members. No. 455 constitutes undue encroachment by respondents into justiciable
controversies over which several courts and tribunals have already
On September 12, 2006, at around 10:45 a.m., Major General Balajadia acquired jurisdiction; and fifth, the subpoenae violated petitioners' rights to
arrested Chairman Sabio in his office at IRC Building, No. 82 EDSA, privacy and against self-incrimination.
Mandaluyong City and brought him to the Senate premises where he was
detained.  In their Consolidated Comment, the above-named respondents
countered: first, the issues raised in the petitions involve political questions
Hence, Chairman Sabio filed with this Court a petition for habeas over which this Court has no jurisdiction; second, Section 4(b) has been
corpus against the Senate Committee on Government Corporations and repealed by the Constitution; third, respondent Senate Committees are
Public Enterprises and Committee on Public Services, their Chairmen, vested with contempt power; fourth, Senate's Rules of Procedure
Senators Richard Gordon and Joker P. Arroyo and Members. The case was Governing Inquiries in Aid of Legislation have been duly published; fifth,
docketed as G.R. No. 174340. respondents have not violated any civil right of the individual petitioners,
such as their (a) right to privacy; and (b) right against self-incrimination;
and sixth, the inquiry does not constitute undue encroachment into The Congress' power of inquiry has been recognized in foreign jurisdictions
justiciable controversies. long before it reached our shores through McGrain v. Daugherty,15 cited
in Arnault v. Nazareno.16 In those earlier days, American courts considered
the power of inquiry as inherent in the power to legislate. The 1864 case
During the oral arguments held on September 21, 2006, the parties were
of Briggs v. MacKellar17 explains the breath and basis of the power, thus:
directed to submit simultaneously their respective memoranda within a
non-extendible period of fifteen (15) days from date. In the meantime, per
agreement of the parties, petitioner Chairman Sabio was allowed to go Where no constitutional limitation or restriction exists, it is
home. Thus, his petition for habeas corpus has become moot. The parties competent for either of the two bodies composing the legislature
also agreed that the service of the arrest warrants issued against all to do, in their separate capacity, whatever may be essential to
petitioners and the proceedings before the respondent Senate Committees enable them to legislate….It is well-established principle of this
are suspended during the pendency of the instant cases.14 parliamentary law, that either house may institute any
investigationhaving reference to its own organization, the
conduct or qualification of its members, its proceedings, rights, or
Crucial to the resolution of the present petitions is the fundamental issue of
privileges or any matter affecting the public interest upon
whether Section 4(b) of E.O. No. 1 is repealed by the 1987
which it may be important that it should have exact
Constitution. On this lone issue hinges the merit of the contention of
information, and in respect to which it would be competent
Chairman Sabio and his Commissioners that their refusal to appear before
for it to legislate. The right to pass laws, necessarily
respondent Senate Committees is justified. With the resolution of this
implies the right to obtain information upon any matter
issue, all the other issues raised by the parties have become
which may become the subject of a law. It is essential to
inconsequential.
the full and intelligent exercise of the legislative
function….In American legislatures the investigation of
Perched on one arm of the scale of justice is Article VI, Section 21 of the public matters before committees, preliminary to
1987 Constitution granting respondent Senate Committees the power of legislation, or with the view of advising the house
legislative inquiry. It reads: appointing the committee is, as a parliamentary usage, well
established as it is in England, and the right of either house to
The Senate or the House of Representatives or any of its compel witnesses to appear and testify before its committee, and
respective committees may conduct inquiries in aid of to punish for disobedience has been frequently enforced….The
legislation in accordance with its duly published rules of right of inquiry, I think, extends to other matters, in respect to
procedure. The rights of persons appearing in or affected which it may be necessary, or may be deemed advisable to apply
by such inquiries shall be respected. for legislative aid.

On the other arm of the scale is Section 4(b) of E.O. No.1 limiting such Remarkably, in Arnault, this Court adhered to a similar theory. Citing
power of legislative inquiry by exempting all PCGG members or staff from McGrain, it recognized that the power of inquiry is "an essential and
testifying in any judicial, legislative or administrative proceeding, thus:  appropriate auxiliary to the legislative function," thus:

No member or staff of the Commission shall be required to Although there is no provision in the "Constitution expressly
testify or produce evidence in any judicial, legislative or investing either House of Congress with power to make
administrative proceeding concerning matters within its investigations and exact testimony to the end that it may exercise
official cognizance. its legislative functions advisedly and effectively, such power is so
far incidental to the legislative function as to be implied. In other
words, the power of inquiry – with process to enforce it – is
To determine whether there exists a clear and unequivocal repugnancy an essential and appropriate auxiliary to the legislative
between the two quoted provisions that warrants a declaration that Section function. A legislative body cannot legislate wisely or
4(b) has been repealed by the 1987 Constitution, a brief consideration of effectively in the absence of information respecting the
the Congress' power of inquiry is imperative.  conditions which the legislation is intended to affect or
change; and where the legislation body does not itself
possess the requisite information – which is not with utmost responsibility, integrity, loyalty, and efficiency, act with
infrequently true – recourse must be had to others who patriotism and justice, and lead modest lives." 
possess it."
The provision presupposes that since an incumbent of a public office is
Dispelling any doubt as to the Philippine Congress' power of inquiry, invested with certain powers and charged with certain duties pertinent to
provisions on such power made their maiden appearance in Article VIII, sovereignty, the powers so delegated to the officer are held in trust for
Section 12 of the 1973 Constitution.18 Then came the 1987 Constitution the people and are to be exercised in behalf of the government or
incorporating the present Article VI, Section 12. What was therefore of all citizens who may need the intervention of the officers. Such
implicit under the 1935 Constitution, as influenced by American trust extends to all matters within the range of duties pertaining to
jurisprudence, became explicit under the 1973 and 1987 Constitutions.19 the office. In other words, public officers are but the servants of
the people, and not their rulers.24
Notably, the 1987 Constitution recognizes the power of investigation, not
just of Congress, but also of "any of its committee." This is significant Section 4(b), being in the nature of an immunity, is inconsistent with
because it constitutes a direct conferral of investigatory power upon the the principle of public accountability. It places the PCGG members and
committees and it means that the mechanisms which the Houses can take staff beyond the reach of courts, Congress and other administrative bodies.
in order to effectively perform its investigative function are also available to Instead of encouraging public accountability, the same provision only
the committees.20 institutionalizes irresponsibility and non-accountability. In Presidential
Commission on Good Government v. Peña,25 Justice Florentino P. Feliciano
characterized as "obiter" the portion of the majority opinion barring, on the
It can be said that the Congress' power of inquiry has gained more solid
basis of Sections 4(a) and (b) of E.O. No. 1, a civil case for damages filed
existence and expansive construal. The Court's high regard to such power
against the PCGG and its Commissioners. He eloquently opined:
is rendered more evident in Senate v. Ermita,21 where it categorically ruled
that "the power of inquiry is broad enough to cover officials of the
executive branch." Verily, the Court reinforced the doctrine in Arnault The above underscored portions are, it is respectfully submitted,
that "the operation of government, being a legitimate subject for clearly obiter. It is important to make clear that the Court is
legislation, is a proper subject for investigation" and that "the not here interpreting, much less upholding as valid and
power of inquiry is co-extensive with the power to legislate." constitutional, the literal terms of Section 4 (a), (b) of
Executive Order No.1. If Section 4 (a) were given its literal
import as immunizing the PCGG or any member thereof from civil
Considering these jurisprudential instructions, we find Section 4(b) directly
liability "for anything done or omitted in the discharge of the task
repugnant with Article VI, Section 21. Section 4(b) exempts the PCGG
contemplated by this Order," the constitutionality of Section 4 (a)
members and staff from the Congress' power of inquiry. This cannot
would, in my submission, be open to most serious doubt. For so
be countenanced. Nowhere in the Constitution is any provision granting
viewed, Section 4 (a) would institutionalize the irresponsibility and
such exemption. The Congress' power of inquiry, being broad,
non-accountability of members and staff of the PCGG, a notion
encompasses everything that concerns the administration of existing laws
that is clearly repugnant to both the 1973 and 1987 Constitution
as well as proposed or possibly needed statutes.22 It even extends "to
and a privileged status not claimed by any other official of the
government agencies created by Congress and officers whose
Republic under the 1987 Constitution. x x x.
positions are within the power of Congress to regulate or even
abolish."23 PCGG belongs to this class.
xxxxxx
Certainly, a mere provision of law cannot pose a limitation to the broad
power of Congress, in the absence of any constitutional basis.  It would seem constitutionally offensive to suppose that a
member or staff member of the PCGG could not be required
to testify before the Sandiganbayan or that such members
Furthermore, Section 4(b) is also inconsistent with Article XI, Section 1 of
were exempted from complying with orders of this Court. 
the Constitution stating that: "Public office is a public trust. Public officers
and employees must at all times be accountable to the people, serve them
Chavez v. Sandiganbayan26 reiterates the same view. Indeed, Section 4(b) Consequently, the conduct of inquiries in aid of legislation is not only
has been frowned upon by this Court even before the filing of the present intended to benefit Congress but also the citizenry. The people are equally
petitions.  concerned with this proceeding and have the right to participate therein in
order to protect their interests. The extent of their participation will largely
depend on the information gathered and made known to them. In other
Corollarily, Section 4(b) also runs counter to the following constitutional
words, the right to information really goes hand-in-hand with the
provisions ensuring the people's access to information: 
constitutional policies of full public disclosure and honesty in the public
service. It is meant to enhance the widening role of the citizenry in
Article II, Section 28 governmental decision-making as well as in checking abuse in the
government.28 The cases of Tañada v. Tuvera29and Legaspi v. Civil Service
Subject to reasonable conditions prescribed by law, the Commission30 have recognized a citizen's interest and personality to
State adopts and implements a policy of full public enforce a public duty and to bring an action to compel public officials and
disclosure of all its transactions involving public interest. employees to perform that duty. 

Article III, Section 7 Section 4(b) limits or obstructs the power of Congress to secure from PCGG
members and staff information and other data in aid of its power to
legislate. Again, this must not be countenanced. In Senate v. Ermita,31 this
The right of the people to information on matters of Court stressed:
public concern shall be recognized. Access to official
records, and to documents, and papers pertaining to
official acts, transactions, or decisions, as well as to To the extent that investigations in aid of legislation are generally
government research data used as basis for policy conducted in public, however, any executive issuance tending
development, shall be afforded the citizen, subject to to unduly limit disclosures of information in such
such limitations as may be provided by law. investigations necessarily deprives the people of
information which, being presumed to be in aid of
legislation, is presumed to be a matter of public concern.
These twin provisions of the Constitution seek to promote transparency in The citizens are thereby denied access to information which they
policy-making and in the operations of the government, as well as provide can use in formulating their own opinions on the matter before
the people sufficient information to enable them to exercise effectively Congress – opinions which they can then communicate to their
their constitutional rights. Armed with the right information, citizens can representatives and other government officials through the
participate in public discussions leading to the formulation of government various legal means allowed by their freedom of expression.
policies and their effective implementation. In Valmonte v. Belmonte,
Jr.27 the Court explained that an informed citizenry is essential to the
existence and proper functioning of any democracy, thus: A statute may be declared unconstitutional because it is not within the
legislative power to enact; or it creates or establishes methods or forms
that infringe constitutional principles; or its purpose or effect violates the
An essential element of these freedoms is to keep open a Constitution or its basic principles. 32 As shown in the above discussion,
continuing dialogue or process of communication between the Section 4(b) is inconsistent with Article VI, Section 21 (Congress' power
government and the people. It is in the interest of the State that of inquiry), Article XI, Section 1 (principle of public
the channels for free political discussion be maintained to the end accountability), Article II, Section 28 (policy of full disclosure)
that the government may perceive and be responsive to the and Article III, Section 7 (right to public information). 
people's will. Yet, this open dialogue can be effective only to the
extent that the citizenry is informed and thus able to formulate its
will intelligently. Only when the participants in the discussion are Significantly, Article XVIII, Section 3 of the Constitution provides:
aware of the issues and have access to information relating
thereto can such bear fruit. All existing laws, decrees, executive orders, proclamations, letters
of instructions, and other executive issuances not
inconsistent with this Constitution shall remain operative until CHAIRMAN SABIO:
amended, repealed, or revoked.
Your Honor, my father was a judge, died being a judge. I
The clear import of this provision is that all existing laws, executive orders, was here in the Supreme Court as Chief of Staff of Justice
proclamations, letters of instructions and other executive issuances Feria. I would definitely honor the Supreme Court and
inconsistent or repugnant to the Constitution are repealed. the rule of law.

Jurisprudence is replete with decisions invalidating laws, decrees, executive CHIEF JUSTICE PANGANIBAN:
orders, proclamations, letters of instructions and other executive issuances
inconsistent with the Constitution. In Pelaez v. Auditor General,33 the Court
You will answer the questions of the Senators if we say
considered repealed Section 68 of the Revised Administrative Code of 1917
that?
authorizing the Executive to change the seat of the government of any
subdivision of local governments, upon the approval of the 1935
Constitution. Section 68 was adjudged incompatible and inconsistent with CHAIRMAN SABIO: 
the Constitutional grant of limited executive supervision over local
governments. In Islamic Da'wah Council of the Philippines, Inc., v. Office of Yes, Your Honor. That is the law already as far as I am
the Executive Secretary,34 the Court declared Executive Order No. 46, concerned.
entitled "Authorizing the Office on Muslim Affairs to Undertake Philippine
Halal Certification," void for encroaching on the religious freedom of
Muslims. In The Province of Batangas v. Romulo,35 the Court declared some With his admission, Chairman Sabio is not fully convinced that he and his
provisions of the General Appropriations Acts of 1999, 2000 and 2001 Commissioners are shielded from testifying before respondent Senate
unconstitutional for violating the Constitutional precept on local autonomy. Committees by Section 4(b) of E.O. No. 1. In effect, his argument that the
And in Ople v. Torres,36the Court likewise declared unconstitutional said provision exempts him and his co-respondent Commissioners from
Administrative Order No. 308, entitled "Adoption of a National testifying before respondent Senate Committees concerning Senate Res.
Computerized Identification Reference System," for being violative of the No. 455 utterly lacks merit.
right to privacy protected by the Constitution.
Incidentally, an argument repeated by Chairman Sabio is that respondent
These Decisions, and many others, highlight that the Constitution is the Senate Committees have no power to punish him and his Commissioners
highest law of the land. It is "the basic and paramount law to which all for contempt of the Senate. 
other laws must conform and to which all persons, including the
highest officials of the land, must defer. No act shall be valid, The argument is misleading. 
however noble its intentions, if it conflicts with the
Constitution."37 Consequently, this Court has no recourse but to declare
Section 4(b) of E.O. No. 1 repealed by the 1987 Constitution.  Article VI, Section 21 provides:

Significantly, during the oral arguments on September 21, 2006, Chairman The Senate or the House of Representatives or any of its
Sabio admitted that should this Court rule that Section 4(b) is respective committees may conduct inquiries in aid of
unconstitutional or that it does not apply to the Senate, he will answer the legislation in accordance with its duly published rules of
questions of the Senators, thus: procedure. The rights of persons appearing in or affected
by such inquiries shall be respected.

CHIEF JUSTICE PANGANIBAN:


It must be stressed that the Order of Arrest for "contempt of Senate
Committees and the Philippine Senate" was approved by Senate
Okay. Now, if the Supreme Court rules that Sec. 4(b) is President Villar and signed by fifteen (15) Senators. From this, it can
unconstitutional or that it does not apply to the Senate, be concluded that the Order is under the authority, not only of the
will you answer the questions of the Senators?  respondent Senate Committees, but of the entire Senate.
At any rate, Article VI, Section 21 grants the power of inquiry not only to In Arnault v. Balagtas,42 the Court further explained that the contempt
the Senate and the House of Representatives, but also to any of their power of Congress is founded upon reason and policy and that the power of
respective committees. Clearly, there is a direct conferral of power to inquiry will not be complete if for every contumacious act, Congress has to
the committees. Father Bernas, in his Commentary on the 1987 resort to judicial interference, thus:
Constitution, correctly pointed out its significance: 
The principle that Congress or any of its bodies has the power
It should also be noted that the Constitution explicitly recognizes to punish recalcitrant witnesses is founded upon reason and
the power of investigation not just of Congress but also of "any of policy. Said power must be considered implied or incidental to the
its committees." This is significant because it constitutes a exercise of legislative power. How could a legislative body
direct conferral of investigatory power upon the obtain the knowledge and information on which to base
committees and it means that the means which the Houses intended legislation if it cannot require and compel the
can take in order to effectively perform its investigative disclosure of such knowledge and information if it is
function are also available to the Committees. 38 impotent to punish a defiance of its power and authority?
When the framers of the Constitution adopted the principle
of separation of powers, making each branch supreme
This is a reasonable conclusion. The conferral of the legislative power of
within the realm of its respective authority, it must have
inquiry upon any committee of Congress must carry with it all powers
intended each department's authority to be full and
necessary and proper for its effective discharge. Otherwise, Article VI,
complete, independently of the other's authority or power.
Section 21 will be meaningless. The indispensability and usefulness of the
And how could the authority and power become complete if
power of contempt in a legislative inquiry is underscored in a catena of
for every act of refusal, every act of defiance, every act of
cases, foreign and local. 
contumacy against it, the legislative body must resort to
the judicial department for the appropriate remedy,
In the 1821 case of Anderson v. Dunn,39 the function of the Houses of because it is impotent by itself to punish or deal therewith,
Congress with respect to the contempt power was likened to that of a with the affronts committed against its authority or
court, thus: dignity.43

…But the court in its reasoning goes beyond this, and though the In Negros Oriental II Electric Cooperative, Inc. v. Sangguniang Panlungsod
grounds of the decision are not very clearly stated, we take them of Dumaguete,44 the Court characterized contempt power as a matter of
to be: that there is in some cases a power in each House of self-preservation, thus:
Congress to punish for contempt; that this power is
analogous to that exercised by courts of justice, and that it
The exercise by the legislature of the contempt power is a matter
being the well established doctrine that when it appears
of self-preservation as that branch of the government vested
that a prisoner is held under the order of a court of general
with the legislative power, independently of the judicial branch,
jurisdiction for a contempt of its authority, no other court
asserts its authority and punishes contempts thereof. The
will discharge the prisoner or make further inquiry into the
contempt power of the legislature is, therefore, sui generis x x x.
cause of his commitment. That this is the general rule…as
regards the relation of one court to another must be conceded.
Meanwhile, with respect to G.R. No. 174177, the petition of Philcomsat
Holdings Corporation and its directors and officers, this Court holds that the
In McGrain,40 the U.S. Supreme Court held: "Experience has shown that
respondent Senate Committees' inquiry does not violate their right to
mere requests for such information are often unavailing, and also
privacy and right against self-incrimination. 
that information which is volunteered is not always accurate or
complete; so some means of compulsion is essential to obtain what
is needed." The Court, in Arnault v. Nazareno,41sustained the Congress' One important limitation on the Congress' power of inquiry is that "the
power of contempt on the basis of this observation. rights of persons appearing in or affected by such inquiries shall be
respected." This is just another way of saying that the power of inquiry
must be "subject to the limitations placed by the Constitution on
government action." As held in Barenblatt v. United States,45"the improprieties in the operations by their respective board of
Congress, in common with all the other branches of the directors." Obviously, the inquiry focus on petitioners' acts committed in
Government, must exercise its powers subject to the limitations the discharge of their duties as officers and directors of the said
placed by the Constitution on governmental action, more corporations, particularly Philcomsat Holdings Corporation. Consequently,
particularly in the context of this case, the relevant limitations of they have no reasonable expectation of privacy over matters
the Bill of Rights."  involving their offices in a corporation where the government has
interest. Certainly, such matters are of public concern and over
which the people have the right to information. 
First is the right to privacy.

This goes to show that the right to privacy is not absolute where there is
Zones of privacy are recognized and protected in our laws.46 Within these
an overriding compelling state interest. In Morfe v. Mutuc,51 the Court,
zones, any form of intrusion is impermissible unless excused by law and in
in line with Whalen v. Roe,52 employed the rational basis relationship test
accordance with customary legal process. The meticulous regard we accord
when it held that there was no infringement of the individual's right to
to these zones arises not only from our conviction that the right to privacy
privacy as the requirement to disclosure information is for a valid purpose,
is a "constitutional right" and "the right most valued by civilized
i.e., to curtail and minimize the opportunities for official corruption,
men,"47 but also from our adherence to the Universal Declaration of Human
maintain a standard of honesty in public service, and promote morality in
Rights which mandates that, "no one shall be subjected to arbitrary
public administration.53 In Valmonte v. Belmonte,54 the Court remarked
interference with his privacy" and "everyone has the right to the protection
that as public figures, the Members of the former Batasang Pambansa
of the law against such interference or attacks."48
enjoy a more limited right to privacy as compared to ordinary
individuals, and their actions are subject to closer scrutiny. Taking this into
Our Bill of Rights, enshrined in Article III of the Constitution, provides at consideration, the Court ruled that the right of the people to access
least two guarantees that explicitly create zones of privacy. It highlights a information on matters of public concern prevails over the right to privacy
person's "right to be let alone" or the "right to determine what, how much, of financial transactions. 
to whom and when information about himself shall be disclosed."49 Section
2 guarantees "the right of the people to be secure in their persons,
Under the present circumstances, the alleged anomalies in the
houses, papers and effects against unreasonable searches and
PHILCOMSAT, PHC and POTC, ranging in millions of pesos, and the
seizures of whatever nature and for any purpose." Section 3 renders
conspiratorial participation of the PCGG and its officials are compelling
inviolable the "privacy of communication and correspondence" and
reasons for the Senate to exact vital information from the directors and
further cautions that "any evidence obtained in violation of this or the
officers of Philcomsat Holdings Corporations, as well as from Chairman
preceding section shall be inadmissible for any purpose in any
Sabio and his Commissioners to aid it in crafting the necessary legislation
proceeding."
to prevent corruption and formulate remedial measures and policy
determination regarding PCGG's efficacy. There being no reasonable
In evaluating a claim for violation of the right to privacy, a court must expectation of privacy on the part of those directors and officers over the
determine whether a person has exhibited a reasonable expectation of subject covered by Senate Res. No. 455, it follows that their right to
privacy and, if so, whether that expectation has been violated by privacy has not been violated by respondent Senate Committees.
unreasonable government intrusion.50 Applying this determination to these
cases, the important inquiries are: first, did the directors and officers of
Anent the right against self-incrimination, it must be emphasized that this
Philcomsat Holdings Corporation exhibit a reasonable expectation of
right maybe invoked by the said directors and officers of Philcomsat
privacy?; and second, did the government violate such expectation?
Holdings Corporation only when the incriminating question is being
asked, since they have no way of knowing in advance the nature or
The answers are in the negative. Petitioners were invited in the Senate's effect of the questions to be asked of them."55 That this right
public hearing to deliberate on Senate Res. No. 455, particularly "on the may possibly be violated or abused is no ground for denying respondent
anomalous losses incurred by the Philippine Overseas Senate Committees their power of inquiry. The consolation is that when
Telecommunications Corporation (POTC), Philippine this power is abused, such issue may be presented before the courts. At
Communications Satellite Corporation (PHILCOMSAT), and this juncture, what is important is that respondent Senate Committees
Philcomsat Holdings Corporations (PHC) due to the alleged
have sufficient Rules to guide them when the right against self- Testificandum issued by respondent Senate Committees directing them to
incrimination is invoked. Sec. 19 reads: appear and testify in public hearings relative to Senate Resolution No. 455.

Sec. 19. Privilege Against Self-Incrimination WHEREFORE, the petition in G.R. No. 174340 for habeas
corpus is DISMISSED, for being moot. The petitions in G.R Nos. 174318
and 174177 are likewise DISMISSED. 
A witness can invoke his right against self-incrimination only when
a question tends to elicit an answer that will incriminate him is
propounded to him. However, he may offer to answer any Section 4(b) of E.O. No. 1 is declared REPEALED by the 1987 Constitution.
question in an executive session. Respondent Senate Committees' power of inquiry relative to Senate
Resolution 455 is upheld. PCGG Chairman Camilo L. Sabio and
Commissioners Ricardo Abcede, Narciso Nario, Nicasio Conti and Tereso
No person can refuse to testify or be placed under oath or
Javier; and Manuel Andal and Julio Jalandoni, PCGG's nominees to
affirmation or answer questions before an incriminatory question
Philcomsat Holdings Corporation, as well as its directors and officers,
is asked. His invocation of such right does not by itself excuse him
petitioners in G.R. No. 174177, are ordered to comply with
from his duty to give testimony.
the Subpoenae Ad Testificandum issued by respondent Senate Committees
directing them to appear and testify in public hearings relative to Senate
In such a case, the Committee, by a majority vote of the members Resolution No. 455.
present there being a quorum, shall determine whether the right
has been properly invoked. If the Committee decides otherwise, it
SO ORDERED.
shall resume its investigation and the question or questions
previously refused to be answered shall be repeated to the
witness. If the latter continues to refuse to answer the question, Panganiban, C.J., Puno, Quisumbing, Ynares-Santiago, Carpio, Austria-
the Committee may punish him for contempt for contumacious Martinez, Corona, Carpio Morales, Callejo, Sr., Azcuna, Chico-Nazario,
conduct. Tinga, Garcia, and Velasco, JJ., concur.

The same directors and officers contend that the Senate is barred from
inquiring into the same issues being litigated before the Court of Appeals
and the Sandiganbayan. Suffice it to state that the Senate Rules of
G.R. No. 180643             March 25, 2008
Procedure Governing Inquiries in Aid of Legislation provide that the filing or
pendency of any prosecution of criminal or administrative action should not
stop or abate any inquiry to carry out a legislative purpose.  ROMULO L. NERI, petitioner, 
vs.
SENATE COMMITTEE ON ACCOUNTABILITY OF PUBLIC OFFICERS
Let it be stressed at this point that so long as the constitutional rights of
AND INVESTIGATIONS, SENATE COMMITTEE ON TRADE AND
witnesses, like Chairman Sabio and his Commissioners, will be respected
COMMERCE, AND SENATE COMMITTEE ON NATIONAL DEFENSE AND
by respondent Senate Committees, it their duty to cooperate with them in
SECURITY, respondents.
their efforts to obtain the facts needed for intelligent legislative action. The
unremitting obligation of every citizen is to respond to subpoenae, to
respect the dignity of the Congress and its Committees, and to testify fully DECISION
with respect to matters within the realm of proper investigation.
LEONARDO-DE CASTRO, J.:
In fine, PCGG Chairman Camilo Sabio and Commissioners Ricardo Abcede,
Narciso Nario, Nicasio Conti, and Tereso Javier; and Manuel Andal and Julio At bar is a petition for certiorari under Rule 65 of the Rules of Court
Jalandoni, PCGG's nominees to Philcomsat Holdings Corporation, as well as assailing the show cause Letter1 dated November 22, 2007 and contempt
its directors and officers, must comply with the Subpoenae Ad Order2 dated January 30, 2008 concurrently issued by respondent 
Senate Committees on Accountability of Public Officers and (4) P.S. Res. No. 136, introduced by Senator Miriam Defensor
Investigations,3 Trade and Commerce,4 and National Defense and Santiago, entitled RESOLUTION DIRECTING THE PROPER SENATE
Security5 against petitioner Romulo L. Neri, former Director General of the COMMITTEE TO CONDUCT AN INQUIRY, IN AID OF LEGISLATION,
National Economic and Development Authority (NEDA). ON THE LEGAL AND ECONOMIC JUSTIFICATION OF THE
NATIONAL BROADBAND NETWORK (NBN) PROJECT OF THE
NATIONAL GOVERNMENT.
The facts, as culled from the pleadings, are as follows:

At the same time, the investigation was claimed to be relevant to the


On April 21, 2007, the Department of Transportation and Communication
consideration of three (3) pending bills in the Senate, to wit: 
(DOTC) entered into a contract with Zhong Xing Telecommunications
Equipment (ZTE) for the supply of equipment and services for the National
Broadband Network (NBN) Project in the amount of U.S. $ 329,481,290 1. Senate Bill No. 1793, introduced by Senator Mar Roxas,
(approximately P16 Billion Pesos). The Project was to be financed by the entitled AN ACT SUBJECTING TREATIES, INTERNATIONAL OR
People's Republic of China. EXECUTIVE AGREEMENTS INVOLVING FUNDING IN THE
PROCUREMENT OF INFRASTRUCTURE PROJECTS, GOODS, AND
CONSULTING SERVICES TO BE INCLUDED IN THE SCOPE AND
In connection with this NBN Project, various Resolutions were introduced in
APPLICATION OF PHILIPPINE PROCUREMENT LAWS, AMENDING
the Senate, as follows: 
FOR THE PURPOSE REPUBLIC ACT NO. 9184, OTHERWISE KNOWN
AS THE GOVERNMENT PROCUREMENT REFORM ACT, AND FOR
(1) P.S. Res. No. 127, introduced by Senator Aquilino Q. OTHER PURPOSES;
Pimentel, Jr., entitled RESOLUTION DIRECTING THE BLUE RIBBON
COMMITTEE AND THE COMMITTEE ON TRADE AND INDUSTRY TO
2. Senate Bill No. 1794, introduced by Senator Mar Roxas,
INVESTIGATE, IN AID OF LEGISLATION, THE CIRCUMSTANCES
entitled AN ACT IMPOSING SAFEGUARDS IN CONTRACTING
LEADING TO THE APPROVAL OF THE BROADBAND CONTRACT
LOANS CLASSIFIED AS OFFICIAL DEVELOPMENT ASSISTANCE,
WITH ZTE AND THE ROLE PLAYED BY THE OFFICIALS CONCERNED
AMENDING FOR THE PURPOSE REPUBLIC ACT NO. 8182, AS
IN GETTING IT CONSUMMATED AND TO MAKE
AMENDED BY REPUBLIC ACT NO. 8555, OTHERWISE KNOWN AS
RECOMMENDATIONS TO HALE TO THE COURTS OF LAW THE
THE OFFICIAL DEVELOPMENT ASSISTANCE ACT OF 1996, AND
PERSONS RESPONSIBLE FOR ANY ANOMALY IN CONNECTION
FOR OTHER PURPOSES; and
THEREWITH AND TO PLUG THE LOOPHOLES, IF ANY IN THE BOT
LAW AND OTHER PERTINENT LEGISLATIONS.
3. Senate Bill No. 1317, introduced by Senator Miriam Defensor
Santiago, entitled AN ACT MANDATING CONCURRENCE TO
(2) P.S. Res. No. 144, introduced by Senator Mar Roxas, entitled
INTERNATIONAL AGREEMENTS AND EXECUTIVE AGREEMENTS.
Á RESOLUTION URGING PRESIDENT GLORIA MACAPAGAL
ARROYO TO DIRECT THE CANCELLATION OF THE ZTE CONTRACT
Respondent Committees initiated the investigation by sending invitations to
certain personalities and cabinet officials involved in the NBN Project.
(3) P.S. Res. No. 129, introduced by Senator Panfilo M. Lacson,
Petitioner was among those invited. He was summoned to appear and
entitled RESOLUTION DIRECTING THE COMMITTEE ON NATIONAL
testify on September 18, 20, and 26 and October 25, 2007. However, he
DEFENSE AND SECURITY TO CONDUCT AN INQUIRY IN AID OF
attended only the September 26 hearing, claiming he was "out of town"
LEGISLATION INTO THE NATIONAL SECURITY IMPLICATIONS OF
during the other dates. 
AWARDING THE NATIONAL BROADBAND NETWORK CONTRACT TO
THE CHINESE FIRM ZHONG XING TELECOMMUNICATIONS
EQUIPMENT COMPANY LIMITED (ZTE CORPORATION) WITH THE In the September 18, 2007 hearing, businessman Jose de Venecia III
END IN VIEW OF PROVIDING REMEDIAL LEGISLATION THAT WILL testified that several high executive officials and power brokers were using
PROTECT OUR NATIONAL SOVEREIGNTY, SECURITY AND their influence to push the approval of the NBN Project by the NEDA. It
TERRITORIAL INTEGRITY. appeared that the Project was initially approved as a Build-Operate-
Transfer (BOT) project but, on March 29, 2007, the NEDA acquiesced to
convert it into a government-to-government project, to be financed c) Whether the President said to go ahead and
through a loan from the Chinese Government. approve the project after being told about the
alleged bribe?
On September 26, 2007, petitioner testified before respondent Committees
for eleven (11) hours. He disclosed that then Commission on Elections Following the ruling in Senate v. Ermita, the foregoing questions
(COMELEC) Chairman Benjamin Abalos offered him P200 Million in fall under conversations and correspondence between the
exchange for his approval of the NBN Project. He further narrated that he President and public officials which are considered executive
informed President Arroyo about the bribery attempt and that she privilege (Almonte v. Vasquez, G.R. 95637, 23 May 1995; Chavez
instructed him not to accept the bribe. However, when probed further on v. PEA, G.R. 133250, July 9, 2002). Maintaining the confidentiality
what they discussed about the NBN Project, petitioner refused to answer, of conversations of the President is necessary in the exercise of
invoking "executive privilege". In particular, he refused to answer the her executive and policy decision making process. The expectation
questions on (a) whether or not President Arroyo followed up the NBN of a President to the confidentiality of her conversations and
Project,6 (b) whether or not she directed him to prioritize it,7 and (c) correspondences, like the value which we accord deference for the
whether or not she directed him to approve.8 privacy of all citizens, is the necessity for protection of the public
interest in candid, objective, and even blunt or harsh opinions in
Presidential decision-making. Disclosure of conversations of the
Unrelenting, respondent Committees issued a Subpoena Ad
President will have a chilling effect on the President, and will
Testificandum to petitioner, requiring him to appear and testify on
hamper her in the effective discharge of her duties and
November 20, 2007. 
responsibilities, if she is not protected by the confidentiality of her
conversations.
However, in the Letter dated November 15, 2007, Executive Secretary
Eduardo R. Ermita requested respondent Committees to dispense with
The context in which executive privilege is being invoked is that
petitioner's testimony on the ground of executive privilege. The pertinent
the information sought to be disclosed might impair our diplomatic
portion of the letter reads:
as well as economic relations with the People's Republic of China.
Given the confidential nature in which these information were
With reference to the subpoena ad testificandum issued to conveyed to the President, he cannot provide the Committee any
Secretary Romulo Neri to appear and testify again on 20 further details of these conversations, without disclosing the very
November 2007 before the Joint Committees you chair, it will be thing the privilege is designed to protect.
recalled that Sec. Neri had already testified and exhaustively
discussed the ZTE / NBN project, including his conversation with
In light of the above considerations, this Office is constrained to
the President thereon last 26 September 2007.
invoke the settled doctrine of executive privilege as refined
in Senate v. Ermita, and has advised Secretary Neri accordingly.
Asked to elaborate further on his conversation with the President,
Sec. Neri asked for time to consult with his superiors in line with
Considering that Sec. Neri has been lengthily interrogated on the
the ruling of the Supreme Court in Senate v. Ermita, 488 SCRA 1
subject in an unprecedented 11-hour hearing, wherein he has
(2006).
answered all questions propounded to him except the foregoing
questions involving executive privilege, we therefore request that
Specifically, Sec. Neri sought guidance on the possible invocation his testimony on 20 November 2007 on the ZTE / NBN project be
of executive privilege on the following questions, to wit: dispensed with.

a) Whether the President followed up the (NBN) On November 20, 2007, petitioner did not appear before respondent
project? Committees. Thus, on November 22, 2007, the latter issued the show
cause Letter requiring him to explain why he should not be cited in
b) Were you dictated to prioritize the ZTE? contempt. The Letter reads:
Since you have failed to appear in the said hearing, the In the interim, on December 7, 2007, petitioner filed with this Court the
Committees on Accountability of Public Officers and Investigations present petition for certiorari assailing the show cause Letter dated
(Blue Ribbon), Trade and Commerce and National Defense and November 22, 2007.
Security require you to show cause why you should not be cited in
contempt under Section 6, Article 6 of the Rules of the Committee
Respondent Committees found petitioner's explanations unsatisfactory.
on Accountability of Public Officers and Investigations (Blue
Without responding to his request for advance notice of the matters that he
Ribbon).
should still clarify, they issued the Order dated January 30, 2008, citing
him in contempt of respondent Committees and ordering his arrest and
The Senate expects your explanation on or before 2 December detention at the Office of the Senate Sergeant-At-Arms until such time that
2007. he would appear and give his testimony. The said Order states:

On November 29, 2007, petitioner replied to respondent Committees, ORDER


manifesting that it was not his intention to ignore the Senate hearing and
that he thought the only remaining questions were those he claimed to be
For failure to appear and testify in the Committee's hearing on
covered by executive privilege, thus: 
Tuesday, September 18, 2007; Thursday, September 20, 2007;
Thursday, October 25, 2007; and Tuesday, November 20, 2007,
It was not my intention to snub the last Senate hearing. In fact, I despite personal notice and Subpoenas Ad Testificandum sent to
have cooperated with the task of the Senate in its inquiry in aid of and received by him, which thereby delays, impedes and
legislation as shown by my almost 11 hours stay during the obstructs, as it has in fact delayed, impeded and obstructed the
hearing on 26 September 2007. During said hearing, I answered inquiry into the subject reported irregularities, AND for failure to
all the questions that were asked of me, save for those which I explain satisfactorily why he should not be cited for contempt
thought was covered by executive privilege, and which was (Neri letter of 29 November 2007), herein attached) ROMULO L.
confirmed by the Executive Secretary in his Letter 15 November NERI is hereby cited in contempt of this (sic) Committees
2007. In good faith, after that exhaustive testimony, I thought and ordered arrested and detained in the Office of the
that what remained were only the three questions, where the Senate Sergeant-At-Arms until such time that he will
Executive Secretary claimed executive privilege. Hence, his appear and give his testimony.
request that my presence be dispensed with.
The Sergeant-At-Arms is hereby directed to carry out and
Be that as it may, should there be new matters that were not yet implement this Order and make a return hereof within twenty four
taken up during the 26 September 2007 hearing, may I be (24) hours from its enforcement.
furnished in advance as to what else I need to clarify, so that as a
resource person, I may adequately prepare myself.
SO ORDERED.

In addition, petitioner submitted a letter prepared by his counsel, Atty.


On the same date, petitioner moved for the reconsideration of the above
Antonio R. Bautista, stating, among others that: (1) his (petitioner) non-
Order.9 He insisted that he has not shown "any contemptible conduct
appearance was upon the order of the President; and (2) his conversation
worthy of contempt and arrest." He emphasized his willingness to testify on
with President Arroyo dealt with delicate and sensitive national security and
new matters, however, respondent Committees did not respond to his
diplomatic matters relating to the impact of the bribery scandal involving
request for advance notice of questions. He also mentioned the petition
high government officials and the possible loss of confidence of foreign
for certiorari he filed on December 7, 2007. According to him, this should
investors and lenders in the Philippines. The letter ended with a reiteration
restrain respondent Committees from enforcing the show cause Letter
of petitioner's request that he "be furnished in advance" as to what else he
"through the issuance of declaration of contempt" and arrest.
needs to clarify so that he may adequately prepare for the hearing.

In view of respondent Committees' issuance of the contempt Order,


petitioner filed on February 1, 2008 a Supplemental Petition for
Certiorari (With Urgent Application for TRO/Preliminary Injunction), seeking of her executive and policy decision-making
to restrain the implementation of the said contempt Order.  and (ii) information, which might impair our diplomatic
as well as economic relations with the People's Republic
of China?
On February 5, 2008, the Court issued a Status Quo Ante Order (a)
enjoining respondent Committees from implementing their contempt Order,
(b) requiring the parties to observe the status quo prevailing prior to the 1.b. Did petitioner Neri correctly invoke executive
issuance of the assailed order, and (c) requiring respondent Committees to privilege to avoid testifying on his conversations with the
file their comment. President on the NBN contract on his assertions that the
said conversations "dealt with delicate and sensitive
national security and diplomatic matters relating to
Petitioner contends that respondent Committees' show cause Letter and
the impact of bribery scandal involving high
contempt Order were issued with grave abuse of discretion amounting to
government officials and the possible loss of
lack or excess of jurisdiction. He stresses that his conversations with
confidence of foreign investors and lenders in the
President Arroyo are "candid discussions meant to explore options in
Philippines" x x x within the principles laid down in
making policy decisions." According to him, these discussions "dwelt on
Senate v. Ermita (488 SCRA 1 [2006])?
the impact of the bribery scandal involving high government
officials on the country's diplomatic relations and economic and
military affairs and the possible loss of confidence of foreign 1.c Will the claim of executive privilege in this case
investors and lenders in the Philippines." He also emphasizes that his violate the following provisions of the Constitution:
claim of executive privilege is upon the order of the President and within
the parameters laid down in Senate v. Ermita10 and United States v.
Sec. 28, Art. II (Full public disclosure of all
Reynolds.11Lastly, he argues that he is precluded from disclosing
transactions involving public interest)
communications made 

Sec. 7, Art. III (The right of the people to


to him in official confidence under Section 712 of Republic Act No. 6713,
information on matters of public concern)
otherwise known as Code of Conduct and Ethical Standards for Public
Officials and Employees, and Section 2413 (e) of Rule 130 of the Rules of
Court. Sec. 1, Art. XI (Public office is a public trust)

Respondent Committees assert the contrary. They argue that (1) Sec. 17, Art. VII (The President shall ensure
petitioner's testimony is material and pertinent in the investigation that the laws be faithfully executed)
conducted in aid of legislation; (2) there is no valid justification for
petitioner to claim executive privilege; (3) there is no abuse of their and the due process clause and the principle of separation of
authority to order petitioner's arrest; and (4) petitioner has not come to powers?
court with clean hands. 

2. What is the proper procedure to be followed in invoking


In the oral argument held last March 4, 2008, the following issues were executive privilege?
ventilated:

3. Did the Senate Committees gravely abuse their discretion in


1. What communications between the President and petitioner ordering the arrest of petitioner for non-compliance with the
Neri are covered by the principle of 'executive privilege'? subpoena?

1.a Did Executive Secretary Ermita correctly invoke the After the oral argument, the parties were directed to manifest to the Court
principle of executive privilege, by order of the President, within twenty-four (24) hours if they are amenable to the Court's proposal
to cover (i) conversations of the President in the exercise of allowing petitioner to immediately resume his testimony before the
Senate Committees to answer the other questions of the Senators without At the core of this controversy are the two (2) crucial queries, to wit: 
prejudice to the decision on the merits of this pending petition. It was
understood that petitioner may invoke executive privilege in the course of
First, are the communications elicited by the subject three (3)
the Senate Committees proceedings, and if the respondent Committees
questions covered by executive privilege? 
disagree thereto, the unanswered questions will be the subject of a
supplemental pleading to be resolved along with the three (3) questions
subject of the present petition.14 At the same time, respondent Committees And second, did respondent Committees commit grave abuse of
were directed to submit several pertinent documents.15 discretion in issuing the contempt Order?

The Senate did not agree with the proposal for the reasons stated in the We grant the petition.
Manifestation dated March 5, 2008. As to the required documents, the
Senate and respondent Committees manifested that they would not be able At the outset, a glimpse at the landmark case of Senate v.
to submit the latter's "Minutes of all meetings" and the "Minute Book" Ermita18 becomes imperative. Senate draws in bold strokes the distinction
because it has never been the "historical and traditional legislative practice between the legislative and oversight powers of the Congress, as
to keep them."16 They instead submitted the Transcript of Stenographic embodied under Sections 21 and 22, respectively, of Article VI of the
Notes of respondent Committees' joint public hearings. Constitution, to wit: 

On March 17, 2008, the Office of the Solicitor General (OSG) filed a Motion SECTION 21. The Senate or the House of Representatives or any
for Leave to Intervene and to Admit Attached Memorandum, founded on of its respective committees may conduct inquiries in aid of
the following arguments: legislation in accordance with its duly published rules of
procedure. The rights of persons appearing in or affected by such
(1) The communications between petitioner and the President are inquiries shall be respected. 
covered by the principle of "executive privilege."
SECTION 22. The heads of department may upon their own
(2) Petitioner was not summoned by respondent Senate initiative, with the consent of the President, or upon the request of
Committees in accordance with the law-making body's power to either House, or as the rules of each House shall provide, appear
conduct inquiries in aid of legislation as laid down in Section 21, before and be heard by such House on any matter pertaining to
Article VI of the Constitution and Senate v. Ermita. their departments. Written questions shall be submitted to the
President of the Senate or the Speaker of the House of
Representatives at least three days before their scheduled
(3) Respondent Senate Committees gravely abused its discretion
appearance. Interpellations shall not be limited to written
for alleged non-compliance with the Subpoena dated November
questions, but may cover matters related thereto. When the
13, 2007.
security of the state or the public interest so requires and the
President so states in writing, the appearance shall be conducted
The Court granted the OSG's motion the next day, March 18, 2008.  in executive session.

As the foregoing facts unfold, related events transpired.  Senate cautions that while the above provisions are closely related and
complementary to each other, they should not be considered as pertaining
to the same power of Congress. Section 21 relates to the power to conduct
On March 6, 2008, President Arroyo issued Memorandum Circular No. 151,
inquiries in aid of legislation. Its aim is to elicit information that may be
revoking Executive Order No. 464 and Memorandum Circular No. 108. She
used for legislation. On the other hand, Section 22 pertains to the power to
advised executive officials and employees to follow and abide by the
conduct a question hour, the objective of which is to obtain information in
Constitution, existing laws and jurisprudence, including, among others, the
pursuit of Congress' oversight function.19 Simply stated, while both powers
case of Senate v. Ermita17 when they are invited to legislative inquiries in
allow Congress or any of its committees to conduct inquiry,
aid of legislation.
their objectives are different. 
This distinction gives birth to another distinction with regard to the use of The Communications Elicited by the Three (3) Questions are
compulsory process. Unlike in Section 21, Congress cannot compel the Covered by Executive Privilege
appearance of executive officials under Section 22. The Court's
pronouncement in Senate v. Ermita20 is clear:
We start with the basic premises where the parties have conceded.

When Congress merely seeks to be informed on how department


The power of Congress to conduct inquiries in aid of legislation is broad.
heads are implementing the statutes which it has issued, its right
This is based on the proposition that a legislative body cannot legislate
to such information is not as imperative as that of the President to
wisely or effectively in the absence of information respecting the conditions
whom, as Chief Executive, such department heads must give a
which the legislation is intended to affect or change.21 Inevitably, adjunct
report of their performance as a matter of duty. In such instances,
thereto is the compulsory process to enforce it. But, the power, broad as it
Section 22, in keeping with the separation of powers, states that
is, has limitations. To be valid, it is imperative that it is done in accordance
Congress may only request their appearance. Nonetheless, when
with the Senate or House duly published rules of procedure and that the
the inquiry in which Congress requires their appearance is 'in aid
rights of the persons appearing in or affected by such inquiries be
of legislation' under Section 21, the appearance is mandatory for
respected. 
the same reasons stated in Arnault.

The power extends even to executive officials and the only way for them to
In fine, the oversight function of Congress may be
be exempted is through a valid claim of executive privilege.22 This directs
facilitated by compulsory process only to the extent that it
us to the consideration of the question -- is there a recognized claim of
is performed in pursuit of legislation. This is consistent with
executive privilege despite the revocation of E.O. 464?
the intent discerned from the deliberations of the Constitutional
Commission 
A- There is a Recognized Claim
of Executive Privilege Despite the
Ultimately, the power of Congress to compel the appearance of
Revocation of E.O. 464
executive officials under section 21 and the lack of it under
Section 22 find their basis in the principle of separation of powers.
While the executive branch is a co-equal branch of the legislature, At this juncture, it must be stressed that the revocation of E.O. 464 does
it cannot frustrate the power of Congress to legislate by refusing not in any way diminish our concept of executive privilege. This is because
to comply with its demands for information. (Emphasis supplied.)  this concept has Constitutional underpinnings. Unlike the United States
which has further accorded the concept with statutory status by enacting
the Freedom of Information Act23 and the Federal Advisory Committee
The availability of the power of judicial review to resolve the issues raised
Act,24 the Philippines has retained its constitutional origination, occasionally
in this case has also been settled in Senate v. Ermita, when it held:
interpreted only by this Court in various cases. The most recent of these is
the case of Senate v. Ermita where this Court declared unconstitutional
As evidenced by the American experience during the so-called substantial portions of E.O. 464. In this regard, it is worthy to note that
"McCarthy era," however, the right of Congress to conduct Executive Ermita's Letter dated November 15, 2007 limits its bases for the
inquiries in aid of legislation is, in theory, no less susceptible to claim of executive privilege to Senate v. Ermita, Almonte v.
abuse than executive or judicial power. It may thus be subjected Vasquez,25 and Chavez v. PEA.26 There was never a mention of E.O. 464. 
to judicial review pursuant to the Court's certiorari powers under
Section 1, Article VIII of the Constitution.
While these cases, especially Senate v. Ermita,27 have comprehensively
discussed the concept of executive privilege, we deem it imperative to
Hence, this decision. explore it once more in view of the clamor for this Court to clearly define
the communications covered by executive privilege. 
I
The Nixon and post-Watergate cases established the broad contours of
the presidential communications privilege.28 In United States v.
Nixon,29 the U.S. Court recognized a great public interest in preserving The situation in Judicial Watch, Inc. v. Department of Justice33 tested
"the confidentiality of conversations that take place in the the In Re: Sealed Case principles. There, while the presidential decision
President's performance of his official duties." It thus considered involved is the exercise of the President's pardon power, a non-delegable,
presidential communications as "presumptively privileged." Apparently, core-presidential function, the Deputy Attorney General and the Pardon
the presumption is founded on the "President's generalized interest in Attorney were deemed to be too remote from the President and his senior
confidentiality." The privilege is said to be necessary to guarantee the White House advisors to be protected. The Court conceded that 
candor of presidential advisors and to provide "the President and those
who assist him… with freedom to explore alternatives in the
functionally those officials were performing a task directly related to the
process of shaping policies and making decisions and to do so in a
President's pardon power, but concluded that an organizational test was
way many would be unwilling to express except privately."
more appropriate for confining the potentially broad sweep that would
result from the In Re: Sealed Case's functional test. The majority
In In Re: Sealed Case,30 the U.S. Court of Appeals delved deeper. It ruled concluded that, the lesser protections of the deliberative process privilege
that there are two (2) kinds of executive privilege; one is the presidential would suffice. That privilege was, however, found insufficient to justify the
communications privilege and, the other is the deliberative process confidentiality of the 4,341 withheld documents. 
privilege. The former pertains to "communications, documents or
other materials that reflect presidential decision-making and
But more specific classifications of communications covered by executive
deliberations and that the President believes should remain
privilege are made in older cases. Courts ruled early that the Executive has
confidential." The latter includes 'advisory opinions,
a right to withhold documents that might reveal military or state
recommendations and deliberations comprising part of a process by
secrets,34identity of government informers in some
which governmental decisions and policies are formulated." 
circumstances,,35 and information related to pending
investigations.36 An area where the privilege is highly revered is
Accordingly, they are characterized by marked distinctions. Presidential in foreign relations. In United States v. Curtiss-Wright Export Corp.37 the
communications privilege applies to decision-making of the U.S. Court, citing President George Washington, pronounced:
President while, the deliberative process privilege, to decision-
making of executive officials. The first is rooted in the constitutional
The nature of foreign negotiations requires caution, and their
principle of separation of power and the President's unique constitutional
success must often depend on secrecy, and even when brought to
role; the second on common law privilege. Unlike the deliberative
a conclusion, a full disclosure of all the measures, demands, or
process privilege, the presidential communications
eventual concessions which may have been proposed or
privilege applies to documents in their entirety, and covers final and
contemplated would be extremely impolitic, for this might have a
post-decisional materials as well as pre-deliberative ones31 As a
pernicious influence on future negotiations or produce immediate
consequence, congressional or judicial negation of the presidential
inconveniences, perhaps danger and mischief, in relation to other
communications privilege is always subject to greater scrutiny than
powers. The necessity of such caution and secrecy was one cogent
denial of the deliberative process privilege. 
reason for vesting the power of making treaties in the President,
with the advice and consent of the Senate, the principle on which
Turning on who are the officials covered by the presidential the body was formed confining it to a small number of members.
communications privilege, In Re: Sealed Caseconfines the privilege only To admit, then, a right in the House of Representatives to demand
to White House Staff that has "operational proximity" to direct presidential and to have as a matter of course all the papers respecting a
decision-making. Thus, the privilege is meant to encompass only those negotiation with a foreign power would be to establish a
functions that form the core of presidential authority, involving what the dangerous precedent.
court characterized as "quintessential and non-delegable Presidential
power," such as commander-in-chief power, appointment and removal
Majority of the above jurisprudence have found their way in our
power, the power to grant pardons and reprieves, the sole-authority to
jurisdiction. In Chavez v. PCGG38, this Court held that there is a
receive ambassadors and other public officers, the power to negotiate
"governmental privilege against public disclosure with respect to state
treaties, etc.32
secrets regarding military, diplomatic and other security matters."
In Chavez v. PEA,39 there is also a recognition of the confidentiality of
Presidential conversations, correspondences, and discussions in closed-
door Cabinet meetings. In Senate v. Ermita, the concept of presidential the power to enter into an executive agreement with other countries. This
communications privilege is fully discussed. authority of the President to enter into executive agreements without the
concurrence of the Legislature has traditionally been recognized in
Philippine jurisprudence.45 Second, the communications are "received" by
As may be gleaned from the above discussion, the claim of executive
a close advisor of the President. Under the "operational proximity" test,
privilege is highly recognized in cases where the subject of inquiry relates
petitioner can be considered a close advisor, being a member of President
to a power textually committed by the Constitution to the President, such
Arroyo's cabinet. And third, there is no adequate showing of a compelling
as the area of military and foreign relations. Under our Constitution, the
need that would justify the limitation of the privilege and of
President is the repository of the commander-in-
the unavailability of the information elsewhere by an appropriate
chief,40appointing,41 pardoning,42 and diplomatic43 powers. Consistent with
investigating authority. 
the doctrine of separation of powers, the information relating to these
powers may enjoy greater confidentiality than others.
The third element deserves a lengthy discussion.
The above cases, especially, Nixon, In Re Sealed Case and Judicial Watch,
somehow provide the elements of presidential communications United States v. Nixon held that a claim of executive privilege is subject
privilege, to wit: to balancing against other interest. In other words, confidentiality in
executive privilege is not absolutely protected by the Constitution. The
U.S. Court held:
1) The protected communication must relate to a "quintessential
and non-delegable presidential power."
[N]either the doctrine of separation of powers, nor the need for
confidentiality of high-level communications, without more, can
2) The communication must be authored or "solicited and
sustain an absolute, unqualified Presidential privilege of immunity
received" by a close advisor of the President or the President
from judicial process under all circumstances. 
himself. The judicial test is that an advisor must be in "operational
proximity" with the President.
The foregoing is consistent with the earlier case of Nixon v. Sirica,46 where
it was held that presidential communications are presumptively
3) The presidential communications privilege remains a
privileged and that the presumption can be overcome only by mere
qualified privilege that may be overcome by a showing of
showing of public need by the branch seeking access to conversations. The
adequate need, such that the information sought "likely contains
courts are enjoined to resolve the competing interests of the political
important evidence" and by the unavailability of the information
branches of the government "in the manner that preserves the essential
elsewhere by an appropriate investigating authority.44
functions of each Branch."47 Here, the record is bereft of any categorical
explanation from respondent Committees to show a compelling or citical
In the case at bar, Executive Secretary Ermita premised his claim of need for the answers to the three (3) questions in the enactment of a law.
executive privilege on the ground that the communications elicited by the Instead, the questions veer more towards the exercise of the legislative
three (3) questions "fall under conversation and correspondence between oversight function under Section 22 of Article VI rather than Section 21 of
the President and public officials" necessary in "her executive and policy the same Article. Senate v. Ermita ruled that the "the oversight function
decision-making process" and, that "the information sought to be disclosed of Congress may be facilitated by compulsory process only to the
might impair our diplomatic as well as economic relations with the People's extent that it is performed in pursuit of legislation." It is conceded
Republic of China." Simply put, the bases are presidential that it is difficult to draw the line between an inquiry in aid of
communications privilege and executive privilege on matters relating legislation and an inquiry in the exercise of oversight function of Congress.
to diplomacy or foreign relations. In this regard, much will depend on the content of the questions and the
manner the inquiry is conducted.
Using the above elements, we are convinced that, indeed, the
communications elicited by the three (3) questions are covered by Respondent Committees argue that a claim of executive privilege does not
the presidential communications privilege. First, the communications guard against a possible disclosure of a crime or wrongdoing. We see no
relate to a "quintessential and non-delegable power" of the President, i.e. dispute on this. It is settled in United States v. Nixon48 that "demonstrated,
specific need for evidence in pending criminal trial" outweighs the task requires and our decision implies no judgment
President's "generalized interest in confidentiality." However, the present whatever concerning possible presidential involvement in
case's distinction with the Nixon case is very evident. In Nixon, there is a culpable activity. On the contrary, we think the sufficiency
pending criminal proceeding where the information is requested and it is of the Committee's showing must depend solely on whether
the demands of due process of law and the fair administration of criminal the subpoenaed evidence is demonstrably critical to the
justice that the information be disclosed. This is the reason why the U.S. responsible fulfillment of the Committee's functions.
Court was quick to "limit the scope of its decision." It stressed that it is
"not concerned here with the balance between the President's
In its initial briefs here, the Committee argued that it has shown
generalized interest in confidentiality x x x and congressional
exactly this. It contended that resolution, on the basis of the
demands for information." Unlike in Nixon, the information here is
subpoenaed tapes, of the conflicts in the testimony before it
elicited, not in a criminal proceeding, but in a legislative inquiry. In this
'would aid in a determination whether legislative involvement in
regard, Senate v. Ermita stressed that the validity of the claim of executive
political campaigns is necessary' and 'could help engender the
privilege depends not only on the ground invoked but, also, on
public support needed for basic reforms in our electoral system.'
the procedural setting or the context in which the claim is made.
Moreover, Congress has, according to the Committee, power to
Furthermore, in Nixon, the President did not interpose any claim of need to
oversee the operations of the executive branch, to investigate
protect military, diplomatic or sensitive national security secrets. In the
instances of possible corruption and malfeasance in office, and to
present case, Executive Secretary Ermita categorically claims executive
expose the results of its investigations to public view. The
privilege on the grounds of presidential communications privilege in
Committee says that with respect to Watergate-related matters,
relation to her executive and policy decision-making process and diplomatic
this power has been delegated to it by the Senate, and that to
secrets. 
exercise its power responsibly, it must have access to the
subpoenaed tapes. 
The respondent Committees should cautiously tread into the investigation
of matters which may present a conflict of interest that may provide a
We turn first to the latter contention. In the circumstances of this
ground to inhibit the Senators participating in the inquiry if later on an
case, we need neither deny that the Congress may have, quite
impeachment proceeding is initiated on the same subject matter of the
apart from its legislative responsibilities, a general oversight
present Senate inquiry. Pertinently, in Senate Select Committee on
power, nor explore what the lawful reach of that power might be
Presidential Campaign Activities v. Nixon,49 it was held that since an
under the Committee's constituent resolution. Since passage of
impeachment proceeding had been initiated by a House Committee, the
that resolution, the House Committee on the Judiciary has begun
Senate Select Committee's immediate oversight need for five presidential
an inquiry into presidential impeachment. The investigative
tapes should give way to the House Judiciary Committee which has the
authority of the Judiciary Committee with respect to presidential
constitutional authority to inquire into presidential impeachment. The Court
conduct has an express constitutional source. x x x We have
expounded on this issue in this wise:
been shown no evidence indicating that Congress itself
attaches any particular value to this interest. In these
It is true, of course, that the Executive cannot, any more than the circumstances, we think the need for the tapes premised
other branches of government, invoke a general confidentiality solely on an asserted power to investigate and inform
privilege to shield its officials and employees from investigations cannot justify enforcement of the Committee's subpoena.
by the proper governmental institutions into possible criminal
wrongdoing. The Congress learned this as to its own privileges
The sufficiency of the Committee's showing of need has come to
in Gravel v. United States, as did the judicial branch, in a sense,
depend, therefore, entirely on whether the subpoenaed materials
in Clark v. United States, and the executive branch itself in Nixon
are critical to the performance of its legislative functions. There is
v. Sirica. But under Nixon v. Sirica, the showing required to
a clear difference between Congress' legislative tasks and the
overcome the presumption favoring confidentiality turned,
responsibility of a grand jury, or any institution engaged in like
not on the nature of the presidential conduct that the subpoenaed
functions. While fact-finding by a legislative committee is
material might reveal, but, instead, on the nature and
undeniably a part of its task, legislative judgments
appropriateness of the function in the performance of
normally depend more on the predicted consequences of
which the material was sought, and the degree to which
proposed legislative actions and their political
the material was necessary to its fulfillment. Here also our
acceptability, than on precise reconstruction of past More than anything else, though, the right of Congress or any of its
events; Congress frequently legislates on the basis of conflicting Committees to obtain information in aid of legislation cannot be equated
information provided in its hearings. In contrast, the responsibility with the people's right to public information. The former cannot claim that
of the grand jury turns entirely on its ability to determine whether every legislative inquiry is an exercise of the people's right to information.
there is probable cause to believe that certain named individuals The distinction between such rights is laid down in Senate v. Ermita:
did or did not commit specific crimes. If, for example, as in Nixon
v. Sirica, one of those crimes is perjury concerning the content of
There are, it bears noting, clear distinctions between the right of
certain conversations, the grand jury's need for the most precise
Congress to information which underlies the power of inquiry and
evidence, the exact text of oral statements recorded in their
the right of people to information on matters of public concern.
original form, is undeniable. We see no comparable need in
For one, the demand of a citizen for the production of documents
the legislative process, at least not in the circumstances of
pursuant to his right to information does not have the same
this case. Indeed, whatever force there might once have been in
obligatory force as a subpoena duces tecum issued by Congress.
the Committee's argument that the subpoenaed materials are
Neither does the right to information grant a citizen the power to
necessary to its legislative judgments has been substantially
exact testimony from government officials. These powers belong
undermined by subsequent events. (Emphasis supplied)
only to Congress, not to an individual citizen.

Respondent Committees further contend that the grant of petitioner's claim


Thus, while Congress is composed of representatives
of executive privilege violates the constitutional provisions on the right of
elected by the people, it does not follow, except in a highly
the people to information on matters of public concern.50 We might have
qualified sense, that in every exercise of its power of
agreed with such contention if petitioner did not appear before them at all.
inquiry, the people are exercising their right to
But petitioner made himself available to them during the September 26
information. 
hearing, where he was questioned for eleven (11) hours. Not only that, he
expressly manifested his willingness to answer more questions from the
Senators, with the exception only of those covered by his claim of The members of respondent Committees should not invoke as justification
executive privilege. in their exercise of power a right properly belonging to the people in
general. This is because when they discharge their power, they do so as
public officials and members of Congress. Be that as it may, the right to
The right to public information, like any other right, is subject to limitation.
information must be balanced with and should give way, in appropriate
Section 7 of Article III provides:
cases, to constitutional precepts particularly those pertaining to delicate
interplay of executive-legislative powers and privileges which is the subject
The right of the people to information on matters of public concern of careful review by numerous decided cases.
shall be recognized. Access to official records, and to documents,
and papers pertaining to official acts, transactions, or decisions, as
B- The Claim of Executive Privilege
well as to government research data used as basis for policy
is Properly Invoked
development, shall be afforded the citizen, subject to such
limitations as may be provided by law.
We now proceed to the issue -- whether the claim is properly invoked
by the President. Jurisprudence teaches that for the claim to be properly
The provision itself expressly provides the limitation, i.e. as may be
invoked, there must be a formal claim of privilege, lodged by the head of
provided by law. Some of these laws are Section 7 of Republic Act (R.A.)
the department which has control over the matter."56 A formal and proper
No. 6713,51 Article 22952 of the Revised Penal Code, Section 3 (k)53 of R.A.
claim of executive privilege requires a "precise and certain reason" for
No. 3019, and Section 24(e)54 of Rule 130 of the Rules of Court. These are
preserving their confidentiality.57
in addition to what our body of jurisprudence classifies as confidential55 and
what our Constitution considers as belonging to the larger concept of
executive privilege. Clearly, there is a recognized public interest in the The Letter dated November 17, 2007 of Executive Secretary Ermita
confidentiality of certain information. We find the information subject of satisfies the requirement. It serves as the formal claim of privilege. There,
this case belonging to such kind.  he expressly states that "this Office is constrained to invoke the
settled doctrine of executive privilege as refined in Senate v.
Ermita, and has advised Secretary Neri accordingly." Obviously, he is that he thought the only remaining questions were the three (3) questions
referring to the Office of the President. That is more than enough he claimed to be covered by executive privilege. In addition thereto, he
compliance. In Senate v. Ermita, a less categorical letter was even submitted Atty. Bautista's letter, stating that his non-appearance was upon
adjudged to be sufficient. the order of the President and specifying the reasons why his conversations
with President Arroyo are covered by executive privilege. Both
correspondences include an expression of his willingness to testify
With regard to the existence of "precise and certain reason," we find the
again, provided he "be furnished in advance" copies of the
grounds relied upon by Executive Secretary Ermita specific enough so as
questions. Without responding to his request for advance list of questions,
not "to leave respondent Committees in the dark on how the requested
respondent Committees issued the Order dated January 30, 2008, citing
information could be classified as privileged." The case of Senate v.
him in contempt of respondent Committees and ordering his arrest and
Ermita only requires that an allegation be made "whether the information
detention at the Office of the Senate Sergeant-At-Arms until such time that
demanded involves military or diplomatic secrets, closed-door Cabinet
he would appear and give his testimony. Thereupon, petitioner filed a
meetings, etc." The particular ground must only be specified. The
motion for reconsideration, informing respondent Committees that he had
enumeration is not even intended to be comprehensive."58 The following
filed the present petition for certiorari. 
statement of grounds satisfies the requirement:

Respondent Committees committed grave abuse of discretion in issuing the


The context in which executive privilege is being invoked is that
contempt Order in view of five (5) reasons. 
the information sought to be disclosed might impair our diplomatic
as well as economic relations with the People's Republic of China.
Given the confidential nature in which these information were First, there being a legitimate claim of executive privilege, the issuance of
conveyed to the President, he cannot provide the Committee any the contempt Order suffers from constitutional infirmity.
further details of these conversations, without disclosing the very
thing the privilege is designed to protect.
Second, respondent Committees did not comply with the requirement laid
down in Senate v. Ermita that the invitations should contain the "possible
At any rate, as held further in Senate v. Ermita, 59 the Congress must not needed statute which prompted the need for the inquiry," along with "the
require the executive to state the reasons for the claim with such usual indication of the subject of inquiry and the questions relative to and
particularity as to compel disclosure of the information which the privilege in furtherance thereof." Compliance with this requirement is imperative,
is meant to protect. This is a matter of respect to a coordinate and co- both under Sections 21 and 22 of Article VI of the Constitution. This must
equal department.  be so to ensure that the rights of both persons appearing in or
affected by such inquiry are respected as mandated by said Section 21
and by virtue of the express language of Section 22. Unfortunately, despite
II
petitioner's repeated demands, respondent Committees did not send him
an advance list of questions. 
Respondent Committees Committed Grave Abuse of Discretion 
in Issuing the Contempt Order
Third, a reading of the transcript of respondent Committees' January 30,
2008 proceeding reveals that only a minority of the members of the Senate
Grave abuse of discretion means "such capricious and whimsical exercise of Blue Ribbon Committee was present during the deliberation. 61 Section 18
judgment as is equivalent to lack of jurisdiction, or, in other words where of the Rules of Procedure Governing Inquiries in Aid of Legislation provides
the power is exercised in an arbitrary or despotic manner by reason of that: 
passion or personal hostility and it must be so patent and gross as to
amount to an evasion of positive duty or to a virtual refusal to perform the
"The Committee, by a vote of majority of all its members, may
duty enjoined or to act at all in contemplation of law."60
punish for contempt any witness before it who disobeys any order
of the Committee or refuses to be sworn or to testify or to answer
It must be reiterated that when respondent Committees issued the show proper questions by the Committee or any of its members." 
cause Letter dated November 22, 2007, petitioner replied immediately,
manifesting that it was not his intention to ignore the Senate hearing and
Clearly, the needed vote is a majority of all the members of the statements. I was merely mentioning that under Section 6 of the
Committee. Apparently, members who did not actually participate in the Rules of the Committee and under Section 6, "The Committee by a
deliberation were made to sign the contempt Order. Thus, there is a cloud vote of a majority of all its members may punish for contempt any
of doubt as to the validity of the contempt Order dated January 30, 2008. witness before it who disobeys any order of the Committee."
We quote the pertinent portion of the transcript, thus: 
So the Blue Ribbon Committee is more than willing to take that
THE CHAIRMAN (SEN. CAYETANO, A). For clarification. x x x responsibility. But we only have six members here today, I
The Chair will call either a caucus or will ask the Committee am the seventh as chair and so we have not met that
on Rules if there is a problem. Meaning, if we do not have number. So I am merely stating that, sir, that when we will
the sufficient numbers. But if we have a sufficient number, prepare the documentation, if a majority of all members sign and
we will just hold a caucus to be able to implement that I am following the Sabio v. Gordon rule wherein I do believe, if I
right away because…Again, our Rules provide that any one am not mistaken, Chairman Gordon prepared the documentation
held in contempt and ordered arrested, need the and then either in caucus or in session asked the other members
concurrence of a majority of all members of the said to sign. And once the signatures are obtained, solely for the
committee and we have three committees conducting this. purpose that Secretary Neri or Mr. Lozada will not be able to
legally question our subpoena as being insufficient in accordance
with law.
So thank you very much to the members…

SEN. PIMENTEL. Mr. Chairman, the caution that the chair is


SEN. PIMENTEL. Mr. Chairman.
suggesting is very well-taken. But I'd like to advert to the fact that
the quorum of the committee is only two as far as I remember.
THE CHAIRMAN (SEN. CAYETANO,A). May I recognize the Any two-member senators attending a Senate committee hearing
Minority Leader and give him the floor, Senator Pimentel. provide that quorum, and therefore there is more than a quorum
demanded by our Rules as far as we are concerned now, and
SEN. PIMENTEL. Mr. Chairman, there is no problem, I think, acting as Blue Ribbon Committee, as Senator Enrile pointed out.
with consulting the other committees. But I am of the In any event, the signatures that will follow by the additional
opinion that the Blue Ribbon Committee is the lead members will only tend to strengthen the determination of this
committee, and therefore, it should have preference in Committee to put its foot forward – put down on what is
enforcing its own decisions. Meaning to say, it is not happening in this country, Mr. Chairman, because it really looks
something that is subject to consultation with other terrible if the primary Committee of the Senate, which is the Blue
committees. I am not sure that is the right interpretation. I Ribbon Committee, cannot even sanction people who openly defy,
think that once we decide here, we enforce what we you know, the summons of this Committee. I know that the Chair
decide, because otherwise, before we know it, our is going through an agonizing moment here. I know that. But
determination is watered down by delay and, you know, the nonetheless, I think we have to uphold, you know, the institution
so-called "consultation" that inevitably will have to take that we are representing because the alternative will be a disaster
place if we follow the premise that has been explained. for all of us, Mr. Chairman. So having said that, I'd like to
reiterate my point.

So my suggestion, Mr. Chairman, is the Blue Ribbon Committee


should not forget it's the lead committee here, and therefore, the THE CHAIRMAN (SEN. CAYETANO, A.) First of all, I agree 100
will of the lead committee prevails over all the other, you, know percent with the intentions of the Minority Leader. But let me
reservations that other committees might have who are only very respectfully disagree with the legal requirements.
secondary or even tertiary committees, Mr. Chairman. Because, yes, we can have a hearing if we are only two but
both under Section 18 of the Rules of the Senate and under
Section 6 of the Rules of the Blue Ribbon Committee, there
THE CHAIRMAN (SEN. CAYETANO, A.) Thank you very much to is a need for a majority of all members if it is a case of
the Minority Leader. And I agree with the wisdom of his contempt and arrest. So, I am simply trying to avoid the court
rebuking the Committee, which will instead of strengthening will Even the courts are repeatedly advised to exercise the power of contempt
weaken us. But I do agree, Mr. Minority Leader, that we should judiciously and sparingly with utmost self-restraint with the end in view of
push for this and show the executive branch that the well-decided utilizing the same for correction and preservation of the dignity of the
– the issue has been decided upon the Sabio versus Gordon case. court, not for retaliation or vindication.63 Respondent Committees should
And it's very clear that we are all allowed to call witnesses. And if have exercised the same restraint, after all petitioner is not even an
they refure or they disobey not only can we cite them in contempt ordinary witness. He holds a high position in a co-equal branch of
and have them arrested. x x x 62 government.

Fourth, we find merit in the argument of the OSG that respondent In this regard, it is important to mention that many incidents of judicial
Committees likewise violated Section 21 of Article VI of the Constitution, review could have been avoided if powers are discharged with
requiring that the inquiry be in accordance with the "duly published rules circumspection and deference. Concomitant with the doctrine of separation
of procedure." We quote the OSG's explanation: of powers is the mandate to observe respect to a co-equal branch of the
government. 
The phrase 'duly published rules of procedure' requires the Senate
of every Congress to publish its rules of procedure governing One last word.
inquiries in aid of legislation because every Senate is distinct from
the one before it or after it. Since Senatorial elections are held
The Court was accused of attempting to abandon its constitutional duty
every three (3) years for one-half of the Senate's membership,
when it required the parties to consider a proposal that would lead to a
the composition of the Senate also changes by the end of each
possible compromise. The accusation is far from the truth. The Court did
term. Each Senate may thus enact a different set of rules as it
so, only to test a tool that other jurisdictions find to be effective in settling
may deem fit. Not having published its Rules of Procedure,
similar cases, to avoid a piecemeal consideration of the questions for
the subject hearings in aid of legislation conducted by the
review and to avert a constitutional crisis between the executive and
14th Senate, are therefore, procedurally infirm. 
legislative branches of government.

And fifth, respondent Committees' issuance of the contempt Order is


In United States v. American Tel. & Tel Co.,64 the court refrained from
arbitrary and precipitate. It must be pointed out that respondent
deciding the case because of its desire to avoid a resolution that might
Committees did not first pass upon the claim of executive privilege and
disturb the balance of power between the two branches and inaccurately
inform petitioner of their ruling. Instead, they curtly dismissed his
reflect their true needs. Instead, it remanded the record to the District
explanation as "unsatisfactory" and simultaneously issued the Order citing
Court for further proceedings during which the parties are required to
him in contempt and ordering his immediate arrest and detention. 
negotiate a settlement. In the subsequent case of United States v.
American Tel. &Tel Co.,65 it was held that "much of this spirit of
A fact worth highlighting is that petitioner is not an unwilling witness. compromise is reflected in the generality of language found in the
He manifested several times his readiness to testify before respondent Constitution." It proceeded to state:
Committees. He refused to answer the three (3) questions because he was
ordered by the President to claim executive privilege. It behooves
Under this view, the coordinate branches do not exist in an
respondent Committees to first rule on the claim of executive privilege and
exclusively adversary relationship to one another when a conflict
inform petitioner of their finding thereon, instead of peremptorily
in authority arises. Rather each branch should take cognizance of
dismissing his explanation as "unsatisfactory." Undoubtedly, respondent
an implicit constitutional mandate to seek optimal accommodation
Committees' actions constitute grave abuse of discretion for being arbitrary
through a realistic evaluation of the needs of the conflicting
and for denying petitioner due process of law. The same quality afflicted
branches in the particular fact situation. 
their conduct when they (a) disregarded petitioner's motion for
reconsideration alleging that he had filed the present petition before this
Court and (b) ignored petitioner's repeated request for an advance list of It thereafter concluded that: "The Separation of Powers often impairs
questions, if there be any aside from the three (3) questions as to which he efficiency, in terms of dispatch and the immediate functioning of
claimed to be covered by executive privilege. government. It is the long-term staying power of government that
is enhanced by the mutual accommodation required by the G.R. No. 170338             December 23, 2008
separation of powers." 
VIRGILIO O. GARCILLANO, petitioner, 
In rendering this decision, the Court emphasizes once more that the basic vs.
principles of constitutional law cannot be subordinated to the needs of a THE HOUSE OF REPRESENTATIVES COMMITTEES ON PUBLIC
particular situation. As magistrates, our mandate is to rule objectively and INFORMATION, PUBLIC ORDER AND SAFETY, NATIONAL DEFENSE
dispassionately, always mindful of Mr. Justice Holmes' warning on the AND SECURITY, INFORMATION AND COMMUNICATIONS
dangers inherent in cases of this nature, thus:  TECHNOLOGY, and SUFFRAGE AND ELECTORAL
REFORMS, respondents.
"some accident of immediate and overwhelming interest…appeals
to the feelings and distorts the judgment. These immediate x----------------------x
interests exercise a kind of hydraulic pressure which makes what
previously was clear seem doubtful, and before which even well
G.R. No. 179275             December 23, 2008
settled principles of law will bend."66

SANTIAGO JAVIER RANADA and OSWALDO D.


In this present crusade to "search for truth," we should turn to the
AGCAOILI, petitioners, 
fundamental constitutional principles which underlie our tripartite system of
vs.
government, where the Legislature enacts the law, the Judiciary interprets
THE SENATE OF THE REPUBLIC OF THE PHILIPPINES,
it and the Executive implements it. They are considered separate, co-equal,
REPRESENTED BY THE SENATE PRESIDENT THE HONORABLE
coordinate and supreme within their respective spheres but, imbued with a
MANUEL VILLAR, respondents.
system of checks and balances to prevent unwarranted exercise of power.
The Court's mandate is to preserve these constitutional principles at all
times to keep the political branches of government within constitutional x----------------------x
bounds in the exercise of their respective powers and prerogatives, even if
it be in the search for truth. This is the only way we can preserve the MAJ. LINDSAY REX SAGGE, petitioner-in-intervention
stability of our democratic institutions and uphold the Rule of Law.

x----------------------x
WHEREFORE, the petition is hereby GRANTED. The subject Order
dated January 30, 2008, citing petitioner Romulo L. Neri in contempt of the
Senate Committees and directing his arrest and detention, is hereby AQUILINO Q. PIMENTEL, JR., BENIGNO NOYNOY C. AQUINO,
nullified.  RODOLFO G. BIAZON, PANFILO M. LACSON, LOREN B. LEGARDA,
M.A. JAMBY A.S. MADRIGAL, and ANTONIO F.
TRILLANES, respondents-intervenors
SO ORDERED.

DECISION

NACHURA, J.:

More than three years ago, tapes ostensibly containing a wiretapped


conversation purportedly between the President of the Philippines and a
high-ranking official of the Commission on Elections (COMELEC) surfaced.
They captured unprecedented public attention and thrust the country into a
controversy that placed the legitimacy of the present administration on the
line, and resulted in the near-collapse of the Arroyo government. The
tapes, notoriously referred to as the "Hello Garci" tapes, allegedly
contained the President’s instructions to COMELEC Commissioner Virgilio willingness of telecommunications providers to participate in nefarious
Garcillano to manipulate in her favor results of the 2004 presidential wiretapping activities. 
elections. These recordings were to become the subject of heated
legislative hearings conducted separately by committees of both Houses of
On motion of Senator Francis Pangilinan, Senator Lacson’s speech was
Congress.1
referred to the Senate Committee on National Defense and Security,
chaired by Senator Rodolfo Biazon, who had previously filed two
In the House of Representatives (House), on June 8, 2005, then Minority bills6 seeking to regulate the sale, purchase and use of wiretapping
Floor Leader Francis G. Escudero delivered a privilege speech, "Tale of Two equipment and to prohibit the Armed Forces of the Philippines (AFP) from
Tapes," and set in motion a congressional investigation jointly conducted performing electoral duties.7
by the Committees on Public Information, Public Order and Safety, National
Defense and Security, Information and Communications Technology, and
In the Senate’s plenary session the following day, a lengthy debate ensued
Suffrage and Electoral Reforms (respondent House Committees). During
when Senator Richard Gordon aired his concern on the possible
the inquiry, several versions of the wiretapped conversation emerged. But
transgression of Republic Act (R.A.) No. 42008 if the body were to conduct
on July 5, 2005, National Bureau of Investigation (NBI) Director Reynaldo
a legislative inquiry on the matter. On August 28, 2007, Senator Miriam
Wycoco, Atty. Alan Paguia and the lawyer of former NBI Deputy Director
Defensor-Santiago delivered a privilege speech, articulating her considered
Samuel Ong submitted to the respondent House Committees seven alleged
view that the Constitution absolutely bans the use, possession, replay or
"original" tape recordings of the supposed three-hour taped conversation.
communication of the contents of the "Hello Garci" tapes. However, she
After prolonged and impassioned debate by the committee members on the
recommended a legislative investigation into the role of the Intelligence
admissibility and authenticity of the recordings, the tapes were eventually
Service of the AFP (ISAFP), the Philippine National Police or other
played in the chambers of the House.2
government entities in the alleged illegal wiretapping of public officials.9

On August 3, 2005, the respondent House Committees decided to suspend


On September 6, 2007, petitioners Santiago Ranada and Oswaldo Agcaoili,
the hearings indefinitely. Nevertheless, they decided to prepare committee
retired justices of the Court of Appeals, filed before this Court a Petition for
reports based on the said recordings and the testimonies of the resource
Prohibition with Prayer for the Issuance of a Temporary Restraining Order
persons.3
and/or Writ of Preliminary Injunction,10 docketed as G.R. No. 179275,
seeking to bar the Senate from conducting its scheduled legislative inquiry.
Alarmed by these developments, petitioner Virgilio O. Garcillano They argued in the main that the intended legislative inquiry violates R.A.
(Garcillano) filed with this Court a Petition for Prohibition and Injunction, No. 4200 and Section 3, Article III of the Constitution.11
with Prayer for Temporary Restraining Order and/or Writ of Preliminary
Injunction4docketed as G.R. No. 170338. He prayed that the respondent
As the Court did not issue an injunctive writ, the Senate proceeded with its
House Committees be restrained from using these tape recordings of the
public hearings on the "Hello Garci" tapes on September 7,12 1713 and
"illegally obtained" wiretapped conversations in their committee reports
October 1,14 2007.
and for any other purpose. He further implored that the said recordings
and any reference thereto be ordered stricken off the records of the
inquiry, and the respondent House Committees directed to desist from Intervening as respondents,15 Senators Aquilino Q. Pimentel, Jr., Benigno
further using the recordings in any of the House proceedings.5 Noynoy C. Aquino, Rodolfo G. Biazon, Panfilo M. Lacson, Loren B. Legarda,
M.A. Jamby A.S. Madrigal and Antonio F. Trillanes filed their Comment16 on
the petition on September 25, 2007. 
Without reaching its denouement, the House discussion and debates on the
"Garci tapes" abruptly stopped. 
The Court subsequently heard the case on oral argument.17
After more than two years of quiescence, Senator Panfilo Lacson roused
the slumbering issue with a privilege speech, "The Lighthouse That Brought On October 26, 2007, Maj. Lindsay Rex Sagge, a member of the ISAFP and
Darkness." In his discourse, Senator Lacson promised to provide the public one of the resource persons summoned by the Senate to appear and testify
"the whole unvarnished truth – the what’s, when’s, where’s, who’s and at its hearings, moved to intervene as petitioner in G.R. No. 179275.18
why’s" of the alleged wiretap, and sought an inquiry into the perceived
On November 20, 2007, the Court resolved to consolidate G.R. Nos. the current policy that "this Court has repeatedly and consistently refused
170338 and 179275.19 to wield procedural barriers as impediments to its addressing and resolving
serious legal questions that greatly impact on public interest, in keeping
with the Court’s duty under the 1987 Constitution to determine whether or
It may be noted that while both petitions involve the "Hello Garci"
not other branches of government have kept themselves within the limits
recordings, they have different objectives–the first is poised at preventing
of the Constitution and the laws, and that they have not abused the
the playing of the tapes in the House and their subsequent inclusion in the
discretion given to them."26
committee reports, and the second seeks to prohibit and stop the conduct
of the Senate inquiry on the wiretapped conversation.
In G.R. No. 170338, petitioner Garcillano justifies his standing to initiate
the petition by alleging that he is the person alluded to in the "Hello Garci"
The Court dismisses the first petition, G.R. No. 170338, and grants the
tapes. Further, his was publicly identified by the members of the
second, G.R. No. 179275. 
respondent committees as one of the voices in the recordings.27 Obviously,
therefore, petitioner Garcillano stands to be directly injured by the House
-I- committees’ actions and charges of electoral fraud. The Court recognizes
his standing to institute the petition for prohibition.
Before delving into the merits of the case, the Court shall first resolve the
issue on the parties’ standing, argued at length in their pleadings.  In G.R. No. 179275, petitioners Ranada and Agcaoili justify their standing
by alleging that they are concerned citizens, taxpayers, and members of
In Tolentino v. COMELEC,20 we explained that "‘[l]egal standing’ or locus the IBP. They are of the firm conviction that any attempt to use the "Hello
standi refers to a personal and substantial interest in a case such that the Garci" tapes will further divide the country. They wish to see the legal and
party has sustained or will sustain direct injury because of the challenged proper use of public funds that will necessarily be defrayed in the ensuing
governmental act x x x," thus, public hearings. They are worried by the continuous violation of the laws
and individual rights, and the blatant attempt to abuse constitutional
processes through the conduct of legislative inquiries purportedly in aid of
generally, a party will be allowed to litigate only when (1) he can legislation.28
show that he has personally suffered some actual or threatened
injury because of the allegedly illegal conduct of the government;
(2) the injury is fairly traceable to the challenged action; and (3) Intervenor Sagge alleges violation of his right to due process considering
the injury is likely to be redressed by a favorable action.21 that he is summoned to attend the Senate hearings without being apprised
not only of his rights therein through the publication of the Senate Rules of
Procedure Governing Inquiries in Aid of Legislation, but also of the intended
The gist of the question of standing is whether a party has "alleged such a legislation which underpins the investigation. He further intervenes as a
personal stake in the outcome of the controversy as to assure that taxpayer bewailing the useless and wasteful expenditure of public funds
concrete adverseness which sharpens the presentation of issues upon involved in the conduct of the questioned hearings.29
which the court so largely depends for illumination of difficult constitutional
questions."22
Given that petitioners Ranada and Agcaoili allege an interest in the
execution of the laws and that intervenor Sagge asserts his constitutional
However, considering that locus standi is a mere procedural technicality, right to due process,30 they satisfy the requisite personal stake in the
the Court, in recent cases, has relaxed the stringent direct injury outcome of the controversy by merely being citizens of the Republic. 
test. David v. Macapagal-Arroyo23 articulates that a "liberal policy has been
observed, allowing ordinary citizens, members of Congress, and civic
organizations to prosecute actions involving the constitutionality or validity Following the Court’s ruling in Francisco, Jr. v. The House of
of laws, regulations and rulings."24 The fairly recent Chavez v. Representatives,31 we find sufficient petitioners Ranada’s and Agcaoili’s and
Gonzales25 even permitted a non-member of the broadcast media, who intervenor Sagge’s allegation that the continuous conduct by the Senate of
failed to allege a personal stake in the outcome of the controversy, to the questioned legislative inquiry will necessarily involve the expenditure of
challenge the acts of the Secretary of Justice and the National public funds.32 It should be noted that in Francisco, rights personal to then
Telecommunications Commission. The majority, in the said case, echoed Chief Justice Hilario G. Davide, Jr. had been injured by the alleged
unconstitutional acts of the House of Representatives, yet the Court submitted to the House in plenary by the respondent committees.40 Having
granted standing to the petitioners therein for, as in this case, they been overtaken by these events, the Garcillano petition has to be
invariably invoked the vindication of their own rights–as taxpayers, dismissed for being moot and academic. After all, prohibition is a
members of Congress, citizens, individually or in a class suit, and members preventive remedy to restrain the doing of an act about to be done, and
of the bar and of the legal profession–which were also supposedly violated not intended to provide a remedy for an act already accomplished.41
by the therein assailed unconstitutional acts.33
- III -
Likewise, a reading of the petition in G.R. No. 179275 shows that the
petitioners and intervenor Sagge advance constitutional issues which
As to the petition in G.R. No. 179275, the Court grants the same. The
deserve the attention of this Court in view of their seriousness, novelty and
Senate cannot be allowed to continue with the conduct of the questioned
weight as precedents. The issues are of transcendental and paramount
legislative inquiry without duly published rules of procedure, in clear
importance not only to the public but also to the Bench and the Bar, and
derogation of the constitutional requirement. 
should be resolved for the guidance of all.34

Section 21, Article VI of the 1987 Constitution explicitly provides that


Thus, in the exercise of its sound discretion and given the liberal attitude it
"[t]he Senate or the House of Representatives, or any of its respective
has shown in prior cases climaxing in the more recent case of Chavez, the
committees may conduct inquiries in aid of legislation in accordance with
Court recognizes the legal standing of petitioners Ranada and Agcaoili and
its duly published rules of procedure." The requisite of publication of the
intervenor Sagge.
rules is intended to satisfy the basic requirements of due
process.42 Publication is indeed imperative, for it will be the height of
- II - injustice to punish or otherwise burden a citizen for the transgression of a
law or rule of which he had no notice whatsoever, not even a constructive
one.43What constitutes publication is set forth in Article 2 of the Civil Code,
The Court, however, dismisses G.R. No. 170338 for being moot and
which provides that "[l]aws shall take effect after 15 days following the
academic. Repeatedly stressed in our prior decisions is the principle that
completion of their publication either in the Official Gazette, or in a
the exercise by this Court of judicial power is limited to the determination
newspaper of general circulation in the Philippines."44
and resolution of actual cases and controversies.35 By actual cases, we
mean existing conflicts appropriate or ripe for judicial determination, not
conjectural or anticipatory, for otherwise the decision of the Court will The respondents in G.R. No. 179275 admit in their pleadings and even on
amount to an advisory opinion. The power of judicial inquiry does not oral argument that the Senate Rules of Procedure Governing Inquiries in
extend to hypothetical questions because any attempt at abstraction could Aid of Legislation had been published in newspapers of general circulation
only lead to dialectics and barren legal questions and to sterile conclusions only in 1995 and in 2006.45 With respect to the present Senate of the
unrelated to actualities.36 Neither will the Court determine a moot question 14th Congress, however, of which the term of half of its members
in a case in which no practical relief can be granted. A case becomes moot commenced on June 30, 2007, no effort was undertaken for the publication
when its purpose has become stale.37 It is unnecessary to indulge in of these rules when they first opened their session. 
academic discussion of a case presenting a moot question as a judgment
thereon cannot have any practical legal effect or, in the nature of things,
Recently, the Court had occasion to rule on this very same question.
cannot be enforced.38
In Neri v. Senate Committee on Accountability of Public Officers and
Investigations,46 we said:
In G.R. No. 170338, petitioner Garcillano implores from the Court, as
aforementioned, the issuance of an injunctive writ to prohibit the
Fourth, we find merit in the argument of the OSG that
respondent House Committees from playing the tape recordings and from
respondent Committees likewise violated Section 21 of Article VI
including the same in their committee report. He likewise prays that the
of the Constitution, requiring that the inquiry be in accordance
said tapes be stricken off the records of the House proceedings. But the
with the "duly published rules of procedure." We quote the
Court notes that the recordings were already played in the House and
OSG’s explanation:
heard by its members.39 There is also the widely publicized fact that the
committee reports on the "Hello Garci" inquiry were completed and
The phrase "duly published rules of procedure" requires SEC. 123. Unfinished business at the end of the session
the Senate of every Congress to publish its rules of shall be taken up at the next session in the same status.
procedure governing inquiries in aid of legislation
because every Senate is distinct from the one before it or
All pending matters and proceedings shall
after it. Since Senatorial elections are held every three
terminate upon the expiration of one (1) Congress,
(3) years for one-half of the Senate’s membership, the
but may be taken by the succeeding Congress as if
composition of the Senate also changes by the end of
present for the first time. 
each term. Each Senate may thus enact a different set of
rules as it may deem fit. Not having published
its Rules of Procedure, the subject hearings in aid Undeniably from the foregoing, all pending matters and
of legislation conducted by the 14th Senate, are proceedings, i.e., unpassed bills and even legislative
therefore, procedurally infirm. investigations, of the Senate of a particular Congress are
considered terminated upon the expiration of that Congress and
it is merely optional on the Senate of the succeeding Congress to
Justice Antonio T. Carpio, in his Dissenting and Concurring Opinion,
take up such unfinished matters, not in the same status, but as
reinforces this ruling with the following rationalization: 
if presented for the first time. The logic and practicality of such
a rule is readily apparent considering that the Senate of the
The present Senate under the 1987 Constitution is no longer a succeeding Congress (which will typically have a different
continuing legislative body. The present Senate has twenty-four composition as that of the previous Congress) should not be
members, twelve of whom are elected every three years for a bound by the acts and deliberations of the Senate of which they
term of six years each. Thus, the term of twelve Senators expires had no part. If the Senate is a continuing body even with respect
every three years, leaving less than a majority of Senators to to the conduct of its business, then pending matters will not be
continue into the next Congress. The 1987 Constitution, like deemed terminated with the expiration of one Congress but will,
the 1935 Constitution, requires a majority of Senators to as a matter of course, continue into the next Congress with the
"constitute a quorum to do business." Applying the same same status.
reasoning in Arnault v. Nazareno, the Senate under the 1987
Constitution is not a continuing body because less than majority of
This dichotomy of the continuity of the Senate as an institution
the Senators continue into the next Congress. The consequence is
and of the opposite nature of the conduct of its business is
that the Rules of Procedure must be republished by the Senate
reflected in its Rules. The Rules of the Senate (i.e. the Senate’s
after every expiry of the term of twelve Senators.47
main rules of procedure) states:

The subject was explained with greater lucidity in our Resolution48 (On the


RULE LI
Motion for Reconsideration) in the same case, viz.:
AMENDMENTS TO, OR REVISIONS OF, THE RULES

On the nature of the Senate as a "continuing body," this Court


SEC. 136. At the start of each session in which the
sees fit to issue a clarification. Certainly, there is no debate that
Senators elected in the preceding elections shall begin
the Senate as an institution is "continuing," as it is not dissolved
their term of office, the President may endorse the Rules
as an entity with each national election or change in the
to the appropriate committee for amendment or revision.
composition of its members. However, in the conduct of its day-
to-day business the Senate of each Congress acts separately and
independently of the Senate of the Congress before it. The Rules The Rules may also be amended by means of a motion
of the Senate itself confirms this when it states:  which should be presented at least one day before its
consideration, and the vote of the majority of the
Senators present in the session shall be required for its
RULE XLIV
approval.
UNFINISHED BUSINESS
RULE LII The Court does not agree. The absence of any amendment to the rules
DATE OF TAKING EFFECT cannot justify the Senate’s defiance of the clear and unambiguous
language of Section 21, Article VI of the Constitution. The organic law
instructs, without more, that the Senate or its committees may conduct
SEC. 137. These Rules shall take effect on the date of
inquiries in aid of legislation only in accordance with duly published rules of
their adoption and shall remain in force until they are
procedure, and does not make any distinction whether or not these rules
amended or repealed.
have undergone amendments or revision. The constitutional mandate to
publish the said rules prevails over any custom, practice or tradition
Section 136 of the Senate Rules quoted above takes into account followed by the Senate.
the new composition of the Senate after an election and the
possibility of the amendment or revision of the Rules at the start
Justice Carpio’s response to the same argument raised by the respondents
of each session in which the newly elected Senators shall begin
is illuminating:
their term.

The publication of the Rules of Procedure in the website of the


However, it is evident that the Senate has determined that its
Senate, or in pamphlet form available at the Senate, is not
main rules are intended to be valid from the date of their adoption
sufficient under the Tañada v. Tuvera ruling which requires
until they are amended or repealed. Such language is
publication either in the Official Gazette or in a newspaper of
conspicuously absent from the Rules. The Rules simply state
general circulation. The Rules of Procedure even provide that the
"(t)hese Rules shall take effect seven (7) days after publication in
rules "shall take effect seven (7) days after publication in two (2)
two (2) newspapers of general circulation." The latter does not
newspapers of general circulation," precluding any other form of
explicitly provide for the continued effectivity of such rules until
publication. Publication in accordance with Tañada is mandatory to
they are amended or repealed. In view of the difference in the
comply with the due process requirement because the Rules of
language of the two sets of Senate rules, it cannot be presumed
Procedure put a person’s liberty at risk. A person who violates
that the Rules (on legislative inquiries) would continue into the
the Rules of Procedure could be arrested and detained by the
next Congress. The Senate of the next Congress may easily adopt
Senate.
different rules for its legislative inquiries which come within the
rule on unfinished business.
The invocation by the respondents of the provisions of R.A. No.
8792,50 otherwise known as the Electronic Commerce Act of 2000, to
The language of Section 21, Article VI of the Constitution requiring
support their claim of valid publication through the internet is all the more
that the inquiry be conducted in accordance with the duly
incorrect. R.A. 8792 considers an electronic data message or an electronic
published rules of procedure is categorical. It is incumbent upon
document as the functional equivalent of a written document only
the Senate to publish the rules for its legislative inquiries in each
for evidentiary purposes.51 In other words, the law merely recognizes the
Congress or otherwise make the published rules clearly state that
admissibility in evidence (for their being the original) of electronic data
the same shall be effective in subsequent Congresses or until they
messages and/or electronic documents.52 It does not make the internet a
are amended or repealed to sufficiently put public on notice.
medium for publishing laws, rules and regulations.

If it was the intention of the Senate for its present rules on


Given this discussion, the respondent Senate Committees, therefore, could
legislative inquiries to be effective even in the next Congress, it
not, in violation of the Constitution, use its unpublished rules in the
could have easily adopted the same language it had used in its
legislative inquiry subject of these consolidated cases. The conduct of
main rules regarding effectivity.
inquiries in aid of legislation by the Senate has to be deferred until it shall
have caused the publication of the rules, because it can do so only "in
Respondents justify their non-observance of the constitutionally mandated accordance with its duly published rules of procedure."
publication by arguing that the rules have never been amended since 1995
and, despite that, they are published in booklet form available to anyone
Very recently, the Senate caused the publication of the Senate Rules of
for free, and accessible to the public at the Senate’s internet web page.49
Procedure Governing Inquiries in Aid of Legislation in the October 31, 2008
issues of Manila Bulletin and Malaya. While we take judicial notice of this [G.R. NO. 169777* : April 20, 2006]
fact, the recent publication does not cure the infirmity of the inquiry sought
to be prohibited by the instant petitions. Insofar as the consolidated cases
SENATE OF THE PHILIPPINES, represented by FRANKLIN M.
are concerned, the legislative investigation subject thereof still could not be
DRILON, in his capacity as Senate President, JUAN M. FLAVIER, in
undertaken by the respondent Senate Committees, because no published
his capacity as Senate President Pro Tempore, FRANCIS N.
rules governed it, in clear contravention of the Constitution.
PANGILINAN, in his capacity as Majority Leader, AQUILINO Q.
PIMENTEL, JR., in his capacity as Minority Leader, SENATORS
With the foregoing disquisition, the Court finds it unnecessary to discuss RODOLFO G. BIAZON, "COMPANERA" PIA S. CAYETANO, JINGGOY
the other issues raised in the consolidated petitions. EJERCITO ESTRADA, LUISA "LOI" EJERCITO ESTRADA, JUAN PONCE
ENRILE, RICHARD J. GORDON, PANFILO M. LACSON, ALFREDO
S.LIM, M. A. MADRIGAL, SERGIO OSMENA III, RALPH G. RECTO, and
WHEREFORE, the petition in G.R. No. 170338 is DISMISSED, and the
MAR ROXAS, Petitioners, v. EDUARDO R. ERMITA, in his capacity as
petition in G.R. No. 179275 is GRANTED. Let a writ of prohibition be issued
Executive Secretary and alter-ego of President Gloria Macapagal-
enjoining the Senate of the Republic of the Philippines and/or any of its
Arroyo, and anyone acting in his stead and in behalf of the
committees from conducting any inquiry in aid of legislation centered on
President of the Philippines, Respondents.
the "Hello Garci" tapes.

[G.R. NO. 169659 : April 20, 2006]


SO ORDERED.

BAYAN MUNA represented by DR. REYNALDO LESACA, JR., Rep.


SATUR OCAMPO, Rep. CRISPIN BELTRAN, Rep. RAFAEL MARIANO,
Rep. LIZA MAZA, Rep. TEODORO CASINO, Rep. JOEL VIRADOR,
COURAGE represented by FERDINAND GAITE, and COUNSELS FOR
THE DEFENSE OF LIBERTIES (CODAL) represented by ATTY.
REMEDIOS BALBIN, Petitioners, v. EDUARDO ERMITA, in his capacity
as Executive Secretary and alter-ego of President Gloria
Macapagal-Arroyo, Respondent.

[G.R. NO. 169660 : April 20, 2006]

FRANCISCO I. CHAVEZ, Petitioner, v. EDUARDO R. ERMITA, in his


capacity as Executive Secretary, AVELINO J. CRUZ, JR., in his
capacity as Secretary of Defense, and GENEROSO S. SENGA, in his
capacity as AFP Chief of Staff, Respondents.

[G.R. NO. 169667 : April 20, 2006]

ALTERNATIVE LAW GROUPS, INC. (ALG), Petitioner, v.HON.


EDUARDO R. ERMITA, in his capacity as Executive
Secretary, Respondent.

[G.R. NO. 169834 : April 20, 2006]

PDP - LABAN, Petitioner, v. EXECUTIVE SECRETARY EDUARDO R.


ERMITA, Respondent.
[G.R. NO. 171246 : April 20, 2006] those employed in Government Owned and Controlled Corporations, the
Armed Forces of the Philippines (AFP), and the Philippine National Police
(PNP). 
JOSE ANSELMO I. CADIZ, FELICIANO M. BAUTISTA, ROMULO R.
RIVERA, JOSE AMOR AMORANDO, ALICIA A. RISOS-VIDAL,
FILEMON C. ABELITA III, MANUEL P. LEGASPI, J. B. JOVY C. On September 21 to 23, 2005, the Committee of the Senate as a whole
BERNABE, BERNARD L. DAGCUTA, ROGELIO V. GARCIA, and the issued invitations to various officials of the Executive Department for them
INTEGRATED BAR FOR THE PHILIPPINES, Petitioners, v. HON. to appear on September 29, 2005 as resource speakers in a public hearing
EXECUTIVE SECRETARY EDUARDO R. ERMITA, Respondent. on the railway project of the North Luzon Railways Corporation with the
China National Machinery and Equipment Group (hereinafter North Rail
Project). The public hearing was sparked by a privilege speech of Senator
DECISION
Juan Ponce Enrile urging the Senate to investigate the alleged overpricing
and other unlawful provisions of the contract covering the North Rail
CARPIO MORALES, J.: Project.

A transparent government is one of the hallmarks of a truly republican The Senate Committee on National Defense and Security likewise issued
state. Even in the early history of republican thought, however, it has been invitations2 dated September 22, 2005 to the following officials of the AFP:
recognized that the head of government may keep certain information the Commanding General of the Philippine Army, Lt. Gen. Hermogenes C.
confidential in pursuit of the public interest. Explaining the reason for Esperon; Inspector General of the AFP Vice Admiral Mateo M. Mayuga;
vesting executive power in only one magistrate, a distinguished delegate to Deputy Chief of Staff for Intelligence of the AFP Rear Admiral Tirso R.
the U.S. Constitutional Convention said: "Decision, activity, secrecy, and Danga; Chief of the Intelligence Service of the AFP Brig. Gen. Marlu Q.
dispatch will generally characterize the proceedings of one man, in a much Quevedo; Assistant Superintendent of the Philippine Military Academy
more eminent degree than the proceedings of any greater number; and in (PMA) Brig. Gen. Francisco V. Gudani; and Assistant Commandant, Corps
proportion as the number is increased, these qualities will be diminished."1 of Cadets of the PMA, Col. Alexander F. Balutan, for them to attend as
resource persons in a public hearing scheduled on September 28, 2005 on
History has been witness, however, to the fact that the power to withhold the following: (1) Privilege Speech of Senator Aquilino Q. Pimentel Jr.,
information lends itself to abuse, hence, the necessity to guard it zealously. delivered on June 6, 2005 entitled "Bunye has Provided Smoking Gun or
has Opened a Can of Worms that Show Massive Electoral Fraud in the
Presidential Election of May 2005"; (2) Privilege Speech of Senator Jinggoy
The present consolidated petitions for certiorari and prohibition proffer that E. Estrada delivered on July 26, 2005 entitled "The Philippines as the Wire-
the President has abused such power by issuing Executive Order No. 464 Tapping Capital of the World"; (3) Privilege Speech of Senator Rodolfo
(E.O. 464) last September 28, 2005. They thus pray for its declaration as Biazon delivered on August 1, 2005 entitled "Clear and Present Danger";
null and void for being unconstitutional.  (4) Senate Resolution No. 285 filed by Senator Maria Ana Consuelo
Madrigal - Resolution Directing the Committee on National Defense and
In resolving the controversy, this Court shall proceed with the recognition Security to Conduct an Inquiry, in Aid of Legislation, and in the National
that the issuance under review has come from a co-equal branch of Interest, on the Role of the Military in the So-called "Gloriagate Scandal";
government, which thus entitles it to a strong presumption of and (5) Senate Resolution No. 295 filed by Senator Biazon - Resolution
constitutionality. Once the challenged order is found to be indeed violative Directing the Committee on National Defense and Security to Conduct an
of the Constitution, it is duty-bound to declare it so. For the Constitution, Inquiry, in Aid of Legislation, on the Wire-Tapping of the President of the
being the highest expression of the sovereign will of the Filipino people, Philippines. 
must prevail over any issuance of the government that contravenes its
mandates.  Also invited to the above-said hearing scheduled on September 28 2005
was the AFP Chief of Staff, General Generoso S. Senga who, by
In the exercise of its legislative power, the Senate of the Philippines, letter3 dated September 27, 2005, requested for its postponement "due to
through its various Senate Committees, conducts inquiries or investigations a pressing operational situation that demands [his utmost personal
in aid of legislation which call for, inter alia, the attendance of officials and attention" while "some of the invited AFP officers are currently attending to
employees of the executive department, bureaus, and offices including other urgent operational matters."
On September 28, 2005, Senate President Franklin M. Drilon received from Conduct and Ethical Standards for Public Officials and Employees provides
Executive Secretary Eduardo R. Ermita a letter4dated September 27, 2005 that Public Officials and Employees shall not use or divulge confidential or
"respectfully request[ing] for the postponement of the hearing [regarding classified information officially known to them by reason of their office and
the NorthRail project] to which various officials of the Executive not made available to the public to prejudice the public interest.
Department have been invited" in order to "afford said officials ample time
and opportunity to study and prepare for the various issues so that they
Executive privilege covers all confidential or classified information between
may better enlighten the Senate Committee on its investigation." 
the President and the public officers covered by this executive order,
including:
Senate President Drilon, however, wrote5 Executive Secretary Ermita that
the Senators "are unable to accede to [his request]" as it "was sent
Conversations and correspondence between the President and the public
belatedly" and "[a]ll preparations and arrangements as well as notices to
official covered by this executive order (Almonte v. Vasquez G.R. No.
all resource persons were completed [the previous] week."
95367, 23 May 1995; Chavez v. Public Estates Authority, G.R. No. 133250,
9 July 2002); 
Senate President Drilon likewise received on September 28, 2005 a
letter6 from the President of the North Luzon Railways Corporation Jose L.
Military, diplomatic and other national security matters which in the
Cortes, Jr. requesting that the hearing on the NorthRail project be
interest of national security should not be divulged (Almonte v. Vasquez,
postponed or cancelled until a copy of the report of the UP Law Center on
G.R. No. 95367, 23 May 1995; Chavez v. Presidential Commission on Good
the contract agreements relative to the project had been secured.
Government, G.R. No. 130716, 9 December 1998). 

On September 28, 2005, the President issued E.O. 464, "Ensuring


Information between inter-government agencies prior to the conclusion of
Observance of the Principle of Separation of Powers, Adherence to the Rule
treaties and executive agreements (Chavez v. Presidential Commission on
on Executive Privilege and Respect for the Rights of Public Officials
Good Government, G.R. No. 130716, 9 December 1998); 
Appearing in Legislative Inquiries in Aid of Legislation Under the
Constitution, and For Other Purposes,"7 which, pursuant to Section 6
thereof, took effect immediately. The salient provisions of the Order are as Discussion in close-door Cabinet meetings (Chavez v. Presidential
follows: Commission on Good Government, G.R. No. 130716, 9 December 1998); 

SECTION 1. Appearance by Heads of Departments Before Congress. - In Matters affecting national security and public order (Chavez v. Public
accordance with Article VI, Section 22 of the Constitution and to implement Estates Authority, G.R. No. 133250, 9 July 2002). 
the Constitutional provisions on the separation of powers between co-equal
branches of the government, all heads of departments of the Executive (b) Who are covered. - The following are covered by this executive order:
Branch of the government shall secure the consent of the President prior to
appearing before either House of Congress.
Senior officials of executive departments who in the judgment of the
department heads are covered by the executive privilege; 
When the security of the State or the public interest so requires and the
President so states in writing, the appearance shall only be conducted in
executive session.  Generals and flag officers of the Armed Forces of the Philippines and such
other officers who in the judgment of the Chief of Staff are covered by the
executive privilege; 
SECTION. 2. Nature, Scope and Coverage of Executive Privilege.' 

Philippine National Police (PNP) officers with rank of chief superintendent or


(a) Nature and Scope. - The rule of confidentiality based on executive higher and such other officers who in the judgment of the Chief of the PNP
privilege is fundamental to the operation of government and rooted in the are covered by the executive privilege; 
separation of powers under the Constitution (Almonte v. Vasquez, G.R. No.
95367, 23 May 1995). Further, Republic Act No. 6713 or the Code of
Senior national security officials who in the judgment of the National (DOJ) Chief State Counsel Ricardo V. Perez, then Presidential Legal Counsel
Security Adviser are covered by the executive privilege; andcralawlibrary Merceditas Gutierrez, Department of Transportation and Communication
(DOTC) Undersecretary Guiling Mamonding, DOTC Secretary Leandro
Mendoza, Philippine National Railways General Manager Jose Serase II,
Such other officers as may be determined by the President. 
Monetary Board Member Juanita Amatong, Bases Conversion Development
Authority Chairperson Gen. Narciso Abaya and Secretary Romulo L.
SECTION 3. Appearance of Other Public Officials Before Congress. - All Neri.10 NorthRail President Cortes sent personal regrets likewise citing E.O.
public officials enumerated in Section 2 (b) hereof shall secure prior 464.11
consent of the President prior to appearing before either House of Congress
to ensure the observance of the principle of separation of powers,
On October 3, 2005, three petitions, docketed as G.R. NOS. 169659,
adherence to the rule on executive privilege and respect for the rights of
169660, and 169667, for certiorari and prohibition, were filed before this
public officials appearing in inquiries in aid of legislation. (Emphasis and
Court challenging the constitutionality of E.O. 464. 
underscoring supplied)ςrαlαωlιbrαrÿ

In G.R. No. 169659, petitioners party-list Bayan Muna, House of


Also on September 28, 2005, Senate President Drilon received from
Representatives Members Satur Ocampo, Crispin Beltran, Rafael Mariano,
Executive Secretary Ermita a copy of E.O. 464, and another
Liza Maza, Joel Virador and Teodoro Casino, Courage, an organization of
letter8 informing him "that officials of the Executive Department invited to
government employees, and Counsels for the Defense of Liberties
appear at the meeting [regarding the NorthRail project] will not be able to
(CODAL), a group of lawyers dedicated to the promotion of justice,
attend the same without the consent of the President, pursuant to [E.O.
democracy and peace, all claiming to have standing to file the suit because
464]" and that "said officials have not secured the required consent from
of the transcendental importance of the issues they posed, pray, in their
the President." On even date which was also the scheduled date of the
petition that E.O. 464 be declared null and void for being unconstitutional;
hearing on the alleged wiretapping, Gen. Senga sent a letter9 to Senator
that respondent Executive Secretary Ermita, in his capacity as Executive
Biazon, Chairperson of the Committee on National Defense and Security,
Secretary and alter-ego of President Arroyo, be prohibited from imposing,
informing him "that per instruction of [President Arroyo], thru the
and threatening to impose sanctions on officials who appear before
Secretary of National Defense, no officer of the [AFP] is authorized to
Congress due to congressional summons. Additionally, petitioners claim
appear before any Senate or Congressional hearings without seeking a
that E.O. 464 infringes on their rights and impedes them from fulfilling
written approval from the President" and "that no approval has been
their respective obligations. Thus, Bayan Muna alleges that E.O. 464
granted by the President to any AFP officer to appear before the public
infringes on its right as a political party entitled to participate in
hearing of the Senate Committee on National Defense and Security
governance; Satur Ocampo, et al. allege that E.O. 464 infringes on their
scheduled [on] 28 September 2005."
rights and duties as members of Congress to conduct investigation in aid of
legislation and conduct oversight functions in the implementation of laws;
Despite the communications received from Executive Secretary Ermita and Courage alleges that the tenure of its members in public office is
Gen. Senga, the investigation scheduled by the Committee on National predicated on, and threatened by, their submission to the requirements of
Defense and Security pushed through, with only Col. Balutan and Brig. E.O. 464 should they be summoned by Congress; and CODAL alleges that
Gen. Gudani among all the AFP officials invited attending. its members have a sworn duty to uphold the rule of law, and their rights
to information and to transparent governance are threatened by the
For defying President Arroyo's order barring military personnel from imposition of E.O. 464.
testifying before legislative inquiries without her approval, Brig. Gen.
Gudani and Col. Balutan were relieved from their military posts and were In G.R. No. 169660, petitioner Francisco I. Chavez, claiming that his
made to face court martial proceedings.  constitutional rights as a citizen, taxpayer and law practitioner, are affected
by the enforcement of E.O. 464, prays in his petition that E.O. 464 be
As to the NorthRail project hearing scheduled on September 29, 2005, declared null and void for being unconstitutional. 
Executive Secretary Ermita, citing E.O. 464, sent letter of regrets, in
response to the invitations sent to the following government officials: Light In G.R. No. 169667, petitioner Alternative Law Groups, Inc.12(ALG),
Railway Transit Authority Administrator Melquiades Robles, Metro Rail alleging that as a coalition of 17 legal resource non-governmental
Transit Authority Administrator Roberto Lastimoso, Department of Justice organizations engaged in developmental lawyering and work with the poor
and marginalized sectors in different parts of the country, and as an In the budget hearings set by the Senate on February 8 and 13, 2006,
organization of citizens of the Philippines and a part of the general public, it Press Secretary and Presidential Spokesperson Ignacio R. Bunye,19 DOJ
has legal standing to institute the petition to enforce its constitutional right Secretary Raul M. Gonzalez20 and Department of Interior and Local
to information on matters of public concern, a right which was denied to Government Undersecretary Marius P. Corpus21 communicated their
the public by E.O. 464,13 prays, that said order be declared null and void inability to attend due to lack of appropriate clearance from the President
for being unconstitutional and that respondent Executive Secretary Ermita pursuant to E.O. 464. During the February 13, 2005 budget hearing,
be ordered to cease from implementing it.  however, Secretary Bunye was allowed to attend by Executive Secretary
Ermita.
On October 11, 2005, Petitioner Senate of the Philippines, alleging that it
has a vital interest in the resolution of the issue of the validity of E.O. 464 On February 13, 2006, Jose Anselmo I. Cadiz and the incumbent members
for it stands to suffer imminent and material injury, as it has already of the Board of Governors of the Integrated Bar of the Philippines, as
sustained the same with its continued enforcement since it directly taxpayers, and the Integrated Bar of the Philippines as the official
interferes with and impedes the valid exercise of the Senate's powers and organization of all Philippine lawyers, all invoking their constitutional right
functions and conceals information of great public interest and concern, to be informed on matters of public interest, filed their petition
filed its petition for certiorari and prohibition, docketed as G.R. No. 169777 for certiorari and prohibition, docketed as G.R. No. 171246, and pray that
and prays that E.O. 464 be declared unconstitutional.  E.O. 464 be declared null and void.

On October 14, 2005, PDP-Laban, a registered political party with members All the petitions pray for the issuance of a Temporary Restraining Order
duly elected into the Philippine Senate and House of Representatives, filed enjoining respondents from implementing, enforcing, and observing E.O.
a similar petition for certiorariand prohibition, docketed as G.R. No. 464.
169834, alleging that it is affected by the challenged E.O. 464 because it
hampers its legislative agenda to be implemented through its members in
In the oral arguments on the petitions conducted on February 21, 2006,
Congress, particularly in the conduct of inquiries in aid of legislation and
the following substantive issues were ventilated: (1) whether respondents
transcendental issues need to be resolved to avert a constitutional crisis
committed grave abuse of discretion in implementing E.O. 464 prior to its
between the executive and legislative branches of the government.
publication in the Official Gazette or in a newspaper of general circulation;
and (2) whether E.O. 464 violates the following provisions of the
Meanwhile, by letter14 dated February 6, 2006, Senator Biazon reiterated Constitution: Art. II, Sec. 28, Art. III, Sec. 4, Art. III, Sec. 7, Art. IV. Sec.
his invitation to Gen. Senga for him and other military officers to attend 1, Art. VI, Sec. 21, Art. VI, Sec. 22, Art. XI, Sec. 1, and Art. XIII, Sec. 16.
the hearing on the alleged wiretapping scheduled on February 10, 2005. The procedural issue of whether there is an actual case or controversy that
Gen. Senga replied, however, by letter15 dated February 8, 2006, that calls for judicial review was not taken up; instead, the parties were
"[p]ursuant to Executive Order No. 464, th[e] Headquarters requested for instructed to discuss it in their respective memoranda. 
a clearance from the President to allow [them] to appear before the public
hearing" and that "they will attend once [their] request is approved by the
After the conclusion of the oral arguments, the parties were directed to
President." As none of those invited appeared, the hearing on February 10,
submit their respective memoranda, paying particular attention to the
2006 was cancelled.16
following propositions: (1) that E.O. 464 is, on its face, unconstitutional;
and (2) assuming that it is not, it is unconstitutional as applied in four
In another investigation conducted jointly by the Senate Committee on instances, namely: (a) the so called Fertilizer scam; (b) the NorthRail
Agriculture and Food and the Blue Ribbon Committee on the alleged investigation (c) the Wiretapping activity of the ISAFP; and (d) the
mismanagement and use of the fertilizer fund under the Ginintuang investigation on the Venable contract.22
Masaganang Ani program of the Department of Agriculture (DA), several
Cabinet officials were invited to the hearings scheduled on October 5 and
Petitioners in G.R. No. 16966023 and G.R. No. 16977724 filed their
26, November 24 and December 12, 2005 but most of them failed to
memoranda on March 7, 2006, while those in G.R. No. 16966725 and G.R.
attend, DA Undersecretary Belinda Gonzales, DA Assistant Secretary Felix
No. 16983426 filed theirs the next day or on March 8, 2006. Petitioners in
Jose Montes, Fertilizer and Pesticide Authority Executive Director Norlito R.
G.R. No. 171246 did not file any memorandum.
Gicana,17 and those from the Department of Budget and
Management18having invoked E.O. 464.
Petitioners Bayan Muna et al. in G.R. No. 169659, after their motion for Before proceeding to resolve the issue of the constitutionality of E.O. 464,
extension to file memorandum27 was granted, subsequently filed a ascertainment of whether the requisites for a valid exercise of the Court's
manifestation28 dated March 14, 2006 that it would no longer file its power of judicial review are present is in order.
memorandum in the interest of having the issues resolved soonest,
prompting this Court to issue a Resolution reprimanding them.29
Like almost all powers conferred by the Constitution, the power of judicial
review is subject to limitations, to wit: (1) there must be an actual case or
Petitioners submit that E.O. 464 violates the following constitutional controversy calling for the exercise of judicial power; (2) the person
provisions: challenging the act must have standing to challenge the validity of the
subject act or issuance; otherwise stated, he must have a personal and
substantial interest in the case such that he has sustained, or will sustain,
Art. VI, Sec. 2130
direct injury as a result of its enforcement; (3) the question of
constitutionality must be raised at the earliest opportunity; and (4) the
Art. VI, Sec. 2231 issue of constitutionality must be the very lis mota of the case.39

Art. VI, Sec. 132 Except with respect to the requisites of standing and existence of an actual
case or controversy where the disagreement between the parties lies,
Art. XI, Sec. 133 discussion of the rest of the requisites shall be omitted.

Art. III, Sec. 734 Standing

Art. III, Sec. 435 Respondents, through the Solicitor General, assert that the allegations in
G.R. NOS. 169659, 169660 and 169667 make it clear that they, adverting
to the non-appearance of several officials of the executive department in
Art. XIII, Sec. 16 36 the investigations called by the different committees of the Senate, were
brought to vindicate the constitutional duty of the Senate or its different
Art. II, Sec. 2837 committees to conduct inquiry in aid of legislation or in the exercise of its
oversight functions. They maintain that Representatives Ocampo et al.
have not shown any specific prerogative, power, and privilege of the House
Respondents Executive Secretary Ermita et al., on the other hand, pray in of Representatives which had been effectively impaired by E.O. 464, there
their consolidated memorandum38 on March 13, 2006 for the dismissal of being no mention of any investigation called by the House of
the petitions for lack of merit.  Representatives or any of its committees which was aborted due to the
implementation of E.O. 464. 
The Court synthesizes the issues to be resolved as follows:
As for Bayan Muna's alleged interest as a party-list representing the
1. Whether E.O. 464 contravenes the power of inquiry vested in Congress;  marginalized and underrepresented, and that of the other petitioner groups
and individuals who profess to have standing as advocates and defenders
of the Constitution, respondents contend that such interest falls short of
2. Whether E.O. 464 violates the right of the people to information on
that required to confer standing on them as parties "injured-in-fact."40
matters of public concern; andcralawlibrary

Respecting petitioner Chavez, respondents contend that Chavez may not


3. Whether respondents have committed grave abuse of discretion when
claim an interest as a taxpayer for the implementation of E.O. 464 does not
they implemented E.O. 464 prior to its publication in a newspaper of
involve the exercise of taxing or spending power.41
general circulation.

Essential requisites for judicial review


With regard to the petition filed by the Senate, respondents argue that in As Bayan Muna and Representatives Ocampo et al. have the standing to
the absence of a personal or direct injury by reason of the issuance of E.O. file their petitions, passing on the standing of their co-petitioners Courage
464, the Senate and its individual members are not the proper parties to and Codal is rendered unnecessary.49
assail the constitutionality of E.O. 464. 
In filing their respective petitions, Chavez, the ALG which claims to be an
Invoking this Court's ruling in National Economic Protectionism Association organization of citizens, and the incumbent members of the IBP Board of
v. Ongpin42 and Valmonte v. Philippine Charity Sweepstakes Governors and the IBP in behalf of its lawyer members,50 invoke their
Office,43 respondents assert that to be considered a proper party, one must constitutional right to information on matters of public concern, asserting
have a personal and substantial interest in the case, such that he has that the right to information, curtailed and violated by E.O. 464, is
sustained or will sustain direct injury due to the enforcement of E.O. 464.44 essential to the effective exercise of other constitutional rights51 and to the
maintenance of the balance of power among the three branches of the
government through the principle of checks and balances.52
That the Senate of the Philippines has a fundamental right essential not
only for intelligent public decision-making in a democratic system, but
more especially for sound legislation45 is not disputed. E.O. 464, however, It is well-settled that when suing as a citizen, the interest of the petitioner
allegedly stifles the ability of the members of Congress to access in assailing the constitutionality of laws, presidential decrees, orders, and
information that is crucial to law-making.46 Verily, the Senate, including its other regulations, must be direct and personal. In Franciso v. House of
individual members, has a substantial and direct interest over the outcome Representatives,53 this Court held that when the proceeding involves the
of the controversy and is the proper party to assail the constitutionality of assertion of a public right, the mere fact that he is a citizen satisfies the
E.O. 464. Indeed, legislators have standing to maintain inviolate the requirement of personal interest.
prerogative, powers and privileges vested by the Constitution in their office
and are allowed to sue to question the validity of any official action which
As for petitioner PDP-Laban, it asseverates that it is clothed with legal
they claim infringes their prerogatives as legislators.47
standing in view of the transcendental issues raised in its petition which
this Court needs to resolve in order to avert a constitutional crisis. For it to
In the same vein, party-list representatives Satur Ocampo (Bayan Muna), be accorded standing on the ground of transcendental importance,
Teodoro Casino (Bayan Muna), Joel Virador (Bayan Muna), Crispin Beltran however, it must establish (1) the character of the funds (that it is public)
(Anakpawis), Rafael Mariano (Anakpawis), and Liza Maza (Gabriela) are or other assets involved in the case, (2) the presence of a clear case of
allowed to sue to question the constitutionality of E.O. 464, the absence of disregard of a constitutional or statutory prohibition by the public
any claim that an investigation called by the House of Representatives or respondent agency or instrumentality of the government, and (3) the lack
any of its committees was aborted due to the implementation of E.O. 464 of any party with a more direct and specific interest in raising the questions
notwithstanding, it being sufficient that a claim is made that E.O. 464 being raised.54The first and last determinants not being present as no
infringes on their constitutional rights and duties as members of Congress public funds or assets are involved and petitioners in G.R. NOS. 169777
to conduct investigation in aid of legislation and conduct oversight and 169659 have direct and specific interests in the resolution of the
functions in the implementation of laws.  controversy, petitioner PDP-Laban is bereft of standing to file its petition.
Its allegation that E.O. 464 hampers its legislative agenda is vague and
uncertain, and at best is only a "generalized interest" which it shares with
The national political party, Bayan Muna, likewise meets the standing
the rest of the political parties. Concrete injury, whether actual or
requirement as it obtained three seats in the House of Representatives in
threatened, is that indispensable element of a dispute which serves in part
the 2004 elections and is, therefore, entitled to participate in the legislative
to cast it in a form traditionally capable of judicial resolution.55 In fine, PDP-
process consonant with the declared policy underlying the party list system
Laban's alleged interest as a political party does not suffice to clothe it with
of affording citizens belonging to marginalized and underrepresented
legal standing.
sectors, organizations and parties who lack well-defined political
constituencies to contribute to the formulation and enactment of legislation
that will benefit the nation.48 Actual Case or Controversy

Petitioners assert that an actual case exists, they citing the absence of the
executive officials invited by the Senate to its hearings after the issuance of
E.O. 464, particularly those on the NorthRail project and the wiretapping The Congress power of inquiry is expressly recognized in Section 21 of
controversy.  Article VI of the Constitution which reads:

Respondents counter that there is no case or controversy, there being no SECTION 21. The Senate or the House of Representatives or any of its
showing that President Arroyo has actually withheld her consent or respective committees may conduct inquiries in aid of legislation in
prohibited the appearance of the invited officials.56 These officials, they accordance with its duly published rules of procedure. The rights of persons
claim, merely communicated to the Senate that they have not yet secured appearing in or affected by such inquiries shall be respected. (Underscoring
the consent of the President, not that the President prohibited their supplied)ςrαlαωlιbrαrÿ
attendance.57 Specifically with regard to the AFP officers who did not attend
the hearing on September 28, 2005, respondents claim that the instruction
This provision is worded exactly as Section 8 of Article VIII of the 1973
not to attend without the President's consent was based on its role as
Constitution except that, in the latter, it vests the power of inquiry in the
Commander-in-Chief of the Armed Forces, not on E.O. 464.
unicameral legislature established therein - the Batasang Pambansa - and
its committees. 
Respondents thus conclude that the petitions merely rest on an unfounded
apprehension that the President will abuse its power of preventing the
The 1935 Constitution did not contain a similar provision. Nonetheless, in
appearance of officials before Congress, and that such apprehension is not
Arnault v. Nazareno,58 a case decided in 1950 under that Constitution, the
sufficient for challenging the validity of E.O. 464. 
Court already recognized that the power of inquiry is inherent in the power
to legislate.
The Court finds respondents' assertion that the President has not withheld
her consent or prohibited the appearance of the officials concerned
Arnault involved a Senate investigation of the reportedly anomalous
immaterial in determining the existence of an actual case or controversy
purchase of the Buenavista and Tambobong Estates by the Rural Progress
insofar as E.O. 464 is concerned. For E.O. 464 does not require either a
Administration. Arnault, who was considered a leading witness in the
deliberate withholding of consent or an express prohibition issuing from the
controversy, was called to testify thereon by the Senate. On account of his
President in order to bar officials from appearing before Congress. 
refusal to answer the questions of the senators on an important point, he
was, by resolution of the Senate, detained for contempt. Upholding the
As the implementation of the challenged order has already resulted in the Senate's power to punish Arnault for contempt, this Court held:
absence of officials invited to the hearings of petitioner Senate of the
Philippines, it would make no sense to wait for any further event before
Although there is no provision in the Constitution expressly investing either
considering the present case ripe for adjudication. Indeed, it would be
House of Congress with power to make investigations and exact testimony
sheer abandonment of duty if this Court would now refrain from passing on
to the end that it may exercise its legislative functions advisedly and
the constitutionality of E.O. 464. 
effectively, such power is so far incidental to the legislative function as to
be implied. In other words, the power of inquiry - with process to enforce it
Constitutionality of E.O. 464  - is an essential and appropriate auxiliary to the legislative function. A
legislative body cannot legislate wisely or effectively in the absence of
information respecting the conditions which the legislation is intended to
E.O. 464, to the extent that it bars the appearance of executive officials
affect or change; and where the legislative body does not itself possess the
before Congress, deprives Congress of the information in the possession of
requisite information - which is not infrequently true - recourse must be
these officials. To resolve the question of whether such withholding of
had to others who do possess it. Experience has shown that mere requests
information violates the Constitution, consideration of the general power of
for such information are often unavailing, and also that information which
Congress to obtain information, otherwise known as the power of inquiry,
is volunteered is not always accurate or complete; so some means of
is in order.
compulsion is essential to obtain what is needed.59 . . . (Emphasis and
underscoring supplied)ςrαlαωlιbrαrÿ
The power of inquiry
That this power of inquiry is broad enough to cover officials of the
executive branch may be deduced from the same case. The power of
inquiry, the Court therein ruled, is co-extensive with the power to done in accordance with the Senate or House's duly published rules of
legislate.60 The matters which may be a proper subject of legislation and procedure, necessarily implying the constitutional infirmity of an inquiry
those which may be a proper subject of investigation are one. It follows conducted without duly published rules of procedure. Section 21 also
that the operation of government, being a legitimate subject for legislation, mandates that the rights of persons appearing in or affected by such
is a proper subject for investigation.  inquiries be respected, an imposition that obligates Congress to adhere to
the guarantees in the Bill of Rights.
Thus, the Court found that the Senate investigation of the government
transaction involved in Arnault was a proper exercise of the power of These abuses are, of course, remediable before the courts, upon the proper
inquiry. Besides being related to the expenditure of public funds of which suit filed by the persons affected, even if they belong to the executive
Congress is the guardian, the transaction, the Court held, "also involved branch. Nonetheless, there may be exceptional circumstances, none
government agencies created by Congress and officers whose positions it is appearing to obtain at present, wherein a clear pattern of abuse of the
within the power of Congress to regulate or even abolish."  legislative power of inquiry might be established, resulting in palpable
violations of the rights guaranteed to members of the executive
department under the Bill of Rights. In such instances, depending on the
Since Congress has authority to inquire into the operations of the executive
particulars of each case, attempts by the Executive Branch to forestall
branch, it would be incongruous to hold that the power of inquiry does not
these abuses may be accorded judicial sanction.
extend to executive officials who are the most familiar with and informed
on executive operations. 
Even where the inquiry is in aid of legislation, there are still recognized
exemptions to the power of inquiry, which exemptions fall under the rubric
As discussed in Arnault, the power of inquiry, "with process to enforce it,"
of "executive privilege." Since this term figures prominently in the
is grounded on the necessity of information in the legislative process. If the
challenged order, it being mentioned in its provisions, its preambular
information possessed by executive officials on the operation of their
clauses,62and in its very title, a discussion of executive privilege is crucial
offices is necessary for wise legislation on that subject, by parity of
for determining the constitutionality of E.O. 464. 
reasoning, Congress has the right to that information and the power to
compel the disclosure thereof.
Executive privilege
As evidenced by the American experience during the so-called "McCarthy
era," however, the right of Congress to conduct inquiries in aid of The phrase "executive privilege" is not new in this jurisdiction. It has been
legislation is, in theory, no less susceptible to abuse than executive or used even prior to the promulgation of the 1986 Constitution.63 Being of
judicial power. It may thus be subjected to judicial review pursuant to the American origin, it is best understood in light of how it has been defined
Court's certiorari powers under Section 1, Article VIII of the Constitution. and used in the legal literature of the United States.

For one, as noted in Bengzon v. Senate Blue Ribbon Committee,61 the Schwartz defines executive privilege as "the power of the Government to
inquiry itself might not properly be in aid of legislation, and thus beyond withhold information from the public, the courts, and the
the constitutional power of Congress. Such inquiry could not usurp judicial Congress."64 Similarly, Rozell defines it as "the right of the President and
functions. Parenthetically, one possible way for Congress to avoid such a high-level executive branch officers to withhold information from Congress,
result as occurred in Bengzon is to indicate in its invitations to the public the courts, and ultimately the public."65
officials concerned, or to any person for that matter, the possible needed
statute which prompted the need for the inquiry. Given such statement in
Executive privilege is, nonetheless, not a clear or unitary concept.66 It has
its invitations, along with the usual indication of the subject of inquiry and
encompassed claims of varying kinds.67Tribe, in fact, comments that while
the questions relative to and in furtherance thereof, there would be less
it is customary to employ the phrase "executive privilege," it may be more
room for speculation on the part of the person invited on whether the
accurate to speak of executive privileges "since presidential refusals to
inquiry is in aid of legislation. 
furnish information may be actuated by any of at least three distinct kinds
of considerations, and may be asserted, with differing degrees of success,
Section 21, Article VI likewise establishes crucial safeguards that proscribe in the context of either judicial or legislative investigations." 
the legislative power of inquiry. The provision requires that the inquiry be
One variety of the privilege, Tribe explains, is the state secrets privilege The leading case on executive privilege in the United States is U.S. v.
invoked by U.S. Presidents, beginning with Washington, on the ground that Nixon, 72 decided in 1974. In issue in that case was the validity of President
the information is of such nature that its disclosure would subvert crucial Nixon's claim of executive privilege against a subpoena issued by a district
military or diplomatic objectives. Another variety is the informer's privilege, court requiring the production of certain tapes and documents relating to
or the privilege of the Government not to disclose the identity of persons the Watergate investigations. The claim of privilege was based on the
who furnish information of violations of law to officers charged with the President's general interest in the confidentiality of his conversations and
enforcement of that law. Finally, a generic privilege for internal correspondence. The U.S. Court held that while there is no explicit
deliberations has been said to attach to intragovernmental documents reference to a privilege of confidentiality in the U.S. Constitution, it is
reflecting advisory opinions, recommendations and deliberations constitutionally based to the extent that it relates to the effective discharge
comprising part of a process by which governmental decisions and policies of a President's powers. The Court, nonetheless, rejected the President's
are formulated.68 claim of privilege, ruling that the privilege must be balanced against the
public interest in the fair administration of criminal justice. Notably, the
Court was careful to clarify that it was not there addressing the issue of
Tribe's comment is supported by the ruling in In re Sealed Case, thus:
claims of privilege in a civil litigation or against congressional demands for
information. 
Since the beginnings of our nation, executive officials have claimed a
variety of privileges to resist disclosure of information the confidentiality of
Cases in the U.S. which involve claims of executive privilege against
which they felt was crucial to fulfillment of the unique role and
Congress are rare.73 Despite frequent assertion of the privilege to deny
responsibilities of the executive branch of our government. Courts ruled
information to Congress, beginning with President Washington's refusal to
early that the executive had a right to withhold documents that might
turn over treaty negotiation records to the House of Representatives, the
reveal military or state secrets. The courts have also granted the executive
U.S. Supreme Court has never adjudicated the issue.74 However, the U.S.
a right to withhold the identity of government informers in some
Court of Appeals for the District of Columbia Circuit, in a case decided
circumstances and a qualified right to withhold information related to
earlier in the same year as Nixon, recognized the President's privilege over
pending investigations. x x x"69 (Emphasis and underscoring
his conversations against a congressional subpoena.75 Anticipating the
supplied)ςrαlαωlιbrαrÿ
balancing approach adopted by the U.S. Supreme Court in Nixon, the Court
of Appeals weighed the public interest protected by the claim of privilege
The entry in Black's Law Dictionary on "executive privilege" is similarly against the interest that would be served by disclosure to the Committee.
instructive regarding the scope of the doctrine. Ruling that the balance favored the President, the Court declined to enforce
the subpoena.76
This privilege, based on the constitutional doctrine of separation of powers,
exempts the executive from disclosure requirements applicable to the In this jurisdiction, the doctrine of executive privilege was recognized by
ordinary citizen or organization where such exemption is necessary to the this Court in Almonte v. Vasquez.77 Almonte used the term in reference to
discharge of highly important executive responsibilities involved in the same privilege subject of Nixon. It quoted the following portion of the
maintaining governmental operations, and extends not only to military and Nixon decision which explains the basis for the privilege: 
diplomatic secrets but also to documents integral to an appropriate
exercise of the executive' domestic decisional and policy making functions,
"The expectation of a President to the confidentiality of his conversations
that is, those documents reflecting the frank expression necessary in intra-
and correspondences, like the claim of confidentiality of judicial
governmental advisory and deliberative communications.70 (Emphasis and
deliberations, for example, has all the values to which we accord deference
underscoring supplied)ςrαlαωlιbrαrÿ
for the privacy of all citizens and, added to those values, is the necessity
for protection of the public interest in candid, objective, and even blunt or
That a type of information is recognized as privileged does not, however, harsh opinions in Presidential decision-making. A President and those who
necessarily mean that it would be considered privileged in all instances. For assist him must be free to explore alternatives in the process of shaping
in determining the validity of a claim of privilege, the question that must be policies and making decisions and to do so in a way many would be
asked is not only whether the requested information falls within one of the unwilling to express except privately. These are the considerations
traditional privileges, but also whether that privilege should be honored in justifying a presumptive privilege for Presidential communications. The
a given procedural setting.71 privilege is fundamental to the operation of government and inextricably
rooted in the separation of powers under the Constitution x x x " however, which constrain this Court to discuss the validity of these
(Emphasis and underscoring supplied)ςrαlαωlιbrαrÿ provisions separately. 

Almonte involved a subpoena duces tecum issued by the Ombudsman Section 1 specifically applies to department heads. It does not, unlike
against the therein petitioners. It did not involve, as expressly stated in the Section 3, require a prior determination by any official whether they are
decision, the right of the people to information.78 Nonetheless, the Court covered by E.O. 464. The President herself has, through the challenged
recognized that there are certain types of information which the order, made the determination that they are. Further, unlike also Section
government may withhold from the public, thus acknowledging, in 3, the coverage of department heads under Section 1 is not made to
substance if not in name, that executive privilege may be claimed against depend on the department heads' possession of any information which
citizens' demands for information. might be covered by executive privilege. In fact, in marked contrast to
Section 3 vis - à-vis Section 2, there is no reference to executive privilege
at all. Rather, the required prior consent under Section 1 is grounded on
In Chavez v. PCGG,79 the Court held that this jurisdiction recognizes the
Article VI, Section 22 of the Constitution on what has been referred to as
common law holding that there is a "governmental privilege against public
the question hour. 
disclosure with respect to state secrets regarding military, diplomatic and
other national security matters."80 The same case held that closed-door
Cabinet meetings are also a recognized limitation on the right to SECTION 22. The heads of departments may upon their own initiative, with
information.  the consent of the President, or upon the request of either House, as the
rules of each House shall provide, appear before and be heard by such
House on any matter pertaining to their departments. Written questions
Similarly, in Chavez v. Public Estates Authority,81 the Court ruled that the
shall be submitted to the President of the Senate or the Speaker of the
right to information does not extend to matters recognized as "privileged
House of Representatives at least three days before their scheduled
information under the separation of powers,"82 by which the Court meant
appearance. Interpellations shall not be limited to written questions, but
Presidential conversations, correspondences, and discussions in closed-
may cover matters related thereto. When the security of the State or the
door Cabinet meetings. It also held that information on military and
public interest so requires and the President so states in writing, the
diplomatic secrets and those affecting national security, and information on
appearance shall be conducted in executive session. 
investigations of crimes by law enforcement agencies before the
prosecution of the accused were exempted from the right to information. 
Determining the validity of Section 1 thus requires an examination of the
meaning of Section 22 of Article VI. Section 22 which provides for the
From the above discussion on the meaning and scope of executive
question hour must be interpreted vis - à-vis Section 21 which provides for
privilege, both in the United States and in this jurisdiction, a clear principle
the power of either House of Congress to "conduct inquiries in aid of
emerges. Executive privilege, whether asserted against Congress, the
legislation." As the following excerpt of the deliberations of the
courts, or the public, is recognized only in relation to certain types of
Constitutional Commission shows, the framers were aware that these two
information of a sensitive character. While executive privilege is a
provisions involved distinct functions of Congress.
constitutional concept, a claim thereof may be valid or not depending on
the ground invoked to justify it and the context in which it is made.
Noticeably absent is any recognition that executive officials are exempt MR. MAAMBONG. x x x When we amended Section 20 [now Section 22 on
from the duty to disclose information by the mere fact of being executive the Question Hour] yesterday, I noticed that members of the Cabinet
officials. Indeed, the extraordinary character of the exemptions indicates cannot be compelled anymore to appear before the House of
that the presumption inclines heavily against executive secrecy and in Representatives or before the Senate. I have a particular problem in this
favor of disclosure.  regard, Madam President, because in our experience in the Regular
Batasang Pambansa - as the Gentleman himself has experienced in the
interim Batasang Pambansa - one of the most competent inputs that we
Validity of Section 1
can put in our committee deliberations, either in aid of legislation or in
congressional investigations, is the testimonies of Cabinet ministers. We
Section 1 is similar to Section 3 in that both require the officials covered by usually invite them, but if they do not come and it is a congressional
them to secure the consent of the President prior to appearing before investigation, we usually issue subpoenas.
Congress. There are significant differences between the two provisions,
I want to be clarified on a statement made by Commissioner Suarez when MR. GUINGONA. I ask Commissioner Maambong to reply, Mr. Presiding
he said that the fact that the Cabinet ministers may refuse to come to the Officer.
House of Representatives or the Senate [when requested under Section
22] does not mean that they need not come when they are invited or
MR. MAAMBONG. Actually, we considered that previously when we
subpoenaed by the committee of either House when it comes to inquiries in
sequenced this but we reasoned that in Section 21, which is Legislative
aid of legislation or congressional investigation. According to Commissioner
Inquiry, it is actually a power of Congress in terms of its own lawmaking;
Suarez, that is allowed and their presence can be had under Section 21.
whereas, a Question Hour is not actually a power in terms of its own
Does the gentleman confirm this, Madam President?cralawlibrary
lawmaking power because in Legislative Inquiry, it is in aid of legislation.
And so we put Question Hour as Section 31. I hope Commissioner Davide
MR. DAVIDE. We confirm that, Madam President, because Section 20 refers will consider this.
only to what was originally the Question Hour, whereas, Section 21 would
refer specifically to inquiries in aid of legislation, under which anybody for
MR. DAVIDE. The Question Hour is closely related with the legislative
that matter, may be summoned and if he refuses, he can be held in
power, and it is precisely as a complement to or a supplement of the
contempt of the House.83 (Emphasis and underscoring
Legislative Inquiry. The appearance of the members of Cabinet would be
supplied)ςrαlαωlιbrαrÿ
very, very essential not only in the application of check and balance but
also, in effect, in aid of legislation.
A distinction was thus made between inquiries in aid of legislation and the
question hour. While attendance was meant to be discretionary in the
MR. MAAMBONG. After conferring with the committee, we find merit in the
question hour, it was compulsory in inquiries in aid of legislation. The
suggestion of Commissioner Davide. In other words, we are accepting that
reference to Commissioner Suarez bears noting, he being one of the
and so this Section 31 would now become Section 22. Would it be,
proponents of the amendment to make the appearance of department
Commissioner Davide?cralawlibrary
heads discretionary in the question hour. 

MR. DAVIDE. Yes.84 (Emphasis and underscoring supplied)ςrαlαωlιbrαrÿ


So clearly was this distinction conveyed to the members of the Commission
that the Committee on Style, precisely in recognition of this distinction,
later moved the provision on question hour from its original position as Consistent with their statements earlier in the deliberations,
Section 20 in the original draft down to Section 31, far from the provision Commissioners Davide and Maambong proceeded from the same
on inquiries in aid of legislation. This gave rise to the following exchange assumption that these provisions pertained to two different functions of the
during the deliberations:  legislature. Both Commissioners understood that the power to conduct
inquiries in aid of legislation is different from the power to conduct inquiries
during the question hour. Commissioner Davide's only concern was that the
MR. GUINGONA. [speaking in his capacity as Chairman of the Committee
two provisions on these distinct powers be placed closely together, they
on Style] We now go, Mr. Presiding Officer, to the Article on Legislative and
being complementary to each other. Neither Commissioner considered
may I request the chairperson of the Legislative Department,
them as identical functions of Congress. 
Commissioner Davide, to give his reaction.

The foregoing opinion was not the two Commissioners' alone. From the
THE PRESIDING OFFICER (Mr. Jamir). Commissioner Davide is
above-quoted exchange, Commissioner Maambong's committee - the
recognized.ςηαñrοblεš νιr†υαl lαω lιbrαrÿ
Committee on Style - shared the view that the two provisions reflected
distinct functions of Congress. Commissioner Davide, on the other hand,
MR. DAVIDE. Thank you, Mr. Presiding Officer. I have only one reaction to was speaking in his capacity as Chairman of the Committee on the
the Question Hour. I propose that instead of putting it as Section 31, it Legislative Department. His views may thus be presumed as representing
should follow Legislative Inquiries. that of his Committee. 

THE PRESIDING OFFICER. What does the committee say?cralawlibrary In the context of a parliamentary system of government, the "question
hour" has a definite meaning. It is a period of confrontation initiated by
Parliament to hold the Prime Minister and the other ministers accountable administration in a system such as ours becomes a power devoid of most
for their acts and the operation of the government,85 corresponding to what of its practical content, since it depends for its effectiveness solely upon
is known in Britain as the question period. There was a specific provision information parceled out ex gratia by the executive.89 (Emphasis and
for a question hour in the 1973 Constitution86 which made the appearance underscoring supplied)ςrαlαωlιbrαrÿ
of ministers mandatory. The same perfectly conformed to the
parliamentary system established by that Constitution, where the ministers
Sections 21 and 22, therefore, while closely related and complementary to
are also members of the legislature and are directly accountable to it. 
each other, should not be considered as pertaining to the same power of
Congress. One specifically relates to the power to conduct inquiries in aid
An essential feature of the parliamentary system of government is the of legislation, the aim of which is to elicit information that may be used for
immediate accountability of the Prime Minister and the Cabinet to the legislation, while the other pertains to the power to conduct a question
National Assembly. They shall be responsible to the National Assembly for hour, the objective of which is to obtain information in pursuit of Congress'
the program of government and shall determine the guidelines of national oversight function. 
policy. Unlike in the presidential system where the tenure of office of all
elected officials cannot be terminated before their term expired, the Prime
When Congress merely seeks to be informed on how department heads are
Minister and the Cabinet remain in office only as long as they enjoy the
implementing the statutes which it has issued, its right to such information
confidence of the National Assembly. The moment this confidence is lost
is not as imperative as that of the President to whom, as Chief Executive,
the Prime Minister and the Cabinet may be changed.87
such department heads must give a report of their performance as a
matter of duty. In such instances, Section 22, in keeping with the
The framers of the 1987 Constitution removed the mandatory nature of separation of powers, states that Congress may only request their
such appearance during the question hour in the present Constitution so as appearance. Nonetheless, when the inquiry in which Congress requires
to conform more fully to a system of separation of powers.88 To that their appearance is "in aid of legislation" under Section 21, the appearance
extent, the question hour, as it is presently understood in this jurisdiction, is mandatory for the same reasons stated in Arnault.90
departs from the question period of the parliamentary system. That
department heads may not be required to appear in a question hour does
In fine, the oversight function of Congress may be facilitated by
not, however, mean that the legislature is rendered powerless to elicit
compulsory process only to the extent that it is performed in pursuit of
information from them in all circumstances. In fact, in light of the absence
legislation. This is consistent with the intent discerned from the
of a mandatory question period, the need to enforce Congress' right to
deliberations of the Constitutional Commission. 
executive information in the performance of its legislative function becomes
more imperative. As Schwartz observes:
Ultimately, the power of Congress to compel the appearance of executive
officials under Section 21 and the lack of it under Section 22 find their
Indeed, if the separation of powers has anything to tell us on the subject
basis in the principle of separation of powers. While the executive branch is
under discussion, it is that the Congress has the right to obtain information
a co-equal branch of the legislature, it cannot frustrate the power of
from any source - even from officials of departments and agencies in the
Congress to legislate by refusing to comply with its demands for
executive branch. In the United States there is, unlike the situation which
information. 
prevails in a parliamentary system such as that in Britain, a clear
separation between the legislative and executive branches. It is this very
separation that makes the congressional right to obtain information from When Congress exercises its power of inquiry, the only way for department
the executive so essential, if the functions of the Congress as the elected heads to exempt themselves therefrom is by a valid claim of privilege.
representatives of the people are adequately to be carried out. The They are not exempt by the mere fact that they are department heads.
absence of close rapport between the legislative and executive branches in Only one executive official may be exempted from this power - the
this country, comparable to those which exist under a parliamentary President on whom executive power is vested, hence, beyond the reach of
system, and the nonexistence in the Congress of an institution such as the Congress except through the power of impeachment. It is based on her
British question period have perforce made reliance by the Congress upon being the highest official of the executive branch, and the due respect
its right to obtain information from the executive essential, if it is accorded to a co-equal branch of government which is sanctioned by a
intelligently to perform its legislative tasks. Unless the Congress possesses long-standing custom. 
the right to obtain executive information, its power of oversight of
By the same token, members of the Supreme Court are also exempt from Executive Privilege" ', it is evident that under the rule of ejusdem generis,
this power of inquiry. Unlike the Presidency, judicial power is vested in a the determination by the President under this provision is intended to be
collegial body; hence, each member thereof is exempt on the basis not based on a similar finding of coverage under executive privilege. 
only of separation of powers but also on the fiscal autonomy and the
constitutional independence of the judiciary. This point is not in dispute, as
En passant, the Court notes that Section 2(b) of E.O. 464 virtually states
even counsel for the Senate, Sen. Joker Arroyo, admitted it during the oral
that executive privilege actually covers persons. Such is a misuse of the
argument upon interpellation of the Chief Justice.
doctrine. Executive privilege, as discussed above, is properly invoked in
relation to specific categories of information and not to categories of
Having established the proper interpretation of Section 22, Article VI of the persons. 
Constitution, the Court now proceeds to pass on the constitutionality of
Section 1 of E.O. 464. 
In light, however, of Sec 2(a) of E.O. 464 which deals with the nature,
scope and coverage of executive privilege, the reference to persons being
Section 1, in view of its specific reference to Section 22 of Article VI of the "covered by the executive privilege" may be read as an abbreviated way of
Constitution and the absence of any reference to inquiries in aid of saying that the person is in possession of information which is, in the
legislation, must be construed as limited in its application to appearances judgment of the head of office concerned, privileged as defined in Section
of department heads in the question hour contemplated in the provision of 2(a). The Court shall thus proceed on the assumption that this is the
said Section 22 of Article VI. The reading is dictated by the basic rule of intention of the challenged order. 
construction that issuances must be interpreted, as much as possible, in a
way that will render it constitutional. 
Upon a determination by the designated head of office or by the President
that an official is "covered by the executive privilege," such official is
The requirement then to secure presidential consent under Section 1, subjected to the requirement that he first secure the consent of the
limited as it is only to appearances in the question hour, is valid on its face. President prior to appearing before Congress. This requirement effectively
For under Section 22, Article VI of the Constitution, the appearance of bars the appearance of the official concerned unless the same is permitted
department heads in the question hour is discretionary on their part.  by the President. The proviso allowing the President to give its consent
means nothing more than that the President may reverse a prohibition
which already exists by virtue of E.O. 464. 
Section 1 cannot, however, be applied to appearances of department heads
in inquiries in aid of legislation. Congress is not bound in such instances to
respect the refusal of the department head to appear in such inquiry, Thus, underlying this requirement of prior consent is the determination by
unless a valid claim of privilege is subsequently made, either by the a head of office, authorized by the President under E.O. 464, or by the
President herself or by the Executive Secretary. President herself, that such official is in possession of information that is
covered by executive privilege. This determination then becomes the basis
for the official's not showing up in the legislative investigation.
Validity of Sections 2 and 3

In view thereof, whenever an official invokes E.O. 464 to justify his failure
Section 3 of E.O. 464 requires all the public officials enumerated in Section
to be present, such invocation must be construed as a declaration to
2(b) to secure the consent of the President prior to appearing before either
Congress that the President, or a head of office authorized by the
house of Congress. The enumeration is broad. It covers all senior officials
President, has determined that the requested information is privileged, and
of executive departments, all officers of the AFP and the PNP, and all senior
that the President has not reversed such determination. Such declaration,
national security officials who, in the judgment of the heads of offices
however, even without mentioning the term "executive privilege," amounts
designated in the same section (i.e. department heads, Chief of Staff of the
to an implied claim that the information is being withheld by the executive
AFP, Chief of the PNP, and the National Security Adviser), are "covered by
branch, by authority of the President, on the basis of executive privilege.
the executive privilege." 
Verily, there is an implied claim of privilege.

The enumeration also includes such other officers as may be determined by


the President. Given the title of Section 2 - "Nature, Scope and Coverage of
The letter dated September 28, 2005 of respondent Executive Secretary There is no claim by PEA that the information demanded by petitioner is
Ermita to Senate President Drilon illustrates the implied nature of the claim privileged information rooted in the separation of powers. The information
of privilege authorized by E.O. 464. It reads: does not cover Presidential conversations, correspondences, or discussions
during closed-door Cabinet meetings which, like internal-deliberations of
the Supreme Court and other collegiate courts, or executive sessions of
In connection with the inquiry to be conducted by the Committee of the
either house of Congress, are recognized as confidential. This kind of
Whole regarding the Northrail Project of the North Luzon Railways
information cannot be pried open by a co-equal branch of government. A
Corporation on 29 September 2005 at 10:00 a.m., please be informed that
frank exchange of exploratory ideas and assessments, free from the glare
officials of the Executive Department invited to appear at the meeting will
of publicity and pressure by interested parties, is essential to protect the
not be able to attend the same without the consent of the President,
independence of decision-making of those tasked to exercise Presidential,
pursuant to Executive Order No. 464 (s. 2005), entitled "Ensuring
Legislative and Judicial power. This is not the situation in the instant
Observance Of The Principle Of Separation Of Powers, Adherence To The
case.91 (Emphasis and underscoring supplied)ςrαlαωlιbrαrÿ
Rule On Executive Privilege And Respect For The Rights Of Public Officials
Appearing In Legislative Inquiries In Aid Of Legislation Under The
Constitution, And For Other Purposes". Said officials have not secured the Section 3 of E.O. 464, therefore, cannot be dismissed outright as invalid by
required consent from the President. (Underscoring supplied)ςrαlαωlιbrαrÿ the mere fact that it sanctions claims of executive privilege. This Court
must look further and assess the claim of privilege authorized by the Order
to determine whether it is valid. 
The letter does not explicitly invoke executive privilege or that the matter
on which these officials are being requested to be resource persons falls
under the recognized grounds of the privilege to justify their absence. Nor While the validity of claims of privilege must be assessed on a case to case
does it expressly state that in view of the lack of consent from the basis, examining the ground invoked therefor and the particular
President under E.O. 464, they cannot attend the hearing. circumstances surrounding it, there is, in an implied claim of privilege, a
defect that renders it invalid per se. By its very nature, and as
demonstrated by the letter of respondent Executive Secretary quoted
Significant premises in this letter, however, are left unstated, deliberately
above, the implied claim authorized by Section 3 of E.O. 464 is not
or not. The letter assumes that the invited officials are covered by E.O.
accompanied by any specific allegation of the basis thereof (e.g., whether
464. As explained earlier, however, to be covered by the order means that
the information demanded involves military or diplomatic secrets, closed-
a determination has been made, by the designated head of office or the
door Cabinet meetings, etc.). While Section 2(a) enumerates the types of
President, that the invited official possesses information that is covered by
information that are covered by the privilege under the challenged order,
executive privilege. Thus, although it is not stated in the letter that such
Congress is left to speculate as to which among them is being referred to
determination has been made, the same must be deemed implied.
by the executive. The enumeration is not even intended to be
Respecting the statement that the invited officials have not secured the
comprehensive, but a mere statement of what is included in the phrase
consent of the President, it only means that the President has not reversed
"confidential or classified information between the President and the public
the standing prohibition against their appearance before Congress. 
officers covered by this executive order." 

Inevitably, Executive Secretary Ermita's letter leads to the conclusion that


Certainly, Congress has the right to know why the executive considers the
the executive branch, either through the President or the heads of offices
requested information privileged. It does not suffice to merely declare that
authorized under E.O. 464, has made a determination that the information
the President, or an authorized head of office, has determined that it is so,
required by the Senate is privileged, and that, at the time of writing, there
and that the President has not overturned that determination. Such
has been no contrary pronouncement from the President. In fine, an
declaration leaves Congress in the dark on how the requested information
implied claim of privilege has been made by the executive.
could be classified as privileged. That the message is couched in terms
that, on first impression, do not seem like a claim of privilege only makes it
While there is no Philippine case that directly addresses the issue of more pernicious. It threatens to make Congress doubly blind to the
whether executive privilege may be invoked against Congress, it is question of why the executive branch is not providing it with the
gathered from Chavez v. PEA that certain information in the possession of information that it has requested. 
the executive may validly be claimed as privileged even against Congress.
Thus, the case holds:
A claim of privilege, being a claim of exemption from an obligation to Mobil Oil Corp. v. Department of Energy99 similarly emphasizes that "an
disclose information, must, therefore, be clearly asserted. As U.S. v. agency must provide 'precise and certain' reasons for preserving the
Reynolds teaches: confidentiality of requested information." 

The privilege belongs to the government and must be asserted by it; it can Black v. Sheraton Corp. of America100 amplifies, thus:
neither be claimed nor waived by a private party. It is not to be lightly
invoked. There must be a formal claim of privilege, lodged by the head of
A formal and proper claim of executive privilege requires a specific
the department which has control over the matter, after actual personal
designation and description of the documents within its scope as well as
consideration by that officer. The court itself must determine whether the
precise and certain reasons for preserving their confidentiality. Without this
circumstances are appropriate for the claim of privilege, and yet do so
specificity, it is impossible for a court to analyze the claim short of
without forcing a disclosure of the very thing the privilege is designed to
disclosure of the very thing sought to be protected. As the affidavit now
protect.92(Underscoring supplied)ςrαlαωlιbrαrÿ
stands, the Court has little more than its sua sponte speculation with which
to weigh the applicability of the claim. An improperly asserted claim of
Absent then a statement of the specific basis of a claim of executive privilege is no claim of privilege. Therefore, despite the fact that a claim
privilege, there is no way of determining whether it falls under one of the was made by the proper executive as Reynolds requires, the Court can not
traditional privileges, or whether, given the circumstances in which it is recognize the claim in the instant case because it is legally insufficient to
made, it should be respected.93 These, in substance, were the same criteria allow the Court to make a just and reasonable determination as to its
in assessing the claim of privilege asserted against the Ombudsman in applicability. To recognize such a broad claim in which the Defendant has
Almonte v. Vasquez94 and, more in point, against a committee of the given no precise or compelling reasons to shield these documents from
Senate in Senate Select Committee on Presidential Campaign Activities v. outside scrutiny, would make a farce of the whole procedure.101(Emphasis
Nixon.95 and underscoring supplied)ςrαlαωlιbrαrÿ

A.O. Smith v. Federal Trade Commission is enlightening:  Due respect for a co-equal branch of government, moreover, demands no
less than a claim of privilege clearly stating the grounds therefor. Apropos
is the following ruling in McPhaul v. U.S:102
[T]he lack of specificity renders an assessment of the potential harm
resulting from disclosure impossible, thereby preventing the Court from
balancing such harm against plaintiffs' needs to determine whether to We think the Court's decision in United States v. Bryan, 339 U.S. 323, 70
override any claims of privilege.96 (Underscoring supplied)ςrαlαωlιbrαrÿ S. Ct. 724, is highly relevant to these questions. For it is as true here as it
was there, that 'if (petitioner) had legitimate reasons for failing to produce
the records of the association, a decent respect for the House of
And so is U.S. v. Article of Drug:97
Representatives, by whose authority the subpoenas issued, would have
required that (he) state (his) reasons for noncompliance upon the return of
On the present state of the record, this Court is not called upon to perform the writ. Such a statement would have given the Subcommittee an
this balancing operation. In stating its objection to claimant's opportunity to avoid the blocking of its inquiry by taking other appropriate
interrogatories, government asserts, and nothing more, that the steps to obtain the records. 'To deny the Committee the opportunity to
disclosures sought by claimant would inhibit the free expression of opinion consider the objection or remedy is in itself a contempt of its authority and
that non-disclosure is designed to protect. The government has not shown an obstruction of its processes. His failure to make any such statement was
- nor even alleged - that those who evaluated claimant's product were "a patent evasion of the duty of one summoned to produce papers before a
involved in internal policymaking, generally, or in this particular instance. congressional committee[, and] cannot be condoned." (Emphasis and
Privilege cannot be set up by an unsupported claim. The facts upon which underscoring supplied; citations omitted)
the privilege is based must be established. To find these interrogatories
objectionable, this Court would have to assume that the evaluation and
Upon the other hand, Congress must not require the executive to state the
classification of claimant's products was a matter of internal policy
reasons for the claim with such particularity as to compel disclosure of the
formulation, an assumption in which this Court is unwilling to indulge sua
information which the privilege is meant to protect.103 A useful analogy in
sponte.98 (Emphasis and underscoring supplied)ςrαlαωlιbrαrÿ
determining the requisite degree of particularity would be the privilege appearance of such official. These provisions thus allow the President to
against self-incrimination. Thus, Hoffman v. U.S.104declares:  authorize claims of privilege by mere silence.

The witness is not exonerated from answering merely because he declares Such presumptive authorization, however, is contrary to the exceptional
that in so doing he would incriminate himself - his say-so does not of itself nature of the privilege. Executive privilege, as already discussed, is
establish the hazard of incrimination. It is for the court to say whether his recognized with respect to information the confidential nature of which is
silence is justified, and to require him to answer if 'it clearly appears to the crucial to the fulfillment of the unique role and responsibilities of the
court that he is mistaken.' However, if the witness, upon interposing his executive branch,105 or in those instances where exemption from disclosure
claim, were required to prove the hazard in the sense in which a claim is is necessary to the discharge of highly important executive
usually required to be established in court, he would be compelled to responsibilities.106 The doctrine of executive privilege is thus premised on
surrender the very protection which the privilege is designed to guarantee. the fact that certain informations must, as a matter of necessity, be kept
To sustain the privilege, it need only be evident from the implications of confidential in pursuit of the public interest. The privilege being, by
the question, in the setting in which it is asked, that a responsive answer definition, an exemption from the obligation to disclose information, in this
to the question or an explanation of why it cannot be answered might be case to Congress, the necessity must be of such high degree as to
dangerous because injurious disclosure could result." x x x (Emphasis and outweigh the public interest in enforcing that obligation in a particular
underscoring supplied)ςrαlαωlιbrαrÿ case. 

The claim of privilege under Section 3 of E.O. 464 in relation to Section In light of this highly exceptional nature of the privilege, the Court finds it
2(b) is thus invalid per se. It is not asserted. It is merely implied. Instead essential to limit to the President the power to invoke the privilege. She
of providing precise and certain reasons for the claim, it merely invokes may of course authorize the Executive Secretary to invoke the privilege on
E.O. 464, coupled with an announcement that the President has not given her behalf, in which case the Executive Secretary must state that the
her consent. It is woefully insufficient for Congress to determine whether authority is "By order of the President," which means that he personally
the withholding of information is justified under the circumstances of each consulted with her. The privilege being an extraordinary power, it must be
case. It severely frustrates the power of inquiry of Congress.  wielded only by the highest official in the executive hierarchy. In other
words, the President may not authorize her subordinates to exercise such
power. There is even less reason to uphold such authorization in the
In fine, Section 3 and Section 2(b) of E.O. 464 must be invalidated. 
instant case where the authorization is not explicit but by mere silence.
Section 3, in relation to Section 2(b), is further invalid on this score.
No infirmity, however, can be imputed to Section 2(a) as it merely provides
guidelines, binding only on the heads of office mentioned in Section 2(b),
It follows, therefore, that when an official is being summoned by Congress
on what is covered by executive privilege. It does not purport to be
on a matter which, in his own judgment, might be covered by executive
conclusive on the other branches of government. It may thus be construed
privilege, he must be afforded reasonable time to inform the President or
as a mere expression of opinion by the President regarding the nature and
the Executive Secretary of the possible need for invoking the privilege. This
scope of executive privilege. 
is necessary in order to provide the President or the Executive Secretary
with fair opportunity to consider whether the matter indeed calls for a claim
Petitioners, however, assert as another ground for invalidating the of executive privilege. If, after the lapse of that reasonable time, neither
challenged order the alleged unlawful delegation of authority to the heads the President nor the Executive Secretary invokes the privilege, Congress is
of offices in Section 2(b). Petitioner Senate of the Philippines, in particular, no longer bound to respect the failure of the official to appear before
cites the case of the United States where, so it claims, only the President Congress and may then opt to avail of the necessary legal means to
can assert executive privilege to withhold information from Congress.  compel his appearance.

Section 2(b) in relation to Section 3 virtually provides that, once the head The Court notes that one of the expressed purposes for requiring officials
of office determines that a certain information is privileged, such to secure the consent of the President under Section 3 of E.O. 464 is to
determination is presumed to bear the President's authority and has the ensure "respect for the rights of public officials appearing in inquiries in aid
effect of prohibiting the official from appearing before Congress, subject of legislation." That such rights must indeed be respected by Congress is
only to the express pronouncement of the President that it is allowing the an echo from Article VI Section 21 of the Constitution mandating that
"[t]he rights of persons appearing in or affected by such inquiries shall be It is in the interest of the State that the channels for free political
respected."  discussion be maintained to the end that the government may perceive and
be responsive to the people's will. Yet, this open dialogue can be effective
only to the extent that the citizenry is informed and thus able to formulate
In light of the above discussion of Section 3, it is clear that it is essentially
its will intelligently. Only when the participants in the discussion are aware
an authorization for implied claims of executive privilege, for which reason
of the issues and have access to information relating thereto can such bear
it must be invalidated. That such authorization is partly motivated by the
fruit.107 (Emphasis and underscoring supplied)ςrαlαωlιbrαrÿ
need to ensure respect for such officials does not change the infirm nature
of the authorization itself. 
The impairment of the right of the people to information as a consequence
of E.O. 464 is, therefore, in the sense explained above, just as direct as its
Right to Information
violation of the legislature's power of inquiry. 

E.O 464 is concerned only with the demands of Congress for the
Implementation of E.O. 464 prior to its publication
appearance of executive officials in the hearings conducted by it, and not
with the demands of citizens for information pursuant to their right to
information on matters of public concern. Petitioners are not amiss in While E.O. 464 applies only to officials of the executive branch, it does not
claiming, however, that what is involved in the present controversy is not follow that the same is exempt from the need for publication. On the need
merely the legislative power of inquiry, but the right of the people to for publishing even those statutes that do not directly apply to people in
information.  general, Tañada v. Tuvera states:

There are, it bears noting, clear distinctions between the right of Congress The term "laws" should refer to all laws and not only to those of general
to information which underlies the power of inquiry and the right of the application, for strictly speaking all laws relate to the people in general
people to information on matters of public concern. For one, the demand of albeit there are some that do not apply to them directly. An example is a
a citizen for the production of documents pursuant to his right to law granting citizenship to a particular individual, like a relative of
information does not have the same obligatory force as a subpoena duces President Marcos who was decreed instant naturalization. It surely cannot
tecum issued by Congress. Neither does the right to information grant a be said that such a law does not affect the public although it
citizen the power to exact testimony from government officials. These unquestionably does not apply directly to all the people. The subject of
powers belong only to Congress and not to an individual citizen.  such law is a matter of public interest which any member of the body
politic may question in the political forums or, if he is a proper party, even
in courts of justice.108(Emphasis and underscoring supplied)ςrαlαωlιbrαrÿ
Thus, while Congress is composed of representatives elected by the people,
it does not follow, except in a highly qualified sense, that in every exercise
of its power of inquiry, the people are exercising their right to information.  Although the above statement was made in reference to statutes, logic
dictates that the challenged order must be covered by the publication
requirement. As explained above, E.O. 464 has a direct effect on the right
To the extent that investigations in aid of legislation are generally
of the people to information on matters of public concern. It is, therefore, a
conducted in public, however, any executive issuance tending to unduly
matter of public interest which members of the body politic may question
limit disclosures of information in such investigations necessarily deprives
before this Court. Due process thus requires that the people should have
the people of information which, being presumed to be in aid of legislation,
been apprised of this issuance before it was implemented. 
is presumed to be a matter of public concern. The citizens are thereby
denied access to information which they can use in formulating their own
opinions on the matter before Congress - opinions which they can then Conclusion
communicate to their representatives and other government officials
through the various legal means allowed by their freedom of expression.
Congress undoubtedly has a right to information from the executive branch
Thus holds Valmonte v. Belmonte: 
whenever it is sought in aid of legislation. If the executive branch withholds
such information on the ground that it is privileged, it must so assert it and
state the reason therefor and why it must be respected.
The infirm provisions of E.O. 464, however, allow the executive branch to
evade congressional requests for information without need of clearly
asserting a right to do so and/or proffering its reasons therefor. By the
mere expedient of invoking said provisions, the power of Congress to
conduct inquiries in aid of legislation is frustrated. That is impermissible.
For 

[w]hat republican theory did accomplish was to reverse the old


[G.R. No. 89914. November 20, 1991.]
presumption in favor of secrecy, based on the divine right of kings and
nobles, and replace it with a presumption in favor of publicity, based on the
JOSE F.S. BENGZON JR., ABELARDO TERMULO, JOSE MANTECON,
doctrine of popular sovereignty. (Underscoring supplied)109
VICENTE MILLS JR., LEONARDO GAMBOA, KURT BACHMANN JR.,
JOSE V.E. JIMENEZ, ERNESTO CALUYA, AGERICO UNGSON, SUSAN
Resort to any means then by which officials of the executive branch could ROXAS, ELVIE CASTILLO, and CYNTHIA SABIDO
refuse to divulge information cannot be presumed valid. Otherwise, we LIMJAP, Petitioners, v. THE SENATE BLUE RIBBON COMMITTEE AND
shall not have merely nullified the power of our legislature to inquire into ITS MEMBERS, represented by and through the CHAIRMAN, HON.
the operations of government, but we shall have given up something of WIGBERTO TAÑADA, Respondents, JOSE S. SANDEJAS, intervenor.
much greater value - our right as a people to take part in government.

WHEREFORE, the petitions are PARTLY GRANTED. Sections 2(b) and 3 of SYLLABUS
Executive Order No. 464 (series of 2005), "Ensuring Observance of the
Principle of Separation of Powers, Adherence to the Rule on Executive 

1. POLITICAL LAW; JUDICIAL DEPARTMENT; HAS THE POWER TO


Privilege and Respect for the Rights of Public Officials Appearing in DETERMINE THE SCOPE AND EXTENT OF THE POWER OF THE LEGISLATIVE
Legislative Inquiries in Aid of Legislation Under the Constitution, and For COMMITTEES TO CONDUCT INQUIRIES INTO PRIVATE AFFAIRS IN
Other Purposes," are declared VOID. Sections 1 and 2(a) are, however, PURPORTED AID OF LEGISLATION. — The "allocation of constitutional
VALID.  boundaries" is a task that this Court must perform under the Constitution.
Moreover, as held in a recent case," (t)he political question doctrine neither
SO ORDERED. interposes an obstacle to judicial determination of the rival claims. The
jurisdiction to delimit constitutional boundaries has been given to this
Court. It cannot abdicate that obligation mandated by the 1987
Constitution, although said provision by no means does away with the
applicability of the principle in appropriate cases." The Court is thus of the
considered view that it has jurisdiction over the present controversy for the
purpose of determining the scope and extent of the power of the Senate
Blue Ribbon Committee to conduct inquiries into private affairs in purported
aid of legislation.

2. ID.; LEGISLATIVE DEPARTMENT; POWER TO CONDUCT INQUIRIES IN


AID OF LEGISLATION; RULE. — The power of both houses of Congress to
conduct inquiries in aid of legislation is not, therefore, absolute or
unlimited. Its exercise is circumscribed by the afore-quoted provision of the
Constitution. Thus, as provided therein, the investigation must be "in aid of
legislation in accordance with its duly published rules of procedure" and
that "the rights of persons appearing in or affected by such inquiries shall
be respected." It follows then that the rights of persons under the Bill of
Rights must be respected, including the right to due process and the right power of the legislature and even as expressly limited by the Constitution.
not to be compelled to testify against one’s self. The power to conduct
formal inquiries or investigations is specifically provided for in Sec. 1 of the 2. ID.; ID.; INDISPENSABLE DUTY TO INQUIRE INTO THE EXPENDITURE
Senate Rules of Procedure Governing Inquiries in Aid of Legislation. Such OF ALL PUBLIC FUNDS. — The inquiry deals with alleged manipulations of
inquiries may refer to the implementation or re-examination of any law or public funds and illicit acquisitions of properties now being claimed by the
in connection with any proposed legislation or the formulation of future PCGG for the Republic of the Philippines. The purpose of the Committee is
legislation. They may also extend to any and all matters vested by the to ascertain if and how such anomalies have been committed. It is settled
Constitution in Congress and/or in the Senate alone. As held in Jean L. that the legislature has a right to investigate the disposition of the public
Aznault v. Leon Nazareno, Et Al., the inquiry, to be within the jurisdiction of funds it has appropriated; indeed, "an inquiry into the expenditure of all
the legislative body making it, must be material or necessary to the public money is an indispensable duty of the legislature." Moreover, an
exercise of a power in it vested by the Constitution, such as to legislate or investigation of a possible violation of a law may be useful in the drafting of
to expel a member. Under Sec. 4 of the aforementioned Rules, the Senate mandatory legislation to correct or strengthen that law.
may refer to any committee or committees any speech or resolution filed
by any Senator which in its judgment requires an appropriate inquiry in aid 3. CONSTITUTIONAL LAW; BILL OF RIGHTS; RIGHT AGAINST SELF
of legislation. In order therefore to ascertain the character or nature of an INCRIMINATION; WHEN AVAILABLE. — The petitioners’ contention that the
inquiry, resort must be had to the speech or resolution under which such questioned investigation would compel them to reveal their defense in the
an inquiry is proposed to be made. cases now pending against them in the Sandiganbayan is untenable. They
know or should know that they cannot be compelled to answer
3. ID.; ID.; ID.; LIMITATIONS; REASONS THEREFOR. — Now to another incriminating questions. The case of Chavez v. Court of Appeals, 24 SCRA
matter. It has been held that "a congressional committee’s right to inquire 663, where we held that an accused may refuse at the outset to take the
is ‘subject to all relevant limitations placed by the Constitution on stand on the ground that the questions to be put by the prosecutor will
governmental action,’ including `the relevant limitations of the Bill of tend to incriminate him is, of course, not applicable to them. They are not
Rights’." In another case —." . . the mere semblance of legislative purpose facing criminal charges before the Blue Ribbon Committee. Like any
would not justify an inquiry in the face of the Bill of Rights. The critical ordinary witness, they can invoke the right against self-incrimination only
element is the existence of, and the weight to be ascribed to, the interest when and as the incriminating question is propounded.
of the Congress in demanding disclosures from an unwilling witness. We
cannot simply assume, however, that every congressional investigation is
justified by a public need that over-balances any private rights affected. To
DECISION
do so would be to abdicate the responsibility placed by the Constitution
upon the judiciary to insure that the Congress does not unjustifiably
encroach upon an individual’s right to privacy nor abridge his liberty of
speech, press, religion or assembly." (Watkins v. US, 354 USS 178 citing This is a petition for prohibition with prayer for the issuance of a temporary
US v. Rumely, 345 US 41). restraining order and/or injunctive relief, to enjoin the respondent Senate
Blue Ribbon Committee from requiring the petitioners to testify and
CRUZ, J., dissenting:chanrob1es virtual 1aw library produce evidence at its inquiry into the alleged sale of the equity of
Benjamin "Kokoy" Romualdez to the Lopa Group in thirty-six (36) or thirty-
1. POLITICAL LAW; LEGISLATIVE DEPARTMENT; POWER TO CONDUCT nine (39) corporations.
INVESTIGATION IN AID OF LEGISLATION; PRESUMED WITH A LEGITIMATE
OBJECT. — Justice Cruz do not agree that the investigation being On 30 July 1987, the Republic of the Philippines, represented by the
conducted by the Blue Ribbon Committee is not in aid of legislation. In Presidential Commission on Good Government (PCGG), assisted by the
Arnault v. Nazareno, 87 Phil. 29, this Court observed that "we are bound to Solicitor General, filed with the Sandiganbayan Civil Case No. 0035 (PCGG
presume that the action of the legislative body was with a legitimate object Case No. 35) entitled "Republic of the Philippines v. Benjamin "Kokoy"
if it is capable of being so construed, and we have no right to assume that Romualdez, Et. Al.", for reconveyance, reversion, accounting, restitution
the contrary was intended." (People ex rel. McDonald v. Keeler, 99 N.Y. and damages.
463; 52 Am. Rep., 49; 2 N.E., 615, quoted with approval by the U.S.
Supreme Court in McGrain v. Daugherty, 273 U.S. 135). As far as He The complaint was amended several times by impleading new defendants
knows, that is still the rule today. More importantly, the presumption is and or amplifying the allegations therein. Under the Second Amended
supported by the established facts. The inquiry is sustainable as an implied
Complaint, 1 the herein petitioners were impleaded as party defendants. Bengzon Law Offices, or specifically Defendants Jose F.S. Bengzon, Jr.,
Jose V.E. Jimenez, Amando V. Faustino, Jr., and Edilberto S. Narciso, Jr.
The complaint insofar as pertinent to herein petitioners, as defendants, manipulated, schemed, and/or executed a series of devices intended to
alleges among others that:jgc:chanrobles.com.ph conceal and place, and/or for the purpose of concealing and placing,
beyond the inquiry and jurisdiction of the Presidential Commission on Good
"14. Defendants Benjamin (Kokoy) Romualdez and Juliette Gomez Government (PCGG) herein Defendants’ individual and collective funds,
Romualdez, acting by themselves and/or in unlawful concert with properties, and assets subject of and/or suited in the instant Complaint.
Defendants Ferdinand E. Marcos and Imelda R. Marcos, and taking undue
advantage of their relationship, influence and connection with the latter (o) maneuvered, with the technical know-how and legalistic talents of the
Defendant spouses, engaged in devices, schemes and stratagems to FMMC senior managers and some of the Bengzon law partners, such as
unjustly enrich themselves at the expense of Plaintiff and the Filipino Attys. Jose F.S. Bengzon, Jr., Edilberto S. Narciso, Jr., Amando V. Faustino,
people, among others:chanrobles virtualawlibrary Jose Vicente E. Jimenez and Leonardo C. Cruz, the purported sale of
chanrobles.com:chanrobles.com.ph defendant Benjamin Romualdez’s interests in the (i) Professional Managers,
Inc., (ii) A & E International Corporation (A & E), (iii) First Manila
(a) obtained, with the active collaboration of Defendants Senen J. Management Corporation (FMMC), (iv) Maguindanao Navigation (MNI), (v)
Gabaldon, Mario D. Camacho, Mamerto Nepomuceno, Carlos J. Valdez, SOLOIL, Inc. (SOLOIL), (vi) Philippine World Travel Inc. (PWTI) and its
Cesar C. Zalamea and Francisco Tantuico, Atty. Jose Bengzon, Jr. and his subsidiaries consisting of 36 corporations in all, to PNI Holdings, Inc.
law partners, namely: Edilberto S. Narciso, Jr., Jose Vicente E. Jimenez, (whose purported incorporators are all members of Atty. Jose F S.
Amando V. Faustino, Jr., and Leonardo C. Cruz; Jose S. Sandejas and his Bengzon’s law firm) for only P5 million on March 3, 1986 or three days
fellow senior managers of FMMC/PNI Holdings groups of companies such as after the creation of the Presidential Commission on Good Government on
Leonardo Gamboa, Vicente T. Mills, Jr., Jose M. Mantecon, Abelardo S. February 28, 1986, for the sole purpose of deceiving and preempting the
Termulo, Rex C. Drilon II and Kurt Bachmann, Jr., control of some of the Government, particularly the PCGG, and making it appear that defendant
biggest business enterprises in the Philippines, such as the Manila Electric Benjamin Romualdez had already divested himself of his ownership of the
Company (MERALCO), Benguet Consolidated Mining Corporation same when in truth and in fact, his interests are well intact and being
(BENGUET), Pilipinas Shell Corporation and the Philippine Commercial protected by Atty. Jose F S. Bengzon, Jr. and some of hie law partners,
International Bank (PCI Bank) by employing devious financial schemes and together with the FMMC senior managers who still control and run the
techniques calculated to require the massive infusion and hemorrhage of affairs of said corporations, and in order to entice the PCGG to approve the
government funds with minimum or negligible ‘cashout’ from Defendant said fictitious sale, the above-named defendants offered P20 million as
Benjamin Romualdez. . . . ‘donation’ to the Government;

x       x       x (p) misused, with the connivance, support and technical assistance of the
Bengzon law finn represented by Atty. Jose F.S. Bengzon, Jr. as legal
counsel, together with defendants Cesar Zalamea, Antonio Ozaeta, Mario
(m) manipulated, with the support, assistance and collaboration of D. Camacho and Senen J. Gabaldon as members of the Board of Directors
Philguarantee officials led by chairman Cesar E.A. Virata and the senior of the Philippine Commercial International Bank (PCIB), the Meralco
managers of FMMC/PNI Holdings, Inc. led by Jose S. Sandejas, Jr., Jose M. Pension Fund (Fund, for short) in the amount of P25 million by causing it to
Mantecon and Kurt S. Bachmann, Jr., among others, the formation of be invested in the PCIB and through the Bank’s TSG, assigned to PCI
Erectors Holdings, Inc. without infusing additional capital solely for the Development and PCI Equity at 50% each, the Fund’s (a) 8,028,011
purpose of Erectors Incorporated with Philguarantee in the amount of common shares in the Bank and (b) ‘Deposit in Subscription’ in the amount
P527,387,440.71 with insufficient securities/collaterals just to enable of P4,929,972.50 but of the agreed consideration of P28 million for the said
Erectors Inc. to appear viable and to borrow more capitals, so much so that assignment, PCI Development and PCI Equity were able to pay only
its obligation with Philguarantee has reached a total of more than P2 Billion P5,500.00 downpayment and the first amortization of P3,937,500.00 thus
as of June 30, 1987. prompting the Fund to rescind its assignment, and the consequent
reversion of the assigned shares brought the total shareholding of the Fund
(n) at the onset of the present Administration and/or within the week to 11,470,555 voting shares or 36.8% of the voting stock of the PCIB, and
following the February 1986 People’s Revolution, in conspiracy with, this development (which the defendants themselves orchestrated or
support, assistance and collaboration of the abovenamed lawyers of the allowed to happen) was used by them as an excuse for the unlawful
dismantling or cancellation of the Fund’s 10 million shares for allegedly
exceeding the 30-percent ceiling prescribed by Section 12-B of the General Corrupt Practices Act." 4 
Banking Act, although they know for a fact that what the law declares as
unlawful and void ab initio are the subscriptions in excess of the 30% On motion of Senator Orlando Mercado, the matter was referred by the
ceiling ‘to the extent of the excess over any of the ceilings prescribed . . .’ Senate to the Committee on Accountability of Public Officers (Blue Ribbon
and not the whole or entire stockholding which they allowed to stay for six Committee). 5 Thereafter, the Senate Blue Ribbon Committee started its
years (from June 30, 1980 to March 24, 1986); investigation on the matter. Petitioners and Ricardo Lopa were subpoenaed
by the Committee to appear before it and testify on "what they know"
(q) cleverly hid behind the veil of corporate entity, through the use of the regarding the "sale of the thirty-six (36) corporations belonging to
names and managerial expertise of the FMMC senior managers and lawyers Benjamin "Kokoy" Romualdez." chanrobles lawlibrary : rednad
identified as Jose B. Sandejas, Leonardo Gamboa, Vicente T. Mills,
Abelardo S. Termulo, Edilberto S. Narciso, Jr., Jose M. Mantecon, Rex C. At the hearing held on 23 May 1989, Ricardo Lopa declined to testify on the
Drilon II, Kurt Bachmann, Jr. together with the legal talents of corporate Found that his testimony may "unduly prejudice" the defendants in Civil
lawyers, such as Attys. Jose F.S. Bengzon, Jr., Jose V.E. Jimenez, Amando Case No. 0035 before the Sandiganbayan. Petitioner Jose F.S. Bengzon, Jr.
V. Faustino, Jr. and Leonardo C. Cruz, the ill-gotten wealth of Benjamin T. likewise refused to testify invoking his constitutional right to due process,
Romualdez including, among others, the 6,229,177 shares in PCIB and averring that the publicity generated by respondent Committee’s
registered in the names of Trans Middle East Phils. Equities, Inc. and inquiry could adversely affect his rights as well as those of the other
Edilberto S. Narciso, Jr. which they refused to surrender to PCGG despite petitioners who are his co-defendants in Civil Case No. 0035 before the
their disclosure as they tried and continue to exert efforts in getting hold of Sandiganbayan.
the same as well as the shares in Benguet registered in the names of Palm
Avenue Holdings and Palm Avenue Realty Development Corp. purportedly The Senate Blue Ribbon Committee, thereupon, suspended its inquiry and
to be applied as payment for the claim of P70 million of a ‘merger company directed the petitioners to file their memorandum on the constitutional
of the First Manila Management Corp. group’ supposedly owned by them issues raised, after which, it issued a resolution 6 dated 5 June 1989
although the truth is that all the said firms are still beneficially owned by rejecting the petitioners’ plea to be excused from testifying, and the
defendants Benjamin Romualdez. Committee voted to pursue and continue its investigation of the matter.
Senator Neptali Gonzales dissented. 7 
x       x       x" 
Claiming that the Senate Blue Ribbon Committee is poised to subpoena
On 28 September 1988, petitioners (as defendants) filed their respective them and require their attendance and testimony in proceedings before the
answers. 2 Meanwhile, from 2 to 6 August 1988, conflicting reports on the Committee, in excess of its jurisdiction and legislative purpose, in clear and
disposition by the PCGG of the "Romualdez corporations" were carried in blatant disregard of their constitutional rights, and to their grave and
various metropolitan newspapers. Thus, one newspaper reported that the irreparable damage, prejudice and injury, and that there is no appeal nor
Romualdez firms had not been sequestered because of the opposition of any other plain, speedy and adequate remedy in the ordinary course of
certain PCGG officials who "had worked previously as lawyers of the Marcos law, the petitioners filed the present petition for prohibition with a prayer
crony firms." Another daily reported otherwise, while others declared that for temporary restraining order and/or injunctive relief.
on 3 March 1986, or shortly after the EDSA February 1986 revolution, the
"Romualdez companies" were sold for P5 million, without PCGG approval, Meanwhile, one of the defendants in Civil Case No. 0035 before the
to a holding company controlled by Romualdez, and that Ricardo Lopa, the Sandiganbayan, Jose S. Sandejas, filed with the Court a motion for
President’s brother-in-law, had effectively taken over the firms, even intervention, 8 which the Court granted in the resolution 9 of 21 December
pending negotiations for the purchase of the corporations, for the same 1989, and required the respondent Senate Blue Ribbon Committee to
price of P5 million which was reportedly way below the fair value of their comment on the petition in intervention. In compliance therewith,
assets. 3  respondent Senate Blue Ribbon Committee filed its comment 10 thereon.

On 13 September 1988, the Senate Minority Floor Leader, Hon. Juan Ponce Before discussing the issues raised by petitioners and intervenor, we will
Enrile delivered a speech "on a matter of personal privilege" before the first tackle the jurisdictional question raised by the respondent Committee.
Senate on the alleged "take-over of SOLOIL Incorporated, the flagship of
the First Manila Management of Companies (FMMC) by Ricardo Lopa" and In its comment, respondent Committee claims that this Court cannot
called upon "the Senate to look into the possible violation of the law in the properly inquire into the motives of the lawmakers in conducting legislative
case, particularly with regard to Republic Act No. 3019, the Anti-Graft and investigations, much less can it enjoin the Congress or any of its regular
and special committees — like what petitioners seek — from making of judicial review under the Constitution. Even then, this power of judicial
inquiries in aid of legislation, under the doctrine of separation of powers, review is limited to actual cases and controversies to be exercised after full
which obtains in our present system of government. opportunity of argument by the parties, and limited further to the
constitutional question raised or the very lis mota presented. Any attempt
The contention is untenable. In Angara v. Electoral Commission, 11 the at abstraction could only lead to dialectics and barren legal questions and
Court held:jgc:chanrobles.com.ph to sterile conclusions unrelated to actualities. Narrowed as its function is in
this manner, the judiciary does not pass upon questions of wisdom, justice
"The separation of powers is a fundamental principle in our system of or expediency of legislation. More than that, courts accord the presumption
government. It obtains not through express provision but by actual division of constitutionality to legislative enactments, not only because the
in our Constitution. Each department of the government has exclusive legislature is presumed to abide by the Constitution but also because the
cognizance of matters within its jurisdiction, and is supreme within its own judiciary in the determination of actual cases and controversies must
sphere. But it does not follow from the fact that the three powers are to be reflect the wisdom and justice of the people as expressed through their
kept separate and distinct that the Constitution intended them to be representatives in the executive and legislative departments of the
absolutely unrestrained and independent of each other. The Constitution government."cralaw virtua1aw library
has provided for an elaborate system of checks and balances to secure
coordination in the workings of the various departments of the The "allocation of constitutional boundaries" is a task that this Court must
government. perform under the Constitution. Moreover, as held in a recent case, 12"
(t)he political question doctrine neither interposes an obstacle to judicial
x       x       x determination of the rival claims. The jurisdiction to delimit constitutional
boundaries has been given to this Court. It cannot abdicate that obligation
mandated by the 1987 Constitution, although said provision by no means
"But in the main, the Constitution has blocked out with deft strokes and in does away with the applicability of the principle in appropriate cases." 13 
bold lines, allotment of power to the executive, the legislative and the
judicial departments of the government. The overlapping and interlacing of The Court is thus of the considered view that it has jurisdiction over the
functions and duties between the several departments, however, present controversy for the purpose of determining the scope and extent of
sometimes makes it hard to say just where the one leaves off and the the power of the Senate Blue Ribbon Committee to conduct inquiries into
other begins. In times of social disquietude or political excitement, the private affairs in purported aid of legislation.
great landmarks of the Constitution are apt to be forgotten or marred, if
not entirely obliterated. In cases of conflict, the judicial department is the Coming to the specific issues raised in this case, petitioners contend that
only constitutional organ which can be called upon to determine the proper (1) the Senate Blue Ribbon Committee’s inquiry has no valid legislative
allocation of powers between the several departments and among the purpose, i.e., it is not done in aid of legislation; (2) the sale or disposition
integral or constituent units thereof.chanrobles.com:cralaw:red of the Romualdez corporations is a "purely private transaction" which is
beyond the power of the Senate Blue Ribbon Committee to inquire into;
and (3) the inquiry violates their right to due process.
x       x       x
The 1987 Constitution expressly recognizes the power of both houses of
Congress to conduct inquiries in aid of legislation. 14 Thus, Section 21,
The Constitution is a definition of the powers of government. Who is to
Article VI thereof provides:jgc:chanrobles.com.ph
determine the nature, scope and extent of such powers? The Constitution
itself has provided for the instrumentality of the judiciary as the rational
"The Senate or the House of Representatives or any of its respective
way. And when the judiciary mediates to allocate constitutional boundaries;
committee may conduct inquiries in aid of legislation in accordance wish its
it does not assert any superiority over the other departments; it does not
duly published rules of procedure. The rights of persons appearing in or
in reality nullify or invalidate an act of the legislature, but only asserts the
affected by such inquiries shall be respected." 15 
solemn and sacred obligation assigned to it by the Constitution to
determine conflicting claims of authority under the Constitution and to
The power of both houses of Congress to conduct inquiries in aid of
establish for the parties in an actual controversy the rights which that
legislation is not, therefore, absolute or unlimited. Its exercise is
instrument secures and guarantees to them. This is in truth all that is
circumscribed by the afore-quoted provision of the Constitution. Thus, as
involved in what is termed ‘judicial supremacy’ which properly is the power
provided therein, the investigation must be "in aid of legislation in
accordance with its duly published rules of procedure" and that "the rights denies categorically that he has taken over the First Manila Management
of persons appearing in or affected by such inquiries shall be respected." It Group of Companies which includes SOLOIL Incorporated.
follows then that the rights of persons under the Bill of Rights must be
respected, including the right to due process and the right not to be x       x       x
compelled to testify against one’s self.chanrobles virtual lawlibrary

The power to conduct formal inquiries or investigations is specifically "In answer to Mr. Lopa, I will quote pertinent portions from an Official
provided for in Sec. 1 of the Senate Rules of Procedure Governing Inquiries Memorandum to the Presidential Commission on Good Government written
in Aid of Legislation. Such inquiries may refer to the implementation or re- and signed by former Governor, now Congressman Jose Ramirez, in his
examination of any law or in connection with any proposed legislation or capacity as head of the PCGG Task Force for Region VIII. In his
the formulation of future legislation. They may also extend to any and all memorandum dated July 3, 1986, then Governor Ramirez stated that when
matters vested by the Constitution in Congress and/or in the Senate alone. he and the members of his task force sought to serve a sequestration order
on the management of SOLOIL in Tanauan, Leyte, management officials
As held in Jean L. Aznault v. Leon Nazareno, Et Al., 16 the inquiry, to be assured him that relatives of the President of the Philippines were
within the jurisdiction of the legislative body making it, must be material or personally discussing and representing SOLOIL so that the order of
necessary to the exercise of a power in it vested by the Constitution, such sequestration would be lifted and that the new owner was Mr. Ricardo A.
as to legislate or to expel a member. Lopa.

Under Sec. 4 of the aforementioned Rules, the Senate may refer to any "I will quote the pertinent portions in the Ramirez’ memorandum.
committee or committees any speech or resolution filed by any Senator
which in its judgment requires an appropriate inquiry in aid of legislation. "The first paragraph of the memorandum reads as follows and I quote, Mr.
In order therefore to ascertain the character or nature of an inquiry, resort President:chanrob1es virtual 1aw library
must be had to the speech or resolution under which such an inquiry is
proposed to be made. ‘Our sequestration work of SOLOIL in Tanauan, Leyte was not heeded by
management because they said another representation was being made to
A perusal of the speech of Senator Enrile reveals that he (Senator Enrile) this Commission for the eventual lifting of our sequestration order. They
made a statement which was published in various newspapers on 2 even assured us that Mr. Ricardo Lopa and Peping Cojuangco were
September 1988 accusing Mr. Ricardo "Baby" Lopa of "having taken over personally discussing and representing SOLOIL, so the order of
the FMMC Group of Companies." As a consequence thereof, Mr. Lopa wrote sequestration will finally be lifted. While we attempted to carry on our
a letter to Senator Enrile on 4 September 1988 categorically denying that order, management refused to cooperate and vehemently turned down our
he had "taken over" the FMMC Group of Companies; that former PCGG request to make available to us the records of the company. In fact it was
Chairman Ramon Diaz himself categorically stated in a telecast interview obviously clear that they will meet us with force the moment we insist on
by Mr. Luis Beltran on Channel 7 on 31 August 1988 that there has been doing normally our assigned task. In view of the impending threat, and to
no takeover by him (Lopa); and that these repeated allegations of a avoid any untoward incident we decided to temporarily suspend our work
"takeover" on his (Lopa’s) part of FMMC are baseless as they are malicious. until there is a more categorical stand of this Commission in view of the
seemingly influential representation being made by SOLOIL for us not to
The Lopa reply prompted Senator Enrile, during the session of the Senate continue our work.’
on 13 September 1988, to avail of the privilege hour, 17 so that he could
respond to the said Lopa letter, and also to vindicate his reputation as a "Another pertinent portion of the same memorandum is paragraph five,
Member of the Senate of the Philippines, considering the claim of Mr. Lopa which reads as follows, and I quote Mr. President:chanrobles.com : virtual
that his (Enrile’s) charges that he (Lopa) had taken over the FMMC Group law library
of Companies are "baseless" and "malicious." Thus, in his speech, 18
Senator Enrile said, among others, as follows:jgc:chanrobles.com.ph ‘The President, Mr. Gamboa, this is, I understand, the President of SOLOIL,
and the Plant Superintendent, Mr. Jimenez including their chief counsel,
"Mr. President, I rise this afternoon on a matter of personal privilege; the Atty. Mandong Mendiola are now saying that there have been divestment,
privilege being that I received, Mr. President, a letter dated September 4, and that the new owner is now Mr. Ricardo Lopa who according to them, is
1988, signed by Mr. Ricardo A. Lopa, a.k.a. or Baby Lopa, wherein he the brother-in-law of the President. They even went further by telling us
that ever Peping Cojuangco who we know is the brother of her excellency is transaction, contract or application filed by him for approval of which is not
also interested in the ownership and management of SOLOIL. When he discretionary on the part of the officials concerned but depends upon
demanded for supporting papers which will indicate aforesaid divestment, compliance with requisites provided by law, nor to any act lawfully
Messrs. Gamboa, Jimenez and Mendiola refused vehemently to submit performed in an official capacity or in the exercise of a profession.’
these papers to us, instead they said it will be submitted directly to this
Commission. To our mind their continuous dropping of names is not good "Mr. President, I have done duty to this Senate and to myself. I leave it to
for this Commission and even to the President if our desire is to achieve this august Body to make its own conclusion."cralaw virtua1aw library
respectability and stability of the government.’
Verily, the speech of Senator Enrile contained no suggestion of
"The contents of the memorandum of then Governor and now contemplated legislation; he merely called upon the Senate to look into a
Congressman Jose Ramirez were personally confirmed by him in a news possible violation of Sec. 5 of RA No. 3019, otherwise known as "The Anti-
interview last September 7, 1988. Graft and Corrupt Practices Act." In other words, the purpose of the inquiry
to be conducted by respondent Blue Ribbon Committee was to find out
x       x       x whether or not the relatives of President Aquino, particularly Mr. Ricardo
Lopa, had violated the law in connection with the alleged sale of the 36 or
39 corporations belonging to Benjamin "Kokoy" Romualdez to the Lopa
"Also relevant to this case, Mr. President, is a letter of Mr. Ricardo Lopa Group. There appears to be, therefore, no intended legislation involved.
himself in August 11, 1988 issue of the newspaper Malaya headlined ‘On
Alleged Takeover of Romualdez Firms.’ The Court is also not impressed with the respondent Committee’s argument
that the questioned inquiry is to be conducted pursuant to Senate
"Mr. Lopa states in the last paragraph of the published letter and I quote Resolution No. 212. The said resolution was introduced by Senator Jose D.
him:chanrob1es virtual 1aw library Lina in view of the representations made by leaders of school youth,
community groups and youth of non-governmental organizations to the
‘12. As of this writing, the sales agreement is under review by the PCGG Senate Committee on Youth and Sports Development, to look into the
solely to determine the appropriate price. The sale of these companies and charges against the PCGG filed by three (3) stockholders of Oriental
our prior right to reacquire them have never been at issue.’ Petroleum, i.e., that it had adopted a "get-rich-quick scheme" for its
nominee-directors in a sequestered oil exploration firm. The pertinent
"Perhaps I could not make it any clearer to Mr. Lopa that I was not really portion of Senate Resolution No. 212 reads as follows:chanrob1es virtual
making baseless and malicious statements."cralaw virtua1aw library 1aw library

Senator Enrile concluded his privilege speech in the following x       x       x


tenor:jgc:chanrobles.com.ph

"Mr. President, it may be worthwhile for the Senate to look into the "WHEREAS, recent developments have shown that no less than the
possible violation of the law in the case particularly with regard to Republic Solicitor-General has stated that the PCGG Chairman and at least three
Act No. 3019, the Anti-Graft and Corrupt Practices Act, Section 5 of which Commissioners should resign and that the agency should rid itself of
reads as follows and I quote:chanrob1es virtual 1aw library ‘ineptness, incompetence and corruption’ and that the Sandiganbayan has
reportedly ordered the PCGG to answer charges filed by three stockholders
‘SECTION 5. Prohibition on certain relatives. — It shall be unlawful for the of Oriental Petroleum that it had adopted a ‘get-rich-quick scheme’ for its
spouse or for any relative, by consanguinity or affinity, within the third civil nominee-directors in a sequestered oil exploration firm;chanrobles virtual
degree, of the President of the Philippines, the Vice-President of the lawlibrary
Philippines, the President of the Senate, or the Speaker of the House of
Representatives, to intervene directly or indirectly, in any business, "WHEREAS, leaders of school youth, community groups and youth of non-
transaction, contract or application with the Government: Provided, that governmental organization had made representations to the Senate
this section shall not apply to any person who prior to the assumption of Committee on Youth and Sports Development to look into the charges
office of any of the above officials to whom he is related, has been already against PCGG since said agency is a symbol of the changes expected by the
dealing with the Government along the same line of business, nor to any people when the EDSA revolution took place and that the ill-gotten wealth
to be recovered will fund priority projects which will benefit our people such functions of congress. This was freely conceded by the Solicitor General in
as CARP, free education in the elementary and secondary levels, his argument in this case. Nor is the Congress a law enforcement or trial
reforestration, and employment generation for rural and urban workers; agency. These are functions of the executive and judicial departments of
government. No inquiry is an end in itself; it must be related to and in
"WHEREAS, the government and the present leadership must demonstrate furtherance of a legitimate task of Congress. Investigations conducted
in their public and private lives integrity, honor and efficient management solely for the personal aggrandizement of the investigators or to ‘punish’
of government services lest our youth become disillusioned and lose hope those investigated are indefensible." (Emphasis supplied)
and return to an ideology and form of government which is repugnant to
true freedom, democratic participation and human rights: Now, therefore, It can not be overlooked that when respondent Committee decided to
be it. conduct its investigation of the petitioners, the complaint in Civil Case No.
0035 had already been filed with the Sandiganbayan. A perusal of that
"Resolved by the Senate, That the activities of the Presidential Commission complaint shows that one of its principal causes of action against herein
on Good Government be investigated by the appropriate Committee in petitioners, as defendants therein, is the alleged sale of the 36 (or 39)
connection with the implementation of Section 26, Article XVIII of the corporations belonging to Benjamin "Kokoy" Romualdez. Since the issues in
Constitution." 19  said complaint had long been joined by the filing of petitioners’ respective
answers thereto, the issue sought to be investigated by the respondent
Thus, the inquiry under Senate Resolution No. 212 is to look into the Committee is one over which jurisdiction had been acquired by the
charges against the PCGG filed by the three (3) stockholders of Oriental Sandiganbayan. In short, the issue has been pre-empted by that court. To
Petroleum in connection with the implementation of Section 26, Article allow the respondent Committee to conduct its own investigation of an
XVIII of the Constitution. issue already before the Sandiganbayan would not only pose the possibility
of conflicting judgments between a legislative committee and a judicial
It cannot, therefore, be said that the contemplated inquiry on the subject tribunal, but if the Committee’s judgment were to be reached before that
of the privilege speech of Senator Juan Ponce Enrile, i.e., the alleged sale of the Sandiganbayan, the possibility of its influence being made to bear on
of the 36 (or 39) corporations belonging to Benjamin "Kokoy" Romualdez the ultimate judgment of the Sandiganbayan can not be discounted.
to the Lopa Group is to be conducted pursuant to Senate Resolution No.
212, because, firstly, Senator Enrile did not indict the PCGG, and, secondly, In fine, for the respondent Committee to probe and inquire into the same
neither Mr. Ricardo Lopa nor the herein petitioners are connected with the justiciable controversy already before the Sandiganbayan, would be an
government but are private citizens. encroachment into the exclusive domain of judicial jurisdiction that had
much earlier set in. In Baremblatt v. United States, 21 it was held
It appears, therefore, that the contemplated inquiry by respondent that:jgc:chanrobles.com.ph
Committee is not really "in aid of legislation" because it is not related to a
purpose within the jurisdiction of Congress, since the aim of the "Broad as it is, the power is not, however, without limitations. Since
investigation is to find out whether or not the relatives of the President or Congress may only investigate into those areas in which it may potentially
Mr. Ricardo Lopa had violated Section 5 of RA No. 3019, the "Anti-Graft legislate or appropriate, it cannot inquire into matters which are within the
and Corrupt Practices Act", a matter that appears more within the province exclusive province of one of the other branches of the government. Lacking
of the courts rather than of the legislature. Besides, the Court may take the judicial power given to the Judiciary, it cannot inquire into matters that
judicial notice that Mr. Ricardo Lopa died during the pendency of this case. are exclusively the concern of the Judiciary. Neither can it supplant the
In John T . Watkins v. United States, 20 it was held:jgc:chanrobles.com.ph Executive in what exclusively belongs to the Executive. . . . ."cralaw
virtua1aw library
". . . . The power of congress to conduct investigations is inherent in the
legislative process. That power is broad. It encompasses inquiries Now to another matter. It has been held that "a congressional committee’s
concerning the administration of existing laws as well as proposed or right to inquire is ‘subject to all relevant limitations placed by the
possibly needed statutes. It includes surveys of defects in our social, Constitution on governmental action,’ including ‘the relevant limitations of
economic, or political system for the purpose of enabling Congress to the Bill of Rights’." 22 
remedy them. It comprehends probes into departments of the Federal
Government to expose corruption, inefficiency or waste. But broad as is In another case —
this power of inquiry, it is not unlimited. There is no general authority to
expose the private affairs of individuals without justification in terms of the ". . . the mere semblance of legislative purpose would not justify an inquiry
in the face of the Bill of Rights. The critical element is the existence of, and produce evidence before it, it is only because we hold that the questioned
the weight to be ascribed to, the interest of the Congress in demanding inquiry is not in aid of legislation and, If pursued, would be violative of the
disclosures from an unwilling witness. We cannot simply assume, however, principle of separation of powers between the legislative and the judicial
that every congressional investigation is justified by a public need that departments of government, ordained by the Constitution.
over-balances any private rights affected. To do so would be to abdicate
the responsibility placed by the Constitution upon the judiciary to insure WHEREFORE, the petition is GRANTED. The Court holds that, under the
that the Congress does not unjustifiably encroach upon an individual’s right facts, including the circumstance that petitioners are presently impleaded
to privacy nor abridge his liberty of speech, press, religion or assembly." as defendants in a case before the Sandiganbayan, which involves issues
23  intimately related to the subject of contemplated inquiry before the
respondent Committee, the respondent Senate Blue Ribbon Committee is
One of the basic rights guaranteed by the Constitution to an individual is hereby enjoined from compelling the petitioners and intervenor to testify
the right against self-incrimination. 24 This right construed as the right to before it and produce evidence at the said inquiry.
remain completely silent may be availed of by the accused in a criminal
case; but it may be invoked by other witnesses only as questions are asked SO ORDERED.
of them. 
G.R. No. 115543 October 30, 1995
This distinction was enunciated by the Court in Romeo Chavez v. The
Honorable Court of Appeals, Et. Al. 25 thus —
RAUL S. ROCO and the INTEGRATED BAR OF THE
Petitioner, as accused, occupies a different tier of protection from an PHILIPPINES, Petitioners, v. THE SECRETARY OF THE DEPARTMENT
ordinary witness. Whereas an ordinary witness may be compelled to take OF FINANCE; THE COMMISSIONERS OF THE BUREAU OF INTERNAL
the witness stand and claim the privilege as each question requiring an REVENUE AND BUREAU OF CUSTOMS, Respondents.
incriminating answer is shot at him, an accused may altogether refuse to
take the witness stand and refuse to answer any and all questions."cralaw G.R. No. 115544 October 30, 1995
virtua1aw library

Moreover, this right of the accused is extended to respondents PHILIPPINE PRESS INSTITUTE, INC.; EGP PUBLISHING CO., INC.;
administrative investigations but only if they partake of the nature of a KAMAHALAN PUBLISHING CORPORATION; PHILIPPINE
criminal proceeding or analogous to a criminal proceeding. In Galman v. JOURNALISTS, INC.; JOSE L. PAVIA; and OFELIA L.
Pamaran, 26 the Court reiterated the doctrine in Cabal v. Kapunan (6 SCRA DIMALANTA, Petitioners, v. HON. LIWAYWAY V. CHATO, in her
1059) to illustrate the right of witnesses to invoke the right against self- capacity as Commissioner of Internal Revenue; HON. TEOFISTO T.
incrimination not only in criminal proceedings but also in all other types of GUINGONA, JR., in his capacity as Executive Secretary; and HON.
suit. ROBERTO B. DE OCAMPO, in his capacity as Secretary of
Finance, Respondents.
It was held that:jgc:chanrobles.com.ph
G.R. No. 115754 October 30, 1995
"We did not therein state that since he is not an accused and the case is
not a criminal case, Cabal cannot refuse to take the witness stand and
testify, and that he can invoke his right against self-incrimination only CHAMBER OF REAL ESTATE AND BUILDERS ASSOCIATIONS, INC.,
when a question which tends to elicit an answer that will incriminate him is (CREBA), Petitioner, v. THE COMMISSIONER OF INTERNAL
propounded to him. Clearly then, it is not the character of the suit involved REVENUE, Respondent.
but the nature of the proceedings that controls. The privilege has
consistently been held to extend to all proceedings sanctioned by law and G.R. No. 115781 October 30, 1995
to all cases in which punishment is sought to be visited upon a witness,
whether a party or not."cralaw virtua1aw library
KILOSBAYAN, INC., JOVITO R. SALONGA, CIRILO A. RIGOS, ERME
CAMBA, EMILIO C. CAPULONG, JR., JOSE T. APOLO, EPHRAIM
We do not here modify these doctrines. If we presently rule that petitioners
TENDERO, FERNANDO SANTIAGO, JOSE ABCEDE, CHRISTINE TAN,
may not be compelled by the respondent Committee to appear, testify and
FELIPE L. GOZON, RAFAEL G. FERNANDO, RAOUL V. VICTORINO, The Solicitor General, representing the respondents, filed a consolidated
JOSE CUNANAN, QUINTIN S. DOROMAL, MOVEMENT OF ATTORNEYS comment, to which the Philippine Airlines, Inc., petitioner in G.R. No.
FOR BROTHERHOOD, INTEGRITY AND NATIONALISM, INC. 115852, and the Philippine Press Institute, Inc., petitioner in G.R. No.
("MABINI"), FREEDOM FROM DEBT COALITION, INC., and 115544, and Juan T. David, petitioner in G.R. No. 115525, each filed a
PHILIPPINE BIBLE SOCIETY, INC. and WIGBERTO reply. In turn the Solicitor General filed on June 1, 1995 a rejoinder to the
TAÑADA, Petitioners, v. THE EXECUTIVE SECRETARY, THE PPI's reply.chanroblesvirtualawlibrarychanrobles virtual law library
SECRETARY OF FINANCE, THE COMMISSIONER OF INTERNAL
REVENUE and THE COMMISSIONER OF CUSTOMS, Respondents.
On June 27, 1995 the matter was submitted for
resolution.chanroblesvirtualawlibrarychanrobles virtual law library
G.R. No. 115852 October 30, 1995
I. Power of the Senate to propose amendments to revenue bills. Some of
PHILIPPINE AIRLINES, INC., Petitioner, v. THE SECRETARY OF the petitioners (Tolentino, Kilosbayan, Inc., Philippine Airlines (PAL), Roco,
FINANCE and COMMISSIONER OF INTERNAL REVENUE, Respondents. and Chamber of Real Estate and Builders Association (CREBA)) reiterate
previous claims made by them that R.A. No. 7716 did not "originate
exclusively" in the House of Representatives as required by Art. VI, �24 of
G.R. No. 115873 October 30, 1995
the Constitution. Although they admit that H. No. 11197 was filed in the
House of Representatives where it passed three readings and that
COOPERATIVE UNION OF THE PHILIPPINES, Petitioner, v. HON. afterward it was sent to the Senate where after first reading it was referred
LIWAYWAY V. CHATO, in her capacity as the Commissioner of to the Senate Ways and Means Committee, they complain that the Senate
Internal Revenue, HON. TEOFISTO T. GUINGONA, JR., in his did not pass it on second and third readings. Instead what the Senate did
capacity as Executive Secretary, and HON. ROBERTO B. DE was to pass its own version (S. No. 1630) which it approved on May 24,
OCAMPO, in his capacity as Secretary of Finance, Respondents. 1994. Petitioner Tolentino adds that what the Senate committee should
have done was to amend H. No. 11197 by striking out the text of the bill
G.R. No. 115931 October 30, 1995 and substituting it with the text of S. No. 1630. That way, it is said, "the
bill remains a House bill and the Senate version just becomes the text
(only the text) of the House bill."chanrobles virtual law library
PHILIPPINE EDUCATIONAL PUBLISHERS ASSOCIATION, INC. and
ASSOCIATION OF PHILIPPINE BOOK SELLERS, Petitioners, v. HON.
ROBERTO B. DE OCAMPO, as the Secretary of Finance; HON. The contention has no merit.chanroblesvirtualawlibrarychanrobles virtual
LIWAYWAY V. CHATO, as the Commissioner of Internal Revenue; law library
and HON. GUILLERMO PARAYNO, JR., in his capacity as the
Commissioner of Customs, Respondents. The enactment of S. No. 1630 is not the only instance in which the Senate
proposed an amendment to a House revenue bill by enacting its own
RESOLUTION version of a revenue bill. On at least two occasions during the Eighth
Congress, the Senate passed its own version of revenue bills, which, in
consolidation with House bills earlier passed, became the enrolled bills.
MENDOZA, J.: These were:chanrobles virtual law library

These are motions seeking reconsideration of our decision dismissing the R.A. No. 7369 (AN ACT TO AMEND THE OMNIBUS INVESTMENTS CODE OF
petitions filed in these cases for the declaration of unconstitutionality of 1987 BY EXTENDING FROM FIVE (5) YEARS TO TEN YEARS THE PERIOD
R.A. No. 7716, otherwise known as the Expanded Value-Added Tax Law. FOR TAX AND DUTY EXEMPTION AND TAX CREDIT ON CAPITAL
The motions, of which there are 10 in all, have been filed by the several EQUIPMENT) which was approved by the President on April 10, 1992. This
petitioners in these cases, with the exception of the Philippine Educational Act is actually a consolidation of H. No. 34254, which was approved by the
Publishers Association, Inc. and the Association of Philippine Booksellers, House on January 29, 1992, and S. No. 1920, which was approved by the
petitioners in G.R. No. 115931.chanroblesvirtualawlibrarychanrobles virtual Senate on February 3, 1992.chanroblesvirtualawlibrarychanrobles virtual
law library law library
R.A. No. 7549 (AN ACT GRANTING TAX EXEMPTIONS TO WHOEVER SHALL OF THE NATIONAL INTERNAL REVENUE CODE, AS AMENDED (February 24,
GIVE REWARD TO ANY FILIPINO ATHLETE WINNING A MEDAL IN OLYMPIC 1993)chanrobles virtual law library
GAMES) which was approved by the President on May 22, 1992. This Act is
a consolidation of H. No. 22232, which was approved by the House of
House Bill No. 1470, October 20, 1992chanrobles virtual law library
Representatives on August 2, 1989, and S. No. 807, which was approved
by the Senate on October 21, 1991.chanroblesvirtualawlibrarychanrobles
virtual law library Senate Bill No. 35, November 19, 1992chanrobles virtual law library

On the other hand, the Ninth Congress passed revenue laws which were 4. R.A. NO. 7649chanrobles virtual law library
also the result of the consolidation of House and Senate bills. These are the
following, with indications of the dates on which the laws were approved by AN ACT REQUIRING THE GOVERNMENT OR ANY OF ITS POLITICAL
the President and dates the separate bills of the two chambers of Congress SUBDIVISIONS, INSTRUMENTALITIES OR AGENCIES INCLUDING
were respectively passed: GOVERNMENT-OWNED OR CONTROLLED CORPORATIONS (GOCCS) TO
DEDUCT AND WITHHOLD THE VALUE-ADDED TAX DUE AT THE RATE OF
1. R.A. NO. 7642chanrobles virtual law library THREE PERCENT (3%) ON GROSS PAYMENT FOR THE PURCHASE OF
GOODS AND SIX PERCENT (6%) ON GROSS RECEIPTS FOR SERVICES
RENDERED BY CONTRACTORS (April 6, 1993)chanrobles virtual law library
AN ACT INCREASING THE PENALTIES FOR TAX EVASION, AMENDING FOR
THIS PURPOSE THE PERTINENT SECTIONS OF THE NATIONAL INTERNAL
REVENUE CODE (December 28, House Bill No. 5260, January 26, 1993chanrobles virtual law library
1992).chanroblesvirtualawlibrarychanrobles virtual law library
Senate Bill No. 1141, March 30, 1993chanrobles virtual law library
House Bill No. 2165, October 5, 1992chanrobles virtual law library
5. R.A. NO. 7656chanrobles virtual law library
Senate Bill No. 32, December 7, 1992chanrobles virtual law library
AN ACT REQUIRING GOVERNMENT-OWNED OR CONTROLLED
2. R.A. NO. 7643chanrobles virtual law library CORPORATIONS TO DECLARE DIVIDENDS UNDER CERTAIN CONDITIONS
TO THE NATIONAL GOVERNMENT, AND FOR OTHER PURPOSES (November
9, 1993)chanrobles virtual law library
AN ACT TO EMPOWER THE COMMISSIONER OF INTERNAL REVENUE TO
REQUIRE THE PAYMENT OF THE VALUE-ADDED TAX EVERY MONTH AND TO
ALLOW LOCAL GOVERNMENT UNITS TO SHARE IN VAT REVENUE, House Bill No. 11024, November 3, 1993chanrobles virtual law library
AMENDING FOR THIS PURPOSE CERTAIN SECTIONS OF THE NATIONAL
INTERNAL REVENUE CODE (December 28, 1992)chanrobles virtual law Senate Bill No. 1168, November 3, 1993chanrobles virtual law library
library

6. R.A. NO. 7660chanrobles virtual law library


House Bill No. 1503, September 3, 1992chanrobles virtual law library

AN ACT RATIONALIZING FURTHER THE STRUCTURE AND ADMINISTRATION


Senate Bill No. 968, December 7, 1992chanrobles virtual law library OF THE DOCUMENTARY STAMP TAX, AMENDING FOR THE PURPOSE
CERTAIN PROVISIONS OF THE NATIONAL INTERNAL REVENUE CODE, AS
3. R.A. NO. 7646chanrobles virtual law library AMENDED, ALLOCATING FUNDS FOR SPECIFIC PROGRAMS, AND FOR
OTHER PURPOSES (December 23, 1993)chanrobles virtual law library
AN ACT AUTHORIZING THE COMMISSIONER OF INTERNAL REVENUE TO
PRESCRIBE THE PLACE FOR PAYMENT OF INTERNAL REVENUE TAXES BY House Bill No. 7789, May 31, 1993chanrobles virtual law library
LARGE TAXPAYERS, AMENDING FOR THIS PURPOSE CERTAIN PROVISIONS
Senate Bill No. 1330, November 18, 1993chanrobles virtual law library No amendment by substitution shall be entertained unless the text thereof
is submitted in writing.chanroblesvirtualawlibrarychanrobles virtual law
library
7. R.A. NO. 7717chanrobles virtual law library

Any of said amendments may be withdrawn before a vote is taken


AN ACT IMPOSING A TAX ON THE SALE, BARTER OR EXCHANGE OF
thereon.chanroblesvirtualawlibrarychanrobles virtual law library
SHARES OF STOCK LISTED AND TRADED THROUGH THE LOCAL STOCK
EXCHANGE OR THROUGH INITIAL PUBLIC OFFERING, AMENDING FOR THE
PURPOSE THE NATIONAL INTERNAL REVENUE CODE, AS AMENDED, BY �69. No amendment which seeks the inclusion of a legislative provision
INSERTING A NEW SECTION AND REPEALING CERTAIN SUBSECTIONS foreign to the subject matter of a bill (rider) shall be entertained.
THEREOF (May 5, 1994)chanrobles virtual law library
xxx xxx xxxchanrobles virtual law library
House Bill No. 9187, November 3, 1993chanrobles virtual law library
�70-A. A bill or resolution shall not be amended by substituting it with
Senate Bill No. 1127, March 23, 1994 another which covers a subject distinct from that proposed in the original
bill or resolution. (emphasis added).
Thus, the enactment of S. No. 1630 is not the only instance in which the
Senate, in the exercise of its power to propose amendments to bills Nor is there merit in petitioners' contention that, with regard to revenue
required to originate in the House, passed its own version of a House bills, the Philippine Senate possesses less power than the U.S. Senate
revenue measure. It is noteworthy that, in the particular case of S. No. because of textual differences between constitutional provisions giving
1630, petitioners Tolentino and Roco, as members of the Senate, voted to them the power to propose or concur with
approve it on second and third amendments.chanroblesvirtualawlibrarychanrobles virtual law library
readings.chanroblesvirtualawlibrarychanrobles virtual law library
Art. I, �7, cl. 1 of the U.S. Constitution reads:
On the other hand, amendment by substitution, in the manner urged by
petitioner Tolentino, concerns a mere matter of form. Petitioner has not
All Bills for raising Revenue shall originate in the House of Representatives;
shown what substantial difference it would make if, as the Senate actually
but the Senate may propose or concur with amendments as on other Bills.
did in this case, a separate bill like S. No. 1630 is instead enacted as a
substitute measure, "taking into Consideration . . . H.B. 11197."chanrobles
virtual law library Art. VI, �24 of our Constitution reads:

Indeed, so far as pertinent, the Rules of the Senate only provide: All appropriation, revenue or tariff bills, bills authorizing increase of the
public debt, bills of local application, and private bills shall originate
exclusively in the House of Representatives, but the Senate may propose
RULE XXIX
or concur with amendments.

AMENDMENTS
The addition of the word "exclusively" in the Philippine Constitution and the
decision to drop the phrase "as on other Bills" in the American version,
xxx xxx xxxchanrobles virtual law library according to petitioners, shows the intention of the framers of our
Constitution to restrict the Senate's power to propose amendments to
revenue bills. Petitioner Tolentino contends that the word "exclusively" was
�68. Not more than one amendment to the original amendment shall be
inserted to modify "originate" and "the words 'as in any other bills' (sic)
considered.
were eliminated so as to show that these bills were not to be like other bills
but must be treated as a special kind."chanrobles virtual law library
The history of this provision does not support this contention. The coequality of the two chambers of
supposed indicia of constitutional intent are nothing but the relics of an Congress.chanroblesvirtualawlibrarychanrobles virtual law library
unsuccessful attempt to limit the power of the Senate. It will be recalled
that the 1935 Constitution originally provided for a unicameral National
That this is also the understanding of book authors of the scope of the
Assembly. When it was decided in 1939 to change to a bicameral
Senate's power to concur is clear from the following commentaries:
legislature, it became necessary to provide for the procedure for lawmaking
by the Senate and the House of Representatives. The work of proposing
amendments to the Constitution was done by the National Assembly, The power of the Senate to propose or concur with amendments is
acting as a constituent assembly, some of whose members, jealous of apparently without restriction. It would seem that by virtue of this power,
preserving the Assembly's lawmaking powers, sought to curtail the powers the Senate can practically re-write a bill required to come from the House
of the proposed Senate. Accordingly they proposed the following provision: and leave only a trace of the original bill. For example, a general revenue
bill passed by the lower house of the United States Congress contained
provisions for the imposition of an inheritance tax . This was changed by
All bills appropriating public funds, revenue or tariff bills, bills of local
the Senate into a corporation tax. The amending authority of the Senate
application, and private bills shall originate exclusively in the Assembly, but
was declared by the United States Supreme Court to be sufficiently broad
the Senate may propose or concur with amendments. In case of
to enable it to make the alteration. [Flint v. Stone Tracy Company, 220
disapproval by the Senate of any such bills, the Assembly may repass the
U.S. 107, 55 L. ed. 389].chanroblesvirtualawlibrarychanrobles virtual law
same by a two-thirds vote of all its members, and thereupon, the bill so
library
repassed shall be deemed enacted and may be submitted to the President
for corresponding action. In the event that the Senate should fail to finally
act on any such bills, the Assembly may, after thirty days from the opening (L. TAÑADA AND F. CARREON, POLITICAL LAW OF THE PHILIPPINES 247
of the next regular session of the same legislative term, reapprove the (1961))chanrobles virtual law library
same with a vote of two-thirds of all the members of the Assembly. And
upon such reapproval, the bill shall be deemed enacted and may be The above-mentioned bills are supposed to be initiated by the House of
submitted to the President for corresponding action. Representatives because it is more numerous in membership and therefore
also more representative of the people. Moreover, its members are
The special committee on the revision of laws of the Second National presumed to be more familiar with the needs of the country in regard to
Assembly vetoed the proposal. It deleted everything after the first the enactment of the legislation
sentence. As rewritten, the proposal was approved by the National involved.chanroblesvirtualawlibrarychanrobles virtual law library
Assembly and embodied in Resolution No. 38, as amended by Resolution
No. 73. (J. ARUEGO, KNOW YOUR CONSTITUTION 65-66 (1950)). The The Senate is, however, allowed much leeway in the exercise of its power
proposed amendment was submitted to the people and ratified by them in to propose or concur with amendments to the bills initiated by the House of
the elections held on June 18, 1940.chanroblesvirtualawlibrarychanrobles Representatives. Thus, in one case, a bill introduced in the U.S. House of
virtual law library Representatives was changed by the Senate to make a proposed
inheritance tax a corporation tax. It is also accepted practice for the Senate
This is the history of Art. VI, �18 (2) of the 1935 Constitution, from which to introduce what is known as an amendment by substitution, which may
Art. VI, �24 of the present Constitution was derived. It explains why the entirely replace the bill initiated in the House of
word "exclusively" was added to the American text from which the framers Representatives.chanroblesvirtualawlibrarychanrobles virtual law library
of the Philippine Constitution borrowed and why the phrase "as on other
Bills" was not copied. Considering the defeat of the proposal, the power of (I. CRUZ, PHILIPPINE POLITICAL LAW 144-145 (1993)).
the Senate to propose amendments must be understood to be full, plenary
and complete "as on other Bills." Thus, because revenue bills are required
to originate exclusively in the House of Representatives, the Senate cannot In sum, while Art. VI, �24 provides that all appropriation, revenue or tariff
enact revenue measures of its own without such bills. After a revenue bill is bills, bills authorizing increase of the public debt, bills of local application,
passed and sent over to it by the House, however, the Senate certainly can and private bills must "originate exclusively in the House of
pass its own version on the same subject matter. This follows from the Representatives," it also adds, "but the Senate may propose or concur with
amendments." In the exercise of this power, the Senate may propose an
entirely new bill as a substitute measure. As petitioner Tolentino states in a three readings. It was enough that after it was passed on first reading it
high school text, a committee to which a bill is referred may do any of the was referred to the Senate Committee on Ways and Means. Neither was it
following: required that S. No. 1630 be passed by the House of Representatives
before the two bills could be referred to the Conference
Committee.chanroblesvirtualawlibrarychanrobles virtual law library
(1) to endorse the bill without changes; (2) to make changes in the bill
omitting or adding sections or altering its language; (3) to make and
endorse an entirely new bill as a substitute, in which case it will be known There is legislative precedent for what was done in the case of H. No.
as a committee bill; or (4) to make no report at 11197 and S. No. 1630. When the House bill and Senate bill, which became
all.chanroblesvirtualawlibrarychanrobles virtual law library R.A. No. 1405 (Act prohibiting the disclosure of bank deposits), were
referred to a conference committee, the question was raised whether the
two bills could be the subject of such conference, considering that the bill
(A. TOLENTINO, THE GOVERNMENT OF THE PHILIPPINES 258 (1950))
from one house had not been passed by the other and vice versa. As
Congressman Duran put the question:
To except from this procedure the amendment of bills which are required to
originate in the House by prescribing that the number of the House bill and
MR. DURAN. Therefore, I raise this question of order as to procedure: If a
its other parts up to the enacting clause must be preserved although the
House bill is passed by the House but not passed by the Senate, and a
text of the Senate amendment may be incorporated in place of the original
Senate bill of a similar nature is passed in the Senate but never passed in
body of the bill is to insist on a mere technicality. At any rate there is no
the House, can the two bills be the subject of a conference, and can a law
rule prescribing this form. S. No. 1630, as a substitute measure, is
be enacted from these two bills? I understand that the Senate bill in this
therefore as much an amendment of H. No. 11197 as any which the Senate
particular instance does not refer to investments in government securities,
could have made.chanroblesvirtualawlibrarychanrobles virtual law library
whereas the bill in the House, which was introduced by the Speaker, covers
two subject matters: not only investigation of deposits in banks but also
II. S. No. 1630 a mere amendment of H. No. 11197. Petitioners' basic error investigation of investments in government securities. Now, since the two
is that they assume that S. No. 1630 is an independent and distinct bill. bills differ in their subject matter, I believe that no law can be enacted.
Hence their repeated references to its certification that it was passed by
the Senate "in substitution of S.B. No. 1129, taking into considerationP.S.
Ruling on the point of order raised, the chair (Speaker Jose B. Laurel, Jr.)
Res. No. 734 and H.B. No. 11197," implying that there is something
said:
substantially different between the reference to S. No. 1129 and the
reference to H. No. 11197. From this premise, they conclude that R.A. No.
7716 originated both in the House and in the Senate and that it is the THE SPEAKER. The report of the conference committee is in order. It is
product of two "half-baked bills because neither H. No. 11197 nor S. No. precisely in cases like this where a conference should be had. If the House
1630 was passed by both houses of Congress."chanrobles virtual law bill had been approved by the Senate, there would have been no need of a
library conference; but precisely because the Senate passed another bill on the
same subject matter, the conference committee had to be created, and we
are now considering the report of that
In point of fact, in several instances the provisions of S. No. 1630, clearly
committee.chanroblesvirtualawlibrarychanrobles virtual law library
appear to be mere amendments of the corresponding provisions of H. No.
11197. The very tabular comparison of the provisions of H. No. 11197 and
S. No. 1630 attached as Supplement A to the basic petition of petitioner (2 CONG. REC. NO. 13, July 27, 1955, pp. 3841-42 (emphasis added))
Tolentino, while showing differences between the two bills, at the same
time indicates that the provisions of the Senate bill were precisely intended
III. The President's certification. The fallacy in thinking that H. No. 11197
to be amendments to the House bill.chanroblesvirtualawlibrarychanrobles
and S. No. 1630 are distinct and unrelated measures also accounts for the
virtual law library
petitioners' (Kilosbayan's and PAL's) contention that because the President
separately certified to the need for the immediate enactment of these
Without H. No. 11197, the Senate could not have enacted S. No. 1630. measures, his certification was ineffectual and void. The certification had to
Because the Senate bill was a mere amendment of the House bill, H. No. be made of the version of the same revenue bill which at the moment was
11197 in its original form did not have to pass the Senate on second and being considered. Otherwise, to follow petitioners' theory, it would be
necessary for the President to certify as many bills as are presented in a (2) No bill passed by either House shall become a law unless it has passed
house of Congress even though the bills are merely versions of the bill he three readings on separate days, and printed copies thereof in its final form
has already certified. It is enough that he certifies the bill which, at the have been distributed to its Members three days before its passage, except
time he makes the certification, is under consideration. Since on March 22, when the President certifies to the necessity of its immediate enactment to
1994 the Senate was considering S. No. 1630, it was that bill which had to meet a public calamity or emergency. Upon the last reading of a bill, no
be certified. For that matter on June 1, 1993 the President had earlier amendment thereto shall be allowed, and the vote thereon shall be taken
certified H. No. 9210 for immediate enactment because it was the one immediately thereafter, and the yeas and nays entered in the Journal.
which at that time was being considered by the House. This bill was later
substituted, together with other bills, by H. No.
The exception is based on the prudential consideration that if in all cases
11197.chanroblesvirtualawlibrarychanrobles virtual law library
three readings on separate days are required and a bill has to be printed in
final form before it can be passed, the need for a law may be rendered
As to what Presidential certification can accomplish, we have already academic by the occurrence of the very emergency or public calamity
explained in the main decision that the phrase "except when the President which it is meant to address.chanroblesvirtualawlibrarychanrobles virtual
certifies to the necessity of its immediate enactment, etc." in Art. VI, �26 law library
(2) qualifies not only the requirement that "printed copies [of a bill] in its
final form [must be] distributed to the members three days before its
Petitioners further contend that a "growing budget deficit" is not an
passage" but also the requirement that before a bill can become a law it
emergency, especially in a country like the Philippines where budget deficit
must have passed "three readings on separate days." There is not only
is a chronic condition. Even if this were the case, an enormous budget
textual support for such construction but historical basis as
deficit does not make the need for R.A. No. 7716 any less urgent or the
well.chanroblesvirtualawlibrarychanrobles virtual law library
situation calling for its enactment any less an
emergency.chanroblesvirtualawlibrarychanrobles virtual law library
Art. VI, �21 (2) of the 1935 Constitution originally provided:
Apparently, the members of the Senate (including some of the petitioners
(2) No bill shall be passed by either House unless it shall have been printed in these cases) believed that there was an urgent need for consideration of
and copies thereof in its final form furnished its Members at least three S. No. 1630, because they responded to the call of the President by voting
calendar days prior to its passage, except when the President shall have on the bill on second and third readings on the same day. While the judicial
certified to the necessity of its immediate enactment. Upon the last reading department is not bound by the Senate's acceptance of the President's
of a bill, no amendment thereof shall be allowed and the question upon its certification, the respect due coequal departments of the government in
passage shall be taken immediately thereafter, and matters committed to them by the Constitution and the absence of a clear
the yeas and naysentered on the Journal. showing of grave abuse of discretion caution a stay of the judicial
hand.chanroblesvirtualawlibrarychanrobles virtual law library
When the 1973 Constitution was adopted, it was provided in Art. VIII, �19
(2): At any rate, we are satisfied that S. No. 1630 received thorough
consideration in the Senate where it was discussed for six days. Only its
distribution in advance in its final printed form was actually dispensed with
(2) No bill shall become a law unless it has passed three readings on
by holding the voting on second and third readings on the same day
separate days, and printed copies thereof in its final form have been
(March 24, 1994). Otherwise, sufficient time between the submission of the
distributed to the Members three days before its passage, except when the
bill on February 8, 1994 on second reading and its approval on March 24,
Prime Minister certifies to the necessity of its immediate enactment to meet
1994 elapsed before it was finally voted on by the Senate on third
a public calamity or emergency. Upon the last reading of a bill, no
reading.chanroblesvirtualawlibrarychanrobles virtual law library
amendment thereto shall be allowed, and the vote thereon shall be taken
immediately thereafter, and the yeas and nays entered in the Journal.
The purpose for which three readings on separate days is required is said
to be two-fold: (1) to inform the members of Congress of what they must
This provision of the 1973 document, with slight modification, was adopted
vote on and (2) to give them notice that a measure is progressing through
in Art. VI, �26 (2) of the present Constitution, thus:
the enacting process, thus enabling them and others interested in the
measure to prepare their positions with reference to it. (1 J. G. MR. BENGZON. My point of order is that it is out of order to consider the
SUTHERLAND, STATUTES AND STATUTORY CONSTRUCTION �10.04, p. report of the conference committee regarding House Bill No. 2557 by
282 (1972)). These purposes were substantially achieved in the case of reason of the provision of Section 11, Article XII, of the Rules of this House
R.A. No. 7716.chanroblesvirtualawlibrarychanrobles virtual law library which provides specifically that the conference report must be accompanied
by a detailed statement of the effects of the amendment on the bill of the
House. This conference committee report is not accompanied by that
IV. Power of Conference Committee. It is contended (principally by
detailed statement, Mr. Speaker. Therefore it is out of order to consider it.
Kilosbayan, Inc. and the Movement of Attorneys for Brotherhood, Integrity
and Nationalism, Inc. (MABINI)) that in violation of the constitutional policy
of full public disclosure and the people's right to know (Art. II, �28 and Petitioner Tolentino, then the Majority Floor Leader, answered:
Art. III, �7) the Conference Committee met for two days in executive
session with only the conferees
MR. TOLENTINO. Mr. Speaker, I should just like to say a few words in
present.chanroblesvirtualawlibrarychanrobles virtual law library
connection with the point of order raised by the gentleman from
Pangasinan.chanroblesvirtualawlibrarychanrobles virtual law library
As pointed out in our main decision, even in the United States it was
customary to hold such sessions with only the conferees and their staffs in
There is no question about the provision of the Rule cited by the gentleman
attendance and it was only in 1975 when a new rule was adopted requiring
from Pangasinan, but this provision applies to those cases where only
open sessions. Unlike its American counterpart, the Philippine Congress has
portions of the bill have been amended. In this case before us an entire bill
not adopted a rule prescribing open hearings for conference
is presented; therefore, it can be easily seen from the reading of the bill
committees.chanroblesvirtualawlibrarychanrobles virtual law library
what the provisions are. Besides, this procedure has been an established
practice.
It is nevertheless claimed that in the United States, before the adoption of
the rule in 1975, at least staff members were present. These were staff
After some interruption, he continued:
members of the Senators and Congressmen, however, who may be
presumed to be their confidential men, not stenographers as in this case
who on the last two days of the conference were excluded. There is no MR. TOLENTINO. As I was saying, Mr. Speaker, we have to look into the
showing that the conferees themselves did not take notes of their reason for the provisions of the Rules, and the reason for the requirement
proceedings so as to give petitioner Kilosbayan basis for claiming that even in the provision cited by the gentleman from Pangasinan is when there are
in secret diplomatic negotiations involving state interests, conferees keep only certain words or phrases inserted in or deleted from the provisions of
notes of their meetings. Above all, the public's right to know was fully the bill included in the conference report, and we cannot understand what
served because the Conference Committee in this case submitted a report those words and phrases mean and their relation to the bill. In that case, it
showing the changes made on the differing versions of the House and the is necessary to make a detailed statement on how those words and
Senate.chanroblesvirtualawlibrarychanrobles virtual law library phrases will affect the bill as a whole; but when the entire bill itself is
copied verbatim in the conference report, that is not necessary. So when
the reason for the Rule does not exist, the Rule does not
Petitioners cite the rules of both houses which provide that conference
exist.chanroblesvirtualawlibrarychanrobles virtual law library
committee reports must contain "a detailed, sufficiently explicit statement
of the changes in or other amendments." These changes are shown in the
bill attached to the Conference Committee Report. The members of both (2 CONG. REC. NO. 2, p. 4056. (emphasis added))
houses could thus ascertain what changes had been made in the original
bills without the need of a statement detailing the Congressman Tolentino was sustained by the chair. The record shows that
changes.chanroblesvirtualawlibrarychanrobles virtual law library when the ruling was appealed, it was upheld by viva voce and when a
division of the House was called, it was sustained by a vote of 48 to 5.
The same question now presented was raised when the bill which became (Id., 
R.A. No. 1400 (Land Reform Act of 1955) was reported by the Conference p. 4058)chanrobles virtual law library
Committee. Congressman Bengzon raised a point of order. He said:
Nor is there any doubt about the power of a conference committee to insert (R. Jackson, Committees in the Philippine Congress, in COMMITTEES AND
new provisions as long as these are germane to the subject of the LEGISLATURES: A COMPARATIVE ANALYSIS 163 (J. D. LEES AND M.
conference. As this Court held in Philippine Judges Association v. Prado, SHAW, eds.)).
227 SCRA 703 (1993), in an opinion written by then Justice Cruz, the
jurisdiction of the conference committee is not limited to resolving
In citing this study, we pass no judgment on the methods of conference
differences between the Senate and the House. It may propose an entirely
committees. We cite it only to say that conference committees here are no
new provision. What is important is that its report is subsequently
different from their counterparts in the United States whose vast powers
approved by the respective houses of Congress. This Court ruled that it
we noted in Philippine Judges Association v. Prado, supra. At all events,
would not entertain allegations that, because new provisions had been
under Art. VI, �16(3) each house has the power "to determine the rules of
added by the conference committee, there was thereby a violation of the
its proceedings," including those of its committees. Any meaningful change
constitutional injunction that "upon the last reading of a bill, no
in the method and procedures of Congress or its committees must
amendment thereto shall be allowed."
therefore be sought in that body itself.chanroblesvirtualawlibrarychanrobles
virtual law library
Applying these principles, we shall decline to look into the petitioners'
charges that an amendment was made upon the last reading of the bill that
V. The titles of S. No. 1630 and H. No. 11197. PAL maintains that R.A. No.
eventually became R.A. No. 7354 and that copiesthereof in its final
7716 violates Art. VI, �26 (1) of the Constitution which provides that
form were not distributed among the members of each House. Both the
"Every bill passed by Congress shall embrace only one subject which shall
enrolled bill and the legislative journals certify that the measure was duly
be expressed in the title thereof." PAL contends that the amendment of its
enacted i.e., in accordance with Article VI, Sec. 26 (2) of the Constitution.
franchise by the withdrawal of its exemption from the VAT is not expressed
We are bound by such official assurances from a coordinate department of
in the title of the law.chanroblesvirtualawlibrarychanrobles virtual law
the government, to which we owe, at the very least, a becoming
library
courtesy.chanroblesvirtualawlibrarychanrobles virtual law library

Pursuant to �13 of P.D. No. 1590, PAL pays a franchise tax of 2% on its
(Id. at 710. (emphasis added))
gross revenue "in lieu of all other taxes, duties, royalties, registration,
license and other fees and charges of any kind, nature, or description,
It is interesting to note the following description of conference committees imposed, levied, established, assessed or collected by any municipal, city,
in the Philippines in a 1979 study: provincial or national authority or government agency, now or in the
future."chanrobles virtual law library
Conference committees may be of two types: free or instructed. These
committees may be given instructions by their parent bodies or they may PAL was exempted from the payment of the VAT along with other entities
be left without instructions. Normally the conference committees are by �103 of the National Internal Revenue Code, which provides as follows:
without instructions, and this is why they are often critically referred to as
"the little legislatures." Once bills have been sent to them, the conferees
�103. Exempt transactions. - The following shall be exempt from the
have almost unlimited authority to change the clauses of the bills and in
value-added tax:
fact sometimes introduce new measures that were not in the original
legislation. No minutes are kept, and members' activities on conference
committees are difficult to determine. One congressman known for his xxx xxx xxxchanrobles virtual law library
idealism put it this way: "I killed a bill on export incentives for my interest
group [copra] in the conference committee but I could not have done so (q) Transactions which are exempt under special laws or international
anywhere else." The conference committee submits a report to both agreements to which the Philippines is a signatory.
houses, and usually it is accepted. If the report is not accepted, then the
committee is discharged and new members are
appointed.chanroblesvirtualawlibrarychanrobles virtual law library R.A. No. 7716 seeks to withdraw certain exemptions, including that
granted to PAL, by amending �103, as follows:
�103. Exempt transactions. - The following shall be exempt from the THEREWITH. It contained a provision repealing all franking privileges. It
value-added tax: was contended that the withdrawal of franking privileges was not
expressed in the title of the law. In holding that there was sufficient
description of the subject of the law in its title, including the repeal of
xxx xxx xxxchanrobles virtual law library
franking privileges, this Court held:

(q) Transactions which are exempt under special laws, except those
To require every end and means necessary for the accomplishment of the
granted under Presidential Decree Nos. 66, 529, 972, 1491, 1590. . . .
general objectives of the statute to be expressed in its title would not only
be unreasonable but would actually render legislation impossible. [Cooley,
The amendment of �103 is expressed in the title of R.A. No. 7716 which Constitutional Limitations, 8th Ed., p. 297] As has been correctly
reads: explained:

AN ACT RESTRUCTURING THE VALUE-ADDED TAX (VAT) SYSTEM, The details of a legislative act need not be specifically stated in its title, but
WIDENING ITS TAX BASE AND ENHANCING ITS ADMINISTRATION, AND matter germane to the subject as expressed in the title, and adopted to the
FOR THESE PURPOSES AMENDING AND REPEALING THE RELEVANT accomplishment of the object in view, may properly be included in the act.
PROVISIONS OF THE NATIONAL INTERNAL REVENUE CODE, AS AMENDED, Thus, it is proper to create in the same act the machinery by which the act
AND FOR OTHER PURPOSES. is to be enforced, to prescribe the penalties for its infraction, and to
remove obstacles in the way of its execution. If such matters are properly
By stating that R.A. No. 7716 seeks to "[RESTRUCTURE] THE VALUE- connected with the subject as expressed in the title, it is unnecessary that
ADDED TAX (VAT) SYSTEM [BY] WIDENING ITS TAX BASE AND they should also have special mention in the title. (Southern Pac. Co. v.
ENHANCING ITS ADMINISTRATION, AND FOR THESE PURPOSES Bartine, 170 Fed. 725)
AMENDING AND REPEALING THE RELEVANT PROVISIONS OF THE
NATIONAL INTERNAL REVENUE CODE, AS AMENDED AND FOR OTHER (227 SCRA at 707-708)
PURPOSES," Congress thereby clearly expresses its intention to amend any
provision of the NIRC which stands in the way of accomplishing the
VI. Claims of press freedom and religious liberty. We have held that, as a
purpose of the law.chanroblesvirtualawlibrarychanrobles virtual law library
general proposition, the press is not exempt from the taxing power of the
State and that what the constitutional guarantee of free press prohibits are
PAL asserts that the amendment of its franchise must be reflected in the laws which single out the press or target a group belonging to the press for
title of the law by specific reference to P.D. No. 1590. It is unnecessary to special treatment or which in any way discriminate against the press on the
do this in order to comply with the constitutional requirement, since it is basis of the content of the publication, and R.A. No. 7716 is none of
already stated in the title that the law seeks to amend the pertinent these.chanroblesvirtualawlibrarychanrobles virtual law library
provisions of the NIRC, among which is �103(q), in order to widen the
base of the VAT. Actually, it is the bill which becomes a law that is required
Now it is contended by the PPI that by removing the exemption of the
to express in its title the subject of legislation. The titles of H. No. 11197
press from the VAT while maintaining those granted to others, the law
and S. No. 1630 in fact specifically referred to �103 of the NIRC as among
discriminates against the press. At any rate, it is averred, "even
the provisions sought to be amended. We are satisfied that sufficient notice
nondiscriminatory taxation of constitutionally guaranteed freedom is
had been given of the pendency of these bills in Congress before they were
unconstitutional."chanrobles virtual law library
enacted into what is now R.A.
No. 7716.chanroblesvirtualawlibrarychanrobles virtual law library
With respect to the first contention, it would suffice to say that since the
law granted the press a privilege, the law could take back the privilege
In Philippine Judges Association v. Prado, supra, a similar argument as that
anytime without offense to the Constitution. The reason is simple: by
now made by PAL was rejected. R.A. No. 7354 is entitled AN ACT
granting exemptions, the State does not forever waive the exercise of its
CREATING THE PHILIPPINE POSTAL CORPORATION, DEFINING ITS
sovereign prerogative.chanroblesvirtualawlibrarychanrobles virtual law
POWERS, FUNCTIONS AND RESPONSIBILITIES, PROVIDING FOR
library
REGULATION OF THE INDUSTRY AND FOR OTHER PURPOSES CONNECTED
Indeed, in withdrawing the exemption, the law merely subjects the press to (a) Goods for consumption or use which are in their original state
the same tax burden to which other businesses have long ago been (agricultural, marine and forest products, cotton seeds in their original
subject. It is thus different from the tax involved in the cases invoked by state, fertilizers, seeds, seedlings, fingerlings, fish, prawn livestock and
the PPI. The license tax in Grosjean v. American Press Co., 297 U.S. 233, poultry feeds) and goods or services to enhance agriculture (milling of
80 L. Ed. 660 (1936) was found to be discriminatory because it was laid on palay, corn, sugar cane and raw sugar, livestock, poultry feeds, fertilizer,
the gross advertising receipts only of newspapers whose weekly circulation ingredients used for the manufacture of
was over 20,000, with the result that the tax applied only to 13 out of 124 feeds).chanroblesvirtualawlibrarychanrobles virtual law library
publishers in Louisiana. These large papers were critical of Senator Huey
Long who controlled the state legislature which enacted the license tax.
(b) Goods used for personal consumption or use (household and personal
The censorial motivation for the law was thus
effects of citizens returning to the Philippines) or for professional use, like
evident.chanroblesvirtualawlibrarychanrobles virtual law library
professional instruments and implements, by persons coming to the
Philippines to settle here.chanroblesvirtualawlibrarychanrobles virtual law
On the other hand, in Minneapolis Star & Tribune Co. v. Minnesota Comm'r library
of Revenue, 460 U.S. 575, 75 L. Ed. 2d 295 (1983), the tax was found to
be discriminatory because although it could have been made liable for the
(c) Goods subject to excise tax such as petroleum products or to be used
sales tax or, in lieu thereof, for the use tax on the privilege of using,
for manufacture of petroleum products subject to excise tax and services
storing or consuming tangible goods, the press was not. Instead, the press
subject to percentage tax.chanroblesvirtualawlibrarychanrobles virtual law
was exempted from both taxes. It was, however, later made to pay
library
a special use tax on the cost of paper and ink which made these items "the
only items subject to the use tax that were component of goods to be sold
at retail." The U.S. Supreme Court held that the differential treatment of (d) Educational services, medical, dental, hospital and veterinary services,
the press "suggests that the goal of regulation is not related to suppression and services rendered under employer-employee
of expression, and such goal is presumptively unconstitutional." It would relationship.chanroblesvirtualawlibrarychanrobles virtual law library
therefore appear that even a law that favors the press is constitutionally
suspect. (See the dissent of Rehnquist, J. in that case)chanrobles virtual (e) Works of art and similar creations sold by the artist
law library himself.chanroblesvirtualawlibrarychanrobles virtual law library

Nor is it true that only two exemptions previously granted by E.O. No. 273 (f) Transactions exempted under special laws, or international
are withdrawn "absolutely and unqualifiedly" by R.A. No. 7716. Other agreements.chanroblesvirtualawlibrarychanrobles virtual law library
exemptions from the VAT, such as those previously granted to PAL,
petroleum concessionaires, enterprises registered with the Export
Processing Zone Authority, and many more are likewise totally withdrawn, (g) Export-sales by persons not VAT-
in addition to exemptions which are partially withdrawn, in an effort to registered.chanroblesvirtualawlibrarychanrobles virtual law library
broaden the base of the tax.chanroblesvirtualawlibrarychanrobles virtual
law library (h) Goods or services with gross annual sale or receipt not
exceeding P500,000.00.chanroblesvirtualawlibrarychanrobles virtual law
The PPI says that the discriminatory treatment of the press is highlighted library
by the fact that transactions, which are profit oriented, continue to enjoy
exemption under R.A. No. 7716. An enumeration of some of these (Respondents' Consolidated Comment on the Motions for Reconsideration,
transactions will suffice to show that by and large this is not so and that pp. 58-60)
the exemptions are granted for a purpose. As the Solicitor General says,
such exemptions are granted, in some cases, to encourage agricultural
production and, in other cases, for the personal benefit of the end-user The PPI asserts that it does not really matter that the law does not
rather than for profit. The exempt transactions are: discriminate against the press because "even nondiscriminatory taxation on
constitutionally guaranteed freedom is unconstitutional." PPI cites in
support of this assertion the following statement in Murdock
v. Pennsylvania, 319 U.S. 105, 87 L. Ed. 1292 (1943):
The fact that the ordinance is "nondiscriminatory" is immaterial. The On the other hand the registration fee of P1,000.00 imposed by �107 of
protection afforded by the First Amendment is not so restricted. A license the NIRC, as amended by �7 of R.A. No. 7716, although fixed in amount,
tax certainly does not acquire constitutional validity because it classifies is really just to pay for the expenses of registration and enforcement of
the privileges protected by the First Amendment along with the wares and provisions such as those relating to accounting in �108 of the NIRC. That
merchandise of hucksters and peddlers and treats them all alike. Such the PBS distributes free bibles and therefore is not liable to pay the VAT
equality in treatment does not save the ordinance. Freedom of press, does not excuse it from the payment of this fee because it also sells some
freedom of speech, freedom of religion are in preferred position. copies. At any rate whether the PBS is liable for the VAT must be decided
in concrete cases, in the event it is assessed this tax by the Commissioner
of Internal Revenue.chanroblesvirtualawlibrarychanrobles virtual law library
The Court was speaking in that case of a license tax, which, unlike an
ordinary tax, is mainly for regulation. Its imposition on the press is
unconstitutional because it lays a prior restraint on the exercise of its right. VII. Alleged violations of the due process, equal protection and contract
Hence, although its application to others, such those selling goods, is valid, clauses and the rule on taxation. CREBA asserts that R.A. No. 7716 (1)
its application to the press or to religious groups, such as the Jehovah's impairs the obligations of contracts, (2) classifies transactions as covered
Witnesses, in connection with the latter's sale of religious books and or exempt without reasonable basis and (3) violates the rule that taxes
pamphlets, is unconstitutional. As the U.S. Supreme Court put it, "it is one should be uniform and equitable and that Congress shall "evolve a
thing to impose a tax on income or property of a preacher. It is quite progressive system of taxation."chanrobles virtual law library
another thing to exact a tax on him for delivering a sermon."chanrobles
virtual law library
With respect to the first contention, it is claimed that the application of the
tax to existing contracts of the sale of real property by installment or on
A similar ruling was made by this Court in American Bible Society v. City of deferred payment basis would result in substantial increases in the monthly
Manila, 101 Phil. 386 (1957) which invalidated a city ordinance requiring a amortizations to be paid because of the 10% VAT. The additional amount,
business license fee on those engaged in the sale of general merchandise. it is pointed out, is something that the buyer did not anticipate at the time
It was held that the tax could not be imposed on the sale of bibles by the he entered into the contract.chanroblesvirtualawlibrarychanrobles virtual
American Bible Society without restraining the free exercise of its right to law library
propagate.chanroblesvirtualawlibrarychanrobles virtual law library
The short answer to this is the one given by this Court in an early case:
The VAT is, however, different. It is not a license tax. It is not a tax on the "Authorities from numerous sources are cited by the plaintiffs, but none of
exercise of a privilege, much less a constitutional right. It is imposed on them show that a lawful tax on a new subject, or an increased tax on an
the sale, barter, lease or exchange of goods or properties or the sale or old one, interferes with a contract or impairs its obligation, within the
exchange of services and the lease of properties purely for revenue meaning of the Constitution. Even though such taxation may affect
purposes. To subject the press to its payment is not to burden the exercise particular contracts, as it may increase the debt of one person and lessen
of its right any more than to make the press pay income tax or subject it to the security of another, or may impose additional burdens upon one class
general regulation is not to violate its freedom under the and release the burdens of another, still the tax must be paid unless
Constitution.chanroblesvirtualawlibrarychanrobles virtual law library prohibited by the Constitution, nor can it be said that it impairs the
obligation of any existing contract in its true legal sense." (La Insular v.
Machuca Go-Tauco and Nubla Co-Siong, 39 Phil. 567, 574 (1919)). Indeed
Additionally, the Philippine Bible Society, Inc. claims that although it sells
not only existing laws but also "the reservation of the essential attributes
bibles, the proceeds derived from the sales are used to subsidize the cost
of sovereignty, is . . . read into contracts as a postulate of the legal order."
of printing copies which are given free to those who cannot afford to pay so
(Philippine-American Life Ins. Co. v. Auditor General, 22 SCRA 135, 147
that to tax the sales would be to increase the price, while reducing the
(1968)) Contracts must be understood as having been made in reference
volume of sale. Granting that to be the case, the resulting burden on the
to the possible exercise of the rightful authority of the government and no
exercise of religious freedom is so incidental as to make it difficult to
obligation of contract can extend to the defeat of that authority. (Norman
differentiate it from any other economic imposition that might make the
v. Baltimore and Ohio R.R., 79 L. Ed. 885
right to disseminate religious doctrines costly. Otherwise, to follow the
(1935)).chanroblesvirtualawlibrarychanrobles virtual law library
petitioner's argument, to increase the tax on the sale of vestments would
be to lay an impermissible burden on the right of the preacher to make a
sermon.chanroblesvirtualawlibrarychanrobles virtual law library
It is next pointed out that while �4 of R.A. No. 7716 exempts such �28(1) of the Constitution." (At 382) Rejecting the challenge to the law,
transactions as the sale of agricultural products, food items, petroleum, this Court held:
and medical and veterinary services, it grants no exemption on the sale of
real property which is equally essential. The sale of real property for
As the Court sees it, EO 273 satisfies all the requirements of a valid tax. It
socialized and low-cost housing is exempted from the tax, but CREBA
is uniform. . . .chanroblesvirtualawlibrarychanrobles virtual law library
claims that real estate transactions of "the less poor," i.e., the middle
class, who are equally homeless, should likewise be
exempted.chanroblesvirtualawlibrarychanrobles virtual law library The sales tax adopted in EO 273 is applied similarly on all goods and
services sold to the public, which are not exempt, at the constant rate of
0% or 10%.chanroblesvirtualawlibrarychanrobles virtual law library
The sale of food items, petroleum, medical and veterinary services, etc.,
which are essential goods and services was already exempt under �103,
pars. (b) (d) (1) of the NIRC before the enactment of R.A. No. 7716. The disputed sales tax is also equitable. It is imposed only on sales of
Petitioner is in error in claiming that R.A. No. 7716 granted exemption to goods or services by persons engaged in business with an aggregate gross
these transactions, while subjecting those of petitioner to the payment of annual sales exceeding P200,000.00. Small corner sari-sari stores are
the VAT. Moreover, there is a difference between the "homeless poor" and consequently exempt from its application. Likewise exempt from the tax
the "homeless less poor" in the example given by petitioner, because the are sales of farm and marine products, so that the costs of basic food and
second group or middle class can afford to rent houses in the meantime other necessities, spared as they are from the incidence of the VAT, are
that they cannot yet buy their own homes. The two social classes are thus expected to be relatively lower and within the reach of the general
differently situated in life. "It is inherent in the power to tax that the State public.chanroblesvirtualawlibrarychanrobles virtual law library
be free to select the subjects of taxation, and it has been repeatedly held
that 'inequalities which result from a singling out of one particular class for (At 382-383)
taxation, or exemption infringe no constitutional limitation.'" (Lutz v.
Araneta, 98 Phil. 148, 153 (1955). Accord, City of Baguio v. De Leon, 134
Phil. 912 (1968); Sison, Jr. v. Ancheta, 130 SCRA 654, 663 (1984); The CREBA claims that the VAT is regressive. A similar claim is made by
Kapatiran ng mga Naglilingkod sa Pamahalaan ng Pilipinas, Inc. v. Tan, 163 the Cooperative Union of the Philippines, Inc. (CUP), while petitioner Juan
SCRA 371 (1988)).chanroblesvirtualawlibrarychanrobles virtual law library T. David argues that the law contravenes the mandate of Congress to
provide for a progressive system of taxation because the law imposes a flat
rate of 10% and thus places the tax burden on all taxpayers without regard
Finally, it is contended, for the reasons already noted, that R.A. No. 7716 to their ability to pay.chanroblesvirtualawlibrarychanrobles virtual law
also violates Art. VI, �28(1) which provides that "The rule of taxation shall library
be uniform and equitable. The Congress shall evolve a progressive system
of taxation."chanrobles virtual law library
The Constitution does not really prohibit the imposition of indirect taxes
which, like the VAT, are regressive. What it simply provides is that
Equality and uniformity of taxation means that all taxable articles or kinds Congress shall "evolve a progressive system of taxation." The
of property of the same class be taxed at the same rate. The taxing power constitutional provision has been interpreted to mean simply that "direct
has the authority to make reasonable and natural classifications for taxes are . . . to be preferred [and] as much as possible, indirect taxes
purposes of taxation. To satisfy this requirement it is enough that the should be minimized." (E. FERNANDO, THE CONSTITUTION OF THE
statute or ordinance applies equally to all persons, forms and corporations PHILIPPINES 221 (Second ed. (1977)). Indeed, the mandate to Congress is
placed in similar situation. (City of Baguio v. De Leon, supra; Sison, Jr. v. not to prescribe, but to evolve, a progressive tax system. Otherwise, sales
Ancheta, supra)chanrobles virtual law library taxes, which perhaps are the oldest form of indirect taxes, would have
been prohibited with the proclamation of Art. VIII, �17(1) of the 1973
Indeed, the VAT was already provided in E.O. No. 273 long before R.A. No. Constitution from which the present Art. VI, �28(1) was taken. Sales taxes
7716 was enacted. R.A. No. 7716 merely expands the base of the tax. The are also regressive.chanroblesvirtualawlibrarychanrobles virtual law library
validity of the original VAT Law was questioned in Kapatiran ng
Naglilingkod sa Pamahalaan ng Pilipinas, Inc. v. Tan, 163 SCRA 383 (1988) Resort to indirect taxes should be minimized but not avoidedentirely
on grounds similar to those made in these cases, namely, that the law was because it is difficult, if not impossible, to avoid them by imposing such
"oppressive, discriminatory, unjust and regressive in violation of Art. VI,
taxes according to the taxpayers' ability to pay. In the case of the VAT, the (h) Goods or services with gross annual sale or receipt not
law minimizes the regressive effects of this imposition by providing for zero exceeding P500,000.00.chanroblesvirtualawlibrarychanrobles virtual law
rating of certain transactions (R.A. No. 7716, �3, amending �102 (b) of library
the NIRC), while granting exemptions to other transactions. (R.A. No.
7716, �4, amending �103 of the
(Respondents' Consolidated Comment on the Motions for Reconsideration,
NIRC).chanroblesvirtualawlibrarychanrobles virtual law library
pp. 58-60)

Thus, the following transactions involving basic and essential goods and
On the other hand, the transactions which are subject to the VAT are those
services are exempted from the VAT:
which involve goods and services which are used or availed of mainly by
higher income groups. These include real properties held primarily for sale
(a) Goods for consumption or use which are in their original state to customers or for lease in the ordinary course of trade or business, the
(agricultural, marine and forest products, cotton seeds in their original right or privilege to use patent, copyright, and other similar property or
state, fertilizers, seeds, seedlings, fingerlings, fish, prawn livestock and right, the right or privilege to use industrial, commercial or scientific
poultry feeds) and goods or services to enhance agriculture (milling of equipment, motion picture films, tapes and discs, radio, television, satellite
palay, corn sugar cane and raw sugar, livestock, poultry feeds, fertilizer, transmission and cable television time, hotels, restaurants and similar
ingredients used for the manufacture of places, securities, lending investments, taxicabs, utility cars for rent,
feeds).chanroblesvirtualawlibrarychanrobles virtual law library tourist buses, and other common carriers, services of franchise grantees of
telephone and telegraph.chanroblesvirtualawlibrarychanrobles virtual law
library
(b) Goods used for personal consumption or use (household and personal
effects of citizens returning to the Philippines) and or professional use, like
professional instruments and implements, by persons coming to the The problem with CREBA's petition is that it presents broad claims of
Philippines to settle here.chanroblesvirtualawlibrarychanrobles virtual law constitutional violations by tendering issues not at retail but at wholesale
library and in the abstract. There is no fully developed record which can impart to
adjudication the impact of actuality. There is no factual foundation to show
in the concrete the application of the law to actual contracts and exemplify
(c) Goods subject to excise tax such as petroleum products or to be used
its effect on property rights. For the fact is that petitioner's members have
for manufacture of petroleum products subject to excise tax and services
not even been assessed the VAT. Petitioner's case is not made concrete by
subject to percentage tax.chanroblesvirtualawlibrarychanrobles virtual law
a series of hypothetical questions asked which are no different from those
library
dealt with in advisory opinions.

(d) Educational services, medical, dental, hospital and veterinary services,


The difficulty confronting petitioner is thus apparent. He alleges
and services rendered under employer-employee
arbitrariness. A mere allegation, as here, does not suffice. There must be a
relationship.chanroblesvirtualawlibrarychanrobles virtual law library
factual foundation of such unconstitutional taint. Considering that petitioner
here would condemn such a provision as void on its face, he has not made
(e) Works of art and similar creations sold by the artist out a case. This is merely to adhere to the authoritative doctrine that
himself.chanroblesvirtualawlibrarychanrobles virtual law library where the due process and equal protection clauses are invoked,
considering that they are not fixed rules but rather broad standards, there
(f) Transactions exempted under special laws, or international is a need for proof of such persuasive character as would lead to such a
agreements.chanroblesvirtualawlibrarychanrobles virtual law library conclusion. Absent such a showing, the presumption of validity must
prevail.chanroblesvirtualawlibrarychanrobles virtual law library

(g) Export-sales by persons not VAT-


registered.chanroblesvirtualawlibrarychanrobles virtual law library (Sison, Jr. v. Ancheta, 130 SCRA at 661)

Adjudication of these broad claims must await the development of a


concrete case. It may be that postponement of adjudication would result in
a multiplicity of suits. This need not be the case, however. Enforcement of P.D. No. 1955; that in 1986, P.D. No. 2008 again granted cooperatives
the law may give rise to such a case. A test case, provided it is an actual exemption from income and sales taxes until December 31, 1991, but, in
case and not an abstract or hypothetical one, may thus be the same year, E.O. No. 93 revoked the exemption; and that finally in
presented.chanroblesvirtualawlibrarychanrobles virtual law library 1987 the framers of the Constitution "repudiated the previous actions of
the government adverse to the interests of the cooperatives, that is, the
repeated revocation of the tax exemption to cooperatives and instead
Nor is hardship to taxpayers alone an adequate justification for
upheld the policy of strengthening the cooperatives by way of the grant of
adjudicating abstract issues. Otherwise, adjudication would be no different
tax exemptions," by providing the following in Art. XII:
from the giving of advisory opinion that does not really settle legal
issues.chanroblesvirtualawlibrarychanrobles virtual law library
�1. The goals of the national economy are a more equitable distribution of
opportunities, income, and wealth; a sustained increase in the amount of
We are told that it is our duty under Art. VIII, �1, �2 to decide whenever
goods and services produced by the nation for the benefit of the people;
a claim is made that "there has been a grave abuse of discretion
and an expanding productivity as the key to raising the quality of life for
amounting to lack or excess of jurisdiction on the part of any branch or
all, especially the underprivileged.chanroblesvirtualawlibrarychanrobles
instrumentality of the government." This duty can only arise if an actual
virtual law library
case or controversy is before us. Under Art . VIII, �5 our jurisdiction is
defined in terms of "cases" and all that Art. VIII, �1, �2 can plausibly
mean is that in the exercise of that jurisdiction we have the judicial The State shall promote industrialization and full employment based on
power to determine questions of grave abuse of discretion by any branch sound agricultural development and agrarian reform, through industries
or instrumentality of the government.chanroblesvirtualawlibrarychanrobles that make full and efficient use of human and natural resources, and which
virtual law library are competitive in both domestic and foreign markets. However, the State
shall protect Filipino enterprises against unfair foreign competition and
trade practices.chanroblesvirtualawlibrarychanrobles virtual law library
Put in another way, what is granted in Art. VIII, �1, �2 is "judicial power,"
which is "the power of a court to hear and decide cases pending between
parties who have the right to sue and be sued in the courts of law and In the pursuit of these goals, all sectors of the economy and all regions of
equity" (Lamb v. Phipps, 22 Phil. 456, 559 (1912)), as distinguished from the country shall be given optimum opportunity to develop. Private
legislative and executive power. This power cannot be directly appropriated enterprises, including corporations, cooperatives, and similar collective
until it is apportioned among several courts either by the Constitution, as in organizations, shall be encouraged to broaden the base of their
the case of Art. VIII, �5, or by statute, as in the case of the Judiciary Act ownership.chanroblesvirtualawlibrarychanrobles virtual law library
of 1948 (R.A. No. 296) and the Judiciary Reorganization Act of 1980 (B.P.
Blg. 129). The power thus apportioned constitutes the court's "jurisdiction,"
�15. The Congress shall create an agency to promote the viability and
defined as "the power conferred by law upon a court or judge to take
growth of cooperatives as instruments for social justice and economic
cognizance of a case, to the exclusion of all others." (United States v.
development.
Arceo, 6 Phil. 29 (1906)) Without an actual case coming within its
jurisdiction, this Court cannot inquire into any allegation of grave abuse of
discretion by the other departments of the Petitioner's contention has no merit. In the first place, it is not true that
government.chanroblesvirtualawlibrarychanrobles virtual law library P.D. No. 1955 singled out cooperatives by withdrawing their exemption
from income and sales taxes under P.D. No. 175, �5. What P.D. No. 1955,
�1 did was to withdraw the exemptions and preferential treatments
VIII. Alleged violation of policy towards cooperatives. On the other hand,
theretofore granted to private business enterprises in general, in view of
the Cooperative Union of the Philippines (CUP), after briefly surveying the
the economic crisis which then beset the nation. It is true that after P.D.
course of legislation, argues that it was to adopt a definite policy of
No. 2008, �2 had restored the tax exemptions of cooperatives in 1986, the
granting tax exemption to cooperatives that the present Constitution
exemption was again repealed by E.O. No. 93, �1, but then again
embodies provisions on cooperatives. To subject cooperatives to the VAT
cooperatives were not the only ones whose exemptions were
would therefore be to infringe a constitutional policy. Petitioner claims that
withdrawn. The withdrawal of tax incentives applied to all, including
in 1973, P.D. No. 175 was promulgated exempting cooperatives from the
government and private entities. In the second place, the Constitution does
payment of income taxes and sales taxes but in 1984, because of the crisis
not really require that cooperatives be granted tax exemptions in order to
which menaced the national economy, this exemption was withdrawn by
promote their growth and viability. Hence, there is no basis for petitioner's WHEREFORE, the motions for reconsideration are denied with finality and
assertion that the government's policy toward cooperatives had been one the temporary restraining order previously issued is hereby
of vacillation, as far as the grant of tax privileges was concerned, and that lifted.chanroblesvirtualawlibrarychanrobles virtual law library
it was to put an end to this indecision that the constitutional provisions
cited were adopted. Perhaps as a matter of policy cooperatives should be
SO ORDERED.
granted tax exemptions, but that is left to the discretion of Congress. If
Congress does not grant exemption and there is no discrimination to
cooperatives, no violation of any constitutional policy can be
charged.chanroblesvirtualawlibrarychanrobles virtual law library

Indeed, petitioner's theory amounts to saying that under the Constitution


cooperatives are exempt from taxation. Such theory is contrary to the
Constitution under which only the following are exempt from taxation:
charitable institutions, churches and parsonages, by reason of Art. VI, �28
(3), and non-stock, non-profit educational institutions by reason of Art.
XIV, �4 (3).chanroblesvirtualawlibrarychanrobles virtual law library

CUP's further ground for seeking the invalidation of R.A. No. 7716 is that it
denies cooperatives the equal protection of the law because electric
cooperatives are exempted from the VAT. The classification between
electric and other cooperatives (farmers cooperatives, producers
cooperatives, marketing cooperatives, etc.) apparently rests on a [G.R. No. 118303. January 31, 1996.]
congressional determination that there is greater need to provide cheaper
electric power to as many people as possible, especially those living in the SENATOR HEHERSON T. ALVAREZ, SENATOR JOSE D. LINA, JR., MR.
rural areas, than there is to provide them with other necessities in life. We NICASIO B. BAUTISTA, MR. JESUS P. GONZAGA, MR. SOLOMON D.
cannot say that such classification is MAYLEM, LEONORA C. MEDINA, CASIANO S. ALIPON, Petitioners, v.
unreasonable.chanroblesvirtualawlibrarychanrobles virtual law library HON. TEOFISTO T. GUINGONA, JR., in his capacity as Executive
Secretary, HON. RAFAEL ALUNAN, in his capacity as Secretary of
Local Government, HON. SALVADOR ENRIQUEZ, in his capacity as
We have carefully read the various arguments raised against the Secretary of Budget, THE COMMISSION ON AUDIT, HON. JOSE
constitutional validity of R.A. No. 7716. We have in fact taken the MIRANDA, in his capacity as Municipal Mayor of Santiago and HON.
extraordinary step of enjoining its enforcement pending resolution of these CHARITO MANUBAY, HON. VICTORINO MIRANDA, JR., HON.
cases. We have now come to the conclusion that the law suffers from none ARTEMIO ALVAREZ, HON. DANILO VERGARA, HON. PETER DE
of the infirmities attributed to it by petitioners and that its enactment by JESUS, HON. NELIA NATIVIDAD, HON. CELSO CALEON and HON.
the other branches of the government does not constitute a grave abuse of ABEL MUSNGI, in their capacity as SANGGUNIANG BAYAN
discretion. Any question as to its necessity, desirability or expediency must MEMBERS, MR. RODRIGO L. SANTOS, in his capacity as Municipal
be addressed to Congress as the body which is electorally responsible, Treasurer, and ATTY. ALFREDO S. DIRIGE, in his capacity as
remembering that, as Justice Holmes has said, "legislators are the ultimate Municipal Administrator, Respondents.
guardians of the liberties and welfare of the people in quite as great a
degree as are the courts." (Missouri, Kansas & Texas Ry. Co. v. May, 194 Belo, Gozon, Elma, Parel, Asuncion & Lucila, for Petitioners.
U.S. 267, 270, 48 L. Ed. 971, 973 (1904)). It is not right, as petitioner in
G.R. No. 115543 does in arguing that we should enforce the public Rene P. Pine, for Private Respondents.
accountability of legislators, that those who took part in passing the law in
question by voting for it in Congress should later thrust to the courts the
burden of reviewing measures in the flush of enactment. This Court does
not sit as a third branch of the legislature, much less exercise a veto power SYLLABUS
over legislation.chanroblesvirtualawlibrarychanrobles virtual law library
constitutional prescription, originate exclusively in the House of
Representatives, the claim of petitioners that Republic Act No. 7720 did not
originate exclusively in the House of Representatives because a bill of the
1. ADMINISTRATIVE LAW; LOCAL GOVERNMENT CODE; LOCAL
same import, SB No. 1243, was passed in the Senate, is untenable because
GOVERNMENT, CONSTRUED. — A local Government Unit is a political
it cannot be denied that HB No. 8817 was filed in the House of
subdivision of the State which is constituted by law and possessed of
Representatives first before SB No. 1243 was filed in the Senate.
substantial control over its own affairs. Remaining to be an intra sovereign
Petitioners themselves cannot disavow their own admission that HB No.
subdivision of one sovereign nation, but not intended, however, to be an
8817 was filed on April 18, 1993 while SB No. 1243 was filed on May 19,
emperium in emperia, the local government unit is autonomous in the
1993. The filing of HB No. 8817 was thus precursive not only of the said
sense that it is given more powers, authority, responsibilities and
Act in question but also of SB No. 1243. Thus, HB No. 8817, was the bill
resources.
that initiated the legislative process that culminated in the enactment of
Republic Act No. 7720. No violation of Section 24, Article VI, of the 1987
2. ID.; ID.; INCOME DEFINED. — Income is defined in the Local
Constitution is perceptible under the circumstances attending the instant
Government Code to be all revenues and receipts collected or received
controversy.
forming the gross accretions of funds of the local government unit.
7. ID.; ID.; FILING IN THE SENATE OF A SUBSTITUTE BILL IN
3. ID.; ID.; INTERNAL REVENUE ALLOTMENT (IRA) ARE ITEMS OF INCOME.
ANTICIPATION OF ITS RECEIPT OF THE HOUSE BILL WITHOUT ACTING
— The IRAs are items of income because they form part of the gross
THEREON DOES NOT CONTRAVENE CONSTITUTIONAL REQUIREMENT. —
accretion of the funds of the local government unit. The IRAs regularly and
Petitioners themselves acknowledge that HB No. 8817 was already
automatically accrue to the local treasury without need of any further
approved on Third Reading and duly transmitted to the Senate when the
action on the part of the local government unit. They thus constitute
Senate Committee on Local Government conducted its public hearing on
income which the local government can invariably rely upon as the source
HB No. 8817. HB No. 8817 was approved on the Third Reading on
of much needed funds.
December 17, 1993 and transmitted to the Senate on January 28, 1994; a
little less than a month thereafter, or on February 23, 1994, the Senate
4. ID.; ID.; ANNUAL INCOME DEFINED. — Department of Finance Order
Committee on Local Government conducted public hearings on SB No.
No. 35-93 correctly encapsulizes the full import of the above disquisition
1243. Clearly, the Senate held in abeyance any action on SB No. 1243 until
when it defined ANNUAL INCOME to be "revenues and receipts realized by
it received HB No. 8817, already approved on the Third Reading, from the
provinces, cities and municipalities from regular sources of the Local
House of Representatives. The filing in the Senate of a substitute bill in
General Fund including the internal revenue allotment and other shares
anticipation of its receipt of the bill from the House, does not contravene
provided for in Sections 284, 290 and 291 of the Code, but exclusive of
the constitutional requirement that a bill of local application should
n.on.-recurring receipts, such as other national aids, grants, financial
originate in the House of Representatives, for as long as the Senate does
assistance, loan proceeds, sales of fixed assets, and similar others"
not act thereupon until it receives the House bill.
(Emphasis ours).
8. REMEDIAL LAW; EVIDENCE; PRESUMPTIONS; EVERY LAW IS PRESUMED
5. STATUTORY CONSTRUCTION; ORDER CONSTITUTING EXECUTIVE OR
CONSTITUTIONAL; CONSTITUTIONALITY OF R.A. 7720 NOT OVERCOME IN
CONTEMPORANEOUS CONSTRUCTION OF A STATUTE BY ADMINISTRATIVE
CASE AT BAR. — It is a well-entrenched jurisprudential rule that on the
AGENCY CHARGED WITH THE TASK OF INTERPRETING THE SAME,
side of every law lies the presumption of constitutionality. Consequently,
ENTITLED TO FULL RESPECT. — Such order, constituting executive or
for RA No. 7720 to be nullified, it must be shown that there is a clear and
contemporaneous construction of a statute by an administrative agency
unequivocal breach of the Constitution, not merely a doubtful and
charged with the task of interpreting and applying the same, is entitled to
equivocal one; in other words, the grounds for nullity must be clear and
full respect and should be accorded great weight by the courts, unless such
beyond reasonable doubt. Those who petition this court to declare a law to
construction is clearly shown to be in sharp conflict with the Constitution,
be unconstitutional must clearly and fully establish the basis that will justify
the governing statute, or other laws.
such a declaration; otherwise, their petition must fail. Taking into
consideration the justification of our stand on the immediately preceding
6. CONSTITUTIONAL LAW; LEGISLATIVE; BILL CONVERTING
ground raised by petitioners to challenge the constitutionality of RA No.
MUNICIPALITY TO CITY MUST ORIGINATE FROM THE HOUSE; PASSING OF
7720, the Court stands on the holding that petitioners have failed to
SUBSEQUENT BILL COVERING THE SAME MUNICIPALITY, NO ADVERSE
overcome the presumption. The dismissal of this petition is, therefore,
EFFECT. — Although a bill of local application like HB No. 8817 should, by
inevitable.
Meanwhile, a counterpart of HB No. 8817, Senate Bill No. 1243, entitled,
"An Act Converting the Municipality of Santiago into an Independent
Component City to be Known as the City of Santiago," was filed in the
DECISION
Senate. It was introduced by Senator Vicente Sotto III, as principal
sponsor, on May 19, 1993. This was just after the House of
Representatives had conducted its first public hearing on HB No.
HERMOSISIMA. JR., J.: 8817.chanroblesvirtuallawlibrary

On February 23, 1994, or a little less than a month after HB No. 8817 was
transmitted to the Senate, the Senate Committee on Local Government
Of main concern to the petitioners is whether Republic Act No. 7720, just
conducted public hearings on SB No. 1243. On March 1, 1994, the said
recently passed by Congress and signed by the President into law, is
committee submitted Committee Report No. 378 on HB No. 8817, with the
constitutionally infirm.
recommendation that it be approved without amendment, taking into
consideration the reality that H.B. No. 8817 was on all fours with SB No.
Indeed, in this Petition for Prohibition with prayer for Temporary
1243. Senator Heherson T. Alvarez, one of the herein petitioners, indicated
Restraining Order and Preliminary Prohibitory Injunction, petitioners assail
his approval thereto by signing said report as member of the Committee on
the validity of Republic Act No. 7720, entitled, "An Act Converting the
Local Government. 
Municipality of Santiago, Isabela into an Independent Component City to be
known as the City of Santiago," mainly because the Act allegedly did not
On March 3, 1994, Committee Report No. 378 was passed by the Senate
originate exclusively in the House of Representatives as mandated by
on Second Reading and was approved on Third Reading on March 14,
Section 24, Article VI of the 1987 Constitution.chanroblesvirtuallawlibrary
1994. On March 22, 1994, the House of Representatives, upon being
apprised of the action of the Senate, approved the amendments proposed
Also, petitioners claim that the Municipality of Santiago has not met the
by the Senate. 
minimum average annual income required under Section 450 of the Local
Government Code of 1991 in order to be converted into a component city. 
The enrolled bill, submitted to the President on April 12, 1994, was signed
by the Chief Executive on May 5, 1994 as Republic Act No. 7720. When a
Undisputed is the following chronicle of the metamorphosis of House Bill
plebiscite on the Act was held on July 13, 1994, a great majority of the
No. 8817 into Republic Act No. 7720:chanrob1es virtual 1aw library
registered voters of Santiago voted in favor of the conversion of Santiago
into a city.chanroblesvirtuallawlibrary
On April 18, 1993, HB No. 8817, entitled "An Act Converting the
Municipality of Santiago into an Independent Component City to be known
The question as to the validity of Republic Act No. 7720 hinges on the
as the City of Santiago," was filed in the House of Representatives with
following twin issues: (I) Whether or not the Internal Revenue Allotments
Representative Antonio Abaya as principal author. Other sponsors included
(IRAs) are to included in the computation of the average annual income of
Representatives Ciriaco Alfelor, Rodolfo Albano, Santiago Respicio and
a municipality for purposes of its conversion into an independent
Faustino Dy. The bill was referred to the House Committee on Local
component city, and (II) Whether or not, considering that the Senate
Government and the House Committee on Appropriations on May 5,
passed SB No. 1243, its own version of HB No. 8817, Republic Act No.
1993.chanroblesvirtuallawlibrary
7720 can be said to have originated in the House of Representatives.
On May 19, 1993, June 1, 1993, November 28, 1993, and December 1,
I. The annual income of a local government unit includes the IRAs.
1993, public hearings on HB No. 8817 were conducted by the House
Committee on Local Government. The committee submitted to the House a
Petitioners claim that Santiago could not qualify into a component city
favorable report, with amendments, on December 9, 1993. 
because its average annual last two (2) consecutive years based on 1991
constant prices falls below the required annual income of Pesos
On December 13, 1993, HB No. 8817 was passed by the House of
(P20,000,000.00) for its conversion into a city, petitioners having
Representatives on Second Reading and was approved on Third Reading on
computed Santiago’s average annual income in the following manner:
December 17, 1993. On January 28, 1994, HB No. 8817 was transmitted to
the Senate. 
Total income (at 1991 constant prices) for 1991 P20,379,057.07 and contextual explication of the meaning of internal revenue allotments
(IRAs) vis-a-vis the notion of income of a local government unit and the
Total income (at 1991 constant prices) for 1992 P21,570,106.87 principles of local autonomy and decentralization underlying the
institutionalization and intensified empowerment of the local government
—————— system. 

Total income for 1991 and 1992 P41,949,163.94 A Local Government Unit is a political subdivision of the State which is
constituted by law and possessed of substantial control over its own affairs.
3 Remaining to be an intra sovereign subdivision of one sovereign nation,
Minus:
but not intended, however, to be an imperium in imperio, 4 the local
government unit is autonomous in the sense that it is given more powers,
IRAs for 1991 and 1992 P15,730,043.00 authority, responsibilities and resources. 5 Power which used to be highly
centralized in Manila, is thereby deconcentrated, enabling especially the
—————— peripheral local government units to develop not only at their own pace
and discretion but also with their own resources and assets. 6
Total income for 1991 and 1992 P26,219,120.94
The practical side to development through a decentralized local
Average Annual Income P13,109,560.47  government system certainly concerns the matter of financial resources.
With its broadened powers and increased responsibilities, a local
==================================== government unit must now operate on a much wider scale. More extensive
operations, in turn, entail more expenses. Understandably, the vesting of
By dividing the total income of Santiago for calendar years 1991 and 1992, duty, responsibility and accountability in every local government unit is
after deducting the IRAs, the average annual income arrived at would only accompanied with a provision for reasonably adequate resources to
be P13,109,560.47 based on the 1991 constant prices. Thus, petitioners discharge its powers and effectively carry out its functions. 7 Availment of
claim that Santiago’s income is far below the aforesaid Twenty Million such resources is effectuated through the vesting in every local
Pesos average annual income requirement.  government unit of (1) the right to create and broaden its own source of
revenue; (2) the right to be allocated a just share in national taxes such
The certification issued by the Bureau of Local Government Finance of the share being in the form of internal revenue allotments (IRAs); and (3) the
Department of Finance, which indicates Santiago’s average annual income right to be given its equitable share in the proceeds of the utilization and
to be P20,974,581.97, is allegedly not accurate as the Internal Revenue development of the national wealth, if any, within its territorial boundaries.
Allotments were not excluded from the computation. Petitioners asseverate 8 
that the IRAs are not actually income but transfers and/or budgetary aid
from the national government and that they fluctuate, increase or The funds generated from local taxes, IRAs and national wealth utilization
decrease, depending on factors like population, land and equal sharing. proceeds accrue to the general fund of the local government and are used
to finance its operations subject to specified modes of spending the same
In this regard, we hold that petitioners’ asseverations are untenable as provided for in the Local Government Code and its implementing rules
because Internal Revenue Allotments form part of the income of Local and regulations. For instance, not less than twenty percent (20%) of the
Government Units.chanroblesvirtuallawlibrary IRAs must be set aside for local development projects. 9 As such, for
purposes of budget preparation, which budget should reflect the estimates
It is true that for a municipality to be converted into a component city, it of the income of the local government unit, among others, the IRAs and
must, among others, have an average annual income of at least Twenty the share in the national wealth utilization proceeds are considered items
Million Pesos for the last two (2) consecutive years based on 1991 constant of income. This is as it should be, since income is defined in the Local
prices. 1 Such income must be duly certified by the Department of Finance. Government Code to be all revenues and receipts collected or received
2  forming the gross accretions of funds of the local government unit. 10 

Resolution of the controversy regarding compliance by the Municipality of The IRAs are items of income because they form part of the gross accretion
Santiago with the aforecited income requirement hinges on a correlative of the funds of the local government unit. The IRAs regularly and
automatically accrue to the local treasury without need of any further
action on the part of the local government unit. 11 They thus constitute that initiated the legislative process that culminated in the enactment of
income which the local government can invariably rely upon as the source Republic Act No. 7720. No violation of Section 24, Article VI, of the 1987
of much needed funds. For purposes of converting the Municipality of Constitution is perceptible under the circumstances attending the instant
Santiago into a city, the Department of Finance certified, among others, controversy.chanroblesvirtuallawlibrary
that the municipality had an average annual income of at least Twenty
Million Pesos for the last two (2) consecutive years based on 1991 constant Furthermore, petitioners themselves acknowledge that HB No. 8817 was
prices. This, the Department of Finance did after including the IRAs in its already approved on Third Reading and duly transmitted to the Senate
computation of said average annual income.chanroblesvirtuallawlibrary when the Senate Committee on Local Government conducted its public
hearing on HB No. 8817. HB No. 8817 was approved on the Third Reading
Furthermore, Section 450 (c) of the Local Government Code provides that on December 17, 1993 and transmitted to the Senate on January 28,
"the average annual income shall include the income accruing to the 1994; a little less than a month thereafter or on February 23, 1994, the
general fund, exclusive of special funds, transfers, and non-recurring Senate Committee on Local Government conducted public hearings on SB
income.’’ To reiterate, IRAs are a regular, recurring item of income; nil is No. 1243. Clearly, the Senate held in abeyance any action on SB No. 1243
there a basis, too, to classify the same as a special fund or transfer, since until it received HB No. 8817, already approved on the Third Reading, from
IRAs have a technical definition and meaning all its own as used in the the House of Representatives. The filing in the Senate of a substitute bill in
Local Government Code that unequivocally makes it distinct from special anticipation of its receipt of the bill from the House, does not contravene
funds or transfers referred to when the Code speaks of "funding support the constitutional requirement that a bill of local application should
from the national government, its instrumentalities and government- originate in the House of Representatives, for as long as the Senate does
owned- or -controlled corporations." 12  not act thereupon until it receives the House bill. 

Thus, Department of Finance Order No. 35-93 13 correctly encapsulizes the We have already addressed this issue in the case of Tolentino v. Secretary
full import of the above disquisition when it defined ANNUAL INCOME to be of Finance. 17 There, on the matter of the Expanded Value Added Tax
"revenues and receipts realized by provinces cities and municipalities from (EVAT) Law, which, as a revenue bill, is nonetheless constitutionally
regular sources of the Local General Fund including the internal revenue required to originate exclusively in the House of Representatives, we
allotment and other shares provided for in Sections 284, 290 and 291 of explained:jgc:chanrobles.com.ph
the Code, but exclusive of non-recurring receipts, such as other national
aids, grants, financial assistance, loan proceeds, sales of fixed assets, and ". . . To begin with, it is not the law — but the revenue bill — which is
similar others" (Underscoring ours). 14 Such order, constituting executive required by the Constitution to ‘originate exclusively’ in the House of
or contemporaneous construction of a statute by an administrative agency Representatives. It is important to emphasize this, because a bill
charged with the task of interpreting and applying the same, is entitled to originating in the House may undergo such extensive changes in the
full respect and should be accorded great weight by the courts, unless such Senate that the result may be a rewriting of the whole. . . . as a result of
construction is clearly shown to be in sharp conflict with the Constitution, the Senate action, a distinct bill may be produced. To insist that a revenue
the governing statute, or other laws. 15  statute — and not only the bill which initiated the legislative process
culminating in the enactment of the law — must substantially be the same
II. In the enactment of RA No. 7720, there was compliance with Section as the House bill would be to deny the Senate’s power not only to ‘concur
24, Article VI of the 1987 Constitution. with amendments’ but also to ‘propose amendments.’ It would be to violate
the co-equality of legislative power of the two houses of Congress and in
Although a bill of local application like HB No. 8817 should, by fact make the House superior to the Senate.
constitutional prescription, 16 originate exclusively in the House of
Representatives, the claim of petitioners that Republic Act No. 7720 did not x       x       x
originate exclusively in the House of Representatives because a bill of the
same import, SB No. 1243, was passed in the Senate, is untenable because
it cannot be denied that HB No. 8817 was filed in the House of It is insisted, however, that S. No. 1630 was passed not in substitution of
Representatives first before SB No. 1243 was filed in the Senate. H. No. 11197 but of another Senate bill (S. No. 1129) earlier filed and that
Petitioners themselves cannot disavow their own admission that HB No. what the Senate did was merely to ‘take [H. No. 11197] into consideration’
8817 was filed on April 18, 1993 while SB No. 1243 was filed on May 19, in enacting S. No. 1630. There is really no difference between the Senate
1993. The filing of HB No. 8817 was thus precursive not only of the said preserving H. No. 11197 up to the enacting clause and then writing its own
Act in question but also of SB No. 1243. Thus, HB No. 8817, was the bill
version following the enacting clause (which, it would seem petitioners
admit is an amendment by substitution), and, on the other hand,
separately presenting a bill of its own on the same subject matter. In either
case the result are two bills on the same subject.

Indeed, what the Constitution simply means is that the initiative for filing
revenue, tariff, or tax bills, bills authorizing an increase of the public debt,
private bills and bills of local application must come from the House of
Representatives on the theory that, elected as they are from the districts,
the members of the House can be expected to be more sensitive to the
local needs and problems. On the other hand, the senators, who are
elected at large, are expected to approach the same problems from the
national perspective. Both views are thereby made to bear n the enactment
of such laws.

Nor does the Constitution prohibit the filing in the Senate of a substitute
bill in anticipation of its receipt of the bill from the House, so long as action
by the Senate as a body is withheld pending receipt of the House Bill. . . ."
18 

III. Every law, including RA No. 7720, has in its favor the presumption of
constitutionality.

It is a well-entrenched jurisprudential rule that on the side of every law lies


the presumption of constitutionality. 19 Consequently, for RA No. 7720 to
be nullified it must be shown that there is a clear and unequivocal breach
of the Constitution, not merely a doubtful and equivocal one; in other
words, the grounds for nullity must be clear and beyond reasonable doubt.
20 Those who petition this court to declare a law to be unconstitutional
must clearly and fully establish the basis that will justify such a
declaration; otherwise, their petition must fail. Taking into consideration LAWYERS AGAINST MONOPOLY AND POVERTY (LAMP),
the justification of our stand on the immediately preceding ground raised REPRESENTED BY ITS CHAIRMAN AND COUNSEL, CEFERINO
by petitioners to challenge the constitutionality of RA No. 7720, the Court PADUA, MEMBERS, ALBERTO ABELEDA, JR., ELEAZAR ANGELES,
stands on the holding that petitioners have failed to overcome the GREGELY FULTON ACOSTA, VICTOR AVECILLA, GALILEO BRION,
presumption. The dismissal of this petition is, therefore, inevitable. ANATALIA BUENAVENTURA, EFREN CARAG, PEDRO CASTILLO,
NAPOLEON CORONADO, ROMEO ECHAUZ, ALFREDO DE GUZMAN,
WHEREFORE, the instant petition is DISMISSED for lack of merit with costs ROGELIO KARAGDAG, JR., MARIA LUZ ARZAGA-MENDOZA, LEO LUIS
against petitioners. MENDOZA, ANTONIO P. PAREDES, AQUILINO PIMENTEL III, MARIO
REYES, EMMANUEL SANTOS, TERESITA SANTOS, RUDEGELIO
SO ORDERED.chanrobles TACORDA, SECRETARY GEN. ROLANDO ARZAGA, BOARD OF
CONSULTANTS, JUSTICE ABRAHAM SARMIENTO, SEN. AQUILINO
PIMENTEL, JR., AND BARTOLOME FERNANDEZ, JR., PETITIONERS,
VS. THE SECRETARY OF BUDGET AND MANAGEMENT, THE
TREASURER OF THE PHILIPPINES, THE COMMISSION ON AUDIT,
AND THE PRESIDENT OF THE SENATE AND THE SPEAKER OF THE
HOUSE OF REPRESENTATIVES IN REPRESENTATION OF THE
MEMBERS OF THE CONGRESS, RESPONDENTS.
DECISION Petitioner’s Position

MENDOZA, J.: According to LAMP, the above provision is silent and, therefore, prohibits
an automatic or direct allocation of lump sums to individual senators and
For consideration of the Court is an original action for certiorari assailing congressmen for the funding of projects.  It does not empower individual
the constitutionality and legality of the implementation of the Priority Members of Congress to propose, select and identify programs and projects
Development Assistance Fund (PDAF) as provided for in Republic to be funded out of PDAF.  “In previous GAAs, said allocation and
Act (R.A.) 9206 or the General Appropriations Act for 2004 (GAA of 2004).  identification of projects were the main features of the ‘pork barrel’ system
Petitioner Lawyers Against Monopoly and Poverty (LAMP), a group of technically known as Countrywide Development Fund (CDF).  Nothing of
lawyers who have banded together with a mission of dismantling all forms the sort is now seen in the present law (R.A. No. 9206 of CY 2004).3  In its
of political, economic or social monopoly in the country,1 also sought the memorandum, LAMP insists that “[t]he silence in the law of direct or even
issuance of a writ of preliminary injunction or temporary restraining order indirect participation by members of Congress betrays a deliberate intent
to enjoin respondent Secretary of the Department of Budget and on the part of the Executive and the Congress to scrap and do away with
Management (DBM) from making, and, thereafter, releasing budgetary the ‘pork barrel’ system.”4  In other words, “[t]he omission of the PDAF
allocations to individual members of Congress as “pork barrel” funds out of provision to specify sums as ‘allocations’ to individual Members of Congress
PDAF.  LAMP likewise aimed to stop the National Treasurer and the is a ‘casus omissus’ signifying an omission intentionally made by Congress
Commission on Audit (COA) from enforcing the questioned provision.cralaw that this Court is forbidden to supply.”5  Hence, LAMP is of the conclusion
that “the pork barrel has become legally defunct under the present state of
On September 14, 2004, the Court required respondents, including the GAA 2004.”6
President of the Senate and the Speaker of the House of Representatives,
to comment on the petition.  On April 7, 2005, petitioner filed a Reply LAMP further decries the supposed flaws in the implementation of the
thereto.2 On April 26, 2005, both parties were required to submit their provision, namely: 1) the DBM illegally made and directly released
respective memoranda. budgetary allocations out of PDAF in favor of individual Members of
Congress; and 2) the latter do not possess the power to propose, select
The GAA of 2004 contains the following provision subject of this petition: and identify which projects are to be actually funded by PDAF.

PRIORITY DEVELOPMENT ASSISTANCE FUND For LAMP, this situation runs afoul against the principle of separation of
powers because in receiving and, thereafter, spending funds for their
For fund requirements of priority development programs and projects, as chosen projects, the Members of Congress in effect intrude into an
indicated hereunder - P8,327,000,000.00 executive function. In other words, they cannot directly spend the funds,
the appropriation for which was made by them.  In their individual
Xxxxx capacities, the Members of Congress cannot “virtually tell or dictate upon
the Executive Department how to spend taxpayer’s money.7  Further, the
Special Provision authority to propose and select projects does not pertain to legislation. “It
is, in fact, a non-legislative function devoid of constitutional
1.  Use and Release of the Fund.  The amount herein appropriated shall be sanction,”8 and, therefore, impermissible and must be considered nothing
used to fund priority programs and projects or to fund the required less than malfeasance. The proposal and identification of the projects do
counterpart for foreign-assisted programs and projects: PROVIDED, That not involve the making of laws or the repeal and amendment thereof,
such amount shall be released directly to the implementing agency or Local which is the only function given to the Congress by the Constitution. Verily,
Government Unit concerned:   PROVIDED, FURTHER, That the allocations the power of appropriation granted to Congress as a collegial body, “does
authorized herein may be realigned to any expense class, if deemed not include the power of the Members thereof to individually propose,
necessary: PROVIDED FURTHERMORE, That a maximum of ten percent select and identify which projects are to be actually implemented and
(10%) of the authorized allocations by district may be used for funded - a function which essentially and exclusively pertains to the
procurement of rice and other basic commodities which shall be purchased Executive Department.”9  By allowing the Members of Congress to receive
from the National Food Authority. direct allotment from the fund, to propose and identify projects to be
funded and to perform the actual spending of the fund, the implementation
of the PDAF provision becomes legally infirm and constitutionally
repugnant. The Court’s Ruling

Respondents’ Position    To the Court, the case boils down to these issues:  1) whether or not the
mandatory requisites for the exercise of judicial review are met in this
For their part, the respondents10 contend that the petition miserably lacks case; and 2) whether or not the implementation of PDAF by the Members
legal and factual grounds.  Although they admit that PDAF traced its roots of Congress is unconstitutional and illegal.
to CDF,11 they argue that the former should not be equated with “pork
barrel,” which has gained a derogatory meaning referring “to government Like almost all powers conferred by the Constitution, the power of judicial
projects affording political opportunism.”12  In the petition, no proof of this review is subject to limitations, to wit: (1) there must be an actual case or
was offered.   It cannot be gainsaid then that the petition cannot stand on controversy calling for the exercise of judicial power; (2) the person
inconclusive media reports, assumptions and conjectures alone.  Without challenging the act must have the standing to question the validity of the
probative value, media reports cited by the petitioner deserve scant subject act or issuance; otherwise stated, he must have a personal and
consideration especially the accusation that corrupt legislators have substantial interest in the case such that he has sustained, or will sustain,
allegedly proposed cuts or slashes from their pork barrel.  Hence, the Court direct injury as a result of its enforcement; (3) the question of
should decline the petitioner’s plea to take judicial notice of the supposed constitutionality must be raised at the earliest opportunity; and (4) the
iniquity of PDAF because there is no concrete proof that PDAF, in the guise issue of constitutionality must be the very lis mota of the case.16
of “pork barrel,” is a source of “dirty money” for unscrupulous lawmakers
and other officials who tend to misuse their allocations.  These “facts” have An aspect of the “case-or-controversy” requirement is the requisite of
no attributes of sufficient notoriety or general recognition accepted by the “ripeness.”  In the United States, courts are centrally concerned with
public without qualification, to be subjected to judicial notice.  This applies, whether a case involves uncertain contingent future events that may not
a fortiori, to the claim that Members of Congress are beneficiaries of occur as anticipated, or indeed may not occur at all. Another concern is the
commissions (kickbacks) taken out of the PDAF allocations and releases evaluation of the twofold aspect of ripeness: first, the fitness of the issues
and preferred by favored contractors representing from 20% to 50% of the for judicial decision; and second, the hardship to the parties entailed by
approved budget for a particular project. 13  Suffice it to say, the withholding court consideration.  In our jurisdiction, the issue of ripeness is
perceptions of LAMP on the implementation of PDAF must not be based on generally treated in terms of actual injury to the plaintiff.  Hence, a
mere speculations circulated in the news media preaching the evils of pork question is ripe for adjudication when the act being challenged has had a
barrel.  Failing to present even an iota of proof that the DBM Secretary has direct adverse effect on the individual challenging it.17
been releasing lump sums from PDAF directly or indirectly to individual
Members of Congress, the petition falls short of its cause. In this case, the petitioner contested the implementation of an alleged
unconstitutional statute, as citizens and taxpayers.  According to LAMP, the
Likewise admitting that CDF and PDAF are “appropriations for substantially practice of direct allocation and release of funds to the Members of
similar, if not the same, beneficial purposes,” 14 the respondents Congress and the authority given to them to propose and select projects is
invoke Philconsa v. Enriquez,15 where CDF was described as an imaginative the core of the law’s flawed execution resulting in a serious constitutional
and innovative process or mechanism of implementing priority transgression involving the expenditure of public funds. Undeniably, as
programs/projects specified in the law.  In Philconsa, the Court upheld the taxpayers, LAMP would somehow be adversely affected by this.  A finding
authority of individual Members of Congress to propose and identify priority of unconstitutionality would necessarily be tantamount to a misapplication
projects because this was merely recommendatory in nature. In said case, of public funds which, in turn, cause injury or hardship to taxpayers. This
it was also recognized that individual members of Congress far more than affords “ripeness” to the present controversy.
the President and their congressional colleagues were likely to be
knowledgeable about the needs of their respective constituents and the Further, the allegations in the petition do not aim to obtain sheer legal
priority to be given each project. opinion in the nature of advice concerning legislative or executive action.
The possibility of constitutional violations in the implementation of PDAF
The Issues surely involves the interplay of legal rights susceptible of judicial
resolution. For LAMP, this is the right to recover public funds possibly
The respondents urge the Court to dismiss the petition for its failure to misapplied by no less than the Members of Congress.  Hence, without
establish factual and legal basis to support its claims, thereby lacking an prejudice to other recourse against erring public officials, allegations of
essential requisite of judicial review—an actual case or controversy. illegal expenditure of public funds reflect a concrete injury that may have
been committed by other branches of government before the court
intervenes.  The possibility that this injury was indeed committed cannot The powers of government are generally divided into three branches: the
be discounted. The petition complains of illegal disbursement of public Legislative, the Executive and the Judiciary. Each branch is supreme within
funds derived from taxation and this is sufficient reason to say that there its own sphere being independent from one another and it is this
indeed exists a definite, concrete, real or substantial controversy before the supremacy which enables the courts to determine whether a law is
Court. constitutional or unconstitutional.24   The Judiciary is the final arbiter on the
question of whether or not a branch of government or any of its officials
Anent locus standi, “the rule is that the person who impugns the validity of has acted without jurisdiction or in excess of jurisdiction or so capriciously
a statute must have a personal and substantial interest in the case such as to constitute an abuse of discretion amounting to excess of jurisdiction.
that he has sustained, or will sustained, direct injury as a result of its This is not only a judicial power but a duty to pass judgment on matters of
enforcement.18  The gist of the question of standing is whether a party this nature.25
alleges “such a personal stake in the outcome of the controversy as to
assure that concrete adverseness which sharpens the presentation of With these long-established precepts in mind, the Court now goes to the
issues upon which the court so largely depends for illumination of difficult crucial question: In allowing the direct allocation and release of PDAF funds
constitutional questions.”19  In public suits, the plaintiff, representing the to the Members of Congress based on their own list of proposed projects,
general public, asserts a “public right” in assailing an allegedly illegal did the implementation of the PDAF provision under the GAA of 2004
official action. The plaintiff may be a person who is affected no differently violate the Constitution or the laws?
from any other person, and could be suing as a “stranger,” or as a “citizen”
or “taxpayer.”20  Thus, taxpayers have been allowed to sue where there is The Court rules in the negative.
a claim that public funds are illegally disbursed or that public money is
being deflected to any improper purpose, or that public funds are wasted In determining whether or not a statute is unconstitutional, the Court does
through the enforcement of an invalid or unconstitutional law.21  Of greater not lose sight of the presumption of validity accorded to statutory acts of
import than the damage caused by the illegal expenditure of public funds is Congress.   In Fariñas v. The Executive Secretary,26 the Court held that:
the mortal wound inflicted upon the fundamental law by the enforcement of
an invalid statute.22 Every statute is presumed valid. The presumption is that the legislature
intended to enact a valid, sensible and just law and one which operates no
Here, the sufficient interest preventing the illegal expenditure of money further than may be necessary to effectuate the specific purpose of the
raised by taxation required in taxpayers’ suits is established.  Thus, in the law.  Every presumption should be indulged in favor of the
claim that PDAF funds have been illegally disbursed and wasted through constitutionality and the burden of proof is on the party alleging
the enforcement of an invalid or unconstitutional law, LAMP should be that there is a clear and unequivocal breach of the Constitution.
allowed to sue.  The case of Pascual v. Secretary of Public Works23 is
authority in support of the petitioner: To justify the nullification of the law or its implementation, there must be a
clear and unequivocal, not a doubtful, breach of the Constitution. In case of
In the determination of the degree of interest essential to give the requisite doubt in the sufficiency of proof establishing unconstitutionality, the Court
standing to attack the constitutionality of a statute, the general rule is that must sustain legislation because “to invalidate [a law] based on x x x
not only persons individually affected, but also taxpayers have sufficient baseless supposition is an affront to the wisdom not only of the legislature
interest in preventing the illegal expenditures of moneys raised by that passed it but also of the executive which approved it.”27 This
taxation and may therefore question the constitutionality of presumption of constitutionality can be overcome only by the clearest
statutes requiring expenditure of public moneys. [11 Am. Jur. 761, showing that there was indeed an infraction of the Constitution, and only
Emphasis supplied.] when such a conclusion is reached by the required majority may the Court
pronounce, in the discharge of the duty it cannot escape, that the
Lastly, the Court is of the view that the petition poses issues impressed challenged act must be struck down.28
with paramount public interest. The ramification of issues involving the
unconstitutional spending of PDAF deserves the consideration of the Court, The petition is miserably wanting in this regard.  LAMP would have the
warranting the assumption of jurisdiction over the petition. Court declare the unconstitutionality of the PDAF’s enforcement based on
the absence of express provision in the GAA allocating PDAF funds to the
Now, on the substantive issue. Members of Congress and the latter’s encroachment on executive power in
proposing and selecting projects to be funded by PDAF.  Regrettably, these
allegations lack substantiation.  No convincing proof was presented appropriation act precisely following the process established by the
showing that, indeed, there were direct releases of funds to the Members Constitution, which specifies that no money may be paid from the Treasury
of Congress, who actually spend them according to their sole discretion.  except in accordance with an appropriation made by law.
Not even a documentation of the disbursement of funds by the DBM in
favor of the Members of Congress was presented by the petitioner to xxx
convince the Court to probe into the truth of their claims. Devoid of any
pertinent evidentiary support that illegal misuse of PDAF in the form of 3. Budget Execution. Tasked on the Executive, the third phase of the
kickbacks has become a common exercise of unscrupulous Members of budget process covers the various operational aspects of budgeting. The
Congress, the Court cannot indulge the petitioner’s request for rejection of establishment of obligation authority ceilings, the evaluation of work and
a law which is outwardly legal and capable of lawful enforcement.  In a financial plans for individual activities, the continuing review of government
case like this, the Court’s hands are tied in deference to the presumption of fiscal position, the regulation of funds releases, the implementation of cash
constitutionality lest the Court commits unpardonable judicial legislation. payment schedules, and other related activities comprise this phase of the
The Court is not endowed with the power of clairvoyance to divine from budget cycle.
scanty allegations in pleadings where justice and truth lie.29  Again,
newspaper or electronic reports showing the appalling effects of PDAF 4. Budget accountability. The fourth phase refers to the evaluation of
cannot be appreciated by the Court, “not because of any issue as to their actual performance and initially approved work targets, obligations
truth, accuracy, or impartiality, but for the simple reason that facts must incurred, personnel hired and work accomplished are compared with the
be established in accordance with the rules of evidence.”30 targets set at the time the agency budgets were approved.

Hence, absent a clear showing that an offense to the principle of separation Under the Constitution, the power of appropriation is vested in the
of powers was committed, much less tolerated by both the Legislative and Legislature, subject to the requirement that appropriation bills originate
Executive, the Court is constrained to hold that a lawful and regular exclusively in the House of Representatives with the option of the Senate
government budgeting and appropriation process ensued during the to propose or concur with amendments.32 While the budgetary process
enactment and all throughout the implementation of the GAA of 2004.  The commences from the proposal submitted by the President to Congress, it is
process was explained in this wise, in Guingona v. Carague:31 the latter which concludes the exercise by crafting an appropriation act it
may deem beneficial to the nation, based on its own judgment, wisdom
1. Budget preparation. The first step is essentially tasked upon the and purposes.  Like any other piece of legislation, the appropriation act
Executive Branch and covers the estimation of government revenues, the may then be susceptible to objection from the branch tasked to implement
determination of budgetary priorities and activities within the constraints it, by way of a Presidential veto.  Thereafter, budget execution comes
imposed by available revenues and by borrowing limits, and the translation under the domain of the Executive branch which deals with the operational
of desired priorities and activities into expenditure levels. aspects of the cycle including the allocation and release of funds earmarked
for various projects.  Simply put, from the regulation of fund releases, the
Budget preparation starts with the budget call issued by the Department of implementation of payment schedules and up to the actual spending of the
Budget and Management. Each agency is required to submit agency budget funds specified in the law, the Executive takes the wheel.  “The DBM lays
estimates in line with the requirements consistent with the general ceilings down the guidelines for the disbursement of the fund. The Members of
set by the Development Budget Coordinating Council (DBCC). Congress are then requested by the President to recommend projects and
programs which may be funded from the PDAF.  The list submitted by the
With regard to debt servicing, the DBCC staff, based on the macro- Members of Congress is endorsed by the Speaker of the House of
economic projections of interest rates (e.g. LIBOR rate) and estimated Representatives to the DBM, which reviews and determines whether such
sources of domestic and foreign financing, estimates debt service levels. list of projects submitted are consistent with the guidelines and the
Upon issuance of budget call, the Bureau of Treasury computes for the priorities set by the Executive.”33  This demonstrates the power given to
interest and principal payments for the year for all direct national the President to execute appropriation laws and therefore, to exercise the
government borrowings and other liabilities assumed by the same. spending per se of the budget.

2. Legislative authorization. -- At this stage, Congress enters the picture As applied to this case, the petition is seriously wanting in establishing that
and deliberates or acts on the budget proposals of the President, and individual Members of Congress receive and thereafter spend funds out of
Congress in the exercise of its own judgment and wisdom formulates an PDAF.  Although the possibility of this unscrupulous practice cannot be
entirely discounted, surmises and conjectures are not sufficient bases for
the Court to strike down the practice for being offensive to the
Constitution. Moreover, the authority granted the Members of Congress to
propose and select projects was already upheld in Philconsa.  This remains
as valid case law.  The Court sees no need to review or reverse the
standing pronouncements in the said case.  So long as there is no showing
of a direct participation of legislators in the actual spending of the budget,
the constitutional boundaries between the Executive and the Legislative in
the budgetary process remain intact.

While the Court is not unaware of the yoke caused by graft and corruption,
the evils propagated by a piece of valid legislation cannot be used as a tool
to overstep constitutional limits and arbitrarily annul acts of Congress. 
Again, “all presumptions are indulged in favor of constitutionality; one who
attacks a statute, alleging unconstitutionality must prove its invalidity
beyond a reasonable doubt; that a law may work hardship does not render
it unconstitutional; that if any reasonable basis may be conceived which
supports the statute, it will be upheld, and the challenger must negate all
possible bases; that the courts are not concerned with the wisdom, justice,
policy, or expediency of a statute; and that a liberal interpretation of the
constitution in favor of the constitutionality of legislation should be
adopted.”34cralaw

There can be no question as to the patriotism and good motive of the


petitioner in filing this petition. Unfortunately, the petition must fail based
on the foregoing reasons.

WHEREFORE, the petition is DISMISSED without pronouncement as to


costs.

G.R. No. L-33713 July 30, 1975

EUSEBIO B. GARCIA, petitioner-appellant, vs. HON. ERNESTO S. MATA,


Secretary of National Defense, and GENERAL MANUEL T. YAN, Chief
of Staff, Armed Forces of the Philippines, Respondents-Appellees.

This is a petition for certiorari to review the decision of the Court of First


Instance of Quezon City, Branch IX, in civil case Q-13466, entitled "Eusebio
B. Garcia, petitioner, versus Hon. Ernesto Mata (Juan Ponce Enrile), et al.,
respondents," declaring paragraph 11 of the "Special Provisions for the
Armed Forces of the Philippines" of Republic Act No. 16001unconstitutional
and therefore invalid and inoperative.chanroblesvirtualawlibrarychanrobles
virtual law library

We affirm the judgment a quo.chanroblesvirtualawlibrarychanrobles virtual


law library
The facts material to this case are embodied in the following stipulation 1600 is "invalid, unconstitutional and inoperative."chanrobles virtual law
submitted jointly by both parties to the lower court: library

Petitioner was a reserve officer on active duty with the Armed Forces of the The petitioner had a total of 9 years, 4 months and 12 days of accumulated
Philippines until his reversion to inactive status on 15 November 1960, active commissioned service in the AFP when Republic Act 1382 took effect
pursuant to the provisions of Republic Act No. 2332. At the time of on June 18, 1955. Section I of this law provided:
reversion, Petitioner held the rank of Captain with a monthly emolument of
P478.00, comprising his base and longevity pay, quarters and subsistence
Reserve officers with at least ten years of active accumulated
allowances;chanrobles virtual law library
commissioned service who are still on active duty at the time of the
approval of this Act shall not be reverted into inactive status except for
On June 18, 1955, the date when Republic Act No. 1382 took effect, cause after proper court-martial proceedings or upon their own
petitioner had a total of 9 years, 4 months and 12 days of accumulated request: Provided, That for purposes of computing the length of service, six
active commissioned service in the Armed Forces of the months or more of active service shall be considered one year. (emphasis
Philippines;chanrobles virtual law library supplied)

On July 11, 1956, the date when Republic Act 1600 took effect, petitioner The petitioner's accumulated active commissioned service was thus short of
had an accumulated active commissioned service of 10 years, 5 months the minimum service requirement prescribed in the aforequoted provision
and 5 days in the Armed Forces of the Philippines;chanrobles virtual law of R.A. 1382.chanroblesvirtualawlibrarychanrobles virtual law library
library
On July 11, 1956, 3while the petitioner was yet in the active service,
Petitioner's reversion to inactive status on 15 November 1960 was Republic Act 1600 was enacted into law. Paragraph 11 of the SPECIAL
pursuant to the provisions of Republic Act 2334, and such reversion was PROVISIONS FOR THE ARMED FORCES OF THE PHILIPPINES (on page 892
neither for cause, at his own request, nor after court-martial of the Act) provided as follows: 
proceedings;chanrobles virtual law library
11. After the approval of this Act, and when there is no emergency, no
From 15 November 1960 up to the present, petitioner has been on inactive reserve officer of the Armed Forces of the Philippines may be called to a
status and as such, he has neither received any emoluments from the tour of active duty for more than two years during any period of five
Armed Forces of the Philippines, nor was he ever employed in the consecutive years: PROVIDED, That hereafter reserve officers of the Armed
Government in any capacity;chanrobles virtual law library Forces of the Philippines on active duty for more than two years on the
date of the approval of this Act except those whose military and
educational training, experience and qualifications are deemed essential to
As a consequence of his reversion to inactive status, petitioner filed the
the needs of the service, shall be reverted to inactive status within one
necessary petitions with the offices of the AFP Chief of Staff, the Secretary
year from the approval of this Act: PROVIDED, FURTHER, That reserve
of National Defense, and the President, respectively, but received reply
officers with at least ten years of active accumulated commissioned service
only from the Chief of Staff through the AFP Adjutant General.
who are still on active duty at the time of the approval of this Act shall not
be reverted to inactive status except for cause after proper court-martial
On September 17, 1969 the petitioner brought an action for proceedings or upon their request; PROVIDED, FURTHER, That any such
"Mandamus and Recovery of a Sum of Money" in the court a quo to compel reserve officer reverted to inactive status who has at least five of active
the respondents Secretary of National Defense and Chief of Staff of the commissioned service shall be entitled to a gratuity equivalent to one
Armed Forces of the Philippines 2to reinstate him in the active month's authorized base and longevity pay in the rank held at the time of
commissioned service of the Armed Forces of the Philippines, to readjust such reversion for every year of active commissioned service; PROVIDED,
his rank, and to pay all the emoluments and allowances due to him from FURTHER, That any reserve officer who receives a gratuity under the
the time of his reversion to inactive status. On December 2, 1970 the trial provisions of this Act shall not except during a National emergency or
court dismissed the petition. The court ruled that paragraph 11 of the mobilization, be called to a tour of active duty within five years from the
"Special Provisions for the Armed Forces of the Philippines" in Republic Act date of reversion: PROVIDED, FURTHER, That the Secretary of National
Defense is authorized to extend the tour of active duty of reserve officers In the language of the respondents-appellees, "it was indeed a non-
who are qualified military pilots and doctors; PROVIDED, FURTHER, That appropriation item inserted in an appropriation measure in violation of the
any savings in the appropriations authorized in this Act for the Department constitutional inhibition against "riders" to the general appropriation act." It
of National Defense notwithstanding any provision of this Act to the was indeed a new and completely unrelated provision attached to the
contrary and any unexpended balance of certification to accounts payable Appropriation Act.chanroblesvirtualawlibrarychanrobles virtual law library
since 1 July 1949 regardless of purpose of the appropriation shall be made
available for the purpose of this paragraph: AND PROVIDED, FINALLY, That
The paragraph in question also violated Art. VI, Sec. 21, par. 1 5of the 1935
the Secretary of National Defense shall render a quarterly report to
Constitution of the Philippines which provided that "No bill which may be
Congress as to the implementation of the provisions of this paragraph.
enacted into law shall embrace more than one subject which shall be
( pp. 892-893, RA 1600) (emphasis supplied)
expressed in the title of the bill." This constitutional requirement nullified
and rendered inoperative any provision contained in the body of an act that
The petitioner consequently argues that his reversion to inactive status on was not fairly included in the subject expressed in the title or was not
November 15, 1960 was in violation of the abovequoted provision which germane to or properly connected with that
prohibits the reversion to inactive status of reserve officers on active duty subject.chanroblesvirtualawlibrarychanrobles virtual law library
with at least ten years of accumulated active commissioned
service.chanroblesvirtualawlibrarychanrobles virtual law library
In determining whether a provision contained in an act is embraced in the
subject and is properly connected therewith, the subject to be considered is
On the other hand, the respondents contend that the said provision has no the one expressed in the title of the act, and every fair intendment and
relevance or pertinence whatsoever to the budget in question or to any reasonable doubt should be indulged in favor of the validity of the
appropriation item contained therein, and is therefore proscribed by Art. legislative enactment. But when an act contains provisions which are
VI, Sec. 19, par. 24of the 1935 Constitution of the Philippines, which reads: clearly not embraced in the subject of the act, as expressed in the title,
such provisions are inoperative and without
effect.chanroblesvirtualawlibrarychanrobles virtual law library
No provision or enactment shall be embraced in the general appropriation
bill unless it relates specifically to some particular appropriation therein;
and any such provision or enactment shall be limited in its operation to We are mindful that the title of an act is not required to be an index to the
such appropriation. body of the act. Thus, in Sumulong vs. Comelec, 73 Phil. 288, 291, this
Court held that it is "a sufficient compliance with such requirement if the
title expresses the general subject and all the provisions of the statute are
A perusal of the challenged provision of R.A. 1600 fails to disclose its
germane to that general subject." The constitutional provision was
relevance or relation to any appropriation item therein, or to the
intended to preclude the insertion of riders in legislation, a rider being a
Appropriation Act as a whole. From the very first clause of paragraph 11
provision not germane to the subject-matter of the bill. 6chanrobles virtual
itself, which reads,
law library

After the approval of this Act, and when there is no emergency, no reserve
The subject of R.A. 1600, as expressed in its title, is restricted to
officer of the Armed Forces of the Philippines may be called to a tour of
"appropriating funds for the operation of the government." Any provision
active duty for more than two years during any period of five consecutive
contained in the body of the act that is fairly included in this restricted
years:
subject or any matter properly connected therewith is valid and operative.
But, if a provision in the body of the act is not fairly included in this
the incongruity and irrelevancy are already evident. While R.A. 1600 restricted subject, like the provision relating to the policy matters of calling
appropriated money for the operation of the Government for the fiscal year to active duty and reversion to inactive duty of reserve officers of the AFP,
1956-1957, the said paragraph 11 refers to the fundamental government such provision is inoperative and of no
policy matters of the calling to active duty and the reversion to inactive effect.chanroblesvirtualawlibrarychanrobles virtual law library
status of reserve officers in the AFP. The incongruity and irrelevancy
continue throughout the entire
To quote the respondents-appellees on this point:
paragraph.chanroblesvirtualawlibrarychanrobles virtual law library
It is obvious that the statutory provision in question refers to security of
reserve officers from reversion to inactive status, whereas the subject or
title of the statute from which it derives its existence refers to
appropriations. Verily, it runs contrary to or is repugnant to the above-
quoted injunctive provision of the Constitution. Where a conflict arises
between a statute and the Constitution, the latter prevails. It should be
emphasized that a Constitution is superior to a statute and is precisely
called the "supreme law of the land" because it is the fundamental or
organic law which states the general principles and builds the substantial
foundation and general framework of law and government, and for that
reason a statute contrary to or in violation of the Constitution is null and
void (Talabon vs. Iloilo Provincial Warden, 78 Phil. 599). If a law, therefore,
happens to infringe upon or violate the fundamental law, courts of justice
may step in to nullify its effectiveness (Mabanag vs. Lopez Vito, 78 Phil. 1).

Upon the foregoing dissertation, we declare Paragraph 11 of the SPECIAL


PROVISIONS FOR THE ARMED FORCES OF THE PHILIPPINES as
unconstitutional, invalid and inoperative. Being unconstitutional, it confers
no right and affords no protection. In legal contemplation it is as though it
has never been passed. 7chanrobles virtual law library

Verily, not having shown a clear legal right to the position to which he
desires to be restored, the petitioner cannot compel the respondents to
reinstate and/or call him to active duty, promote or readjust his rank,
much less pay him back emoluments and
allowances.chanroblesvirtualawlibrarychanrobles virtual law library

ACCORDINGLY, the instant petition is denied, and the decision of the lower
court dismissing the complaint is hereby affirmed. No pronouncement as to DEMETRIO G. DEMETRIA, M.P., AUGUSTO S. SANCHEZ, M.P.,
costs. ORLANDO S. MERCADO, M.P., HONORATO Y. AQUINO, M.P., ZAFIRO
L. RESPICIO, M.P., DOUGLAS R. CAGAS, M.P., OSCAR F. SANTOS,
M.P., ALBERTO G. ROMULO, M.P., CIRIACO R. ALFELOR, M.P.,
ISIDORO E. REAL, M.P., EMIGDIO L. LINGAD, M.P., ROLANDO C.
MARCIAL, M.P., PEDRO M. MARCELLANA, M.P., VICTOR S. ZIGA,
M.P., and ROGELIO V. GARCIA, M.P., Petitioners, v. HON. MANUEL
ALBA in his capacity as the MINISTER OF THE BUDGET and VICTOR
MACALINGCAG in his capacity as the TREASURER OF THE
PHILIPPINES, Respondents.

SYLLABUS

1. REMEDIAL LAW; CIVIL PROCEDURE; PROPER PARTY; ISSUE OF


CONSTITUTIONALITY OF STATUTES MAY BE RAISED AT THE INSTANCE OF
A TAXPAYER. — The case of Pascual v. Secretary of Public Works, Et Al., government against a coordinate branch to enjoin the performance of
110 Phil. 331 is authority in support of petitioners’ locus standi. Thus: duties within the latter’s sphere of responsibility. where the legislature or
"Again, it is well-settled that the validity of a statute may be contested only the executive branch is acting within the limits of its authority, the judiciary
by one who will sustain a direct injury in consequence of its enforcement. cannot and ought not to interfere with the former, But where the
Yet, there are many decisions nullifying at the instance of taxpayers, laws legislature or the executive acts beyond the scope of its constitutional
providing for the disbursement of public funds, upon the theory that the power, it becomes the duty of the judiciary to declare what the other
expenditure of public funds by an officer of the state for the purpose of branches of the government had assumed to do as void. This is the
administering an unconstitutional act constitute a misapplication of such essence of judicial power conferred by the Constitution "in one Supreme
funds’ which may be enjoined at the request of a taxpayer. Moreover, in Court and in such lower courts as may be established by law" [Art. VIII,
Tan v. Macapagal, 43 SCRA 677 and Sanidad v. Comelec, 73 SCRA 333, we Section 1 of the 1935 Constitution; Art. X, Section 1 of the 1973
said that as regards taxpayers’ suits, this Court enjoys that open discretion Constitution and which was adopted as part of the Freedom Constitution]
to entertain the same or not. and Art. VIII, Section 1 of the 1987 Constitution] and which power this
Court has exercised in many instances. Public respondents are being
2. CONSTITUTIONAL LAW; NATIONAL ASSEMBLY; TRANSFER TO enjoined from acting under a provision of law which we have earlier
APPROPRIATION; LIMITATIONS. — The prohibition to transfer an mentioned to be constitutionally infirm. The general principle relied upon
appropriation for one item to another was explicit and categorical under the cannot therefore accord them the protection sought as they are not acting
1973 Constitution. However, to afford the heads of the different branches within their "sphere of responsibility" but without it.
of the government and those of the constitutional commissions
considerable flexibility in the use of public funds and resources, the
constitution allowed the enactment of a law authorizing the transfer of
DECISION
funds for the purpose of augmenting an item from savings in another item
in the appropriation of the government branch on constitutional body
concerned. The leeway granted was thus limited. Transferred were
specified, i.e. transfer may be allowed for the purpose of augmenting an FERNAN, J.:
item and such transfer may be allowed for the purpose of augmenting an
item and such transfer may be made only if there are savings form another
item in the appropriation of the government branch or constitutional body.
Assailed in this petition for prohibition with prayer for a writ of preliminary
injunction is the constitutionality of the first paragraph of Section 44 of
3. ID.; PAR. 1, SEC. 44 OF PRESIDENTIAL DECREE NO. 1177
Presidential Decree No. 1177, otherwise known as the "Budget Reform
EMPOWERING THE PRESIDENT TO INDISCRIMINATELY TRANSFER FUNDS
Decree of 1977."cralaw virtua1aw library
DECLARED UNCONSTITUTIONAL. — Paragraph 1 of Section 44 of P.D. 1177
unduly over-extends the privilege granted under said Section 16 [5]. It
Petitioners, who filed the instant petition as concerned citizens of this
empowers the President to indiscriminately transfer of funds form one
country, as members of the National Assembly/Batasan Pambansa
department, bureau, office or agency of the Executive Department to any
representing their millions of constituents, as parties with general interest
program, project or activity of any department, bureau or office included in
common to all the people of the Philippines, and as taxpayers whose vital
the General Appropriations Act or approved after its enactment, without
interests may be affected by the outcome of the reliefs prayed for" 1 listed
regard as to whether or not funds to be transferred are actually savings in
the grounds relied upon in this petition as follows:chanrobles law library :
the item from which the same are to be taken, or whether or not the
red
transfer is for the purpose of augmenting the item to which said transfer is
to be made. It does not only completely disregard the standards set in the
"A. SECTION 44 OF THE ‘BUDGET REFORM DECREE OF 1977’ INFRINGES
fundamental law, thereby amounting to an undue delegation of legislative
UPON THE FUNDAMENTAL LAW BY AUTHORIZING THE ILLEGAL TRANSFER
powers, but likewise goes beyond the tenor thereof. Indeed, such
OF PUBLIC MONEYS.
constitutional infirmities render the provision in question null and void.
"B. SECTION 44 OF PRESIDENTIAL DECREE NO. 1177 IS REPUGNANT TO
4. ID.; SUPREME COURT; MAY ISSUE WRIT OF PROHIBITION AGAINST A
THE CONSTITUTION AS IT FAILS TO SPECIFY THE OBJECTIVES AND
COORDINATE BRANCH ACTING BEYOND THE SCOPE OF ITS
PURPOSES FOR WHICH THE PROPOSED TRANSFER OF FUNDS ARE TO BE
CONSTITUTIONAL POWERS. — Another theory advanced by public
MADE.
respondents is that prohibition will not lie form one branch of the
simplistic and tolerant pretext that the case has become moot and
"C. SECTION 44 OF PRESIDENTIAL DECREE NO. 1177 ALLOWS THE academic.
PRESIDENT TO OVERRIDE THE SAFEGUARDS, FORM AND PROCEDURE
PRESCRIBED BY THE CONSTITUTION IN APPROVING APPROPRIATIONS. "The Supreme Court is not only the highest arbiter of legal questions but
also the conscience of the government. The citizen comes to us in quest of
"D. SECTION 44 OF THE SAME DECREE AMOUNTS TO AN UNDUE law but we must also give him justice. The two are not always the same.
DELEGATION OF LEGISLATIVE POWERS TO THE EXECUTIVE. There are times when we cannot grant the latter because the issue has
been settled and decision is no longer possible according to the law. But
"E. THE THREATENED AND CONTINUING TRANSFER OF FUNDS BY THE there are also times when although the dispute has disappeared, as in this
PRESIDENT AND THE IMPLEMENTATION THEREOF BY THE BUDGET case, it nevertheless cries out to be resolved. Justice demands that we act
MINISTER AND THE TREASURER OF THE PHILIPPINES ARE WITHOUT OR IN then, not only for the vindication of the outraged right, though gone, but
EXCESS OF THEIR AUTHORITY AND JURISDICTION." 2  also for the guidance of and as a restraint upon the future."cralaw
virtua1aw library
Commenting on the petition in compliance with the Court resolution dated
September 19, 1985, the Solicitor General, for the public respondents, It is in the discharge of our role in society, as above-quoted, as well as to
questioned the legal standing of petitioners, who were allegedly merely avoid great disservice to national interest that We take cognizance of this
begging an advisory opinion from the Court, there being no justiciable petition and thus deny public respondents’ motion to dismiss. Likewise
controversy fit for resolution or determination. He further contended that noteworthy is the fact that the new Constitution, ratified by the Filipino
the provision under consideration was enacted pursuant to Section 16[5], people in the plebiscite held on February 2, 1987, carries verbatim section
Article VIII of the 1973 Constitution; and that at any rate, prohibition will 16[5], Article VIII of the 1973 Constitution under Section 24[5], Article VI.
not lie from one branch of the government to a coordinate branch to enjoin And while Congress has not officially reconvened, We see no cogent reason
the performance of duties within the latter’s sphere of responsibility. for further delaying the resolution of the case at bar.

On February 27, 1986, the Court required the petitioners to file a Reply to The exception taken to petitioners’ legal standing deserves scant
the Comment. This, they did, stating, among others, that as a result of the consideration. The case of Pascual v. Secretary of Public Works, Et Al., 110
change in the administration, there is a need to hold the resolution of the Phil. 331, is authority in support of petitioners’ locus standi.
present case in abeyance "until developments arise to enable the parties to Thus:jgc:chanrobles.com.ph
concretize their respective stands." 3 
"Again, it is well-settled that the validity of a statute may be contested only
Thereafter, We required public respondents to file a rejoinder. The Solicitor by one who will sustain a direct injury in consequence of its enforcement.
General filed a rejoinder with a motion to dismiss, setting forth as grounds Yet, there are many decisions nullifying at the instance of taxpayers, laws
therefor the abrogation of Section 16[5], Article VIII of the 1973 providing for the disbursement of public funds, upon the theory that ‘the
Constitution by the Freedom Constitution of March 25, 1986, which has expenditure of public funds by an officer of the state for the purpose of
allegedly rendered the instant petition moot and academic. He likewise administering an unconstitutional act constitutes a misapplication of such
cited the "seven pillars" enunciated by Justice Brandeis in Ashwander v. funds which may be enjoined at the request of a taxpayer. Although there
TVA, 297 U.S. 288 (1936) 4 as basis for the petition’s dismissal.cralawnad are some decisions to the contrary, the prevailing view in the United States
is stated in the American Jurisprudence as follows:chanrob1es virtual 1aw
In the case of Evelio B. Javier v. The Commission on Elections and Arturo library
F. Pacificador, G.R. Nos. 68379-81, September 22, 1986, We stated
that:jgc:chanrobles.com.ph ‘In the determination of the degree of interest essential to give the
requisite standing to attack the constitutionality of a statute, the general
"The abolition of the Batasang Pambansa and the disappearance of the rule is that not only persons individually affected, but also taxpayers have
office in dispute between the petitioner and the private respondents — both sufficient interest in preventing the illegal expenditures of moneys raised
of whom have gone their separate ways-could be a convenient justification by taxation and may therefore question the constitutionality of statutes
for dismissing the case. But there are larger issues involved that must be requiring expenditure of public moneys. [11 Am. Jur. 761, Emphasis
resolved now, once and for all, not only to dispel the legal ambiguities here supplied.]’" 
raised. The more important purpose is to manifest in the clearest possible
terms that this Court will not disregard and in effect condone wrong on the Moreover, in Tan v. Macapagal, 43 SCRA 677 and Sanidad v. Comelec, 73
SCRA 333. We said that as regards taxpayers’ suits, this Court enjoys that amounting to an undue delegation of legislative powers, but likewise goes
open discretion to entertain the same or not.chanrobles law library beyond the tenor thereof. Indeed, such constitutional infirmities render the
provision in question null and void.
The conflict between paragraph 1 of Section 44 of Presidential-Decree No.
1177 and Section 16[5], Article VIII of the 1973 Constitution is readily "For the love of money is the root of all evil: . . ." and money belonging to
perceivable from a mere cursory reading thereof. Said paragraph 1 of no one in particular, i.e. public funds, provide an even greater temptation
Section 44 provides:jgc:chanrobles.com.ph for misappropriation and embezzlement. This, evidently, was foremost in
the minds of the framers of the constitution in meticulously prescribing the
"The President shall have the authority to transfer any fund, appropriated rules regarding the appropriation and disposition of public funds as
for the different departments, bureaus, offices and agencies of the embodied in Sections 16 and 18 of Article VIII of the 1973 Constitution.
Executive Department, which are included in the General Appropriations Hence, the conditions on the release of money from the treasury [Sec.
Act, to any program, project or activity of any department, bureau, or 18(1)]; the restrictions on the use of public funds for public purpose [Sec.
office included in the General Appropriations Act or approved after its 18(2)]; the prohibition to transfer an appropriation for an item to another
enactment."cralaw virtua1aw library [Sec. 16(5) and the requirement of specifications [Sec. 16(2)], among
others, were all safeguards designed to forestall abuses in the expenditure
On the other hand, the constitutional provision under consideration reads of public funds. Paragraph 1 of Section 44 puts all these safeguards to
as follows:jgc:chanrobles.com.ph naught. For, as correctly observed by petitioners, in view of the unlimited
authority bestowed upon the President,." . . Pres. Decree No. 1177 opens
"Sec. 16[5]. No law shall be passed authorizing any transfer of the floodgates for the enactment of unfounded appropriations, results in
appropriations, however, the President, the Prime Minister, the Speaker, uncontrolled executive expenditures, diffuses accountability for budgetary
the Chief Justice of the Supreme Court, and the heads of constitutional performance and entrenches the pork barrel system as the ruling party
commissions may by law be authorized to augment any item in the general may well expand [sic] public money not on the basis of development
appropriations law for their respective offices from savings in other items of priorities but on political and personal expediency." 5 The contention of
their respective appropriations."cralaw virtua1aw library public respondents that paragraph 1 of Section 44 of P.D. 1177 was
enacted pursuant to Section 16(5) of Article VIII of the 1973 Constitution
The prohibition to transfer an appropriation for one item to another was must perforce fall flat on its face.chanrobles virtual lawlibrary
explicit and categorical under the 1973 Constitution. However, to afford the
heads of the different branches of the government and those of the Another theory advanced by public respondents is that prohibition will not
constitutional commissions considerable flexibility in the use of public funds lie from one branch of the government against a coordinate branch to
and resources, the constitution allowed the enactment of a law authorizing enjoin the performance of duties within the latter’s sphere of responsibility.
the transfer of funds for the purpose of augmenting an item from savings
in another item in the appropriation of the government branch or Thomas M. Cooley in his "A Treatise on the Constitutional Limitations," Vol.
constitutional body concerned. The leeway granted was thus limited. The I, Eight Edition, Little, Brown and Company, Boston,
purpose and conditions for which funds may be transferred were specified, explained:jgc:chanrobles.com.ph
i.e. transfer may be allowed for the purpose of augmenting an item and
such transfer may be made only if there are savings from another item in ". . . The legislative and judicial are coordinate departments of the
the appropriation of the government branch or constitutional body. government, of equal dignity; each is alike supreme in the exercise of its
proper functions, and cannot directly or indirectly, while acting within the
Paragraph 1 of Section 44 of P.D. No. 1177 unduly overextends the limits of its authority, be subjected to the control or supervision of the
privilege granted under said Section 16[5]. It empowers the President to other, without an unwarrantable assumption by that other of power which,
indiscriminately transfer funds from one department, bureau, office or by the Constitution, is not conferred upon it. The Constitution apportions
agency of the Executive Department to any program, project or activity of the powers of government, but it does not make any one of the three
any department, bureau or office included in the General Appropriations departments subordinate to another, when exercising the trust committed
Act or approved after its enactment, without regard as to whether or not to it. The courts may declare legislative enactments unconstitutional and
the funds to be transferred are actually savings in the item from which the void in some cases, but not because the judicial power is superior in degree
same are to be taken, or whether or not the transfer is for the purpose of or dignity to the legislative. Being required to declare what the law is in the
augmenting the item to which said transfer is to be made. It does not only cases which come before them, they must enforce the Constitution, as the
completely disregard the standards set in the fundamental law, thereby paramount law, whenever a legislative enactment comes in conflict with it.
But the courts sit, not to review or revise the legislative action, but to
enforce the legislative will, and it is only where they find that the
legislature has failed to keep within its constitutional limits, that they are at
liberty to disregard its action; and in doing so, they only do what every
private citizen may do in respect to the mandates of the courts when the
judges assume to act and to render judgments or decrees without
jurisdiction.’In exercising this high authority, the judges claim no judicial
supremacy; they are only the administrators of the public will. If an act of
the legislature is held void, it is not because the judges have any control
over the legislative power, but because the act is forbidden by the
Constitution, and because the will of the people, which is therein declared,
is paramount to that of their representatives expressed in any law.’
[Lindsay v. Commissioners, & c., 2 Bay, 38, 61; People v. Rucker, 5 Col. 5;
Russ v. Com., 210 Pa. St. 544; 60 Atl. 169, 1 L.R.A. [N.S.] 409, 105 Am.
St. Rep. 825]" (pp. 332-334).

Indeed, where the legislature or the executive branch is acting within the
limits of its authority, the judiciary cannot and ought not to interfere with
the former. But where the legislature or the executive acts beyond the
scope of its constitutional powers, it becomes the duty of the judiciary to
declare what the other branches of the government had assumed to do as
void. This is the essence of judicial power conferred by the Constitution "in
one Supreme Court and in such lower courts as may be established by law"
[Art. VIII, Section 1 of the 1935 Constitution; Art. X, Section 1 of the 1973
Constitution and which was adopted as part of the Freedom Constitution,
and Art. VIII, Section 1 of the 1987 Constitutional and which power this
Court has exercised in many instances. ** 

Public respondents are being enjoined from acting under a provision of law
which We have earlier mentioned to be constitutionally infirm. The general
principle relied upon cannot therefore accord them the protection sought as
they are not acting within their "sphere of responsibility" but without it.

The nation has not recovered from the shock, and worst, the economic
destitution brought about by the plundering of the Treasury by the deposed
PHILIPPINE CONSTITUTION ASSOCIATION, EXEQUIEL B. GARCIA
dictator and his cohorts. A provision which allows even the slightest
and A. GONZALES, Petitioners, v. HON. SALVADOR ENRIQUEZ, as
possibility of a repetition of this sad experience cannot remain written in
Secretary of Budget and Management; HON. VICENTE T. TAN, as
our statute books.
National Treasurer and COMMISSION ON AUDIT, Respondents.
WHEREFORE, the instant petition is granted. Paragraph 1 of Section 44 of
Presidential Decree No. 1177 is hereby declared null and void for being QUIASON, J.:
unconstitutional.ch
Once again this Court is called upon to rule on the conflicting claims of
authority between the Legislative and the Executive in the clash of the
powers of the purse and the sword. Providing the focus for the contest
between the President and the Congress over control of the national
budget are the four cases at bench. Judicial intervention is being sought by
a group of concerned taxpayers on the claim that Congress and the In G.R. No. 113105, the Philippine Constitution Association, Exequiel B.
President have impermissibly exceeded their respective authorities, and by Garcia and Ramon A. Gonzales as taxpayers, prayed for a writ of
several Senators on the claim that the President has committed grave prohibition to declare as unconstitutional and void: (a) Article XLI on the
abuse of discretion or acted without jurisdiction in the exercise of his veto Countrywide Development Fund, the special provision in Article I entitled
power. Realignment of Allocation for Operational Expenses, and Article XLVIII on
the Appropriation for Debt Service or the amount appropriated under said
Article XLVIII in excess of the P37.9 Billion allocated for the Department of
Ichanrobles virtual law library
Education, Culture and Sports; and (b) the veto of the President of the
Special Provision of 
House Bill No. 10900, the General Appropriation Bill of 1994 (GAB of Article XLVIII of the GAA of 1994 (Rollo, pp. 88-90, 104-105)chanrobles
1994), was passed and approved by both houses of Congress on December virtual law library
17, 1993. As passed, it imposed conditions and limitations on certain items
of appropriations in the proposed budget previously submitted by the
In G.R. No. 113174, sixteen members of the Senate led by Senate
President. It also authorized members of Congress to propose and identify
President Edgardo J. Angara, Senator Neptali A. Gonzales, the Chairman of
projects in the "pork barrels" allotted to them and to realign their
the Committee on Finance, and Senator Raul S. Roco, sought the issuance
respective operating budgets.chanroblesvirtualawlibrarychanrobles virtual
of the writs of certiorari, prohibition and mandamus against the Executive
law library
Secretary, the Secretary of the Department of Budget and Management,
and the National Treasurer.chanroblesvirtualawlibrarychanrobles virtual law
Pursuant to the procedure on the passage and enactment of bills as library
prescribed by the Constitution, Congress presented the said bill to the
President for consideration and
Suing as members of the Senate and taxpayers, petitioners question: (1)
approval.chanroblesvirtualawlibrarychanrobles virtual law library
the constitutionality of the conditions imposed by the President in the items
of the GAA of 1994: (a) for the Supreme Court, (b) Commission on Audit
On December 30, 1993, the President signed the bill into law, and declared (COA), (c) Ombudsman, (d) Commission on Human Rights (CHR), (e)
the same to have become Republic Act No. 7663, entitled "AN ACT Citizen Armed Forces Geographical Units (CAFGU'S) and (f) State
APPROPRIATING FUNDS FOR THE OPERATION OF THE GOVERNMENT OF Universities and Colleges (SUC's); and (2) the constitutionality of the veto
THE PHILIPPINES FROM JANUARY ONE TO DECEMBER THIRTY ONE, of the special provision in the appropriation for debt
NINETEEN HUNDRED AND NINETY-FOUR, AND FOR OTHER PURPOSES" service.chanroblesvirtualawlibrarychanrobles virtual law library
(GAA of 1994). On the same day, the President delivered his Presidential
Veto Message, specifying the provisions of the bill he vetoed and on which
In G.R. No. 113766, Senators Alberto G. Romulo and Wigberto Tañada (a
he imposed certain conditions.chanroblesvirtualawlibrarychanrobles virtual
co-petitioner in G.R. No. 113174), together with the Freedom from Debt
law library
Coalition, a non-stock domestic corporation, sought the issuance of the
writs of prohibition and mandamus against the Executive Secretary, the
No step was taken in either House of Congress to override the Secretary of the Department of Budget and Management, the National
vetoes.chanroblesvirtualawlibrarychanrobles virtual law library Treasurer, and the COA.chanroblesvirtualawlibrarychanrobles virtual law
library

Petitioners Tañada and Romulo sued as members of the Philippine Senate


and taxpayers, while petitioner Freedom from Debt Coalition sued as a
taxpayer. They challenge the constitutionality of the Presidential veto of
the special provision in the appropriations for debt service and the
automatic appropriation of funds
therefor.chanroblesvirtualawlibrarychanrobles virtual law library
In G.R. No. 11388, Senators Tañada and Romulo sought the issuance of The legal standing of the Senate, as an institution, was recognized
the writs of prohibition and mandamus against the same respondents in in Gonzales v. Macaraig, Jr., 191 SCRA 452 (1990). In said case, 23
G.R. No. 113766. In this petition, petitioners contest the constitutionality Senators, comprising the entire membership of the Upper House of
of: (1) the veto on four special provision added to items in the GAA of 1994 Congress, filed a petition to nullify the presidential veto of Section 55 of the
for the Armed Forces of the Philippines (AFP) and the Department of Public GAA of 1989. The filing of the suit was authorized by Senate Resolution No.
Works and Highways (DPWH); and (2) the conditions imposed by the 381, adopted on February 2, 1989, and which reads as follows: 
President in the implementation of certain appropriations for the CAFGU's,
the DPWH, and the National Housing Authority
Authorizing and Directing the Committee on Finance to Bring in the Name
(NHA).chanroblesvirtualawlibrarychanrobles virtual law library
of the Senate of the Philippines the Proper Suit with the Supreme Court of
the Philippines contesting the Constitutionality of the Veto by the President
Petitioners also sought the issuance of temporary restraining orders to of Special and General Provisions, particularly Section 55, of the General
enjoin respondents Secretary of Budget and Management, National Appropriation Bill of 1989 (H.B. No. 19186) and For Other Purposes.
Treasurer and COA from enforcing the questioned provisions of the GAA of
1994, but the Court declined to grant said provisional reliefs on the time-
In the United States, the legal standing of a House of Congress to sue has
honored principle of according the presumption of validity to statutes and
been recognized (United States v. American Tel. & Tel. Co., 551 F. 2d 384,
the presumption of regularity to official
391 [1976]; Notes: Congressional Access To The Federal Courts, 90
acts.chanroblesvirtualawlibrarychanrobles virtual law library
Harvard Law Review 1632 [1977]).chanroblesvirtualawlibrarychanrobles
virtual law library
In view of the importance and novelty of most of the issues raised in the
four petitions, the Court invited former Chief Justice Enrique M. Fernando
While the petition in G.R. No. 113174 was filed by 16 Senators, including
and former Associate Justice Irene Cortes to submit their respective
the Senate President and the Chairman of the Committee on Finance, the
memoranda as Amicus curiae, which they graciously did.
suit was not authorized by the Senate itself. Likewise, the petitions in 
G.R. Nos. 113766 and 113888 were filed without an enabling resolution for
II the purpose.chanroblesvirtualawlibrarychanrobles virtual law library

Locus Standi Therefore, the question of the legal standing of petitioners in the three
cases becomes a preliminary issue before this Court can inquire into the
validity of the presidential veto and the conditions for the implementation
When issues of constitutionality are raised, the Court can exercise its
of some items in the GAA of 1994.chanroblesvirtualawlibrarychanrobles
power of judicial review only if the following requisites are compresent: (1)
virtual law library
the existence of an actual and appropriate case; (2) a personal and
substantial interest of the party raising the constitutional question; (3) the
exercise of judicial review is pleaded at the earliest opportunity; and (4) We rule that a member of the Senate, and of the House of Representatives
the constitutional question is the lis mota of the case (Luz Farms v. for that matter, has the legal standing to question the validity of a
Secretary of the Department of Agrarian Reform, 192 SCRA 51 [1990]; presidential veto or a condition imposed on an item in an appropriation
Dumlao v. Commission on Elections, 95 SCRA 392 [1980]; People v. Vera, bill.chanroblesvirtualawlibrarychanrobles virtual law library
65 Phil. 56 [1937]).chanroblesvirtualawlibrarychanrobles virtual law library
Where the veto is claimed to have been made without or in excess of the
While the Solicitor General did not question the locus standiof petitioners in authority vested on the President by the Constitution, the issue of an
G.R. No. 113105, he claimed that the remedy of the Senators in the other impermissible intrusion of the Executive into the domain of the Legislature
petitions is political (i.e., to override the vetoes) in effect saying that they arises (Notes: Congressional Standing To Challenge Executive Action, 122
do not have the requisite legal standing to bring the University of Pennsylvania Law Review 1366
suits.chanroblesvirtualawlibrarychanrobles virtual law library [1974]).chanroblesvirtualawlibrarychanrobles virtual law library
To the extent the power of Congress are impaired, so is the power of each For Fund requirements of countrywide 
member thereof, since his office confers a right to participate in the development projects P 2,977,000,000
exercise of the powers of that institution (Coleman v. Miller, 307 U.S. 433 -------
[1939]; Holtzman v. Schlesinger, 484 F. 2d 1307
[1973]).chanroblesvirtualawlibrarychanrobles virtual law library
New Appropriations, by Purpose
Current Operating Expenditures
An act of the Executive which injures the institution of Congress causes a
derivative but nonetheless substantial injury, which can be questioned by a
A. PURPOSE
member of Congress (Kennedy v. Jones, 412 F. Supp. 353 [1976]). In such
a case, any member of Congress can have a resort to the
courts.chanroblesvirtualawlibrarychanrobles virtual law library Personal Maintenance Capital Total
Services and Other Outlays
Operating
Former Chief Justice Enrique M. Fernando, as Amicus Curiae, noted:
Expenses

This is, then, the clearest case of the Senate as a whole or individual
1. For Countrywide
Senators as such having a substantial interest in the question at issue. It
Developments Projects P250,000,000 P2,727,000,000 P2,977,000,000
could likewise be said that there was the requisite injury to their rights as
Senators. It would then be futile to raise any locus standi issue. Any
TOTAL NEW
intrusion into the domain appertaining to the Senate is to be resisted.
APPROPRIATIONS P250,000,000 P2,727,000,000 P2,977,000,000
Similarly, if the situation were reversed, and it is the Executive Branch that
could allege a transgression, its officials could likewise file the
corresponding action. What cannot be denied is that a Senator has Special Provisionschanrobles virtual law library
standing to maintain inviolate the prerogatives, powers and privileges
vested by the Constitution in his office (Memorandum, p. 14). 1. Use and Release of Funds. The amount herein appropriated shall be used
for infrastructure, purchase of ambulances and computers and other
It is true that the Constitution provides a mechanism for overriding a veto priority projects and activities, and credit facilities to qualified beneficiaries
(Art. VI, Sec. 27 [1]). Said remedy, however, is available only when the as proposed and identified by officials concerned according to the following
presidential veto is based on policy or political considerations but not when allocations: Representatives, P12,500,000 each; Senators, P18,000,000
the veto is claimed to be ultra vires. In the latter case, it becomes the duty each; Vice-President, P20,000,000; PROVIDED, That, the said credit
of the Court to draw the dividing line where the exercise of executive facilities shall be constituted as a revolving fund to be administered by a
power ends and the bounds of legislative jurisdiction begin. government financial institution (GFI) as a trust fund for lending
operations. Prior years releases to local government units and national
government agencies for this purpose shall be turned over to the
III
government financial institution which shall be the sole administrator of
credit facilities released from this
G.R. No. 113105 fund.chanroblesvirtualawlibrarychanrobles virtual law library

1. Countrywide Development Fund The fund shall be automatically released quarterly by way of Advice of
Allotments and Notice of Cash Allocation directly to the assigned
implementing agency not later than five (5) days after the beginning of
Article XLI of the GAA of 1994 sets up a Countrywide Development Fund of
each quarter upon submission of the list of projects and activities by the
P2,977,000,000.00 to "be used for infrastructure, purchase of ambulances
officials concerned.chanroblesvirtualawlibrarychanrobles virtual law library
and computers and other priority projects and activities and credit facilities
to qualified beneficiaries." Said Article provides:
2. Submission of Quarterly Reports. The Department of Budget and
Management shall submit within thirty (30) days after the end of each
COUNTRYWIDE DEVELOPMENT FUND
quarter a report to the Senate Committee on Finance and the House proposals and identifications made by the members of Congress are merely
Committee on Appropriations on the releases made from this Fund. The recommendatory.chanroblesvirtualawlibrarychanrobles virtual law library
report shall include the listing of the projects, locations, implementing
agencies and the endorsing officials (GAA of 1994, p. 1245).
The procedure of proposing and identifying by members of Congress of
particular projects or activities under Article XLI of the GAA of 1994 is
Petitioners claim that the power given to the members of Congress to imaginative as it is innovative.chanroblesvirtualawlibrarychanrobles virtual
propose and identify the projects and activities to be funded by the law library
Countrywide Development Fund is an encroachment by the legislature on
executive power, since said power in an appropriation act in
The Constitution is a framework of a workable government and its
implementation of a law. They argue that the proposal and identification of
interpretation must take into account the complexities, realities and politics
the projects do not involve the making of laws or the repeal and
attendant to the operation of the political branches of government. Prior to
amendment thereof, the only function given to the Congress by the
the GAA of 1991, there was an uneven allocation of appropriations for the
Constitution (Rollo, pp. 78- 86).chanroblesvirtualawlibrarychanrobles
constituents of the members of Congress, with the members close to the
virtual law library
Congressional leadership or who hold cards for "horse-trading," getting
more than their less favored colleagues. The members of Congress also
Under the Constitution, the spending power called by James Madison as had to reckon with an unsympathetic President, who could exercise his
"the power of the purse," belongs to Congress, subject only to the veto veto power to cancel from the appropriation bill a pet project of a
power of the President. The President may propose the budget, but still the Representative or Senator.chanroblesvirtualawlibrarychanrobles virtual law
final say on the matter of appropriations is lodged in the library
Congress.chanroblesvirtualawlibrarychanrobles virtual law library
The Countrywide Development Fund attempts to make equal the unequal.
The power of appropriation carries with it the power to specify the project It is also a recognition that individual members of Congress, far more than
or activity to be funded under the appropriation law. It can be as detailed the President and their congressional colleagues are likely to be
and as broad as Congress wants it to knowledgeable about the needs of their respective constituents and the
be.chanroblesvirtualawlibrarychanrobles virtual law library priority to be given each project.

The Countrywide Development Fund is explicit that it shall be used "for 2. Realignment of Operating Expenses
infrastructure, purchase of ambulances and computers and other priority
projects and activities and credit facilities to qualified beneficiaries . . ." It
Under the GAA of 1994, the appropriation for the Senate is
was Congress itself that determined the purposes for the
P472,000,000.00 of which P464,447,000.00 is appropriated for current
appropriation.chanroblesvirtualawlibrarychanrobles virtual law library
operating expenditures, while the appropriation for the House of
Representatives is P1,171,924,000.00 of which P1,165,297,000.00 is
Executive function under the Countrywide Development Fund involves appropriated for current operating expenditures (GAA of 1994, pp. 2, 4, 9,
implementation of the priority projects specified in the 12).chanroblesvirtualawlibrarychanrobles virtual law library
law.chanroblesvirtualawlibrarychanrobles virtual law library
The 1994 operating expenditures for the Senate are as follows:
The authority given to the members of Congress is only to propose and
identify projects to be implemented by the President. Under Article XLI of
Personal Serviceschanrobles virtual law library
the GAA of 1994, the President must perforce examine whether the
proposals submitted by the members of Congress fall within the specific
items of expenditures for which the Fund was set up, and if qualified, he Salaries, Permanent 153,347
next determines whether they are in line with other projects planned for Salaries/Wage, Contractual/Emergency 6,870
the locality. Thereafter, if the proposed projects qualify for funding under ----
the Funds, it is the President who shall implement them. In short, the Total Salaries and Wages 160,217
=======chanrobles virtual law library
Other Compensationchanrobles virtual law library Personal Services

Step Increments 1,073 Salaries, Permanent 261,557


Honoraria and Commutable Allowances 3,731 Salaries/Wages, Contractual/Emergency 143,643
Compensation Insurance Premiums 1,579 ----
Pag-I.B.I.G. Contributions 1,184 Total Salaries and Wages 405,200
Medicare Premiums 888 =======
Bonus and Cash Gift 14,791
Terminal Leave Benefits 2,000
Other Compensation
Personnel Economic Relief Allowance 10,266
Additional Compensation of P500 under A.O. 53 11,130
Others 57,173 Step Increments 4,312
---- Honoraria and Commutable
Total Other Compensation 103,815 Allowances 4,764
---- Compensation Insurance
01 Total Personal Services 264,032 Premiums 1,159
======= Pag-I.B.I.G. Contributions 5,231
Medicare Premiums 2,281
Maintenance and Other Operating Expenses
Bonus and Cash Gift 35,669
Terminal Leave Benefits 29
02 Traveling Expenses 32,841 Personnel Economic Relief
03 Communication Services 7,666 Allowance 21,150
04 Repair and Maintenance of Government Facilities 1,220 Additional Compensation of P500 under A.O. 53
05 Repair and Maintenance of Government Vehicles 318 Others 106,140
06 Transportation Services 128 ----
07 Supplies and Materials 20,189 Total Other Compensation 202,863
08 Rents 24,584 ----
14 Water/Illumination and Power 6,561 01 Total Personal Services 608,063
15 Social Security Benefits and Other Claims 3,270 =======
17 Training and Seminars Expenses 2,225
18 Extraordinary and Miscellaneous Expenses 9,360
Maintenance and Other Operating Expenses
23 Advertising and Publication
24 Fidelity Bonds and Insurance Premiums 1,325
29 Other Services 89,778 02 Traveling Expenses 139,611
---- 03 Communication Services 22,514
Total Maintenance and Other Operating Expenditures 200,415 04 Repair and Maintenance of Government Facilities 5,116
---- 05 Repair and Maintenance of Government Vehicles 1,863
Total Current Operating Expenditures 464,447 06 Transportation Services 178
======= 07 Supplies and Materials 55,248
10 Grants/Subsidies/Contributions 940
14 Water/Illumination and Power 14,458
(GAA of 1994, pp. 3-4)
15 Social Security Benefits and Other Claims 325
17 Training and Seminars Expenses 7,236
The 1994 operating expenditures for the House of Representatives are as 18 Extraordinary and Miscellaneous Expenses 14,474
follows: 20 Anti-Insurgency/Contingency Emergency Expenses 9,400
23 Advertising and Publication 242
24 Fidelity Bonds and Insurance Premiums 1,420 authorizing such augmentation.chanroblesvirtualawlibrarychanrobles virtual
29 Other Services 284,209 law library
----
Total Maintenance and Other Operating Expenditures 557,234
The special provision on realignment of the operating expenses of
----
members of Congress is authorized by Section 16 of the General Provisions
Total Current Operating Expenditures 1,165,297
of the GAA of 1994, which provides:
=======

Expenditure Components. Except by act of the Congress of the Philippines,


(GAA of 1994, pp. 11-12)
no change or modification shall be made in the expenditure items
authorized in this Act and other appropriation laws unless in cases 
The Special Provision Applicable to the Congress of the Philippines of augmentations from savings in appropriations as authorized under
provides: Section 25(5) of Article VI of the Constitution (GAA of 1994, p. 1273).

4. Realignment of Allocation for Operational Expenses. A member of Petitioners argue that the Senate President and the Speaker of the House
Congress may realign his allocation for operational expenses to any other of Representatives, but not the individual members of Congress are the
expenses category provide the total of said allocation is not exceeded. ones authorized to realign the savings as
(GAA of 1994, p. 14). appropriated.chanroblesvirtualawlibrarychanrobles virtual law library

The appropriation for operating expenditures for each House is further Under the Special Provisions applicable to the Congress of the Philippines,
divided into expenditures for salaries, personal services, other the members of Congress only determine the necessity of the realignment
compensation benefits, maintenance expenses and other operating of the savings in the allotments for their operating expenses. They are in
expenses. In turn, each member of Congress is allotted for his own the best position to do so because they are the ones who know whether
operating expenditure a proportionate share of the appropriation for the there are savings available in some items and whether there are
House to which he belongs. If he does not spend for one items of expense, deficiencies in other items of their operating expenses that need
the provision in question allows him to transfer his allocation in said item to augmentation. However, it is the Senate President and the Speaker of the
another item of expense.chanroblesvirtualawlibrarychanrobles virtual law House of Representatives, as the case may be, who shall approve the
library realignment. Before giving their stamp of approval, these two officials will
have to see to it that:chanrobles virtual law library
Petitioners assail the special provision allowing a member of Congress to
realign his allocation for operational expenses to any other expense (1) The funds to be realigned or transferred are actually savings in the
category (Rollo, pp. 82-92), claiming that this practice is prohibited by items of expenditures from which the same are to be taken;
Section 25(5), Article VI of the Constitution. Said section provides: and chanrobles virtual law library

No law shall be passed authorizing any transfer of appropriations: however, (2) The transfer or realignment is for the purposes of augmenting the
the President, the President of the Senate, the Speaker of the House of items of expenditure to which said transfer or realignment is to be made.
Representatives, the Chief Justice of the Supreme Court, and the heads of
Constitutional Commissions may, by law, be authorized to augment any
3. Highest Priority for Debt Service
item in the general appropriations law for their respective offices from
savings in other items of their respective appropriations.
While Congress appropriated P86,323,438,000.00 for debt service (Article
XLVII of the GAA of 1994), it appropriated only P37,780,450,000.00 for the
The proviso of said Article of the Constitution grants the President of the
Department of Education Culture and Sports. Petitioners urged that
Senate and the Speaker of the House of Representatives the power to
Congress cannot give debt service the highest priority in the GAA of 1994
augment items in an appropriation act for their respective offices from
(Rollo, pp. 93-94) because under the Constitution it should be education
savings in other items of their appropriations, whenever there is a law
that is entitled to the highest funding. They invoke Section 5(5), Article XIV Veto of Provision on Debt Ceiling
thereof, which provides:
The Congress added a Special Provision to Article XLVIII (Appropriations for
(5) The State shall assign the highest budgetary priority to education and Debt Service) of the GAA of 1994 which provides:
ensure that teaching will attract and retain its rightful share of the best
available talents through adequate remuneration and other means of job
Special Provisionschanrobles virtual law library
satisfaction and fulfillment.

1. Use of the Fund. The appropriation authorized herein shall be used for
This issue was raised in Guingona, Jr. v. Carague, 196 SCRA 221 (1991),
payment of principal and interest of foreign and domestic
where this Court held that Section 5(5), Article XIV of the Constitution, is
indebtedness; PROVIDED, That any payment in excess of the amount
merely directory, thus:
herein appropriated shall be subject to the approval of the President of the
Philippines with the concurrence of the Congress of the
While it is true that under Section 5(5), Article XIV of the Constitution, Philippines; PROVIDED, FURTHER, That in no case shall this fund be used
Congress is mandated to "assign the highest budgetary priority to to pay for the liabilities of the Central Bank Board of
education" in order to "insure that teaching will attract and retain its Liquidators.chanroblesvirtualawlibrarychanrobles virtual law library
rightful share of the best available talents through adequate remuneration
and other means of job satisfaction and fulfillment," it does not thereby
2. Reporting Requirement. The Bangko Sentral ng Pilipinas and the
follow that the hands of Congress are so hamstrung as to deprive it the
Department of Finance shall submit a quarterly report of actual foreign and
power to respond to the imperatives of the national interest and for the
domestic debt service payments to the House Committee on Appropriations
attainment of other state policies or
and Senate Finance Committee within one (1) month after each quarter
objectives.chanroblesvirtualawlibrarychanrobles virtual law library
(GAA of 1944, pp. 1266).

As aptly observed by respondents, since 1985, the budget for education


The President vetoed the first Special Provision, without vetoing the
has tripled to upgrade and improve the facility of the public school system.
P86,323,438,000.00 appropriation for debt service in said Article.
The compensation of teachers has been doubled. The amount of
According to the President's Veto Message:
P29,740,611,000.00 set aside for the Department of Education, Culture
and Sports under the General Appropriations Act (R.A. No. 6381), is the
highest budgetary allocation among all department budgets. This is a clear IV. APPROPRIATIONS FOR DEBT SERVICEchanrobles virtual law library
compliance with the aforesaid constitutional mandate according highest
priority to education.chanroblesvirtualawlibrarychanrobles virtual law I would like to emphasize that I concur fully with the desire of Congress to
library reduce the debt burden by decreasing the appropriation for debt service as
well as the inclusion of the Special Provision quoted below. Nevertheless, I
Having faithfully complied therewith, Congress is certainly not without any believe that this debt reduction scheme cannot be validly done through the
power, guided only by its good judgment, to provide an appropriation, that 1994 GAA. This must be addressed by revising our debt policy by way of
can reasonably service our enormous debt, the greater portion of which innovative and comprehensive debt reduction programs conceptualized
was inherited from the previous administration. It is not only a matter of within the ambit of the Medium-Term Philippine Development
honor and to protect the credit standing of the country. More especially, Plan.chanroblesvirtualawlibrarychanrobles virtual law library
the very survival of our economy is at stake. Thus, if in the process
Congress appropriated an amount for debt service bigger than the share Appropriations for payment of public debt, whether foreign or domestic, are
allocated to education, the Court finds and so holds that said appropriation automatically appropriated pursuant to the Foreign Borrowing Act and
cannot be thereby assailed as unconstitutional. Section 31 of P.D. No. 1177 as reiterated under Section 26, Chapter 4,
Book VI of E.O. No. 292, the Administrative Code of 1987. I wish to
G.R. No. 113105 emphasize that the constitutionality of such automatic provisions on debt
G.R. No. 113174chanrobles virtual law library servicing has been upheld by the Supreme Court in the case of "Teofisto T.
Guingona, Jr., and Aquilino Q. Pimentel, Jr. v. Hon. Guillermo N. Carague,
in his capacity as Secretary of Budget and Management, et al.," G.R. No. The bases of the petition in Gonzales, which are similar to those invoked in
94571, dated April 22, 1991.chanroblesvirtualawlibrarychanrobles virtual the present case, are stated as follows:
law library
In essence, petitioners' cause is anchored on the following grounds: (1) the
I am, therefore vetoing the following special provision for the reason that President's line-veto power as regards appropriation bills is limited to
the GAA is not the appropriate legislative measure to amend the provisions item/s and does not cover provision/s; therefore, she exceeded her
of the Foreign Borrowing Act, P.D. No. 1177 and E.O. No. 292: authority when she vetoed Section 55 (FY '89) and Section 16 (FY '90)
which are provisions; (2) when the President objects to a provision of an
appropriation bill, she cannot exercise the item-veto power but should veto
Use of the Fund. The appropriation authorized herein shall be used for
the entire bill; (3) the item-veto power does not carry with it the power to
payment of principal and interest of foreign and domestic
strike out conditions or restrictions for that would be legislation, in violation
indebtedness: PROVIDED, That any payment in excess of the amount
of the doctrine of separation of powers; and (4) the power of augmentation
herein appropriated shall be subject to the approval of the President of the
in Article VI, Section 25 [5] of the 1987 Constitution, has to be provided for
Philippines with the concurrence of the Congress of the
by law and, therefore, Congress is also vested with the prerogative to
Philippines: PROVIDED, FURTHER, That in no case shall this fund be used
impose restrictions on the exercise of that
to pay for the liabilities of the Central Bank Board of Liquidators (GAA of
power.chanroblesvirtualawlibrarychanrobles virtual law library
1994, p. 1290).

The restrictive interpretation urged by petitioners that the President may


Petitioners claim that the President cannot veto the Special Provision on
not veto a provision without vetoing the entire bill not only disregards the
the appropriation for debt service without vetoing the entire amount of
basic principle that a distinct and severable part of a bill may be the
P86,323,438.00 for said purpose (Rollo, G.R. No. 113105, pp. 93-98; Rollo,
subject of a separate veto but also overlooks the Constitutional mandate
G.R. No. 113174, pp. 16-18). The Solicitor General counterposed that the
that any provision in the general appropriations bill shall relate specifically
Special Provision did not relate to the item of appropriation for debt service
to some particular appropriation therein and that any such provision shall
and could therefore be the subject of an item veto (Rollo, G.R. No. 113105,
be limited in its operation to the appropriation to which it relates (1987
pp. 54-60; Rollo, G.R. No. 113174, pp. 72-
Constitution, Article VI, Section 25 [2]). In other words, in the true sense
82).chanroblesvirtualawlibrarychanrobles virtual law library
of the term, a provision in an Appropriations Bill is limited in its operation
to some particular appropriation to which it relates, and does not relate to
This issue is a mere rehash of the one put to rest in Gonzales v. Macaraig, the entire bill.
Jr., 191 SCRA 452 (1990). In that case, the issue was stated by the Court,
thus:
The Court went one step further and ruled that even
assuming arguendo that "provisions" are beyond the executive power to
The fundamental issue raised is whether or not the veto by the President of veto, and Section 55 
Section 55 of the 1989 Appropriations Bill (Section 55  (FY '89) and Section 16 (FY '90) were not "provisions" in the budgetary
FY '89), and subsequently of its counterpart Section 16 of the 1990 sense of the term, they are "inappropriate provisions" that should be
Appropriations Bill (Section 16 FY '90), is unconstitutional and without treated as "items" for the purpose of the President's veto
effect. power.chanroblesvirtualawlibrarychanrobles virtual law library

The Court re-stated the issue, just so there would not be any The Court, citing Henry v. Edwards, La., 346 So. 2d 153 (1977), said that
misunderstanding about it, thus: Congress cannot include in a general appropriations bill matters that should
be more properly enacted in separate legislation, and if it does that, the
The focal issue for resolution is whether or not the President exceeded the inappropriate provisions inserted by it must be treated as "item", which can
item-veto power accorded by the Constitution. Or differently put, has the be vetoed by the President in the exercise of his item-veto
President the power to veto "provisions" of an Appropriations Bill? power.chanroblesvirtualawlibrarychanrobles virtual law library
It is readily apparent that the Special Provision applicable to the Cognizant of the legislative practice of inserting provisions, including
appropriation for debt service insofar as it refers to funds in excess of the conditions, restrictions and limitations, to items in appropriations bills, the
amount appropriated in the bill, is an "inappropriate" provision referring to Constitutional Convention added the following sentence to Section 20(2),
funds other than the P86,323,438,000.00 appropriated in the General Article VI of the 1935 Constitution:
Appropriations Act of 1991.chanroblesvirtualawlibrarychanrobles virtual law
library
. . . When a provision of an appropriation bill affect one or more items of
the same, the President cannot veto the provision without at the same time
Likewise the vetoed provision is clearly an attempt to repeal Section 31 of vetoing the particular item or items to which it relates . . . .
P.D. No. 1177 (Foreign Borrowing Act) and E.O. No. 292, and to reverse
the debt payment policy. As held by the Court in Gonzales, the repeal of
In short, under the 1935 Constitution, the President was empowered to
these laws should be done in a separate law, not in the appropriations
veto separately not only items in an appropriations bill but also
law.chanroblesvirtualawlibrarychanrobles virtual law library
"provisions".chanroblesvirtualawlibrarychanrobles virtual law library

The Court will indulge every intendment in favor of the constitutionality of


While the 1987 Constitution did not retain the aforementioned sentence
a veto, the same as it will presume the constitutionality of an act of
added to Section 11(2) of Article VI of the 1935 Constitution, it included
Congress (Texas Co. v. State, 254 P. 1060; 31 Ariz, 485, 53 A.L.R. 258
the following provision:
[1927]).chanroblesvirtualawlibrarychanrobles virtual law library

No provision or enactment shall be embraced in the general appropriations


The veto power, while exercisable by the President, is actually a part of the
bill unless it relates specifically to some particular appropriation therein.
legislative process (Memorandum of Justice Irene Cortes
Any such provision or enactment shall be limited in its operation to the
as Amicus Curiae, pp. 3-7). That is why it is found in Article VI on the
appropriation to which it relates (Art. VI, Sec. 25[2]).
Legislative Department rather than in Article VII on the Executive
Department in the Constitution. There is, therefore, sound basis to indulge
in the presumption of validity of a veto. The burden shifts on those In Gonzales, we made it clear that the omission of that sentence of Section
questioning the validity thereof to show that its use is a violation of the 16(2) of the 1935 Constitution in the 1987 Constitution should not be
Constitution.chanroblesvirtualawlibrarychanrobles virtual law library interpreted to mean the disallowance of the power of the President to veto
a "provision".chanroblesvirtualawlibrarychanrobles virtual law library
Under his general veto power, the President has to veto the entire bill, not
merely parts thereof (1987 Constitution, Art. VI, Sec. 27[1]). The As the Constitution is explicit that the provision which Congress can include
exception to the general veto power is the power given to the President to in an appropriations bill must "relate specifically to some particular
veto any particular item or items in a general appropriations bill (1987 appropriation therein" and "be limited in its operation to the appropriation
Constitution, Art. VI,  to which it relates," it follows that any provision which does not relate to
Sec. 27[2]). In so doing, the President must veto the entire any particular item, or which extends in its operation beyond an item of
item.chanroblesvirtualawlibrarychanrobles virtual law library appropriation, is considered "an inappropriate provision" which can be
vetoed separately from an item. Also to be included in the category of
"inappropriate provisions" are unconstitutional provisions and provisions
A general appropriations bill is a special type of legislation, whose content
which are intended to amend other laws, because clearly these kind of laws
is limited to specified sums of money dedicated to a specific purpose or a
have no place in an appropriations bill. These are matters of general
separate fiscal unit (Beckman, The Item Veto Power of the Executive, 
legislation more appropriately dealt with in separate enactments. Former
31 Temple Law Quarterly 27 [1957]).chanroblesvirtualawlibrarychanrobles
Justice Irene Cortes, as Amicus Curiae, commented that Congress cannot
virtual law library
by law establish conditions for and regulate the exercise of powers of the
President given by the Constitution for that would be an unconstitutional
The item veto was first introduced by the Organic Act of the Philippines intrusion into executive prerogative.chanroblesvirtualawlibrarychanrobles
passed by the U.S. Congress on August 29, 1916. The concept was virtual law library
adopted from some State
Constitutions.chanroblesvirtualawlibrarychanrobles virtual law library
The doctrine of "inappropriate provision" was well elucidated in Henry Petitioners cannot anticipate that the President will not faithfully execute
v. Edwards, supra., thus: the laws. The writ of prohibition will not issue on the fear that official
actions will be done in contravention of the
laws.chanroblesvirtualawlibrarychanrobles virtual law library
Just as the President may not use his item-veto to usurp constitutional
powers conferred on the legislature, neither can the legislature deprive the
Governor of the constitutional powers conferred on him as chief executive The President vetoed the entire paragraph one of the Special Provision of
officer of the state by including in a general appropriation bill matters more the item on debt service, including the provisions that the appropriation
properly enacted in separate legislation. The Governor's constitutional authorized in said item "shall be used for payment of the principal and
power to veto bills of general legislation . . . cannot be abridged by the interest of foreign and domestic indebtedness" and that "in no case shall
careful placement of such measures in a general appropriation bill, thereby this fund be used to pay for the liabilities of the Central Bank Board of
forcing the Governor to choose between approving unacceptable Liquidators." These provisions are germane to and have a direct connection
substantive legislation or vetoing "items" of expenditures essential to the with the item on debt service. Inherent in the power of appropriation is the
operation of government. The legislature cannot by location of a bill give it power to specify how the money shall be spent (Henry v. Edwards, LA, 346
immunity from executive veto. Nor can it circumvent the Governor's veto So., 2d., 153). The said provisos, being appropriate provisions, cannot be
power over substantive legislation by artfully drafting general law vetoed separately. Hence the item veto of said provisions is
measures so that they appear to be true conditions or limitations on an void.chanroblesvirtualawlibrarychanrobles virtual law library
item of appropriation. Otherwise, the legislature would be permitted to
impair the constitutional responsibilities and functions of a co-equal branch
We reiterate, in order to obviate any misunderstanding, that we are
of government in contravention of the separation of powers doctrine . . .
sustaining the veto of the Special Provision of the item on debt service only
We are no more willing to allow the legislature to use its appropriation
with respect to the proviso therein requiring that "any payment in excess
power to infringe on the Governor's constitutional right to veto matters of
of the amount herein, appropriated shall be subject to the approval of the
substantive legislation than we are to allow the Governor to encroach on
President of the Philippines with the concurrence of the Congress of the
the Constitutional powers of the legislature. In order to avoid this result,
Philippines . . ."
we hold that, when the legislature inserts inappropriate provisions in a
general appropriation bill, such provisions must be treated as "items" for
purposes of the Governor's item veto power over general appropriation G.R. NO. 113174
bills. G.R. NO. 113766
G.R. NO. 11388
xxx xxx xxxchanrobles virtual law library
1. Veto of provisions for revolving funds of SUC's.
. . . Legislative control cannot be exercised in such a manner as to
encumber the general appropriation bill with veto-proof "logrolling In the appropriation for State Universities and Colleges (SUC's), the
measures", special interest provisions which could not succeed if separately President vetoed special provisions which authorize the use of income and
enacted, or "riders", substantive pieces of legislation incorporated in a bill the creation, operation and maintenance of revolving funds. The Special
to insure passage without veto . . . (Emphasis supplied). Provisions vetoed are the following:

Petitioners contend that granting arguendo that the veto of the Special (H. 7) West Visayas State Universitychanrobles virtual law library
Provision on the ceiling for debt payment is valid, the President cannot
automatically appropriate funds for debt payment without complying with Equal Sharing of Income. Income earned by the University subject to
the conditions for automatic appropriation under the provisions of R.A. No. Section 13 of the special provisions applicable to all State Universities and
4860 as amended by P.D. No. 81 and the provisions of P.D. No. 1177 as Colleges shall be equally shared by the University and the University
amended by the Administrative Code of 1987 and P.D. No. 1967 (Rollo, Hospital (GAA of 1994, p. 395).
G.R. No. 113766, pp. 9-15).chanroblesvirtualawlibrarychanrobles virtual
law library
xxx xxx xxxchanrobles virtual law library
(J. 3) Leyte State Collegechanrobles virtual law library All collections of the State Universities and Colleges for fees, charges and
receipts intended for private recipient units, including private foundations
affiliated with these institutions shall be duly acknowledged with official
Revolving Fund for the Operation of LSC House and Human Resources
receipts and deposited as a trust receipt before said income shall be
Development Center (HRDC). The income of Leyte State College derived
subject to Section 35, Chapter 5, Book VI of E.O. No. 292 
from the operation of its LSC House and HRDC shall be constituted into a
(GAA of 1994, p. 490).
Revolving Fund to be deposited in an authorized government depository
bank for the operational expenses of these projects/services. The net
income of the Revolving Fund at the end of the year shall be remitted to The President gave his reason for the veto thus:
the National Treasury and shall accrue to the General Fund. The
implementing guidelines shall be issued by the Department of Budget and
Pursuant to Section 65 of the Government Auditing Code of the Philippines,
Management (GAA of 1994, p. 415).
Section 44, Chapter 5, Book VI of E.O. No. 292, s. 1987 and Section 22,
Article VII of the Constitution, all income earned by all Government offices
The vetoed Special Provisions applicable to all SUC's are the following: and agencies shall accrue to the General Fund of the Government in line
with the One Fund Policy enunciated by Section 29 (1), Article VI and
Section 22, Article VII of the Constitution. Likewise, the creation and
12. Use of Income from Extension Services. State Universities and Colleges
establishment of revolving funds shall be authorized by substantive law
are authorized to use their income from their extension services. Subject to
pursuant to Section 66 of the Government Auditing Code of the Philippines
the approval of the Board of Regents and the approval of a special budget
and Section 45, Chapter 5, Book VI of E.O. No.
pursuant to Sec. 35, Chapter 5, Book VI of E.O. 
292.chanroblesvirtualawlibrarychanrobles virtual law library
No. 292, such income shall be utilized solely for faculty development,
instructional materials and work study program (GAA of 1994, p. 490).
Notwithstanding the aforementioned provisions of the Constitution and
existing law, I have noted the proliferation of special provisions authorizing
xxx xxx xxxchanrobles virtual law library
the use of agency income as well as the creation, operation and
maintenance of revolving funds.chanroblesvirtualawlibrarychanrobles
13. Income of State Universities and Colleges. The income of State virtual law library
Universities and Colleges derived from tuition fees and other sources as
may be imposed by governing boards other than those accruing to
I would like to underscore the facts that such income were already
revolving funds created under LOI Nos. 872 and 1026 and those authorized
considered as integral part of the revenue and financing sources of the
to be recorded as trust receipts pursuant to Section 40, Chapter 5, Book VI
National Expenditure Program which I previously submitted to Congress.
of E.O. No. 292 shall be deposited with the National Treasury and recorded
Hence, the grant of new special provisions authorizing the use of agency
as a Special Account in the General Fund pursuant to P.D. No. 1234 and
income and the establishment of revolving funds over and above the
P.D. No. 1437 for the use of the institution, subject to Section 35, Chapter
agency appropriations authorized in this Act shall effectively reduce the
5, Book VI of E.O. No. 292L PROVIDED, That disbursements from the
financing sources of the 1994 GAA and, at the same time, increase the
Special Account shall not exceed the amount actually earned and
level of expenditures of some agencies beyond the well-coordinated,
deposited: PROVIDED,FURTHER, That a cash advance on such income may
rationalized levels for such agencies. This corresponding increases the
be allowed State half of income actually realized during the preceding year
overall deficit of the National Government (Veto Message, p. 3).
and this cash advance shall be charged against income actually earned
during the budget year: AND PROVIDED, FINALLY, That in no case shall
such funds be used to create positions, nor for payment of salaries, wages Petitioners claim that the President acted with grave abuse of discretion
or allowances, except as may be specifically approved by the Department when he disallowed by his veto the "use of income" and the creation of
of Budge and Management for income-producing activities, or to purchase "revolving fund" by the Western Visayas State University and Leyte State
equipment or books, without the prior approval of the President of the Colleges when he allowed other government offices, like the National Stud
Philippines pursuant to Letter of Implementation No. Farm, to use their income for their operating expenses (Rollo, G.R. No.
29.chanroblesvirtualawlibrarychanrobles virtual law library 113174, pp. 15-16).chanroblesvirtualawlibrarychanrobles virtual law library
There was no undue discrimination when the President vetoed said special No retention or deduction as reserves or overhead expenses shall be made,
provisions while allowing similar provisions in other government agencies. except as authorized by law or upon direction of the President 
If some government agencies were allowed to use their income and (GAA of 1994, pp. 785-786; Emphasis supplied).
maintain a revolving fund for that purpose, it is because these agencies
have been enjoying such privilege before by virtue of the special laws
The President gave the following reason for the veto:
authorizing such practices as exceptions to the "one-fund policy" (e.g., R.A.
No. 4618 for the National Stud Farm, P.D. No. 902-A for the Securities and
Exchange Commission; E.O. No. 359 for the Department of Budget and While I am cognizant of the well-intended desire of Congress to impose
Management's Procurement Service). certain restrictions contained in some special provisions, I am equally
aware that many programs, projects and activities of agencies would
require some degree of flexibility to ensure their successful implementation
2. Veto of provision on 70% (administrative)/30% (contract) ratio for road
and therefore risk their completion. Furthermore, not only could these
maintenance.
restrictions and limitations derail and impede program implementation but
they may also result in a breach of contractual
In the appropriation for the Department of Public Works and Highways, the obligations.chanroblesvirtualawlibrarychanrobles virtual law library
President vetoed the second paragraph of Special Provision No. 2,
specifying the 30% maximum ration of works to be contracted for the
D.1.a. A study conducted by the Infrastructure Agencies show that for
maintenance of national roads and bridges. The said paragraph reads as
practical intent and purposes, maintenance by contract could be
follows:
undertaken to an optimum of seventy percent (70%) and the remaining
thirty percent (30%) by force account. Moreover, the policy of maximizing
2. Release and Use of Road Maintenance Funds. Funds allotted for the implementation through contract maintenance is a covenant of the Road
maintenance and repair of roads which are provided in this Act for the and Road Transport Program Loan from the Asian Development Bank (ADB
Department of Public Works and Highways shall be released to the Loan No. 1047-PHI-1990) and Overseas Economic Cooperation Fund (OECF
respective Engineering District, subject to such rules and regulations as Loan No. PH-C17-199). The same is a covenant under the World Bank
may be prescribed by the Department of Budget and Management. (IBRD) Loan for the Highway Management Project (IBRD Loan 
Maintenance funds for roads and bridges shall be exempt from budgetary No. PH-3430) obtained in 1992.chanroblesvirtualawlibrarychanrobles
reserve. virtual law library

Of the amount herein appropriated for the maintenance of national roads In the light of the foregoing and considering the policy of the government
and bridges, a maximum of thirty percent (30%) shall be contracted out in to encourage and maximize private sector participation in the regular
accordance with guidelines to be issued by the Department of Public Works repair and maintenance of infrastructure facilities, I am directly vetoing the
and Highways. The balance shall be used for maintenance by force underlined second paragraph of Special Provision No. 2 of the Department
account.chanroblesvirtualawlibrarychanrobles virtual law library of Public Works and Highways (Veto Message, p. 11).

Five percent (5%) of the total road maintenance fund appropriated herein The second paragraph of Special Provision No. 2 brings to fore the
to be applied across the board to the allocation of each region shall be set divergence in policy of Congress and the President. While Congress
aside for the maintenance of roads which may be converted to or taken expressly laid down the condition that only 30% of the total appropriation
over as national roads during the current year and the same shall be for road maintenance should be contracted out, the President, on the basis
released to the central office of the said department for eventual  of a comprehensive study, believed that contracting out road maintenance
sub-allotment to the concerned region and district: PROVIDED, That any projects at an option of 70% would be more efficient, economical and
balance of the said five percent (5%) shall be restored to the regions on practical.chanroblesvirtualawlibrarychanrobles virtual law library
a pro-rata basis for the maintenance of existing national
roads.chanroblesvirtualawlibrary chanrobles virtual law library
The Special Provision in question is not an inappropriate provision which
can be the subject of a veto. It is not alien to the appropriation for road
maintenance, and on the other hand, it specified how the said item shall be
expended - 70% by administrative and 30% by period for the smooth implementation of the law in the case of purchases
contract.chanroblesvirtualawlibrarychanrobles virtual law library by the Armed Forces of the Philippines, as implied by Section 11 (Education
Drive) of the law itself. This belief, however, cannot justify his veto of the
provision on the purchase of medicines by the
The 1987 Constitution allows the addition by Congress of special
AFP.chanroblesvirtualawlibrarychanrobles virtual law library
provisions, conditions to items in an expenditure bill, which cannot be
vetoed separately from the items to which they relate so long as they are
"appropriate" in the budgetary sense (Art. VII, Sec. Being directly related to and inseparable from the appropriation item on
25[2]).chanroblesvirtualawlibrarychanrobles virtual law library purchases of medicines by the AFP, the special provision cannot be vetoed
by the President without also vetoing the said item (Bolinao Electronics
Corporation v. Valencia, 11 SCRA 486 [1964]).
The Solicitor General was hard put in justifying the veto of this special
provision. He merely argued that the provision is a complete turnabout
from an entrenched practice of the government to maximize contract 4. Veto of provision on prior approval of Congress for purchase of military
maintenance (Rollo, G.R. No. 113888, pp. 85-86). That is not a ground to equipment.
veto a provision separate from the item to which it
refers.chanroblesvirtualawlibrarychanrobles virtual law library
In the appropriation for the modernization of the AFP, the President vetoed
the underlined proviso of Special Provision No. 2 on the "Use of Fund,"
The veto of the second paragraph of Special Provision No. 2 of the item for which requires the prior approval of Congress for the release of the
the DPWH is therefore unconstitutional. corresponding modernization funds, as well as the entire Special
Provisions 
No. 3 on the "Specific Prohibition":
3. Veto of provision on purchase of medicines by AFP.

2. Use of the Fund. Of the amount herein appropriated, priority shall be


In the appropriation for the Armed Forces of the Philippines (AFP), the
given for the acquisition of AFP assets necessary for protecting marine,
President vetoed the special provision on the purchase by the AFP of
mineral, forest and other resources within Philippine territorial borders and
medicines in compliance with the Generics Drugs Law (R.A. No. 6675). The
its economic zone, detection, prevention or deterrence of air or surface
vetoed provision reads:
intrusions and to support diplomatic moves aimed at preserving national
dignity, sovereignty and patrimony: PROVIDED, That the said
12. Purchase of Medicines. The purchase of medicines by all Armed Forces modernization fund shall not be released until a Table of Organization and
of the Philippines units, hospitals and clinics shall strictly comply with the Equipment for FY 1994-2000 is submitted to and approved by
formulary embodied in the National Drug Policy of the Department of Congress.chanroblesvirtualawlibrarychanrobles virtual law library
Health (GAA of 1994, p. 748).
3. Specific Prohibition. The said Modernization Fund shall not be used for
According to the President, while it is desirable to subject the purchase of payment of six (6) additional S-211 Trainer planes, 18 SF-260 Trainer
medicines to a standard formulary, "it is believed more prudent to provide planes and 150 armored personnel carriers (GAA of 1994, p. 747).
for a transition period for its adoption and smooth implementation in the
Armed Forces of the Philippines" (Veto Message, p.
As reason for the veto, the President stated that the said condition and
12).chanroblesvirtualawlibrarychanrobles virtual law library
prohibition violate the Constitutional mandate of non-impairment of
contractual obligations, and if allowed, "shall effectively alter the original
The Special Provision which requires that all purchases of medicines by the intent of the AFP Modernization Fund to cover all military equipment
AFP should strictly comply with the formulary embodied in the National deemed necessary to modernize the Armed Forces of the Philippines" (Veto
Drug Policy of the Department of Health is an "appropriate" provision. it is Message, p. 12).chanroblesvirtualawlibrarychanrobles virtual law library
a mere advertence by Congress to the fact that there is an existing law,
the Generics Act of 1988, that requires "the extensive use of drugs with
Petitioners claim that Special Provision No. 2 on the "Use of Fund" and
generic names through a rational system of procurement and distribution."
Special Provision No. 3 are conditions or limitations related to the item on
The President believes that it is more prudent to provide for a transition
the AFP modernization plan.chanroblesvirtualawlibrarychanrobles virtual The veto of said special provision is therefore valid.
law library
5. Veto of provision on use of savings to augment AFP pension funds.
The requirement in Special Provision No. 2 on the "Use of Fund" for the AFP
modernization program that the President must submit all purchases of
In the appropriation for the AFP Pension and Gratuity Fund, the President
military equipment to Congress for its approval, is an exercise of the
vetoed the new provision authorizing the Chief of Staff to use savings in
"congressional or legislative veto." By way of definition, a congressional
the AFP to augment pension and gratuity funds. The vetoed provision
veto is a means whereby the legislature can block or modify administrative
reads:
action taken under a statute. It is a form of legislative control in the
implementation of particular executive actions. The form may be either
negative, that is requiring disapproval of the executive action, or 2. Use of Savings. The Chief of Staff, AFP, is authorized, subject to the
affirmative, requiring approval of the executive action. This device approval of the Secretary of National Defense, to use savings in the
represents a significant attempt by Congress to move from oversight of the appropriations provided herein to augment the pension fund being
executive to shared administration (Dixon, The Congressional Veto and managed by the AFP Retirement and Separation Benefits System as
Separation of Powers: The Executive on a Leash,  provided under Sections 2(a) and 3 of P.D. No. 361 (GAA of 1994, 
56 North Carolina Law Review, 423 p. 746).
[1978]).chanroblesvirtualawlibrarychanrobles virtual law library
According to the President, the grant of retirement and separation benefits
A congressional veto is subject to serious questions involving the principle should be covered by direct appropriations specifically approved for the
of separation of powers.chanroblesvirtualawlibrarychanrobles virtual law purpose pursuant to Section 29(1) of Article VI of the Constitution.
library Moreover, he stated that the authority to use savings is lodged in the
officials enumerated in Section 25(5) of Article VI of the Constitution (Veto
Message, pp. 7-8).chanroblesvirtualawlibrarychanrobles virtual law library
However the case at bench is not the proper occasion to resolve the issues
of the validity of the legislative veto as provided in Special Provisions Nos.
2 and 3 because the issues at hand can be disposed of on other grounds. Petitioners claim that the Special Provision on AFP Pension and Gratuity
Any provision blocking an administrative action in implementing a law or Fund is a condition or limitation which is so intertwined with the item of
requiring legislative approval of executive acts must be incorporated in a appropriation that it could not be separated
separate and substantive bill. Therefore, being "inappropriate" provisions, therefrom.chanroblesvirtualawlibrarychanrobles virtual law library
Special Provisions Nos. 2 and 3 were properly
vetoed.chanroblesvirtualawlibrarychanrobles virtual law library The Special Provision, which allows the Chief of Staff to use savings to
augment the pension fund for the AFP being managed by the AFP
As commented by Justice Irene Cortes in her memorandum as Amicus Retirement and Separation Benefits System is violative of Sections 25(5)
Curiae: "What Congress cannot do directly by law it cannot do indirectly by and 29(1) of the Article VI of the
attaching conditions to the exercise of that power (of the President as Constitution.chanroblesvirtualawlibrarychanrobles virtual law library
Commander-in-Chief) through provisions in the appropriation
law."chanrobles virtual law library Under Section 25(5), no law shall be passed authorizing any transfer of
appropriations, and under Section 29(1), no money shall be paid out of 
Furthermore, Special Provision No. 3, prohibiting the use of the the Treasury except in pursuance of an appropriation made by law. While
Modernization Funds for payment of the trainer planes and armored Section 25(5) allows as an exception the realignment of savings to
personnel carriers, which have been contracted for by the AFP, is violative augment items in the general appropriations law for the executive branch,
of the Constitutional prohibition on the passage of laws that impair the such right must and can be exercised only by the President pursuant to a
obligation of contracts (Art. III, Sec. 10), more so, contracts entered into specific law.
by the Government itself.chanroblesvirtualawlibrarychanrobles virtual law
library 6. Condition on the deactivation of the CAFGU's.
Congress appropriated compensation for the CAFGU's, including the The Solicitor General contends that it is the President, as Commander-in-
payment of separation benefits but it added the following Special Provision: Chief of the Armed Forces of the Philippines, who should determine when
the services of the CAFGU's are no longer needed (Rollo, G.R. No. 113888, 
pp. 92-95.).chanroblesvirtualawlibrarychanrobles virtual law library
1. CAFGU Compensation and Separation Benefit. The appropriation
authorized herein shall be used for the compensation of CAFGU's including
the payment of their separation benefit not exceeding one (1) year This is the first case before this Court where the power of the President to
subsistence allowance for the 11,000 members who will be deactivated in impound is put in issue. Impoundment refers to a refusal by the President,
1994. The Chief of Staff, AFP, shall, subject to the approval of the for whatever reason, to spend funds made available by Congress. It is the
Secretary of National Defense, promulgate policies and procedures for the failure to spend or obligate budget authority of any type
payment of separation benefit (GAA of 1994, p. 740). (Notes: Impoundment of Funds, 86 Harvard Law Review 1505
[1973]).chanroblesvirtualawlibrarychanrobles virtual law library
The President declared in his Veto Message that the implementation of this
Special Provision to the item on the CAFGU's shall be subject to prior Those who deny to the President the power to impound argue that once
Presidential approval pursuant to P.D. No. 1597 and R.A.. No. 6758. He Congress has set aside the fund for a specific purpose in an appropriations
gave the following reasons for imposing the condition: act, it becomes mandatory on the part of the President to implement the
project and to spend the money appropriated therefor. The President has
no discretion on the matter, for the Constitution imposes on him the duty
I am well cognizant of the laudable intention of Congress in proposing the
to faithfully execute the laws.chanroblesvirtualawlibrarychanrobles virtual
amendment of Special Provision No. 1 of the CAFGU. However, it is
law library
premature at this point in time of our peace process to earmark and
declare through special provision the actual number of CAFGU members to
be deactivated in CY 1994. I understand that the number to be deactivated In refusing or deferring the implementation of an appropriation item, the
would largely depend on the result or degree of success of the on-going President in effect exercises a veto power that is not expressly granted by
peace initiatives which are not yet precisely determinable today. I have the Constitution. As a matter of fact, the Constitution does not say
desisted, therefore, to directly veto said provisions because this would anything about impounding. The source of the Executive authority must be
mean the loss of the entire special provision to the prejudice of its found elsewhere.chanroblesvirtualawlibrarychanrobles virtual law library
beneficient provisions. I therefore declare that the actual implementation of
this special provision shall be subject to prior Presidential approval
Proponents of impoundment have invoked at least three principal sources
pursuant to the provisions of P.D. No. 1597 and 
of the authority of the President. Foremost is the authority to impound
R.A. No. 6758 (Veto Message, p. 13).
given to him either expressly or impliedly by Congress. Second is the
executive power drawn from the President's role as Commander-in-Chief.
Petitioners claim that the Congress has required the deactivation of the Third is the Faithful Execution Clause which ironically is the same provision
CAFGU's when it appropriated the money for payment of the separation invoked by petitioners herein.chanroblesvirtualawlibrarychanrobles virtual
pay of the members of thereof. The President, however, directed that the law library
deactivation should be done in accordance to his timetable, taking into
consideration the peace and order situation in the affected
The proponents insist that a faithful execution of the laws requires that the
localities.chanroblesvirtualawlibrarychanrobles virtual law library
President desist from implementing the law if doing so would prejudice
public interest. An example given is when through efficient and prudent
Petitioners complain that the directive of the President was tantamount to management of a project, substantial savings are made. In such a case, it
an administrative embargo of the congressional will to implement the is sheer folly to expect the President to spend the entire amount budgeted
Constitution's command to dissolve the CAFGU's (Rollo, G.R. No. 113174,  in the law (Notes: Presidential Impoundment: Constitutional Theories and
p. 14; G.R. No. 113888, pp. 9, 14-16). They argue that the President Political Realities, 61 Georgetown Law Journal 1295 [1973];
cannot impair or withhold expenditures authorized and appropriated by Notes; Protecting the Fisc: Executive Impoundment and Congressional
Congress when neither the Appropriations Act nor other legislation Power, 82 Yale Law Journal 1686
authorize such impounding (Rollo, G.R. No. 113888, pp. 15- [1973).chanroblesvirtualawlibrarychanrobles virtual law library
16).chanroblesvirtualawlibrarychanrobles virtual law library
We do not find anything in the language used in the challenged Special P.D. No. 985 and other pertinent laws (GAA of 1994, p. 1128; Emphasis
Provision that would imply that Congress intended to deny to the President supplied).
the right to defer or reduce the spending, much less to deactivate 11,000
CAFGU members all at once in 1994. But even if such is the intention, the
xxx xxx xxxchanrobles virtual law library
appropriation law is not the proper vehicle for such purpose. Such intention
must be embodied and manifested in another law considering that it
abrades the powers of the Commander-in-Chief and there are existing laws Commission on Audit
on the creation of the CAFGU's to be amended. Again we state: a provision
in an appropriations act cannot  xxx xxx xxxchanrobles virtual law library
be used to repeal or amend other laws, in this case, P.D. No. 1597 and
R.A. No. 6758.
5. Use of Savings. The Chairman of the Commission on Audit is hereby
authorized, subject to appropriate accounting and auditing rules and
7. Condition on the appropriation for the Supreme Court, etc. regulations, to use savings for the payment of fringe benefits as may be
authorized by law for officials and personnel of the Commission (GAA of
(a) In the appropriations for the Supreme Court, Ombudsman, COA, and 1994, p. 1161; Emphasis supplied).
CHR, the Congress added the following provisions:
xxx xxx xxxchanrobles virtual law library
The Judiciary
Office of the Ombudsman
xxx xxx xxxchanrobles virtual law library
xxx xxx xxxchanrobles virtual law library
Special Provisionschanrobles virtual law library
6. Augmentation of Items in the appropriation of the Office of the
1. Augmentation of any Item in the Court's Appropriations. Any savings in Ombudsman. The Ombudsman is hereby authorized, subject to appropriate
the appropriations for the Supreme Court and the Lower Courts may be accounting and auditing rules and regulations to augment items of
utilized by the Chief Justice of the Supreme Court to augment any item of appropriation in the Office of the Ombudsman from savings in other items
the Court's appropriations for (a) printing of decisions and publication of of appropriation actually released, for: (a) printing and/or publication of
"Philippine Reports"; (b) Commutable terminal leaves of Justices and other decisions, resolutions, training and information materials; (b) repair,
personnel of the Supreme Court and payment of adjusted pension rates to maintenance and improvement of OMB Central and Area/Sectoral facilities;
retired Justices entitled thereto pursuant to Administrative Matter No. 91- (c) purchase of books, journals, periodicals and equipment; 
8-225-C.A.; (c) repair, maintenance, improvement and other operating (d) payment of commutable representation and transportation allowances
expenses of the courts' libraries, including purchase of books and of officials and employees who by reason of their positions are entitled
periodicals; (d) purchase, maintenance and improvement of printing thereto and fringe benefits as may be authorized specifically by law for
equipment; (e) necessary expenses for the employment of temporary officials and personnel of OMB pursuant to Section 8 of Article IX-B of the
employees, contractual and casual employees, for judicial administration; Constitution; and (e) for other official purposes subject to accounting and
(f) maintenance and improvement of the Court's Electronic Data  auditing rules and regulations (GAA of 1994, p. 1174; Emphasis supplied).
Processing System; (g) extraordinary expenses of the Chief Justice,
attendance in international conferences and conduct of training programs; xxx xxx xxxchanrobles virtual law library
(h) commutable transportation and representation allowances and fringe
benefits for Justices, Clerks of Court, Court Administrator, Chiefs of Offices
and other Court personnel in accordance with the rates prescribed by law; Commission on Human Rights
and (i) compensation of attorney-de-officio: PROVIDED, That as mandated
by LOI No. 489 any increase in salary and allowances shall be subject to xxx xxx xxxchanrobles virtual law library
the usual procedures and policies as provided for under 
1. Use of Savings. The Chairman of the Commission on Human Rights The provisions subject to said condition reads:
(CHR) is hereby authorized, subject to appropriate accounting and auditing
rules and regulations, to augment any item of appropriation in the office of
xxx xxx xxxchanrobles virtual law library
the CHR from savings in other items of appropriations actually released,
for: (a) printing and/or publication of decisions, resolutions, training
materials and educational publications; (b) repair, maintenance and 3. Revolving Fund. The income of the Commission on Audit derived from
improvement of Commission's central and regional facilities; (c) purchase sources authorized by the Government Auditing Code of the Philippines
of books, journals, periodicals and equipment, (d) payment of commutable (P.D. No. 1445) not exceeding Ten Million Pesos (P10,000,000) shall be
representation and transportation allowances of officials and employees constituted into a revolving fund which shall be used for maintenance,
who by reason of their positions are entitled thereto and fringe benefits, as operating and other incidental expenses to enhance audit services and
may be authorized by law for officials and personnel of CHR, subject to audit-related activities. The fund shall be deposited in an authorized
accounting and auditing rules and regulations (GAA of 1994, p. 1178; government depository ban, and withdrawals therefrom shall be made in
Emphasis supplied). accordance with the procedure prescribed by law and implementing rules
and regulations: PROVIDED, That any interests earned on such deposit
shall be remitted at the end of each quarter to the national Treasury and
In his Veto Message, the President expressed his approval of the conditions
shall accrue to the General Fund: PROVIDED FURTHER, That the
included in the GAA of 1994. He noted that:
Commission on Audit shall submit to the Department of Budget and
Management a quarterly report of income and expenditures of said
The said condition is consistent with the Constitutional injunction revolving fund (GAA of 1994, pp. 1160-1161).
prescribed under Section 8, Article IX-B of the Constitution which states
that "no elective or appointive public officer or employee shall receive
The President cited the "imperative need to rationalize" the
additional, double, or indirect compensation unless specifically authorized
implementation, applicability and operation of use of income and revolving
by law." I am, therefore, confident that the heads of the said offices shall
funds. The Veto Message stated:
maintain fidelity to the law and faithfully adhere to the well-established
principle on compensation standardization (Veto Message, p. 10).
. . . I have observed that there are old and long existing special provisions
authorizing the use of income and the creation of revolving funds. As a
Petitioners claim that the conditions imposed by the President violated the
rule, such authorizations should be discouraged. However, I take it that
independence and fiscal autonomy of the Supreme Court, the Ombudsman,
these authorizations have legal/statutory basis aside from being already a
the COA and the CHR.chanroblesvirtualawlibrarychanrobles virtual law
vested right to the agencies concerned which should not be jeopardized
library
through the Veto Message. There is, however, imperative need to
rationalize their implementation, applicability and operation. Thus, in order
In the first place, the conditions questioned by petitioners were placed in to substantiate the purpose and intention of said provisions, I hereby
the GAB by Congress itself, not by the President. The Veto Message merely declare that the operationalization of the following provisions during budget
highlighted the Constitutional mandate that additional or indirect implementation shall be subject to the guidelines to be issued by the
compensation can only be given pursuant to President pursuant to Section 35, Chapter 5, Book VI of E.O. No. 292 and
law.chanroblesvirtualawlibrarychanrobles virtual law library Sections 65 and 66 of P.D. No. 1445 in relation to Sections 2 and 3 of the
General Provisions of this Act (Veto Message, p. 6; Emphasis Supplied.)
In the second place, such statements are mere reminders that the
disbursements of appropriations must be made in accordance with law. (c) In the appropriation for the DPWH, the President imposed the condition
Such statements may, at worse, be treated as that in the implementation of DPWH projects, the administrative and
superfluities.chanroblesvirtualawlibrary chanrobles virtual law library engineering overhead of 5% and 3% "shall be subject to the necessary
administrative guidelines to be formulated by the Executive pursuant to
existing laws." The condition was imposed because the provision "needs
(b) In the appropriation for the COA, the President imposed the condition
further study" according to the
that the implementation of the budget of the COA be subject to "the
President.chanroblesvirtualawlibrarychanrobles virtual law library
guidelines to be issued by the President."chanrobles virtual law library
The following provision was made subject to said condition: 4. Allocation of Funds. Out of the amount appropriated for the
implementation of various projects in resettlement areas, Seven Million
Five Hundred Thousand Pesos (P7,500,000) shall be allocated to the
9. Engineering and Administrative Overhead. Not more than five percent
Dasmariñas Bagong Bayan resettlement area, Eighteen Million Pesos
(5%) of the amount for infrastructure project released by the Department
(P18,000,000) to the Carmona Relocation Center Area (Gen. Mariano
of Budget and Management shall be deducted by DPWH for administrative
Alvarez) and Three Million Pesos (P3,000,000) to the Bulihan Sites and
overhead, detailed engineering and construction supervision, testing and
Services, all of which will be for the cementing of roads in accordance with
quality control, and the like, thus insuring that at least ninety-five percent
DPWH standards.chanroblesvirtualawlibrarychanrobles virtual law library
(95%) of the released fund is available for direct implementation of the
project. PROVIDED, HOWEVER, That for school buildings, health centers,
day-care centers and barangay halls, the deductible amount shall not 5. Allocation for Sapang Palay. An allocation of Eight Million Pesos
exceed three percent (3%).chanroblesvirtualawlibrarychanrobles virtual (P8,000,000) shall be set aside for the asphalting of seven (7) kilometer
law library main road of Sapang Palay, San Jose Del Monte, Bulacan 
(GAA of 1994, p. 1216).
Violation of, or non-compliance with, this provision shall subject the
government official or employee concerned to administrative, civil and/or The President imposed the conditions: (a) that the "operationalization" of
criminal sanction under Sections 43 and 80, Book VI of E.O.  the special provision on revolving funds of the COA "shall be subject to
No. 292 (GAA of 1994, p. 786). guidelines to be issued by the President pursuant to Section 35, Chapter
5, 
Book VI of E.O. 292 and Sections 65 and 66 of P.D. No. 1445 in relation to
(d) In the appropriation for the National Housing Authority (NHA), the
Sections 2 and 3 of the General Provisions of this Act" (Rollo, G.R. 
President imposed the condition that allocations for specific projects shall
No. 113174, pp. 5,7-8); (b) that the implementation of Special Provision
be released and disbursed "in accordance with the housing program of the
No. 9 of the DPWH on the mandatory retention of 5% and 3% of the
government, subject to prior Executive approval."chanrobles virtual law
amounts released by said Department "be subject to the necessary
library
administrative guidelines to be formulated by the Executive pursuant to
existing law" (Rollo, G.R. No. 113888; pp. 10, 14-16); and (c) that the
The provision subject to the said condition reads: appropriations authorized for the NHA can be released only "in accordance
with the housing program of the government subject to prior Executive
3. Allocations for Specified Projects. The following allocations for the approval" (Rollo, G.R. No. 113888, pp. 10-11; 
specified projects shall be set aside for corollary works and used 14-16).chanroblesvirtualawlibrarychanrobles virtual law library
exclusively for the repair, rehabilitation and construction of buildings,
roads, pathwalks, drainage, waterworks systems, facilities and amenities in The conditions objected to by petitioners are mere reminders that the
the area: PROVIDED, That any road to be constructed or rehabilitated shall implementation of the items on which the said conditions were imposed,
conform with the specifications and standards set by the Department of should be done in accordance with existing laws, regulations or policies.
Public Works and Highways for such kind of They did not add anything to what was already in place at the time of the
road: PROVIDED, FURTHER, That savings that may be available in the approval of the GAA of 1994.chanroblesvirtualawlibrarychanrobles virtual
future shall be used for road repair, rehabilitation and construction: law library

(1) Maharlika Village Road - Not less than P5,000,000chanrobles virtual law There is less basis to complain when the President said that the
library expenditures shall be subject to guidelines he will issue. Until the
guidelines are issued, it cannot be determined whether they are proper or
(2) Tenement Housing Project (Taguig) - Not less than inappropriate. The issuance of administrative guidelines on the use of
P3,000,000chanrobles virtual law library public funds authorized by Congress is simply an exercise by the President
of his constitutional duty to see that the laws are faithfully executed (1987
Constitution, Art. VII, Sec. 17; Planas v. Gil 67 Phil. 62 [1939]). Under the
(3) Bagong Lipunan Condominium Project (Taguig) - Not less than Faithful Execution Clause, the President has the power to take "necessary
P2,000,000 and proper steps" to carry into execution the law (Schwartz, On
Constitutional Law, p. 147 [1977]). These steps are the ones to be 1994, p. 748), which is GRANTED.chanroblesvirtualawlibrarychanrobles
embodied in the guidelines.  virtual law library

IVchanrobles virtual law library SO ORDERED.

Petitioners chose to avail of the special civil actions but those remedies can Narvasa, C.J., Feliciano, Bidin, Regalado, Davide, Jr., Romero, Bellosillo,
be used only when respondents have acted "without or in excess" of Melo, Puno, Kapunan and Mendoza, JJ., concur.
jurisdiction, or "with grave abuse of discretion," (Revised Rules of Court, 
Rule 65, Section 2). How can we begrudge the President for vetoing the
Special Provision on the appropriation for debt payment when he merely
followed our decision in Gonzales? How can we say that Congress has
abused its discretion when it appropriated a bigger sum for debt payment
than the amount appropriated for education, when it merely followed our
dictum in Guingona?chanrobles virtual law library

Article 8 of the Civil Code of Philippines, provides:

Judicial decisions applying or interpreting the laws or the constitution shall


from a part of the legal system of the Philippines.

The Court's interpretation of the law is part of that law as of the date of its
enactment since the court's interpretation merely establishes the
contemporary legislative intent that the construed law purports to carry
into effect (People v. Licera, 65 SCRA 270 [1975]). Decisions of the
Supreme Court assume the same authority as statutes (Floresca v. Philex
Mining Corporation, 136 SCRA 141
[1985]).chanroblesvirtualawlibrarychanrobles virtual law library

Even if Guingona and Gonzales are considered hard cases that make bad


laws and should be reversed, such reversal cannot nullify prior acts done in
reliance thereof.chanroblesvirtualawlibrarychanrobles virtual law library

WHEREFORE, the petitions are DISMISSED, except with respect to 


(1) G.R. Nos. 113105 and 113766 only insofar as they pray for the
annulment of the veto of the special provision on debt service specifying
that the fund therein appropriated "shall be used for payment of the
principal and interest of foreign and domestic indebtedness" prohibiting the
use of the said funds "to pay for the liabilities of the Central Bank Board of
Liquidators", and (2) G.R. No. 113888 only insofar as it prays for the
annulment of the veto of: (a) the second paragraph of Special Provision
No. 2 of the item of appropriation for the Department of Public Works and
Highways (GAA of 1994, pp. 785-786); and (b) Special Provision No. 12 on
the purchase of medicines by the Armed Forces of the Philippines (GAA of
I. Pork Barrel: General Concept.
G.R. No. 208566, November 19, 2013
"Pork Barrel” is political parlance of American–English origin.3 Historically,
its usage may be traced to the degrading ritual of rolling out a barrel
GRECO ANTONIOUS BEDA B. BELGICA, JOSE M. VILLEGAS, JR., JOSE stuffed with pork to a multitude of black slaves who would cast their
L. GONZALEZ, REUBEN M. ABANTE, AND QUINTIN PAREDES SAN famished bodies into the porcine feast to assuage their hunger with
DIEGO, Petitioners, v. HONORABLE EXECUTIVE SECRETARY PAQUITO morsels coming from the generosity of their well–fed master.4 This practice
N. OCHOA, JR., SECRETARY OF BUDGET AND MANAGEMENT was later compared to the actions of American legislators in trying to direct
FLORENCIO B. ABAD, NATIONAL TREASURER ROSALIA V. DE LEON, federal budgets in favor of their districts.5 While the advent of refrigeration
SENATE OF THE PHILIPPINES, REPRESENTED BY FRANKLIN M. has made the actual pork barrel obsolete, it persists in reference to political
DRILON IN HIS CAPACITY AS SENATE PRESIDENT, AND HOUSE OF bills that “bring home the bacon” to a legislator’s district and
REPRESENTATIVES, REPRESENTED BY FELICIANO S. BELMONTE, JR. constituents.6 In a more technical sense, “Pork Barrel” refers to an
IN HIS CAPACITY AS SPEAKER OF THE HOUSE, Respondents.  appropriation of government spending meant for localized
projects and secured solely or primarily to bring money to a
[G.R. NO. 208493] representative’s district.7 Some scholars on the subject further use it to
refer to legislative control of local appropriations.8cralawlawlibrary 
SOCIAL JUSTICE SOCIETY (SJS) PRESIDENT SAMSON S.
ALCANTARA, Petitioner, v. HONORABLE FRANKLIN M. DRILON, IN In the Philippines, “Pork Barrel” has been commonly referred to as lump–
HIS CAPACITY AS SENATE PRESIDENT, AND HONORABLE sum, discretionary funds of Members of the Legislature,9 although, as will
FELICIANO S. BELMONTE, JR., IN HIS CAPACITY AS SPEAKER OF be later discussed, its usage would evolve in reference to certain funds of
THE HOUSE OF REPRESENTATIVES, Respondents. the Executive.

[G.R. NO. 209251] II. History of Congressional Pork Barrel in the Philippines.

PEDRITO M. NEPOMUCENO, FORMER MAYOR–BOAC, MARINDUQUE A. Pre–Martial Law Era (1922–1972).


FORMER PROVINCIAL BOARD MEMBER – PROVINCE OF
MARINDUQUE, Petitioner, v. PRESIDENT BENIGNO SIMEON C. Act 3044,10 or the Public Works Act of 1922, is considered11 as the earliest
AQUINO III* AND SECRETARY FLORENCIO “BUTCH” ABAD, form of “Congressional Pork Barrel” in the Philippines since the utilization of
DEPARTMENT OF BUDGET AND MANAGEMENT, Respondents. the funds appropriated therein were subjected to post– enactment
legislator approval. Particularly, in the area of fund release, Section
DECISION 312 provides that the sums appropriated for certain public works
projects13 “shall be distributed x x x subject to the approval of a joint
committee elected by the Senate and the House of
PERLAS–BERNABE, J.: Representatives.” “[T]he committee from each House may [also]
authorize one of its members to approve the distribution made by the
“Experience is the oracle of truth.”1 Secretary of Commerce and Communications.”14 Also, in the area of fund
realignment, the same section provides that the said secretary, “with the
– James Madison approval of said joint committee, or of the authorized members
thereof, may, for the purposes of said distribution, transfer unexpended
Before the Court are consolidated petitions2 taken under Rule 65 of the portions of any item of appropriation under this Act to any other item
Rules of Court, all of which assail the constitutionality of the Pork Barrel hereunder.”
System. Due to the complexity of the subject matter, the Court shall
heretofore discuss the system’s conceptual underpinnings before detailing In 1950, it has been documented15 that post–enactment legislator
the particulars of the constitutional challenge. participation broadened from the areas of fund release and realignment to
the area of project identification. During that year, the mechanics of the
public works act was modified to the extent that the discretion of choosing
The Facts
projects was transferred from the Secretary of Commerce and
Communications to legislators. “For the first time, the law carried a list of Fund” (CDF) which was integrated into the 1990 GAA24 with an initial
projects selected by Members of Congress, they ‘being the representatives funding of P2.3 Billion to cover “small local infrastructure and other priority
of the people, either on their own account or by consultation with local community projects.”
officials or civil leaders.’ “16 During this period, the pork barrel process
commenced with local government councils, civil groups, and individuals Under the GAAs for the years 1991 and 1992,25 CDF funds were, with the
appealing to Congressmen or Senators for projects. Petitions that were approval of the President, to be released directly to the implementing
accommodated formed part of a legislator’s allocation, and the amount agencies but “subject to the submission of the required list of
each legislator would eventually get is determined in a caucus convened by projects and activities.” Although the GAAs from 1990 to 1992 were
the majority. The amount was then integrated into the administration bill silent as to the amounts of allocations of the individual legislators, as well
prepared by the Department of Public Works and Communications. as their participation in the identification of projects, it has been
Thereafter, the Senate and the House of Representatives added their own reported26 that by 1992, Representatives were receiving P12.5 Million each
provisions to the bill until it was signed into law by the President – the in CDF funds, while Senators were receiving P18 Million each, without any
Public Works Act.17 In the 1960’s , however, pork barrel legislation limitation or qualification, and that they could identify any kind of
reportedly ceased in view of the stalemate between the House of project, from hard or infrastructure projects such as roads, bridges, and
Representatives and the Senate.18 buildings to “soft projects” such as textbooks, medicines, and
scholarships.27
B. Martial Law Era (1972–1986).
D. Fidel Valdez Ramos (Ramos) Administration (1992–1998).
While the previous “Congressional Pork Barrel” was apparently
discontinued in 1972 after Martial Law was declared, an era when “one The following year, or in 1993,28 the GAA explicitly stated that the release
man controlled the legislature,”19 the reprieve was only temporary. By of CDF funds was to be made upon the submission of the list of
1982, the Batasang Pambansa had already introduced a new item in the projects and activities identified by, among others, individual
General Appropriations Act (GAA) called the “Support for Local legislators. For the first time, the 1993 CDF Article included an
Development Projects” (SLDP) under the article on “National Aid to Local allocation for the Vice–President.29 As such, Representatives were
Government Units”. Based on reports,20 it was under the SLDP that the allocated P12.5 Million each in CDF funds, Senators, P18 Million each, and
practice of giving lump–sum allocations to individual legislators the Vice– President, P20 Million.
began, with each assemblyman receiving P500,000.00. Thereafter,
assemblymen would communicate their project preferences to the In 1994,301995,31 and 1996,32 the GAAs contained the same provisions on
Ministry of Budget and Management for approval. Then, the said ministry project identification and fund release as found in the 1993 CDF Article. In
would release the allocation papers to the Ministry of Local Governments, addition, however, the Department of Budget and Management (DBM) was
which would, in turn, issue the checks to the city or municipal treasurers in directed to submit reports to the Senate Committee on Finance and
the assemblyman’s locality. It has been further reported that the House Committee on Appropriations on the releases made from
“Congressional Pork Barrel” projects under the SLDP also began to cover the funds.33
not only public works projects, or so–called “hard projects”, but also “soft
projects”,21 or non–public works projects such as those which would fall Under the 199734 CDF Article, Members of Congress and the Vice–
under the categories of, among others, education, health and livelihood.22 President, in consultation with the implementing agency concerned,
were directed to submit to the DBM the list of 50% of projects to be funded
C. Post–Martial Law Era:  from their respective CDF allocations which shall be duly endorsed by (a)
     Corazon Cojuangco Aquino Administration (1986–1992).   the Senate President and the Chairman of the Committee on Finance, in
the case of the Senate, and (b) the Speaker of the House of
After the EDSA People Power Revolution in 1986 and the restoration of Representatives and the Chairman of the Committee on Appropriations, in
Philippine democracy, “Congressional Pork Barrel” was revived in the form the case of the House of Representatives; while the list for the remaining
of the “Mindanao Development Fund” and the “Visayas Development 50% was to be submitted within six (6) months thereafter. The same
Fund” which were created with lump–sum appropriations of P480 Million article also stated that the project list, which would be published by the
and P240 Million, respectively, for the funding of development projects in DBM,35 “shall be the basis for the release of funds” and that “[n]o
the Mindanao and Visayas areas in 1989. It has been documented23 that funds appropriated herein shall be disbursed for projects not
the clamor raised by the Senators and the Luzon legislators for a similar included in the list herein required.”
funding, prompted the creation of the “Countrywide Development
The following year, or in 1998,36 the foregoing provisions regarding the provision was present, with simply an expansion of purpose and express
required lists and endorsements were reproduced, except that the authority to realign. Nevertheless, the provisions in the 2003 budgets of
publication of the project list was no longer required as the list itself the Department of Public Works and Highways51 (DPWH) and the
sufficed for the release of CDF Funds. DepEd52required prior consultation with Members of Congress on the
aspects of implementation delegation and project list submission,
The CDF was not, however, the lone form of “Congressional Pork Barrel” at respectively. In 2004, the 2003 GAA was re–enacted.53
that time. Other forms of “Congressional Pork Barrel” were reportedly
fashioned and inserted into the GAA (called “Congressional Insertions” In 2005, 54 the PDAF Article provided that the PDAF shall be used “to fund
or “CIs”) in order to perpetuate the administration’s political agenda.37 It priority programs and projects under the ten point agenda of the national
has been articulated that since CIs “formed part and parcel of the government and shall be released directly to the implementing agencies.”
budgets of executive departments, they were not easily identifiable It also introduced the program menu concept,55 which is essentially a list
and were thus harder to monitor.” Nonetheless, the lawmakers of general programs and implementing agencies from which a
themselves as well as the finance and budget officials of the implementing particular PDAF project may be subsequently chosen by the
agencies, as well as the DBM, purportedly knew about the identifying authority. The 2005 GAA was re–enacted56 in 2006 and
insertions.38 Examples of these CIs are the Department of Education hence, operated on the same bases. In similar regard, the program menu
(DepEd) School Building Fund, the Congressional Initiative Allocations, the concept was consistently integrated into the 2007,572008,582009,59 and
Public Works Fund, the El Niño Fund, and the Poverty Alleviation 201060 GAAs.
Fund.39 The allocations for the School Building Fund, particularly, “shall be
made upon prior consultation with the representative of the Textually, the PDAF Articles from 2002 to 2010 were silent with respect
legislative district concerned.”40Similarly, the legislators had the to the specific amounts allocated for the individual legislators, as well as
power to direct how, where and when these appropriations were to be their participation in the proposal and identification of PDAF projects to be
spent.41 funded. In contrast to the PDAF Articles, however, the provisions under the
DepEd School Building Program and the DPWH budget, similar to its
E. Joseph Ejercito Estrada (Estrada) Administration (1998–2001). predecessors, explicitly required prior consultation with the concerned
Member of Congress61 anent certain aspects of project implementation.
In 1999,42 the CDF was removed in the GAA and replaced by three (3)
separate forms of CIs, namely, the “Food Security Program Fund,”43 the Significantly, it was during this era that provisions which allowed formal
“Lingap Para Sa Mahihirap Program Fund,”44 and the “Rural/Urban participation of non–governmental organizations (NGO) in the
Development Infrastructure Program Fund,”45 all of which contained a implementation of government projects were introduced. In the
special provision requiring “prior consultation” with the Members of Supplemental Budget for 2006, with respect to the appropriation for school
Congress for the release of the funds. buildings, NGOs were, by law, encouraged to participate. For such purpose,
the law stated that “the amount of at least P250 Million of the P500 Million
It was in the year 200046 that the “Priority Development Assistance allotted for the construction and completion of school buildings shall be
Fund” (PDAF) appeared in the GAA. The requirement of “prior consultation made available to NGOs including the Federation of Filipino–Chinese
with the respective Representative of the District” before PDAF funds were Chambers of Commerce and Industry, Inc. for its “Operation Barrio School”
directly released to the implementing agency concerned was explicitly program[,] with capability and proven track records in the construction of
stated in the 2000 PDAF Article. Moreover, realignment of funds to any public school buildings x x x.”62 The same allocation was made available to
expense category was expressly allowed, with the sole condition that no NGOs in the 2007 and 2009 GAAs under the DepEd Budget.63 Also, it was in
amount shall be used to fund personal services and other personnel 2007 that the Government Procurement Policy Board64 (GPPB)
benefits.47 The succeeding PDAF provisions remained the same in view of issued Resolution No. 12–2007 dated June 29, 2007 (GPPB Resolution
the re–enactment48 of the 2000 GAA for the year 2001. 12–2007), amending the implementing rules and regulations65 of RA
9184,66 the Government Procurement Reform Act, to include, as a form of
F. Gloria Macapagal–Arroyo (Arroyo) Administration (2001–2010). negotiated procurement,67 the procedure whereby the Procuring
Entity68 (the implementing agency) may enter into a memorandum of
The 200249 PDAF Article was brief and straightforward as it merely agreement with an NGO, provided that “an appropriation law or
contained a single special provision ordering the release of the funds ordinance earmarks an amount to be specifically contracted out to NGOs.”69
directly to the implementing agency or local government unit concerned,
without further qualifications. The following year, 2003,50 the same single G. Present Administration (2010–Present).
Differing from previous PDAF Articles but similar to the CDF Articles, On the one hand, the Malampaya Funds was created as a special fund
the 201170 PDAF Article included an express statement on lump– sum under Section 880 of Presidential Decree No. (PD) 910,81 issued by then
amounts allocated for individual legislators and the Vice–President: President Ferdinand E. Marcos (Marcos) on March 22, 1976. In enacting the
Representatives were given P70 Million each, broken down into P40 Million said law, Marcos recognized the need to set up a special fund to help
for “hard projects” and P30 Million for “soft projects”; while P200 Million intensify, strengthen, and consolidate government efforts relating to the
was given to each Senator as well as the Vice–President, with a P100 exploration, exploitation, and development of indigenous energy resources
Million allocation each for “hard” and “soft projects.” Likewise, a provision vital to economic growth.82 Due to the energy–related activities of the
on realignment of funds was included, but with the qualification that it may government in the Malampaya natural gas field in Palawan, or the
be allowed only once. The same provision also allowed the Secretaries of “Malampaya Deep Water Gas–to–Power Project”,83 the special fund created
Education, Health, Social Welfare and Development, Interior and Local under PD 910 has been currently labeled as Malampaya Funds.
Government, Environment and Natural Resources, Energy, and Public
Works and Highways to realign PDAF Funds, with the further conditions On the other hand the Presidential Social Fund was created under Section
that: (a) realignment is within the same implementing unit and same 12, Title IV84 of PD 1869,85 or the Charter of the Philippine Amusement and
project category as the original project, for infrastructure projects; (b) Gaming Corporation (PAGCOR). PD 1869 was similarly issued by Marcos on
allotment released has not yet been obligated for the original scope of July 11, 1983. More than two (2) years after, he amended PD 1869
work, and (c) the request for realignment is with the concurrence of the and accordingly issued PD 1993 on October 31, 1985,86 amending
legislator concerned.71 Section 1287 of the former law. As it stands, the Presidential Social
Fund has been described as a special funding facility managed and
In the 201272 and 201373 PDAF Articles, it is stated that the administered by the Presidential Management Staff through which the
“[i]dentification of projects and/or designation of beneficiaries shall President provides direct assistance to priority programs and projects not
conform to the priority list, standard or design prepared by each funded under the regular budget. It is sourced from the share of the
implementing agency [(priority list requirement)] x x x.” However, as government in the aggregate gross earnings of PAGCOR.88
practiced, it would still be the individual legislator who would choose and
identify the project from the said priority list.74 IV. Controversies in the Philippines.

Provisions on legislator allocations75 as well as fund realignment76 were Over the decades, “pork” funds in the Philippines have increased
included in the 2012 and 2013 PDAF Articles; but the allocation for the tremendously,89 owing in no small part to previous Presidents who
Vice–President, which was pegged at P200 Million in the 2011 GAA, had reportedly used the “Pork Barrel” in order to gain congressional
been deleted. In addition, the 2013 PDAF Article now allowed LGUs to be support.90 It was in 1996 when the first controversy surrounding the “Pork
identified as implementing agencies if they have the technical Barrel” erupted. Former Marikina City Representative Romeo Candazo
capability to implement the projects.77 Legislators were also allowed to (Candazo), then an anonymous source, “blew the lid on the huge sums of
identify programs/projects, except for assistance to indigent patients and government money that regularly went into the pockets of legislators in the
scholarships, outside of his legislative district provided that he secures form of kickbacks.”91 He said that the kickbacks were ‘SOP’ (standard
the written concurrence of the legislator of the intended outside–district, operating procedure) among legislators and ranged from a low 19 percent
endorsed by the Speaker of the House.78 Finally, any realignment of to a high 52 percent of the cost of each project, which could be anything
PDAF funds, modification and revision of project identification, as from dredging, rip rapping, asphalting, concreting, and construction of
well as requests for release of funds, were all required to be school buildings.”92 “Other sources of kickbacks that Candazo identified
favorably endorsed by the House Committee on Appropriations and were public funds intended for medicines and textbooks. A few days later,
the Senate Committee on Finance, as the case may be.79 the tale of the money trail became the banner story of the [Philippine
Daily] Inquirer issue of [August] 13, 1996, accompanied by an illustration
III. History of Presidential Pork Barrel in the Philippines. of a roasted pig.”93 “The publication of the stories, including those about
congressional initiative allocations of certain lawmakers, including P3.6
While the term “Pork Barrel” has been typically associated with lump– sum, [B]illion for a [C]ongressman, sparked public outrage.”94
discretionary funds of Members of Congress, the present cases and the
recent controversies on the matter have, however, shown that the term’s Thereafter, or in 2004, several concerned citizens sought the nullification of
usage has expanded to include certain funds of the President such as the the PDAF as enacted in the 2004 GAA for being unconstitutional.
Malampaya Funds and the Presidential Social Fund. Unfortunately, for lack of “any pertinent evidentiary support that illegal
misuse of PDAF in the form of kickbacks has become a common exercise of  Infrastructure projects were constructed on private lots
unscrupulous Members of Congress,” the petition was dismissed.95 without these having been turned over to the
government. 
Recently, or in July of the present year, the National Bureau of  Significant amounts were released to [implementing
Investigation (NBI) began its probe into allegations that “the government agencies] without the latter’s endorsement and without
has been defrauded of some P10 Billion over the past 10 years by a considering their mandated functions, administrative and
syndicate using funds from the pork barrel of lawmakers and various technical capabilities to implement projects. 
government agencies for scores of ghost projects.”96 The investigation was  Implementation of most livelihood projects was not
spawned by sworn affidavits of six (6) whistle–blowers who declared that undertaken by the [implementing agencies] themselves
JLN Corporation – “JLN” standing for Janet Lim Napoles (Napoles) – had but by [NGOs] endorsed by the proponent legislators to
swindled billions of pesos from the public coffers for “ghost projects” using which the Funds were transferred. • The funds were
no fewer than 20 dummy NGOs for an entire decade. While the NGOs were transferred to the NGOs in spite of the absence of any
supposedly the ultimate recipients of PDAF funds, the whistle–blowers appropriation law or ordinance. 
declared that the money was diverted into Napoles’ private  Selection of the NGOs were not compliant with law and
accounts.97 Thus, after its investigation on the Napoles controversy, regulations.
criminal complaints were filed before the Office of the Ombudsman,  Eighty–Two (82) NGOs entrusted with implementation of
charging five (5) lawmakers for Plunder, and three (3) other lawmakers for seven hundred seventy two (772) projects amount to
Malversation, Direct Bribery, and Violation of the Anti–Graft and Corrupt [P]6.156 Billion were either found questionable, or
Practices Act. Also recommended to be charged in the complaints are some submitted questionable/spurious documents, or failed to
of the lawmakers’ chiefs–of–staff or representatives, the heads and other liquidate in whole or in part their utilization of the Funds. 
officials of three (3) implementing agencies, and the several presidents of  Procurement by the NGOs, as well as some implementing
the NGOs set up by Napoles.98 agencies, of goods and services reportedly used in the
projects were not compliant with law.
On August 16, 2013, the Commission on Audit (CoA) released the results
of a three–year audit investigation99 covering the use of legislators’ PDAF
from 2007 to 2009, or during the last three (3) years of the Arroyo
As for the “Presidential Pork Barrel”, whistle–blowers alleged that “[a]t
administration. The purpose of the audit was to determine the propriety of
least P900 Million from royalties in the operation of the Malampaya gas
releases of funds under PDAF and the Various Infrastructures including
project off Palawan province intended for agrarian reform beneficiaries has
Local Projects (VILP)100 by the DBM, the application of these funds and the
gone into a dummy [NGO].”104 According to incumbent CoA Chairperson
implementation of projects by the appropriate implementing agencies and
Maria Gracia Pulido Tan (CoA Chairperson), the CoA is, as of this writing, in
several government–owned–and–controlled corporations (GOCCs).101 The
the process of preparing “one consolidated report” on the Malampaya
total releases covered by the audit amounted to P8.374 Billion in PDAF and
Funds.105
P32.664 Billion in VILP, representing 58% and 32%, respectively, of the
total PDAF and VILP releases that were found to have been made
V. The Procedural Antecedents.
nationwide during the audit period.102 Accordingly, the CoA’s findings
contained in its Report No. 2012–03 (CoA Report), entitled “Priority
Spurred in large part by the findings contained in the CoA Report and the
Development Assistance Fund (PDAF) and Various Infrastructures including
Napoles controversy, several petitions were lodged before the Court
Local Projects (VILP),” were made public, the highlights of which are as
similarly seeking that the “Pork Barrel System” be declared
follows:103
unconstitutional. To recount, the relevant procedural antecedents in these
cases are as follows:
 Amounts released for projects identified by a
considerable number of legislators significantly exceeded On August 28, 2013, petitioner Samson S. Alcantara (Alcantara), President
their respective allocations. of the Social Justice Society, filed a Petition for Prohibition of even date
 Amounts were released for projects outside of legislative under Rule 65 of the Rules of Court (Alcantara Petition), seeking that the
districts of sponsoring members of the Lower House. “Pork Barrel System” be declared unconstitutional, and a writ of prohibition
 Total VILP releases for the period exceeded the total be issued permanently restraining respondents Franklin M. Drilon and
amount appropriated under the 2007 to 2009 GAAs.  Feliciano S. Belmonte, Jr., in their respective capacities as the incumbent
Senate President and Speaker of the House of Representatives, from 14951.112
further taking any steps to enact legislation appropriating funds for the
“Pork Barrel System,” in whatever form and by whatever name it may be On September 10, 2013, the Court issued a Resolution of even date (a)
called, and from approving further releases pursuant thereto.106 The consolidating all cases; (b) requiring public respondents to comment on the
Alcantara Petition was docketed as G.R. No. 208493. consolidated petitions; (c) issuing a TRO (September 10, 2013 TRO)
enjoining the DBM, National Treasurer, the Executive Secretary, or any of
On September 3, 2013, petitioners Greco Antonious Beda B. Belgica, Jose the persons acting under their authority from releasing (1) the remaining
L. Gonzalez, Reuben M. Abante, Quintin Paredes San Diego (Belgica, et PDAF allocated to Members of Congress under the GAA of 2013, and (2)
al.), and Jose M. Villegas, Jr. (Villegas) filed an Urgent Petition Malampaya Funds under the phrase “for such other purposes as may be
For Certiorari and Prohibition With Prayer For The Immediate Issuance of hereafter directed by the President” pursuant to Section 8 of PD 910 but
Temporary Restraining Order (TRO) and/or Writ of Preliminary Injunction not for the purpose of “financ[ing] energy resource development and
dated August 27, 2013 under Rule 65 of the Rules of Court (Belgica exploitation programs and projects of the government” under the same
Petition), seeking that the annual “Pork Barrel System,” presently provision; and (d) setting the consolidated cases for Oral Arguments on
embodied in the provisions of the GAA of 2013 which provided for the 2013 October 8, 2013.
PDAF, and the Executive’s lump–sum, discretionary funds, such as the
Malampaya Funds and the Presidential Social Fund,107 be declared On September 23, 2013, the Office of the Solicitor General (OSG) filed a
unconstitutional and null and void for being acts constituting grave abuse Consolidated Comment (Comment) of even date before the Court, seeking
of discretion. Also, they pray that the Court issue a TRO against the lifting, or in the alternative, the partial lifting with respect to
respondents Paquito N. Ochoa, Jr., Florencio B. Abad (Secretary Abad) and educational and medical assistance purposes, of the Court’s September 10,
Rosalia V. De Leon, in their respective capacities as the incumbent 2013 TRO, and that the consolidated petitions be dismissed for lack of
Executive Secretary, Secretary of the Department of Budget and merit.113
Management (DBM), and National Treasurer, or their agents, for them to
immediately cease any expenditure under the aforesaid funds. Further, On September 24, 2013, the Court issued a Resolution of even date
they pray that the Court order the foregoing respondents to release to the directing petitioners to reply to the Comment.
CoA and to the public: (a) “the complete schedule/list of legislators who
have availed of their PDAF and VILP from the years 2003 to 2013, Petitioners, with the exception of Nepomuceno, filed their respective replies
specifying the use of the funds, the project or activity and the recipient to the Comment: (a) on September 30, 2013, Villegas filed a separate
entities or individuals, and all pertinent data thereto”; and (b) “the use of Reply dated September 27, 2013 (Villegas Reply); (b) on October 1, 2013,
the Executive’s [lump–sum, discretionary] funds, including the proceeds Belgica, et al. filed a Reply dated September 30, 2013 (Belgica Reply); and
from the x x x Malampaya Fund[s] [and] remittances from the [PAGCOR] x (c) on October 2, 2013, Alcantara filed a Reply dated October 1, 2013.
x x from 2003 to 2013, specifying the x x x project or activity and the
recipient entities or individuals, and all pertinent data thereto.”108 Also, they On October 1, 2013, the Court issued an Advisory providing for the
pray for the “inclusion in budgetary deliberations with the Congress of all guidelines to be observed by the parties for the Oral Arguments scheduled
presently off–budget, [lump–sum], discretionary funds including, but not on October 8, 2013. In view of the technicality of the issues material to the
limited to, proceeds from the Malampaya Fund[s] [and] remittances from present cases, incumbent Solicitor General Francis H. Jardeleza (Solicitor
the [PAGCOR].”109 The Belgica Petition was docketed as G.R. No. 208566.110 General) was directed to bring with him during the Oral Arguments
representative/s from the DBM and Congress who would be able to
Lastly, on September 5, 2013, petitioner Pedrito M. Nepomuceno competently and completely answer questions related to, among others,
(Nepomuceno), filed a Petition dated August 23, 2012 (Nepomuceno the budgeting process and its implementation. Further, the CoA
Petition), seeking that the PDAF be declared unconstitutional, and a cease Chairperson was appointed as amicus curiae and thereby requested to
and desist order be issued restraining President Benigno Simeon S. Aquino appear before the Court during the Oral Arguments.
III (President Aquino) and Secretary Abad from releasing such funds to
Members of Congress and, instead, allow their release to fund priority On October 8 and 10, 2013, the Oral Arguments were conducted.
projects identified and approved by the Local Development Councils in Thereafter, the Court directed the parties to submit their respective
consultation with the executive departments, such as the DPWH, the memoranda within a period of seven (7) days, or until October 17, 2013,
Department of Tourism, the Department of Health, the Department of which the parties subsequently did.
Transportation, and Communication and the National Economic
Development Authority.111The Nepomuceno Petition was docketed as UDK–
The Issues Before the Court I. Procedural Issues.

Based on the pleadings, and as refined during the Oral Arguments, the The prevailing rule in constitutional litigation is that no question involving
following are the main issues for the Court’s resolution: the constitutionality or validity of a law or governmental act may be heard
and decided by the Court unless there is compliance with the legal
I. Procedural Issues. requisites for judicial inquiry,117 namely: (a) there must be an actual case
or controversy calling for the exercise of judicial power; (b) the person
Whether or not (a) the issues raised in the consolidated petitions involve challenging the act must have the standing to question the validity of the
an actual and justiciable controversy; (b) the issues raised in the subject act or issuance; (c) the question of constitutionality must be raised
consolidated petitions are matters of policy not subject to judicial review; at the earliest opportunity; and (d) the issue of constitutionality must be
(c) petitioners have legal standing to sue; and (d) the Court’s Decision the very lis mota of the case.118 Of these requisites, case law states that
dated August 19, 1994 in G.R. Nos. 113105, 113174, 113766, and 113888, the first two are the most important119 and, therefore, shall be discussed
entitled “Philippine Constitution Association v. Enriquez”114 (Philconsa) and forthwith.
Decision dated April 24, 2012 in G.R. No. 164987, entitled “Lawyers
Against Monopoly and Poverty v. Secretary of Budget and A. Existence of an Actual Case or Controversy.
Management”115 (LAMP) bar the re– litigation of the issue of
constitutionality of the “Pork Barrel System” under the principles of res By constitutional fiat, judicial power operates only when there is an actual
judicataand stare decisis. case or controversy.120 This is embodied in Section 1, Article VIII of the
1987 Constitution which pertinently states that “[j]udicial power includes
II. Substantive Issues on the “Congressional Pork Barrel.” the duty of the courts of justice to settle actual controversies involving
rights which are legally demandable and enforceable x x x.”
Whether or not the 2013 PDAF Article and all other Congressional Pork Jurisprudence provides that an actual case or controversy is one which
Barrel Laws similar thereto are unconstitutional considering that they “involves a conflict of legal rights, an assertion of opposite legal claims,
violate the principles of/constitutional provisions on (a) separation of susceptible of judicial resolution as distinguished from a hypothetical or
powers; (b) non–delegability of legislative power; (c) checks and balances; abstract difference or dispute.”121 In other words, “[t]here must be
(d) accountability; (e) political dynasties; and (f) local autonomy. a contrariety of legal rights that can be interpreted and enforced on
the basis of existing law and jurisprudence.”122 Related to the
III. Substantive Issues on the “Presidential Pork Barrel.” requirement of an actual case or controversy is the requirement of
“ripeness,” meaning that the questions raised for constitutional scrutiny are
already ripe for adjudication. “A question is ripe for adjudication when the
Whether or not the phrases (a) “and for such other purposes as may be
act being challenged has had a direct adverse effect on the individual
hereafter directed by the President” under Section 8 of PD 910,116 relating
challenging it. It is a prerequisite that something had then been
to the Malampaya Funds, and (b) “to finance the priority infrastructure
accomplished or performed by either branch before a court may come into
development projects and to finance the restoration of damaged or
the picture, and the petitioner must allege the existence of an
destroyed facilities due to calamities, as may be directed and authorized by
immediate or threatened injury to itself as a result of the
the Office of the President of the Philippines” under Section 12 of PD 1869,
challenged action.”123 “Withal, courts will decline to pass upon
as amended by PD 1993, relating to the Presidential Social Fund, are
constitutional issues through advisory opinions, bereft as they are of
unconstitutional insofar as they constitute undue delegations of legislative
authority to resolve hypothetical or moot questions.”124
power.
Based on these principles, the Court finds that there exists an actual and
These main issues shall be resolved in the order that they have been
justiciable controversy in these cases.
stated. In addition, the Court shall also tackle certain ancillary issues as
prompted by the present cases.
The requirement of contrariety of legal rights is clearly satisfied by the
antagonistic positions of the parties on the constitutionality of the “Pork
The Court’s Ruling Barrel System.” Also, the questions in these consolidated cases are ripe for
adjudication since the challenged funds and the provisions allowing for
The petitions are partly granted. their utilization – such as the 2013 GAA for the PDAF, PD 910 for the
Malampaya Funds and PD 1869, as amended by PD 1993, for the
Presidential Social Fund – are currently existing and operational; hence, outside of the COA Report, you have the report of the whistle–blowers, the
there exists an immediate or threatened injury to petitioners as a result of President was just exercising precisely the duty ….
the unconstitutional use of these public funds.
xxx
As for the PDAF, the Court must dispel the notion that the issues related
thereto had been rendered moot and academic by the reforms undertaken Justice Carpio: Yes, and that is correct. You‘ve seen the CoA Report, there
by respondents. A case becomes moot when there is no more actual are anomalies, you stop and investigate, and prosecute, he has done
controversy between the parties or no useful purpose can be served in that. But, does that mean that PDAF has been repealed?
passing upon the merits.125Differing from this description, the Court
observes that respondents’ proposed line–item budgeting scheme would Solicitor General Jardeleza: No, Your Honor x x x. 
not terminate the controversy nor diminish the useful purpose for its
resolution since said reform is geared towards the 2014 budget, and not xxx
the 2013 PDAF Article which, being a distinct subject matter, remains
legally effective and existing. Neither will the President’s declaration that Justice Carpio: So that PDAF can be legally abolished only in two (2)
he had already “abolished the PDAF” render the issues on PDAF moot cases. Congress passes a law to repeal it, or this Court declares it
precisely because the Executive branch of government has no unconstitutional, correct?
constitutional authority to nullify or annul its legal existence. By
constitutional design, the annulment or nullification of a law may be done Solictor General Jardeleza: Yes, Your Honor.
either by Congress, through the passage of a repealing law, or by the
Court, through a declaration of unconstitutionality. Instructive on this point Justice Carpio: The President has no power to legally abolish PDAF.
is the following exchange between Associate Justice Antonio T. Carpio (Emphases supplied)
(Justice Carpio) and the Solicitor General during the Oral Arguments:126
Even on the assumption of mootness, jurisprudence, nevertheless, dictates
Justice Carpio: [T]he President has taken an oath to faithfully that “the ‘moot and academic’ principle is not a magical formula that can
execute the law,127correct? automatically dissuade the Court in resolving a case.” The Court will decide
cases, otherwise moot, if: first, there is a grave violation of the
Solicitor General Jardeleza: Yes, Your Honor. Constitution; second, the exceptional character of the situation and the
paramount public interest is involved; third, when the constitutional issue
Justice Carpio: And so the President cannot refuse to implement the raised requires formulation of controlling principles to guide the bench, the
General Appropriations Act, correct? bar, and the public; and fourth, the case is capable of repetition yet
evading review.129
Solicitor General Jardeleza: Well, that is our answer, Your Honor. In the
case, for example of the PDAF, the President has a duty to execute the The applicability of the first exception is clear from the fundamental
laws but in the face of the outrage over PDAF, the President was saying, “I posture of petitioners – they essentially allege grave violations of the
am not sure that I will continue the release of the soft projects,” and that Constitution with respect to, inter alia, the principles of separation of
started, Your Honor. Now, whether or not that … (interrupted) powers, non–delegability of legislative power, checks and balances,
accountability and local autonomy.
Justice Carpio: Yeah. I will grant the President if there are anomalies in the
project, he has the power to stop the releases in the meantime, to The applicability of the second exception is also apparent from the nature
investigate, and that is Section [38] of Chapter 5 of Book 6 of the Revised of the interests involved – the constitutionality of the very system within
Administrative Code128 x x x. So at most the President can suspend, now if which significant amounts of public funds have been and continue to be
the President believes that the PDAF is unconstitutional, can he just refuse utilized and expended undoubtedly presents a situation of exceptional
to implement it? character as well as a matter of paramount public interest. The present
petitions, in fact, have been lodged at a time when the system’s flaws have
Solicitor General Jardeleza: No, Your Honor, as we were trying to say in the never before been magnified. To the Court’s mind, the coalescence of the
specific case of the PDAF because of the CoA Report, because of the CoA Report, the accounts of numerous whistle–blowers, and the
reported irregularities and this Court can take judicial notice, even outside, government’s own recognition that reforms are needed “to address the
reported abuses of the PDAF”130demonstrates a prima facie pattern of
abuse which only underscores the importance of the matter. It is also by Finally, the application of the fourth exception is called for by the
this finding that the Court finds petitioners’ claims as not merely theorized, recognition that the preparation and passage of the national budget is, by
speculative or hypothetical. Of note is the weight accorded by the Court to constitutional imprimatur, an affair of annual occurrence.133 The relevance
the findings made by the CoA which is the constitutionally–mandated audit of the issues before the Court does not cease with the passage of a “PDAF–
arm of the government. In Delos Santos v. CoA,131 a recent case wherein free budget for 2014.”134 The evolution of the “Pork Barrel System,” by its
the Court upheld the CoA’s disallowance of irregularly disbursed PDAF multifarious iterations throughout the course of history, lends a semblance
funds, it was emphasized that: of truth to petitioners’ claim that “the same dog will just resurface wearing
a different collar.”135 In Sanlakas v. Executive Secretary,136 the government
[T]he CoA is endowed with enough latitude to determine, prevent, had already backtracked on a previous course of action yet the Court used
and disallow irregular, unnecessary, excessive, extravagant or the “capable of repetition but evading review” exception in order “[t]o
unconscionable expenditures of government funds. It is tasked to be prevent similar questions from re–emerging.”137 The situation similarly
vigilant and conscientious in safeguarding the proper use of the holds true to these cases. Indeed, the myriad of issues underlying the
government’s, and ultimately the people’s, property. The exercise of its manner in which certain public funds are spent, if not resolved at this most
general audit power is among the constitutional mechanisms that opportune time, are capable of repetition and hence, must not evade
gives life to the check and balance system inherent in our form of judicial review.
government.
B. Matters of Policy: the Political Question Doctrine.
[I]t is the general policy of the Court to sustain the decisions of
administrative authorities, especially one which is constitutionally–created, The “limitation on the power of judicial review to actual cases and
such as the CoA, not only on the basis of the doctrine of separation controversies” carries the assurance that “the courts will not intrude into
of powers but also for their presumed expertise in the laws they areas committed to the other branches of government.”138Essentially, the
are entrusted to enforce. Findings of administrative agencies are foregoing limitation is a restatement of the political question doctrine
accorded not only respect but also finality when the decision and order are which, under the classic formulation of Baker v. Carr,139 applies when there
not tainted with unfairness or arbitrariness that would amount to grave is found, among others, “a textually demonstrable constitutional
abuse of discretion. It is only when the CoA has acted without or in excess commitment of the issue to a coordinate political department,” “a lack of
of jurisdiction, or with grave abuse of discretion amounting to lack or judicially discoverable and manageable standards for resolving it” or “the
excess of jurisdiction, that this Court entertains a petition questioning its impossibility of deciding without an initial policy determination of a kind
rulings. x x x. (Emphases supplied) clearly for non–judicial discretion.” Cast against this light, respondents
submit that the “[t]he political branches are in the best position not only to
Thus, if only for the purpose of validating the existence of an actual perform budget–related reforms but also to do them in response to the
and justiciable controversy in these cases, the Court deems the specific demands of their constituents” and, as such, “urge [the Court] not
findings under the CoA Report to be sufficient. to impose a solution at this stage.”140

The Court also finds the third exception to be applicable largely due to The Court must deny respondents’ submission.
the practical need for a definitive ruling on the system’s constitutionality.
As disclosed during the Oral Arguments, the CoA Chairperson estimates Suffice it to state that the issues raised before the Court do not present
that thousands of notices of disallowances will be issued by her office in political but legal questions which are within its province to resolve. A
connection with the findings made in the CoA Report. In this relation, political question refers to “those questions which, under the Constitution,
Associate Justice Marvic Mario Victor F. Leonen (Justice Leonen) pointed are to be decided by the people in their sovereign capacity, or in regard to
out that all of these would eventually find their way to the which full discretionary authority has been delegated to the Legislature or
courts.132 Accordingly, there is a compelling need to formulate controlling executive branch of the Government. It is concerned with issues dependent
principles relative to the issues raised herein in order to guide the bench, upon the wisdom, not legality, of a particular measure.”141The intrinsic
the bar, and the public, not just for the expeditious resolution of the constitutionality of the “Pork Barrel System” is not an issue
anticipated disallowance cases, but more importantly, so that the dependent upon the wisdom of the political branches of
government may be guided on how public funds should be utilized in government but rather a legal one which the Constitution itself has
accordance with constitutional principles. commanded the Court to act upon. Scrutinizing the contours of the
system along constitutional lines is a task that the political branches of
government are incapable of rendering precisely because it is an exercise
of judicial power. More importantly, the present Constitution has not only personal stake in the outcome of the controversy as to assure that
vested the Judiciary the right to exercise judicial power but essentially concrete adverseness which sharpens the presentation of issues upon
makes it a duty to proceed therewith. Section 1, Article VIII of the 1987 which the court depends for illumination of difficult constitutional questions.
Constitution cannot be any clearer: “The judicial power shall be vested in Unless a person is injuriously affected in any of his constitutional rights by
one Supreme Court and in such lower courts as may be established by law. the operation of statute or ordinance, he has no standing.”145
[It] includes the duty of the courts of justice to settle actual controversies
involving rights which are legally demandable and enforceable, and to Petitioners have come before the Court in their respective capacities as
determine whether or not there has been a grave abuse of discretion citizen–taxpayers and accordingly, assert that they “dutifully contribute to
amounting to lack or excess of jurisdiction on the part of any branch or the coffers of the National Treasury.”146 Clearly, as taxpayers, they possess
instrumentality of the Government.” In Estrada v. Desierto,142 the the requisite standing to question the validity of the existing “Pork Barrel
expanded concept of judicial power under the 1987 Constitution and its System” under which the taxes they pay have been and continue to be
effect on the political question doctrine was explained as follows:143 utilized. It is undeniable that petitioners, as taxpayers, are bound to suffer
from the unconstitutional usage of public funds, if the Court so rules.
To a great degree, the 1987 Constitution has narrowed the reach of the Invariably, taxpayers have been allowed to sue where there is a claim that
political question doctrine when it expanded the power of judicial review public funds are illegally disbursed or that public money is being deflected
of this court not only to settle actual controversies involving rights which to any improper purpose, or that public funds are wasted through the
are legally demandable and enforceable but also to determine whether enforcement of an invalid or unconstitutional law,147 as in these cases.
or not there has been a grave abuse of discretion amounting to lack
or excess of jurisdiction on the part of any branch or Moreover, as citizens, petitioners have equally fulfilled the standing
instrumentality of government. Heretofore, the judiciary has focused on requirement given that the issues they have raised may be classified as
the “thou shalt not’s” of the Constitution directed against the exercise of its matters “of transcendental importance, of overreaching significance to
jurisdiction. With the new provision, however, courts are given a greater society, or of paramount public interest.”148 The CoA Chairperson’s
prerogative to determine what it can do to prevent grave abuse of statement during the Oral Arguments that the present controversy involves
discretion amounting to lack or excess of jurisdiction on the part of any “not [merely] a systems failure” but a “complete breakdown of
branch or instrumentality of government. Clearly, the new provision did controls”149 amplifies, in addition to the matters above–discussed, the
not just grant the Court power of doing nothing. x x x (Emphases seriousness of the issues involved herein. Indeed, of greater import than
supplied) the damage caused by the illegal expenditure of public funds is the mortal
wound inflicted upon the fundamental law by the enforcement of an invalid
It must also be borne in mind that “when the judiciary mediates to allocate statute.150 All told, petitioners have sufficient locus standi to file the instant
constitutional boundaries, it does not assert any superiority over the other cases.
departments; does not in reality nullify or invalidate an act of the
legislature [or the executive], but only asserts the solemn and sacred D. Res Judicata and Stare Decisis.
obligation assigned to it by the Constitution.”144 To a great extent, the
Court is laudably cognizant of the reforms undertaken by its co–equal Res judicata (which means a “matter adjudged”) and stare decisisnon
branches of government. But it is by constitutional force that the Court quieta et movere  ([or simply, stare decisis] which means “follow past
must faithfully perform its duty. Ultimately, it is the Court’s avowed precedents and do not disturb what has been settled”) are general
intention that a resolution of these cases would not arrest or in any manner procedural law principles which both deal with the effects of previous but
impede the endeavors of the two other branches but, in fact, help ensure factually similar dispositions to subsequent cases. For the cases at bar, the
that the pillars of change are erected on firm constitutional grounds. After Court examines the applicability of these principles in relation to its prior
all, it is in the best interest of the people that each great branch of rulings in Philconsa and LAMP.
government, within its own sphere, contributes its share towards achieving
a holistic and genuine solution to the problems of society. For all these The focal point of res judicata is the judgment. The principle states
reasons, the Court cannot heed respondents’ plea for judicial restraint. that a judgment on the merits in a previous case rendered by a court of
competent jurisdiction would bind a subsequent case if, between the first
C. Locus Standi. and second actions, there exists an identity of parties, of subject
matter, and of causes of action.151 This required identity is not,
"The gist of the question of standing is whether a party alleges such however, attendant hereto since Philconsa and LAMP, respectively involved
constitutional challenges against the 1994 CDF Article and 2004 PDAF
Article, whereas the cases at bar call for a broader constitutional scrutiny with each other, formative as they are of the entire “Pork Barrel System”
of the entire “Pork Barrel System.”Also, the ruling in LAMP is essentially as well as (b) the intra–relation of post–enactment measures contained
a dismissal based on a procedural technicality – and, thus, hardly a within a particular CDF or PDAF Article, including not only those related to
judgment on the merits – in that petitioners therein failed to present any the area of project identification but also to the areas of fund release and
“convincing proof x x x showing that, indeed, there were direct releases realignment. The complexity of the issues and the broader legal analyses
of funds to the Members of Congress, who actually spend them according herein warranted may be, therefore, considered as a powerful
to their sole discretion” or “pertinent evidentiary support [to demonstrate countervailing reason against a wholesale application of the stare
the] illegal misuse of PDAF in the form of kickbacks [and] has become a decisis principle.
common exercise of unscrupulous Members of Congress.” As such, the
Court upheld, in view of the presumption of constitutionality accorded to In addition, the Court observes that the Philconsa ruling was actually
every law, the 2004 PDAF Article, and saw “no need to review or reverse riddled with inherent constitutional inconsistencies which similarly
the standing pronouncements in the said case.” Hence, for the foregoing countervail against a full resort to stare decisis. As may be deduced from
reasons, the res judicata principle, insofar as the Philconsa and LAMP cases the main conclusions of the case, Philconsa’ s fundamental premise in
are concerned, cannot apply. allowing Members of Congress to propose and identify of projects would be
that the said identification authority is but an aspect of the power of
On the other hand, the focal point of stare decisis is the doctrine appropriation which has been constitutionally lodged in Congress. From this
created. The principle, entrenched under Article 8152 of the Civil Code, premise, the contradictions may be easily seen. If the authority to identify
evokes the general rule that, for the sake of certainty, a conclusion projects is an aspect of appropriationand the power of appropriation is a
reached in one case should be doctrinally applied to those that follow if the form of legislative power thereby lodged in Congress, then it follows
facts are substantially the same, even though the parties may be different. that: (a) it is Congress which should exercise such authority, and not its
It proceeds from the first principle of justice that, absent any powerful individual Members; (b)such authority must be exercised within the
countervailing considerations, like cases ought to be decided alike. prescribed procedure of law passage and, hence, should not be exercised
Thus, where the same questions relating to the same event have been after the GAA has already been passed; and (c) such authority, as
put forward by the parties similarly situated as in a previous case litigated embodied in the GAA, has the force of law and, hence, cannot be merely
and decided by a competent court, the rule of stare decisis is a bar to any recommendatory. Justice Vitug’s Concurring Opinion in the same case
attempt to re–litigate the same issue.153 sums up the Philconsa quandary in this wise: Neither would it be
objectionable for Congress, by law, to appropriate funds for such specific
Philconsa was the first case where a constitutional challenge against a Pork projects as it may be minded; to give that authority, however, to the
Barrel provision, i.e., the 1994 CDF Article, was resolved by the Court. To individual members of Congress in whatever guise, I am afraid, would be
properly understand its context, petitioners’ posturing was that “the power constitutionally impermissible.” As the Court now largely benefits from
given to the [M]embers of Congress to propose and identify projects and hindsight and current findings on the matter, among others, the CoA
activities to be funded by the [CDF] is an encroachment by the legislature Report, the Court must partially abandon its previous ruling
on executive power, since said power in an appropriation act is in in Philconsa insofar as it validated the post–enactment identification
implementation of the law” and that “the proposal and identification of the authority of Members of Congress on the guise that the same was
projects do not involve the making of laws or the repeal and amendment merely recommendatory. This postulate raises serious constitutional
thereof, the only function given to the Congress by the Constitution.”154 In inconsistencies which cannot be simply excused on the ground that such
deference to the foregoing submissions, the Court reached the following mechanism is “imaginative as it is innovative.” Moreover, it must be
main conclusions: one, under the Constitution, the power of appropriation, pointed out that the recent case of Abakada Guro Party List v.
or the “power of the purse,” belongs to Congress; two, the power of Purisima155 (Abakada) has effectively overturned Philconsa’ s allowance of
appropriation carries with it the power to specify the project or activity to post–enactment legislator participation in view of the separation of powers
be funded under the appropriation law and it can be detailed and as broad principle. These constitutional inconsistencies and the Abakada rule will be
as Congress wants it to be; and, three, the proposals and identifications discussed in greater detail in the ensuing section of this Decision.
made by Members of Congress are merely recommendatory. At once, it is
apparent that the Philconsa resolution was a limited response to a As for LAMP, suffice it to restate that the said case was dismissed on a
separation of powers problem, specifically on the propriety of procedural technicality and, hence, has not set any controlling doctrine
conferring post–enactment identification authority to Members of susceptible of current application to the substantive issues in these cases.
Congress. On the contrary, the present cases call for a more holistic In fine, stare decisis would not apply.
examination of (a) the inter–relation between the CDF and PDAF Articles
to the Malampaya Funds and the Presidential Social Fund.
II. Substantive Issues.
A. Definition of Terms. With these definitions in mind, the Court shall now proceed to discuss the
Before the Court proceeds to resolve the substantive issues of these cases, substantive issues of these cases.
it must first define the terms “Pork Barrel System,” “Congressional Pork
Barrel,” and “Presidential Pork Barrel” as they are essential to the ensuing B. Substantive Issues on the Congressional Pork Barrel.
discourse. 1. Separation of Powers.
a. Statement of Principle.
Petitioners define the term “Pork Barrel System” as the “collusion between The principle of separation of powers refers to the constitutional
the Legislative and Executive branches of government to accumulate lump– demarcation of the three fundamental powers of government. In the
sum public funds in their offices with unchecked discretionary powers to celebrated words of Justice Laurel in Angara v. Electoral Commission, 162 it
determine its distribution as political largesse.”156 They assert that the means that the “Constitution has blocked out with deft strokes and in bold
following elements make up the Pork Barrel System: (a) lump–sum funds lines, allotment of power to the executive, the legislative and the judicial
are allocated through the appropriations process to an individual officer; departments of the government.”163 To the legislative branch of
(b) the officer is given sole and broad discretion in determining how the government, through Congress,164 belongs the power to make laws; to the
funds will be used or expended; (c) the guidelines on how to spend or use executive branch of government, through the President,165 belongs the
the funds in the appropriation are either vague, overbroad or inexistent; power to enforce laws; and to the judicial branch of government, through
and (d) projects funded are intended to benefit a definite constituency in a the Court,166 belongs the power to interpret laws. Because the three great
particular part of the country and to help the political careers of the powers have been, by constitutional design, ordained in this respect,
disbursing official by yielding rich patronage benefits.157 They further state “[e]ach department of the government has exclusive cognizance of matters
that the Pork Barrel System is comprised of two (2) kinds of discretionary within its jurisdiction, and is supreme within its own sphere.”167 Thus, “the
public funds: first, the Congressional (or Legislative) Pork Barrel, currently legislature has no authority to execute or construe the law, the executive
known as the PDAF;158 and, second, the Presidential (or Executive) Pork has no authority to make or construe the law, and the judiciary has no
Barrel, specifically, the Malampaya Funds under PD 910 and the power to make or execute the law.”168 The principle of separation of powers
Presidential Social Fund under PD 1869, as amended by PD 1993.159 and its concepts of autonomy and independence stem from the notion that
the powers of government must be divided to avoid concentration of these
Considering petitioners’ submission and in reference to its local concept powers in any one branch; the division, it is hoped, would avoid any single
and legal history, the Court defines the Pork Barrel System as the branch from lording its power over the other branches or the
collective body of rules and practices that govern the manner by citizenry.169 To achieve this purpose, the divided power must be wielded by
which lump–sum, discretionary funds, primarily intended for local co–equal branches of government that are equally capable of independent
projects, are utilized through the respective participations of the action in exercising their respective mandates. Lack of independence would
Legislative and Executive branches of government, including its result in the inability of one branch of government to check the arbitrary or
members. The Pork Barrel System involves two (2) kinds of lump–sum self– interest assertions of another or others.170
discretionary funds:
Broadly speaking, there is a violation of the separation of powers principle
First, there is the Congressional Pork Barrel which is herein defined when one branch of government unduly encroaches on the domain of
as a kind of lump–sum, discretionary fund wherein legislators, another. US Supreme Court decisions instruct that the principle of
either individually or collectively organized into committees, are separation of powers may be violated in two (2) ways: firstly, “[o]ne
able to effectively control certain aspects of the fund’s utilization branch may interfere impermissibly with the other’s performance
through various post–enactment measures and/or practices. In of its constitutionally assigned function”;171 and “[a]lternatively, the
particular, petitioners consider the PDAF, as it appears under the 2013 doctrine may be violated when one branch assumes a function that more
GAA, as Congressional Pork Barrel since it is, inter alia, a post–enactment properly is entrusted to another.”172 In other words, there is a violation of
measure that allows individual legislators to wield a collective power;160 and the principle when there is impermissible (a) interference with and/or
(b) assumption of another department’s functions.
Second, there is the Presidential Pork Barrel which is herein defined
as a kind of lump–sum, discretionary fund which allows the The enforcement of the national budget, as primarily contained in the GAA,
President to determine the manner of its utilization. For reasons is indisputably a function both constitutionally assigned and properly
earlier stated,161 the Court shall delimit the use of such term to refer only entrusted to the Executive branch of government. In Guingona, Jr. v. Hon.
Carague173 (Guingona, Jr.), the Court explained that the phase of budget departments to appear before and be heard by either of its Houses on any
execution “covers the various operational aspects of budgeting” and matter pertaining to their departments and its power of confirmation; and
accordingly includes “the evaluation of work and financial plans for
individual activities,” the “regulation and release of funds” as well as (2) investigation and monitoring of the implementation of laws pursuant to
all “other related activities” that comprise the budget execution the power of Congress to conduct inquiries in aid of legislation.
cycle.174 This is rooted in the principle that the allocation of power in the Any action or step beyond that will undermine the separation of
three principal branches of government is a grant of all powers inherent in powers guaranteed by the Constitution. (Emphases supplied)
them.175 Thus, unless the Constitution provides otherwise, the Executive b. Application.
department should exclusively exercise all roles and prerogatives which go In these cases, petitioners submit that the Congressional Pork Barrel –
into the implementation of the national budget as provided under the GAA among others, the 2013 PDAF Article – “wrecks the assignment of
as well as any other appropriation law. responsibilities between the political branches” as it is designed to allow
individual legislators to interfere “way past the time it should have ceased”
In view of the foregoing, the Legislative branch of government, much more or, particularly, “after the GAA is passed.”179 They state that the findings
any of its members, should not cross over the field of implementing the and recommendations in the CoA Report provide ?an illustration of how
national budget since, as earlier stated, the same is properly the domain of absolute and definitive the power of legislators wield over project
the Executive. Again, in Guingona, Jr., the Court stated that “Congress implementation in complete violation of the constitutional [principle of
enters the picture [when it] deliberates or acts on the budget proposals of separation of powers.]”180 Further, they point out that the Court in
the President. Thereafter, Congress, “in the exercise of its own judgment the Philconsa case only allowed the CDF to exist on the condition that
and wisdom, formulates an appropriation act precisely following the individual legislators limited their role to recommending projects and not if
process established by the Constitution, which specifies that no money may they actually dictate their implementation.181
be paid from the Treasury except in accordance with an appropriation
made by law.” Upon approval and passage of the GAA, Congress’ law– For their part, respondents counter that the separations of powers principle
making role necessarily comes to an end and from there the Executive’s has not been violated since the President maintains “ultimate authority to
role of implementing the national budget begins. So as not to blur the control the execution of the GAA” and that he “retains the final discretion
constitutional boundaries between them, Congress must “not concern itself to reject” the legislators’ proposals.182 They maintain that the Court,
with details for implementation by the Executive.”176 in Philconsa, “upheld the constitutionality of the power of members of
Congress to propose and identify projects so long as such proposal and
The foregoing cardinal postulates were definitively enunciated identification are recommendatory.”183 As such, they claim that
in Abakada where the Court held that “[f]rom the moment the law “[e]verything in the Special Provisions [of the 2013 PDAF Article] follows
becomes effective, any provision of law that empowers Congress or the Philconsa framework, and hence, remains constitutional.”184
any of its members to play any role in the implementation or
enforcement of the lawviolates the principle of separation of The Court rules in favor of petitioners.
powers and is thus unconstitutional.”177 It must be clarified, however,
that since the restriction only pertains to “any role in the implementation or As may be observed from its legal history, the defining feature of all forms
enforcement of the law,” Congress may still exercise its oversight function of Congressional Pork Barrel would be the authority of legislators to
which is a mechanism of checks and balances that the Constitution itself participate in the post–enactment phases of project implementation.
allows. But it must be made clear that Congress’ role must be confined to
mere oversight. Any post– enactment–measure allowing legislator At its core, legislators – may it be through project lists,185 prior
participation beyond oversight is bereft of any constitutional basis and consultations186 or program menus187 – have been consistently accorded
hence, tantamount to impermissible interference and/or assumption of post–enactment authority to identify the projects they desire to be
executive functions. As the Court ruled in Abakada:178 funded through various Congressional Pork Barrel allocations. Under the
2013 PDAF Article, the statutory authority of legislators to identify projects
[A]ny post–enactment congressional measure x x x should be post– GAA may be construed from the import of Special Provisions 1 to 3
limited to scrutiny and investigation. In particular, congressional as well as the second paragraph of Special Provision 4. To elucidate,
oversight must be confined to the following:chanRoblesvirtualLawlibrary Special Provision 1 embodies the program menu feature which, as evinced
(1) scrutiny based primarily on Congress’ power of appropriation and the from past PDAF Articles, allows individual legislators to identify PDAF
budget hearings conducted in connection with it, its power to ask heads of projects for as long as the identified project falls under a general program
listed in the said menu. Relatedly, Special Provision 2 provides that the
implementing agencies shall, within 90 days from the GAA is passed, powers principle. The fundamental rule, as categorically articulated
submit to Congress a more detailed priority list, standard or design in Abakada, cannot be overstated – from the moment the law becomes
prepared and submitted by implementing agencies from which the effective, any provision of law that empowers Congress or any of
legislator may make his choice. The same provision further authorizes its members to play any role in the implementation or enforcement
legislators to identify PDAF projects outside his district for as long as the of the law violates the principle of separation of powers and is thus
representative of the district concerned concurs in writing. Meanwhile, unconstitutional.191 That the said authority is treated as merely
Special Provision 3 clarifies that PDAF projects refer to “projects to be recommendatory in nature does not alter its unconstitutional tenor since
identified by legislators”188 and thereunder provides the allocation limit for the prohibition, to repeat, covers any role in the implementation or
the total amount of projects identified by each legislator. Finally, paragraph enforcement of the law. Towards this end, the Court must therefore
2 of Special Provision 4 requires that any modification and revision of the abandon its ruling in Philconsawhich sanctioned the conduct of legislator
project identification “shall be submitted to the House Committee on identification on the guise that the same is merely recommendatory and,
Appropriations and the Senate Committee on Finance for favorable as such, respondents’ reliance on the same falters altogether.
endorsement to the DBM or the implementing agency, as the case may
be.” From the foregoing special provisions, it cannot be seriously doubted Besides, it must be pointed out that respondents have nonetheless failed to
that legislators have been accorded post–enactment authority to identify substantiate their position that the identification authority of legislators is
PDAF projects. only of recommendatory import. Quite the contrary, respondents – through
the statements of the Solicitor General during the Oral Arguments – have
Aside from the area of project identification, legislators have also been admitted that the identification of the legislator constitutes a mandatory
accorded post–enactment authority in the areas of fund release and requirement before his PDAF can be tapped as a funding source, thereby
realignment. Under the 2013 PDAF Article, the statutory authority of highlighting the indispensability of the said act to the entire budget
legislators to participate in the area of fund release through congressional execution process:192
committees is contained in Special Provision 5 which explicitly states that
“[a]ll request for release of funds shall be supported by the documents Justice Bernabe: Now, without the individual legislator’s
prescribed under Special Provision No. 1 and favorably endorsed by House identification of the project, can the PDAF of the legislator be
Committee on Appropriations and the Senate Committee on Finance, as the utilized? 
case may be”; while their statutory authority to participate in the area
of fund realignment is contained in: first, paragraph 2, Special Provision Solicitor General Jardeleza: No, Your Honor. 
4189 which explicitly states, among others, that “[a]ny realignment [of
funds] shall be submitted to the House Committee on Appropriations and Justice Bernabe: It cannot?
the Senate Committee on Finance for favorable endorsement to the DBM or
the implementing agency, as the case may be”; and, second, paragraph 1, Solicitor General Jardeleza: It cannot… (interrupted)
also of Special Provision 4 which authorizes the “Secretaries of Agriculture,
Education, Energy, Interior and Local Government, Labor and Employment, Justice Bernabe: So meaning you should have the identification of
Public Works and Highways, Social Welfare and Development and Trade the project by the individual legislator?
and Industry190 x x x to approve realignment from one project/scope to
another within the allotment received from this Fund, subject to [among Solicitor General Jardeleza: Yes, Your Honor. 
others] (iii) the request is with the concurrence of the legislator
concerned.” xxx

Clearly, these post–enactment measures which govern the areas of project Justice Bernabe: In short, the act of identification is mandatory?
identification, fund release and fund realignment are not related to
functions of congressional oversight and, hence, allow legislators to Solictor General Jardeleza: Yes, Your Honor. In the sense that if it is
intervene and/or assume duties that properly belong to the sphere of not done and then there is no identification.
budget execution. Indeed, by virtue of the foregoing, legislators have been,
in one form or another, authorized to participate in – as Guingona, Jr. puts xxx
it – “the various operational aspects of budgeting,” including “the
evaluation of work and financial plans for individual activities” and Justice Bernabe: Now, would you know of specific instances when a project
the “regulation and release of funds” in violation of the separation of
was implemented without the identification by the individual legislator?
Ultimately, legislators cannot exercise powers which they do not have,
Solicitor General Jardeleza: I do not know, Your Honor; I do not think so whether through formal measures written into the law or informal practices
but I have no specific examples. I would doubt very much, Your Honor, institutionalized in government agencies, else the Executive department be
because to implement, there is a need [for] a SARO and the NCA. And the deprived of what the Constitution has vested as its own.
SARO and the NCA are triggered by an identification from the 2. Non–delegability of Legislative Power.
legislator. a. Statement of Principle.
As an adjunct to the separation of powers principle,194 legislative power
xxx shall be exclusively exercised by the body to which the Constitution has
conferred the same. In particular, Section 1, Article VI of the 1987
Solictor General Jardeleza: What we mean by mandatory, Your Honor, is Constitution states that such power shall be vested in the Congress of the
we were replying to a question, “How can a legislator make sure that he is Philippines which shall consist of a Senate and a House of Representatives,
able to get PDAF Funds?” It is mandatory in the sense that he must except to the extent reserved to the people by the provision on initiative
identify, in that sense, Your Honor. Otherwise, if he does not identify, he and referendum.195 Based on this provision, it is clear that only Congress,
cannot avail of the PDAF Funds and his district would not be able to have acting as a bicameral body, and the people, through the process of
PDAF Funds, only in that sense, Your Honor. (Emphases supplied) initiative and referendum, may constitutionally wield legislative power and
no other. This premise embodies the principle of non–delegability of
Thus, for all the foregoing reasons, the Court hereby declares the 2013 legislative power, and the only recognized exceptions thereto would be: (a)
PDAF Article as well as all other provisions of law which similarly allow delegated legislative power to local governments which, by immemorial
legislators to wield any form of post–enactment authority in the practice, are allowed to legislate on purely local matters;196and (b)
implementation or enforcement of the budget, unrelated to constitutionally–grafted exceptions such as the authority of the President
congressional oversight, as violative of the separation of powers principle to, by law, exercise powers necessary and proper to carry out a declared
and thus unconstitutional. Corollary thereto, informal practices, through national policy in times of war or other national emergency,197 or fix within
which legislators have effectively intruded into the proper phases of budget specified limits, and subject to such limitations and restrictions as Congress
execution, must be deemed as acts of grave abuse of may impose, tariff rates, import and export quotas, tonnage and wharfage
discretion amounting to lack or excess of jurisdiction and, hence, dues, and other duties or imposts within the framework of the national
accorded the same unconstitutional treatment. That such informal practices development program of the Government.198
do exist and have, in fact, been constantly observed throughout the years
has not been substantially disputed here. As pointed out by Chief Justice Notably, the principle of non–delegability should not be confused as a
Maria Lourdes P.A. Sereno (Chief Justice Sereno) during the Oral restriction to delegate rule–making authority to implementing agencies
Arguments of these cases:193 for the limited purpose of either filling up the details of the law for its
enforcement (supplementary rule–making) or ascertaining facts to
Chief Justice Sereno: bring the law into actual operation (contingent rule–making).199 The
conceptual treatment and limitations of delegated rule–making were
Now, from the responses of the representative of both, the DBM and two explained in the case of People v. Maceren200 as follows:
(2) Houses of Congress, if we enforces the initial thought that I have, after
I had seen the extent of this research made by my staff, that neither the The grant of the rule–making power to administrative agencies is
Executive nor Congress frontally faced the question of constitutional a relaxation of the principle of separation of powers and is an
compatibility of how they were engineering the budget process. In fact, the exception to the nondelegation of legislative powers. Administrative
words you have been using, as the three lawyers [of the DBM, and both regulations or “subordinate legislation” calculated to promote the public
Houses of Congress] has also been using is surprise; surprised that all of interest are necessary because of “the growing complexity of modern life,
these things are now surfacing. In fact, I thought that what the 2013 the multiplication of the subjects of governmental regulations, and the
PDAF provisions did was to codify in one section all the past increased difficulty of administering the law.”
practice that [had] been done since 1991. In a certain sense, we
should be thankful that they are all now in the PDAF Special Provisions. x x xxx
x (Emphasis and underscoring supplied)
[Nevertheless, it must be emphasized that] [t]he rule–making
power must be confined to details for regulating the mode or Sec. 27. x x x. 
proceeding to carry into effect the law as it has been enacted. The
power cannot be extended to amending or expanding the statutory xxx
requirements or to embrace matters not covered by the statute. Rules that
subvert the statute cannot be sanctioned. (Emphases (2) The President shall have the power to veto any particular item or items
supplied)chanroblesvirtualawlibrary in an appropriation, revenue, or tariff bill, but the veto shall not affect the
b. Application. item or items to which he does not object.
In the cases at bar, the Court observes that the 2013 PDAF Article, insofar
as it confers post–enactment identification authority to individual The presentment of appropriation, revenue or tariff bills to the President,
legislators, violates the principle of non–delegability since said legislators wherein he may exercise his power of item–veto, forms part of the “single,
are effectively allowed to individually exercise the power of finely wrought and exhaustively considered, procedures” for law–
appropriation, which – as settled in Philconsa – is lodged in passage as specified under the Constitution.204 As stated in Abakada, the
Congress.201 That the power to appropriate must be exercised only final step in the law–making process is the “submission [of the bill] to the
through legislation is clear from Section 29(1), Article VI of the 1987 President for approval. Once approved, it takes effect as law after the
Constitution which states that: “No money shall be paid out of the Treasury required publication.”205 Elaborating on the President’s item–veto power
except in pursuance of an appropriation made by law.” To understand and its relevance as a check on the legislature, the Court, in Bengzon,
what constitutes an act of appropriation, the Court, in Bengzon v. explained that:206
Secretary of Justice and Insular Auditor202 (Bengzon), held that the power
of appropriation involves (a) the setting apart by law of a certain The former Organic Act and the present Constitution of the Philippines
sum from the public revenue for (b) a specified purpose. Essentially, make the Chief Executive an integral part of the law–making power. His
under the 2013 PDAF Article, individual legislators are given a personal disapproval of a bill, commonly known as a veto, is essentially a
lump–sum fund from which they are able to dictate (a) how much from legislative act. The questions presented to the mind of the Chief
such fund would go to (b) a specific project or beneficiary that they Executive are precisely the same as those the legislature must determine
themselves also determine. As these two (2) acts comprise the exercise of in passing a bill, except that his will be a broader point of view.
the power of appropriation as described in Bengzon, and given that the
2013 PDAF Article authorizes individual legislators to perform the same, The Constitution is a limitation upon the power of the legislative
undoubtedly, said legislators have been conferred the power to legislate department of the government, but in this respect it is a grant of
which the Constitution does not, however, allow. Thus, keeping with the power to the executive department. The Legislature has the affirmative
principle of non–delegability of legislative power, the Court hereby declares power to enact laws; the Chief Executive has the negative power by
the 2013 PDAF Article, as well as all other forms of Congressional Pork the constitutional exercise of which he may defeat the will of the
Barrel which contain the similar legislative identification feature as herein Legislature. It follows that the Chief Executive must find his authority in
discussed, as unconstitutional. the Constitution. But in exercising that authority he may not be confined to
3. Checks and Balances. rules of strict construction or hampered by the unwise interference of the
a. Statement of Principle; Item–Veto Power. judiciary. The courts will indulge every intendment in favor of the
The fact that the three great powers of government are intended to be kept constitutionality of a veto [in the same manner] as they will presume the
separate and distinct does not mean that they are absolutely unrestrained constitutionality of an act as originally passed by the Legislature.
and independent of each other. The Constitution has also provided for an (Emphases supplied)
elaborate system of checks and balances to secure coordination in the
workings of the various departments of the government.203 The justification for the President’s item–veto power rests on a variety of
policy goals such as to prevent log–rolling legislation,207 impose fiscal
A prime example of a constitutional check and balance would be restrictions on the legislature, as well as to fortify the executive branch’s
the President’s power to veto an item written into an appropriation, role in the budgetary process.208 In Immigration and Naturalization Service
revenue or tariff bill submitted to him by Congress for approval through v. Chadha, the US Supreme Court characterized the President’s item–
a process known as “bill presentment.” The President’s item–veto power is power as “a salutary check upon the legislative body, calculated to guard
found in Section 27(2), Article VI of the 1987 Constitution which reads as the community against the effects of factions, precipitancy, or of any
follows: impulse unfriendly to the public good, which may happen to influence a
majority of that body”; phrased differently, it is meant to “increase the
chances in favor of the community against the passing of bad laws, actually available as certified by the National Treasurer, or to be
through haste, inadvertence, or design.”209 raised by a corresponding revenue proposal therein.” Meanwhile,
with respect to discretionary funds, Section 25(6), Article VI of the 1987
For the President to exercise his item–veto power, it necessarily Constitution requires that said funds “shall be disbursed only for public
follows that there exists a proper “item” which may be the object purposes to be supported by appropriate vouchers and subject to
of the veto. An item, as defined in the field of appropriations, pertains to such guidelines as may be prescribed by law.”
“the particulars, the details, the distinct and severable parts of the
appropriation or of the bill.” In the case of Bengzon v. Secretary of Justice In contrast, what beckons constitutional infirmity are appropriations which
of the Philippine Islands,210the US Supreme Court characterized an item of merely provide for a singular lump–sum amount to be tapped as a
appropriation as follows: source of funding for multiple purposes. Since such appropriation type
necessitates the further determination of both the actual amount to be
An item of an appropriation bill obviously means an item which, in itself, is expended and the actual purpose of the appropriation which must still be
a specific appropriation of money, not some general provision of chosen from the multiple purposes stated in the law, it cannot be said that
law which happens to be put into an appropriation bill. (Emphases the appropriation law already indicates a “specific appropriation of money”
supplied) and hence, without a proper line–item which the President may veto. As a
practical result, the President would then be faced with the predicament of
On this premise, it may be concluded that an appropriation bill, to ensure either vetoing the entire appropriation if he finds some of its purposes
that the President may be able to exercise his power of item veto, wasteful or undesirable, or approving the entire appropriation so as not to
must contain “specific appropriations of money” and not only “general hinder some of its legitimate purposes. Finally, it may not be amiss to state
provisions” which provide for parameters of appropriation. that such arrangement also raises non–delegability issues considering that
the implementing authority would still have to determine, again, both the
Further, it is significant to point out that an item of appropriation must be actual amount to be expended and the actual purpose of the appropriation.
an item characterized by singular correspondence – meaning an Since the foregoing determinations constitute the integral aspects of the
allocation of a specified singular amount for a specified singular power to appropriate, the implementing authority would, in effect, be
purpose, otherwise known as a “line–item.”211 This treatment not only exercising legislative prerogatives in violation of the principle of non–
allows the item to be consistent with its definition as a “specific delegability.
appropriation of money” but also ensures that the President may b. Application.
discernibly veto the same. Based on the foregoing formulation, the existing In these cases, petitioners claim that “[i]n the current x x x system where
Calamity Fund, Contingent Fund and the Intelligence Fund, being the PDAF is a lump–sum appropriation, the legislator’s identification of the
appropriations which state a specified amount for a specific purpose, would projects after the passage of the GAA denies the President the chance to
then be considered as “line–item” appropriations which are rightfully veto that item later on.”212 Accordingly, they submit that the “item veto
subject to item veto. Likewise, it must be observed that an power of the President mandates that appropriations bills adopt line–item
appropriation may be validly apportioned into component budgeting” and that “Congress cannot choose a mode of budgeting [which]
percentages or values; however, it is crucial that each percentage or effectively renders the constitutionally–given power of the President
value must be allocated for its own corresponding purpose for such useless.”213
component to be considered as a proper line–item. Moreover, as Justice
Carpio correctly pointed out, a valid appropriation may even have several On the other hand, respondents maintain that the text of the Constitution
related purposes that are by accounting and budgeting practice considered envisions a process which is intended to meet the demands of a
as one purpose, e.g., MOOE (maintenance and other operating expenses), modernizing economy and, as such, lump–sum appropriations are essential
in which case the related purposes shall be deemed sufficiently specific for to financially address situations which are barely foreseen when a GAA is
the exercise of the President’s item veto power. Finally, special purpose enacted. They argue that the decision of the Congress to create some
funds and discretionary funds would equally square with the constitutional lump– sum appropriations is constitutionally allowed and textually–
mechanism of item–veto for as long as they follow the rule on grounded.214
singular correspondence as herein discussed. Anent special purpose
funds, it must be added that Section 25(4), Article VI of the 1987 The Court agrees with petitioners.
Constitution requires that the “‘special appropriations bill shall specify the
purpose for which it is intended, and shall be supported by funds Under the 2013 PDAF Article, the amount of P24.79 Billion only appears as
a collective allocation limit since the said amount would be further divided
among individual legislators who would then receive personal lump–sum financial interest in the smooth, speedy passing of the yearly budget”
allocations and could, after the GAA is passed, effectively appropriate PDAF which turns them “from fiscalizers” into “financially–interested
funds based on their own discretion. As these intermediate appropriations partners.”219 They also claim that the system has an effect on re–election
are made by legislators only after the GAA is passed and hence, outside of as “the PDAF excels in self–perpetuation of elective officials.” Finally, they
the law, it necessarily means that the actual items of PDAF appropriation add that the “PDAF impairs the power of impeachment” as such “funds are
would not have been written into the General Appropriations Bill and thus indeed quite useful, ‘to well, accelerate the decisions of senators.’ “220
effectuated without veto consideration. This kind of lump–sum/post–
enactment legislative identification budgeting system fosters the The Court agrees in part.
creation of a “budget within a budget” which subverts the prescribed
procedure of presentment and consequently impairs the President’s power The aphorism forged under Section 1, Article XI of the 1987 Constitution,
of item veto. As petitioners aptly point out, the above– described system which states that “public office is a public trust,” is an overarching
forces the President to decide between (a) accepting the entire P24.79 reminder that every instrumentality of government should exercise their
Billion PDAF allocation without knowing the specific projects of the official functions only in accordance with the principles of the Constitution
legislators, which may or may not be consistent with his national agenda which embodies the parameters of the people’s trust. The notion of a public
and (b) rejecting the whole PDAF to the detriment of all other legislators trust connotes accountability,221 hence, the various mechanisms in the
with legitimate projects.215 Constitution which are designed to exact accountability from public officers.

Moreover, even without its post–enactment legislative identification Among others, an accountability mechanism with which the proper
feature, the 2013 PDAF Article would remain constitutionally flawed since it expenditure of public funds may be checked is the power of congressional
would then operate as a prohibited form of lump–sum appropriation as oversight. As mentioned in Abakada,222 congressional oversight may be
above–characterized. In particular, the lump–sum amount of P24.79 Billion performed either through: (a) scrutiny based primarily on Congress’
would be treated as a mere funding source allotted for multiple purposes of power of appropriation and the budget hearings conducted in connection
spending, i.e., scholarships, medical missions, assistance to indigents, with it, its power to ask heads of departments to appear before and be
preservation of historical materials, construction of roads, flood control, heard by either of its Houses on any matter pertaining to their departments
etc. This setup connotes that the appropriation law leaves the actual and its power of confirmation;223 or (b) investigation and monitoring of
amounts and purposes of the appropriation for further determination and, the implementation of laws pursuant to the power of Congress to
therefore, does not readily indicate a discernible item which may be subject conduct inquiries in aid of legislation.224
to the President’s power of item veto.
The Court agrees with petitioners that certain features embedded in some
In fact, on the accountability side, the same lump–sum budgeting scheme forms of Congressional Pork Barrel, among others the 2013 PDAF Article,
has, as the CoA Chairperson relays, “limit[ed] state auditors from obtaining has an effect on congressional oversight. The fact that individual legislators
relevant data and information that would aid in more stringently auditing are given post–enactment roles in the implementation of the budget makes
the utilization of said Funds.”216 Accordingly, she recommends the adoption it difficult for them to become disinterested “observers” when scrutinizing,
of a “line by line budget or amount per proposed program, activity or investigating or monitoring the implementation of the appropriation law. To
project, and per implementing agency.”217 a certain extent, the conduct of oversight would be tainted as said
legislators, who are vested with post–enactment authority, would, in effect,
Hence, in view of the reasons above–stated, the Court finds the 2013 PDAF be checking on activities in which they themselves participate. Also, it must
Article, as well as all Congressional Pork Barrel Laws of similar operation, be pointed out that this very same concept of post– enactment
to be unconstitutional. That such budgeting system provides for a greater authorization runs afoul of Section 14, Article VI of the 1987 Constitution
degree of flexibility to account for future contingencies cannot be an excuse which provides that:
to defeat what the Constitution requires. Clearly, the first and essential
truth of the matter is that unconstitutional means do not justify even Sec. 14. No Senator or Member of the House of Representatives may
commendable ends.218 personally appear as counsel before any court of justice or before the
c. Accountability. Electoral Tribunals, or quasi–judicial and other administrative bodies.
Petitioners further relate that the system under which various forms of Neither shall he, directly or indirectly, be interested financially in any
Congressional Pork Barrel operate defies public accountability as it renders contract with, or in any franchise or special privilege granted by the
Congress incapable of checking itself or its Members. In particular, they Government, or any subdivision, agency, or instrumentality thereof,
point out that the Congressional Pork Barrel “gives each legislator a direct,
including any government–owned or controlled corporation, or its In any event, the Court finds the above–stated argument on this score to
subsidiary, during his term of office. He shall not intervene in any be largely speculative since it has not been properly demonstrated how the
matter before any office of the Government for his pecuniary Pork Barrel System would be able to propagate political dynasties.
benefit or where he may be called upon to act on account of his 5. Local Autonomy.
office. (Emphasis supplied) The State’s policy on local autonomy is principally stated in Section 25,
Article II and Sections 2 and 3, Article X of the 1987 Constitution which
Clearly, allowing legislators to intervene in the various phases of project read as follows:
implementation – a matter before another office of government – renders
them susceptible to taking undue advantage of their own office. ARTICLE II

The Court, however, cannot completely agree that the same post– Sec. 25. The State shall ensure the autonomy of local governments.
enactment authority and/or the individual legislator’s control of his PDAF
per se would allow him to perpetuate himself in office. Indeed, while the
ARTICLE X
Congressional Pork Barrel and a legislator’s use thereof may be linked to
this area of interest, the use of his PDAF for re–election purposes is a
Sec. 2. The territorial and political subdivisions shall enjoy local autonomy.
matter which must be analyzed based on particular facts and on a case–to–
case basis.
Sec. 3. The Congress shall enact a local government code which shall
provide for a more responsive and accountable local government structure
Finally, while the Court accounts for the possibility that the close
instituted through a system of decentralization with effective mechanisms
operational proximity between legislators and the Executive department,
of recall, initiative, and referendum, allocate among the different local
through the former’s post–enactment participation, may affect the process
government units their powers, responsibilities, and resources, and provide
of impeachment, this matter largely borders on the domain of politics and
for the qualifications, election, appointment and removal, term, salaries,
does not strictly concern the Pork Barrel System’s intrinsic constitutionality.
powers and functions and duties of local officials, and all other matters
As such, it is an improper subject of judicial assessment.
relating to the organization and operation of the local units.
In sum, insofar as its post–enactment features dilute congressional
Pursuant thereto, Congress enacted RA 7160,227 otherwise known as the
oversight and violate Section 14, Article VI of the 1987 Constitution, thus
“Local Government Code of 1991” (LGC), wherein the policy on local
impairing public accountability, the 2013 PDAF Article and other forms of
autonomy had been more specifically explicated as follows:
Congressional Pork Barrel of similar nature are deemed as unconstitutional.
4. Political Dynasties.
One of the petitioners submits that the Pork Barrel System enables Sec. 2. Declaration of Policy. – (a) It is hereby declared the policy of the
politicians who are members of political dynasties to accumulate funds to State that the territorial and political subdivisions of the State shall enjoy
perpetuate themselves in power, in contravention of Section 26, Article II genuine and meaningful local autonomy to enable them to attain
of the 1987 Constitution225 which states that: their fullest development as self–reliant communities and make
them more effective partners in the attainment of national goals.
Toward this end, the State shall provide for a more responsive and
Sec. 26. The State shall guarantee equal access to opportunities for public
accountable local government structure instituted through a system of
service, and prohibit political dynasties as may be defined by law.
decentralization whereby local government units shall be given more
(Emphasis and underscoring supplied)
powers, authority, responsibilities, and resources. The process of
decentralization shall proceed from the National Government to the local
At the outset, suffice it to state that the foregoing provision is considered
government units.
as not self–executing due to the qualifying phrase “as may be
defined by law.” In this respect, said provision does not, by and of itself,
xxx
provide a judicially enforceable constitutional right but merely specifies a
guideline for legislative or executive action.226 Therefore, since there
(c) It is likewise the policy of the State to require all national agencies
appears to be no standing law which crystallizes the policy on political
and offices to conduct periodic consultations with appropriate local
dynasties for enforcement, the Court must defer from ruling on this issue.
government units, nongovernmental and people’s organizations, and
other concerned sectors of the community before any project or
program is implemented in their respective jurisdictions. (Emphases Notwithstanding these declarations, the Court, however, finds an inherent
and underscoring supplied) defect in the system which actually belies the avowed intention of “making
equal the unequal.” In particular, the Court observes that the gauge of
The above–quoted provisions of the Constitution and the LGC reveal the PDAF and CDF allocation/division is based solely on the fact of
policy of the State to empower local government units (LGUs) to develop office, without taking into account the specific interests and
and ultimately, become self–sustaining and effective contributors to the peculiarities of the district the legislator represents. In this regard,
national economy. As explained by the Court in Philippine Gamefowl the allocation/division limits are clearly not based on genuine parameters
Commission v. Intermediate Appellate Court:228 of equality, wherein economic or geographic indicators have been taken
into consideration. As a result, a district representative of a highly–
This is as good an occasion as any to stress the commitment of the urbanized metropolis gets the same amount of funding as a district
Constitution to the policy of local autonomy which is intended to representative of a far–flung rural province which would be relatively
provide the needed impetus and encouragement to the “underdeveloped” compared to the former. To add, what rouses graver
development of our local political subdivisions as “self–reliant scrutiny is that even Senators and Party–List Representatives – and in
communities.” In the words of Jefferson, ?Municipal corporations are the some years, even the Vice– President – who do not represent any locality,
small republics from which the great one derives its strength.” The receive funding from the Congressional Pork Barrel as well. These certainly
vitalization of local governments will enable their inhabitants to fully exploit are anathema to the Congressional Pork Barrel’s original intent which is “to
their resources and more important, imbue them with a deepened sense of make equal the unequal.” Ultimately, the PDAF and CDF had become
involvement in public affairs as members of the body politic. This personal funds under the effective control of each legislator and given unto
objective could be blunted by undue interference by the national them on the sole account of their office.
government in purely local affairs which are best resolved by the
officials and inhabitants of such political units. The decision we reach The Court also observes that this concept of legislator control underlying
today conforms not only to the letter of the pertinent laws but also to the the CDF and PDAF conflicts with the functions of the various Local
spirit of the Constitution.229(Emphases and underscoring supplied) Development Councils (LDCs) which are already legally mandated to “assist
the corresponding sanggunian in setting the direction of economic and
In the cases at bar, petitioners contend that the Congressional Pork Barrel social development, and coordinating development efforts within its
goes against the constitutional principles on local autonomy since it allows territorial jurisdiction.”234 Considering that LDCs are instrumentalities
district representatives, who are national officers, to substitute their whose functions are essentially geared towards managing local
judgments in utilizing public funds for local development.230 affairs,235 their programs, policies and resolutions should not be overridden
nor duplicated by individual legislators, who are national officers that have
The Court agrees with petitioners. no law–making authority except only when acting as a body. The
undermining effect on local autonomy caused by the post–enactment
Philconsa described the 1994 CDF as an attempt “to make equal the authority conferred to the latter was succinctly put by petitioners in the
unequal” and that ?[i]t is also a recognition that individual members of following wise:236
Congress, far more than the President and their congressional colleagues,
are likely to be knowledgeable about the needs of their respective With PDAF, a Congressman can simply bypass the local development
constituents and the priority to be given each project.”231 Drawing strength council and initiate projects on his own, and even take sole credit for its
from this pronouncement, previous legislators justified its existence by execution. Indeed, this type of personality–driven project identification has
stating that ?the relatively small projects implemented under [the not only contributed little to the overall development of the district, but has
Congressional Pork Barrel] complement and link the national development even contributed to ?further weakening infrastructure planning and
goals to the countryside and grassroots as well as to depressed areas coordination efforts of the government.”
which are overlooked by central agencies which are preoccupied with
mega–projects.232 Similarly, in his August 23, 2013 speech on the Thus, insofar as individual legislators are authorized to intervene in purely
“abolition” of PDAF and budgetary reforms, President Aquino mentioned local matters and thereby subvert genuine local autonomy, the 2013 PDAF
that the Congressional Pork Barrel was originally established for a worthy Article as well as all other similar forms of Congressional Pork Barrel is
goal, which is to enable the representatives to identify projects for deemed unconstitutional.
communities that the LGU concerned cannot afford.233
With this final issue on the Congressional Pork Barrel resolved, the Court appropriation may be made in general as well as in specific terms. The
now turns to the substantive issues involving the Presidential Pork Barrel. Congressional authorization may be embodied in annual laws, such as a
C. Substantive Issues on the Presidential Pork Barrel. general appropriations act or in special provisions of laws of general or
1. Validity of Appropriation. special application which appropriate public funds for specific public
Petitioners preliminarily assail Section 8 of PD 910 and Section 12 of purposes, such as the questioned decrees. An appropriation measure is
PD1869 (now, amended by PD 1993), which respectively provide for the sufficient if the legislative intention clearly and certainly appears
Malampaya Funds and the Presidential Social Fund, as invalid from the language employed (In re Continuing Appropriations, 32
appropriations laws since they do not have the “primary and specific” P. 272), whether in the past or in the present.(Emphases and
purpose of authorizing the release of public funds from the National underscoring supplied)
Treasury. Petitioners submit that Section 8 of PD 910 is not an
appropriation law since the “primary and specific” purpose of PD 910 is the Likewise, as ruled by the US Supreme Court in State of Nevada v. La
creation of an Energy Development Board and Section 8 thereof only Grave:242
created a Special Fund incidental thereto.237 In similar regard, petitioners
argue that Section 12 of PD 1869 is neither a valid appropriations law since To constitute an appropriation there must be money placed in a fund
the allocation of the Presidential Social Fund is merely incidental to the applicable to the designated purpose. The word appropriate means to
“primary and specific” purpose of PD 1869 which is the amendment of the allot, assign, set apart or apply to a particular use or purpose. An
Franchise and Powers of PAGCOR.238 In view of the foregoing, petitioners appropriation in the sense of the constitution means the setting apart a
suppose that such funds are being used without any valid law allowing for portion of the public funds for a public purpose. No particular form
their proper appropriation in violation of Section 29(1), Article VI of the of words is necessary for the purpose, if the intention to
1987 Constitution which states that: “No money shall be paid out of the appropriate is plainly manifested. (Emphases supplied)
Treasury except in pursuance of an appropriation made by law.”239
Thus, based on the foregoing, the Court cannot sustain the argument that
The Court disagrees. the appropriation must be the “primary and specific” purpose of the law in
order for a valid appropriation law to exist. To reiterate, if a legal provision
"An appropriation made by law” under the contemplation of Section 29(1), designates a determinate or determinable amount of money and allocates
Article VI of the 1987 Constitution exists when a provision of law (a) sets the same for a particular public purpose, then the legislative intent to
apart a determinate or determinable 240amountof money and (b) appropriate becomes apparent and, hence, already sufficient to satisfy the
allocates the same for a particular public purpose. These two minimum requirement of an “appropriation made by law” under contemplation of the
designations of amount and purpose stem from the very definition of the Constitution.
word “appropriation,” which means “to allot, assign, set apart or apply to a
particular use or purpose,” and hence, if written into the Section 8 of PD 910 pertinently provides:chanRoblesvirtualLawlibrary
law, demonstrate that the legislative intent to appropriate exists. As Section 8. Appropriations. x x x
the Constitution “does not provide or prescribe any particular form of
words or religious recitals in which an authorization or appropriation by All fees, revenues and receipts of the Board from any and all
Congress shall be made, except that it be ‘made by law,’ “ an appropriation sources including receipts from service contracts and agreements such as
law may – according to Philconsa – be “detailed and as broad as Congress application and processing fees, signature bonus, discovery bonus,
wants it to be” for as long as the intent to appropriate may be gleaned production bonus; all money collected from concessionaires, representing
from the same. As held in the case of Guingona, Jr.: 241 unspent work obligations, fines and penalties under the Petroleum Act of
1949; as well as the government share representing royalties, rentals,
[T]here is no provision in our Constitution that provides or production share on service contracts and similar payments on the
prescribes any particular form of words or religious recitals in exploration, development and exploitation of energy resources, shall form
which an authorization or appropriation by Congress shall be made, part of a Special Fund to be used to finance energy resource
except that it be “made by law,” such as precisely the authorization or development and exploitation programs and projects of the
appropriation under the questioned presidential decrees. In other words, in government and for such other purposes as may be hereafter
terms of time horizons, an appropriation may be made impliedly (as by directed by the President. (Emphases supplied)
past but subsisting legislations) as well as expressly for the current fiscal
year (as by enactment of laws by the present Congress), just as said Whereas Section 12 of PD 1869, as amended by PD 1993, reads:
Sec. 12. Special Condition of Franchise. — After deducting five (5%) apply the principle of ejusdem generis to the same section and thus,
percent as Franchise Tax, the Fifty (50%) percent share of the construe the phrase “and for such other purposes as may be hereafter
Government in the aggregate gross earnings of the Corporation directed by the President” to refer only to other purposes related “to
from this Franchise, or 60% if the aggregate gross earnings be less energy resource development and exploitation programs and projects of
than P150,000,000.00 shall be set aside and shall accrue to the General the government.”244
Fund to finance the priority infrastructure development projects and
to finance the restoration of damaged or destroyed facilities due to The Court agrees with petitioners’ submissions.
calamities, as may be directed and authorized by the Office of the
President of the Philippines.(Emphases supplied) While the designation of a determinate or determinable amount for a
particular public purpose is sufficient for a legal appropriation to exist, the
Analyzing the legal text vis–à–vis the above–mentioned principles, it may appropriation law must contain adequate legislative guidelines if the
then be concluded that (a) Section 8 of PD 910, which creates a Special same law delegates rule–making authority to the Executive245 either for
Fund comprised of “all fees, revenues, and receipts of the [Energy the purpose of (a) filling up the details of the law for its enforcement,
Development] Board from any and all sources” (a determinable amount) known as supplementary rule–making, or (b) ascertaining facts to bring
“to be used to finance energy resource development and exploitation the law into actual operation, referred to as contingent rule–
programs and projects of the government and for such other purposes as making.246There are two (2) fundamental tests to ensure that the
may be hereafter directed by the President” (a specified public legislative guidelines for delegated rule– making are indeed adequate. The
purpose), and (b) Section 12 of PD 1869, as amended by PD 1993, which first test is called the “completeness test.” Case law states that a law is
similarly sets aside, “[a]fter deducting five (5%) percent as Franchise Tax, complete when it sets forth therein the policy to be executed, carried out,
the Fifty (50%) percent share of the Government in the aggregate gross or implemented by the delegate. On the other hand, the second test is
earnings of [PAGCOR], or 60%[,] if the aggregate gross earnings be less called the “sufficient standard test.” Jurisprudence holds that a law lays
than P150,000,000.00” (also a determinable amount) “to finance the down a sufficient standard when it provides adequate guidelines or
priority infrastructure development projects and x x x the restoration of limitations in the law to map out the boundaries of the delegate’s authority
damaged or destroyed facilities due to calamities, as may be directed and and prevent the delegation from running riot.247 To be sufficient, the
authorized by the Office of the President of the Philippines” (also a standard must specify the limits of the delegate’s authority, announce the
specified public purpose), are legal appropriations under Section 29(1), legislative policy, and identify the conditions under which it is to be
Article VI of the 1987 Constitution. implemented.248

In this relation, it is apropos to note that the 2013 PDAF Article cannot be In view of the foregoing, the Court agrees with petitioners that the phrase
properly deemed as a legal appropriation under the said constitutional “and for such other purposes as may be hereafter directed by the
provision precisely because, as earlier stated, it contains post– enactment President” under Section 8 of PD 910 constitutes an undue delegation of
measures which effectively create a system of intermediate appropriations. legislative power insofar as it does not lay down a sufficient standard to
These intermediate appropriations are the actual appropriations meant for adequately determine the limits of the President’s authority with respect to
enforcement and since they are made by individual legislators after the the purpose for which the Malampaya Funds may be used. As it reads,
GAA is passed, they occur outside the law. As such, the Court observes the said phrase gives the President wide latitude to use the
that the real appropriation made under the 2013 PDAF Article is not the Malampaya Funds for any other purpose he may direct and, in
P24.79 Billion allocated for the entire PDAF, but rather the post–enactment effect, allows him to unilaterally appropriate public funds beyond
determinations made by the individual legislators which are, to repeat, the purview of the law. That the subject phrase may be confined only to
occurrences outside of the law. Irrefragably, the 2013 PDAF Article does “energy resource development and exploitation programs and projects of
not constitute an “appropriation made by law” since it, in its truest sense, the government” under the principle of ejusdem generis, meaning that the
only authorizes individual legislators to appropriate in violation of the general word or phrase is to be construed to include – or be restricted to –
non–delegability principle as afore–discussed. things akin to, resembling, or of the same kind or class as those specifically
2. Undue Delegation. mentioned,249 is belied by three (3) reasons: first, the phrase “energy
On a related matter, petitioners contend that Section 8 of PD 910 resource development and exploitation programs and projects of the
constitutes an undue delegation of legislative power since the phrase “and government” states a singular and general class and hence, cannot be
for such other purposes as may be hereafter directed by the President” treated as a statutory reference of specific things from which the general
gives the President “unbridled discretion to determine for what purpose the phrase “for such other purposes” may be limited; second, the said phrase
funds will be used.”243 Respondents, on the other hand, urged the Court to also exhausts the class it represents, namely energy development
programs of the government;250 and, third, the Executive department has, amended by PD 1993, remains legally effective and subsisting.
in fact, used the Malampaya Funds for non–energy related purposes under
the subject phrase, thereby contradicting respondents’ own position that it D. Ancillary Prayers.
is limited only to “energy resource development and exploitation programs
and projects of the government.”251 Thus, while Section 8 of PD 910 may 1. Petitioners’ Prayer to be Furnished Lists and Detailed Reports.
have passed the completeness test since the policy of energy development
is clearly deducible from its text, the phrase “and for such other purposes Aside from seeking the Court to declare the Pork Barrel System
as may be hereafter directed by the President” under the same provision of unconstitutional – as the Court did so in the context of its pronouncements
law should nonetheless be stricken down as unconstitutional as it lies made in this Decision – petitioners equally pray that the Executive
independently unfettered by any sufficient standard of the delegating law. Secretary and/or the DBM be ordered to release to the CoA and to the
This notwithstanding, it must be underscored that the rest of Section 8, public: (a) “the complete schedule/list of legislators who have availed of
insofar as it allows for the use of the Malampaya Funds “to finance energy their PDAF and VILP from the years 2003 to 2013, specifying the use of the
resource development and exploitation programs and projects of the funds, the project or activity and the recipient entities or individuals, and
government,” remains legally effective and subsisting. Truth be told, the all pertinent data thereto” (PDAF Use Schedule/List);254 and (b) “the use of
declared unconstitutionality of the aforementioned phrase is but an the Executive’s [lump–sum, discretionary] funds, including the proceeds
assurance that the Malampaya Funds would be used – as it should be used from the x x x Malampaya Fund[s] [and] remittances from the [PAGCOR] x
– only in accordance with the avowed purpose and intention of PD 910. x x from 2003 to 2013, specifying the x x x project or activity and the
recipient entities or individuals, and all pertinent data
As for the Presidential Social Fund, the Court takes judicial notice of the thereto”255 (Presidential Pork Use Report). Petitioners’ prayer is grounded
fact that Section 12 of PD 1869 has already been amended by PD 1993 on Section 28, Article II and Section 7, Article III of the 1987 Constitution
which thus moots the parties’ submissions on the same.252Nevertheless, which read as follows:
since the amendatory provision may be readily examined under the current
parameters of discussion, the Court proceeds to resolve its ARTICLE II
constitutionality.
Sec. 28. Subject to reasonable conditions prescribed by law, the State
Primarily, Section 12 of PD 1869, as amended by PD 1993, indicates that adopts and implements a policy of full public disclosure of all its
the Presidential Social Fund may be used “to [first,] finance the priority transactions involving public interest.
infrastructure development projects and [second,] to finance the
restoration of damaged or destroyed facilities due to calamities, as may be
ARTICLE III
directed and authorized by the Office of the President of the Philippines.”
The Court finds that while the second indicated purpose adequately curtails
Sec. 7. The right of the people to information on matters of public concern
the authority of the President to spend the Presidential Social Fund only for
shall be recognized. Access to official records, and to documents and
restoration purposes which arise from calamities, the first indicated
papers pertaining to official acts, transactions, or decisions, as well as to
purpose, however, gives him carte blanche authority to use the same fund
government research data used as basis for policy development, shall be
for any infrastructure project he may so determine as a “priority”. Verily,
afforded the citizen, subject to such limitations as may be provided by law.
the law does not supply a definition of “priority infrastructure development
projects” and hence, leaves the President without any guideline to construe
The Court denies petitioners’ submission.
the same. To note, the delimitation of a project as one of “infrastructure” is
too broad of a classification since the said term could pertain to any kind of
Case law instructs that the proper remedy to invoke the right to
facility. This may be deduced from its lexicographic definition as follows:
information is to file a petition for mandamus. As explained in the case
“[t]he underlying framework of a system, [especially] public services and
of Legaspi v. Civil Service Commission:256
facilities (such as highways, schools, bridges, sewers, and water–systems)
needed to support commerce as well as economic and residential
development.”253 In fine, the phrase “to finance the priority infrastructure [W]hile the manner of examining public records may be subject to
development projects” must be stricken down as unconstitutional since – reasonable regulation by the government agency in custody thereof, the
similar to the above– assailed provision under Section 8 of PD 910 – it lies duty to disclose the information of public concern, and to afford access to
independently unfettered by any sufficient standard of the delegating law. public records cannot be discretionary on the part of said agencies.
As they are severable, all other provisions of Section 12 of PD 1869, as Certainly, its performance cannot be made contingent upon the discretion
of such agencies. Otherwise, the enjoyment of the constitutional right may petitioners pray that said information be equally released to the CoA, it
be rendered nugatory by any whimsical exercise of agency discretion. The must be pointed out that the CoA has not been impleaded as a party to
constitutional duty, not being discretionary, its performance may be these cases nor has it filed any petition before the Court to be allowed
compelled by a writ of mandamus in a proper case. access to or to compel the release of any official document relevant to the
conduct of its audit investigations. While the Court recognizes that the
But what is a proper case for Mandamus to issue? In the case before Us, information requested is a matter of significant public concern, however, if
the public right to be enforced and the concomitant duty of the State are only to ensure that the parameters of disclosure are properly foisted and so
unequivocably set forth in the Constitution. The decisive question on the as not to unduly hamper the equally important interests of the
propriety of the issuance of the writ of mandamus in this case is, government, it is constrained to deny petitioners’ prayer on this score,
whether the information sought by the petitioner is within the without prejudice to a proper mandamus case which they, or even the CoA,
ambit of the constitutional guarantee. (Emphases may choose to pursue through a separate petition.
supplied)chanroblesvirtualawlibrary
It bears clarification that the Court’s denial herein should only cover
Corollarily, in the case of Valmonte v. Belmonte Jr.257 (Valmonte), it has petitioners’ plea to be furnished with such schedule/list and report and not
been clarified that the right to information does not include the right to in any way deny them, or the general public, access to official documents
compel the preparation of “lists, abstracts, summaries and the like.” In the which are already existing and of public record. Subject to reasonable
same case, it was stressed that it is essential that the “applicant has a regulation and absent any valid statutory prohibition, access to
well– defined, clear and certain legal right to the thing demanded and that these documents should not be proscribed. Thus, in Valmonte, while
it is the imperative duty of defendant to perform the act required.” Hence, the Court denied the application for mandamus towards the preparation of
without the foregoing substantiations, the Court cannot grant a particular the list requested by petitioners therein, it nonetheless allowed access to
request for information. The pertinent portions of Valmonte are hereunder the documents sought for by the latter, subject, however, to the
quoted:258 custodian’s reasonable regulations, viz.:259

Although citizens are afforded the right to information and, pursuant In fine, petitioners are entitled to access to the documents evidencing
thereto, are entitled to “access to official records,” the Constitution does loans granted by the GSIS, subject to reasonable regulations that the latter
not accord them a right to compel custodians of official records to may promulgate relating to the manner and hours of examination, to the
prepare lists, abstracts, summaries and the like in their desire to end that damage to or loss of the records may be avoided, that undue
acquire information on matters of public concern. interference with the duties of the custodian of the records may be
prevented and that the right of other persons entitled to inspect the
It must be stressed that it is essential for a writ of mandamus to issue records may be insured [Legaspi v. Civil Service Commission, supra at p.
that the applicant has a well–defined, clear and certain legal right 538, quoting Subido v. Ozaeta,80 Phil. 383, 387.] The petition, as to the
to the thing demanded and that it is the imperative duty of second and third alternative acts sought to be done by petitioners, is
defendant to perform the act required. The corresponding duty of the meritorious.
respondent to perform the required act must be clear and specific [Lemi v.
Valencia, G.R. No. L–20768, November 29,1968,126 SCRA 203; Ocampo v. However, the same cannot be said with regard to the first act sought by
Subido, G.R. No. L–28344, August 27, 1976, 72 SCRA 443.] The request petitioners, i.e., “to furnish petitioners the list of the names of the
of the petitioners fails to meet this standard, there being no duty Batasang Pambansa members belonging to the UNIDO and PDP– Laban
on the part of respondent to prepare the list requested. (Emphases who were able to secure clean loans immediately before the February 7
supplied) election thru the intercession/marginal note of the then First Lady Imelda
Marcos.”
In these cases, aside from the fact that none of the petitions are in the
nature of mandamus actions, the Court finds that petitioners have failed to The Court, therefore, applies the same treatment here.
establish a “a well–defined, clear and certain legal right” to be furnished by
the Executive Secretary and/or the DBM of their requested PDAF Use 2. Petitioners’ Prayer to Include Matters in Congressional
Schedule/List and Presidential Pork Use Report. Neither did petitioners Deliberations.
assert any law or administrative issuance which would form the bases of
the latter’s duty to furnish them with the documents requested. While Petitioners further seek that the Court “[order] the inclusion in budgetary
deliberations with the Congress of all presently, off–budget, lump sum, At the outset, it must be observed that the issue of whether or not the
discretionary funds including but not limited to, proceeds from the x x x Court’s September 10, 2013 TRO should be lifted is a matter rendered
Malampaya Fund, remittances from the [PAGCOR] and the [PCSO] or the moot by the present Decision. The unconstitutionality of the 2013 PDAF
Executive’s Social Funds[.]”260 Article as declared herein has the consequential effect of converting the
temporary injunction into a permanent one. Hence, from the
Suffice it to state that the above–stated relief sought by petitioners covers promulgation of this Decision, the release of the remaining PDAF
a matter which is generally left to the prerogative of the political branches funds for 2013, among others, is now permanently enjoined.
of government. Hence, lest the Court itself overreach, it must equally deny
their prayer on this score. The propriety of the DBM’s interpretation of the concept of “release” must,
nevertheless, be resolved as it has a practical impact on the execution of
3. Respondents’ Prayer to Lift TRO; Consequential Effects of the current Decision. In particular, the Court must resolve the issue of
Decision. whether or not PDAF funds covered by obligated SAROs, at the time this
Decision is promulgated, may still be disbursed following the DBM’s
The final issue to be resolved stems from the interpretation accorded by interpretation in DBM Circular 2013–8.
the DBM to the concept of released funds. In response to the Court’s
September 10, 2013 TRO that enjoined the release of the remaining PDAF On this score, the Court agrees with petitioners’ posturing for the
allocated for the year 2013, the DBM issued Circular Letter No. 2013–8 fundamental reason that funds covered by an obligated SARO are yet to be
dated September 27, 2013 (DBM Circular 2013–8) which pertinently reads “released” under legal contemplation. A SARO, as defined by the DBM itself
as follows: in its website, is “[a]specific authority issued to identified agencies to
incur obligations not exceeding a given amount during a specified period
3.0 Nonetheless, PDAF projects funded under the FY 2013 GAA, where a for the purpose indicated. It shall cover expenditures the release of
Special Allotment Release Order (SARO) has been issued by the DBM and which is subject to compliance with specific laws or regulations, or
such SARO has been obligated by the implementing agencies prior to the is subject to separate approval or clearance by competent
issuance of the TRO, may continually be implemented and disbursements authority.”263 Based on this definition, it may be gleaned that a SARO only
thereto effected by the agencies concerned. evinces the existence of an obligation and not the directive to pay.
Practically speaking, the SARO does not have the direct and immediate
Based on the text of the foregoing, the DBM authorized the continued effect of placing public funds beyond the control of the disbursing
implementation and disbursement of PDAF funds as long as they are: first, authority. In fact, a SARO may even be withdrawn under certain
covered by a SARO; and, second, that said SARO had been obligated by circumstances which will prevent the actual release of funds. On the other
the implementing agency concerned prior to the issuance of the Court’s hand, the actual release of funds is brought about by the issuance of the
September 10, 2013 TRO. NCA,264 which is subsequent to the issuance of a SARO. As may be
determined from the statements of the DBM representative during the Oral
Petitioners take issue with the foregoing circular, arguing that ?the Arguments:265
issuance of the SARO does not yet involve the release of funds under the
PDAF, as release is only triggered by the issuance of a Notice of Cash Justice Bernabe: Is the notice of allocation issued simultaneously with the
Allocation [(NCA)].”261 As such, PDAF disbursements, even if covered by an SARO?
obligated SARO, should remain enjoined.
xxx
For their part, respondents espouse that the subject TRO only covers
“unreleased and unobligated allotments.” They explain that once a SARO Atty. Ruiz: It comes after. The SARO, Your Honor, is only the go signal
has been issued and obligated by the implementing agency concerned, the for the agencies to obligate or to enter into commitments. The NCA,
PDAF funds covered by the same are already “beyond the reach of the TRO Your Honor, is already the go signal to the treasury for us to be
because they cannot be considered as ‘remaining PDAF.’ “ They conclude able to pay or to liquidate the amounts obligated in the SARO; so it
that this is a reasonable interpretation of the TRO by the DBM.262 comes after. x x x The NCA, Your Honor, is the go signal for the MDS for
the authorized government–disbursing banks to, therefore, pay the payees
The Court agrees with petitioners in part. depending on the projects or projects covered by the SARO and the NCA.
Justice Bernabe: Are there instances that SAROs are cancelled or revoked? complied with. As explained in the recent case of Commissioner of Internal
Revenue v. San Roque Power Corporation,266the doctrine merely “reflect[s]
Atty. Ruiz: Your Honor, I would like to instead submit that there are awareness that precisely because the judiciary is the governmental organ
instances that the SAROs issued are withdrawn by the DBM. which has the final say on whether or not a legislative or executive
measure is valid, a period of time may have elapsed before it can exercise
Justice Bernabe: They are withdrawn? the power of judicial review that may lead to a declaration of nullity. It
would be to deprive the law of its quality of fairness and justice then, if
Atty. Ruiz: Yes, Your Honor x x x. (Emphases and underscoring supplied) there be no recognition of what had transpired prior to such
adjudication.”267 “In the language of an American Supreme Court decision:
Thus, unless an NCA has been issued, public funds should not be treated as ‘The actual existence of a statute, prior to such a determination [of
funds which have been “released.” In this respect, therefore, the unconstitutionality], is an operative fact and may have consequences
disbursement of 2013 PDAF funds which are only covered by obligated which cannot justly be ignored.’ “268
SAROs, and without any corresponding NCAs issued, must, at the time of
this Decision’s promulgation, be enjoined and consequently reverted For these reasons, this Decision should be heretofore applied prospectively.
to the unappropriated surplus of the general fund. Verily, in view of
the declared unconstitutionality of the 2013 PDAF Article, the funds Conclusion
appropriated pursuant thereto cannot be disbursed even though already
obligated, else the Court sanctions the dealing of funds coming from an The Court renders this Decision to rectify an error which has persisted in
unconstitutional source. the chronicles of our history. In the final analysis, the Court must strike
down the Pork Barrel System as unconstitutional in view of the inherent
This same pronouncement must be equally applied to (a) the Malampaya defects in the rules within which it operates. To recount, insofar as it has
Funds which have been obligated but not released – meaning, those merely allowed legislators to wield, in varying gradations, non–oversight, post–
covered by a SARO – under the phrase “and for such other purposes as enactment authority in vital areas of budget execution, the system has
may be hereafter directed by the President” pursuant to Section 8 of PD violated the principle of separation of powers; insofar as it has
910; and (b) funds sourced from the Presidential Social Fund under the conferred unto legislators the power of appropriation by giving them
phrase “to finance the priority infrastructure development projects” personal, discretionary funds from which they are able to fund specific
pursuant to Section 12 of PD 1869, as amended by PD 1993, which were projects which they themselves determine, it has similarly violated
altogether declared by the Court as unconstitutional. However, these funds the principle of non– delegability of legislative power; insofar as it
should not be reverted to the general fund as afore–stated but instead, has created a system of budgeting wherein items are not textualized into
respectively remain under the Malampaya Funds and the Presidential Social the appropriations bill, it has flouted the prescribed procedure of
Fund to be utilized for their corresponding special purposes not otherwise presentment and, in the process, denied the President the power to
declared as unconstitutional. veto items; insofar as it has diluted the effectiveness of congressional
oversight by giving legislators a stake in the affairs of budget execution, an
E. Consequential Effects of Decision. aspect of governance which they may be called to monitor and scrutinize,
the system has equally impaired public accountability; insofar as it has
As a final point, it must be stressed that the Court’s pronouncement anent authorized legislators, who are national officers, to intervene in affairs of
the unconstitutionality of (a) the 2013 PDAF Article and its Special purely local nature, despite the existence of capable local institutions, it
Provisions, (b) all other Congressional Pork Barrel provisions similar has likewise subverted genuine local autonomy; and again, insofar as it
thereto, and (c) the phrases (1) ?and for such other purposes as may be has conferred to the President the power to appropriate funds intended by
hereafter directed by the President” under Section 8 of PD 910, and (2) “to law for energy–related purposes only to other purposes he may deem fit as
finance the priority infrastructure development projects” under Section 12 well as other public funds under the broad classification of “priority
of PD 1869, as amended by PD 1993, must only be treated as prospective infrastructure development projects,” it has once more transgressed the
in effect in view of the operative fact doctrine. principle of non–delegability.

To explain, the operative fact doctrine exhorts the recognition that until the For as long as this nation adheres to the rule of law, any of the multifarious
judiciary, in an appropriate case, declares the invalidity of a certain unconstitutional methods and mechanisms the Court has herein pointed
legislative or executive act, such act is presumed constitutional and thus, out should never again be adopted in any system of governance, by any
entitled to obedience and respect and should be properly enforced and
name or form, by any semblance or similarity, by any influence or effect. remaining PDAF funds covered by this permanent injunction shall not be
Disconcerting as it is to think that a system so constitutionally unsound has disbursed/released but instead reverted to the unappropriated surplus of
monumentally endured, the Court urges the people and its co– stewards in the general fund, while the funds under the Malampaya Funds and the
government to look forward with the optimism of change and the Presidential Social Fund shall remain therein to be utilized for their
awareness of the past. At a time of great civic unrest and vociferous public respective special purposes not otherwise declared as unconstitutional.
debate, the Court fervently hopes that its Decision today, while it may not
purge all the wrongs of society nor bring back what has been lost, guides On the other hand, due to improper recourse and lack of proper
this nation to the path forged by the Constitution so that no one may substantiation, the Court hereby DENIES petitioners’ prayer seeking that
heretofore detract from its cause nor stray from its course. After all, this is the Executive Secretary and/or the Department of Budget and
the Court’s bounden duty and no other’s . Management be ordered to provide the public and the Commission on Audit
complete lists/schedules or detailed reports related to the availments and
WHEREFORE, the petitions are PARTLY GRANTED. In view of the utilization of the funds subject of these cases. Petitioners’ access to official
constitutional violations discussed in this Decision, the Court hereby documents already available and of public record which are related to these
declares as UNCONSTITUTIONAL: (a) the entire 2013 PDAF Article; (b) funds must, however, not be prohibited but merely subjected to the
all legal provisions of past and present Congressional Pork Barrel Laws, custodian’s reasonable regulations or any valid statutory prohibition on the
such as the previous PDAF and CDF Articles and the various Congressional same. This denial is without prejudice to a proper mandamus case which
Insertions, which authorize/d legislators – whether individually or they or the Commission on Audit may choose to pursue through a separate
collectively organized into committees – to intervene, assume or petition.
participate in any of the various post–enactment stages of the budget
execution, such as but not limited to the areas of project identification, The Court also DENIES petitioners’ prayer to order the inclusion of the
modification and revision of project identification, fund release and/or fund funds subject of these cases in the budgetary deliberations of Congress as
realignment, unrelated to the power of congressional oversight; (c) all the same is a matter left to the prerogative of the political branches of
legal provisions of past and present Congressional Pork Barrel Laws, such government.
as the previous PDAF and CDF Articles and the various Congressional
Insertions, which confer/red personal, lump–sum allocations to legislators Finally, the Court hereby DIRECTS all prosecutorial organs of the
from which they are able to fund specific projects which they themselves government to, within the bounds of reasonable dispatch, investigate and
determine; (d) all informal practices of similar import and effect, which the accordingly prosecute all government officials and/or private individuals for
Court similarly deems to be acts of grave abuse of discretion amounting to possible criminal offenses related to the irregular, improper and/or unlawful
lack or excess of jurisdiction; and (e) the phrases (1) ?and for such other disbursement/utilization of all funds under the Pork Barrel System.
purposes as may be hereafter directed by the President” under Section 8 of
Presidential Decree No. 910 and (2) “to finance the priority infrastructure This Decision is immediately executory but prospective in
development projects” under Section 12 of Presidential Decree No. 1869, effect.ChanRoblesVirtualawlibrary
as amended by Presidential Decree No. 1993, for both failing the sufficient
standard test in violation of the principle of non–delegability of legislative SO ORDERED.
power.

Accordingly, the Court’s temporary injunction dated September 10, 2013 is


hereby declared to be PERMANENT. Thus, the disbursement/release of
the remaining PDAF funds allocated for the year 2013, as well as for all
previous years, and the funds sourced from (1) the Malampaya Funds
under the phrase “and for such other purposes as may be hereafter
directed by the President” pursuant to Section 8 of Presidential Decree No.
910, and (2) the Presidential Social Fund under the phrase ?to finance the
priority infrastructure development projects” pursuant to Section 12 of
Presidential Decree No. 1869, as amended by Presidential Decree No.
1993, which are, at the time this Decision is promulgated, not covered by
Notice of Cash Allocations (NCAs) but only by Special Allotment Release
Orders (SAROs), whether obligated or not, are hereby ENJOINED. The
x-----------------------x "incentive" for voting in favor of the impeachment of Chief Justice Renato
C. Corona.
G.R. No. 209135
Responding to Sen. Estrada’s revelation, Secretary Florencio Abad of the
DBM issued a public statement entitled Abad: Releases to Senators Part of
AUGUSTO L. SY JUCO JR., Ph.D., Petitioner, 
Spending Acceleration Program,1 explaining that the funds released to the
vs.
Senators had been part of the DAP, a program designed by the DBM to
FLORENCIO B. ABAD, IN HIS CAPACITY AS THE SECRETARY OF
ramp up spending to accelerate economic expansion. He clarified that the
DEPARTMENT OF BUDGET AND MANAGEMENT; AND HON. FRANKLIN
funds had been released to the Senators based on their letters of request
MAGTUNAO DRILON, IN HIS CAP A CITY AS THE SENATE
for funding; and that it was not the first time that releases from the DAP
PRESIDENT OF THE PHILIPPINES, Respondents.
had been made because the DAP had already been instituted in 2011 to
ramp up spending after sluggish disbursements had caused the growth of
x-----------------------x the gross domestic product (GDP) to slow down. He explained that the
funds under the DAP were usually taken from (1) unreleased
DECISION appropriations under Personnel Services;2 (2) unprogrammed funds; (3)
carry-over appropriations unreleased from the previous year; and (4)
budgets for slow-moving items or projects that had been realigned to
BERSAMIN, J.: support faster-disbursing projects.

For resolution are the consolidated petitions assailing the constitutionality The DBM soon came out to claim in its website3 that the DAP releases had
of the Disbursement Acceleration Program(DAP), National Budget Circular been sourced from savings generated by the Government, and from
(NBC) No. 541, and related issuances of the Department of Budget and unprogrammed funds; and that the savings had been derived from (1) the
Management (DBM) implementing the DAP. pooling of unreleased appropriations, like unreleased Personnel
Services4 appropriations that would lapse at the end of the year,
At the core of the controversy is Section 29(1) of Article VI of the 1987 unreleased appropriations of slow-moving projects and discontinued
Constitution, a provision of the fundamental law that firmly ordains that projects per zero based budgeting findings;5 and (2) the withdrawal of
"[n]o money shall be paid out of the Treasury except in pursuance of an unobligated allotments also for slow-moving programs and projects that
appropriation made by law." The tenor and context of the challenges posed had been earlier released to the agencies of the National Government.
by the petitioners against the DAP indicate that the DAP contravened this
provision by allowing the Executive to allocate public money pooled from The DBM listed the following as the legal bases for the DAP’s use of
programmed and unprogrammed funds of its various agencies in the guise savings,6 namely: (1) Section 25(5), Article VI of the 1987 Constitution,
of the President exercising his constitutional authority under Section 25(5) which granted to the President the authority to augment an item for his
of the 1987 Constitution to transfer funds out of savings to augment the office in the general appropriations law; (2) Section 49 (Authority to Use
appropriations of offices within the Executive Branch of the Government. Savings for Certain Purposes) and Section 38 (Suspension of Expenditure
But the challenges are further complicated by the interjection of allegations Appropriations), Chapter 5, Book VI of Executive Order (EO) No. 292
of transfer of funds to agencies or offices outside of the Executive. (Administrative Code of 1987); and (3) the General Appropriations Acts
(GAAs) of 2011, 2012 and 2013, particularly their provisions on the (a) use
Antecedents of savings; (b) meanings of savings and augmentation; and (c) priority in
the use of savings.
What has precipitated the controversy?
As for the use of unprogrammed funds under the DAP, the DBM cited as
legal bases the special provisions on unprogrammed fund contained in the
On September 25, 2013, Sen. Jinggoy Ejercito Estrada delivered a privilege GAAs of 2011, 2012 and 2013.
speech in the Senate of the Philippines to reveal that some Senators,
including himself, had been allotted an additional ₱50 Million each as
The revelation of Sen. Estrada and the reactions of Sec. Abad and the DBM this issue are whether there is a controversy ripe for judicial determination,
brought the DAP to the consciousness of the Nation for the first time, and and the standing of petitioners.
made this present controversy inevitable. That the issues against the DAP
came at a time when the Nation was still seething in anger over
Substantive Issues:
Congressional pork barrel – "an appropriation of government spending
meant for localized projects and secured solely or primarily to bring money
to a representative’s district"7 – excited the Nation as heatedly as the pork B. Whether or not the DAP violates Sec. 29, Art. VI of the 1987
barrel controversy. Constitution, which provides: "No money shall be paid out of the Treasury
except in pursuance of an appropriation made by law."
Nine petitions assailing the constitutionality of the DAP and the issuances
relating to the DAP were filed within days of each other, as follows: G.R. C. Whether or not the DAP, NBC No. 541, and all other executive issuances
No. 209135 (Syjuco), on October 7, 2013; G.R. No. 209136 (Luna), on allegedly implementing the DAP violate Sec. 25(5), Art. VI of the 1987
October 7, 2013; G.R. No. 209155 (Villegas),8 on October 16, 2013; G.R. Constitution insofar as:
No. 209164 (PHILCONSA), on October 8, 2013; G.R. No. 209260 (IBP), on
October 16, 2013; G.R. No. 209287 (Araullo), on October 17, 2013; G.R. (a)They treat the unreleased appropriations and
No. 209442 (Belgica), on October 29, 2013; G.R. No. 209517 (COURAGE), unobligated allotments withdrawn from government
on November6, 2013; and G.R. No. 209569 (VACC), on November 8, 2013. agencies as "savings" as the term is used in Sec. 25(5),
in relation to the provisions of the GAAs of 2011, 2012
In G.R. No. 209287 (Araullo), the petitioners brought to the Court’s and 2013;
attention NBC No. 541 (Adoption of Operational Efficiency Measure –
Withdrawal of Agencies’ Unobligated Allotments as of June 30, 2012), (b)They authorize the disbursement of funds for projects
alleging that NBC No. 541, which was issued to implement the DAP, or programs not provided in the GAAs for the Executive
directed the withdrawal of unobligated allotments as of June 30, 2012 of Department; and
government agencies and offices with low levels of obligations, both for
continuing and current allotments.
(c)They "augment" discretionary lump sum
appropriations in the GAAs.
In due time, the respondents filed their Consolidated Comment through the
Office of the Solicitor General (OSG).
D. Whether or not the DAP violates: (1) the Equal Protection Clause, (2)
the system of checks and balances, and (3) the principle of public
The Court directed the holding of oral arguments on the significant issues accountability enshrined in the 1987 Constitution considering that it
raised and joined. authorizes the release of funds upon the request of legislators.

Issues E. Whether or not factual and legal justification exists to issue a temporary
restraining order to restrain the implementation of the DAP, NBC No. 541,
Under the Advisory issued on November 14, 2013, the presentations of the and all other executive issuances allegedly implementing the DAP.
parties during the oral arguments were limited to the following, to wit:
In its Consolidated Comment, the OSG raised the matter of unprogrammed
Procedural Issue: funds in order to support its argument regarding the President’s power to
spend. During the oral arguments, the propriety of releasing
unprogrammed funds to support projects under the DAP was considerably
A. Whether or not certiorari, prohibition, and mandamus are proper
discussed. The petitioners in G.R. No. 209287 (Araullo) and G.R. No.
remedies to assail the constitutionality and validity of the Disbursement
209442 (Belgica) dwelled on unprogrammed funds in their respective
Acceleration Program (DAP), National Budget Circular (NBC) No. 541, and
memoranda. Hence, an additional issue for the oral arguments is stated as
all other executive issuances allegedly implementing the DAP. Subsumed in
follows:
F. Whether or not the release of unprogrammed funds under the DAP was (3) A breakdown of the sources of savings, including savings from
in accord with the GAAs. discontinued projects and unpaid appropriations for compensation
from 2011 to 2013
During the oral arguments held on November 19, 2013, the Court directed
Sec. Abad to submit a list of savings brought under the DAP that had been On January 28, 2014, the OSG, to comply with the Resolution issued on
sourced from (a) completed programs; (b) discontinued or abandoned January 21, 2014 directing the respondents to submit the documents not
programs; (c) unpaid appropriations for compensation; (d) a certified copy yet submitted in compliance with the directives of the Court or its
of the President’s directive dated June 27, 2012 referred to in NBC No. Members, submitted several evidence packets to aid the Court in
541; and (e) all circulars or orders issued in relation to the DAP.9 understanding the factual bases of the DAP, to wit:

In compliance, the OSG submitted several documents, as follows: (1) First Evidence Packet11 – containing seven memoranda issued
by the DBM through Sec. Abad, inclusive of annexes, listing in
detail the 116 DAP identified projects approved and duly signed by
(1) A certified copy of the Memorandum for the President dated
the President, as follows:
June 25, 2012 (Omnibus Authority to Consolidate
Savings/Unutilized Balances and their Realignment);10
a. Memorandum for the President dated October 12,
2011 (FY 2011 Proposed Disbursement Acceleration
(2) Circulars and orders, which the respondents identified as
Program (Projects and Sources of Funds);
related to the DAP, namely:

b. Memorandum for the President dated December 12,


a. NBC No. 528 dated January 3, 2011 (Guidelines on the
2011 (Omnibus Authority to Consolidate
Release of Funds for FY 2011);
Savings/Unutilized Balances and its Realignment);

b. NBC No. 535 dated December 29, 2011 (Guidelines on


c. Memorandum for the President dated June 25, 2012
the Release of Funds for FY 2012);
(Omnibus Authority to Consolidate Savings/Unutilized
Balances and their Realignment);
c. NBC No. 541 dated July 18, 2012 (Adoption of
Operational Efficiency Measure – Withdrawal of Agencies’
d. Memorandum for the President dated September 4,
Unobligated Allotments as of June 30, 2012);
2012 (Release of funds for other priority projects and
expenditures of the Government);
d. NBC No. 545 dated January 2, 2013 (Guidelines on the
Release of Funds for FY 2013);
e. Memorandum for the President dated December 19,
2012 (Proposed Priority Projects and Expenditures of the
e. DBM Circular Letter No. 2004-2 dated January 26, Government);
2004 (Budgetary Treatment of Commitments/Obligations
of the National Government);
f. Memorandum for the President dated May 20, 2013
(Omnibus Authority to Consolidate Savings/Unutilized
f. COA-DBM Joint Circular No. 2013-1 dated March 15, Balances and their Realignment to Fund the Quarterly
2013 (Revised Guidelines on the Submission of Quarterly Disbursement Acceleration Program); and
Accountability Reports on Appropriations, Allotments,
Obligations and Disbursements);
g. Memorandum for the President dated September 25,
2013 (Funding for the Task Force Pablo Rehabilitation
g. NBC No. 440 dated January 30, 1995 (Adoption of a Plan).
Simplified Fund Release System in the Government).
(2) Second Evidence Packet12 – consisting of 15 applications of the a) The petitions under Rule 65 are proper remedies
DAP, with their corresponding Special Allotment Release Orders
(SAROs) and appropriation covers;
All the petitions are filed under Rule 65 of the Rules of Court, and include
applications for the issuance of writs of preliminary prohibitory injunction or
(3) Third Evidence Packet13 – containing a list and descriptions of temporary restraining orders. More specifically, the nature of the petitions
12 projects under the DAP; is individually set forth hereunder, to wit:

(4) Fourth Evidence Packet14 – identifying the DAP-related portions


Certiorari, Prohibition and
of the Annual Financial Report (AFR) of the Commission on Audit G.R. No. 209135 (Syjuco) 
Mandamus
for 2011 and 2012;
G.R. No. 209136 (Luna)  Certiorariand Prohibition
(5) Fifth Evidence Packet15 – containing a letter of Department of
Transportation and Communications(DOTC) Sec. Joseph Abaya G.R. No. 209155 (Villegas)  Certiorariand Prohibition
addressed to Sec. Abad recommending the withdrawal of funds
from his agency, inclusive of annexes; and G.R. No. 209164
Certiorariand Prohibition
(PHILCONSA) 
(6) Sixth Evidence Packet16 – a print-out of the Solicitor General’s
visual presentation for the January 28, 2014 oral arguments. G.R. No. 209260 (IBP)  Prohibition

G.R. No. 209287 (Araullo)  Certiorariand Prohibition


On February 5, 2014,17 the OSG forwarded the Seventh Evidence
Packet,18 which listed the sources of funds brought under the DAP, the uses G.R. No. 209442 (Belgica)  Certiorari
of such funds per project or activity pursuant to DAP, and the legal bases
thereof. G.R. No. 209517
Certiorari and Prohibition
(COURAGE) 
On February 14, 2014, the OSG submitted another set of documents in
further compliance with the Resolution dated January 28, 2014, viz: G.R. No. 209569 (VACC)  Certiorari and Prohibition

(1) Certified copies of the certifications issued by the Bureau of Treasury to


The respondents submit that there is no actual controversy that is ripe for
the effect that the revenue collections exceeded the original revenue
adjudication in the absence of adverse claims between the parties;19 that
targets for the years 2011, 2012 and 2013, including collections arising
the petitioners lacked legal standing to sue because no allegations were
from sources not considered in the original revenue targets, which
made to the effect that they had suffered any injury as a result of the
certifications were required for the release of the unprogrammed funds as
adoption of the DAP and issuance of NBC No. 541; that their being
provided in Special Provision No. 1 of Article XLV, Article XVI, and Article
taxpayers did not immediately confer upon the petitioners the legal
XLV of the 2011, 2012 and 2013 GAAs; and (2) A report on releases of
standing to sue considering that the adoption and implementation of the
savings of the Executive Department for the use of the Constitutional
DAP and the issuance of NBC No. 541 were not in the exercise of the taxing
Commissions and other branches of the Government, as well as the fund
or spending power of Congress;20 and that even if the petitioners had
releases to the Senate and the Commission on Elections (COMELEC).
suffered injury, there were plain, speedy and adequate remedies in the
ordinary course of law available to them, like assailing the regularity of the
RULING DAP and related issuances before the Commission on Audit (COA) or in the
trial courts.21
I.
The respondents aver that the special civil actions of certiorari and
Procedural Issue: prohibition are not proper actions for directly assailing the constitutionality
and validity of the DAP, NBC No. 541, and the other executive issuances constitutionally created court, the rest being created by Congress in its
implementing the DAP.22 exercise of the legislative power.

In their memorandum, the respondents further contend that there is no The Constitution states that judicial power includes the duty of the courts
authorized proceeding under the Constitution and the Rules of Court for of justice not only "to settle actual controversies involving rights which are
questioning the validity of any law unless there is an actual case or legally demandable and enforceable" but also "to determine whether or not
controversy the resolution of which requires the determination of the there has been a grave abuse of discretion amounting to lack or excess of
constitutional question; that the jurisdiction of the Court is largely jurisdiction on the part of any branch or instrumentality of the
appellate; that for a court of law to pass upon the constitutionality of a law Government." It has thereby expanded the concept of judicial power, which
or any act of the Government when there is no case or controversy is for up to then was confined to its traditional ambit of settling actual
that court to set itself up as a reviewer of the acts of Congress and of the controversies involving rights that were legally demandable and
President in violation of the principle of separation of powers; and that, in enforceable.
the absence of a pending case or controversy involving the DAP and NBC
No. 541, any decision herein could amount to a mere advisory opinion that
The background and rationale of the expansion of judicial power under the
no court can validly render.23
1987 Constitution were laid out during the deliberations of the 1986
Constitutional Commission by Commissioner Roberto R. Concepcion (a
The respondents argue that it is the application of the DAP to actual former Chief Justice of the Philippines) in his sponsorship of the proposed
situations that the petitioners can question either in the trial courts or in provisions on the Judiciary, where he said:–
the COA; that if the petitioners are dissatisfied with the ruling either of the
trial courts or of the COA, they can appeal the decision of the trial courts
The Supreme Court, like all other courts, has one main function: to settle
by petition for review on certiorari, or assail the decision or final order of
actual controversies involving conflicts of rights which are demandable and
the COA by special civil action for certiorari under Rule 64 of the Rules of
enforceable. There are rights which are guaranteed by law but cannot be
Court.24
enforced by a judicial party. In a decided case, a husband complained that
his wife was unwilling to perform her duties as a wife. The Court said: "We
The respondents’ arguments and submissions on the procedural issue are can tell your wife what her duties as such are and that she is bound to
bereft of merit. comply with them, but we cannot force her physically to discharge her
main marital duty to her husband. There are some rights guaranteed by
law, but they are so personal that to enforce them by actual compulsion
Section 1, Article VIII of the 1987 Constitution expressly provides:
would be highly derogatory to human dignity." This is why the first part of
the second paragraph of Section 1 provides that: Judicial power includes
Section 1. The judicial power shall be vested in one Supreme Court and in the duty of courts to settle actual controversies involving rights which are
such lower courts as may be established by law. legally demandable or enforceable…

Judicial power includes the duty of the courts of justice to settle actual The courts, therefore, cannot entertain, much less decide, hypothetical
controversies involving rights which are legally demandable and questions. In a presidential system of government, the Supreme Court has,
enforceable, and to determine whether or not there has been a grave also, another important function. The powers of government are generally
abuse of discretion amounting to lack or excess of jurisdiction on the part considered divided into three branches: the Legislative, the Executive and
of any branch or instrumentality of the Government. the Judiciary. Each one is supreme within its own sphere and independent
of the others. Because of that supremacy power to determine whether a
Thus, the Constitution vests judicial power in the Court and in such lower given law is valid or not is vested in courts of justice.
courts as may be established by law. In creating a lower court, Congress
concomitantly determines the jurisdiction of that court, and that court, Briefly stated, courts of justice determine the limits of power of the
upon its creation, becomes by operation of the Constitution one of the agencies and offices of the government as well as those of its officers. In
repositories of judicial power.25 However, only the Court is a other words, the judiciary is the final arbiter on the question whether or
not a branch of government or any of its officials has acted without
jurisdiction or in excess of jurisdiction, or so capriciously as to constitute an MR. CONCEPCION. No. Judicial power, as I said, refers to ordinary cases
abuse of discretion amounting to excess of jurisdiction or lack of but where there is a question as to whether the government had authority
jurisdiction. This is not only a judicial power but a duty to pass judgmenton or had abused its authority to the extent of lacking jurisdiction or excess of
matters of this nature. jurisdiction, that is not a political question. Therefore, the court has the
duty to decide.27
This is the background of paragraph 2 of Section 1, which means that the
courts cannot hereafter evade the duty to settle matters of this nature, by Our previous Constitutions equally recognized the extent of the power of
claiming that such matters constitute a political question. (Bold emphasis judicial review and the great responsibility of the Judiciary in maintaining
supplied)26 the allocation of powers among the three great branches of Government.
Speaking for the Court in Angara v. Electoral Commission,28 Justice Jose P.
Laurel intoned:
Upon interpellation by Commissioner Nolledo, Commissioner Concepcion
clarified the scope of judicial power in the following manner:–
x x x In times of social disquietude or political excitement, the great
landmarks of the Constitution are apt to be forgotten or marred, if not
MR. NOLLEDO. x x x
entirely obliterated. In cases of conflict, the judicial department is the only
constitutional organ which can be called upon to determine the proper
The second paragraph of Section 1 states: "Judicial power includes the duty allocation of powers between the several department and among the
of courts of justice to settle actual controversies…" The term "actual integral or constituent units thereof.
controversies" according to the Commissioner should refer to questions
which are political in nature and, therefore, the courts should not refuse to
xxxx
decide those political questions. But do I understand it right that this is
restrictive or only an example? I know there are cases which are not actual
yet the court can assume jurisdiction. An example is the petition for The Constitution is a definition of the powers of government. Who is to
declaratory relief. determine the nature, scope and extent of such powers? The Constitution
itself has provided for the instrumentality of the judiciary as the rational
way. And when the judiciary mediates to allocate constitutional boundaries,
May I ask the Commissioner’s opinion about that?
it does not assert any superiority over the other department; it does not in
reality nullify or invalidate an act of the legislature, but only asserts the
MR. CONCEPCION. The Supreme Court has no jurisdiction to grant solemn and sacred obligation assigned to it by the Constitution to
declaratory judgments. determine conflicting claims of authority under the Constitution and to
establish for the parties in an actual controversy the rights which that
MR. NOLLEDO. The Gentleman used the term "judicial power" but judicial instrument secures and guarantees to them. This is in truth all that is
power is not vested in the Supreme Court alone but also in other lower involved in what is termed "judicial supremacy" which properly is the
courts as may be created by law. power of judicial review under the Constitution. x x x29

MR. CONCEPCION. Yes. What are the remedies by which the grave abuse of discretion amounting
to lack or excess of jurisdiction on the part of any branch or instrumentality
of the Government may be determined under the Constitution?
MR. NOLLEDO. And so, is this only an example?

The present Rules of Court uses two special civil actions for determining
MR. CONCEPCION. No, I know this is not. The Gentleman seems to identify and correcting grave abuse of discretion amounting to lack or excess of
political questions with jurisdictional questions. But there is a difference. jurisdiction. These are the special civil actions for certiorari and prohibition,
and both are governed by Rule 65. A similar remedy of certiorari exists
MR. NOLLEDO. Because of the expression "judicial power"? under Rule 64, but the remedy is expressly applicable only to the
judgments and final orders or resolutions of the Commission on Elections
and the Commission on Audit.
The ordinary nature and function of the writ of certiorari in our present court and not to the court itself, while prohibition is a preventative remedy
system are aptly explained in Delos Santos v. Metropolitan Bank and Trust issuing to restrain future action, and is directed to the court itself.32 The
Company:30 Court expounded on the nature and function of the writ of prohibition in
Holy Spirit Homeowners Association, Inc. v. Defensor:33
In the common law, from which the remedy of certiorari evolved, the writ
of certiorari was issued out of Chancery, or the King’s Bench, commanding A petition for prohibition is also not the proper remedy to assail an IRR
agents or officers of the inferior courts to return the record of a cause issued in the exercise of a quasi-legislative function. Prohibition is an
pending before them, so as to give the party more sure and speedy justice, extraordinary writ directed against any tribunal, corporation, board, officer
for the writ would enable the superior court to determine from an or person, whether exercising judicial, quasi-judicial or ministerial
inspection of the record whether the inferior court’s judgment was functions, ordering said entity or person to desist from further proceedings
rendered without authority. The errors were of such a nature that, if when said proceedings are without or in excess of said entity’s or person’s
allowed to stand, they would result in a substantial injury to the petitioner jurisdiction, or are accompanied with grave abuse of discretion, and there
to whom no other remedy was available. If the inferior court acted without is no appeal or any other plain, speedy and adequate remedy in the
authority, the record was then revised and corrected in matters of law. The ordinary course of law. Prohibition lies against judicial or ministerial
writ of certiorari was limited to cases in which the inferior court was said to functions, but not against legislative or quasi-legislative functions.
be exceeding its jurisdiction or was not proceeding according to essential Generally, the purpose of a writ of prohibition is to keep a lower court
requirements of law and would lie only to review judicial or quasi-judicial within the limits of its jurisdiction in order to maintain the administration of
acts. justice in orderly channels. Prohibition is the proper remedy to afford relief
against usurpation of jurisdiction or power by an inferior court, or when, in
the exercise of jurisdiction in handling matters clearly within its cognizance
The concept of the remedy of certiorari in our judicial system remains
the inferior court transgresses the bounds prescribed to it by the law, or
much the same as it has been in the common law. In this jurisdiction,
where there is no adequate remedy available in the ordinary course of law
however, the exercise of the power to issue the writ of certiorari is largely
by which such relief can be obtained. Where the principal relief sought is to
regulated by laying down the instances or situations in the Rules of Court
invalidate an IRR, petitioners’ remedy is an ordinary action for its
in which a superior court may issue the writ of certiorari to an inferior court
nullification, an action which properly falls under the jurisdiction of the
or officer. Section 1, Rule 65 of the Rules of Court compellingly provides
Regional Trial Court. In any case, petitioners’ allegation that "respondents
the requirements for that purpose, viz:
are performing or threatening to perform functions without or in excess of
their jurisdiction" may appropriately be enjoined by the trial court through
xxxx a writ of injunction or a temporary restraining order.

The sole office of the writ of certiorari is the correction of errors of With respect to the Court, however, the remedies of certiorari and
jurisdiction, which includes the commission of grave abuse of discretion prohibition are necessarily broader in scope and reach, and the writ of
amounting to lack of jurisdiction. In this regard, mere abuse of discretion is certiorari or prohibition may be issued to correct errors of jurisdiction
not enough to warrant the issuance of the writ. The abuse of discretion committed not only by a tribunal, corporation, board or officer exercising
must be grave, which means either that the judicial or quasi-judicial power judicial, quasi-judicial or ministerial functions but also to set right, undo
was exercised in an arbitrary or despotic manner by reason of passion or and restrain any act of grave abuse of discretion amounting to lack or
personal hostility, or that the respondent judge, tribunal or board evaded a excess of jurisdiction by any branch or instrumentality of the Government,
positive duty, or virtually refused to perform the duty enjoined or to act in even if the latter does not exercise judicial, quasi-judicial or ministerial
contemplation of law, such as when such judge, tribunal or board functions. This application is expressly authorized by the text of the second
exercising judicial or quasi-judicial powers acted in a capricious or paragraph of Section 1, supra.
whimsical manner as to be equivalent to lack of jurisdiction.31
Thus, petitions for certiorari and prohibition are appropriate remedies to
Although similar to prohibition in that it will lie for want or excess of raise constitutional issues and to review and/or prohibit or nullify the acts
jurisdiction, certiorari is to be distinguished from prohibition by the fact of legislative and executive officials.34
that it is a corrective remedy used for the re-examination of some action of
an inferior tribunal, and is directed to the cause or proceeding in the lower
Necessarily, in discharging its duty under Section 1, supra, to set right and opinions, bereft as they are of authority to resolve hypothetical or moot
undo any act of grave abuse of discretion amounting to lack or excess of questions."
jurisdiction by any branch or instrumentality of the Government, the Court
is not at all precluded from making the inquiry provided the challenge was
An actual and justiciable controversy exists in these consolidated cases.
properly brought by interested or affected parties. The Court has been
The incompatibility of the perspectives of the parties on the
thereby entrusted expressly or by necessary implication with both the duty
constitutionality of the DAP and its relevant issuances satisfy the
and the obligation of determining, in appropriate cases, the validity of any
requirement for a conflict between legal rights. The issues being raised
assailed legislative or executive action. This entrustment is consistent with
herein meet the requisite ripeness considering that the challenged
the republican system of checks and balances.35
executive acts were already being implemented by the DBM, and there are
averments by the petitioners that such implementation was repugnant to
Following our recent dispositions concerning the congressional pork barrel, the letter and spirit of the Constitution. Moreover, the implementation of
the Court has become more alert to discharge its constitutional duty. We the DAP entailed the allocation and expenditure of huge sums of public
will not now refrain from exercising our expanded judicial power in order to funds. The fact that public funds have been allocated, disbursed or utilized
review and determine, with authority, the limitations on the Chief by reason or on account of such challenged executive acts gave rise,
Executive’s spending power. therefore, to an actual controversy that is ripe for adjudication by the
Court.
b) Requisites for the exercise of the
power of judicial review were It is true that Sec. Abad manifested during the January 28, 2014 oral
complied with arguments that the DAP as a program had been meanwhile discontinued
because it had fully served its purpose, saying: "In conclusion, Your
Honors, may I inform the Court that because the DAP has already fully
The requisites for the exercise of the power of judicial review are the
served its purpose, the Administration’s economic managers have
following, namely: (1) there must bean actual case or justiciable
recommended its termination to the President. x x x."39
controversy before the Court; (2) the question before the Court must be
ripe for adjudication; (3) the person challenging the act must be a proper
party; and (4) the issue of constitutionality must be raised at the earliest The Solicitor General then quickly confirmed the termination of the DAP as
opportunity and must be the very litis mota of the case.36 a program, and urged that its termination had already mooted the
challenges to the DAP’s constitutionality, viz:
The first requisite demands that there be an actual case calling for the
exercise of judicial power by the Court.37 An actual case or controversy, in DAP as a program, no longer exists, thereby mooting these present cases
the words of Belgica v. Executive Secretary Ochoa:38 brought to challenge its constitutionality. Any constitutional challenge
should no longer be at the level of the program, which is now extinct, but
at the level of its prior applications or the specific disbursements under the
x x x is one which involves a conflict of legal rights, an assertion of
now defunct policy. We challenge the petitioners to pick and choose which
opposite legal claims, susceptible of judicial resolution as distinguished
among the 116 DAP projects they wish to nullify, the full details we will
from a hypothetical or abstract difference or dispute. In other words,
have provided by February 5. We urge this Court to be cautious in limiting
"[t]here must be a contrariety of legal rights that can be interpreted and
the constitutional authority of the President and the Legislature to respond
enforced on the basis of existing law and jurisprudence." Related to the
to the dynamic needs of the country and the evolving demands of
requirement of an actual case or controversy is the requirement of
governance, lest we end up straight jacketing our elected representatives
"ripeness," meaning that the questions raised for constitutional scrutiny are
in ways not consistent with our constitutional structure and democratic
already ripe for adjudication. "A question is ripe for adjudication when the
principles.40
act being challenged has had a direct adverse effect on the individual
challenging it. It is a prerequisite that something had then been
accomplished or performed by either branch before a court may come into A moot and academic case is one that ceases to present a justiciable
the picture, and the petitioner must allege the existence of an immediate controversy by virtue of supervening events, so that a declaration thereon
or threatened injury to itself as a result of the challenged action." "Withal, would be of no practical use or value.41
courts will decline to pass upon constitutional issues through advisory
The Court cannot agree that the termination of the DAP as a program was It is true that as early as in 1937, in People v. Vera, the Court adopted the
a supervening event that effectively mooted these consolidated cases. direct injury test for determining whether a petitioner in a public action had
Verily, the Court had in the past exercised its power of judicial review locus standi. There, the Court held that the person who would assail the
despite the cases being rendered moot and academic by supervening validity of a statute must have "a personal and substantial interest in the
events, like: (1) when there was a grave violation of the Constitution; (2) case such that he has sustained, or will sustain direct injury as a result."
when the case involved a situation of exceptional character and was of Vera was followed in Custodio v. President of the Senate, Manila Race
paramount public interest; (3) when the constitutional issue raised Horse Trainers’ Association v. De la Fuente, Anti-Chinese League of the
required the formulation of controlling principles to guide the Bench, the Philippines v. Felix, and Pascual v. Secretary of Public Works.
Bar and the public; and (4) when the case was capable of repetition yet
evading review.42
Yet, the Court has also held that the requirement of locus standi, being a
mere procedural technicality, can be waived by the Court in the exercise of
Assuming that the petitioners’ several submissions against the DAP were its discretion. For instance, in 1949, in Araneta v. Dinglasan, the Court
ultimately sustained by the Court here, these cases would definitely come liberalized the approach when the cases had "transcendental importance."
under all the exceptions. Hence, the Court should not abstain from Some notable controversies whose petitioners did not pass the direct injury
exercising its power of judicial review. test were allowed to be treated in the same way as in Araneta v.
Dinglasan.
Did the petitioners have the legal standing to sue?
In the 1975 decision in Aquino v. Commission on Elections, this Court
decided to resolve the issues raised by the petition due to their "far
Legal standing, as a requisite for the exercise of judicial review, refers to "a
reaching implications," even if the petitioner had no personality to file the
right of appearance in a court of justice on a given question."43 The concept
suit. The liberal approach of Aquino v. Commission on Elections has been
of legal standing, or locus standi, was particularly discussed in De Castro v.
adopted in several notable cases, permitting ordinary citizens, legislators,
Judicial and Bar Council,44 where the Court said:
and civic organizations to bring their suits involving the constitutionality or
validity of laws, regulations, and rulings.
In public or constitutional litigations, the Court is often burdened with the
determination of the locus standi of the petitioners due to the ever-present
However, the assertion of a public right as a predicate for challenging a
need to regulate the invocation of the intervention of the Court to correct
supposedly illegal or unconstitutional executive or legislative action rests
any official action or policy in order to avoid obstructing the efficient
on the theory that the petitioner represents the public in general. Although
functioning of public officials and offices involved in public service. It is
such petitioner may not be as adversely affected by the action complained
required, therefore, that the petitioner must have a personal stake in the
against as are others, it is enough that he sufficiently demonstrates in his
outcome of the controversy, for, as indicated in Agan, Jr. v. Philippine
petition that he is entitled to protection or relief from the Court in the
International Air Terminals Co., Inc.:
vindication of a public right.

The question on legal standing is whether such parties have "alleged such a
Quite often, as here, the petitioner in a public action sues as a citizen or
personal stake in the outcome of the controversy as to assure that
taxpayer to gain locus standi. That is not surprising, for even if the issue
concrete adverseness which sharpens the presentation of issues upon
may appear to concern only the public in general, such capacities
which the court so largely depends for illumination of difficult constitutional
nonetheless equip the petitioner with adequate interest to sue. In David v.
questions." Accordingly, it has been held that the interest of a person
Macapagal-Arroyo, the Court aptly explains why:
assailing the constitutionality of a statute must be direct and personal. He
must be able to show, not only that the law or any government act is
invalid, but also that he sustained or is in imminent danger of sustaining Case law in most jurisdiction snow allows both "citizen" and "taxpayer"
some direct injury as a result of its enforcement, and not merely that he standing in public actions. The distinction was first laid down in Beauchamp
suffers thereby in some indefinite way. It must appear that the person v. Silk, where it was held that the plaintiff in a taxpayer’s suit is in a
complaining has been or is about to be denied some right or privilege to different category from the plaintiff in a citizen’s suit. In the former, the
which he is lawfully entitled or that he is about to be subjected to some plaintiff is affected by the expenditure of public funds, while in the latter,
burdens or penalties by reason of the statute or act complained of. he is but the mere instrument of the public concern. As held by the New
York Supreme Court in People ex rel Case v. Collins: "In matter of mere II.
public right, however…the people are the real parties…It is at least the Substantive Issues
right, if not the duty, of every citizen to interfere and see that a public
offence be properly pursued and punished, and that a public grievance be
1.
remedied." With respect to taxpayer’s suits, Terr v. Jordan held that "the
Overview of the Budget System
right of a citizen and a taxpayer to maintain an action in courts to restrain
the unlawful use of public funds to his injury cannot be denied."45
An understanding of the Budget System of the Philippines will aid the Court
in properly appreciating and justly resolving the substantive issues.
The Court has cogently observed in Agan, Jr. v. Philippine International Air
Terminals Co., Inc.46 that "[s]tanding is a peculiar concept in constitutional
law because in some cases, suits are not brought by parties who have been a) Origin of the Budget System
personally injured by the operation of a law or any other government act
but by concerned citizens, taxpayers or voters who actually sue in the The term "budget" originated from the Middle English word bouget that had
public interest." derived from the Latin word bulga (which means bag or purse).51

Except for PHILCONSA, a petitioner in G.R. No. 209164, the petitioners In the Philippine setting, Commonwealth Act (CA) No. 246 (Budget Act)
have invoked their capacities as taxpayers who, by averring that the defined "budget" as the financial program of the National Government for a
issuance and implementation of the DAP and its relevant issuances designated fiscal year, consisting of the statements of estimated receipts
involved the illegal disbursements of public funds, have an interest in and expenditures for the fiscal year for which it was intended to be
preventing the further dissipation of public funds. The petitioners in G.R. effective based on the results of operations during the preceding fiscal
No. 209287 (Araullo) and G.R. No. 209442 (Belgica) also assert their right years. The term was given a different meaning under Republic Act No. 992
as citizens to sue for the enforcement and observance of the constitutional (Revised Budget Act) by describing the budget as the delineation of the
limitations on the political branches of the Government.47 services and products, or benefits that would accrue to the public together
with the estimated unit cost of each type of service, product or
On its part, PHILCONSA simply reminds that the Court has long recognized benefit.52 For a forthright definition, budget should simply be identified as
its legal standing to bring cases upon constitutional issues.48 Luna, the the financial plan of the Government,53 or "the master plan of
petitioner in G.R. No. 209136, cites his additional capacity as a lawyer. The government."54
IBP, the petitioner in G.R. No. 209260, stands by "its avowed duty to work
for the rule of law and of paramount importance of the question in this The concept of budgeting has not been the product of recent economies. In
action, not to mention its civic duty as the official association of all lawyers reality, financing public goals and activities was an idea that existed from
in this country."49 the creation of the State.55 To protect the people, the territory and
sovereignty of the State, its government must perform vital functions that
Under their respective circumstances, each of the petitioners has required public expenditures. At the beginning, enormous public
established sufficient interest in the outcome of the controversy as to expenditures were spent for war activities, preservation of peace and
confer locus standi on each of them. order, security, administration of justice, religion, and supply of limited
goods and services.56 In order to finance those expenditures, the State
raised revenues through taxes and impositions.57 Thus, budgeting became
In addition, considering that the issues center on the extent of the power
necessary to allocate public revenues for specific government
of the Chief Executive to disburse and allocate public funds, whether
functions.58 The State’s budgeting mechanism eventually developed
appropriated by Congress or not, these cases pose issues that are of
through the years with the growing functions of its government and
transcendental importance to the entire Nation, the petitioners included. As
changes in its market economy.
such, the determination of such important issues call for the Court’s
exercise of its broad and wise discretion "to waive the requirement and so
remove the impediment to its addressing and resolving the serious The Philippine Budget System has been greatly influenced by western
constitutional questions raised."50 public financial institutions. This is because of the country’s past as a
colony successively of Spain and the United States for a long period of
time. Many aspects of the country’s public fiscal administration, including The Ministry of Budget was later renamed the Office of Budget and
its Budget System, have been naturally patterned after the practices and Management (OBM) under EO No. 711. The OBM became the DBM
experiences of the western public financial institutions. At any rate, the pursuant to EO No. 292 effective on November 24, 1989.
Philippine Budget System is presently guided by two principal objectives
that are vital to the development of a progressive democratic government,
c) The Philippine Budget Cycle66
namely: (1) to carry on all government activities under a comprehensive
fiscal plan developed, authorized and executed in accordance with the
Constitution, prevailing statutes and the principles of sound public Four phases comprise the Philippine budget process, specifically: (1)
management; and (2) to provide for the periodic review and disclosure of Budget Preparation; (2) Budget Legislation; (3) Budget Execution; and (4)
the budgetary status of the Government in such detail so that persons Accountability. Each phase is distinctly separate from the others but they
entrusted by law with the responsibility as well as the enlightened citizenry overlap in the implementation of the budget during the budget year.
can determine the adequacy of the budget actions taken, authorized or
proposed, as well as the true financial position of the Government.59 c.1.Budget Preparation67

b) Evolution of the Philippine Budget System The budget preparation phase is commenced through the issuance of a
Budget Call by the DBM. The Budget Call contains budget parameters
The budget process in the Philippines evolved from the early years of the earlier set by the Development Budget Coordination Committee (DBCC) as
American Regime up to the passage of the Jones Law in 1916. A Budget well as policy guidelines and procedures to aid government agencies in the
Office was created within the Department of Finance by the Jones Law to preparation and submission of their budget proposals. The Budget Call is of
discharge the budgeting function, and was given the responsibility to assist two kinds, namely: (1) a National Budget Call, which is addressed to all
in the preparation of an executive budget for submission to the Philippine agencies, including state universities and colleges; and (2) a Corporate
Legislature.60 Budget Call, which is addressed to all government-owned and -controlled
corporations (GOCCs) and government financial institutions (GFIs).
As early as under the 1935 Constitution, a budget policy and a budget
procedure were established, and subsequently strengthened through the Following the issuance of the Budget Call, the various departments and
enactment of laws and executive acts.61 EO No. 25, issued by President agencies submit their respective Agency Budget Proposals to the DBM. To
Manuel L. Quezon on April 25, 1936, created the Budget Commission to boost citizen participation, the current administration has tasked the
serve as the agency that carried out the President’s responsibility of various departments and agencies to partner with civil society
preparing the budget.62 CA No. 246, the first budget law, went into effect organizations and other citizen-stakeholders in the preparation of the
on January 1, 1938 and established the Philippine budget process. The law Agency Budget Proposals, which proposals are then presented before a
also provided a line-item budget as the framework of the Government’s technical panel of the DBM in scheduled budget hearings wherein the
budgeting system,63 with emphasis on the observance of a "balanced various departments and agencies are given the opportunity to defend
budget" to tie up proposed expenditures with existing revenues. their budget proposals. DBM bureaus thereafter review the Agency Budget
Proposals and come up with recommendations for the Executive Review
Board, comprised by the DBM Secretary and the DBM’s senior officials. The
CA No. 246 governed the budget process until the passage on June 4, 1954
discussions of the Executive Review Board cover the prioritization of
of Republic Act (RA) No. 992,whereby Congress introduced performance-
programs and their corresponding support vis-à-vis the priority agenda of
budgeting to give importance to functions, projects and activities in terms
the National Government, and their implementation.
of expected results.64 RA No. 992 also enhanced the role of the Budget
Commission as the fiscal arm of the Government.65
The DBM next consolidates the recommended agency budgets into the
National Expenditure Program (NEP)and a Budget of Expenditures and
The 1973 Constitution and various presidential decrees directed a series of
Sources of Financing (BESF). The NEP provides the details of spending for
budgetary reforms that culminated in the enactment of PD No. 1177 that
each department and agency by program, activity or project (PAP), and is
President Marcos issued on July30, 1977, and of PD No. 1405, issued on
submitted in the form of a proposed GAA. The Details of Selected Programs
June 11, 1978. The latter decree converted the Budget Commission into
and Projects is the more detailed disaggregation of key PAPs in the NEP,
the Ministry of Budget, and gave its head the rank of a Cabinet member.
especially those in line with the National Government’s development plan.
The Staffing Summary provides the staffing complement of each On the other hand, public revenues complement public expenditures and
department and agency, including the number of positions and amounts cover all income or receipts of the government treasury used to support
allocated. government expenditures.77

The NEP and BESF are thereafter presented by the DBM and the DBCC to Classical economist Adam Smith categorized public revenues based on two
the President and the Cabinet for further refinements or reprioritization. principal sources, stating: "The revenue which must defray…the necessary
Once the NEP and the BESF are approved by the President and the Cabinet, expenses of government may be drawn either, first from some fund which
the DBM prepares the budget documents for submission to Congress. The peculiarly belongs to the sovereign or commonwealth, and which is
budget documents consist of: (1) the President’s Budget Message, through independent of the revenue of the people, or, secondly, from the revenue
which the President explains the policy framework and budget priorities; of the people."78 Adam Smith’s classification relied on the two aspects of
(2) the BESF, mandated by Section 22, Article VII of the the nature of the State: first, the State as a juristic person with an artificial
Constitution,68 which contains the macroeconomic assumptions, public personality, and, second, the State as a sovereign or entity possessing
sector context, breakdown of the expenditures and funding sources for the supreme power. Under the first aspect, the State could hold property and
fiscal year and the two previous years; and (3) the NEP. engage in trade, thereby deriving what is called its quasi private income or
revenues, and which "peculiarly belonged to the sovereign." Under the
second aspect, the State could collect by imposing charges on the revenues
Public or government expenditures are generally classified into two
of its subjects in the form of taxes.79
categories, specifically: (1) capital expenditures or outlays; and (2) current
operating expenditures. Capital expenditures are the expenses whose
usefulness lasts for more than one year, and which add to the assets of the In the Philippines, public revenues are generally derived from the following
Government, including investments in the capital of government-owned or sources, to wit: (1) tax revenues(i.e., compulsory contributions to finance
controlled corporations and their subsidiaries.69 Current operating government activities); 80 (2) capital revenues(i.e., proceeds from sales of
expenditures are the purchases of goods and services in current fixed capital assets or scrap thereof and public domain, and gains on such
consumption the benefit of which does not extend beyond the fiscal sales like sale of public lands, buildings and other structures, equipment,
year.70 The two components of current expenditures are those for personal and other properties recorded as fixed assets); 81 (3) grants(i.e.,
services (PS), and those for maintenance and other operating voluntary contributions and aids given to the Government for its operation
expenses(MOOE). on specific purposes in the form of money and/or materials, and do not
require any monetary commitment on the part of the recipient);82 (4)
extraordinary income(i.e., repayment of loans and advances made by
Public expenditures are also broadly grouped according to their functions
government corporations and local governments and the receipts and
into: (1) economic development expenditures (i.e., expenditures on
shares in income of the Banko Sentral ng Pilipinas, and other
agriculture and natural resources, transportation and communications,
receipts);83 and (5) public borrowings(i.e., proceeds of repayable
commerce and industry, and other economic development efforts);71 (2)
obligations generally with interest from domestic and foreign creditors of
social services or social development expenditures (i.e., government outlay
the Government in general, including the National Government and its
on education, public health and medicare, labor and welfare and
political subdivisions).84
others);72 (3) general government or general public services expenditures
(i.e., expenditures for the general government, legislative services, the
administration of justice, and for pensions and gratuities);73 (4) national More specifically, public revenues are classified as follows:85
defense expenditures (i.e., sub-divided into national security expenditures
and expenditures for the maintenance of peace and order);74 and (5) public
debt.75 General Income Specific Income
Subsidy Income from National Income Taxes
Public expenditures may further be classified according to the nature of Government Property Taxes
funds, i.e., general fund, special fund or bond fund.76 Subsidy from Central Office  Taxes on Goods and Services
Subsidy from Regional  Taxes on International Trade and
Office/Staff Bureaus  Transactions
session. As with other laws, the GAB is approved on Third Reading before
Income from Government  Other Taxes 6.Fines and Penalties-Tax Revenue
the House of Representatives’ version is transmitted to the Senate.88
Services  Other Specific Income
Income from Government 
Business Operations After transmission, the Senate conducts its own committee hearings on the
GAB. To expedite proceedings, the Senate may conduct its committee
Sales Revenue hearings simultaneously with the House of Representatives’ deliberations.
Rent Income The Senate’s Finance Committee and its Sub-Committees may submit the
proposed amendments to the GAB to the plenary of the Senate only after
Insurance Income the House of Representatives has formally transmitted its version to the
Dividend Income Senate. The Senate version of the GAB is likewise approved on Third
Reading.89
Interest Income
Sale of Confiscated Goods and
The House of Representatives and the Senate then constitute a panel each
Properties
to sit in the Bicameral Conference Committee for the purpose of discussing
Foreign Exchange (FOREX) and harmonizing the conflicting provisions of their versions of the GAB. The
Gains "harmonized" version of the GAB is next presented to the President for
Miscellaneous Operating and approval.90 The President reviews the GAB, and prepares the Veto Message
Service Income where budget items are subjected to direct veto,91 or are identified for
conditional implementation.
Fines and Penalties-Government
Services and Business Operations
If, by the end of any fiscal year, the Congress shall have failed to pass the
Income from Grants and GAB for the ensuing fiscal year, the GAA for the preceding fiscal year shall
Donations be deemed re-enacted and shall remain in force and effect until the GAB is
passed by the Congress.92

c.3. Budget Execution93


c.2. Budget Legislation86
With the GAA now in full force and effect, the next step is the
implementation of the budget. The Budget Execution Phase is primarily the
The Budget Legislation Phase covers the period commencing from the time
function of the DBM, which is tasked to perform the following procedures,
Congress receives the President’s Budget, which is inclusive of the NEPand
namely: (1) to issue the programs and guidelines for the release of funds;
the BESF, up to the President’s approval of the GAA. This phase is also
(2) to prepare an Allotment and Cash Release Program; (3) to release
known as the Budget Authorization Phase, and involves the significant
allotments; and (4) to issue disbursement authorities.
participation of the Legislative through its deliberations.

The implementation of the GAA is directed by the guidelines issued by the


Initially, the President’s Budget is assigned to the House of
DBM. Prior to this, the various departments and agencies are required to
Representatives’ Appropriations Committee on First Reading. The
submit Budget Execution Documents(BED) to outline their plans and
Appropriations Committee and its various Sub-Committees schedule and
performance targets by laying down the physical and financial plan, the
conduct budget hearings to examine the PAPs of the departments and
monthly cash program, the estimate of monthly income, and the list of
agencies. Thereafter, the House of Representatives drafts the General
obligations that are not yet due and demandable.
Appropriations Bill (GAB).87

Thereafter, the DBM prepares an Allotment Release Program (ARP)and a


The GABis sponsored, presented and defended by the House of
Cash Release Program (CRP).The ARP sets a limit for allotments issued in
Representatives’ Appropriations Committee and Sub-Committees in plenary
general and to a specific agency. The CRP fixes the monthly, quarterly and review of agency performance; and (4) audit conducted by the Commission
annual disbursement levels. on Audit(COA).

Allotments, which authorize an agency to enter into obligations, are issued 2.


by the DBM. Allotments are lesser in scope than appropriations, in that the
latter embrace the general legislative authority to spend. Allotments may
Nature of the DAP as a fiscal plan
be released in two forms – through a comprehensive Agency Budget Matrix
(ABM),94 or, individually, by SARO.95
a. DAP was a program designed to
promote economic growth
Armed with either the ABM or the SARO, agencies become authorized to
incur obligations96 on behalf of the Government in order to implement their
PAPs. Obligations may be incurred in various ways, like hiring of personnel, Policy is always a part of every budget and fiscal decision of any
entering into contracts for the supply of goods and services, and using Administration.99 The national budget the Executive prepares and presents
utilities. to Congress represents the Administration’s "blueprint for public policy"
and reflects the Government’s goals and strategies.100 As such, the national
budget becomes a tangible representation of the programs of the
In order to settle the obligations incurred by the agencies, the DBM issues
Government in monetary terms, specifying therein the PAPs and services
a disbursement authority so that cash may be allocated in payment of the
for which specific amounts of public funds are proposed and
obligations. A cash or disbursement authority that is periodically issued is
allocated.101 Embodied in every national budget is government spending.102
referred to as a Notice of Cash Allocation (NCA),97 which issuance is based
upon an agency’s submission of its Monthly Cash Program and other
required documents. The NCA specifies the maximum amount of cash that When he assumed office in the middle of 2010, President Aquino made
can be withdrawn from a government servicing bank for the period efficiency and transparency in government spending a significant focus of
indicated. Apart from the NCA, the DBM may issue a Non-Cash Availment his Administration. Yet, although such focus resulted in an improved fiscal
Authority(NCAA) to authorize non-cash disbursements, or a Cash deficit of 0.5% in the gross domestic product (GDP) from January to July of
Disbursement Ceiling(CDC) for departments with overseas operations to 2011, it also unfortunately decelerated government project implementation
allow the use of income collected by their foreign posts for their operating and payment schedules.103 The World Bank observed that the Philippines’
requirements. economic growth could be reduced, and potential growth could be
weakened should the Government continue with its underspending and fail
to address the large deficiencies in infrastructure.104 The economic situation
Actual disbursement or spending of government funds terminates the
prevailing in the middle of 2011 thus paved the way for the development
Budget Execution Phase and is usually accomplished through the Modified
and implementation of the DAP as a stimulus package intended to fast-
Disbursement Scheme under which disbursements chargeable against the
track public spending and to push economic growth by investing on high-
National Treasury are coursed through the government servicing banks.
impact budgetary PAPs to be funded from the "savings" generated during
the year as well as from unprogrammed funds.105 In that respect, the DAP
c.4. Accountability98 was the product of "plain executive policy-making" to stimulate the
economy by way of accelerated spending.106The Administration would
thereby accelerate government spending by: (1) streamlining the
Accountability is a significant phase of the budget cycle because it ensures
implementation process through the clustering of infrastructure projects of
that the government funds have been effectively and efficiently utilized to
the Department of Public Works and Highways (DPWH) and the
achieve the State’s socio-economic goals. It also allows the DBM to assess
Department of Education (DepEd),and (2) front loading PPP-related
the performance of agencies during the fiscal year for the purpose of
projects107 due for implementation in the following year.108
implementing reforms and establishing new policies.

Did the stimulus package work?


An agency’s accountability may be examined and evaluated through (1)
performance targets and outcomes; (2) budget accountability reports; (3)
The March 2012 report of the World Bank,109 released after the initial A. Fund Sources for the Acceleration Program
implementation of the DAP, revealed that the DAP was partially successful.
The disbursements under the DAP contributed 1.3 percentage points to
GDP growth by the fourth quarter of 2011.110 The continued Amount
implementation of the DAP strengthened growth by 11.8% year on year Action
Fund Sources (In million Description
while infrastructure spending rebounded from a 29% contraction to a 34% Requested
Php)
growth as of September 2013.111

FY 2011 30,000 Unreleased Personnel Declare as


The DAP thus proved to be a demonstration that expenditure was a policy
Unreleased Services (PS) savings and
instrument that the Government could use to direct the economies towards
Personal appropriations which approve/
growth and development.112 The Government, by spending on public
Services (PS) will lapse at the end of authorize its
infrastructure, would signify its commitment of ensuring profitability for
Appropriations FY 2011 but may be use
prospective investors.113 The PAPs funded under the DAP were chosen for
pooled as savings and for the 2011
this reason based on their: (1) multiplier impact on the economy and
realigned for priority Disbursement
infrastructure development; (2) beneficial effect on the poor; and (3)
programs that require Acceleration
translation into disbursements.114
immediate funding Program

b. History of the implementation of


the DAP, and sources of funds FY 2011 482 Unreleased  
under the DAP Unreleased appropriations (slow
Appropriations moving projects and
programs for
How the Administration’s economic managers conceptualized and discontinuance)
developed the DAP, and finally presented it to the President remains
unknown because the relevant documents appear to be scarce.
FY 2010 12,336 Supported by the GFI Approve and
Unprogrammed Dividends authorize its
The earliest available document relating to the genesis of the DAP was the Fund use
memorandum of October 12,2011 from Sec. Abad seeking the approval of for the 2011
the President to implement the proposed DAP. The memorandum, which Disbursement
contained a list of the funding sources for ₱72.11 billion and of the Acceleration
proposed priority projects to be funded,115 reads: Program

MEMORANDUM FOR THE PRESIDENT


FY 2010 21,544 Unreleased With prior
Carryover appropriations (slow approval from
xxxx Appropriation moving projects and the President in
programs for November
SUBJECT: FY 2011 PROPOSED DISBURSEMENT ACCELERATION PROGRAM discontinuance) and 2010
(PROJECTS AND SOURCES OF FUNDS) savings from Zero-based to declare as
Budgeting savings and
Initiative with
DATE: OCTOBER 12, 2011 authority to
use
Mr. President, this is to formally confirm your approval of the Disbursement for priority
Acceleration Program totaling ₱72.11 billion. We are already working with projects
all the agencies concerned for the immediate execution of the projects
therein.
FY 2011 Budget 7,748 FY 2011 Agency For information 6. HGC: Equity infusion for credit insurance 400
items for Budget items that can and mortgage guaranty operations of HGC
realignment be realigned within the
agency to fund new fast 7. PHIC: Obligations incurred (premium 1,496
disbursing projects subsidy for indigent families) in January-June
DPWH-3.981 Billion 2010, booked for payment in Jul[y] – Dec
DA – 2.497 Billion 2010. The delay in payment is due to the
DOT – 1.000 Billion delay in the certification of the LGU
DepEd – 270 Million counterpart. Without it, the NG is obliged to
pay the full amount.
TOTAL 72.110    
8. Philpost: Purchase of foreclosed property. 644
Payment of Mandatory Obligations, (GSIS,
PhilHealth, ECC), Franking Privilege
B. Projects in the Disbursement Acceleration Program
9. BSP: First equity infusion out of Php 40B 10,000
(Descriptions of projects attached as Annex A) capitalization under the BSP Law

10. PCMC: Capital and Equipment Renovation 280


GOCCs and GFIs
11. LCOP: 105
Agency/Project Allotment a. Pediatric Pulmonary Program
(SARO and NCA Release) (in Million Php) b. Bio-regenerative Technology Program 35
(Stem-Cell Research – subject to legal
1. LRTA: Rehabilitation of LRT 1 and 2 1,868 review and presentation) 70

2. NHA: 11,050 12. TIDCORP: NG Equity infusion 570

a. Resettlement of North Triangle residents to 450 TOTAL 26,945


Camarin A7
b. Housing for BFP/BJMP 500
c. On-site development for families living 10,000
along dangerous NGAs/LGUs
d. Relocation sites for informal settlers 100
along Iloilo River and its tributaries Agency/Project Allotment
(SARO) Cash
3. PHIL. HEART CENTER: Upgrading of 357 (In Million Requirement
ageing physical plant and medical equipment Php) (NCA)

4. CREDIT INFO CORP: Establishment of 75 13. DOF-BIR: NPSTAR


centralized credit information system centralization of data    
processing and others (To be    
5. PIDS: purchase of land to relocate the PIDS 100 synchronized with GFMIS    
office and building construction activities) 758 758
14. COA: IT infrastructure a. Establishment of National
program and hiring of     Meterological and Climate    
additional litigational experts 144 144 Center 275 275
b. Enhancement of Doppler
15. DND-PAF: On Base Housing Radar Network for National    
Facilities and Communication     Weather Watch, Accurate    
Equipment 30 30 Forecasting and Flood Early    
Warning 190 190
16. DA: 2,959 2,223
a. Irrigation, FMRs and 23. DOF-BOC: To settle the
Integrated Community Based Multi-     principal obligations with    
Species     PDIC consistent with the    
Hatchery and Aquasilvi 1,629 1,629 agreement with the CISS and    
Farming SGS 2,800 2,800
b. Mindanao Rural 919 183
Development Project 24. OEO-FDCP: Establishment of
the National Film Archive and    
c. NIA Agno River Integrated local cinematheques, and other    
Irrigation Project 411 411 local activities 20 20

17. DAR: 1,293 1,293 25. DPWH: Various infrastructure


a. Agrarian Reform projects 5,500 5,500
Communities Project 2 1,293 132
b. Landowners Compensation 5,432 26. DepEd/ERDT/DOST: Thin
Client Cloud Computing    
18. DBM: Conduct of National Project 270 270
Survey of    
Farmers/Fisherfolks/Ips 625 625 27. DOH: Hiring of nurses and
midwives 294 294
19. DOJ: Operating requirements
of 50 investigation agents and     28. TESDA: Training Program in
15 state attorneys 11 11 partnership with BPO industry    
and other sectors 1,100 1,100
20. DOT: Preservation of the Cine
Corregidor Complex 25 25 29. DILG: Performance Challenge
Fund (People Empowered    
21. OPAPP: Activities for Peace Community Driven    
Process (PAMANA- Project     Development with DSWD and    
details: budget breakdown,     NAPC) 250 50
implementation plan, and    
conditions on fund release     30. ARMM: Comprehensive Peace
attached as Annex B) 1,819 1,819 and Development Intervention 8,592 8,592

22. DOST 425 425 31. DOTC-MRT: Purchase of


fiscal year 2011. Pertinent portions of the memorandum of December 12,
additional MRT cars 4,500 - 2011 read:

32. LGU Support Fund 6,500 6,500


MEMORANDUM FOR THE PRESIDENT
33. Various Other Local Projects 6,500 6,500
xxxx
34. Development Assistance to the
Province of Quezon 750 750 SUBJECT: Omnibus Authority to Consolidate Savings/Unutilized Balances
and its Realignment
TOTAL 45,165 44,000
DATE: December 12, 2011

C. Summary
This is to respectfully request for the grant of Omnibus Authority to
consolidate savings/unutilized balances in FY 2011 corresponding to
completed or discontinued projects which may be pooled to fund additional
  Fund Sources
projects or expenditures.
Identified for Allotments Cash
Approval for Release Requirements for
(In Million Release in FY In addition, Mr. President, this measure will allow us to undertake projects
Php) 2011 even if their implementation carries over to 2012 without necessarily
impacting on our budget deficit cap next year.
Total 72,110 72,110 70,895
BACKGROUND
GOCCs 26,895 26,895
1.0 The DBM, during the course of performance reviews
NGAs/LGUs 45,165 44,000 conducted on the agencies’ operations, particularly on the
implementation of their projects/activities, including
expenses incurred in undertaking the same, have
For His Excellency’s Consideration identified savings out of the 2011 General Appropriations
Act. Said savings correspond to completed or
(Sgd.) FLORENCIO B. ABAD discontinued projects under certain
departments/agencies which may be pooled, for the
following:
[/] APPROVED

1.1 to provide for new activities which have not


[ ] DISAPPROVED been anticipated during preparation of the
budget;
(Sgd.) H.E. BENIGNO S. AQUINO, III
1.2 to augment additional requirements of on-
OCT 12, 2011 going priority projects; and

The memorandum of October 12, 2011 was followed by another 1.3 to provide for deficiencies under the Special
memorandum for the President dated December 12, 2011116 requesting Purpose Funds, e.g., PDAF, Calamity Fund,
omnibus authority to consolidate the savings and unutilized balances for Contingent Fund
1.4 to cover for the modifications of the original 6.0 Among others, the following are such proposed
allotment class allocation as a result of on-going additional projects that have been chosen given their
priority projects and implementation of new multiplier impact on economy and infrastructure
activities development, their beneficial effect on the poor, and their
translation into disbursements. Please note that we have
classified the list of proposed projects as follows:
2.0 x x x x

7.0 x x x
2.1 x x x

FOR THE PRESIDENT’S APPROVAL


2.2 x x x

8.0 Foregoing considered, may we respectfully request


ON THE UTILIZATION OF POOLED SAVINGS
for the President’s approval for the following:

3.0 It may be recalled that the President approved our


8.1 Grant of omnibus authority to consolidate FY
request for omnibus authority to pool savings/unutilized
2011 savings/unutilized balances and its
balances in FY 2010 last November 25, 2010.
realignment; and

4.0 It is understood that in the utilization of the pooled


8.2 The proposed additional projects identified
savings, the DBM shall secure the corresponding
for funding.
approval/confirmation of the President. Furthermore, it is
assured that the proposed realignments shall be within
the authorized Expenditure level. For His Excellency’s consideration and approval.

5.0 Relative thereto, we have identified some (Sgd.)


expenditure items that may be sourced from the said
pooled appropriations in FY 2010 that will expire on
[/] APPROVED
December 31, 2011 and appropriations in FY 2011 that
may be declared as savings to fund additional
expenditures. [ ] DISAPPROVED

5.1 The 2010 Continuing Appropriations (pooled (Sgd.) H.E. BENIGNO S. AQUINO, III
savings) is proposed to be spent for the projects
that we have identified to be immediate actual DEC 21, 2011
disbursements considering that this same fund
source will expire on December 31, 2011.
Substantially identical requests for authority to pool savings and to fund
proposed projects were contained in various other memoranda from Sec.
5.2 With respect to the proposed expenditure Abad dated June 25, 2012,117 September 4, 2012,118 December 19,
items to be funded from the FY 2011 Unreleased 2012,119 May 20, 2013,120 and September 25, 2013.121 The President
Appropriations, most of these are the same apparently approved all the requests, withholding approval only of the
projects for which the DBM is directed by the proposed projects contained in the June 25, 2012 memorandum, as borne
Office of the President, thru the Executive out by his marginal note therein to the effect that the proposed projects
Secretary, to source funds. should still be "subject to further discussions."122
In order to implement the June25, 2012 memorandum, Sec. Abad issued outside the agencies’ spheres of control). Also, they are asked to formulate
NBC No. 541 (Adoption of Operational Efficiency Measure – Withdrawal of strategies and improvement plans for the rest of 2012.
Agencies’ Unobligated Allotments as of June 30, 2012),123 reproduced
herein as follows:
Notwithstanding these initiatives, some departments/agencies have
continued to post low obligation levels as of end of first semester, thus
NATIONAL BUDGET CIRCULAR No. 541 resulting to substantial unobligated allotments.

July 18, 2012 In line with this, the President, per directive dated June 27, 2012
authorized the withdrawal of unobligated allotments of agencies with low
levels of obligations as of June 30, 2012, both for continuing and current
TO: All Heads of Departments/Agencies/State Universities and Colleges and
allotments. This measure will allow the maximum utilization of available
other Offices of the National Government, Budget and Planning Officers;
allotments to fund and undertake other priority expenditures of the
Heads of Accounting Units and All Others Concerned
national government.

SUBJECT : Adoption of Operational Efficiency Measure – Withdrawal of


2.0 Purpose
Agencies’ Unobligated Allotments as of June 30, 2012

2.1 To provide the conditions and parameters on the


1.0 Rationale
withdrawal of unobligated allotments of agencies as of
June 30, 2012 to fund priority and/or fast-moving
The DBM, as mandated by Executive Order (EO) No. 292 (Administrative programs/projects of the national government;
Code of 1987), periodically reviews and evaluates the
departments/agencies’ efficiency and effectiveness in utilizing budgeted
2.2 To prescribe the reports and documents to be used as
funds for the delivery of services and production of goods, consistent with
bases on the withdrawal of said unobligated allotments;
the government priorities.
and

In the event that a measure is necessary to further improve the


2.3 To provide guidelines in the utilization or reallocation
operational efficiency of the government, the President is authorized to
of the withdrawn allotments.
suspend or stop further use of funds allotted for any agency or expenditure
authorized in the General Appropriations Act. Withdrawal and pooling of
unutilized allotment releases can be effected by DBM based on authority of 3.0 Coverage
the President, as mandated under Sections 38 and 39, Chapter 5, Book VI
of EO 292.
3.1 These guidelines shall cover the withdrawal of
unobligated allotments as of June 30, 2012 of all national
For the first five months of 2012, the National Government has not met its government agencies (NGAs) charged against FY 2011
spending targets. In order to accelerate spending and sustain the fiscal Continuing Appropriation (R.A. No.10147) and FY 2012
targets during the year, expenditure measures have to be implemented to Current Appropriation (R.A. No. 10155), pertaining to:
optimize the utilization of available resources.
3.1.1 Capital Outlays (CO);
Departments/agencies have registered low spending levels, in terms of
obligations and disbursements per initial review of their 2012 performance.
3.1.2 Maintenance and Other Operating
To enhance agencies’ performance, the DBM conducts continuous
Expenses (MOOE) related to the implementation
consultation meetings and/or send call-up letters, requesting them to
of programs and projects, as well as capitalized
identify slow-moving programs/projects and the factors/issues affecting
MOOE; and
their performance (both pertaining to internal systems and those which are
3.1.3 Personal Services corresponding to • Savings from mandatory expenditures
unutilized pension benefits declared as savings which can be realigned only in the last
by the agencies concerned based on their quarter after taking into consideration
updated/validated list of pensioners. the agency’s full year requirements,
i.e., Petroleum, Oil and Lubricants,
Water, Illumination, Power Services,
3.2 The withdrawal of unobligated allotments may cover
Telephone, other Communication
the identified programs, projects and activities of the
Services and Rent.
departments/agencies reflected in the DBM list shown as
Annex A or specific programs and projects as may be
identified by the agencies. 4.2.3 Foreign-Assisted Projects (loan proceeds
and peso counterpart);
4.0 Exemption
4.2.4 Special Purpose Funds such as: E-
Government Fund, International Commitments
These guidelines shall not apply to the following:
Fund, PAMANA, Priority Development Assistance
Fund, Calamity Fund, Budgetary Support to
4.1 NGAs GOCCs and Allocation to LGUs, among others;

4.1.1 Constitutional Offices/Fiscal Autonomy 4.2.5 Quick Response Funds; and


Group, granted fiscal autonomy under the
Philippine Constitution; and
4.2.6 Automatic Appropriations i.e., Retirement
Life Insurance Premium and Special Accounts in
4.1.2 State Universities and Colleges, adopting the General Fund.
the Normative Funding allocation scheme i.e.,
distribution of a predetermined budget ceiling.
5.0 Guidelines

4.2 Fund Sources


5.1 National government agencies shall continue to
undertake procurement activities notwithstanding the
4.2.1 Personal Services other than pension implementation of the policy of withdrawal of unobligated
benefits; allotments until the end of the third quarter, FY 2012.
Even without the allotments, the agency shall proceed in
4.2.2 MOOE items earmarked for specific undertaking the procurement processes (i.e.,
purposes or subject to realignment conditions procurement planning up to the conduct of bidding but
per General Provisions of the GAA: short of awarding of contract) pursuant to GPPB Circular
Nos. 02-2008 and 01-2009 and DBM Circular Letter No.
2010-9.
• Confidential and Intelligence Fund;

5.2 For the purpose of determining the amount of


• Savings from Traveling, unobligated allotments that shall be withdrawn, all
Communication, Transportation and departments/agencies/operating units (OUs) shall submit
Delivery, Repair and Maintenance, to DBM not later than July 30, 2012, the following budget
Supplies and Materials and Utility which accountability reports as of June 30, 2012;
shall be used for the grant of Collective
Negotiation Agreement incentive
benefit;
• Statement of Allotments, Obligations and report shall highlight the agencies which failed to submit
Balances (SAOB); the June 30 reports required under this Circular.

• Financial Report of Operations (FRO); and 5.7 The withdrawn allotments may be:

• Physical Report of Operations. 5.7.1 Reissued for the original programs and
projects of the agencies/OUs concerned, from
which the allotments were withdrawn;
5.3 In the absence of the June 30, 2012 reports cited
under item 5.2 of this Circular, the agency’s latest report
available shall be used by DBM as basis for withdrawal of 5.7.2 Realigned to cover additional funding for
allotment. The DBM shall compute/approximate the other existing programs and projects of the
agency’s obligation level as of June 30 to derive its agency/OU; or
unobligated allotments as of same period. Example: If
the March 31 SAOB or FRO reflects actual obligations of P
5.7.3 Used to augment existing programs and
800M then the June 30 obligation level shall approximate
projects of any agency and to fund priority
to ₱1,600 M (i.e., ₱800 M x 2 quarters).
programs and projects not considered in the
2012 budget but expected to be started or
5.4 All released allotments in FY 2011 charged against implemented during the current year.
R.A. No. 10147 which remained unobligated as of June
30, 2012 shall be immediately considered for withdrawal.
5.8 For items 5.7.1 and 5.7.2 above, agencies/OUs
This policy is based on the following considerations:
concerned may submit to DBM a Special Budget Request
(SBR), supported with the following:
5.4.1 The departments/agencies’ approved
priority programs and projects are assumed to
5.8.1 Physical and Financial Plan (PFP);
be implementation-ready and doable during the
given fiscal year; and
5.8.2 Monthly Cash Program (MCP); and
5.4.2 The practice of having substantial
carryover appropriations may imply that the 5.8.3 Proof that the project/activity has started
agency has a slower-than-programmed the procurement processes i.e., Proof of Posting
implementation capacity or agency tends to and/or Advertisement of the Invitation to Bid.
implement projects within a two-year
timeframe. 5.9 The deadline for submission of request/s pertaining to
these categories shall be until the end of the third quarter
5.5. Consistent with the President’s directive, the DBM i.e., September 30, 2012. After said cut-off date, the
shall, based on evaluation of the reports cited above and withdrawn allotments shall be pooled and form part of
results of consultations with the departments/agencies, the overall savings of the national government.
withdraw the unobligated allotments as of June 30, 2012
through issuance of negative Special Allotment Release 5.10 Utilization of the consolidated withdrawn allotments
Orders (SAROs). for other priority programs and projects as cited under
item 5.7.3 of this Circular, shall be subject to approval of
5.6 DBM shall prepare and submit to the President, a the President. Based on the approval of the President,
report on the magnitude of withdrawn allotments. The DBM shall issue the SARO to cover the approved priority
expenditures subject to submission by the agency/OU the various departments and agencies derived from pooling unobligated
concerned of the SBR and supported with PFP and MCP. allotments and withdrawing unreleased appropriations; (2) releasing
unprogrammed funds; and (3) applying the "savings" and unprogrammed
funds to augment existing PAPs or to support other priority PAPs.
5.11 It is understood that all releases to be made out of
the withdrawn allotments (both 2011 and 2012
unobligated allotments) shall be within the approved c. DAP was not an appropriation
Expenditure Program level of the national government for measure; hence, no appropriation
the current year. The SAROs to be issued shall properly law was required to adopt or to
disclose the appropriation source of the release to implement it
determine the extent of allotment validity, as follows:
Petitioners Syjuco, Luna, Villegas and PHILCONSA state that Congress did
• For charges under R.A. 10147 – allotments not enact a law to establish the DAP, or to authorize the disbursement and
shall be valid up to December 31, 2012; and release of public funds to implement the DAP. Villegas, PHILCONSA, IBP,
Araullo, and COURAGE observe that the appropriations funded under the
DAP were not included in the 2011, 2012 and 2013 GAAs. To petitioners
• For charges under R.A. 10155 – allotments
IBP, Araullo, and COURAGE, the DAP, being actually an appropriation that
shall be valid up to December 31, 2013.
set aside public funds for public use, should require an enabling law for its
validity. VACC maintains that the DAP, because it involved huge allocations
5.12 Timely compliance with the submission of existing that were separate and distinct from the GAAs, circumvented and
BARs and other reportorial requirements is reiterated for duplicated the GAAs without congressional authorization and control.
monitoring purposes.
The petitioners contend in unison that based on how it was developed and
6.0 Effectivity implemented the DAP violated the mandate of Section 29(1), Article VI of
the 1987 Constitution that "[n]o money shall be paid out of the Treasury
This circular shall take effect immediately. except in pursuance of an appropriation made by law."

(Sgd.) FLORENCIO B. ABAD The OSG posits, however, that no law was necessary for the adoption and
Secretary implementation of the DAP because of its being neither a fund nor an
appropriation, but a program or an administrative system of prioritizing
spending; and that the adoption of the DAP was by virtue of the authority
As can be seen, NBC No. 541 specified that the unobligated allotments of of the President as the Chief Executive to ensure that laws were faithfully
all agencies and departments as of June 30, 2012 that were charged executed.
against the continuing appropriations for fiscal year 2011 and the 2012
GAA (R.A. No. 10155) were subject to withdrawal through the issuance of
negative SAROs, but such allotments could be either: (1) reissued for the We agree with the OSG’s position.
original PAPs of the concerned agencies from which they were withdrawn;
or (2) realigned to cover additional funding for other existing PAPs of the The DAP was a government policy or strategy designed to stimulate the
concerned agencies; or (3) used to augment existing PAPs of any agency economy through accelerated spending. In the context of the DAP’s
and to fund priority PAPs not considered in the 2012 budget but expected adoption and implementation being a function pertaining to the Executive
to be started or implemented in 2012. Financing the other priority PAPs as the main actor during the Budget Execution Stage under its
was made subject to the approval of the President. Note here that NBC No. constitutional mandate to faithfully execute the laws, including the GAAs,
541 used terminologies like "realignment" and "augmentation" in the Congress did not need to legislate to adopt or to implement the DAP.
application of the withdrawn unobligated allotments. Congress could appropriate but would have nothing more to do during the
Budget Execution Stage. Indeed, appropriation was the act by which
Taken together, all the issuances showed how the DAP was to be Congress "designates a particular fund, or sets apart a specified portion of
implemented and funded, that is — (1) by declaring "savings" coming from the public revenue or of the money in the public treasury, to be applied to
some general object of governmental expenditure, or to some individual execution stage. Executive discretion is necessary at that stage to achieve
purchase or expense."124 As pointed out in Gonzales v. Raquiza:125 ‘"In a a sound fiscal administration and assure effective budget implementation.
strict sense, appropriation has been defined ‘as nothing more than the The heads of offices, particularly the President, require flexibility in their
legislative authorization prescribed by the Constitution that money may be operations under performance budgeting to enable them to make whatever
paid out of the Treasury,’ while appropriation made by law refers to ‘the adjustments are needed to meet established work goals under changing
act of the legislature setting apart or assigning to a particular use a certain conditions.128 In particular, the power to transfer funds can give the
sum to be used in the payment of debt or dues from the State to its President the flexibility to meet unforeseen events that may otherwise
creditors.’"126 impede the efficient implementation of the PAPs set by Congress in the
GAA.
On the other hand, the President, in keeping with his duty to faithfully
execute the laws, had sufficient discretion during the execution of the Congress has traditionally allowed much flexibility to the President in
budget to adapt the budget to changes in the country’s economic allocating funds pursuant to the GAAs,129particularly when the funds are
situation.127 He could adopt a plan like the DAP for the purpose. He could grouped to form lump sum accounts.130 It is assumed that the agencies of
pool the savings and identify the PAPs to be funded under the DAP. The the Government enjoy more flexibility when the GAAs provide broader
pooling of savings pursuant to the DAP, and the identification of the PAPs appropriation items.131 This flexibility comes in the form of policies that the
to be funded under the DAP did not involve appropriation in the strict sense Executive may adopt during the budget execution phase. The DAP – as a
because the money had been already set apart from the public treasury by strategy to improve the country’s economic position – was one policy that
Congress through the GAAs. In such actions, the Executive did not usurp the President decided to carry out in order to fulfill his mandate under the
the power vested in Congress under Section 29(1), Article VI of the GAAs.
Constitution.
Denying to the Executive flexibility in the expenditure process would be
3. counterproductive. In Presidential Spending Power,132 Prof. Louis Fisher, an
Unreleased appropriations and withdrawn American constitutional scholar whose specialties have included budget
unobligated allotments under the DAP policy, has justified extending discretionary authority to the Executive
were not savings, and the use of such thusly:
appropriations contravened Section 25(5),
Article VI of the 1987 Constitution.
[T]he impulse to deny discretionary authority altogether should be resisted.
There are many number of reasons why obligations and outlays by
Notwithstanding our appreciation of the DAP as a plan or strategy validly administrators may have to differ from appropriations by legislators.
adopted by the Executive to ramp up spending to accelerate economic Appropriations are made many months, and sometimes years, in advance
growth, the challenges posed by the petitioners constrain us to dissect the of expenditures. Congress acts with imperfect knowledge in trying to
mechanics of the actual execution of the DAP. The management and legislate in fields that are highly technical and constantly undergoing
utilization of the public wealth inevitably demands a most careful scrutiny change. New circumstances will develop to make obsolete and mistaken
of whether the Executive’s implementation of the DAP was consistent with the decisions reached by Congress at the appropriation stage. It is not
the Constitution, the relevant GAAs and other existing laws. practicable for Congress to adjust to each new development by passing
separate supplemental appropriation bills. Were Congress to control
expenditures by confining administrators to narrow statutory details, it
a. Although executive discretion
would perhaps protect its power of the purse but it would not protect the
and flexibility are necessary in
purse itself. The realities and complexities of public policy require executive
the execution of the budget, any
discretion for the sound management of public funds.
transfer of appropriated funds
should conform to Section 25(5),
Article VI of the Constitution xxxx

We begin this dissection by reiterating that Congress cannot anticipate all x x x The expenditure process, by its very nature, requires substantial
issues and needs that may come into play once the budget reaches its discretion for administrators. They need to exercise judgment and take
responsibility for their actions, but those actions ought to be directed During the Commonwealth period, the power of the President to transfer
toward executing congressional, not administrative policy. Let there be funds continued to be governed by the GAAs despite the enactment of the
discretion, but channel it and use it to satisfy the programs and priorities Constitution in 1935. It is notable that the 1935 Constitution did not
established by Congress. include a provision on the power to transfer funds. At any rate, a shift in
the extent of the President’s power to transfer funds was again experienced
during this era, with the President being given more flexibility in
In contrast, by allowing to the heads of offices some power to transfer
implementing the budget. The GAAs provided that the power to transfer all
funds within their respective offices, the Constitution itself ensures the
or portions of the appropriations in the Executive Department could be
fiscal autonomy of their offices, and at the same time maintains the
made in the "interest of the public, as the President may determine."136
separation of powers among the three main branches of the Government.
The Court has recognized this, and emphasized so in Bengzon v.
Drilon,133 viz: In its time, the 1971 Constitutional Convention wanted to curtail the
President’s seemingly unbounded discretion in transferring funds.137 Its
Committee on the Budget and Appropriation proposed to prohibit the
The Judiciary, the Constitutional Commissions, and the Ombudsman must
transfer of funds among the separate branches of the Government and the
have the independence and flexibility needed in the discharge of their
independent constitutional bodies, but to allow instead their respective
constitutional duties. The imposition of restrictions and constraints on the
heads to augment items of appropriations from savings in their respective
manner the independent constitutional offices allocate and utilize the funds
budgets under certain limitations.138 The clear intention of the Convention
appropriated for their operations is anathema to fiscal autonomy and
was to further restrict, not to liberalize, the power to transfer
violative not only of the express mandate of the Constitution but especially
appropriations.139 Thus, the Committee on the Budget and Appropriation
as regards the Supreme Court, of the independence and separation of
initially considered setting stringent limitations on the power to augment,
powers upon which the entire fabric of our constitutional system is based.
and suggested that the augmentation of an item of appropriation could be
made "by not more than ten percent if the original item of appropriation to
In the case of the President, the power to transfer funds from one item to be augmented does not exceed one million pesos, or by not more than five
another within the Executive has not been the mere offshoot of established percent if the original item of appropriation to be augmented exceeds one
usage, but has emanated from law itself. It has existed since the time of million pesos."140 But two members of the Committee objected to the
the American Governors-General.134 Act No. 1902 (An Act authorizing the ₱1,000,000.00 threshold, saying that the amount was arbitrary and might
Governor-General to direct any unexpended balances of appropriations be not be reasonable in the future. The Committee agreed to eliminate the
returned to the general fund of the Insular Treasury and to transfer from ₱1,000,000.00 threshold, and settled on the ten percent limitation.141
the general fund moneys which have been returned thereto), passed on
May 18, 1909 by the First Philippine Legislature,135 was the first enabling
In the end, the ten percent limitation was discarded during the plenary of
law that granted statutory authority to the President to transfer funds. The
the Convention, which adopted the following final version under Section 16,
authority was without any limitation, for the Act explicitly empowered the
Article VIII of the 1973 Constitution, to wit:
Governor-General to transfer any unexpended balance of appropriations for
any bureau or office to another, and to spend such balance as if it had
originally been appropriated for that bureau or office. (5) No law shall be passed authorizing any transfer of appropriations;
however, the President, the Prime Minister, the Speaker, the Chief Justice
of the Supreme Court, and the heads of Constitutional Commissions may
From 1916 until 1920, the appropriations laws set a cap on the amounts of
by law be authorized to augment any item in the general appropriations
funds that could be transferred, thereby limiting the power to transfer
law for their respective offices from savings in other items of their
funds. Only 10% of the amounts appropriated for contingent or
respective appropriations.
miscellaneous expenses could be transferred to a bureau or office, and the
transferred funds were to be used to cover deficiencies in the
appropriations also for miscellaneous expenses of said bureau or office. The 1973 Constitution explicitly and categorically prohibited the transfer of
funds from one item to another, unless Congress enacted a law authorizing
the President, the Prime Minister, the Speaker, the Chief Justice of the
In 1921, the ceiling on the amounts of funds to be transferred from items
Supreme Court, and the heads of the Constitutional omissions to transfer
under miscellaneous expenses to any other item of a certain bureau or
funds for the purpose of augmenting any item from savings in another item
office was removed.
in the GAA of their respective offices. The leeway was limited to
augmentation only, and was further constricted by the condition that the xxxx
funds to be transferred should come from savings from another item in the
appropriation of the office.142
5) No law shall be passed authorizing any transfer of appropriations;
however, the President, the President of the Senate, the Speaker of the
On July 30, 1977, President Marcos issued PD No. 1177, providing in its House of Representatives, the Chief Justice of the Supreme Court, and the
Section 44 that: heads of Constitutional Commissions may, by law, be authorized to
augment any item in the general appropriations law for their respective
offices from savings in other items of their respective appropriations.
Section 44. Authority to Approve Fund Transfers. The President shall have
the authority to transfer any fund appropriated for the different
departments, bureaus, offices and agencies of the Executive Department xxxx
which are included in the General Appropriations Act, to any program,
project, or activity of any department, bureau or office included in the
The foregoing history makes it evident that the Constitutional Commission
General Appropriations Act or approved after its enactment.
included Section 25(5), supra, to keep a tight rein on the exercise of the
power to transfer funds appropriated by Congress by the President and the
The President shall, likewise, have the authority to augment any other high officials of the Government named therein. The Court stated in
appropriation of the Executive Department in the General Appropriations Nazareth v. Villar:144
Act, from savings in the appropriations of another department, bureau,
office or agency within the Executive Branch, pursuant to the provisions of
In the funding of current activities, projects, and programs, the general
Article VIII, Section 16 (5) of the Constitution.
rule should still be that the budgetary amount contained in the
appropriations bill is the extent Congress will determine as sufficient for the
In Demetria v. Alba, however, the Court struck down the first paragraph of budgetary allocation for the proponent agency. The only exception is found
Section 44 for contravening Section 16(5)of the 1973 Constitution, ruling: in Section 25 (5), Article VI of the Constitution, by which the President, the
President of the Senate, the Speaker of the House of Representatives, the
Chief Justice of the Supreme Court, and the heads of Constitutional
Paragraph 1 of Section 44 of P.D. No. 1177 unduly over-extends the
Commissions are authorized to transfer appropriations to augmentany item
privilege granted under said Section 16. It empowers the President to
in the GAA for their respective offices from the savings in other items of
indiscriminately transfer funds from one department, bureau, office or
their respective appropriations. The plain language of the constitutional
agency of the Executive Department to any program, project or activity of
restriction leaves no room for the petitioner’s posture, which we should
any department, bureau or office included in the General Appropriations
now dispose of as untenable.
Act or approved after its enactment, without regard as to whether or not
the funds to be transferred are actually savings in the item from which the
same are to be taken, or whether or not the transfer is for the purpose of It bears emphasizing that the exception in favor of the high officials named
augmenting the item to which said transfer is to be made. It does not only in Section 25(5), Article VI of the Constitution limiting the authority to
completely disregard the standards set in the fundamental law, thereby transfer savings only to augment another item in the GAA is strictly but
amounting to an undue delegation of legislative powers, but likewise goes reasonably construed as exclusive. As the Court has expounded in Lokin,
beyond the tenor thereof. Indeed, such constitutional infirmities render the Jr. v. Commission on Elections:
provision in question null and void.143
When the statute itself enumerates the exceptions to the application of the
It is significant that Demetria was promulgated 25 days after the general rule, the exceptions are strictly but reasonably construed. The
ratification by the people of the 1987 Constitution, whose Section 25(5) of exceptions extend only as far as their language fairly warrants, and all
Article VI is identical to Section 16(5), Article VIII of the 1973 Constitution, doubts should be resolved in favor of the general provision rather than the
to wit: exceptions. Where the general rule is established by a statute with
exceptions, none but the enacting authority can curtail the former. Not
even the courts may add to the latter by implication, and it is a rule that an
Section 25. x x x
express exception excludes all others, although it is always proper in
determining the applicability of the rule to inquire whether, in a particular law, generally, is the GAA of a given fiscal year. To comply with the first
case, it accords with reason and justice. requisite, the GAAs should expressly authorize the transfer of funds.

The appropriate and natural office of the exception is to exempt something Did the GAAs expressly authorize the transfer of funds?
from the scope of the general words of a statute, which is otherwise within
the scope and meaning of such general words. Consequently, the existence
In the 2011 GAA, the provision that gave the President and the other high
of an exception in a statute clarifies the intent that the statute shall apply
officials the authority to transfer funds was Section 59, as follows:
to all cases not excepted. Exceptions are subject to the rule of strict
construction; hence, any doubt will be resolved in favor of the general
provision and against the exception. Indeed, the liberal construction of a Section 59. Use of Savings. The President of the Philippines, the Senate
statute will seem to require in many circumstances that the exception, by President, the Speaker of the House of Representatives, the Chief Justice of
which the operation of the statute is limited or abridged, should receive a the Supreme Court, the Heads of Constitutional Commissions enjoying
restricted construction. fiscal autonomy, and the Ombudsman are hereby authorized to augment
any item in this Act from savings in other items of their respective
appropriations.
Accordingly, we should interpret Section 25(5), supra, in the context of a
limitation on the President’s discretion over the appropriations during the
Budget Execution Phase. In the 2012 GAA, the empowering provision was Section 53, to wit:

b. Requisites for the valid transfer of Section 53. Use of Savings. The President of the Philippines, the Senate
appropriated funds under Section President, the Speaker of the House of Representatives, the Chief Justice of
25(5), Article VI of the 1987 the Supreme Court, the Heads of Constitutional Commissions enjoying
Constitution fiscal autonomy, and the Ombudsman are hereby authorized to augment
any item in this Act from savings in other items of their respective
appropriations.
The transfer of appropriated funds, to be valid under Section 25(5), supra,
must be made upon a concurrence of the following requisites, namely:
In fact, the foregoing provisions of the 2011 and 2012 GAAs were cited by
the DBM as justification for the use of savings under the DAP.145
(1) There is a law authorizing the President, the President of the
Senate, the Speaker of the House of Representatives, the Chief
Justice of the Supreme Court, and the heads of the Constitutional A reading shows, however, that the aforequoted provisions of the GAAs of
Commissions to transfer funds within their respective offices; 2011 and 2012 were textually unfaithful to the Constitution for not carrying
the phrase "for their respective offices" contained in Section 25(5), supra.
The impact of the phrase "for their respective offices" was to authorize only
(2) The funds to be transferred are savings generated from the
transfers of funds within their offices (i.e., in the case of the President, the
appropriations for their respective offices; and (3) The purpose of
transfer was to an item of appropriation within the Executive). The
the transfer is to augment an item in the general appropriations
provisions carried a different phrase ("to augment any item in this Act"),
law for their respective offices.
and the effect was that the 2011 and 2012 GAAs thereby literally allowed
the transfer of funds from savings to augment any item in the GAAs even if
b.1. First Requisite–GAAs of 2011 and the item belonged to an office outside the Executive. To that extent did the
2012 lacked valid provisions to 2011 and 2012 GAAs contravene the Constitution. At the very least, the
authorize transfers of funds under aforequoted provisions cannot be used to claim authority to transfer
the DAP; hence, transfers under the appropriations from the Executive to another branch, or to a constitutional
DAP were unconstitutional commission.

Section 25(5), supra, not being a self-executing provision of the Apparently realizing the problem, Congress inserted the omitted phrase in
Constitution, must have an implementing law for it to be operative. That the counterpart provision in the 2013 GAA, to wit:
Section 52. Use of Savings. The President of the Philippines, the Senate clearly "portions or balances of any programmed appropriation…free from
President, the Speaker of the House of Representatives, the Chief Justice of any obligation or encumbrances which are (i) still available after the
the Supreme Court, the Heads of Constitutional Commissions enjoying completion or final discontinuance or abandonment of the work, activity or
fiscal autonomy, and the Ombudsman are hereby authorized to use savings purpose for which the appropriation is authorized…"
in their respective appropriations to augment actual deficiencies incurred
for the current year in any item of their respective appropriations.
We partially find for the petitioners.

Even had a valid law authorizing the transfer of funds pursuant to Section
In ascertaining the meaning of savings, certain principles should be borne
25(5), supra, existed, there still remained two other requisites to be met,
in mind. The first principle is that Congress wields the power of the purse.
namely: that the source of funds to be transferred were savings from
Congress decides how the budget will be spent; what PAPs to fund; and the
appropriations within the respective offices; and that the transfer must be
amounts of money to be spent for each PAP. The second principle is that
for the purpose of augmenting an item of appropriation within the
the Executive, as the department of the Government tasked to enforce the
respective offices.
laws, is expected to faithfully execute the GAA and to spend the budget in
accordance with the provisions of the GAA.149 The Executive is expected to
b.2. Second Requisite – There were faithfully implement the PAPs for which Congress allocated funds, and to
no savings from which funds limit the expenditures within the allocations, unless exigencies result to
could be sourced for the DAP deficiencies for which augmentation is authorized, subject to the conditions
Were the funds used in the DAP actually savings? provided by law. The third principle is that in making the President’s power
to augment operative under the GAA, Congress recognizes the need for
flexibility in budget execution. In so doing, Congress diminishes its own
The petitioners claim that the funds used in the DAP — the unreleased
power of the purse, for it delegates a fraction of its power to the Executive.
appropriations and withdrawn unobligated allotments — were not actual
But Congress does not thereby allow the Executive to override its authority
savings within the context of Section 25(5), supra, and the relevant
over the purse as to let the Executive exceed its delegated authority. And
provisions of the GAAs. Belgica argues that "savings" should be understood
the fourth principle is that savings should be actual. "Actual" denotes
to refer to the excess money after the items that needed to be funded have
something that is real or substantial, or something that exists presently in
been funded, or those that needed to be paid have been paid pursuant to
fact, as opposed to something that is merely theoretical, possible, potential
the budget.146 The petitioners posit that there could be savings only when
or hypothetical.150
the PAPs for which the funds had been appropriated were actually
implemented and completed, or finally discontinued or abandoned. They
insist that savings could not be realized with certainty in the middle of the The foregoing principles caution us to construe savings strictly against
fiscal year; and that the funds for "slow-moving" PAPs could not be expanding the scope of the power to augment. It is then indubitable that
considered as savings because such PAPs had not actually been abandoned the power to augment was to be used only when the purpose for which the
or discontinued yet.147 They stress that NBC No. 541, by allowing the funds had been allocated were already satisfied, or the need for such funds
withdrawn funds to be reissued to the "original program or project from had ceased to exist, for only then could savings be properly realized. This
which it was withdrawn," conceded that the PAPs from which the supposed interpretation prevents the Executive from unduly transgressing Congress’
savings were taken had not been completed, abandoned or discontinued.148 power of the purse.

The OSG represents that "savings" were "appropriations balances," being The definition of "savings" in the GAAs, particularly for 2011, 2012 and
the difference between the appropriation authorized by Congress and the 2013, reflected this interpretation and made it operational, viz:
actual amount allotted for the appropriation; that the definition of "savings"
in the GAAs set only the parameters for determining when savings
Savings refer to portions or balances of any programmed appropriation in
occurred; that it was still the President (as well as the other officers vested
this Act free from any obligation or encumbrance which are: (i) still
by the Constitution with the authority to augment) who ultimately
available after the completion or final discontinuance or abandonment of
determined when savings actually existed because savings could be
the work, activity or purpose for which the appropriation is authorized; (ii)
determined only during the stage of budget execution; that the President
from appropriations balances arising from unpaid compensation and related
must be given a wide discretion to accomplish his tasks; and that the
costs pertaining to vacant positions and leaves of absence without pay; and
withdrawn unobligated allotments were savings inasmuch as they were
(iii) from appropriations balances realized from the implementation of and have remained with the DBM technically speaking. Ergo, unreleased
measures resulting in improved systems and efficiencies and thus enabled appropriations refer to appropriations with allotments but without
agencies to meet and deliver the required or planned targets, programs disbursement authority.
and services approved in this Act at a lesser cost.
For us to consider unreleased appropriations as savings, unless these met
The three instances listed in the GAAs’ aforequoted definition were a sure the statutory definition of savings, would seriously undercut the
indication that savings could be generated only upon the purpose of the congressional power of the purse, because such appropriations had not
appropriation being fulfilled, or upon the need for the appropriation being even reached and been used by the agency concerned vis-à-vis the PAPs
no longer existent. for which Congress had allocated them. However, if an agency has unfilled
positions in its plantilla and did not receive an allotment and NCA for such
vacancies, appropriations for such positions, although unreleased, may
The phrase "free from any obligation or encumbrance" in the definition of
already constitute savings for that agency under the second instance.
savings in the GAAs conveyed the notion that the appropriation was at that
stage when the appropriation was already obligated and the appropriation
was already released. This interpretation was reinforced by the Unobligated allotments, on the other hand, were encompassed by the first
enumeration of the three instances for savings to arise, which showed that part of the definition of "savings" in the GAA, that is, as "portions or
the appropriation referred to had reached the agency level. It could not be balances of any programmed appropriation in this Act free from any
otherwise, considering that only when the appropriation had reached the obligation or encumbrance." But the first part of the definition was further
agency level could it be determined whether (a) the PAP for which the qualified by the three enumerated instances of when savings would be
appropriation had been authorized was completed, finally discontinued, or realized. As such, unobligated allotments could not be indiscriminately
abandoned; or (b) there were vacant positions and leaves of absence declared as savings without first determining whether any of the three
without pay; or (c) the required or planned targets, programs and services instances existed. This signified that the DBM’s withdrawal of unobligated
were realized at a lesser cost because of the implementation of measures allotments had disregarded the definition of savings under the GAAs.
resulting in improved systems and efficiencies.
Justice Carpio has validly observed in his Separate Concurring Opinion that
The DBM declares that part of the savings brought under the DAP came MOOE appropriations are deemed divided into twelve monthly allocations
from "pooling of unreleased appropriations such as unreleased Personnel within the fiscal year; hence, savings could be generated monthly from the
Services appropriations which will lapse at the end of the year, unreleased excess or unused MOOE appropriations other than the Mandatory
appropriations of slow moving projects and discontinued projects per Zero- Expenditures and Expenditures for Business-type Activities because of the
Based Budgeting findings." physical impossibility to obligate and spend such funds as MOOE for a
period that already lapsed. Following this observation, MOOE for future
months are not savings and cannot be transferred.
The declaration of the DBM by itself does not state the clear legal basis for
the treatment of unreleased or unalloted appropriations as savings.
The DBM’s Memorandum for the President dated June 25, 2012 (which
became the basis of NBC No. 541) stated:
The fact alone that the appropriations are unreleased or unalloted is a
mere description of the status of the items as unalloted or unreleased.
They have not yet ripened into categories of items from which savings can ON THE AUTHORITY TO WITHDRAW UNOBLIGATED ALLOTMENTS
be generated. Appropriations have been considered "released" if there has
already been an allotment or authorization to incur obligations and
5.0 The DBM, during the course of performance reviews conducted
disbursement authority. This means that the DBM has issued either an ABM
on the agencies’ operations, particularly on the implementation of
(for those not needing clearance), or a SARO (for those needing
their projects/activities, including expenses incurred in
clearance), and consequently an NCA, NCAA or CDC, as the case may be.
undertaking the same, have been continuously calling the
Appropriations remain unreleased, for instance, because of noncompliance
attention of all National Government agencies (NGAs) with low
with documentary requirements (like the Special Budget Request), or
levels of obligations as of end of the first quarter to speedup the
simply because of the unavailability of funds. But the appropriations do not
implementation of their programs and projects in the second
actually reach the agencies to which they were allocated under the GAAs,
quarter.
6.0 Said reminders were made in a series of consultation meetings "[r]eissued for the original programs and projects of the agencies/OUs
with the concerned agencies and with call-up letters sent. concerned, from which the allotments were withdrawn"153 supported the
conclusion that the PAPs had not yet been finally discontinued or
abandoned. Thus, the purpose for which the withdrawn funds had been
7.0 Despite said reminders and the availability of funds at the
appropriated was not yet fulfilled, or did not yet cease to exist, rendering
department’s disposal, the level of financial performance of some
the declaration of the funds as savings impossible.
departments registered below program, with the targeted
obligations/disbursements for the first semester still not being
met. Worse, NBC No. 541 immediately considered for withdrawal all released
allotments in 2011 charged against the 2011 GAA that had remained
unobligated based on the following considerations, to wit:
8.0 In order to maximize the use of the available allotment, all
unobligated balances as of June 30, 2012, both for continuing and
current allotments shall be withdrawn and pooled to fund fast 5.4.1 The departments/agencies’ approved priority programs and
moving programs/projects. projects are assumed to be implementation-ready and doable
during the given fiscal year; and
9.0 It may be emphasized that the allotments to be withdrawn will
be based on the list of slow moving projects to be identified by the 5.4.2 The practice of having substantial carryover appropriations
agencies and their catch up plans to be evaluated by the DBM. may imply that the agency has a slower-than-programmed
implementation capacity or agency tends to implement projects
within a two-year timeframe.
It is apparent from the foregoing text that the withdrawal of unobligated
allotments would be based on whether the allotments pertained to slow-
moving projects, or not. However, NBC No. 541 did not set in clear terms Such withdrawals pursuant to NBC No. 541, the circular that affected the
the criteria for the withdrawal of unobligated allotments, viz: unobligated allotments for continuing and current appropriations as of June
30, 2012, disregarded the 2-year period of availability of the appropriations
for MOOE and capital outlay extended under Section 65, General Provisions
3.1. These guidelines shall cover the withdrawal of unobligated
of the 2011 GAA, viz:
allotments as of June 30, 2012 ofall national government agencies
(NGAs) charged against FY 2011 Continuing Appropriation (R.A.
No. 10147) and FY 2012 Current Appropriation (R.A. No. 10155), Section 65. Availability of Appropriations. — Appropriations for MOOE and
pertaining to: capital outlays authorized in this Act shall be available for release and
obligation for the purpose specified, and under the same special provisions
applicable thereto, for a period extending to one fiscal year after the end of
3.1.1 Capital Outlays (CO);
the year in which such items were appropriated: PROVIDED, That
appropriations for MOOE and capital outlays under R.A. No. 9970 shall be
3.1.2 Maintenance and Other Operating Expenses made available up to the end of FY 2011: PROVIDED, FURTHER, That a
(MOOE) related to the implementation of programs and report on these releases and obligations shall be submitted to the Senate
projects, as well as capitalized MOOE; and Committee on Finance and the House Committee on Appropriations.

3.1.3 Personal Services corresponding to unutilized and Section 63 General Provisions of the 2012 GAA, viz:
pension benefits declared as savings by the agencies
concerned based on their undated/validated list of
Section 63. Availability of Appropriations. — Appropriations for MOOE and
pensioners.
capital outlays authorized in this Act shall be available for release and
obligation for the purpose specified, and under the same special provisions
A perusal of its various provisions reveals that NBC No. 541 targeted the applicable thereto, for a period extending to one fiscal year after the end of
"withdrawal of unobligated allotments of agencies with low levels of the year in which such items were appropriated: PROVIDED, That a report
obligations"151 "to fund priority and/or fast-moving on these releases and obligations shall be submitted to the Senate
programs/projects."152 But the fact that the withdrawn allotments could be
Committee on Finance and the House Committee on Appropriations, either middle of the fiscal year, in effect deprived funding for PAPs with existing
in printed form or by way of electronic document.154 appropriations under the GAAs.155

Thus, another alleged area of constitutional infirmity was that the DAP and The respondents belie the accusation, insisting that the unobligated
its relevant issuances shortened the period of availability of the allotments were being withdrawn upon the instance of the implementing
appropriations for MOOE and capital outlays. agencies based on their own assessment that they could not obligate those
allotments pursuant to the President’s directive for them to spend their
appropriations as quickly as they could in order to ramp up the economy.156
Congress provided a one-year period of availability of the funds for all
allotment classes in the 2013 GAA (R.A. No. 10352), to wit:
We agree with the petitioners.
Section 63. Availability of Appropriations.— All appropriations authorized in
this Act shall be available for release and obligation for the purposes Contrary to the respondents’ insistence, the withdrawals were upon the
specified, and under the same special provisions applicable thereto, until initiative of the DBM itself. The text of NBC No. 541 bears this out, to wit:
the end of FY 2013: PROVIDED, That a report on these releases and
obligations shall be submitted to the Senate Committee on Finance and
5.2 For the purpose of determining the amount of unobligated allotments
House Committee on Appropriations, either in printed form or by way of
that shall be withdrawn, all departments/agencies/operating units (OUs)
electronic document.
shall submit to DBM not later than July 30, 2012, the following budget
accountability reports as of June 30, 2012;
Yet, in his memorandum for the President dated May 20, 2013, Sec. Abad
sought omnibus authority to consolidate savings and unutilized balances to
• Statement of Allotments, Obligation and Balances (SAOB);
fund the DAP on a quarterly basis, viz:

• Financial Report of Operations (FRO); and


7.0 If the level of financial performance of some department will
register below program, even with the availability of funds at their
disposal, the targeted obligations/disbursements for each quarter • Physical Report of Operations.
will not be met. It is important to note that these funds will lapse
at the end of the fiscal year if these remain unobligated. 5.3 In the absence of the June 30, 2012 reports cited under item 5.2 of
this Circular, the agency’s latest report available shall be used by DBM as
8.0 To maximize the use of the available allotment, all unobligated basis for withdrawal of allotment. The DBM shall compute/approximate the
balances at the end of every quarter, both for continuing and agency’s obligation level as of June 30 to derive its unobligated allotments
current allotments shall be withdrawn and pooled to fund fast as of same period. Example: If the March 31 SAOB or FRO reflects actual
moving programs/projects. obligations of P 800M then the June 30 obligation level shall approximate
to ₱1,600 M (i.e., ₱800 M x 2 quarters).
9.0 It may be emphasized that the allotments to be withdrawn will
be based on the list of slow moving projects to be identified by the The petitioners assert that no law had authorized the withdrawal and
agencies and their catch up plans to be evaluated by the DBM. transfer of unobligated allotments and the pooling of unreleased
appropriations; and that the unbridled withdrawal of unobligated allotments
and the retention of appropriated funds were akin to the impoundment of
The validity period of the affected appropriations, already given the brief
appropriations that could be allowed only in case of "unmanageable
Lifes pan of one year, was further shortened to only a quarter of a year
national government budget deficit" under the GAAs,157 thus violating the
under the DBM’s memorandum dated May 20, 2013.
provisions of the GAAs of 2011, 2012 and 2013 prohibiting the retention or
deduction of allotments.158
The petitioners accuse the respondents of forcing the generation of savings
in order to have a larger fund available for discretionary spending. They
aver that the respondents, by withdrawing unobligated allotments in the
In contrast, the respondents emphasize that NBC No. 541 adopted a Constitution, or (ii) there are clear economic indications of an impending
spending, not saving, policy as a last-ditch effort of the Executive to push occurrence of such condition, as determined by the Development Budget
agencies into actually spending their appropriations; that such policy did Coordinating Committee and approved by the President.
not amount to an impoundment scheme, because impoundment referred to
the decision of the Executive to refuse to spend funds for political or
The 2012 and 2013 GAAs contained similar provisions.
ideological reasons; and that the withdrawal of allotments under NBC No.
541 was made pursuant to Section 38, Chapter 5, Book VI of the
Administrative Code, by which the President was granted the authority to The withdrawal of unobligated allotments under the DAP should not be
suspend or otherwise stop further expenditure of funds allotted to any regarded as impoundment because it entailed only the transfer of funds,
agency whenever in his judgment the public interest so required. not the retention or deduction of appropriations.

The assertions of the petitioners are upheld. The withdrawal and transfer of Nor could Section 68 of the 2011 GAA (and the similar provisions of the
unobligated allotments and the pooling of unreleased appropriations were 2012 and 2013 GAAs) be applicable. They uniformly stated:
invalid for being bereft of legal support. Nonetheless, such withdrawal of
unobligated allotments and the retention of appropriated funds cannot be Section 68. Prohibition Against Retention/Deduction of Allotment. Fund
considered as impoundment. releases from appropriations provided in this Act shall be transmitted intact
or in full to the office or agency concerned. No retention or deduction as
According to Philippine Constitution Association v. reserves or overhead shall be made, except as authorized by law, or upon
Enriquez:159 "Impoundment refers to a refusal by the President, for direction of the President of the Philippines. The COA shall ensure
whatever reason, to spend funds made available by Congress. It is the compliance with this provision to the extent that sub-allotments by
failure to spend or obligate budget authority of any type." Impoundment agencies to their subordinate offices are in conformity with the release
under the GAA is understood to mean the retention or deduction of documents issued by the DBM.
appropriations. The 2011 GAA authorized impoundment only in case of
unmanageable National Government budget deficit, to wit: The provision obviously pertained to the retention or deduction of
allotments upon their release from the DBM, which was a different matter
Section 66. Prohibition Against Impoundment of Appropriations. No altogether. The Court should not expand the meaning of the provision by
appropriations authorized under this Act shall be impounded through applying it to the withdrawal of allotments.
retention or deduction, unless in accordance with the rules and regulations
to be issued by the DBM: PROVIDED, That all the funds appropriated for The respondents rely on Section 38, Chapter 5, Book VI of the
the purposes, programs, projects and activities authorized under this Act, Administrative Code of 1987 to justify the withdrawal of unobligated
except those covered under the Unprogrammed Fund, shall be released allotments. But the provision authorized only the suspension or stoppage of
pursuant to Section 33 (3), Chapter 5, Book VI of E.O. No. 292. further expenditures, not the withdrawal of unobligated allotments, to wit:

Section 67. Unmanageable National Government Budget Deficit. Retention Section 38. Suspension of Expenditure of Appropriations.- Except as
or deduction of appropriations authorized in this Act shall be effected only otherwise provided in the General Appropriations Act and whenever in his
in cases where there is an unmanageable national government budget judgment the public interest so requires, the President, upon notice to the
deficit. head of office concerned, is authorized to suspend or otherwise stop further
expenditure of funds allotted for any agency, or any other expenditure
Unmanageable national government budget deficit as used in this section authorized in the General Appropriations Act, except for personal services
shall be construed to mean that (i) the actual national government budget appropriations used for permanent officials and employees.
deficit has exceeded the quarterly budget deficit targets consistent with the
full-year target deficit as indicated in the FY 2011 Budget of Moreover, the DBM did not suspend or stop further expenditures in
accordance with Section 38, supra, but instead transferred the funds to
Expenditures and Sources of Financing submitted by the President and other PAPs.
approved by Congress pursuant to Section 22, Article VII of the
It is relevant to remind at this juncture that the balances of appropriations x x x Augmentation implies the existence in this Act of a program, activity,
that remained unexpended at the end of the fiscal year were to be reverted or project with an appropriation, which upon implementation, or
to the General Fund.1âwphi1 This was the mandate of Section 28, Chapter subsequent evaluation of needed resources, is determined to be deficient.
IV, Book VI of the Administrative Code, to wit: In no case shall a non-existent program, activity, or project, be funded by
augmentation from savings or by the use of appropriations otherwise
authorized in this Act.
Section 28. Reversion of Unexpended Balances of Appropriations,
Continuing Appropriations.- Unexpended balances of appropriations
authorized in the General Appropriation Act shall revert to the In other words, an appropriation for any PAP must first be determined to
unappropriated surplus of the General Fund at the end of the fiscal year be deficient before it could be augmented from savings. Note is taken of
and shall not thereafter be available for expenditure except by subsequent the fact that the 2013 GAA already made this quite clear, thus:
legislative enactment: Provided, that appropriations for capital outlays shall
remain valid until fully spent or reverted: provided, further, that continuing
Section 52. Use of Savings. The President of the Philippines, the Senate
appropriations for current operating expenditures may be specifically
President, the Speaker of the House of Representatives, the Chief Justice of
recommended and approved as such in support of projects whose effective
the Supreme Court, the Heads of Constitutional Commissions enjoying
implementation calls for multi-year expenditure commitments: provided,
fiscal autonomy, and the Ombudsman are hereby authorized to use savings
finally, that the President may authorize the use of savings realized by an
in their respective appropriations to augment actual deficiencies incurred
agency during given year to meet non-recurring expenditures in a
for the current year in any item of their respective appropriations.
subsequent year.

As of 2013, a total of ₱144.4 billion worth of PAPs were implemented


The balances of continuing appropriations shall be reviewed as part of the
through the DAP.161
annual budget preparation process and the preparation process and the
President may approve upon recommendation of the Secretary, the
reversion of funds no longer needed in connection with the activities Of this amount ₱82.5 billion were released in 2011 and ₱54.8 billion in
funded by said continuing appropriations. 2012.162 Sec. Abad has reported that 9% of the total DAP releases were
applied to the PAPs identified by the legislators.163
The Executive could not circumvent this provision by declaring unreleased
appropriations and unobligated allotments as savings prior to the end of The petitioners disagree, however, and insist that the DAP supported the
the fiscal year. following PAPs that had not been covered with appropriations in the
respective GAAs, namely:
b.3. Third Requisite – No funds from
savings could be transferred under (i) ₱1.5 billion for the Cordillera People’s Liberation Army;
the DAP to augment deficient items
not provided in the GAA (ii) ₱1.8 billion for the Moro National Liberation Front;

The third requisite for a valid transfer of funds is that the purpose of the (iii) ₱700 million for assistance to Quezon Province;164
transfer should be "to augment an item in the general appropriations law
for the respective offices." The term "augment" means to enlarge or
increase in size, amount, or degree.160 (iv) ₱50 million to ₱100 (million) each to certain senators;165

The GAAs for 2011, 2012 and 2013 set as a condition for augmentation (v) ₱10 billion for the relocation of families living along dangerous
that the appropriation for the PAP item to be augmented must be deficient, zones under the National Housing Authority;
to wit: –
(vi) ₱10 billion and ₱20 billion equity infusion under the Bangko
Sentral;
(vii) ₱5.4 billion landowners’ compensation under the Department
of Agrarian Reform; and research capability
building in priority areas
identified as strategic to P 43,504,024
(viii) ₱8.6 billion for the ARMM comprehensive peace and National Development 1,164,517,589
development program; Personnel Services 391,978,387
Maintenance and Other P 1,600,000,000
(ix) ₱6.5 billion augmentation of LGU internal revenue allotments Operating Expenses 
Capital Outlays
(x) ₱5 billion for crucial projects like tourism road construction
under the Department of Tourism and the Department of Public
the pertinent provision of the 2011 GAA (R.A. No. 10147) showed that
Works and Highways;
Congress had appropriated only ₱537,910,000 for MOOE, but nothing for
personnel services and capital outlays, to wit:
(xi) ₱1.8 billion for the DAR-DPWH Tulay ng Pangulo;

Personne Maintenan Capital TOTAL


(xii) ₱1.96 billion for the DOH-DPWH rehabilitation of regional
l ce Outlays
health units; and
Services and Other
Operating
(xiii) ₱4 billion for the DepEd-PPP school infrastructure projects.166 Expenditu
res
In refutation, the OSG argues that a total of 116 DAP-financed PAPs were
implemented, had appropriation covers, and could properly be accounted III Operations
for because the funds were released following and pursuant to the standard .
practices adopted by the DBM.167 In support of its argument, the OSG has a Funding 177,406, 1,887,365 49,090, 2,113,861
submitted seven evidence packets containing memoranda, SAROs, and . Assistance to 000 ,000 000 ,000
other pertinent documents relative to the implementation and fund Science
transfers under the DAP.168 and
Technology
Activities
Upon careful review of the documents contained in the seven evidence
packets, we conclude that the "savings" pooled under the DAP were 1 Central Office 1,554,238 1,554,238
allocated to PAPs that were not covered by any appropriations in the . ,000 ,000
pertinent GAAs.
a. 537,910,0 537,910,0
For example, the SARO issued on December 22, 2011 for the highly Generati 00 00
vaunted Disaster Risk, Exposure, Assessment and Mitigation (DREAM) on of
project under the Department of Science and Technology (DOST) covered new
the amount of ₱1.6 Billion,169 broken down as follows: knowledg
e and
technolo
APPROPRIATION PARTICULARS AMOUNT gies and
CODE AUTHORIZED research
capability
A.03.a.01.a Generation of new building
knowledge and technologies in
the appropriation code and the particulars appearing in the SARO did not
priority
correspond to the program specified in the GAA, whose particulars were
areas
Research and Management Services(inclusive of the following activities: (1)
identified
Technological and Economic Assessment for Industry, Energy and Utilities;
as
(2) Dissemination of Science and Technology Information; and (3)
strategic
Management of PCIERD Information System for Industry, Energy and
to
Utilities. Even assuming that Development, integration and coordination of
National
the National Research System for Industry, Energy and Emerging
Develop
Technology and Related Fields– the particulars stated in the SARO – could
ment
fall under the broad program description of Research and Management
Services– as appearing in the SARO, it would nonetheless remain a new
activity by reason of its not being specifically stated in the GAA. As such,
Aside from this transfer under the DAP to the DREAM project exceeding by
the DBM, sans legislative authorization, could not validly fund and
almost 300% the appropriation by Congress for the program Generation of
implement such PAP under the DAP.
new knowledge and technologies and research capability building in priority
areas identified as strategic to National Development, the Executive
allotted funds for personnel services and capital outlays. The Executive In defending the disbursements, however, the OSG contends that the
thereby substituted its will to that of Congress. Worse, the Executive had Executive enjoyed sound discretion in implementing the budget given the
not earlier proposed any amount for personnel services and capital outlays generality in the language and the broad policy objectives identified under
in the NEP that became the basis of the 2011 GAA.170 the GAAs;172 and that the President enjoyed unlimited authority to spend
the initial appropriations under his authority to declare and utilize
savings,173 and in keeping with his duty to faithfully execute the laws.
It is worth stressing in this connection that the failure of the GAAs to set
aside any amounts for an expense category sufficiently indicated that
Congress purposely did not see fit to fund, much less implement, the PAP Although the OSG rightly contends that the Executive was authorized to
concerned. This indication becomes clearer when even the President spend in line with its mandate to faithfully execute the laws (which
himself did not recommend in the NEP to fund the PAP. The consequence included the GAAs), such authority did not translate to unfettered
was that any PAP requiring expenditure that did not receive any discretion that allowed the President to substitute his own will for that of
appropriation under the GAAs could only be a new PAP, any funding for Congress. He was still required to remain faithful to the provisions of the
which would go beyond the authority laid down by Congress in enacting the GAAs, given that his power to spend pursuant to the GAAs was but a
GAAs. That happened in some instances under the DAP. delegation to him from Congress. Verily, the power to spend the public
wealth resided in Congress, not in the Executive.174 Moreover, leaving the
spending power of the Executive unrestricted would threaten to undo the
In relation to the December 22, 2011 SARO issued to the Philippine Council
principle of separation of powers.175
for Industry, Energy and Emerging Technology Research and Development
(DOST-PCIEETRD)171 for Establishment of the Advanced Failure Analysis
Laboratory, which reads: Congress acts as the guardian of the public treasury in faithful discharge of
its power of the purse whenever it deliberates and acts on the budget
proposal submitted by the Executive.176 Its power of the purse is touted as
APPROPRIATIO PARTICULARS AMOUNTthe very foundation of its institutional strength,177 and underpins "all other
N AUTHORIZED
legislative decisions and regulating the balance of influence between the
CODE legislative and executive branches of government."178 Such enormous
power encompasses the capacity to generate money for the Government,
Development, integration and coordination of the National to appropriate public funds, and to spend the money.179 Pertinently, when it
Research System for Industry, Energy and Emerging Technology exercises its power of the purse, Congress wields control by specifying the
A.02.a
and Related Fields PAPs for which public money should be spent.
Capital Outlays P 300,000,000
It is the President who proposes the budget but it is Congress that has the
final say on matters of appropriations.180For this purpose, appropriation
involves two governing principles, namely: (1) "a Principle of the Public Well, in the Memos that we submitted to you, such an instance, Your Honor
Fisc, asserting that all monies received from whatever source by any part
of the government are public funds;" and (2) "a Principle of Appropriations
JUSTICE BERSAMIN:
Control, prohibiting expenditure of any public money without legislative
authorization."181To conform with the governing principles, the Executive
cannot circumvent the prohibition by Congress of an expenditure for a PAP Can you tell me two instances? I don’t recall having read your material.
by resorting to either public or private funds.182 Nor could the Executive
transfer appropriated funds resulting in an increase in the budget for one SECRETARY ABAD:
PAP, for by so doing the appropriation for another PAP is necessarily
decreased. The terms of both appropriations will thereby be violated.
Well, the first instance had to do with a request from the House of
Representatives. They started building their e-library in 2010 and they had
b.4 Third Requisite – Cross-border a budget for about 207 Million but they lack about 43 Million to complete its
augmentations from savings were 250 Million requirements. Prior to that, the COA, in an audit observation
prohibited by the Constitution informed the Speaker that they had to continue with that construction
otherwise the whole building, as well as the equipments therein may suffer
By providing that the President, the President of the Senate, the Speaker from serious deterioration. And at that time, since the budget of the House
of the House of Representatives, the Chief Justice of the Supreme Court, of Representatives was not enough to complete 250 Million, they wrote to
and the Heads of the Constitutional Commissions may be authorized to the President requesting for an augmentation of that particular item, which
augment any item in the GAA "for their respective offices," Section 25(5), was granted, Your Honor. The second instance in the Memos is a request
supra, has delineated borders between their offices, such that funds from the Commission on Audit. At the time they were pushing very
appropriated for one office are prohibited from crossing over to another strongly the good governance programs of the government and therefore,
office even in the guise of augmentation of a deficient item or items. Thus, part of that is a requirement to conduct audits as well as review financial
we call such transfers of funds cross-border transfers or cross-border reports of many agencies. And in the performance of that function, the
augmentations. Commission on Audit needed information technology equipment as well as
hire consultants and litigators to help them with their audit work and for
that they requested funds from the Executive and the President saw that it
To be sure, the phrase "respective offices" used in Section 25(5), supra,
was important for the Commission to be provided with those IT equipments
refers to the entire Executive, with respect to the President; the Senate,
and litigators and consultants and the request was granted, Your Honor.
with respect to the Senate President; the House of Representatives, with
respect to the Speaker; the Judiciary, with respect to the Chief Justice; the
Constitutional Commissions, with respect to their respective Chairpersons. JUSTICE BERSAMIN:

Did any cross-border transfers or augmentations transpire? These cross border examples, cross border augmentations were not
supported by appropriations…
During the oral arguments on January 28, 2014, Sec. Abad admitted
making some cross-border augmentations, to wit: SECRETARY ABAD:

JUSTICE BERSAMIN: They were, we were augmenting existing items within their… (interrupted)

Alright, the whole time that you have been Secretary of Department of JUSTICE BERSAMIN:
Budget and Management, did the Executive Department ever redirect any
part of savings of the National Government under your control cross border No, appropriations before you augmented because this is a cross border
to another department? and the tenor or text of the Constitution is quite clear as far as I am
concerned. It says here, "The power to augment may only be made to
SECRETARY ABAD:
increase any item in the General Appropriations Law for their respective
AMOUNT
offices." Did you not feel constricted by this provision?
DATE (In thousand pesos)
OFFICE PURPOSE RELEASE
SECRETARY ABAD: D Reserve Releases
Imposed
Well, as the Constitution provides, the prohibition we felt was on the
transfer of appropriations, Your Honor. What we thought we did was to Commission on IT Infrastructure 11/11/11   143,700
transfer savings which was needed by the Commission to address Audit Program and hiring of
deficiency in an existing item in both the Commission as well as in the additional litigation
House of Representatives; that’s how we saw…(interrupted) experts

Congress – Completion of the 07/23/12 207,034 250,000


JUSTICE BERSAMIN: House of construction of the (Savings of
Representative Legislative Library and HOR)
So your position as Secretary of Budget is that you could do that? s Archives
Building/Congressional
e-library
SECRETARY ABAD:

In an extreme instances because…(interrupted) The respondents further stated in their memorandum that the President
"made available" to the "Commission on Elections the savings of his
department upon [its] request for funds…"187 This was another instance of a
JUSTICE BERSAMIN:
cross-border augmentation.

No, no, in all instances, extreme or not extreme, you could do that, that’s
The respondents justified all the cross-border transfers thusly:
your feeling.

99. The Constitution does not prevent the President from transferring
SECRETARY ABAD:
savings of his department to another department upon the latter’s request,
provided it is the recipient department that uses such funds to augment its
Well, in that particular situation when the request was made by the own appropriation. In such a case, the President merely gives the other
Commission and the House of Representatives, we felt that we needed to department access to public funds but he cannot dictate how they shall be
respond because we felt…(interrupted).183 applied by that department whose fiscal autonomy is guaranteed by the
Constitution.188
The records show, indeed, that funds amounting to ₱143,700,000.00 and
₱250,000,000.00 were transferred under the DAP respectively to the In the oral arguments held on February 18, 2014, Justice Vicente V.
COA184 and the House of Representatives.185 Those transfers of funds, which Mendoza, representing Congress, announced a different characterization of
constituted cross-border augmentations for being from the Executive to the the cross-border transfers of funds as in the nature of "aid" instead of
COA and the House of Representatives, are graphed as follows:186 "augmentation," viz:

HONORABLE MENDOZA:

The cross-border transfers, if Your Honors please, is not an application of


the DAP. What were these cross-border transfers? They are transfers of
savings as defined in the various General Appropriations Act. So, that
makes it similar to the DAP, the use of savings. There was a cross-border
which appears to be in violation of Section 25, paragraph 5 of Article VI, in So, the residual powers labelled in Marcos v. Manglapus would be the basis
the sense that the border was crossed. But never has it been claimed that for this theory of the government?
the purpose was to augment a deficient item in another department of the
government or agency of the government. The cross-border transfers, if
HONORABLE MENDOZA:
Your Honors please, were in the nature of [aid] rather than augmentations.
Here is a government entity separate and independent from the Executive
Department solely in need of public funds. The President is there 24 hours Yes, if Your Honor, please.
a day, 7 days a week. He’s in charge of the whole operation although six or
seven heads of government offices are given the power to augment. Only JUSTICE LEONEN:
the President stationed there and in effect in-charge and has the
responsibility for the failure of any part of the government. You have
election, for one reason or another, the money is not enough to hold A while ago, Justice Carpio mentioned that the remedy is might be to go to
election. There would be chaos if no money is given as an aid, not to Congress. That there are opportunities and there have been opportunities
augment, but as an aid to a department like COA. The President is of the President to actually go to Congress and ask for supplemental
responsible in a way that the other heads, given the power to augment, are budgets?
not. So, he cannot very well allow this, if Your Honor please.189
HONORABLE MENDOZA:
JUSTICE LEONEN:
If there is time to do that, I would say yes.
May I move to another point, maybe just briefly. I am curious that the
position now, I think, of government is that some transfers of savings is JUSTICE LEONEN:
now considered to be, if I’m not mistaken, aid not augmentation. Am I
correct in my hearing of your argument?
So, the theory of aid rather than augmentation applies in extra-ordinary
situation?
HONORABLE MENDOZA:
HONORABLE MENDOZA:
That’s our submission, if Your Honor, please.
Very extra-ordinary situations.
JUSTICE LEONEN:
JUSTICE LEONEN:
May I know, Justice, where can we situate this in the text of the
Constitution? Where do we actually derive the concepts that transfers of
appropriation from one branch to the other or what happened in DAP can But Counsel, this would be new doctrine, in case?
be considered a said? What particular text in the Constitution can we
situate this? HONORABLE MENDOZA:

HONORABLE MENDOZA: Yes, if Your Honor please.190

There is no particular provision or statutory provision for that matter, if Regardless of the variant characterizations of the cross-border transfers of
Your Honor please. It is drawn from the fact that the Executive is the funds, the plain text of Section 25(5), supra, disallowing cross border
executive in-charge of the success of the government. transfers was disobeyed. Cross-border transfers, whether as augmentation,
or as aid, were prohibited under Section 25(5), supra.
JUSTICE LEONEN:
4. BESFs – was not included. This meant that the collection of additional
Sourcing the DAP from unprogrammed revenues from new sources did not warrant the release of the
funds despite the original revenue targets unprogrammed funds. Hence, even if the revenues not considered in the
not having been exceeded was invalid BESFs were collected or generated, the basic condition that the revenue
collections should exceed the revenue targets must still be complied with in
order to justify the release of the unprogrammed funds.
Funding under the DAP were also sourced from unprogrammed funds
provided in the GAAs for 2011, 2012,and 2013. The respondents stress,
however, that the unprogrammed funds were not brought under the DAP The view that there were only two instances when the unprogrammed
as savings, but as separate sources of funds; and that, consequently, the funds could be released was bolstered by the following texts of the Special
release and use of unprogrammed funds were not subject to the Provisions of the 2011 and 2012 GAAs, to wit:
restrictions under Section 25(5), supra.
2011 GAA
The documents contained in the Evidence Packets by the OSG have
confirmed that the unprogrammed funds were treated as separate sources
1. Release of Fund. The amounts authorized herein shall be released only
of funds. Even so, the release and use of the unprogrammed funds were
when the revenue collections exceed the original revenue targets submitted
still subject to restrictions, for, to start with, the GAAs precisely specified
by the President of the Philippines to Congress pursuant to Section 22,
the instances when the unprogrammed funds could be released and the
Article VII of the Constitution, including savings generated from
purposes for which they could be used.
programmed appropriations for the year: PROVIDED, That collections
arising from sources not considered in the aforesaid original revenue
The petitioners point out that a condition for the release of the targets may be used to cover releases from appropriations in this Fund:
unprogrammed funds was that the revenue collections must exceed PROVIDED, FURTHER, That in case of newly approved loans for foreign-
revenue targets; and that the release of the unprogrammed funds was assisted projects, the existence of a perfected loan agreement for the
illegal because such condition was not met.191 purpose shall be sufficient basis for the issuance of a SARO covering the
loan proceeds: PROVIDED, FURTHERMORE, That if there are savings
generated from the programmed appropriations for the first two quarters of
The respondents disagree, holding that the release and use of the
the year, the DBM may, subject to the approval of the President, release
unprogrammed funds under the DAP were in accordance with the pertinent
the pertinent appropriations under the Unprogrammed Fund corresponding
provisions of the GAAs. In particular, the DBM avers that the
to only fifty percent (50%) of the said savings net of revenue shortfall:
unprogrammed funds could be availed of when any of the following three
PROVIDED, FINALLY, That the release of the balance of the total savings
instances occur, to wit: (1) the revenue collections exceeded the original
from programmed appropriations for the year shall be subject to fiscal
revenue targets proposed in the BESFs submitted by the President to
programming and approval of the President.
Congress; (2) new revenues were collected or realized from sources not
originally considered in the BESFs; or(3) newly-approved loans for foreign
assisted projects were secured, or when conditions were triggered for other 2012 GAA
sources of funds, such as perfected loan agreements for foreign-assisted
projects.192 This view of the DBM was adopted by all the respondents in
1. Release of the Fund. The amounts authorized herein shall be released
their Consolidated Comment.193
only when the revenue collections exceed the original revenue targets
submitted by the President of the Philippines to Congress pursuant to
The BESFs for 2011, 2012 and 2013 uniformly defined "unprogrammed Section 22, Article VII of the Constitution: PROVIDED, That collections
appropriations" as appropriations that provided standby authority to incur arising from sources not considered in the aforesaid original revenue
additional agency obligations for priority PAPs when revenue collections targets may be used to cover releases from appropriations in this Fund:
exceeded targets, and when additional foreign funds are PROVIDED, FURTHER, That in case of newly approved loans for foreign-
generated.194 Contrary to the DBM’s averment that there were three assisted projects, the existence of a perfected loan agreement for the
instances when unprogrammed funds could be released, the BESFs purpose shall be sufficient basis for the issuance of a SARO covering the
envisioned only two instances. The third mentioned by the DBM – the loan proceeds.
collection of new revenues from sources not originally considered in the
As can be noted, the provisos in both provisions to the effect that condition being deemed complied with once the revenue collections from a
"collections arising from sources not considered in the aforesaid original particular source already exceeded the stated target.
revenue targets may be used to cover releases from appropriations in this
Fund" gave the authority to use such additional revenues for appropriations
The BESF provided for the following sources of revenue, with the
funded from the unprogrammed funds. They did not at all waive
corresponding revenue target stated for each source of revenue, to wit:
compliance with the basic requirement that revenue collections must still
exceed the original revenue targets.
TAX REVENUES
In contrast, the texts of the provisos with regard to additional revenues
generated from newly-approved foreign loans were clear to the effect that Taxes on Net Income and Profits
the perfected loan agreement would be in itself "sufficient basis" for the Taxes on Property
issuance of a SARO to release the funds but only to the extent of the Taxes on Domestic Goods and Services
amount of the loan. In such instance, the revenue collections need not
exceed the revenue targets to warrant the release of the loan proceeds, General Sales, Turnover or VAT
and the mere perfection of the loan agreement would suffice. Selected Excises on Goods

It can be inferred from the foregoing that under these provisions of the Selected Taxes on Services
GAAs the additional revenues from sources not considered in the BESFs Taxes on the Use of Goods or Property or Permission to Perform
must be taken into account in determining if the revenue collections Activities
exceeded the revenue targets. The text of the relevant provision of the Other Taxes
2013 GAA, which was substantially similar to those of the GAAs for 2011 Taxes on International Trade and Transactions
and 2012, already made this explicit, thus:

NON-TAX REVENUES
1. Release of the Fund. The amounts authorized herein shall be released
only when the revenue collections exceed the original revenue targets
submitted by the President of the Philippines to Congress pursuant to Fees and Charges
Section 22, Article VII of the Constitution, including collections arising from BTR Income
sources not considered in the aforesaid original revenue target, as certified
by the BTr: PROVIDED, That in case of newly approved loans for foreign- Government Services
assisted projects, the existence of a perfected loan agreement for the Interest on NG Deposits
purpose shall be sufficient basis for the issuance of a SARO covering the Interest on Advances to Government Corporations
loan proceeds. Income from Investments

Consequently, that there were additional revenues from sources not Interest on Bond Holdings
considered in the revenue target would not be enough. The total revenue
collections must still exceed the original revenue targets to justify the
release of the unprogrammed funds (other than those from newly- Guarantee Fee
approved foreign loans). Gain on Foreign Exchange
NG Income Collected by BTr

The present controversy on the unprogrammed funds was rooted in the


correct interpretation of the phrase "revenue collections should exceed the Dividends on Stocks
original revenue targets." The petitioners take the phrase to mean that the NG Share from Airport Terminal Fee
total revenue collections must exceed the total revenue target stated in the NG Share from PAGCOR Income
BESF, but the respondents understand the phrase to refer only to the NG Share from MIAA Profit
collections for each source of revenue as enumerated in the BESF, with the
Privatization The certifications reflected that by collecting dividends amounting to ₱23.8
Foreign Grants billion in 2011, ₱19.419 billion in 2012, and ₱12.438 billion in 2013 the BTr
had exceeded only the ₱5.5 billion in target revenues in the form of
dividends from stocks in each of 2011 and 2012, and only the ₱10 billion in
Thus, when the Court required the respondents to submit a certification
target revenues in the form of dividends from stocks in 2013.
from the Bureau of Treasury (BTr) to the effect that the revenue collections
had exceeded the original revenue targets,195 they complied by submitting
certifications from the BTr and Department of Finance (DOF) pertaining to However, the requirement that revenue collections exceed the original
only one identified source of revenue – the dividends from the shares of revenue targets was to be construed in light of the purpose for which the
stock held by the Government in government-owned and controlled unprogrammed funds were incorporated in the GAAs as standby
corporations. appropriations to support additional expenditures for certain priority PAPs
should the revenue collections exceed the resource targets assumed in the
budget or when additional foreign project loan proceeds were realized. The
To justify the release of the unprogrammed funds for 2011, the OSG
unprogrammed funds were included in the GAAs to provide ready cover so
presented the certification dated March 4, 2011 issued by DOF
as not to delay the implementation of the PAPs should new or additional
Undersecretary Gil S. Beltran, as follows:
revenue sources be realized during the year.200 Given the tenor of the
certifications, the unprogrammed funds were thus not yet supported by the
This is to certify that under the Budget for Expenditures and Sources of corresponding resources.201
Financing for 2011, the programmed income from dividends from shares of
stock in government-owned and controlled corporations is 5.5 billion.
The revenue targets stated in the BESF were intended to address the
funding requirements of the proposed programmed appropriations. In
This is to certify further that based on the records of the Bureau of contrast, the unprogrammed funds, as standby appropriations, were to be
Treasury, the National Government has recorded dividend income released only when there were revenues in excess of what the
amounting to ₱23.8 billion as of 31 January 2011.196 programmed appropriations required. As such, the revenue targets should
be considered as a whole, not individually; otherwise, we would be dealing
For 2012, the OSG submitted the certification dated April 26, 2012 issued with artificial revenue surpluses. The requirement that revenue collections
by National Treasurer Roberto B. Tan, viz: must exceed revenue target should be understood to mean that the
revenue collections must exceed the total of the revenue targets stated in
the BESF. Moreover, to release the unprogrammed funds simply because
This is to certify that the actual dividend collections remitted to the there was an excess revenue as to one source of revenue would be an
National Government for the period January to March 2012 amounted to unsound fiscal management measure because it would disregard the
₱19.419 billion compared to the full year program of ₱5.5 billion for budget plan and foster budget deficits, in contravention of the
2012.197 Government’s surplus budget policy.202

And, finally, for 2013, the OSG presented the certification dated July 3, We cannot, therefore, subscribe to the respondents’ view.
2013 issued by National Treasurer Rosalia V. De Leon, to wit:

5.
This is to certify that the actual dividend collections remitted to the Equal protection, checks and balances,
National Government for the period January to May 2013 amounted to and public accountability challenges
₱12.438 billion compared to the full year program of ₱10.0198 billion for
2013.
The DAP is further challenged as violative of the Equal Protection Clause,
the system of checks and balances, and the principle of public
Moreover, the National Government accounted for the sale of the right to accountability.
build and operate the NAIA expressway amounting to ₱11.0 billion in June
2013.199
With respect to the challenge against the DAP under the Equal Protection
Clause,203 Luna argues that the implementation of the DAP was "unfair as it
[was] selective" because the funds released under the DAP was not made could not of itself warrant a finding of contravention of the Equal Protection
available to all the legislators, with some of them refusing to avail Clause. The denial of equal protection of any law should be an issue to be
themselves of the DAP funds, and others being unaware of the availability raised only by parties who supposedly suffer it, and, in these cases, such
of such funds. Thus, the DAP practised "undue favoritism" in favor of select parties would be the few legislators claimed to have been discriminated
legislators in contravention of the Equal Protection Clause. against in the releases of funds under the DAP. The reason for the
requirement is that only such affected legislators could properly and fully
bring to the fore when and how the denial of equal protection occurred,
Similarly, COURAGE contends that the DAP violated the Equal Protection
and explain why there was a denial in their situation. The requirement was
Clause because no reasonable classification was used in distributing the
not met here. Consequently, the Court was not put in the position to
funds under the DAP; and that the Senators who supposedly availed
determine if there was a denial of equal protection. To have the Court do
themselves of said funds were differently treated as to the amounts they
so despite the inadequacy of the showing of factual and legal support
respectively received.
would be to compel it to speculate, and the outcome would not do justice
to those for whose supposed benefit the claim of denial of equal protection
Anent the petitioners’ theory that the DAP violated the system of checks has been made.
and balances, Luna submits that the grant of the funds under the DAP to
some legislators forced their silence about the issues and anomalies
The argument that the release of funds under the DAP effectively stayed
surrounding the DAP. Meanwhile, Belgica stresses that the DAP, by allowing
the hands of the legislators from conducting congressional inquiries into
the legislators to identify PAPs, authorized them to take part in the
the legality and propriety of the DAP is speculative. That deficiency
implementation and execution of the GAAs, a function that exclusively
eliminated any need to consider and resolve the argument, for it is
belonged to the Executive; that such situation constituted undue and
fundamental that speculation would not support any proper judicial
unjustified legislative encroachment in the functions of the Executive; and
determination of an issue simply because nothing concrete can thereby be
that the President arrogated unto himself the power of appropriation
gained. In order to sustain their constitutional challenges against official
vested in Congress because NBC No. 541 authorized the use of the funds
acts of the Government, the petitioners must discharge the basic burden of
under the DAP for PAPs not considered in the 2012 budget.
proving that the constitutional infirmities actually existed.205 Simply put,
guesswork and speculation cannot overcome the presumption of the
Finally, the petitioners insist that the DAP was repugnant to the principle of constitutionality of the assailed executive act.
public accountability enshrined in the Constitution,204 because the
legislators relinquished the power of appropriation to the Executive, and
We do not need to discuss whether or not the DAP and its implementation
exhibited a reluctance to inquire into the legality of the DAP.
through the various circulars and memoranda of the DBM transgressed the
system of checks and balances in place in our constitutional system. Our
The OSG counters the challenges, stating that the supposed discrimination earlier expositions on the DAP and its implementing issuances infringing
in the release of funds under the DAP could be raised only by the affected the doctrine of separation of powers effectively addressed this particular
Members of Congress themselves, and if the challenge based on the concern.
violation of the Equal Protection Clause was really against the
constitutionality of the DAP, the arguments of the petitioners should be
Anent the principle of public accountability being transgressed because the
directed to the entitlement of the legislators to the funds, not to the
adoption and implementation of the DAP constituted an assumption by the
proposition that all of the legislators should have been given such
Executive of Congress’ power of appropriation, we have already held that
entitlement.
the DAP and its implementing issuances were policies and acts that the
Executive could properly adopt and do in the execution of the GAAs to the
The challenge based on the contravention of the Equal Protection Clause, extent that they sought to implement strategies to ramp up or accelerate
which focuses on the release of funds under the DAP to legislators, lacks the economy of the country.
factual and legal basis. The allegations about Senators and Congressmen
being unaware of the existence and implementation of the DAP, and about
6.
some of them having refused to accept such funds were unsupported with
Doctrine of operative fact was applicable
relevant data. Also, the claim that the Executive discriminated against
some legislators on the ground alone of their receiving less than the others
After declaring the DAP and its implementing issuances constitutionally after the judiciary, in an appropriate case, declares its invalidity, it is
infirm, we must now deal with the consequences of the declaration. entitled to obedience and respect. Parties may have acted under it and
may have changed their positions. What could be more fitting than that in
a subsequent litigation regard be had to what has been done while such
Article 7 of the Civil Code provides:
legislative or executive act was in operation and presumed to be valid in all
respects. It is now accepted as a doctrine that prior to its being nullified, its
Article 7. Laws are repealed only by subsequent ones, and their violation or existence as a fact must be reckoned with. This is merely to reflect
non-observance shall not be excused by disuse, or custom or practice to awareness that precisely because the judiciary is the governmental organ
the contrary. which has the final say on whether or not a legislative or executive
measure is valid, a period of time may have elapsed before it can exercise
When the courts declared a law to be inconsistent with the Constitution, the power of judicial review that may lead to a declaration of nullity. It
the former shall be void and the latter shall govern. would be to deprive the law of its quality of fairness and justice then, if
there be no recognition of what had transpired prior to such adjudication.

Administrative or executive acts, orders and regulations shall be valid only


when they are not contrary to the laws or the Constitution. In the language of an American Supreme Court decision: ‘The actual
existence of a statute, prior to such a determination [of
unconstitutionality], is an operative fact and may have consequences which
A legislative or executive act that is declared void for being unconstitutional cannot justly be ignored. The past cannot always be erased by a new
cannot give rise to any right or obligation.206 However, the generality of the judicial declaration. The effect of the subsequent ruling as to invalidity may
rule makes us ponder whether rigidly applying the rule may at times be have to be considered in various aspects, with respect to particular
impracticable or wasteful. Should we not recognize the need to except from relations, individual and corporate, and particular conduct, private and
the rigid application of the rule the instances in which the void law or official.’"
executive act produced an almost irreversible result?

The doctrine of operative fact recognizes the existence of the law or


The need is answered by the doctrine of operative fact. The doctrine, executive act prior to the determination of its unconstitutionality as an
definitely not a novel one, has been exhaustively explained in De Agbayani operative fact that produced consequences that cannot always be erased,
v. Philippine National Bank:207 ignored or disregarded. In short, it nullifies the void law or executive act
but sustains its effects. It provides an exception to the general rule that a
The decision now on appeal reflects the orthodox view that an void or unconstitutional law produces no effect.208 But its use must be
unconstitutional act, for that matter an executive order or a municipal subjected to great scrutiny and circumspection, and it cannot be invoked to
ordinance likewise suffering from that infirmity, cannot be the source of validate an unconstitutional law or executive act, but is resorted to only as
any legal rights or duties. Nor can it justify any official act taken under it. a matter of equity and fair play.209 It applies only to cases where
Its repugnancy to the fundamental law once judicially declared results in its extraordinary circumstances exist, and only when the extraordinary
being to all intents and purposes a mere scrap of paper. As the new Civil circumstances have met the stringent conditions that will permit its
Code puts it: ‘When the courts declare a law to be inconsistent with the application.
Constitution, the former shall be void and the latter shall govern.’
Administrative or executive acts, orders and regulations shall be valid only We find the doctrine of operative fact applicable to the adoption and
when they are not contrary to the laws of the Constitution. It is implementation of the DAP. Its application to the DAP proceeds from equity
understandable why it should be so, the Constitution being supreme and and fair play. The consequences resulting from the DAP and its related
paramount. Any legislative or executive act contrary to its terms cannot issuances could not be ignored or could no longer be undone.
survive.

To be clear, the doctrine of operative fact extends to a void or


Such a view has support in logic and possesses the merit of simplicity. It unconstitutional executive act. The term executive act is broad enough to
may not however be sufficiently realistic. It does not admit of doubt that include any and all acts of the Executive, including those that are quasi
prior to the declaration of nullity such challenged legislative or executive legislative and quasi-judicial in nature. The Court held so in Hacienda
act must have been in force and had to be complied with. This is so as until Luisita, Inc. v. Presidential Agrarian Reform Council:210
Nonetheless, the minority is of the persistent view that the applicability of utilized under his orders or those of his authorized military
the operative fact doctrine should be limited to statutes and rules and representatives.’
regulations issued by the executive department that are accorded the same
status as that of a statute or those which are quasi-legislative in nature.
Evidently, the operative fact doctrine is not confined to statutes and rules
Thus, the minority concludes that the phrase ‘executive act’ used in the
and regulations issued by the executive department that are accorded the
case of De Agbayani v. Philippine National Bank refers only to acts, orders,
same status as that of a statute or those which are quasi-legislative in
and rules and regulations that have the force and effect of law. The
nature.
minority also made mention of the Concurring Opinion of Justice Enrique
Fernando in Municipality of Malabang v. Benito, where it was supposedly
made explicit that the operative fact doctrine applies to executive acts, Even assuming that De Agbayani initially applied the operative fact doctrine
which are ultimately quasi-legislative in nature. only to executive issuances like orders and rules and regulations, said
principle can nonetheless be applied, by analogy, to decisions made by the
President or the agencies under the executive department. This doctrine, in
We disagree. For one, neither the De Agbayani case nor the Municipality of
the interest of justice and equity, can be applied liberally and in a broad
Malabang case elaborates what ‘executive act’ mean. Moreover, while
sense to encompass said decisions of the executive branch. In keeping with
orders, rules and regulations issued by the President or the executive
the demands of equity, the Court can apply the operative fact doctrine to
branch have fixed definitions and meaning in the Administrative Code and
acts and consequences that resulted from the reliance not only on a law or
jurisprudence, the phrase ‘executive act’ does not have such specific
executive act which is quasi-legislative in nature but also on decisions or
definition under existing laws. It should be noted that in the cases cited by
orders of the executive branch which were later nullified. This Court is not
the minority, nowhere can it be found that the term ‘executive act’ is
unmindful that such acts and consequences must be recognized in the
confined to the foregoing. Contrarily, the term ‘executive act’ is broad
higher interest of justice, equity and fairness.
enough to encompass decisions of administrative bodies and agencies
under the executive department which are subsequently revoked by the
agency in question or nullified by the Court. Significantly, a decision made by the President or the administrative
agencies has to be complied with because it has the force and effect of law,
springing from the powers of the President under the Constitution and
A case in point is the concurrent appointment of Magdangal B. Elma (Elma)
existing laws. Prior to the nullification or recall of said decision, it may have
as Chairman of the Presidential Commission on Good Government (PCGG)
produced acts and consequences in conformity to and in reliance of said
and as Chief Presidential Legal Counsel (CPLC) which was declared
decision, which must be respected. It is on this score that the operative
unconstitutional by this Court in Public Interest Center, Inc. v. Elma. In
fact doctrine should be applied to acts and consequences that resulted from
said case, this Court ruled that the concurrent appointment of Elma to
the implementation of the PARC Resolution approving the SDP of HLI. (Bold
these offices is in violation of Section 7, par. 2, Article IX-B of the 1987
underscoring supplied for emphasis)
Constitution, since these are incompatible offices. Notably, the
appointment of Elma as Chairman of the PCGG and as CPLC is, without a
question, an executive act. Prior to the declaration of unconstitutionality of In Commissioner of Internal Revenue v. San Roque Power
the said executive act, certain acts or transactions were made in good faith Corporation,211 the Court likewise declared that "for the operative fact
and in reliance of the appointment of Elma which cannot just be set aside doctrine to apply, there must be a ‘legislative or executive measure,’
or invalidated by its subsequent invalidation. meaning a law or executive issuance." Thus, the Court opined there that
the operative fact doctrine did not apply to a mere administrative practice
of the Bureau of Internal Revenue, viz:
In Tan v. Barrios, this Court, in applying the operative fact doctrine, held
that despite the invalidity of the jurisdiction of the military courts over
civilians, certain operative facts must be acknowledged to have existed so Under Section 246, taxpayers may rely upon a rule or ruling issued by the
as not to trample upon the rights of the accused therein. Relevant thereto, Commissioner from the time the rule or ruling is issued up to its reversal
in Olaguer v. Military Commission No. 34, it was ruled that ‘military by the Commissioner or this Court. The reversal is not given retroactive
tribunals pertain to the Executive Department of the Government and are effect. This, in essence, is the doctrine of operative fact. There must,
simply instrumentalities of the executive power, provided by the legislature however, be a rule or ruling issued by the Commissioner that is relied upon
for the President as Commander-in-Chief to aid him in properly by the taxpayer in good faith. A mere administrative practice, not
commanding the army and navy and enforcing discipline therein, and formalized into a rule or ruling, will not suffice because such a mere
administrative practice may not be uniformly and consistently applied. An consequence of every declaration of constitutional invalidity. It can be
administrative practice, if not formalized as a rule or ruling, will not be invoked only in situations where the nullification of the effects of what used
known to the general public and can be availed of only by those with to be a valid law would result in inequity and injustice;212but where no such
informal contacts with the government agency. result would ensue, the general rule that an unconstitutional law is totally
ineffective should apply.
It is clear from the foregoing that the adoption and the implementation of
the DAP and its related issuances were executive acts.1avvphi1 The DAP In that context, as Justice Brion has clarified, the doctrine of operative fact
itself, as a policy, transcended a merely administrative practice especially can apply only to the PAPs that can no longer be undone, and whose
after the Executive, through the DBM, implemented it by issuing various beneficiaries relied in good faith on the validity of the DAP, but cannot
memoranda and circulars. The pooling of savings pursuant to the DAP from apply to the authors, proponents and implementors of the DAP, unless
the allotments made available to the different agencies and departments there are concrete findings of good faith in their favor by the proper
was consistently applied throughout the entire Executive. With the tribunals determining their criminal, civil, administrative and other
Executive, through the DBM, being in charge of the third phase of the liabilities.
budget cycle – the budget execution phase, the President could legitimately
adopt a policy like the DAP by virtue of his primary responsibility as the
WHEREFORE, the Court PARTIALLY GRANTS the petitions for certiorari and
Chief Executive of directing the national economy towards growth and
prohibition; and DECLARES the following acts and practices under the
development. This is simply because savings could and should be
Disbursement Acceleration Program, National Budget Circular No. 541 and
determined only during the budget execution phase.
related executive issuances UNCONSTITUTIONAL for being in violation of
Section 25(5), Article VI of the 1987 Constitution and the doctrine of
As already mentioned, the implementation of the DAP resulted into the use separation of powers, namely:
of savings pooled by the Executive to finance the PAPs that were not
covered in the GAA, or that did not have proper appropriation covers, as
(a) The withdrawal of unobligated allotments from the
well as to augment items pertaining to other departments of the
implementing agencies, and the declaration of the withdrawn
Government in clear violation of the Constitution. To declare the
unobligated allotments and unreleased appropriations as savings
implementation of the DAP unconstitutional without recognizing that its
prior to the end of the fiscal year and without complying with the
prior implementation constituted an operative fact that produced
statutory definition of savings contained in the General
consequences in the real as well as juristic worlds of the Government and
Appropriations Acts;
the Nation is to be impractical and unfair. Unless the doctrine is held to
apply, the Executive as the disburser and the offices under it and
elsewhere as the recipients could be required to undo everything that they (b) The cross-border transfers of the savings of the Executive to
had implemented in good faith under the DAP. That scenario would be augment the appropriations of other offices outside the Executive;
enormously burdensome for the Government. Equity alleviates such and
burden.
(c) The funding of projects, activities and programs that were not
The other side of the coin is that it has been adequately shown as to be covered by any appropriation in the General Appropriations Act.
beyond debate that the implementation of the DAP yielded undeniably
positive results that enhanced the economic welfare of the country. To The Court further DECLARES VOID the use of unprogrammed funds despite
count the positive results may be impossible, but the visible ones, like the absence of a certification by the National Treasurer that the revenue
public infrastructure, could easily include roads, bridges, homes for the collections exceeded the revenue targets for non-compliance with the
homeless, hospitals, classrooms and the like. Not to apply the doctrine of conditions provided in the relevant General Appropriations Acts.
operative fact to the DAP could literally cause the physical undoing of such
worthy results by destruction, and would result in most undesirable
wastefulness. SO ORDERED.

Nonetheless, as Justice Brion has pointed out during the deliberations, the
doctrine of operative fact does not always apply, and is not always the
PHILIPPINE CONSTITUTION ASSOCIATION, INC., JOSE E. ROMERO, years. In the third place, all government officers and employees are given
SALVADOR ARANETA, GUILLERMO B. GUEVARA, PIO PEDROSA, only one retirement benefit irrespective of their length of service in the
CONRADO BENITEZ, JOSE M. ARUEGO, SOTERO H. LAUREL, government, whereas, under Republic Act 3836 because of no age
FELIXBERTO M. SERRANO and ROMAN OZAETA, Petitioners, v. limitation, a Senator or Member of the House of Representatives upon
PEDRO M. GIMENEZ, JOSE VELASCO, ELADIO SALITA, and JOSE being elected for 24 years will be entitled to two retirement benefits or
AVILES, Respondents. equivalent to six years salary. Also, while the payment of retirement
benefits (annuity) to an employee who had been retired and reappointed is
Roman Ozaeta, Guillermo B. Guevara, Jose M. Aruego, Sotero H . suspended during his new employment (under Commonwealth Act 186, as
Laurel and Felixberto M. Serrano, for Petitioners. amended), this is not so under Republic Act 3836. Lastly, Republic Act
3836 grants retirement benefits to officials who are not members of the
Solicitor General for Respondents. Government Service Insurance System. Most grantees of retirement
benefits under the various retirement laws have to be members or must at
least contribute a portion of their monthly salaries to the System.
SYLLABUS
4. ID.; ID.; TITLE OF LAW NOT GERMANE TO THE SUBJECT MATTER. —
Under Republic Act No. 3836, amending the first paragraph of section 12,
subsection (c) of Commonwealth Act 186, as amended by Republic Acts
1. CONSTITUTIONAL LAW; STATUTE INVOLVING EXPENDITURES OF Nos. 660 and 3096, the retirement benefits are granted to members of the
PUBLIC FUNDS; PERSONALITY OF TAXPAYERS TO ATTACK ITS Government Service Insurance System who have rendered at least twenty
CONSTITUTIONALITY. — In the determination of the degree of interest years of service regardless of age. This provision is related and germane to
essential to give the requisite standing to attack the constitutionality of a the subject of Commonwealth Act 186. On the other hand, the succeeding
statute, the general rule is that not only persons individually affected, but paragraph of Republic Act No. 3836 refers to members of Congress and to
also taxpayers have sufficient interest in preventing the illegal expenditure elective officers thereof who are not members of the Government Service
of moneys raised by taxation and they may, therefore, question the Insurance System. To provide retirement benefits, therefore, for these
constitutionality of statutes requiring expenditure of public moneys. (11 officials, would relate to subject matter, not germane to Commonwealth
Am. Jur. 761) Act No. 186.

2. ID.; REPUBLIC ACT 3836; INCREASE IN EMOLUMENTS OF MEMBERS OF 5. ID.; ID.; ID.; DUTY OF COURT TO DECLARE VOID THE STATUTE. — The
CONGRESS. — Republic Act No. 3836 provides for the retirement benefits requirement that the subject of an act shall be expressed in its title is not a
for members of Congress which in effect are increases in the emoluments mere rule of legislative procedure, directory to Congress; it is mandatory.
of Senators and Members of the House of Representatives, to take effect It is the duty of the courts to declare void any statute not conforming to
upon the approval of the Act, which was on June 22, 1963. Retirement the constitutional provision. (See Walker v. State, 49 Alabama 329;
benefits were immediately available thereunder without awaiting the Cooley, Constitutional Limitations, 8th Ed., Volume I, pp. 162-164.)
expiration of the full term of all the Members of the Senate and the House
of Representatives approving such increase. Such provision clearly runs
counter to the prohibition in Article VI, Section 14 of the Constitution.
DECISION
3. ID.; ID.; LAW VIOLATES EQUAL PROTECTION CLAUSE OF THE
CONSTITUTION. — The features of Republic Act 3836 are discriminatory,
and therefore violate the equal protection clause of the Constitution. (Art. REGALA, J.:
III, Sec. 1, par. 1.) In the first place, while the said law grants retirement
benefits to Senators and Members of the House of Representatives who are
elective officials, it does not include other elective officials of the provinces,
We are called upon in this case to decide the grave and fundamental
municipalities and chartered cities. Secondly, all members of Congress
problem of the constitutionality of Republic Act No. 3836 "insofar as the
under Republic Act 3836 are given retirement benefits after serving twelve
same allows retirement gratuity and commutation of vacation and sick
years, not necessarily continuous, whereas, most government officers and
leave to Senators and Representatives, and to the elective officials of both
employees are given retirement benefits after serving for at least twenty
Houses (of Congress). The suit was instituted by the Philippine Constitution
Association, Inc. (Philconsa, for short), a non-profit, civic organization, duly
incorporated under Philippine laws, by way of petition for prohibition with Be it enacted by the Senate and House of Representatives of the
preliminary injunction to restrain the Auditor General of the Philippines and Philippines in Congress assembled:jgc:chanrobles.com.ph
the disbursing officers of both Houses of Congress from "passing in audit
the vouchers, and from countersigning the checks or treasury warrants for "SECTION 1, Subsection (c), Section twelve of Commonwealth Act
the payment to any former Senator or former Member of the House of Numbered One Hundred eighty-six as amended by Republic Act Numbered
Representatives of retirement and vacation gratuities pursuant to Republic Thirty hundred ninety-six, is further amended to read as
Act No. 3836; and likewise restraining the respondent disbursing officers of follows:chanrob1es virtual 1aw library
the House and Senate, respectively, and their successors in office from
paying the said retirement and vacation gratuities."cralaw virtua1aw library ‘(c) Retirement is likewise allowed to a member, regardless of age, who
has rendered at least twenty years of service. The benefit shall in addition
It is argued that the above-numbered Republic Act, at least to the end that to the return of his personal contributions plus interest and the payment of
it provided for the retirement of the members of Congress in the manner the corresponding employer’s premiums described in subsection (a) of
and terms that it did, is unconstitutional and void. The challenge to the Section five hereof, without interest, be only a gratuity equivalent to one
constitutionality of the law is centered on the following month’s salary for every year of service, based on the highest rate
propositions:chanrob1es virtual 1aw library received, but not to exceed twenty-four months; Provided, That the retiring
officer or employee has been in the service of the said employer or office
1. The provision for the retirement of the members and certain officers of for at least four years immediately preceding his retirement.
Congress is not expressed in the title of the bill, in violation of Section
21(1) of Article VI of the Constitution. ‘Retirement is also allowed to a senator or a member of the House of
Representatives and to an elective officer of either House of the Congress,
2. The provision on retirement gratuity is an attempt to circumvent the regardless of age, provided that in the case of a Senator or Member, he
Constitutional ban on increase of salaries of the members of Congress must have served at least twelve years as a Senator and/or as a member
during their term of office, contrary to the provisions of Article VI, Section of the House of Representatives, and, in the case of an elective officer of
14 of the Constitution. either House, he must have served the government for at least twelve
years, not less than four years of which must have been rendered as such
3. The same provision constitutes "selfish class legislation" because it elective officer: Provided, That the gratuity payable to a retiring senator,
allows members and officers of Congress to retire after twelve (12) years member of the House of Representatives, or elective officer, of either
of service and gives them a gratuity equivalent to one year salary for every House, shall be equivalent to one year’s salary for every four years of
four years of service, which is not refundable in case of reinstatement or service in the government and the same shall be exempt from any tax
re-election of the retiree, while all other officers and employees of the whatsoever and shall be neither liable to attachment or execution nor
government can retire only after at least twenty (20) years of service and refundable in case of reinstatement or re-election of the retiree.
are given a gratuity which is only equivalent to one month salary for every
year of service, which, in any case, can not exceed 24 months. This gratuity is payable by the employer of office concerned which is
hereby authorized to provide the necessary appropriation or pay the same
4. The provision on vacation and sick leave, commutable at the highest from any unexpended items of appropriations or savings in its
rate received, insofar as members of Congress are concerned, is another appropriations.
attempt of the legislator to further increase their compensation in violation
of the Constitution. ‘Elective or appointive officials and employees paid gratuity under this
subsection shall be entitled to the commutation of the unused vacation and
The text of Republic Act No. 3836 sick leave, based on the highest rate received, which they may have to
their credit at the time of retirement."cralaw virtua1aw library
The text of Republic Act No. 3836 reads:jgc:chanrobles.com.ph
"SECTION 2. This Act shall take effect upon its approval.
"AN ACT AMENDING SUBSECTION (c), SECTION TWELVE OF
COMMONWEALTH ACT NUMBERED ONE HUNDRED EIGHTY-SIX AS "Approved, June 22, 1963."cralaw virtua1aw library
AMENDED BY REPUBLIC ACT NUMBERED THIRTY HUNDRED NINETY-
SIX:chanrob1es virtual 1aw library The Solicitor General’s Office, in representation of the respondents, filed its
answer on September 8, 1964, and contends, by way of special and "The inclusion of members of Congress in subsection (c), Section 12 C. A.
affirmative defenses, that:chanrob1es virtual 1aw library 186, as amended, will enable them to retire voluntarily, regardless of age,
after serving a minimum of twenty years as a Member of Congress. This
1. The grant of retirement or pension benefits under Republic Act No. 3836 gratuity will insure the security of the family of the retiring member of
to the officers objected to by the petitioner does not constitute "forbidden Congress without the latter engaging in other activities which may detract
compensation" within the meaning of Section 14 of Article VI of the from his exalted position and usefulness as lawmaker. It is expected that
Philippine Constitution. with this assurance of security for his loved ones, deserving and well-
intentioned but poor men will be attracted to serve their people in
2. The title of the law in question sufficiently complies with the provisions Congress."cralaw virtua1aw library
of Section 21, Article VI, of the Constitution that "no bill which may be
enacted into law shall embrace more than one subject which shall be As finally approved, the law (subsection [c], paragraph 2, Section 1, R.A.
expressed in the title of the bill."cralaw virtua1aw library 3836) allows a Senator or a Member of the House of Representatives and
an elective officer of either House of Congress to retire regardless of age.
3. The law in question does not constitute class legislation. To be eligible for retirement, he must have served for at least twelve years
as such Senator and/or as member of the House of Representatives. For an
4. Certain indispensable parties, specifically the elected officers of Congress elective officer of either House, he must have served the government for at
who are authorized to approve vouchers for payments for funds under the least twelve years, of which not less than four years must have been
law in question, and the claimants to the vouchers to be presented for rendered as such elective officer. The gratuity payable by the employer or
payment under said items, were not included in the petition. office concerned is equivalent to one year’s salary for every four years of
service in the government. Said gratuity is exempt from taxation, not liable
5. The petitioner has no standing to institute this suit. to attachment or execution, and not refundable in case of reinstatement or
re-election of the retiree.
6. The payment of commutable vacation and sick leave benefits under the
said Act is merely "in the nature of a basis for computing the gratuity due First legal point — personality of the Petitioner to bring suit.
each retiring member" and, therefore, is not an indirect scheme to increase
their salary. The first point to be considered is whether petitioner Philconsa has a
standing to institute this action. This Court has not hesitated to examine
A brief historical background of Republic Act No. 3836. past decisions involving this matter. This Court has repeatedly held that
when the petitioner, like in this case, is composed of substantial taxpayers,
Republic Act No. 3836 was originally House Bill No. 6051, which was and the outcome will affect their vital interests, they are allowed to bring
introduced by Congressmen Marcial R. Pimentel of Camarines Norte and this suit. (Pascual v. Secretary, G.R. No. L-10405, December 29, 1960;
Marcelino R. Veloso of the Third District of Leyte, on May 6, 1963. On the and Gonzales v. Hechanova, 60 Off. Gaz. 802 ([1963]).
same date, it was referred to the Committee on Civil Service, which on the
following May 8, submitted its REPORT No. 3129, recommending approval The petitioner, Philconsa, is precisely a non-profit, civic organization
of the bill with amendments, among others, that the word "TWENTY" in the composed of several leaders from all walks of life whose main objective is
bill as filed — representing the number of years that a senator or member to uphold the principles of the Constitution.
must serve in Congress to entitle him to retirement under the bill — must
be reduced to "TWELVE" years, and that the following words were inserted, In rejecting the motion to dismiss in the case of Pascual v. Secretary,
namely, "AND THE SAME (referring to GRATUITY) SHALL BE EXEMPT FROM supra, this Court stated, among other things, that "there are many
ANY TAX WHATSOEVER AND SHALL NOT BE LIABLE FROM ATTACHMENT decisions nullifying, at the instance of the taxpayers, laws providing the
OR EXECUTION NOR REFUNDABLE IN CASE OF REINSTATEMENT OR RE- disbursement of public funds, upon the theory that the expenditures of
ELECTION OF THE RETIREE." On May 8, 1963, the bill with the proposed public funds by an officer of the State for the purpose of administering an
amendments was approved on second reading. It was passed on third unconstitutional act constitutes a misappropriation of such funds, which
reading on May 13, 1963, and on the same day was sent to the Senate, may be enjoined at the request of the taxpayers." 1 This legislation
which, in turn, on May 23, 1963, passed it without amendment. The bill (Republic Act 3836) involves the disbursement of public funds.
was finally approved on June 22, 1963. As explained in the EXPLANATORY
NOTE attached to the bill, among others — We are not, however, unmindful of the ruling laid down by the Supreme
Court of the United States in the case of Massachusetts v. Mellon, 262 U.S.
447, holding that:jgc:chanrobles.com.ph embodies some limitations and prohibitions upon the members of
Congress, to wit:chanrob1es virtual 1aw library
". . . the relation of a taxpayer of the United States to the Federal
Government is very different. His interest in the money of the Treasury — 1. They may not hold any other office or employment in the Government
partly realized from taxation and partly from other sources — is shared without forfeiting their respective seats;
with millions of others; is comparatively minute and indeterminable; and
the effect upon future taxation of any payment out of the funds, so remote, 2. They shall not be appointed, during the time for which they are elected,
fluctuating and uncertain, that no basis is afforded for an appeal to the to any civil office which may have been created or the emoluments whereof
preventive powers of equity"  shall have been increased while they were members of Congress; (Section
16, Article VI, Constitution)
The general view in the United States, which is followed here, is stated in
the American Jurisprudence, thus — 3. They cannot be financially interested in any franchise;

"In the determination of the degree of interest essential to give the 4. They cannot appear in any civil case wherein the Government is an
requisite standing to attack the constitutionality of a statute the general adverse party;
rule is that not only persons individually affected, but also taxpayers have
sufficient interest in preventing the illegal expenditure of moneys raised by 5. They cannot appear as counsel before any Electoral Tribunal; and
taxation and may therefore question the constitutionality of statutes
requiring expenditures of public moneys." (II Am. Jur. Emphasis supplied) 6. They cannot appear as counsel in any criminal case where an officer or
employee of the Government is accused. (Section 17, Article VI,
As far as the first point is concerned, We hold, therefore, that the Constitution)
contention of the Solicitor General is untenable.
In addition to the above prohibitions, the Anti-Graft Law (Republic Act
Second legal point — Whether or not Republic Act No. 3836 falls within the 3019) also prohibits members of Congress to have any special interest in
any specific business which will directly or indirectly be favored by any law
prohibition embodied in Art. VI, section 14 of the or resolution authorized by them during their term of office.

Constitution. It is thus clear that the Constitutional Convention wisely surrounded the
Constitution with these limitations and prohibitions upon Members of
The first constitutional question is whether Republic Act 3836 violates Congress. This is a practical demonstration or application of the principle of
Section 14, Article VI, of the Constitution, which reads as checks and balance which is one of the peculiar characteristics of our
follows:jgc:chanrobles.com.ph Constitution. In the light of this background, can We conclude that
Congress can validly enact Republic Act 3836, providing retirement benefits
"The senators and the Members of the House of Representatives shall, to its members, without violating the provisions in the aforementioned
unless otherwise provided by law, receive an annual compensation of Article VI, Section 14, of the Constitution, regarding increase of the
seven thousand two hundred pesos each, including per diems and other compensation as including other emoluments?
emoluments or allowances, and exclusive only of travelling expenses to
and from their respective district in the case of Members of the House of It is worthy to note that the original salary for the members of the National
Representatives and to and from their places of residence in the case of Assembly (unicameral body) was fixed at P5,000.00 per annum each. This
Senators, when attending sessions of the Congress. No increase in said was raised to P7,200 per annum by the enactment of the 1940
compensation shall take effect until after the expiration of the full term of Constitutional amendment, when the unicameral body, the National
all the Members of the Senate and of the House of Representatives Assembly, was changed to Congress, composed of two bodies, the Senate
approving such increase. Until otherwise provided by law, the President of and the House of Representatives. Again, in 1964, by the enactment of
the Senate and the Speaker of the House of Representatives shall each Republic Act 4143, the salary for the Members of Congress was raised to
receive an annual compensation of sixteen thousand pesos." (Emphasis P32,000.00 per annum for each of them; and for the President of the
supplied) Senate and the Speaker of the House of Representatives, to P40,000.00
per annum each.
Before discussing this point, it is worthy to note that the Constitution
Likewise, it is significant that, as stated above, when the Constitutional stated that "as used in Article 4, section 9, of the Constitution of
Convention first determined the compensation for the Members of Minnesota, providing that no Senator or Representative shall hold any
Congress, the amount fixed by it was only P5,000.00 per annum but it office, the emoluments of which have been increased during the session of
embodies a special proviso which reads as follows: "No increase in said the Legislature of which he was member, until after the expiration of his
compensation shall take effect until after the expiration of the full term of term of office in the Legislature, the word "emoluments" does not refer to
all the members of the National Assembly elected subsequent to approval the fixed salary alone, but includes fees and compensation as the
of such increase." In other words, under the original constitutional incumbent of the office is by law entitled to receive because he holds such
provision regarding the power of the National Assembly to increase the office and performed some service required of the occupant
salaries of its members, no increase would take effect until after the thereof ."cralaw virtua1aw library
expiration of the full term of the members of the Assembly elected
subsequent to the approval of such increase. (See Aruego, The Framing of From the decisions of this cases, it is evident that retirement benefit is a
the Constitution, Vol. 1, pp. 296-300; Sinco, Philippine Government and form or another species of emolument, because it is a part of
Political Law, 4th ed., p. 187) compensation for services of one possessing any office.

This goes to show how, zealous were the members of the Constitutional Republic Act 3836 provides for an increase in the emoluments of Senators
Convention in guarding against the temptation for members of Congress to and Members of the House of Representatives, to take effect upon the
increase their salaries. However, the original strict prohibition was modified approval of said Act, which was on June 22, 1963. Retirement were
by the subsequent provision when the Constitutional amendments were immediately available thereunder, without awaiting the expiration of the
approved in 1940. 2  full term of all the Members of the Senate and the House of
Representatives approving such increase. Such provision clearly runs
The Constitutional provision in the aforementioned Section 14, Article VI, counter to the prohibition in Article VI, Section 14 of the Constitution.
includes in the term compensation "other emoluments." This is the pivotal
point on this fundamental question as to whether the retirement benefit as Third Legal Point — Whether or not the law in question violates the equal
provided for in Republic Act 3836 fall within the purview of the term "other
emoluments."cralaw virtua1aw library protection clause of the Constitution.

Most of the authorities and decided cases have regarded "emolument" as Another reason in support of the conclusion reached herein is that the
"the profit arising from office or employment; that which is received as features of said Republic Act 3836 are patently discriminatory, and
compensation for services or which is annexed to the possession of an therefore violate the equal protection clause of the Constitution. (Art III,
office, as salary, fees and perquisites." 3  Sec. 1, par. 1.)

In another set of cases, "emolument" has been defined as "the profit In the first place, while the said law grants retirement benefits to Senators
arising from office or employment; that which is received as compensation and Members of the House of Representatives who are elective officials, it
for services, or which is annexed to the possession of office, as salary, fees does not include other elective officials such as the governors of provinces
and perquisites; advantage, gain public or private. The gain, profit or and the members of the provincial boards, and the elective officials of the
advantage which is contemplated in the definition or significance of the municipalities and chartered cities.
word "emolument" as applied to public officers, clearly comprehends, We
think, a gain, profit, or advantage which is pecuniary in character. (citing The principles of equal protection of law embodied in our Constitution has
Taxpayers’ League of Cargon County v. McPherson, 54 P. 2d. 897, 901, 49 been fully explained by Us in the case of People v. Vera, 65 Phil. 56, 126,
Wy. 26; 106 A.L.R. 767). where we stated that the classification to be reasonable must be based
upon substantial distinctions which make real differences and must be
In Schieffelin v. Berry, 216 N.Y.S. (citing Wright v. Craig, 202 App. Div. germane to the purposes of the law.
684, 195 N.Y.S. 391, affirmed 234 N.Y. 548, 138 N.E. 441), it has been
established that pensions and retirement allowances are part of As well stated by Willoughby on the Constitution of the United States
compensation of public officials; otherwise their payment would be (second edition) p. 1937, the principle of the requirement of equal
unconstitutional. protection of law applies to all persons similarly situated. Why limit the
application of the benefits of Republic Act 3836 to the elected members of
In another case, State v. Schmahl, 145 N. W. 795, 125 Minn. 104, it is Congress? We feel that the classification here is not reasonable. (See also
Sinco, Philippine Political Law, 11th ed. [1962]; Selected Essays on We are not unmindful of the fact that there has been a general disposition
Constitutional Law [1938-62], p. 789; The Equal Protection of the Laws, 37 in all courts to construe the constitutional provision with reference to the
Cal. Law Rev. 341.) subject and title of the Act, liberally.

Secondly, all members of Congress under Republic Act 3836 are given It is the contention of petitioner that the said title of Republic Act 3836
retirement benefits after serving twelve years, not necessarily continuous, gives no inkling or notice whatsoever to the public regarding the retirement
whereas, most government officers and employees are given retirement gratuities and commutable vacation and sick leave privileges to members
benefits after serving for at least twenty years. In fact, the original bill of of Congress. It is claimed that petitioner learned of this law for the first
Act 3836 provided for twenty years of service. time only when Jose Velasco, disbursing officer of the House, testified on
January 30, 1964, before Justice Labrador, in connection with the hearing
In the third place, all government officers and employees are given only of the case, and he revealed that in 1963, Congress enacted the retirement
one retirement benefits irrespective of their length of service in the law for its members. In fact the Appropriation Act for the fiscal year 1964-
government, whereas, under Republic Act 3836, because of no age 1965, Republic Act No. 4164, provides:jgc:chanrobles.com.ph
limitation, a Senator or Member of the House of Representatives upon
being elected for 24 years will be entitled to two retirement benefits or "13. For payment of retirement gratuities of members of the Senate
equivalent to six years’ salary. pursuant to the provisions of Republic Act No. 3836: PROVIDED, That no
portion of this Appropriation shall be transferred to any other item until
Also, while the payment of retirement benefits (annuity) to an employee approved claims shall have been paid — P210,000.000.
who had been retired and reappointed is suspended during his new
employment (under Commonwealth Act 186, as amended), this is not so In the appropriation for the House of Representatives, the following items
under Republic Act 3836. appear:jgc:chanrobles.com.ph

Lastly, it is peculiar that Republic Act 3836 grants retirement benefits to "7. For government share of premiums on life insurance and retirement of
officials who are not members of the Government Service Insurance Members and employees of the House of Representatives, as provided for
System. Lost grantees of retirement benefits under the various retirement under Republic Act No. 1616 — P1,300,000.00
laws have to be members or must at least contribute a portion of their
monthly salaries to the system. 4  "8. For payment of the cash commutation of the accumulated vacation and
sick leaves as provided for under Republic Act. No. 611, and retirement
The arguments advanced against the discriminatory features of Republic gratuities of Members and employees of the House of Representatives
Act 3836, as far as Members of Congress are concerned, apply with equal under Republic Act No. 1616 — P1,300,000.00."cralaw virtua1aw library
force to the elected officers of each House, such as the Secretaries and the
Sergeants-at-arms. Under Republic Act 3836, the Secretary and Sergeants- In the Appropriations Act of 1965 1966 (Republic Act No. 4642), the
at-arms of each House are given the benefits of retirement without having following item appears in the appropriations for the
served for twenty years as required with other officers and employees of Senate:jgc:chanrobles.com.ph
the Government.
"13. For payment of retirement gratuities of Senate personnel pursuant to
Fourth Legal Point — Whether or not the title of Republic Act No. 3836 is the provisions of Republic Act No. 1616: PROVIDED. That no portion of this
appropriation shall be transferred to any other item until all approved
germane to the subject matter expressed in the act. claims shall have been paid — P100,000.00."cralaw virtua1aw library

Another Constitutional point to determine is whether the title of Republic It is thus clear that in the Appropriations Act for 1965-1966, the item in the
Act 3836 complies with the requirement of paragraph 1 section 21, Article Senate for P210,000.00 to implement Republic Act 3836, was eliminated.
VI of the Constitution, which reads as follows:jgc:chanrobles.com.ph
In the appropriations for the House (1965-1966) the following items
"No bill which may be enacted into law shall embrace more than one appear:jgc:chanrobles.com.ph
subject which shall be expressed in the title of the bill."cralaw virtua1aw
library "7. For government share of premiums on life insurance and retirement of
members and employees of the House of Representatives, as provided for
under Republic Act No. 1616 — P1,200,000.00 compliance with such requirement if the title expresses the general subject
and all the provisions of the statute are germane to that general subject."
"8. For payment of the cash commutation of the accumulated vacation and (Sumulong v. The Commission on Elections, 73 Phil. 288, 291)
sick leaves as provided for under Republic Act No. 611, and retirement
gratuities of Members and employees of the House of Representatives The requirement that the subject of an act shall be expressed in its title is
under Republic Act No. 1616 — P1,700,000.00. wholly illustrated and explained in Central Capiz v. Ramirez, 40 Phil. 883.
In this case, the question raised was whether Commonwealth Act 2784,
It is to be observed that under Republic Act 3836, amending the first known as the Public Land Act, was limited in its application to lands of the
paragraph of section 12, subsection (c) of Commonwealth Act 186, as public domain or whether its provisions also extended to agricultural lands
amended by Republic Act Nos. 660 and 3096, the retirement benefits are held in private ownership. The Court held that the act was limited to lands
granted to members of the Government Service Insurance System, who of the public domain as indicated in its title, and did not include private
have rendered at least twenty years of service regardless of age. This agricultural lands. The Court further stated that this provision of the
paragraph is related and germane to the subject of Commonwealth Act No. Constitution expressing the subject matter of an Act in its title, is not a
186. mere rule of legislative procedure, directory to Congress, but it is
mandatory. It is the duty of the Court to declare void any statute not
On the other hand, the succeeding paragraph of Republic Act 3836 refers conforming to this constitutional provision. (See Walker v. State, 49
to members of Congress and to elective officers thereof who are not Alabama 329; Cooley, Constitutional Limitations, pp. 162-164 5; See also
members of the Government Service Insurance System. To provide Agcaoili v. Suguitan, 48 Phil. 676; Sutherland on Statutory Construction,
retirement benefits, therefore, for these officials, would relate to subject Sec. 111.)
matter which is not germane to Commonwealth Act No. 186. In other
words, this portion of the amendment (re retirement benefits for Members In the light of the history and analysis of Republic Act 3836, We conclude
of Congress and elected officers, such as the Secretary and Sergeant-at- that the title of said Republic Act 3836 is void as it is not germane to the
arms for each House) is not related in any manner to the subject of subject matter and is a violation of the aforementioned paragraph 1,
Commonwealth Act 186 establishing the Government Service Insurance section 21, Article VI of the Constitution.
System and which provides for both retirement and insurance benefits to
its members. In short, Republic Act 3836 violates three constitutional provisions,
namely: first, the pr prohibition regarding increase in the salaries of
Parenthetically, it may be added that the purpose of the requirement that Members of Congress; second, the equal protection clause; and third, the
the subject of an act should be expressed in its title is fully explained by prohibition that the title of a bill shall not embrace more than one subject.
Cooley, thus: (1) to prevent surprise or fraud upon the Legislature; and (2)
to fairly apprise the people, through such publication of legislation that are IN VIEW OF THE FOREGOING CONSIDERATIONS, Republic Act No. 3836 is
being considered, in order that they may have the opportunity of being hereby declared null and void, in so far as it refers to the retirement of
heard thereon by petition or otherwise, if they shall so desire. (Cooley, Members of Congress and the elected officials thereof, as being-
Constitutional Limitations, 8th ed., Vol. I, p. 162; See also Martin, Political unconstitutional. The restraining order issued in our resolution on
Law Reviewer, Book One [1965] p. 119) December 6, 1965 is hereby made permanent. No costs.

With respect to sufficiency of title this Court has ruled in two


cases:jgc:chanrobles.com.ph

"The Constitutional requirements with respect to titles of statutes as


sufficient to reflect their contents is satisfied if all parts of a law relate to
the subject expressed in its title, and it is not necessary that the title be a
complete index of the content." (People v. Carlos, 78 Phil. 535)

"The Constitutional requirement that the subject of an act shall be


expressed in its title should be reasonably construed so as not to interfere
unduly with the enactment of necessary legislation. It should be given a
practical, rather than technical, construction. It should be a sufficient
unregulated proliferation of film piracy." The Intervenors were thereafter
allowed to file their Comment in
Intervention.chanroblesvirtualawlibrary chanrobles virtual law library

The rationale behind the enactment of the DECREE, is set out in its
G.R. No. L-75697 June 18, 1987
preambular clauses as follows: 

VALENTIN TIO doing business under the name and style of OMI
1. WHEREAS, the proliferation and unregulated circulation of videograms
ENTERPRISES, Petitioner, vs. VIDEOGRAM REGULATORY BOARD,
including, among others, videotapes, discs, cassettes or any technical
MINISTER OF FINANCE, METRO MANILA COMMISSION, CITY
improvement or variation thereof, have greatly prejudiced the operations
MAYOR and CITY TREASURER OF MANILA, Respondents.
of moviehouses and theaters, and have caused a sharp decline in theatrical
attendance by at least forty percent (40%) and a tremendous drop in the
Nelson Y. Ng for petitioner.chanrobles virtual law library collection of sales, contractor's specific, amusement and other taxes,
thereby resulting in substantial losses estimated at P450 Million annually in
The City Legal Officer for respondents City Mayor and City Treasurer.  government revenues; chanrobles virtual law library

MELENCIO-HERRERA, J.: 2. WHEREAS, videogram(s) establishments collectively earn around P600


Million per annum from rentals, sales and disposition of videograms, and
such earnings have not been subjected to tax, thereby depriving the
This petition was filed on September 1, 1986 by petitioner on his own Government of approximately P180 Million in taxes each year; chanrobles
behalf and purportedly on behalf of other videogram operators adversely virtual law library
affected. It assails the constitutionality of Presidential Decree No. 1987
entitled "An Act Creating the Videogram Regulatory Board" with broad
powers to regulate and supervise the videogram industry (hereinafter 3. WHEREAS, the unregulated activities of videogram establishments have
briefly referred to as the BOARD). The Decree was promulgated on October also affected the viability of the movie industry, particularly the more than
5, 1985 and took effect on April 10, 1986, fifteen (15) days after 1,200 movie houses and theaters throughout the country, and occasioned
completion of its publication in the Official industry-wide displacement and unemployment due to the shutdown of
Gazette.chanroblesvirtualawlibrary chanrobles virtual law library numerous moviehouses and theaters; chanrobles virtual law library

On November 5, 1985, a month after the promulgation of the 4. "WHEREAS, in order to ensure national economic recovery, it is
abovementioned decree, Presidential Decree No. 1994 amended the imperative for the Government to create an environment conducive to
National Internal Revenue Code providing, inter alia:  growth and development of all business industries, including the movie
industry which has an accumulated investment of about P3
Billion; chanrobles virtual law library
SEC. 134. Video Tapes. - There shall be collected on each processed video-
tape cassette, ready for playback, regardless of length, an annual tax of
five pesos; Provided, That locally manufactured or imported blank video 5. WHEREAS, proper taxation of the activities of videogram establishments
tapes shall be subject to sales tax. will not only alleviate the dire financial condition of the movie industry
upon which more than 75,000 families and 500,000 workers depend for
their livelihood, but also provide an additional source of revenue for the
On October 23, 1986, the Greater Manila Theaters Association, Integrated Government, and at the same time rationalize the heretofore uncontrolled
Movie Producers, Importers and Distributors Association of the Philippines, distribution of videograms; chanrobles virtual law library
and Philippine Motion Pictures Producers Association, hereinafter
collectively referred to as the Intervenors, were permitted by the Court to
intervene in the case, over petitioner's opposition, upon the allegations 6. WHEREAS, the rampant and unregulated showing of obscene videogram
that intervention was necessary for the complete protection of their rights features constitutes a clear and present danger to the moral and spiritual
and that their "survival and very existence is threatened by the well-being of the youth, and impairs the mandate of the Constitution for
the State to support the rearing of the youth for civic efficiency and the express each and every end that the statute wishes to accomplish. The
development of moral character and promote their physical, intellectual, requirement is satisfied if all the parts of the statute are related, and are
and social well-being; chanrobles virtual law library germane to the subject matter expressed in the title, or as long as they are
not inconsistent with or foreign to the general subject and title. 2 An act
having a single general subject, indicated in the title, may contain any
7. WHEREAS, civic-minded citizens and groups have called for remedial
number of provisions, no matter how diverse they may be, so long as they
measures to curb these blatant malpractices which have flaunted our
are not inconsistent with or foreign to the general subject, and may be
censorship and copyright laws; chanrobles virtual law library
considered in furtherance of such subject by providing for the method and
means of carrying out the general object." 3 The rule also is that the
8. WHEREAS, in the face of these grave emergencies corroding the moral constitutional requirement as to the title of a bill should not be so narrowly
values of the people and betraying the national economic recovery construed as to cripple or impede the power of legislation. 4 It should be
program, bold emergency measures must be adopted with dispatch; ... given practical rather than technical construction. 5chanrobles virtual law
(Numbering of paragraphs supplied).  library

Petitioner's attack on the constitutionality of the DECREE rests on the Tested by the foregoing criteria, petitioner's contention that the tax
following grounds:  provision of the DECREE is a rider is without merit. That section
reads, inter alia: 
1. Section 10 thereof, which imposes a tax of 30% on the gross receipts
payable to the local government is a RIDER and the same is not germane Section 10. Tax on Sale, Lease or Disposition of Videograms. -
to the subject matter thereof; chanrobles virtual law library Notwithstanding any provision of law to the contrary, the province shall
collect a tax of thirty percent (30%) of the purchase price or rental rate, as
2. The tax imposed is harsh, confiscatory, oppressive and/or in unlawful the case may be, for every sale, lease or disposition of a videogram
restraint of trade in violation of the due process clause of the containing a reproduction of any motion picture or audiovisual program.
Constitution; chanrobles virtual law library Fifty percent (50%) of the proceeds of the tax collected shall accrue to the
province, and the other fifty percent (50%) shall acrrue to the municipality
where the tax is collected; PROVIDED, That in Metropolitan Manila, the tax
3. There is no factual nor legal basis for the exercise by the President of shall be shared equally by the City/Municipality and the Metropolitan Manila
the vast powers conferred upon him by Amendment No. 6; chanrobles Commission.chanroblesvirtualawlibrary chanrobles virtual law library
virtual law library

xxx xxx xxx


4. There is undue delegation of power and authority; chanrobles virtual law
library
The foregoing provision is allied and germane to, and is reasonably
necessary for the accomplishment of, the general object of the DECREE,
5. The Decree is an ex-post facto law; and chanrobles virtual law library which is the regulation of the video industry through the Videogram
Regulatory Board as expressed in its title. The tax provision is not
6. There is over regulation of the video industry as if it were a nuisance, inconsistent with, nor foreign to that general subject and title. As a tool for
which it is not.  regulation 6 it is simply one of the regulatory and control mechanisms
scattered throughout the DECREE. The express purpose of the DECREE to
include taxation of the video industry in order to regulate and rationalize
We shall consider the foregoing objections the heretofore uncontrolled distribution of videograms is evident from
in seriatim.chanroblesvirtualawlibrary chanrobles virtual law library Preambles 2 and 5, supra. Those preambles explain the motives of the
lawmaker in presenting the measure. The title of the DECREE, which is the
1. The Constitutional requirement that "every bill shall embrace only one creation of the Videogram Regulatory Board, is comprehensive enough to
subject which shall be expressed in the title thereof" 1 is sufficiently include the purposes expressed in its Preamble and reasonably covers all
complied with if the title be comprehensive enough to include the general its provisions. It is unnecessary to express all those objectives in the title
purpose which a statute seeks to achieve. It is not necessary that the title
or that the latter be an index to the body of the DECREE. 7 chanrobles 3. Petitioner argues that there was no legal nor factual basis for the
virtual law library promulgation of the DECREE by the former President under Amendment
No. 6 of the 1973 Constitution providing that "whenever in the judgment of
the President ... , there exists a grave emergency or a threat or imminence
2. Petitioner also submits that the thirty percent (30%) tax imposed is
thereof, or whenever the interim Batasang Pambansa or the regular
harsh and oppressive, confiscatory, and in restraint of trade. However, it is
National Assembly fails or is unable to act adequately on any matter for
beyond serious question that a tax does not cease to be valid merely
any reason that in his judgment requires immediate action, he may, in
because it regulates, discourages, or even definitely deters the activities
order to meet the exigency, issue the necessary decrees, orders, or letters
taxed. 8The power to impose taxes is one so unlimited in force and so
of instructions, which shall form part of the law of the land." chanrobles
searching in extent, that the courts scarcely venture to declare that it is
virtual law library
subject to any restrictions whatever, except such as rest in the discretion
of the authority which exercises it. 9 In imposing a tax, the legislature acts
upon its constituents. This is, in general, a sufficient security against In refutation, the Intervenors and the Solicitor General's Office aver that
erroneous and oppressive taxation. 10 chanrobles virtual law library the 8th "whereas" clause sufficiently summarizes the justification in that
grave emergencies corroding the moral values of the people and betraying
the national economic recovery program necessitated bold emergency
The tax imposed by the DECREE is not only a regulatory but also a revenue
measures to be adopted with dispatch. Whatever the reasons "in the
measure prompted by the realization that earnings of videogram
judgment" of the then President, considering that the issue of the validity
establishments of around P600 million per annum have not been subjected
of the exercise of legislative power under the said Amendment still pends
to tax, thereby depriving the Government of an additional source of
resolution in several other cases, we reserve resolution of the question
revenue. It is an end-user tax, imposed on retailers for every videogram
raised at the proper time.chanroblesvirtualawlibrary chanrobles virtual law
they make available for public viewing. It is similar to the 30% amusement
library
tax imposed or borne by the movie industry which the theater-owners pay
to the government, but which is passed on to the entire cost of the
admission ticket, thus shifting the tax burden on the buying or the viewing 4. Neither can it be successfully argued that the DECREE contains an undue
public. It is a tax that is imposed uniformly on all videogram delegation of legislative power. The grant in Section 11 of the DECREE of
operators.chanroblesvirtualawlibrary chanrobles virtual law library authority to the BOARD to "solicit the direct assistance of other agencies
and units of the government and deputize, for a fixed and limited period,
the heads or personnel of such agencies and units to perform enforcement
The levy of the 30% tax is for a public purpose. It was imposed primarily to
functions for the Board" is not a delegation of the power to legislate but
answer the need for regulating the video industry, particularly because of
merely a conferment of authority or discretion as to its execution,
the rampant film piracy, the flagrant violation of intellectual property
enforcement, and implementation. "The true distinction is between the
rights, and the proliferation of pornographic video tapes. And while it was
delegation of power to make the law, which necessarily involves a
also an objective of the DECREE to protect the movie industry, the tax
discretion as to what it shall be, and conferring authority or discretion as to
remains a valid imposition. 
its execution to be exercised under and in pursuance of the law. The first
cannot be done; to the latter, no valid objection can be made." 14 Besides,
The public purpose of a tax may legally exist even if the motive which in the very language of the decree, the authority of the BOARD to solicit
impelled the legislature to impose the tax was to favor one industry over such assistance is for a "fixed and limited period" with the deputized
another. 11 chanrobles virtual law library agencies concerned being "subject to the direction and control of the
BOARD." That the grant of such authority might be the source of graft and
It is inherent in the power to tax that a state be free to select the subjects corruption would not stigmatize the DECREE as unconstitutional. Should
of taxation, and it has been repeatedly held that "inequities which result the eventuality occur, the aggrieved parties will not be without adequate
from a singling out of one particular class for taxation or exemption remedy in law.chanroblesvirtualawlibrary chanrobles virtual law library
infringe no constitutional limitation". 12Taxation has been made the
implement of the state's police power. 13 chanrobles virtual law library 5. The DECREE is not violative of the ex post facto principle. An ex post
facto law is, among other categories, one which "alters the legal rules of
At bottom, the rate of tax is a matter better addressed to the taxing evidence, and authorizes conviction upon less or different testimony than
legislature.chanroblesvirtualawlibrary chanrobles virtual law library
the law required at the time of the commission of the offense." It is 6. We do not share petitioner's fears that the video industry is being over-
petitioner's position that Section 15 of the DECREE in providing that:  regulated and being eased out of existence as if it were a nuisance. Being a
relatively new industry, the need for its regulation was apparent. While the
underlying objective of the DECREE is to protect the moribund movie
All videogram establishments in the Philippines are hereby given a period
industry, there is no question that public welfare is at bottom of its
of forty-five (45) days after the effectivity of this Decree within which to
enactment, considering "the unfair competition posed by rampant film
register with and secure a permit from the BOARD to engage in the
piracy; the erosion of the moral fiber of the viewing public brought about
videogram business and to register with the BOARD all their inventories of
by the availability of unclassified and unreviewed video tapes containing
videograms, including videotapes, discs, cassettes or other technical
pornographic films and films with brutally violent sequences; and losses in
improvements or variations thereof, before they could be sold, leased, or
government revenues due to the drop in theatrical attendance, not to
otherwise disposed of. Thereafter any videogram found in the possession of
mention the fact that the activities of video establishments are virtually
any person engaged in the videogram business without the required proof
untaxed since mere payment of Mayor's permit and municipal license fees
of registration by the BOARD, shall be prima facie evidence of violation of
are required to engage in business. 17 
the Decree, whether the possession of such videogram be for private
showing and/or public exhibition. 
The enactment of the Decree since April 10, 1986 has not brought about
the "demise" of the video industry. On the contrary, video establishments
raises immediately a prima facie evidence of violation of the DECREE when
are seen to have proliferated in many places notwithstanding the 30% tax
the required proof of registration of any videogram cannot be presented
imposed.chanroblesvirtualawlibrary chanrobles virtual law library
and thus partakes of the nature of an ex post
facto law.chanroblesvirtualawlibrary chanrobles virtual law library
In the last analysis, what petitioner basically questions is the necessity,
wisdom and expediency of the DECREE. These considerations, however,
The argument is untenable. As this Court held in the recent case of Vallarta
are primarily and exclusively a matter of legislative concern. 
vs. Court of Appeals, et al. 15

Only congressional power or competence, not the wisdom of the action


... it is now well settled that "there is no constitutional objection to the
taken, may be the basis for declaring a statute invalid. This is as it ought to
passage of a law providing that the presumption of innocence may be
be. The principle of separation of powers has in the main wisely allocated
overcome by a contrary presumption founded upon the experience of
the respective authority of each department and confined its jurisdiction to
human conduct, and enacting what evidence shall be sufficient to
such a sphere. There would then be intrusion not allowable under the
overcome such presumption of innocence" (People vs. Mingoa 92 Phil. 856
Constitution if on a matter left to the discretion of a coordinate branch, the
[1953] at 858-59, citing 1 COOLEY, A TREATISE ON THE CONSTITUTIONAL
judiciary would substitute its own. If there be adherence to the rule of law,
LIMITATIONS, 639-641). And the "legislature may enact that when certain
as there ought to be, the last offender should be courts of justice, to which
facts have been proved that they shall be prima facie evidence of the
rightly litigants submit their controversy precisely to maintain unimpaired
existence of the guilt of the accused and shift the burden of proof provided
the supremacy of legal norms and prescriptions. The attack on the validity
there be a rational connection between the facts proved and the ultimate
of the challenged provision likewise insofar as there may be objections,
facts presumed so that the inference of the one from proof of the others is
even if valid and cogent on its wisdom cannot be sustained. 18
not unreasonable and arbitrary because of lack of connection between the
two in common experience". 16
In fine, petitioner has not overcome the presumption of validity which
attaches to a challenged statute. We find no clear violation of the
Applied to the challenged provision, there is no question that there is a
Constitution which would justify us in pronouncing Presidential Decree No.
rational connection between the fact proved, which is non-registration, and
1987 as unconstitutional and void.chanroblesvirtualawlibrary chanrobles
the ultimate fact presumed which is violation of the DECREE, besides the
virtual law library
fact that the prima facie presumption of violation of the DECREE attaches
only after a forty-five-day period counted from its effectivity and is,
therefore, neither retrospective in WHEREFORE, the instant Petition is hereby
character.chanroblesvirtualawlibrary chanrobles virtual law library dismissed.chanroblesvirtualawlibrary chanrobles virtual law library
No costs.chanroblesvirtualawlibrary chanrobles virtual law library The petitioners are members of the lower courts who feel that their official
functions as judges will be prejudiced by the above-named measures. The
National Land Registration Authority has taken common cause with them
SO ORDERED.
insofar as its own activities, such as sending of requisite notices in
registration cases, affect judicial proceedings. On its motion, it has been
THE PHILIPPINE JUDGES ASSOCIATION, duly rep. by its President, allowed to intervene.chanroblesvirtualawlibrarychanrobles virtual law
BERNARDO P. ABESAMIS, Vice-President for Legal Affairs, library
MARIANO M. UMALI, Director for Pasig, Makati, and Pasay, Metro
Manila, ALFREDO C. FLORES, and Chairman of the Committee on
The petition assails the constitutionality of R.A. No. 7354 on the grounds
Legal Aid, JESUS G. BERSAMIRA, Presiding Judges of the Regional
that: (1) its title embraces more than one subject and does not express its
Trial Court, Branch 85, Quezon City and Branches 160, 167 and
purposes; (2) it did not pass the required readings in both Houses of
166, Pasig, Metro Manila, respectively: the NATIONAL
Congress and printed copies of the bill in its final form were not distributed
CONFEDERATION OF THE JUDGES ASSOCIATION OF THE
among the members before its passage; and (3) it is discriminatory and
PHILIPPINES, composed of the METROPOLITAN TRIAL COURT
encroaches on the independence of the
JUDGES ASSOCIATION rep. by its President. REINATO QUILALA of
Judiciary.chanroblesvirtualawlibrarychanrobles virtual law library
the MUNICIPAL TRIAL CIRCUIT COURT, Manila; THE MUNICIPAL
JUDGES LEAGUE OF THE PHILIPPINES rep. by its President, TOMAS
G. TALAVERA; by themselves and in behalf of all the Judges of the We approach these issues with one important principle in mind, to wit, the
Regional Trial and Shari'a Courts, Metropolitan Trial Courts and presumption of the constitutionality of statutes. The theory is that as the
Municipal Courts throughout the Country, Petitioners, vs. HON. PETE joint act of the Legislature and the Executive, every statute is supposed to
PRADO, in his capacity as Secretary of the Department of have first been carefully studied and determined to be constitutional before
Transportation and Communications, JORGE V. SARMIENTO, in his it was finally enacted. Hence, unless it is clearly shown that it is
capacity as Postmaster General, and the PHILIPPINE POSTAL constitutionally flawed, the attack against its validity must be rejected and
CORP., Respondents.chanrobles virtual law library the law itself upheld. To doubt is to sustain.

CRUZ, J.: Ichanrobles virtual law library

The basic issue raised in this petition is the independence of the Judiciary. We consider first the objection based on Article VI, Sec. 26(l), of the
It is asserted by the petitioners that this hallmark of republicanism is Constitution providing that "Every bill passed by the Congress shall
impaired by the statute and circular they are here challenging. The embrace only one subject which shall be expressed in the title
Supreme Court is itself affected by these measures and is thus an thereof." chanrobles virtual law library
interested party that should ordinarily not also be a judge at the same
time. Under our system of government, however, it cannot inhibit itself and
The purposes of this rule are: (1) to prevent hodge-podge or "log-rolling"
must rule upon the challenge, because no other office has the authority to
legislation; (2) to prevent surprise or fraud upon the legislature by means
do so. We shall therefore act upon this matter not with officiousness but in
of provisions in bills of which the title gives no intimation, and which might
the discharge of an unavoidable duty and, as always, with detachment and
therefore be overlooked and carelessly and unintentionally adopted; and
fairness.chanroblesvirtualawlibrarychanrobles virtual law library
(3) to fairly apprise the people, through such publication of legislative
proceedings as is usually made, of the subject of legislation that is being
The main target of this petition is Section 35 of R.A. No. 7354 as considered, in order that they may have opportunity of being heard
implemented by the Philippine Postal Corporation through its Circular No. thereon, by petition or otherwise, if they shall so desire. 1chanrobles virtual
92-28. These measures withdraw the franking privilege from the Supreme law library
Court, the Court of Appeals, the Regional Trial Courts, the Metropolitan
Trial Courts, the Municipal Trial Courts, and the Land Registration
It is the submission of the petitioners that Section 35 of R.A. No. 7354
Commission and its Registers of Deeds, along with certain other
which withdrew the franking privilege from the Judiciary is not expressed in
government offices.chanroblesvirtualawlibrarychanrobles virtual law library
the title of the law, nor does it reflect its
purposes.chanroblesvirtualawlibrarychanrobles virtual law library
R.A. No. 7354 is entitled "An Act Creating the Philippine Postal Corporation, The petitioners' contention is untenable. We do not agree that the title of
Defining its Powers, Functions and Responsibilities, Providing for Regulation the challenged act violates the
of the Industry and for Other Purposes Connected Therewith."chanrobles Constitution.chanroblesvirtualawlibrarychanrobles virtual law library
virtual law library
The title of the bill is not required to be an index to the body of the act, or
The objectives of the law are enumerated in Section 3, which provides: to be as comprehensive as to cover every single detail of the measure. It
has been held that if the title fairly indicates the general subject, and
reasonably covers all the provisions of the act, and is not calculated to
The State shall pursue the following objectives of a nationwide postal
mislead the legislature or the people, there is sufficient compliance with the
system:chanrobles virtual law library
constitutional requirement. 2chanrobles virtual law library

a) to enable the economical and speedy transfer of mail and other postal
To require every end and means necessary for the accomplishment of the
matters, from sender to addressee, with full recognition of their privacy or
general objectives of the statute to be expressed in its title would not only
confidentiality;chanrobles virtual law library
be unreasonable but would actually render legislation impossible. 3As has
been correctly explained:
b) to promote international interchange, cooperation and understanding
through the unhampered flow or exchange of postal matters between
The details of a legislative act need not be specifically stated in its title, but
nations;chanrobles virtual law library
matter germane to the subject as expressed in the title, and adopted to the
accomplishment of the object in view, may properly be included in the act.
c) to cause or effect a wide range of postal services to cater to different Thus, it is proper to create in the same act the machinery by which the act
users and changing needs, including but not limited to, philately, transfer is to be enforced, to prescribe the penalties for its infraction, and to
of monies and valuables, and the like;chanrobles virtual law library remove obstacles in the way of its execution. If such matters are properly
connected with the subject as expressed in the title, it is unnecessary that
d) to ensure that sufficient revenues are generated by and within the they should also have special mention in the title (Southern Pac. Co. v.
industry to finance the overall cost of providing the varied range of postal Bartine, 170 Fed. 725).
delivery and messengerial services as well as the expansion and continuous
upgrading of service standards by the same. This is particularly true of the repealing clause, on which Cooley writes:
"The repeal of a statute on a given subject is properly connected with the
Sec. 35 of R.A. No. 7354, which is the principal target of the petition, reads subject matter of a new statute on the same subject; and therefore a
as follows: repealing section in the new statute is valid, notwithstanding that the title
is silent on the subject. It would be difficult to conceive of a matter more
germane to an act and to the object to be accomplished thereby than the
Sec. 35. Repealing Clause. - All acts, decrees, orders, executive orders, repeal of previous legislations connected therewith." 4chanrobles virtual law
instructions, rules and regulations or parts thereof inconsistent with the library
provisions of this Act are repealed or modified
accordingly.chanroblesvirtualawlibrarychanrobles virtual law library
The reason is that where a statute repeals a former law, such repeal is the
effect and not the subject of the statute; and it is the subject, not the
All franking privileges authorized by law are hereby repealed, except those effect of a law, which is required to be briefly expressed in its title. 5As
provided for under Commonwealth Act No. 265, Republic Acts Numbered observed in one case, 6if the title of an act embraces only one subject, we
69, 180, 1414, 2087 and 5059. The Corporation may continue the franking apprehend it was never claimed that every other act which repeals it or
privilege under Circular No. 35 dated October 24, 1977 and that of the Vice alters by implication must be mentioned in the title of the new act. Any
President, under such arrangements and conditions as may obviate abuse such rule would be neither within the reason of the Constitution, nor
or unauthorized use thereof. practicable.chanroblesvirtualawlibrarychanrobles virtual law library
We are convinced that the withdrawal of the franking privilege from some subject matter can be inserted into the conference bill. But occasionally a
agencies is germane to the accomplishment of the principal objective of conference committee produces unexpected results, results beyond its
R.A. No. 7354, which is the creation of a more efficient and effective postal mandate, These excursions occur even where the rules impose strict
service system. Our ruling is that, by virtue of its nature as a repealing limitations on conference committee jurisdiction. This is symptomatic of the
clause, Section 35 did not have to be expressly included in the title of the authoritarian power of conference committee (Davies, Legislative Law and
said law. Process: In a Nutshell, 1986 Ed., p.81).

IIchanrobles virtual law library It is a matter of record that the conference Committee Report on the bill in
question was returned to and duly approved by both the Senate and the
House of Representatives. Thereafter, the bill was enrolled with its
The petitioners maintain that the second paragraph of Sec. 35 covering the
certification by Senate President Neptali A. Gonzales and Speaker Ramon
repeal of the franking privilege from the petitioners and this Court under
V. Mitra of the House of Representatives as having been duly passed by
E.O. 207, PD 1882 and PD 26 was not included in the original version of
both Houses of Congress. It was then presented to and approved by
Senate Bill No. 720 or House Bill No. 4200. As this paragraph appeared
President Corazon C. Aquino on April 3,
only in the Conference Committee Report, its addition, violates Article VI,
1992.chanroblesvirtualawlibrarychanrobles virtual law library
Sec. 26(2) of the Constitution, reading as follows:

Under the doctrine of separation powers, the Court may not inquire beyond
(2) No bill passed by either House shall become a law unless it has passed
the certification of the approval of a bill from the presiding officers of
three readings on separate days, and printed copies thereof in its final form
Congress. Casco Philippine Chemical Co. v. Gimenez 7 laid down the rule
have been distributed to its Members three days before its passage, except
that the enrolled bill, is conclusive upon the Judiciary (except in matters
when the President certifies to the necessity of its immediate enactment to
that have to be entered in the journals like the yeas and nays on the final
meet a public calamity or emergency. Upon the last reading of a bill, no
reading of the
amendment thereto shall be allowed, and the vote thereon shall be taken
bill). 8The journals are themselves also binding on the Supreme Court, as
immediately thereafter, and the yeas andnays entered in the Journal.
we held in the old (but still valid) case of U.S. vs. Pons, 9where we
explained the reason thus:
The petitioners also invoke Sec. 74 of the Rules of the House of
Representatives, requiring that amendment to any bill when the House and
To inquire into the veracity of the journals of the Philippine legislature
the Senate shall have differences thereon may be settled by a conference
when they are, as we have said, clear and explicit, would be to violate both
committee of both chambers. They stress that Sec. 35 was never a subject
the, letter and spirit of the organic laws by which the Philippine
of any disagreement between both Houses and so the second paragraph
Government was brought into existence, to invade a coordinate and
could not have been validly added as an
independent department of the Government, and to interfere with the
amendment.chanroblesvirtualawlibrarychanrobles virtual law library
legitimate powers and functions, of the Legislature.

These argument are unacceptable.chanroblesvirtualawlibrarychanrobles


Applying these principles, we shall decline to look into the petitioners'
virtual law library
charges that an amendment was made upon the last reading of the bill that
eventually became R.A. No. 7354 and that copies thereof in its final form
While it is true that a conference committee is the mechanism for were not distributed among the members of each House. Both the enrolled
compromising differences between the Senate and the House, it is not bill and the legislative journals certify that the measure was duly
limited in its jurisdiction to this question. Its broader function is described enacted i.e., in accordance with Article VI, Sec. 26(2) of the Constitution.
thus: We are bound by such official assurances from a coordinate department of
the government, to which we owe, at the very least, a becoming courtesy.
A conference committee may, deal generally with the subject matter or it
may be limited to resolving the precise differences between the two IIIchanrobles virtual law library
houses. Even where the conference committee is not by rule limited in its
jurisdiction, legislative custom severely limits the freedom with which new
The third and most serious challenge of the petitioners is based on the the morals of the youth but violate the liberty of adults. What the clause
equal protection clause.chanroblesvirtualawlibrary chanrobles virtual law requires is equality among equals as determined according to a valid
library classification. By classification is meant the grouping of persons or things
similar to each other in certain particulars and different from all others in
these same particulars. 13chanrobles virtual law library
It is alleged that R.A. No. 7354 is discriminatory because while withdrawing
the franking privilege from the Judiciary, it retains the same for the
President of the Philippines, the Vice President of the Philippines; Senators What is the reason for the grant of the franking privilege in the first place?
and Members of the House of Representatives, the Commission on Is the franking privilege extended to the President of the Philippines or the
Elections; former Presidents of the Philippines; the National Census and Commission on Elections or to former Presidents of the Philippines purely
Statistics Office; and the general public in the filing of complaints against as a courtesyfrom the lawmaking body? Is it offered because of
public offices and officers. 10chanrobles virtual law library the importance or status of the grantee or because of its need for the
privilege? Or have the grantees been chosen pell-mell, as it were, without
any basis at all for the selection?chanrobles virtual law library
The respondents counter that there is no discrimination because the law is
based on a valid classification in accordance with the equal protection
clause. In fact, the franking privilege has been withdrawn not only from the We reject outright the last conjecture as there is no doubt that the statute
Judiciary but also the Office of Adult Education, the Institute of National as a whole was carefully deliberated upon, by the political departments
Language; the Telecommunications Office; the Philippine Deposit Insurance before it was finally enacted. There is reason to suspect, however, that not
Corporation; the National Historical Commission; the Armed Forces of the enough care or attention was given to its repealing clause, resulting in the
Philippines; the Armed Forces of the Philippines Ladies Steering unwitting withdrawal of the franking privilege from the
Committee; the City and Provincial Prosecutors; the Tanodbayan (Office of Judiciary.chanroblesvirtualawlibrary chanrobles virtual law library
Special Prosecutor); the Kabataang Barangay; the Commission on the
Filipino Language; the Provincial and City Assessors; and the National
We also do not believe that the basis of the classification was mere
Council for the Welfare of Disabled Persons. 11chanrobles virtual law library
courtesy, for it is unimaginable that the political departments would have
intended this serious slight to the Judiciary as the third of the major and
The equal protection of the laws is embraced in the concept of due process, equal departments the government. The same observations are made if the
as every unfair discrimination offends the requirements of justice and fair importance or status of the grantee was the criterion used for the
play. It has nonetheless been embodied in a separate clause in Article III extension of the franking privilege, which is enjoyed by the National
Sec. 1., of the Constitution to provide for a more, specific guaranty against Census and Statistics Office and even some private individuals but not the
any form of undue favoritism or hostility from the government. courts of justice.chanroblesvirtualawlibrarychanrobles virtual law library
Arbitrariness in general may be challenged on the basis of the due process
clause. But if the particular act assailed partakes of an unwarranted
In our view, the only acceptable reason for the grant of the franking
partiality or prejudice, the sharper weapon to cut it down is the equal
privilege was the perceived need of the grantee for the accommodation,
protection clause.chanroblesvirtualawlibrarychanrobles virtual law library
which would justify a waiver of substantial revenue by the Corporation in
the interest of providing for a smoother flow of communication between the
According to a long line of decisions, equal protection simply requires that government and the people.chanroblesvirtualawlibrarychanrobles virtual
all persons or things similarly situated should be treated alike, both as to law library
rights conferred and responsibilities imposed, 12Similar subjects, in other
words, should not be treated differently, so as to give undue favor to some
Assuming that basis, we cannot understand why, of all the departments of
and unjustly discriminate against
the government, it is the Judiciary, that has been denied the franking
others.chanroblesvirtualawlibrarychanrobles virtual law library
privilege. There is no question that if there is any major branch of the
government that needs the privilege, it is the Judicial Department, as the
The equal protection clause does not require the universal application of respondents themselves point out. Curiously, the respondents would justify
the laws on all persons or things without distinction. This might in fact the distinction on the basis precisely of this need and, on this basis, deny
sometimes result in unequal protection, as where, for example, a law the Judiciary the franking privilege while extending it to others less
prohibiting mature books to all persons, regardless of age, would benefit deserving.chanroblesvirtualawlibrarychanrobles virtual law library
In their Comment, the respondents point out that available data from the their widows, does not send as much frank mail as the
Postal Service Office show that from January 1988 to June 1992, the total Judiciary.)chanrobles virtual law library
volume of frank mails amounted to P90,424,175.00. Of this amount, frank
mails from the Judiciary and other agencies whose functions include the
It is worth observing that the Philippine Postal Corporation, as a
service of judicial processes, such as the intervenor, the Department of
government-controlled corporation, was created and is expected to operate
Justice and the Office of the Ombudsman, amounted to P86,481,759. Frank
for the purpose of promoting the public service. While it may have been
mails coming fromthe Judiciary amounted to P73,574,864.00, and those
established primarily for private gain, it cannot excuse itself from
coming from the petitioners reached the total amount of P60,991,431.00.
performing certain functions for the benefit of the public in exchange for
The respondents' conclusion is that because of this considerable volume of
the franchise extended to it by the government and the many advantages
mail from the Judiciary, the franking privilege must be withdrawn from
it enjoys under its charter. 14 Among the services it should be prepared to
it.chanroblesvirtualawlibrarychanrobles virtual law library
extend is free carriage of mail for certain offices of the government that
need the franking privilege in the discharge of their own public
The argument is self-defeating. The respondents are in effect saying that functions.chanroblesvirtualawlibrarychanrobles virtual law library
the franking privilege should be extended only to those who do not need it
very much, if at all, (like the widows of former Presidents) but not to those
We also note that under Section 9 of the law, the Corporation is capitalized
who need it badly (especially the courts of justice). It is like saying that a
at P10 billion pesos, 55% of which is supplied by the Government, and that
person may be allowed cosmetic surgery although it is not really necessary
it derives substantial revenues from the sources enumerated in Section 10,
but not an operation that can save his
on top of the exemptions it enjoys. It is not likely that the retention of the
life.chanroblesvirtualawlibrarychanrobles virtual law library
franking privilege of the Judiciary will cripple the
Corporation.chanroblesvirtualawlibrarychanrobles virtual law library
If the problem of the respondents is the loss of revenues from the franking
privilege, the remedy, it seems to us, is to withdraw it altogether
At this time when the Judiciary is being faulted for the delay in the
from all agencies of government, including those who do not need it. The
administration of justice, the withdrawal from it of the franking privilege
problem is not solved by retaining it for some and withdrawing it from
can only further deepen this serious problem. The volume of judicial mail,
others, especially where there is no substantial distinction between those
as emphasized by the respondents themselves, should stress the
favored, which may or may not need it at all, and the Judiciary, which
dependence of the courts of justice on the postal service for
definitely needs it. The problem is not solved by violating the
communicating with lawyers and litigants as part of the judicial process.
Constitution.chanroblesvirtualawlibrarychanrobles virtual law library
The Judiciary has the lowest appropriation in the national budget compared
to the Legislative and Executive Departments; of the P309 billion budgeted
In lumping the Judiciary with the other offices from which the franking for 1993, only .84%, or less than 1%, is alloted for the judiciary. It should
privilege has been withdrawn, Section 35 has placed the courts of justice in not be hard to imagine the increased difficulties of our courts if they have
a category to which it does not belong. If it recognizes the need of the to affix a purchased stamp to every process they send in the discharge of
President of the Philippines and the members of Congress for the franking their judicial functions.chanroblesvirtualawlibrarychanrobles virtual law
privilege, there is no reason why it should not recognize a similar and in library
fact greater need on the part of the Judiciary for such privilege. While we
may appreciate the withdrawal of the franking privilege from the Armed
We are unable to agree with the respondents that Section 35 of R.A. No.
Forces of the Philippines Ladies Steering Committee, we fail to understand
7354 represents a valid exercise of discretion by the Legislature under the
why the Supreme Court should be similarly treated as that Committee. And
police power. On the contrary, we find its repealing clause to be a
while we may concede the need of the National Census and Statistics Office
discriminatory provision that denies the Judiciary the equal protection of
for the franking privilege, we are intrigued that a similar if not greater need
the laws guaranteed for all persons or things similarly situated. The
is not recognized in the courts of
distinction made by the law is superficial. It is not based on substantial
justice.chanroblesvirtualawlibrarychanrobles virtual law library
distinctions that make real differences between the Judiciary and the
grantees of the franking privilege.chanroblesvirtualawlibrarychanrobles
(On second thought, there does not seem to be any justifiable need for virtual law library
withdrawing the privilege from the Armed Forces of the Philippines Ladies
Steering Committee, which, like former Presidents of the Philippines or
This is not a question of wisdom or power into which the Judiciary may not
intrude. It is a matter of arbitrariness that this Court has the duty and
power to correct.

IVchanrobles virtual law library

In sum, we sustain R.A. No. 7354 against the attack that its subject is not
expressed in its title and that it was not passed in accordance with the
prescribed procedure. However, we annul Section 35 of the law as violative
of Article 3, Sec. 1, of the Constitution providing that no person shall "be
deprived of the equal protection of laws."chanrobles virtual law library

We arrive at these conclusions with a full awareness of the criticism it is


certain to provoke. While ruling against the discrimination in this case, we
may ourselves be accused of similar discrimination through the exercise of
our ultimate power in our own favor. This is inevitable. Criticism of judicial
conduct, however undeserved, is a fact of life in the political system that
we are prepared to accept.. As judges, we cannot debate with our
detractors. We can only decide the cases before us as law imposes on us
the duty to be fair and our own conscience gives us the light to be
right.chanroblesvirtualawlibrarychanrobles virtual law library

ACCORDINGLY, the petition is partially GRANTED and Section 35 of R.A.


No. 7354 is declared UNCONSTITUTIONAL. Circular No. 92-28 is SET ASIDE
insofar as it withdraws the franking privilege from the Supreme Court, the
Court of Appeals, the Regional trail Courts, the Municipal trial Courts, and
the National Land Registration Authority and its Register of Deeds to all of
which offices the said privilege shall be RESTORED. The temporary
restraining order dated June 2, 1992, is made
permanent.chanroblesvirtualawlibrarychanrobles virtual law library

SO ORDERED.
Article VI, Section 26(1) - Every bill passed by the Congress shall embrace
only one subject which shall be expressed in the title
thereof.chanroblesvirtualawlibrarychanrobles virtual law library

Article VI, Section 28(1) - The rule of taxation shall be uniform and
equitable. The Congress shall evolve a progressive system of
taxation.chanroblesvirtualawlibrarychanrobles virtual law library

G.R. No. 109289 October 3, 1994


Article III, Section 1 - No person shall be deprived of . . . property without
due process of law, nor shall any person be denied the equal protection of
RUFINO R. TAN, Petitioner, v. RAMON R. DEL ROSARIO, JR., as the laws.
SECRETARY OF FINANCE & JOSE U. ONG, as COMMISSIONER OF
INTERNAL REVENUE, Respondents.
In G.R. No. 109446, petitioners, assailing Section 6 of Revenue Regulations
No. 2-93, argue that public respondents have exceeded their rule-making
G.R. No. 109446 October 3, 1994 authority in applying SNIT to general professional
partnerships.chanroblesvirtualawlibrarychanrobles virtual law library
CARAG, CABALLES, JAMORA AND SOMERA LAW OFFICES, CARLO A.
CARAG, MANUELITO O. CABALLES, ELPIDIO C. JAMORA, JR. and The Solicitor General espouses the position taken by public
BENJAMIN A. SOMERA, JR., Petitioners, v. RAMON R. DEL ROSARIO, respondents.chanroblesvirtualawlibrarychanrobles virtual law library
in his capacity as SECRETARY OF FINANCE and JOSE U. ONG, in his
capacity as COMMISSIONER OF INTERNAL REVENUE, Respondents.
The Court has given due course to both petitions. The parties, in
compliance with the Court's directive, have filed their respective
Rufino R. Tan for and in his own behalf.chanrobles virtual law library memoranda.

Carag, Caballes, Jamora & Zomera Law Offices for petitioners in G.R. G.R. No. 109289
109446.
Petitioner contends that the title of House Bill No. 34314, progenitor of
VITUG, J.: Republic Act No. 7496, is a misnomer or, at least, deficient for being
merely entitled, "Simplified Net Income Taxation Scheme for the Self-
Employed
These two consolidated special civil actions for prohibition challenge, in
and Professionals Engaged in the Practice of their Profession" (Petition in
G.R. No. 109289, the constitutionality of Republic Act No. 7496, also
G.R. No. 109289).chanroblesvirtualawlibrarychanrobles virtual law library
commonly known as the Simplified Net Income Taxation Scheme ("SNIT"),
amending certain provisions of the National Internal Revenue Code and, in
G.R. No. 109446, the validity of Section 6, Revenue Regulations No. 2-93, The full text of the title actually reads:
promulgated by public respondents pursuant to said
law.chanroblesvirtualawlibrarychanrobles virtual law library
An Act Adopting the Simplified Net Income Taxation Scheme For The Self-
Employed and Professionals Engaged In The Practice of Their Profession,
Petitioners claim to be taxpayers adversely affected by the continued Amending Sections 21 and 29 of the National Internal Revenue Code, as
implementation of the amendatory Amended.
legislation.chanroblesvirtualawlibrarychanrobles virtual law library
The pertinent provisions of Sections 21 and 29, so referred to, of the
In G.R. No. 109289, it is asserted that the enactment of Republic Act National Internal Revenue Code, as now amended, provide:
No. 7496 violates the following provisions of the Constitution:
Sec. 21. Tax on citizens or residents. - (d) Business rentals;chanrobles virtual law library

xxx xxx xxxchanrobles virtual law library (e) Depreciation;chanrobles virtual law library

(f) Simplified Net Income Tax for the Self-Employed and/or Professionals (f) Contributions made to the Government and accredited relief
Engaged in the Practice of Profession. - A tax is hereby imposed upon the organizations for the rehabilitation of calamity stricken areas declared by
taxable net income as determined in Section 27 received during each the President; andchanrobles virtual law library
taxable year from all sources, other than income covered by paragraphs
(b), (c), (d) and (e) of this section by every individual whether
(g) Interest paid or accrued within a taxable year on loans contracted from
a citizen of the Philippines or an alien residing in the Philippines who is self-
accredited financial institutions which must be proven to have been
employed or practices his profession herein, determined in accordance with
incurred in connection with the conduct of a taxpayer's profession, trade or
the following schedule:
business.chanroblesvirtualawlibrarychanrobles virtual law library

Not over P10,000 3%


For individuals whose cost of goods sold and direct costs are difficult to
determine, a maximum of forty per cent (40%) of their gross receipts shall
Over P10,000 P300 + 9% be allowed as deductions to answer for business or professional expenses
but not over P30,000 of excess over P10,000 as the case may be.

Over P30,000 P2,100 + 15% On the basis of the above language of the law, it would be difficult to
but not over P120,00 of excess over P30,000 accept petitioner's view that the amendatory law should be considered as
having now adopted a gross income, instead of as having still retained
the net income, taxation scheme. The allowance for deductible items, it is
Over P120,000 P15,600 + 20%
true, may have significantly been reduced by the questioned law in
but not over P350,000 of excess over P120,000
comparison with that which has prevailed prior to the amendment; limiting,
however, allowable deductions from gross income is neither discordant
Over P350,000 P61,600 + 30% with, nor opposed to, the net income tax concept. The fact of the matter is
of excess over P350,000chanrobles virtual law library still that various deductions, which are by no means inconsequential,
continue to be well provided under the new
Sec. 29. Deductions from gross income. - In computing taxable income law.chanroblesvirtualawlibrarychanrobles virtual law library
subject to tax under Sections 21(a), 24(a), (b) and (c); and 25 (a)(1),
there shall be allowed as deductions the items specified in paragraphs (a) Article VI, Section 26(1), of the Constitution has been envisioned so as (a)
to (i) of this section:Provided, however, That in computing taxable income to prevent log-rolling legislation intended to unite the members of the
subject to tax under Section 21 (f) in the case of individuals engaged in legislature who favor any one of unrelated subjects in support of the whole
business or practice of profession, only the following direct costs shall be act, (b) to avoid surprises or even fraud upon the legislature, and (c) to
allowed as deductions:chanrobles virtual law library fairly apprise the people, through such publications of its proceedings as
are usually made, of the subjects of legislation. 1The above objectives of
(a) Raw materials, supplies and direct labor;chanrobles virtual law library the fundamental law appear to us to have been sufficiently met. Anything
else would be to require a virtual compendium of the law which could not
have been the intendment of the constitutional
(b) Salaries of employees directly engaged in activities in the course of or mandate.chanroblesvirtualawlibrarychanrobles virtual law library
pursuant to the business or practice of their profession;chanrobles virtual
law library
Petitioner intimates that Republic Act No. 7496 desecrates the
constitutional requirement that taxation "shall be uniform and equitable" in
(c) Telecommunications, electricity, fuel, light and water;chanrobles virtual that the law would now attempt to tax single proprietorships and
law library
professionals differently from the manner it imposes the tax on G.R. No. 109446
corporations and partnerships. The contention clearly forgets, however,
that such a system of income taxation has long been the prevailing rule
The several propositions advanced by petitioners revolve around the
even prior to Republic Act No. 7496.chanroblesvirtualawlibrarychanrobles
question of whether or not public respondents have exceeded their
virtual law library
authority in promulgating Section 6, Revenue Regulations No. 2-93, to
carry out Republic Act No. 7496.chanroblesvirtualawlibrarychanrobles
Uniformity of taxation, like the kindred concept of equal protection, merely virtual law library
requires that all subjects or objects of taxation, similarly situated, are to be
treated alike both in privileges and liabilities (Juan Luna Subdivision vs.
The questioned regulation reads:
Sarmiento, 91 Phil. 371). Uniformity does not forfend classification as long
as: (1) the standards that are used therefor are substantial and not
arbitrary, (2) the categorization is germane to achieve the legislative Sec. 6. General Professional Partnership - The general professional
purpose, (3) the law applies, all things being equal, to both present and partnership (GPP) and the partners comprising the GPP are covered by R.
future conditions, and (4) the classification applies equally well to all those A. No. 7496. Thus, in determining the net profit of the partnership, only
belonging to the same class (Pepsi Cola vs. City of Butuan, 24 SCRA the direct costs mentioned in said law are to be deducted from partnership
3; Basco vs. PAGCOR, 197 SCRA 52).chanroblesvirtualawlibrarychanrobles income. Also, the expenses paid or incurred by partners in their individual
virtual law library capacities in the practice of their profession which are not reimbursed or
paid by the partnership but are not considered as direct cost, are not
deductible from his gross income.
What may instead be perceived to be apparent from the amendatory law is
the legislative intent to increasingly shift the income tax system towards
the schedular approach 2 in the income taxation of individual taxpayers and The real objection of petitioners is focused on the administrative
to maintain, by and large, the present global treatment 3 on taxable interpretation of public respondents that would apply SNIT to partners in
corporations. We certainly do not view this classification to be arbitrary and general professional partnerships. Petitioners cite the pertinent
inappropriate.chanroblesvirtualawlibrarychanrobles virtual law library deliberations in Congress during its enactment of Republic Act No. 7496,
also quoted by the Honorable Hernando B. Perez, minority floor leader of
the House of Representatives, in the latter's privilege speech by way of
Petitioner gives a fairly extensive discussion on the merits of the law,
commenting on the questioned implementing regulation of public
illustrating, in the process, what he believes to be an imbalance between
respondents following the effectivity of the law, thusly:
the tax liabilities of those covered by the amendatory law and those who
are not. With the legislature primarily lies the discretion to determine the
nature (kind), object (purpose), extent (rate), coverage (subjects) MR. ALBANO, Now Mr. Speaker, I would like to get the correct impression
and situs(place) of taxation. This court cannot freely delve into those of this bill. Do we speak here of individuals who are earning, I mean, who
matters which, by constitutional fiat, rightly rest on legislative judgment. earn through business enterprises and therefore, should file an income tax
Of course, where a tax measure becomes so unconscionable and unjust as return?chanrobles virtual law library
to amount to confiscation of property, courts will not hesitate to strike it
down, for, despite all its plenitude, the power to tax cannot override MR. PEREZ. That is correct, Mr. Speaker. This does not apply to
constitutional proscriptions. This stage, however, has not been corporations. It applies only to individuals.
demonstrated to have been reached within any appreciable distance in this
controversy before us.chanroblesvirtualawlibrarychanrobles virtual law
library (See Deliberations on H. B. No. 34314, August 6, 1991, 6:15 P.M.;
Emphasis ours).

Having arrived at this conclusion, the plea of petitioner to have the law
declared unconstitutional for being violative of due process must perforce Other deliberations support this position, to wit:chanrobles virtual law
fail. The due process clause may correctly be invoked only when there is a library
clear contravention of inherent or constitutional limitations in the exercise
of the tax power. No such transgression is so evident to us. MR. ABAYA . . . Now, Mr. Speaker, did I hear the Gentleman from Batangas
say that this bill is intended to increase collections as far as individuals are
concerned and to make collection of taxes equitable?chanrobles virtual law (2) Shall be deemed to have elected the itemized deductions, unless he
library declares his distributive share of the gross income undiminished by his
share of the deductions.
MR. PEREZ. That is correct, Mr. Speaker.
There is, then and now, no distinction in income tax liability between a
person who practices his profession alone or individually and one who does
(Id. at 6:40 P.M.; Emphasis ours).chanroblesvirtualawlibrarychanrobles
it through partnership (whether registered or not) with others in the
virtual law library
exercise of a common profession. Indeed, outside of the gross
compensation income tax and the final tax on passive investment income,
In fact, in the sponsorship speech of Senator Mamintal Tamano on the under the present income tax system all individuals deriving income from
Senate version of the SNITS, it is categorically stated, thus: any source whatsoever are treated in almost invariably the same manner
and under a common set of rules.chanroblesvirtualawlibrarychanrobles
This bill, Mr. President, is not applicable to business corporations or to virtual law library
partnerships; it is only with respect to individuals and professionals.
(Emphasis ours) We can well appreciate the concern taken by petitioners if perhaps we were
to consider Republic Act No. 7496 as an entirely independent, not merely
The Court, first of all, should like to correct the apparent misconception as an amendatory, piece of legislation. The view can easily become myopic,
that general professional partnerships are subject to the payment of however, when the law is understood, as it should be, as only forming part
income tax or that there is a difference in the tax treatment between of, and subject to, the whole income tax concept and precepts long
individuals engaged in business or in the practice of their respective obtaining under the National Internal Revenue Code. To elaborate a little,
professions and partners in general professional partnerships. The fact of the phrase "income taxpayers" is an all embracing term used in the Tax
the matter is that a general professional partnership, unlike an ordinary Code, and it practically covers all persons who derive taxable income. The
business partnership (which is treated as a corporation for income tax law, in levying the tax, adopts the most comprehensive tax situs of
purposes and so subject to the corporate income tax), is not itself an nationality and residence of the taxpayer (that renders citizens, regardless
income taxpayer. The income tax is imposed not on the professional of residence, and resident aliens subject to income tax liability on their
partnership, which is tax exempt, but on the partners themselves in their income from all sources) and of the generally accepted and internationally
individual capacity computed on their distributive shares of partnership recognized income taxable base (that can subject non-resident aliens and
profits. Section 23 of the Tax Code, which has not been amended at all by foreign corporations to income tax on their income from Philippine
Republic Act 7496, is explicit: sources). In the process, the Code classifies taxpayers into four main
groups, namely: (1) Individuals, (2) Corporations, (3) Estates under
Judicial Settlement and (4) Irrevocable Trusts (irrevocable both as
Sec. 23. Tax liability of members of general professional partnerships. - (a) to corpus and as to income).chanroblesvirtualawlibrarychanrobles virtual
Persons exercising a common profession in general partnership shall be law library
liable for income tax only in their individual capacity, and the share in the
net profits of the general professional partnership to which any taxable
partner would be entitled whether distributed or otherwise, shall be Partnerships are, under the Code, either "taxable partnerships" or "exempt
returned for taxation and the tax paid in accordance with the provisions of partnerships." Ordinarily, partnerships, no matter how created or
this Title.chanroblesvirtualawlibrarychanrobles virtual law library organized, are subject to income tax (and thus alluded to as "taxable
partnerships") which, for purposes of the above categorization, are by law
assimilated to be within the context of, and so legally contemplated as,
(b) In determining his distributive share in the net income of the corporations. Except for few variances, such as in the application of the
partnership, each partner - "constructive receipt rule" in the derivation of income, the income tax
approach is alike to both juridical persons. Obviously, SNIT is not intended
(1) Shall take into account separately his distributive share of the or envisioned, as so correctly pointed out in the discussions in Congress
partnership's income, gain, loss, deduction, or credit to the extent provided during its deliberations on Republic Act 7496, aforequoted, to cover
by the pertinent provisions of this Code, andchanrobles virtual law library corporations and partnerships which are independently subject to the
payment of income tax.chanroblesvirtualawlibrarychanrobles virtual law
library

"Exempt partnerships," upon the other hand, are not similarly identified as
corporations nor even considered as independent taxable entities for
income tax purposes. A generalprofessional partnership is such an
example. 4 Here, the partners themselves, not the partnership (although it
is still obligated to file an income tax return [mainly for administration and
data]), are liable for the payment of income tax in their individual capacity
computed on their respective and distributive shares of profits. In the
determination of the tax liability, a partner does so as an individual, and
there is no choice on the matter. In fine, under the Tax Code on income
taxation, the general professional partnership is deemed to be no more
than a mere mechanism or a flow-through entity in the generation of
income by, and the ultimate distribution of such income to, respectively,
each of the individual partners.chanroblesvirtualawlibrarychanrobles virtual
law library

Section 6 of Revenue Regulation No. 2-93 did not alter, but merely
confirmed, the above standing rule as now so modified by Republic Act
No. 7496 on basically the extent of allowable deductions applicable
to all individual income taxpayers on their non-compensation income.
There is no evident intention of the law, either before or after the
amendatory legislation, to place in an unequal footing or in significant
variance the income tax treatment of professionals who practice their
respective professions individually and of those who do it through a general
professional partnership.chanroblesvirtualawlibrarychanrobles virtual law
library

WHEREFORE, the petitions are DISMISSED. No special pronouncement on


costs.chanroblesvirtualawlibrarychanrobles virtual law library

SO ORDERED.
represented by its President,
MERCEDITAS A. GARCIA;
ROSARIO ANTONIO doing
business under the name and
style of 'ANB NORTH SHELL
SERVICE STATION; LOURDES
THE HONORABLE EXECUTIVE     MARTINEZ doing business under
SECRETARY EDUARDO ERMITA; the name and style of 'SHELL
HONORABLE SECRETARY OF THE GATE ' N. DOMINGO; BETHZAIDA
DEPARTMENT OF FINANCE CESAR TAN doing business under the
PURISIMA; and HONORABLE name and style of 'ADVANCE
COMMISSIONER OF INTERNAL SHELL STATION; REYNALDO P.
REVENUE GUILLERMO PARAYNO, MONTOYA doing business under
JR., the name and style of 'NEW
Respondents.     LAMUAN SHELL SERVICE
      STATION; EFREN SOTTO doing
x-----------------------     business under the name and
- - x  style of 'RED FIELD SHELL
      SERVICE STATION; DONICA
AQUILINO Q. PIMENTEL, JR.,   G.R. No. 168207 CORPORATION represented by its
LUISA P. EJERCITO-ESTRADA, President, DESI TOMACRUZ; RUTH
JINGGOY E. ESTRADA, PANFILO M. E. MARBIBI doing business under
LACSON, ALFREDO S. LIM, JAMBY the name and style of 'R&R
A.S. MADRIGAL, AND SERGIO R. PETRON STATION; PETER M.
OSMEA III, UNGSON doing business under the
Petitioners,      name and style of 'CLASSIC STAR
      GASOLINE SERVICE STATION;
- versus -     MARIAN SHEILA A. LEE doing
      business under the name and
EXECUTIVE SECRETARY EDUARDO     style of 'NTE GASOLINE &
R. ERMITA, CESAR V. PURISIMA, SERVICE STATION; JULIAN CESAR
SECRETARY OF FINANCE, P. POSADAS doing business under
GUILLERMO L. PARAYNO, JR., the name and style of
COMMISSIONER OF THE BUREAU 'STARCARGA ENTERPRISES' ;
OF INTERNAL REVENUE, ADORACION MAEBO doing
Respondents.     business under the name and
      style of 'CMA MOTORISTS CENTER;
x-----------------------     SUSAN M. ENTRATA doing
--x business under the name and
      style of 'LEONA'S GASOLINE
ASSOCIATION OF PILIPINAS   G.R. No. 168461 STATION and SERVICE CENTER;
SHELL DEALERS, INC. represented CARMELITA BALDONADO doing
by its President, ROSARIO business under the name and
ANTONIO; PETRON DEALERS' style of 'FIRST CHOICE SERVICE
ASSOCIATION represented by its CENTER; MERCEDITAS A. GARCIA
President, RUTH E. BARBIBI; doing business under the name
ASSOCIATION OF CALTEX and style of 'LORPED SERVICE
DEALERS' OF THE PHILIPPINES CENTER; RHEAMAR A. RAMOS
doing business under the name BENJAMIN C. AGARAO, JR. JUAN
and style of 'RJRAM PTT GAS EDGARDO M. ANGARA, JUSTIN
STATION; MA. ISABEL VIOLAGO MARC SB. CHIPECO, FLORENCIO
doing business under the name G. NOEL, MUJIV S. HATAMAN,
and style of 'VIOLAGO-PTT RENATO B. MAGTUBO, JOSEPH A.
SERVICE CENTER; MOTORISTS' SANTIAGO, TEOFISTO DL.
HEART CORPORATION GUINGONA III, RUY ELIAS C.
represented by its Vice-President LOPEZ, RODOLFO Q. AGBAYANI
for Operations, JOSELITO F. and TEODORO A. CASIO,
FLORDELIZA; MOTORISTS' Petitioners,     
HARVARD CORPORATION      
represented by its Vice-President - versus -    
for Operations, JOSELITO F.      
FLORDELIZA; MOTORISTS'
HERITAGE CORPORATION
CESAR V. PURISIMA, in his  
represented by its Vice-President
capacity as Secretary of Finance,
for Operations, JOSELITO F.
GUILLERMO L. PARAYNO, JR., in
FLORDELIZA; PHILIPPINE  
his capacity as Commissioner of
STANDARD OIL CORPORATION
Internal Revenue, and EDUARDO
represented by its Vice-President  
R. ERMITA, in his capacity as
for Operations, JOSELITO F.
Executive Secretary,
FLORDELIZA; ROMEO MANUEL
doing business under the name  
and style of 'ROMMAN GASOLINE
STATION; ANTHONY ALBERT CRUZ  
III doing business under the Respondents.    
name and style of 'TRUE SERVICE      
STATION',  x-----------------------    
'Petitioners,     - - x 
           
- versus -     BATAAN GOVERNOR ENRIQUE T.   G.R. No. 168730
      GARCIA, JR.
CESAR V. PURISIMA, in his     Petitioner,     
capacity as Secretary of the      
Department of Finance and - versus -    
GUILLERMO L. PARAYNO, JR., in      
his capacity as Commissioner of HON. EDUARDO R. ERMITA, in his    
Internal Revenue, capacity as the Executive
Respondents.     Secretary; HON. MARGARITO
       
TEVES, in his capacity as
x-----------------------     Secretary of Finance; HON. JOSE
- - x  MARIO BUNAG, in his capacity as  
      the OIC Commissioner of the
FRANCIS JOSEPH G. ESCUDERO,   G.R. No. 168463 Bureau of Internal Revenue; and  
VINCENT CRISOLOGO, EMMANUEL HON. ALEXANDER AREVALO, in his
JOEL J. VILLANUEVA, RODOLFO G. capacity as the OIC Commissioner
PLAZA, DARLENE ANTONINO- of the Bureau of Customs,
CUSTODIO, OSCAR G. MALAPITAN,
'French statesman and economist
 

 
 

  Mounting budget deficit, revenue generation, inadequate fiscal allocation

  for education, increased emoluments for health workers, and wider

coverage for full value-added tax benefits ' these are the reasons why
Promulgated:
Respondents.   September 1, 2005 Republic Act No. 9337 (R.A. No. 9337) [1] was enacted. Reasons, the

wisdom of which, the Court even with its extensive constitutional power of
 
review, cannot probe. The petitioners in these cases, however, question
x----------------------------------------------
not only the wisdom of the law, but also perceived constitutional infirmities
-------------x
in its passage.
 
 
 

DECISION Every law enjoys in its favor the presumption of constitutionality. Their

arguments notwithstanding, petitioners failed to justify their call for the


 
invalidity of the law. Hence, R.A. No. 9337 is not unconstitutional.
 
 
AUSTRIA-MARTINEZ, J .:

LEGISLATIVE HISTORY
 
 
 

The expenses of government, having for their object the interest of all, R.A. No. 9337 is a consolidation of three legislative bills namely, House Bill
should be borne by everyone, and the more man enjoys the advantages of
society, the more he ought to hold himself honored in contributing to those Nos. 3555 and 3705, and Senate Bill No. 1950. 
expenses.
 
-Anne Robert Jacques Turgot (1727-1781)
 
House Bill No. 3555 [2] was introduced on first reading on January 7,

2005. The House Committee on Ways and Means approved the bill, in
On the same date, April 13, 2005, the Senate agreed to the request of the
substitution of House Bill No. 1468, which Representative (Rep.) Eric D.
House of Representatives for a committee conference on the disagreeing
Singson introduced on August 8, 2004. The President certified the bill
provisions of the proposed bills. 
on January 7, 2005 for immediate enactment. On January 27, 2005, the
 
House of Representatives approved the bill on second and third reading.

  Before long, the Conference Committee on the Disagreeing Provisions of

House Bill No. 3555, House Bill No. 3705, and Senate Bill No. 1950, 'after
House Bill No. 3705 [3] on the other hand, substituted House Bill No.
having met and discussed in full free and conference, recommended the
3105 introduced by Rep. Salacnib F. Baterina, and House Bill No. 3381
approval of its report, which the Senate did on May 10, 2005, and with the
introduced by Rep. Jacinto V. Paras. Its 'mother bill is House Bill No. 3555.
House of Representatives agreeing thereto the next day, May 11, 2005. 
The House Committee on Ways and Means approved the bill on February 2,
 
2005. The President also certified it as urgent on February 8, 2005. The

House of Representatives approved the bill on second and third reading


On May 23, 2005, the enrolled copy of the consolidated House and Senate
on February 28, 2005.
version was transmitted to the President, who signed the same into law

  on May 24, 2005. Thus, came R.A. No. 9337. 

 
Meanwhile, the Senate Committee on Ways and Means approved Senate

Bill No. 1950 [4] on March 7, 2005, 'in substitution of Senate Bill Nos.


July 1, 2005 is the effectivity date of R.A. No. 9337. [5] When said date
1337, 1838 and 1873, taking into consideration House Bill Nos. 3555 and
came, the Court issued a temporary restraining order, effective
3705. Senator Ralph G. Recto sponsored Senate Bill No. 1337, while
immediately and continuing until further orders, enjoining respondents
Senate Bill Nos. 1838 and 1873 were both sponsored by Sens. Franklin M.
from enforcing and implementing the law. 
Drilon, Juan M. Flavier and Francis N. Pangilinan. The President certified
 
the bill on March 11, 2005, and was approved by the Senate on second and

third reading on April 13, 2005.


J. PANGANIBAN : That's correct . . .
Oral arguments were held on July 14, 2005. Significantly, during the  
ATTY. BANIQUED : . . . and therefore that was meant to
hearing, the Court speaking through Mr. Justice Artemio V. Panganiban, temper the impact . . .
interrupted
voiced the rationale for its issuance of the temporary restraining order
 
on July 1, 2005, to wit:  
J. PANGANIBAN : . . . mitigating measures . . .
 
J. PANGANIBAN : . . . But before I go into the details of ATTY. BANIQUED : Yes, Your Honor.
your presentation, let me just  
tell you a little background. J. PANGANIBAN : As a matter of fact a part of the
You know when the law took mitigating measures would be
effect on July 1, 2005, the the elimination of the Excise
Court issued a TRO at about 5 Tax and the import duties.
oclock in the afternoon. But That is why, it is not correct to
before that, there was a lot of say that the VAT as to
complaints aired on television petroleum dealers increased
and on radio. Some people in prices by 10%.
a gas station were complaining  
that the gas prices went up by ATTY. BANIQUED : Yes, Your Honor.
10%. Some people were  
complaining that their electric J. PANGANIBAN : And therefore, there is no justification
bill will go up by 10%. Other for increasing the retail price
times people riding in domestic by 10% to cover the E-Vat tax.
air carrier were complaining If you consider the excise tax
that the prices that theyll have and the import duties, the Net
to pay would have to go up by Tax would probably be in the
10%. While all that was being neighborhood of 7%? We are
aired, per your presentation not going into exact figures I
and per our own am just trying to deliver a
understanding of the law, point that different industries,
that's not true. It's not true different products, different
that the e-vat law necessarily services are hit differently. So
increased prices by 10% it's not correct to say that all
uniformly isnt it? prices must go up by 10%.
  ATTY. BANIQUED : Youre right, Your Honor.
ATTY. BANIQUED : No, Your Honor.  
   
J. PANGANIBAN : It is not? J. PANGANIBAN : Now. For instance, Domestic Airline
  companies, Mr. Counsel, are at
ATTY. BANIQUED : It's not, because, Your Honor, there is present imposed a Sales Tax
an Executive Order that of 3%. When this E-Vat law
granted the Petroleum took effect the Sales Tax was
companies some subsidy . . . also removed as a mitigating
interrupted measure. So, therefore, there
  is no justification to increase
 
the fares by 10% at best 7%, The Court also directed the parties to file their respective Memoranda.
correct?
 
 
ATTY. BANIQUED : I guess so, Your Honor, yes.
 
J. PANGANIBAN : There are other products that the
people were complaining on G.R. No. 168056
that first day, were being
increased arbitrarily by 10%.  
And that's one reason among
many others this Court had to
issue TRO because of the
Before R.A. No. 9337 took effect, petitioners ABAKADA GURO Party List, et
confusion in the
implementation. That's why we al., filed a petition for prohibition on May 27, 2005. They question the
added as an issue in this case,
even if it's tangentially taken constitutionality of Sections 4, 5 and 6 of R.A. No. 9337, amending
up by the pleadings of the
parties, the confusion in the Sections 106, 107 and 108, respectively, of the National Internal Revenue
implementation of the E-vat.
Our people were subjected to Code (NIRC). Section 4 imposes a 10% VAT on sale of goods and
the mercy of that confusion of
an across the board increase properties, Section 5 imposes a 10% VAT on importation of goods, and
of 10%, which you yourself
Section 6 imposes a 10% VAT on sale of services and use or lease of
now admit and I think even
the Government will admit is properties. These questioned provisions contain a
incorrect. In some cases, it
should be 3% only, in some uniform proviso authorizing the President, upon recommendation of the
cases it should be 6%
depending on these mitigating Secretary of Finance, to raise the VAT rate to 12%, effective January 1,
measures and the location and
situation of each product, of 2006, after any of the following conditions have been satisfied, to wit:
each service, of each
company, isnt it?  
 
ATTY. BANIQUED : Yes, Your Honor.
  . . . That the President, upon the recommendation of the
J. PANGANIBAN : Alright. So that's one reason why we Secretary of Finance, shall, effective January 1, 2006,
had to issue a TRO pending raise the rate of value-added tax to twelve percent
the clarification of all these (12%), after any of the following conditions has been
and we wish the government satisfied:
will take time to clarify all  
these by means of a more (i) Value-added tax collection as a percentage of Gross
detailed implementing rules, in Domestic Product (GDP) of the previous year exceeds two
case the law is upheld by this and four-fifth percent (2 4/5%); or
Court. . . . [6]  
  (ii) National government deficit as a percentage of GDP of
  the previous year exceeds one and one-half percent (1
'%).
 
President to raise the VAT collection to at least 2 4/5 of the GDP of the
  previous year, should only be based on fiscal adequacy. 

 
Petitioners argue that the law is unconstitutional, as it constitutes

abandonment by Congress of its exclusive authority to fix the rate of taxes


Petitioners further claim that the inclusion of a stand-by authority granted
under Article VI, Section 28(2) of the 1987 Philippine Constitution. 
to the President by the Bicameral Conference Committee is a violation of
  the 'no-amendment rule upon last reading of a bill laid down in Article VI,

Section 26(2) of the Constitution.


G.R. No. 168207
 
 

G.R. No. 168461


On June 9, 2005, Sen. Aquilino Q. Pimentel, Jr., et al., filed a petition
 
for certiorari likewise assailing the constitutionality of Sections 4, 5 and 6

of R.A. No. 9337. 


Thereafter, a petition for prohibition was filed on June 29, 2005, by the

Association of Pilipinas Shell Dealers, Inc., et al., assailing the following


Aside from questioning the so-called stand-by authority of the President to
provisions of R.A. No. 9337:
increase the VAT rate to 12%, on the ground that it amounts to an undue

delegation of legislative power, petitioners also contend that the increase in 1) Section 8, amending Section 110 (A)(2) of the NIRC,
requiring that the input tax on depreciable goods
the VAT rate to 12% contingent on any of the two conditions being shall be amortized over a 60-month period, if
the acquisition, excluding the VAT components,
satisfied violates the due process clause embodied in Article III, Section 1 exceeds One Million Pesos (P1, 000,000.00);
 
of the Constitution, as it imposes an unfair and additional tax burden on
2) Section 8, amending Section 110 (B) of the NIRC,
the people, in that: (1) the 12% increase is ambiguous because it does not imposing a 70% limit on the amount of input tax
to be credited against the output tax; and 
state if the rate would be returned to the original 10% if the conditions are  
3) Section 12, amending Section 114 (c) of the NIRC,
no longer satisfied; (2) the rate is unfair and unreasonable, as the people authorizing the Government or any of its
political subdivisions, instrumentalities or
are unsure of the applicable VAT rate from year to year; and (3) the agencies, including GOCCs, to deduct a 5% final
withholding tax on gross payments of goods and
increase in the VAT rate, which is supposed to be an incentive to the services, which are subject to 10% VAT under
Sections 106 (sale of goods and properties) and
108 (sale of services and use or lease of several transactions with the government, is not based on real and
properties) of the NIRC. 
substantial differences to meet a valid classification.

 
 

 
Lastly, petitioners contend that the 70% limit is anything but progressive,

Petitioners contend that these provisions are unconstitutional for being violative of Article VI, Section 28(1) of the Constitution, and that it is the

arbitrary, oppressive, excessive, and confiscatory.  smaller businesses with higher input tax to output tax ratio that will suffer

the consequences thereof for it wipes out whatever meager margins the
 
petitioners make.

Petitioners' argument is premised on the constitutional right of non-


 
deprivation of life, liberty or property without due process of law under

Article III, Section 1 of the Constitution. According to petitioners, the G.R. No. 168463

contested sections impose limitations on the amount of input tax that may
 
be claimed. Petitioners also argue that the input tax partakes the nature of

a property that may not be confiscated, appropriated, or limited without Several members of the House of Representatives led by Rep. Francis
due process of law. Petitioners further contend that like any other property Joseph G. Escudero filed this petition for certiorari on June 30, 2005. They
or property right, the input tax credit may be transferred or disposed of, question the constitutionality of R.A. No. 9337 on the following grounds:
and that by limiting the same, the government gets to tax a profit or
 
value-added even if there is no profit or value-added.
1) Sections 4, 5, and 6 of R.A. No. 9337 constitute an
  undue delegation of legislative power, in
violation of Article VI, Section 28(2) of the
Constitution;
Petitioners also believe that these provisions violate the constitutional  
2) The Bicameral Conference Committee acted without
guarantee of equal protection of the law under Article III, Section 1 of the jurisdiction in deleting the no pass on provisions
present in Senate Bill No. 1950 and House Bill
Constitution, as the limitation on the creditable input tax if: (1) the entity No. 3705; and 
 
has a high ratio of input tax; or (2) invests in capital equipment; or (3) has 3) Insertion by the Bicameral Conference Committee of
Sections 27, 28, 34, 116, 117, 119, 121,
125, [7] 148, 151, 236, 237 and 288, which
were present in Senate Bill No. 1950, violates the presumption of constitutionality and petitioners failed to cast doubt on
Article VI, Section 24(1) of the Constitution,
which provides that all appropriation, revenue or its validity.
tariff bills shall originate exclusively in the House
of Representatives  

 
Relying on the case of Tolentino vs. Secretary of Finance, 235 SCRA 

G.R. No. 168730


630 (1994), respondents argue that the procedural issues raised by
  petitioners, i.e., legality of the bicameral proceedings, exclusive origination

of revenue measures and the power of the Senate concomitant thereto,


On the eleventh hour, Governor Enrique T. Garcia filed a petition
have already been settled. With regard to the issue of undue delegation of
for certiorariand prohibition on July 20, 2005, alleging unconstitutionality of
legislative power to the President, respondents contend that the law is
the law on the ground that the limitation on the creditable input tax in
complete and leaves no discretion to the President but to increase the rate
effect allows VAT-registered establishments to retain a portion of the taxes
to 12% once any of the two conditions provided therein arise.
they collect, thus violating the principle that tax collection and revenue
 
should be solely allocated for public purposes and expenditures. Petitioner

Garcia further claims that allowing these establishments to pass on the tax
Respondents also refute petitioners' argument that the increase to 12%, as
to the consumers is inequitable, in violation of Article VI, Section 28(1) of
well as the 70% limitation on the creditable input tax, the 60-month
the Constitution.
amortization on the purchase or importation of capital goods
  exceeding P1,000,000.00, and the 5% final withholding tax by government

agencies, is arbitrary, oppressive, and confiscatory, and that it violates the


RESPONDENTS' COMMENT
constitutional principle on progressive taxation, among others. 

 
 

The Office of the Solicitor General (OSG) filed a Comment in behalf of


Finally, respondents manifest that R.A. No. 9337 is the anchor of the
respondents. Preliminarily, respondents contend that R.A. No. 9337 enjoys
government's fiscal reform agenda. A reform in the value-added system of
taxation is the core revenue measure that will tilt the balance towards a RULING OF THE COURT

sustainable macroeconomic environment necessary for economic growth.


 

 
As a prelude, the Court deems it apt to restate the general principles and

ISSUES concepts of value-added tax (VAT), as the confusion and inevitably,

litigation, breeds from a fallacious notion of its nature.


 

 
The Court defined the issues, as follows:

The VAT is a tax on spending or consumption. It is levied on the sale,


 
barter, exchange or lease of goods or properties and services. [8] Being an
PROCEDURAL ISSUE
  indirect tax on expenditure, the seller of goods or services may pass on the
Whether R.A. No. 9337 violates the following provisions
amount of tax paid to the buyer, [9] with the seller acting merely as a tax
of the Constitution:
  collector. [10] The burden of VAT is intended to fall on the immediate
a. Article VI, Section 24, and
b. Article VI, Section 26(2) buyers and ultimately, the end-consumers. 
 
SUBSTANTIVE ISSUES
   
1. Whether Sections 4, 5 and 6 of R.A. No. 9337,
amending Sections 106, 107 and 108 of the NIRC, violate
the following provisions of the Constitution: In contrast, a direct tax is a tax for which a taxpayer is directly liable on
 
a. Article VI, Section 28(1), and the transaction or business it engages in, without transferring the burden
b. Article VI, Section 28(2)
  to someone else. [11] Examples are individual and corporate income taxes,
2. Whether Section 8 of R.A. No. 9337, amending
transfer taxes, and residence taxes. [12]
Sections 110(A)(2) and 110(B) of the NIRC; and Section
12 of R.A. No. 9337, amending Section 114(C) of the
NIRC, violate the following provisions of the Constitution:  
 
a. Article VI, Section 28(1), and 
b. Article III, Section 1 In the Philippines, the value-added system of sales taxation has long been

  in existence, albeit in a different mode. Prior to 1978, the system was a

single-stage tax computed under the 'cost deduction method and was
 
Whether R.A. No. 9337 violates the following provisions of the Constitution:
payable only by the original sellers. The single-stage system was

subsequently modified, and a mixture of the 'cost deduction method and  


a. Article VI, Section 24, and
'tax credit method was used to determine the value-added tax b. Article VI, Section 26(2)

payable. [13] Under the 'tax credit method, an entity can credit against or  

subtract from the VAT charged on its sales or outputs the VAT paid on its
A. The Bicameral Conference Committee
purchases, inputs and imports. [14]
 
 

Petitioners Escudero, et al., and Pimentel, et al., allege that the Bicameral
It was only in 1987, when President Corazon C. Aquino issued Executive
Conference Committee exceeded its authority by:
Order No. 273, that the VAT system was rationalized by imposing a multi-
 
stage tax rate of 0% or 10% on all sales using the 'tax credit method. [15]

1) Inserting the stand-by authority in favor of the


  President in Sections 4, 5, and 6 of R.A. No. 9337;
 
  2) Deleting entirely the no pass-on provisions found in
both the House and Senate bills; 
 
3) Inserting the provision imposing a 70% limit on the
E.O. No. 273 was followed by R.A. No. 7716 or the Expanded VAT
amount of input tax to be credited against the output
Law, [16] R.A. No. 8241 or the Improved VAT Law, [17] R.A. No. 8424 or tax; and
 
the Tax Reform Act of 1997, [18] and finally, the presently beleaguered 4) Including the amendments introduced only by Senate
Bill No. 1950 regarding other kinds of taxes in addition to
R.A. No. 9337, also referred to by respondents as the VAT Reform Act. the value-added tax.

   

 
The Court will now discuss the issues in logical sequence.

  Petitioners now beseech the Court to define the powers of the Bicameral

Conference Committee. 
PROCEDURAL ISSUE
 
I.
 
It should be borne in mind that the power of internal regulation and The Chairman of the House panel may be interpellated on
the Conference Committee Report prior to the voting
discipline are intrinsic in any legislative body for, as unerringly elucidated thereon. The House shall vote on the Conference
Committee Report in the same manner and procedure as
by Justice Story, '[i]f the power did not exist, it would be utterly
it votes on a bill on third and final reading.
impracticable to transact the business of the nation, either at all, or
 
at least with decency, deliberation, and order. [19] Thus, Article VI,

Section 16 (3) of the Constitution provides that 'each House may  

determine the rules of its proceedings. Pursuant to this inherent


Rule XII, Section 35 of the Rules of the Senate states:
constitutional power to promulgate and implement its own rules of

procedure, the respective rules of each house of Congress provided for the  

creation of a Bicameral Conference Committee. 


Sec. 35. In the event that the Senate does not agree with
the House of Representatives on the provision of any bill
  or joint resolution, the differences shall be settled by a
conference committee of both Houses which shall meet
within ten (10) days after their composition. The
Thus, Rule XIV, Sections 88 and 89 of the Rules of House of President shall designate the members of the Senate
Panel in the conference committee with the approval of
Representatives provides as follows: the Senate.
 
Each Conference Committee Report shall contain a
 
detailed and sufficiently explicit statement of the changes
in, or amendments to the subject measure, and shall be
Sec. 88. Conference Committee. ' In the event that the signed by a majority of the members of each House
House does not agree with the Senate on the amendment panel, voting separately.
to any bill or joint resolution, the differences may be  
settled by the conference committees of both chambers. A comparative presentation of the conflicting House and
  Senate provisions and a reconciled version thereof with
In resolving the differences with the Senate, the House the explanatory statement of the conference committee
panel shall, as much as possible, adhere to and support shall be attached to the report.
the House Bill. If the differences with the Senate are so  
substantial that they materially impair the House Bill, the
panel shall report such fact to the House for the latter's
...
appropriate action.
 
Sec. 89. Conference Committee Reports. ' . . . Each  
report shall contain a detailed, sufficiently explicit
statement of the changes in or amendments to the
 
subject measure.
 
. . . 
 
The creation of such conference committee was apparently in response to a

problem, not addressed by any constitutional provision, where the two


Striking down such argument, the Court held thus:
houses of Congress find themselves in disagreement over changes or
 
amendments introduced by the other house in a legislative bill. Given that

one of the most basic powers of the legislative branch is to formulate and Under the 'enrolled bill doctrine, the signing of a bill by the Speaker of the
House and the Senate President and the certification of the Secretaries of
implement its own rules of proceedings and to discipline its members, may both Houses of Congress that it was passed are conclusive of its due
enactment. A review of cases reveals the Court's consistent adherence to
the Court then delve into the details of how Congress complies with its the rule. The Court finds no reason to deviate from the salutary rule
in this case where the irregularities alleged by the petitioners
internal rules or how it conducts its business of passing legislation? Note mostly involved the internal rules of Congress, e.g., creation of the
2nd or 3rd Bicameral Conference Committee by the House. This Court
that in the present petitions, the issue is not whether provisions of the
is not the proper forum for the enforcement of these internal rules
rules of both houses creating the bicameral conference committee are of Congress, whether House or Senate. Parliamentary rules are
merely procedural and with their observance the courts have no
unconstitutional, but whether the bicameral conference committee concern. Whatever doubts there may be as to the formal validity of
Rep. Act No. 9006 must be resolved in its favor. The Court reiterates
has strictly complied with the rules of both houses, thereby its ruling in Arroyo vs. De Venecia, viz.:

remaining within the jurisdiction conferred upon it by Congress. 


 
But the cases, both here and
  abroad, in varying forms of
expression, all deny to the courts
the power to inquire into
In the recent case of Farias vs. The Executive Secretary, [20] the Court En allegations that, in enacting a law,
a House of Congress failed to
Banc, unanimously reiterated and emphasized its adherence to the comply with its own rules, in the
absence of showing that there was
'enrolled bill doctrine, thus, declining therein petitioners' plea for the Court a violation of a constitutional
provision or the rights of private
to go behind the enrolled copy of the bill. Assailed in said case was individuals. In Osmea v. Pendatun, it
was held: 'At any rate, courts have
Congress's creation of two sets of bicameral conference committees, the
declared that 'the rules adopted by
lack of records of said committees' proceedings, the alleged violation of deliberative bodies are subject to
revocation, modification or waiver at
said committees of the rules of both houses, and the disappearance or the pleasure of the body adopting
them. And it has been said that
deletion of one of the provisions in the compromise bill submitted by the 'Parliamentary rules are merely
procedural, and with their
bicameral conference committee. It was argued that such irregularities in observance, the courts have no
concern. They may be waived or
the passage of the law nullified R.A. No. 9006, or the Fair Election Act.  disregarded by the legislative body.
Consequently, 'mere failure to
conform to parliamentary usage
will not invalidate the action (taken Moreover, as far back as 1994 or more than ten years ago, in the case
by a deliberative body) when the
requisite number of members have of Tolentino vs. Secretary of Finance, [23] the Court already made the
agreed to a particular
pronouncement that '[i]f a change is desired in the practice [of the
measure. [21] (Emphasis supplied)
Bicameral Conference Committee] it must be sought in Congress
 
since this question is not covered by any constitutional provision

  but is only an internal rule of each house. [24] To date, Congress has

not seen it fit to make such changes adverted to by the Court. It seems,
The foregoing declaration is exactly in point with the present cases, where
therefore, that Congress finds the practices of the bicameral conference
petitioners allege irregularities committed by the conference committee in
committee to be very useful for purposes of prompt and efficient legislative
introducing changes or deleting provisions in the House and Senate bills.
action.
Akin to the Farias case, [22] the present petitions also raise an issue
 
regarding the actions taken by the conference committee on matters

regarding Congress' compliance with its own internal rules. As stated


Nevertheless, just to put minds at ease that no blatant irregularities tainted
earlier, one of the most basic and inherent power of the legislature is the
the proceedings of the bicameral conference committees, the Court deems
power to formulate rules for its proceedings and the discipline of its
it necessary to dwell on the issue. The Court observes that there was a
members. Congress is the best judge of how it should conduct its own
necessity for a conference committee because a comparison of the
business' expeditiously and in the most orderly manner. It is also the sole 
provisions of House Bill Nos. 3555 and 3705 on one hand, and Senate Bill

No. 1950 on the other, reveals that there were indeed disagreements. As
concern of Congress to instill discipline among the members of its
pointed out in the petitions, said disagreements were as follows: 
conference committee if it believes that said members violated any of its

rules of proceedings. Even the expanded jurisdiction of this Court cannot          

apply to questions regarding only the internal operation of Congress, thus,


House Bill No. House Bill Senate Bill No.
the Court is wont to deny a review of the internal proceedings of a co-equal 3555 No.3705 1950

branch of government.   
 
 
petroleum products companies and
shall be absorbed by services of
With regard to 'Stand-By Authority in favor of President
generation transmission
companies or companies and
  sellers, respectively, distribution
Provides for 12%   Provides for 12%   Provides for a single and shall not be companies, as well as
VAT on every sale VAT in general on rate of 10% VAT on passed on to those of franchise
of goods or sales of goods or sale of goods or consumers grantees of electric
properties properties and properties (amending utilities shall not
(amending Sec. reduced rates for Sec. 106 of NIRC), apply to residential 
106 of NIRC); sale of certain locally 10% VAT on sale of
12% VAT on manufactured goods services including
end-users. VAT shall
importation of and petroleum sale of electricity by
be absorbed by
goods (amending products and raw generation
generation,
Sec. 107 of materials to be used companies,
transmission, and
NIRC); and 12% in the manufacture transmission and
distribution
VAT on sale of thereof (amending distribution
companies.
services and use Sec. 106 of NIRC); companies, and use
With regard to 70% limit on input tax credit
or lease of 12% VAT on or lease of properties
properties importation of goods (amending Sec. 108
(amending Sec. and reduced rates of NIRC)  
108 of NIRC) for certain imported Provides that the   No similar provision    Provides that the
products including input tax credit input tax credit for
petroleum products for capital goods capital goods on
(amending Sec. 107 on which a VAT which a VAT has
of NIRC); and 12% has been paid been paid shall be
VAT on sale of shall be equally equally distributed
services and use or distributed over 5 over 5 years or the
lease of properties years or the depreciable life of
and a reduced rate depreciable life of such capital goods;
for certain services such capital the input tax credit
including power goods; the input for goods and
generation tax credit for services other than
(amending Sec. 108 goods and capital goods shall
of NIRC) services other not exceed 90% of
  than capital the output VAT.
goods shall not
exceed 5% of the
 
total amount of
such goods and
With regard to the 'no pass-on provision services; and for
persons engaged
  in retail trading of
No similar   Provides that the   Provides that the VAT goods, the
provision  VAT imposed on imposed on sales of allowable input
power generation electricity by tax credit shall
and on the sale of generation not exceed 11%
of the total
amount of goods
purchased. There being differences and/or disagreements on the foregoing provisions

of the House and Senate bills, the Bicameral Conference Committee was
 
mandated by the rules of both houses of Congress to act on the same by
 
settling said differences and/or disagreements. The Bicameral Conference

With regard to amendments to be made to NIRC provisions Committee acted on the disagreeing provisions by making the following
regarding income and excise taxes
changes:

 
No similar   No similar provision   Provided for  
provision amendments to
several NIRC 1. With regard to the disagreement on the rate of VAT to be imposed, it
provisions regarding would appear from the Conference Committee Report that the Bicameral
corporate income, Conference Committee tried to bridge the gap in the difference between
percentage, franchise the 10% VAT rate proposed by the Senate, and the various rates with 12%
and excise taxes as the highest VAT rate proposed by the House, by striking a compromise
whereby the present 10% VAT rate would be retained until certain
  conditions arise, i.e., the value-added tax collection as a percentage of
gross domestic product (GDP) of the previous year exceeds 2 4/5%, or
National Government deficit as a percentage of GDP of the previous year
  exceeds 1%, when the President, upon recommendation of the Secretary
of Finance shall raise the rate of VAT to 12% effective January 1, 2006. 

The disagreements between the provisions in the House bills and the
 
Senate bill were with regard to (1) what rate of VAT is to be imposed; (2)
2. With regard to the disagreement on whether only the VAT imposed on
whether only the VAT imposed on electricity generation, transmission and electricity generation, transmission and distribution companies should not
be passed on to consumers or whether both the VAT imposed on electricity
distribution companies should not be passed on to consumers, as proposed generation, transmission and distribution companies and the VAT imposed
on sale of petroleum products may be passed on to consumers, the
in the Senate bill, or both the VAT imposed on electricity generation,
Bicameral Conference Committee chose to settle such disagreement by
transmission and distribution companies and the VAT imposed on sale of altogether deleting from its Report any no pass-on provision.

petroleum products should not be passed on to consumers, as proposed in  


the House bill; (3) in what manner input tax credits should be limited; (4)
3. With regard to the disagreement on whether input tax credits should be
and whether the NIRC provisions on corporate income taxes, percentage, limited or not, the Bicameral Conference Committee decided to adopt the
position of the House by putting a limitation on the amount of input tax
franchise and excise taxes should be amended.  that may be credited against the output tax, although it crafted its own
language as to the amount of the limitation on input tax credits and the
  manner of computing the same by providing thus:
  adopted the provisions found in Senate Bill No. 1950, with some changes
(A) Creditable Input Tax. ' . . . as to the rate of the tax to be imposed.
 
. . . 
 
 
Provided, The input tax on goods
purchased or imported in a calendar
month for use in trade or business for Under the provisions of both the Rules of the House of Representatives and
which deduction for depreciation is
Senate Rules, the Bicameral Conference Committee is mandated to settle
allowed under this Code, shall be
spread evenly over the month of the differences between the disagreeing provisions in the House bill and
acquisition and the fifty-nine (59)
succeeding months if the aggregate the Senate bill. The term settle is synonymous to 'reconcile and
acquisition cost for such goods,
excluding the VAT component thereof, 'harmonize. [25] To reconcile or harmonize disagreeing provisions, the
exceeds one million Pesos
(P1,000,000.00): PROVIDED, however, Bicameral Conference Committee may then (a) adopt the specific
that if the estimated useful life of the
capital good is less than five (5) years, provisions of either the House bill or Senate bill, (b) decide that neither
as used for depreciation purposes, then
the input VAT shall be spread over such provisions in the House bill or the provisions in the Senate bill would 
shorter period: . . .
 
(B) Excess Output or Input Tax. ' If at be carried into the final form of the bill, and/or (c) try to arrive at a
the end of any taxable quarter the
output tax exceeds the input tax, the compromise between the disagreeing provisions.
excess shall be paid by the VAT-
registered person. If the input tax  
exceeds the output tax, the excess shall
be carried over to the succeeding
quarter or quarters: PROVIDED that the
 
input tax inclusive of input VAT carried
over from the previous quarter that
may be credited in every quarter shall
not exceed seventy percent (70%) of In the present case, the changes introduced by the Bicameral Conference
the output VAT: PROVIDED, HOWEVER,
THAT any input tax attributable to zero- Committee on disagreeing provisions were meant only to reconcile and
rated sales by a VAT-registered person
harmonize the disagreeing provisions for it did not inject any idea or intent
may at his option be refunded or
credited against other internal revenue that is wholly foreign to the subject embraced by the original provisions. 
taxes, . . .
 
   

4. With regard to the amendments to other provisions of the NIRC on


The so-called stand-by authority in favor of the President, whereby the rate
corporate income tax, franchise, percentage and excise taxes, the
conference committee decided to include such amendments and basically
of 10% VAT wanted by the Senate is retained until such time that certain
conditions arise when the 12% VAT wanted by the House shall be imposed, the change introduced by the Bicameral Conference Committee was totally

appears to be a compromise to try to bridge the difference in the rate of within the intent of both houses' to 'put 'a cap on input tax that may be 

VAT proposed by the two houses of Congress. Nevertheless, such


credited against the output tax. From the inception of the subject revenue
compromise is still totally within the subject of what rate of VAT should be
bill in the House of Representatives, one of the major objectives was to
imposed on taxpayers. 
'plug a glaring loophole in the tax policy and administration by creating
 
vital restrictions on the claiming of input VAT tax credits . . . and '[b]y

introducing limitations on the claiming of tax credit, we are capping a


The no pass-on provision was deleted altogether. In the transcripts of the
major leakage that has placed our collection efforts at an apparent
proceedings of the Bicameral Conference Committee held on May 10, 2005,
disadvantage. [28]
Sen. Ralph Recto, Chairman of the Senate Panel, explained the reason for

deleting the no pass-on provision in this wise:  

 
As to the amendments to NIRC provisions on taxes other than the value-
. . . the thinking was just to keep the VAT law or the VAT bill simple. And added tax proposed in Senate Bill No. 1950, since said provisions were
we were thinking that no sector should be a beneficiary of legislative grace,
neither should any sector be discriminated on. The VAT is an indirect among those referred to it, the conference committee had to act on the
tax. It is a pass on-tax. And let's keep it plain and simple. Let's not
confuse the bill and put a no pass-on provision. Two-thirds of the world same and it basically adopted the version of the Senate.
have a VAT system and in this two-thirds of the globe, I have yet to see a
VAT with a no pass-though provision. So, the thinking of the Senate is  
basically simple, let's keep the VAT simple. [26] (Emphasis supplied)

Thus, all the changes or modifications made by the Bicameral Conference


Rep. Teodoro Locsin further made the manifestation that the no pass-
Committee were germane to subjects of the provisions' referred 
on provision 'never really enjoyed the support of either House. [27]

  to it for reconciliation. Such being the case, the Court does not see any

grave abuse of discretion amounting to lack or excess of jurisdiction


With regard to the amount of input tax to be credited against output tax,
committed by the Bicameral Conference Committee. In the earlier cases
the Bicameral Conference Committee came to a compromise on the
of Philippine Judges Association vs. Prado [29] and Tolentino vs. Secretary
percentage rate of the limitation or cap on such input tax credit, but again,
of Finance, [30] the Court recognized the long-standing legislative practice
emergency. Upon the last reading of a bill, no
of giving said conference committee ample latitude for compromising amendment thereto shall be allowed, and the vote
thereon shall be taken immediately thereafter, and the
differences' between the Senate and the House. Thus, in yeas and nays entered in the Journal. 
the Tolentino case, it was held that:
 
 
 
. . . it is within the power of a conference committee to
include in its report an entirely new provision that is not
found either in the House bill or in the Senate bill. If the Petitioners' argument that the practice where a bicameral conference
committee can propose an amendment consisting of one
committee is allowed to add or delete provisions in the House bill and the
or two provisions, there is no reason why it cannot
propose several provisions, collectively considered as an
Senate bill after these had passed three readings is in effect a
'amendment in the nature of a substitute, so long as such
amendment is germane to the subject of the bills before circumvention of the 'no amendment rule (Sec. 26 (2), Art. VI of the 1987
the committee. After all, its report was not final but
needed the approval of both houses of Congress to Constitution), fails to convince the Court to deviate from its ruling in
become valid as an act of the legislative department. The
charge that in this case the Conference Committee the Tolentino case that:
acted as a third legislative chamber is thus without
any basis.[31] (Emphasis supplied)  
 
 
B. R.A. No. 9337 Does Not Violate Nor is there any reason for requiring that the
Article VI, Section 26(2) of the Committee's Report in these cases must have undergone
Constitution on the 'No- three readings in each of the two houses. If that be the
Amendment Rule case, there would be no end to negotiation since each
house may seek modification of the compromise bill. . . .
 
  Art. VI. ' 26 (2) must, therefore, be construed as
referring only to bills introduced for the first time
  in either house of Congress, not to the conference
committee report. [32] (Emphasis supplied)

Article VI, Sec. 26 (2) of the Constitution, states:   

   

No bill passed by either House shall become a law unless


it has passed three readings on separate days, and The Court reiterates here that the no-amendment rule refers only to
printed copies thereof in its final form have been
distributed to its Members three days before its passage, the procedure to be followed by each house of Congress with
except when the President certifies to the necessity of its
immediate enactment to meet a public calamity or regard to bills initiated in each of said respective houses, before
28(B) Inter-corporate Dividends
said bill is transmitted to the other house for its concurrence or (1)

amendment. Verily, to construe said provision in a way as to proscribe 34(B) Inter-corporate Dividends
(1)
any further changes to a bill after one house has voted on it would lead to
116 Tax on Persons Exempt
absurdity as this would mean that the other house of Congress would be from VAT

deprived of its constitutional power to amend or introduce changes to said 117 Percentage Tax on
domestic carriers and
bill. Thus, Art. VI, Sec. 26 (2) of the Constitution cannot be taken to mean keepers of Garage
119 Tax on franchises
that the introduction by the Bicameral Conference Committee of
121 Tax on banks and Non-
amendments and modifications to disagreeing provisions in bills that have Bank Financial
Intermediaries
been acted upon by both houses of Congress is prohibited. 
148 Excise Tax on
  manufactured oils and
other fuels

C. R.A. No. 9337 Does Not Violate 151 Excise Tax on mineral
Article VI, Section 24 of the products
Constitution on Exclusive
236 Registration requirements
Origination of Revenue Bills
237 Issuance of receipts or
  sales or commercial
invoices

  288 Disposition of
Incremental Revenue

Coming to the issue of the validity of the amendments made regarding the
 
NIRC provisions on corporate income taxes and percentage, excise taxes.
 
Petitioners refer to the following provisions, to wit:

  Petitioners claim that the amendments to these provisions of the NIRC did

not at all originate from the House. They aver that House Bill No. 3555
Section  
27
Rates of Income Tax on proposed amendments only regarding Sections 106, 107, 108, 110 and
Domestic Corporation
114 of the NIRC, while House Bill No. 3705 proposed amendments only to
28(A)(1) Tax on Resident Foreign
Corporation Sections 106, 107,108, 109, 110 and 111 of the NIRC; thus, the other
sections of the NIRC which the Senate amended but which amendments introduction by the Senate of provisions not dealing directly with the value-

were not found in the House bills are not intended to be amended by the added tax, which is the only kind of tax being amended in the House bills,

House of Representatives. Hence, they argue that since the proposed still within the purview of the constitutional provision authorizing the

amendments did not originate from the House, such amendments are a Senate to propose or concur with amendments to a revenue bill that

violation of Article VI, Section 24 of the Constitution.  originated from the House? 

   

The argument does not hold water. The foregoing question had been squarely answered in the Tolentino case,

wherein the Court held, thus:


 

 
Article VI, Section 24 of the Constitution reads:
. . . To begin with, it is not the law ' but the revenue bill '
which is required by the Constitution to 'originate
 
exclusively in the House of Representatives. It is
important to emphasize this, because a bill originating in
Sec. 24. All appropriation, revenue or tariff bills, bills the House may undergo such extensive changes in the
authorizing increase of the public debt, bills of local Senate that the result may be a rewriting of the whole. . .
application, and private bills shall originate exclusively in . At this point, what is important to note is that, as a
the House of Representatives but the Senate may result of the Senate action, a distinct bill may be
propose or concur with amendments. produced. To insist that a revenue statute ' and not
only the bill which initiated the legislative process
  culminating in the enactment of the law must
substantially be the same as the House bill would
be to deny the Senate's power not only to 'concur
  with amendments but also to 'propose
amendments. It would be to violate the coequality of
legislative power of the two houses of Congress and in
In the present cases, petitioners admit that it was indeed House Bill Nos. fact make the House superior to the Senate.
 
3555 and 3705 that initiated the move for amending provisions of the NIRC  
Given, then, the power of the Senate to propose
dealing mainly with the value-added tax. Upon transmittal of said House amendments, the Senate can propose its own
version even with respect to bills which are
bills to the Senate, the Senate came out with Senate Bill No. 1950
required by the Constitution to originate in the
House.
proposing amendments not only to NIRC provisions on the value-added tax
...
but also amendments to NIRC provisions on other kinds of taxes. Is the  
Indeed, what the Constitution simply means is that the
initiative for filing revenue, tariff or tax bills, bills introduced on the floor, which was later substituted by House Bill No. 3555,
authorizing an increase of the public debt, private bills
and bills of local application must come from the House of stated:
Representatives on the theory that, elected as they are
from the districts, the members of the House can be  
expected to be more sensitive to the local needs
and problems. On the other hand, the senators,
One of the challenges faced by the present administration
who are elected at large, are expected to approach
is the urgent and daunting task of solving the country's
the same problems from the national perspective.
serious financial problems. To do this, government
Both views are thereby made to bear on the
expenditures must be strictly monitored and controlled
enactment of such laws. [33] (Emphasis supplied)
and revenues must be significantly increased. This may
be easier said than done, but our fiscal authorities are
  still optimistic the government will be operating on a
balanced budget by the year 2009. In fact, several
  measures that will result to significant expenditure
savings have been identified by the administration. It is
supported with a credible package of revenue
measures that include measures to improve tax
Since there is no question that the revenue bill exclusively originated in the administration and control the leakages in
revenues from income taxes and the value-added
House of Representatives, the Senate was acting within its 
tax (VAT). (Emphasis supplied)

constitutional power to introduce amendments to the House bill when it  

included provisions in Senate Bill No. 1950 amending corporate income  


taxes, percentage, excise and franchise taxes. Verily, Article VI, Section 24

of the Constitution does not contain any prohibition or limitation on the Rep. Eric D. Singson, in his sponsorship speech for House Bill No. 3555,

extent of the amendments that may be introduced by the Senate to the declared that:

House revenue bill.


 

 
In the budget message of our President in the year 2005,
she reiterated that we all acknowledged that on top of
our agenda must be the restoration of the health of our
Furthermore, the amendments introduced by the Senate to the NIRC fiscal system. 
 
provisions that had not been touched in the House bills are still in In order to considerably lower the consolidated public
sector deficit and eventually achieve a balanced budget
furtherance of the intent of the House in initiating the subject revenue bills.
by the year 2009, we need to seize windows of
The Explanatory Note of House Bill No. 1468, the very first House bill opportunities which might seem poignant in the
beginning, but in the long run prove effective and
beneficial to the overall status of our economy. One
such opportunity is a review of existing tax rates, What we therefore prescribe is a burden sharing between
evaluating the relevance given our present corporate Philippines and the consumer. Why should the
conditions. [34] (Emphasis supplied) latter bear all the pain? Why should the fiscal salvation
be only on the burden of the consumer?
 
 
The corporate world's equity is in form of the increase in
the corporate income tax from 32 to 35 percent, but up
  to 2008 only. This will raise P10.5 billion a year. After
that, the rate will slide back, not to its old rate of 32
percent, but two notches lower, to 30 percent.
Notably therefore, the main purpose of the bills emanating from the House  
Clearly, we are telling those with the capacity to pay,
of Representatives is to bring in sizeable revenues for the government  corporations, to bear with this emergency provision that
will be in effect for 1,200 days, while we put our fiscal
house in order. This fiscal medicine will have an expiry
to supplement our country's serious financial problems, and improve tax date.
 
administration and control of the leakages in revenues from income taxes For their assistance, a reward of tax reduction awaits
them. We intend to keep the length of their sacrifice
and value-added taxes. As these house bills were transmitted to the brief. We would like to assure them that not because
there is a light at the end of the tunnel, this government
Senate, the latter, approaching the measures from the point of national will keep on making the tunnel long.
 
perspective, can introduce amendments within the purposes of those bills. The responsibility will not rest solely on the weary
shoulders of the small man. Big business will be there to
It can provide for ways that would soften the impact of the VAT measure
share the burden. [35]
on the consumer, i.e., by distributing the burden across all sectors instead
 
of putting it entirely on the shoulders of the consumers. The sponsorship

speech of Sen. Ralph Recto on why the provisions on income tax on  

corporation were included is worth quoting:


As the Court has said, the Senate can propose amendments and in fact,
 
the amendments made on provisions in the tax on income of corporations

All in all, the proposal of the Senate Committee on Ways are germane to the purpose of the house bills which is to raise revenues for
and Means will raise P64.3 billion in additional revenues
annually even while by mitigating prices of power, the government. 
services and petroleum products.
   
However, not all of this will be wrung out of VAT. In fact,
only P48.7 billion amount is from the VAT on twelve
goods and services. The rest of the tab ' P10.5 billion-  
will be picked by corporations.
 
Likewise, the Court finds the sections referring to other percentage and The other sections amended by the Senate pertained to matters of tax

excise taxes germane to the reforms to the VAT system, as these sections administration which are necessary for the implementation of the changes

would cushion the effects of VAT on consumers. Considering that certain in the VAT system. 

goods and services which were subject to percentage tax and excise tax
 
would no longer be VAT-exempt, the consumer would be burdened more as

they would be paying the VAT in addition to these taxes. Thus, there is a To reiterate, the sections introduced by the Senate are germane to the

need to amend these sections to soften the impact of VAT. Again, in his subject matter and purposes of the house bills, which is to supplement our

sponsorship speech, Sen. Recto said: country's fiscal deficit, among others. Thus, the Senate acted within its

power to propose those amendments. 


 

 
However, for power plants that run on oil, we will reduce to zero the
present excise tax on bunker fuel, to lessen the effect of a VAT on this
product. SUBSTANTIVE ISSUES

  I.
For electric utilities like Meralco, we will wipe out the
franchise tax in exchange for a VAT.
Whether Sections 4, 5 and 6 of R.A. No. 9337, amending Sections 106, 107
 
and 108 of the NIRC, violate the following provisions of the Constitution:
And in the case of petroleum, while we will levy the VAT
on oil products, so as not to destroy the VAT chain, we
will however bring down the excise tax on socially  
sensitive products such as diesel, bunker, fuel and
kerosene. a. Article VI, Section 28(1), and
 
...
  b. Article VI, Section 28(2)
What do all these exercises point to? These are not
contortions of giving to the left hand what was taken A. No Undue Delegation of
from the right. Rather, these sprang from our concern of Legislative Power
softening the impact of VAT, so that the people can
cushion the blow of higher prices they will have to pay as
a result of VAT. [36]  

   
(GDP) of the previous year
Petitioners ABAKADA GURO Party List, et al., Pimentel, Jr., et al., and exceeds two and four-fifth
percent (2 4/5%) or
Escudero,et al. contend in common that Sections 4, 5 and 6 of R.A. No.  
(ii) national government deficit as
9337, amending Sections 106, 107 and 108, respectively, of the NIRC
a percentage of GDP of the
giving the President the stand-by authority to raise the VAT rate from 10% previous year exceeds one
and one-half percent (1
to 12% when a certain condition is met, constitutes undue delegation of '%).
 
the legislative power to tax.  SEC. 5. Section 107 of the same Code, as amended, is
hereby further amended to read as follows:
 
 
SEC. 107. Value-Added Tax on Importation of Goods. ' 
(A) In General. ' There shall be levied,
assessed and collected on every
The assailed provisions read as follows: importation of goods a value-added tax
equivalent to ten percent (10%) based
  on the total value used by the Bureau
of Customs in determining tariff and
customs duties, plus customs duties,
SEC. 4. Sec. 106 of the same Code, as amended, is excise taxes, if any, and other charges,
hereby further amended to read as follows: such tax to be paid by the importer
  prior to the release of such goods from
SEC. 106. Value-Added Tax on Sale of Goods or customs custody: Provided, That where
Properties. ' the customs duties are determined on
  the basis of the quantity or volume of
(A) Rate and Base of Tax. ' There shall the goods, the value-added tax shall be
be levied, assessed and collected on based on the landed cost plus excise
every sale, barter or exchange of goods taxes, if any: provided, further, that
or properties, a value-added tax the President, upon the
equivalent to ten percent (10%) of the recommendation of the Secretary
gross selling price or gross value in of Finance, shall, effective January
money of the goods or properties sold, 1, 2006, raise the rate of value-
bartered or exchanged, such tax to be added tax to twelve percent (12%)
paid by the seller or after any of the following
transferor: provided, that the conditions has been satisfied.
President, upon the  
recommendation of the Secretary (i) value-added tax collection as a
of Finance, shall, effective January percentage of Gross
1, 2006, raise the rate of value- Domestic Product (GDP) of
added tax to twelve percent (12%), the previous year exceeds
after any of the following two and four-fifth percent
conditions has been satisfied. (2 4/5%) or
  (ii) national government deficit as
(i)                 value-added tax a percentage of GDP of the
collection as a percentage previous year exceeds one
of Gross Domestic Product
and one-half percent (1
'%). power to tax because such delegation is not within the purview of Section
 
28 (2), Article VI of the Constitution, which provides:

SEC. 6. Section 108 of the same Code, as amended, is hereby further


amended to read as follows:  

  The Congress may, by law, authorize the President to fix within specified
limits, and may impose, tariff rates, import and export quotas, tonnage
and wharfage dues, and other duties or imposts within the framework of
SEC. 108. Value-added Tax on Sale of the national development program of the government.
Services and Use or Lease of Properties
'
 
 
(A) Rate and Base of Tax. ' There shall
 
be levied, assessed and collected, a
value-added tax equivalent to ten
percent (10%) of gross receipts derived
from the sale or exchange of They argue that the VAT is a tax levied on the sale, barter or exchange of
services: provided, that the
President, upon the goods and properties as well as on the sale or exchange of services, which
recommendation of the Secretary
cannot be included within the purview of tariffs under the exempted
of Finance, shall, effective January
1, 2006, raise the rate of value- delegation as the latter refers to customs duties, tolls or tribute payable
added tax to twelve percent (12%),
after any of the following upon merchandise to the government and usually imposed on goods or
conditions has been satisfied.
  merchandise imported or exported. 
(i) value-added tax collection as a
percentage of Gross
 
Domestic Product (GDP) of
the previous year exceeds
two and four-fifth percent
(2 4/5%) or Petitioners ABAKADA GURO Party List, et al., further contend that
(ii) national government deficit as
delegating to the President the legislative power to tax is contrary to
a percentage of GDP of the
previous year exceeds one
republicanism. They insist that accountability, responsibility and
and one-half percent (1
'%). (Emphasis supplied) transparency should dictate the actions of Congress and they should not

  pass to the President the decision to impose taxes. They also argue that

the law also effectively nullified the President's power of control, which

Petitioners allege that the grant of the stand-by authority to the President includes the authority to set aside and nullify the acts of her subordinates

to increase the VAT rate is a virtual abdication by Congress of its exclusive


like the Secretary of Finance, by mandating the fixing of the tax rate by the The principle of separation of powers ordains that each of the three great

President upon the recommendation of the Secretary of Finance. branches of government has exclusive cognizance of and is supreme in

matters' falling within its' own constitutionally allocated sphere. [37] A


 
logical 

Petitioners Pimentel, et al. aver that the President has ample powers to


corollary to the doctrine of separation of powers is the principle of non-
cause, influence or create the conditions provided by the law to bring about
delegation of powers, as expressed in the Latin maxim: potestas delegata
either or both the conditions precedent.
non delegari potest which means 'what has been delegated, cannot be
 
delegated.[38] This doctrine is based on the ethical principle that such as

delegated power constitutes not only a right but a duty to be performed by


On the other hand, petitioners Escudero, et al. find bizarre and revolting
the delegate through the instrumentality of his own judgment and not
the situation that the imposition of the 12% rate would be subject to the
through the intervening mind of another. [39]
whim of the Secretary of Finance, an unelected bureaucrat, contrary to the

principle of no taxation without representation. They submit that the  

Secretary of Finance is not mandated to give a favorable recommendation


With respect to the Legislature, Section 1 of Article VI of the Constitution
and he may not even give his recommendation. Moreover, they allege that
provides that 'the Legislative power shall be vested in the Congress of
no guiding standards are provided in the law on what basis and as to how
the Philippines which shall consist of a Senate and a House of
he will make his recommendation. They claim, nonetheless, that any
Representatives . The powers which Congress is prohibited from delegating
recommendation of the Secretary of Finance can easily be brushed aside by
are those which are strictly, or inherently and exclusively, legislative.
the President since the former is a mere alter ego of the latter, such that,
Purely legislative power, which can never be delegated, has been described
ultimately, it is the President who decides whether to impose the increased
as the authority to make a complete law ' complete as to the time
tax rate or not.
when it shall take effect and as to whom it shall be applicable ' and
 
to determine the expediency of its enactment. [40] Thus, the rule is

that in order that a court may be justified in holding a statute


A brief discourse on the principle of non-delegation of powers is instructive.
unconstitutional as a delegation of legislative power, it must appear that
 
the power involved is purely legislative in nature ' that is, one appertaining
exclusively to the legislative department. It is the nature of the power, and one which defines legislative policy, marks its limits, maps out its

not the liability of its use or the manner of its exercise, which determines boundaries and specifies the public agency to apply it. It indicates the

the validity of its delegation.  circumstances under which the legislative command is to be

effected. [43] Both tests are intended to prevent a total transference of


 
legislative authority to the delegate, who is not allowed to step into the

Nonetheless, the general rule barring delegation of legislative powers is shoes of the legislature and exercise a power essentially legislative.[44]

subject to the following recognized limitations or exceptions:  

 
In People vs. Vera, [45] the Court, through eminent Justice Jose P. Laurel,
(1) Delegation of tariff powers to the President under Section 28
(2) of Article VI of the Constitution; expounded on the concept and extent of delegation of power in this wise:

(2) Delegation of emergency powers to the President under  


Section 23 (2) of Article VI of the Constitution;
In testing whether a statute constitutes an undue
(3) Delegation to the people at large; delegation of legislative power or not, it is usual to
inquire whether the statute was complete in all its terms
and provisions when it left the hands of the legislature so
(4) Delegation to local governments; and that nothing was left to the judgment of any other
appointee or delegate of the legislature.
(5) Delegation to administrative bodies.  
...
 
  The true distinction', says Judge Ranney, 'is
between the delegation of power to make the law,
  which necessarily involves a discretion as to what it
shall be, and conferring an authority or discretion
as to its execution, to be exercised under and in
In every case of permissible delegation, there must be a showing that the pursuance of the law. The first cannot be done; to
the latter no valid objection can be made.
delegation itself is valid. It is valid only if the law (a) is complete in itself,  
...
setting forth therein the policy to be executed, carried out, or implemented  
It is contended, however, that a legislative act may be
by the delegate; [41] and (b) fixes a standard ' the limits of which are made to the effect as law after it leaves the hands of the
legislature. It is true that laws may be made effective on
sufficiently determinate and determinable ' to which the delegate must certain contingencies, as by proclamation of the
executive or the adoption by the people of a particular
conform in the performance of his functions. [42] A sufficient standard is community. In Wayman vs. Southard, the Supreme Court
of the United States ruled that the legislature may  
delegate a power not legislative which it may itself
rightfully exercise. The power to ascertain facts is
such a power which may be delegated. There is In Edu vs. Ericta, [47] the Court reiterated:
nothing essentially legislative in ascertaining the
existence of facts or conditions as the basis of the
taking into effect of a law. That is a mental process  
common to all branches of the
government. Notwithstanding the apparent tendency, What cannot be delegated is the authority under the
however, to relax the rule prohibiting delegation of Constitution to make laws and to alter and repeal them;
legislative authority on account of the complexity arising the test is the completeness of the statute in all its terms
from social and economic forces at work in this modern and provisions when it leaves the hands of the
industrial age, the orthodox pronouncement of Judge legislature. To determine whether or not there is an
Cooley in his work on Constitutional Limitations finds undue delegation of legislative power, the inquiry must
restatement in Prof. Willoughby's treatise on the be directed to the scope and definiteness of the measure
Constitution of the United States in the following enacted. The legislative does not abdicate its
language ' speaking of declaration of legislative power to functions when it describes what job must be done,
administrative agencies: The principle which permits who is to do it, and what is the scope of his
the legislature to provide that the administrative authority. For a complex economy, that may be the only
agent may determine when the circumstances are way in which the legislative process can go forward. A
such as require the application of a law is defended distinction has rightfully been made between
upon the ground that at the time this authority is delegation of power to make the laws which
granted, the rule of public policy, which is the necessarily involves a discretion as to what it shall
essence of the legislative act, is determined by the be, which constitutionally may not be done, and
legislature. In other words, the legislature, as it is delegation of authority or discretion as to its
its duty to do, determines that, under given execution to be exercised under and in pursuance
circumstances, certain executive or administrative of the law, to which no valid objection can be
action is to be taken, and that, under other made. 'The Constitution is thus not to be regarded as
circumstances, different or no action at all is to be denying the legislature the necessary resources of
taken. What is thus left to the administrative flexibility and practicability. (Emphasis supplied). [48]
official is not the legislative determination of what
public policy demands, but simply the
 
ascertainment of what the facts of the case require
to be done according to the terms of the law by
which he is governed. The efficiency of an Act as a  
declaration of legislative will must, of course, come
from Congress, but the ascertainment of the
contingency upon which the Act shall take effect Clearly, the legislature may delegate to executive officers or bodies the
may be left to such agencies as it may designate.
The legislature, then, may provide that a law shall power to determine certain facts or conditions, or the happening of
take effect upon the happening of future specified
contingencies leaving to some other person or body contingencies, on which the operation of a statute is, by its terms, made to
the power to determine when the specified
contingency has arisen. (Emphasis supplied). [46] depend, but the legislature must prescribe sufficient standards, policies or

limitations on their authority. [49] While the power to tax cannot be


 
delegated to executive agencies, details as to the enforcement and
That the President, upon the recommendation of the
administration of an exercise of such power may be left to them, including Secretary of Finance, shall, effective January 1, 2006,
raise the rate of value-added tax to twelve percent
the power to determine the existence of facts on which its operation (12%), after any of the following conditions has been
satisfied:
depends. [50]
 
(i) Value-added tax collection as a
  percentage of Gross Domestic Product
(GDP) of the previous year exceeds two
and four-fifth percent (2 4/5%); or
The rationale for this is that the preliminary ascertainment of facts as basis  
(ii) National government deficit as a
for the enactment of legislation is not of itself a legislative function, but is percentage of GDP of the previous year
exceeds one and one-half percent (1
simply ancillary to legislation. Thus, the duty of correlating information and '%).

making recommendations is the kind of subsidiary activity which the


 
legislature may perform through its members, or which it may delegate to
 
others to perform. Intelligent legislation on the complicated problems of

modern society is impossible in the absence of accurate information on the The case before the Court is not a delegation of legislative power. It is
simply a delegation of ascertainment of facts upon which enforcement and
part of the legislators, and any reasonable method of securing such administration of the increase rate under the law is contingent. The
legislature has made the operation of the 12% rate effective January 1,
information is proper. [51] The Constitution as a continuously operative 2006, contingent upon a specified fact or condition. It leaves the entire
operation or non-operation of the 12% rate upon factual matters outside of
charter of government does not require that Congress find for itself 
the control of the executive. 

 
every fact upon which it desires to base legislative action or that it make

for itself detailed determinations which it has declared to be prerequisite to


No discretion would be exercised by the President. Highlighting the absence
application of legislative policy to particular facts and circumstances
of discretion is the fact that the word shall is used in the common proviso.
impossible for Congress itself properly to investigate. [52]
The use of the word shall connotes a mandatory order.  Its use in a statute
 
denotes an imperative obligation and is inconsistent with the idea of

discretion. [53] 'Where the law is clear and unambiguous, it must be taken


In the present case, the challenged section of R.A. No. 9337 is the
to mean exactly what it says, and courts have no choice but to see to it
common proviso in Sections 4, 5 and 6 which reads as follows:
that the mandate is obeyed. [54]
 
 
Thus, it is the ministerial duty of the President to immediately impose the When one speaks of the Secretary of Finance as the alter ego of the

12% rate upon the existence of any of the conditions specified by President, it simply means that as head of the Department of Finance he is

Congress. This is a duty which cannot be evaded by the President. the assistant and agent of the Chief Executive. The multifarious executive

Inasmuch as the law specifically uses the word shall, the exercise of and administrative functions of the Chief Executive are performed by and

discretion by the President does not come into play. It is a clear directive to through the executive departments, and the acts of the secretaries of such

impose the 12% VAT rate when the specified conditions are present. The departments, such as the Department of Finance, performed and

time of taking into effect of the 12% VAT rate is based on the happening of promulgated in the regular course of business, are, unless disapproved or

a certain specified contingency, or upon the ascertainment of certain facts reprobated by the Chief Executive, presumptively the acts of the Chief

or conditions by a person or body other than the legislature itself. Executive. The Secretary of Finance, as such, occupies a political position

and holds office in an advisory capacity, and, in the language of Thomas


 
Jefferson, "should be of the President's bosom confidence" and, in the

The Court finds no merit to the contention of petitioners ABAKADA language of Attorney-General Cushing, is 'subject to the direction of the

GURO Party List, et al. that the law effectively nullified the President's President." [55]

power of control over the Secretary of Finance by mandating the fixing of


 
the tax rate by the President upon the recommendation of the Secretary of

Finance. The Court cannot also subscribe to the position of petitioners   

Pimentel, et al. that the word shall should be interpreted to mean may in In the present case, in making his recommendation to the President on the
view of the phrase upon the recommendation of the Secretary of Finance. existence of either of the two conditions, the Secretary of Finance is not
Neither does the Court find persuasive the submission of petitioners acting as the alter ego of the President or even her subordinate. In such
Escudero, et al. that any recommendation by the Secretary of Finance can instance, he is not subject to the power of control and direction of the
easily be brushed aside by the President since the former is a mere alter President. He is acting as the agent of the legislative department, to
ego of the latter. determine and declare the event upon which its expressed will is to take

  effect. [56] The Secretary of Finance becomes the means or tool by which

legislative policy is determined and implemented, considering that he


possesses all the facilities to gather data and information and has a much frequently the only way in which the legislative process can go

broader perspective to properly evaluate them. His function is to gather forward. [58]

and collate statistical data and other pertinent information and verify if any
 
of the two conditions laid out by Congress is present. His personality in

such instance is in reality but a projection of that of Congress.Thus, being As to the argument of petitioners ABAKADA GURO Party List, et al. that

the agent of Congress and not of the President, the President cannot alter delegating to the President the legislative power to tax is contrary to the

or modify or nullify, or set aside the findings of the Secretary of Finance principle of republicanism, the same deserves scant consideration.

and to substitute the judgment of the former for that of the latter.  Congress did not delegate the power to tax but the mere implementation of

the law. The intent and will to increase the VAT rate to 12% came from
 
Congress and the task of the President is to simply execute the legislative

Congress simply granted the Secretary of Finance the authority to policy. That Congress chose to do so in such a manner is not within the

ascertain the existence of a fact, namely, whether by December 31, 2005, province of the Court to inquire into, its task being to interpret the

the value-added tax collection as a percentage of Gross Domestic Product law. [59]

(GDP) of the previous year exceeds two and four-fifth percent (24/5%) or
 
the national government deficit as a percentage of GDP of the previous

year exceeds one and one-half percent (1%). If either of these two The insinuation by petitioners Pimentel, et al. that the President has ample

instances has occurred, the Secretary of Finance, by legislative mandate, powers to cause, influence or create the conditions to bring about either or

must submit such information to the President. Then the 12% VAT rate both the conditions precedent does not deserve any merit as this argument

must be imposed by the President effective January 1, 2006. There is no is highly speculative. The Court does not rule on allegations which are

undue delegation of legislative power but only of the discretion as manifestly conjectural, as these may not exist at all. The Court deals with

to the execution of a law. This is constitutionally facts, not fancies; on realities, not appearances. When the Court acts on

permissible. [57] Congress does not abdicate its functions or unduly appearances instead of realities, justice and law will be short-lived.

delegate power when it describes what job must be done, who must do it,
 
and what is the scope of his authority; in our complex economy that is
B. The 12% Increase VAT Rate Does
Not Impose an Unfair and
Unnecessary Additional Tax  
Burden

  Thus, in the absence of any provision providing for a return to the 10%

rate, which in this case the Court finds none, petitioners' argument is, at
 
best, purely speculative. There is no basis for petitioners' fear of a
Petitioners Pimentel, et al. argue that the 12% increase in the VAT rate
fluctuating VAT rate because the law itself does not provide that the rate
imposes an unfair and additional tax burden on the people. Petitioners also
argue that the 12% increase, dependent on any of the 2 conditions set should go back to 10% if the conditions provided in Sections 4, 5 and 6 are
forth in the contested provisions, is ambiguous because it does not state if
the VAT rate would be returned to the original 10% if the rates are no no longer present. The rule is that where the provision of the law is clear
longer satisfied. Petitioners also argue that such rate is unfair and
unreasonable, as the people are unsure of the applicable VAT rate from and unambiguous, so that there is no occasion for the court's seeking the
year to year.
legislative intent, the law must be taken as it is, devoid of judicial addition
  or subtraction. [61]

 
Under the common provisos of Sections 4, 5 and 6 of R.A. No. 9337, if any

of the two conditions set forth therein are satisfied, the President shall
Petitioners also contend that the increase in the VAT rate, which was
increase the VAT rate to 12%. The provisions of the law are clear. It does
allegedly an incentive to the President to raise the VAT collection to at least
not provide for a return to the 10% rate nor does it empower the President
2 4/5 of the GDP of the previous year, should be based on fiscal adequacy. 
to so revert if, after the rate is increased to 12%, the VAT collection goes

below the 24/5 of the GDP of the previous year or that the national  

government deficit as a percentage of GDP of the previous year does not


Petitioners obviously overlooked that increase in VAT collection is not
exceed 1%. 
the onlycondition. There is another condition, i.e., the national government
 
deficit as a percentage of GDP of the previous year exceeds one and one-

half percent (1 '%). 


Therefore, no statutory construction or interpretation is needed.  Neither

can conditions or limitations be introduced where none is provided for.   

Rewriting the law is a forbidden ground that only Congress may tread
Respondents explained the philosophy behind these alternative conditions:
upon. [60]

 
1.      VAT/GDP Ratio > 2.8% little as possible over and above what it brings
  into the public treasury of the state. [63]
The condition set for increasing VAT rate to 12% have
economic or fiscal meaning. If VAT/GDP is less than
 
2.8%, it means that government has weak or no
capability of implementing the VAT or that VAT is not
effective in the function of the tax collection. 'Therefore,  
there is no value to increase it to 12% because such
action will also be ineffectual.
  It simply means that sources of revenues must be adequate to meet
2.      Natl Govt Deficit/GDP >1.5%
  government expenditures and their variations. [64]
The condition set for increasing VAT when deficit/GDP is
1.5% or less means the fiscal condition of government
 
has reached a relatively sound position or is towards the
direction of a balanced budget position. Therefore, there
is no need to increase the VAT rate since the fiscal house
is in a relatively healthy position. Otherwise stated, if the The dire need for revenue cannot be ignored. Our country is in a quagmire
ratio is more than 1.5%, there is indeed a need to
of financial woe. During the Bicameral Conference Committee hearing, then
increase the VAT rate. [62]
Finance Secretary Purisima bluntly depicted the country's gloomy state of
 
economic affairs, thus:
 
 

That the first condition amounts to an incentive to the President to increase First, let me explain the position that the Philippines finds
itself in right now. We are in a position where 90 percent
the VAT collection does not render it unconstitutional so long as there is a of our revenue is used for debt service. So, for every
peso of revenue that we currently raise, 90 goes to debt
public purpose for which the law was passed, which in this case, is mainly service. That's interest plus amortization of our debt. So
clearly, this is not a sustainable situation. That's the first
to raise revenue. In fact, fiscal adequacy dictated the need for a raise in
fact.
 
revenue. 
The second fact is that our debt to GDP level is way out
of line compared to other peer countries that borrow
  money from that international financial markets. Our
debt to GDP is approximately equal to our GDP. Again,
that shows you that this is not a sustainable situation.
The principle of fiscal adequacy as a characteristic of a sound tax system  
The third thing that Id like to point out is the
was originally stated by Adam Smith in his Canons of Taxation (1776), as: environment that we are presently operating in is not as
benign as what it used to be the past five years.
IV. Every tax ought to be so contrived as both to take out  
What do I mean by that?
and to keep out of the pockets of the people as
 
In the past five years, weve been lucky because we were
operating in a period of basically global growth and low the Farias case, the Court refused to consider the various arguments raised
interest rates. The past few months, we have seen an
inching up, in fact, a rapid increase in the interest rates therein that dwelt on the wisdom of Section 14 of R.A. No. 9006 (The Fair
in the leading economies of the world. And, therefore,
Election Act), pronouncing that:
our ability to borrow at reasonable prices is going to be
challenged. In fact, ultimately, the question is our ability
to access the financial markets.   
 
When the President made her speech in July last year, . . . policy matters are not the concern of the Court.
the environment was not as bad as it is now, at least Government policy is within the exclusive dominion of the
based on the forecast of most financial institutions. So, political branches of the government. It is not for this
we were assuming that raising 80 billion would put us in Court to look into the wisdom or propriety of legislative
a position where we can then convince them to improve determination. Indeed, whether an enactment is wise or
our ability to borrow at lower rates. But conditions have unwise, whether it is based on sound economic theory,
changed on us because the interest rates have gone up. whether it is the best means to achieve the desired
'In fact, just within this room, we tried to access the results, whether, in short, the legislative discretion within
market for a billion dollars because for this year alone, its prescribed limits should be exercised in a particular
the Philippines will have to borrow 4 billion dollars. Of manner are matters for the judgment of the legislature,
that amount, we have borrowed 1.5 billion. We issued and the serious conflict of opinions does not suffice to
last January a 25-year bond at 9.7 percent cost. We were bring them within the range of judicial cognizance. [66]
trying to access last week and the market was not as
favorable and up to now we have not accessed and we
might pull back because the conditions are not very  
good.
   
So given this situation, we at the Department of Finance
believe that we really need to front-end our deficit
reduction. Because it is deficit that is causing the
In the same vein, the Court in this case will not dawdle on the purpose of
increase of the debt and we are in what we call a debt
spiral. The more debt you have, the more deficit you Congress or the executive policy, given that it is not for the judiciary to
have because interest and debt service eats and eats
more of your revenue. We need to get out of this debt "pass upon questions of wisdom, justice or expediency of legislation. [67]
spiral. And the only way, I think, we can get out of this
debt spiral is really have a front-end adjustment in our
revenue base.[65]  

  II.

  Whether Section 8 of R.A. No. 9337, amending Sections 110(A)(2) and


110(B) of the NIRC; and Section 12 of R.A. No. 9337, amending Section
114(C) of the NIRC, violate the following provisions of the Constitution:
The image portrayed is chilling. Congress passed the law hoping for rescue
 
from an inevitable catastrophe. Whether the law is indeed sufficient to
a. Article VI, Section 28(1), and
answer the state's economic dilemma is not for the Court to judge. In
b. Article III, Section 1  

 
Section 8 of R.A. No. 9337, amending Section 110(B) of the NIRC imposes
  a limitation on the amount of input tax that may be credited against the

A. Due Process and Equal Protection Clauses output tax. It states, in part: '[P]rovided, that the input tax inclusive of the

input VAT carried over from the previous quarter that may be credited in
 
every quarter shall not exceed seventy percent (70%) of the output VAT: 
 
 

Petitioners Association of Pilipinas Shell Dealers, Inc., et al. argue that


Input Tax is defined under Section 110(A) of the NIRC, as amended, as the
Section 8 of R.A. No. 9337, amending Sections 110 (A)(2), 110 (B), and
value-added tax due from or paid by a VAT-registered person on the
Section 12 of R.A. No. 9337, amending Section 114 (C) of the NIRC are
importation of goods or local purchase of good and services, including lease
arbitrary, oppressive, excessive and confiscatory. Their argument is
or use of property, in the course of trade or business, from a VAT-
premised on the constitutional right against deprivation of life, liberty of
registered person, and Output Tax is the value-added tax due on the sale
property without due process of law, as embodied in Article III, Section 1
or lease of taxable goods or properties or services by any person registered
of the Constitution.
or required to register under the law.
 
 

Petitioners also contend that these provisions violate the constitutional


 
guarantee of equal protection of the law.

Petitioners claim that the contested sections impose limitations on the


The doctrine is that where the due process and equal protection clauses are
amount of input tax that may be claimed. In effect, a portion of the input
invoked, considering that they are not fixed rules but rather broad
tax that has already been paid cannot now be credited against the output
standards, there is a need for proof of such persuasive character as would
tax.
lead to such a conclusion. Absent such a showing, the presumption of

validity must prevail.[68]  


 
Petitioners' argument is not absolute. It assumes that the input tax

exceeds 70% of the output tax, and therefore, the input tax in excess of
Therefore, petitioners' argument must be rejected.
70% remains uncredited. However, to the extent that the input tax is less
 
than 70% of the output tax, then 100% of such input tax is still creditable. 

  On the other hand, it appears that petitioner Garcia failed to comprehend

the operation of the 70% limitation on the input tax. According to


More importantly, the excess input tax, if any, is retained in a business's
petitioner, the limitation on the creditable input tax in effect allows VAT-
books of accounts and remains creditable in the succeeding quarter/s. This
registered establishments to retain a portion of the taxes they collect,
is explicitly allowed by Section 110(B), which provides that 'if the input tax
which violates the principle that tax collection and revenue should be for
exceeds the output tax, the excess shall be carried over to the succeeding
public purposes and expenditures
quarter or quarters. In addition, Section 112(B) allows a VAT-registered
 
person to apply for the issuance of a tax credit certificate or refund for any

unused input taxes, to the extent that such input taxes have not been
As earlier stated, the input tax is the tax paid by a person, passed on to
applied against the output taxes. Such unused input tax may be used in
him by the seller, when he buys goods. Output tax meanwhile is the tax
payment of his other internal revenue taxes.
due to the person when he sells goods. In computing the VAT payable,

  three possible scenarios may arise:

 
The non-application of the unutilized input tax in a given quarter is not ad

infinitum, as petitioners exaggeratedly contend. Their analysis of the effect


First, if at the end of a taxable quarter the output taxes charged by the
of the 70% limitation is incomplete and one-sided. It ends at the net effect
seller are equal to the input taxes that he paid and passed on by the
that there will be unapplied/unutilized inputs VAT for a given quarter. It
suppliers, then no payment is required; 
does not proceed further to the fact that such unapplied/unutilized input
 
tax may be credited in the subsequent periods as allowed by the carry-over

provision of Section 110(B) or that it may later on be refunded through a

tax credit certificate under Section 112(B). 


Second, when the output taxes exceed the input taxes, the person shall be Petitioners Association of Pilipinas Shell Dealers, Inc., et al. also argue that

liable for the excess, which has to be paid to the Bureau of Internal the input tax partakes the nature of a property that may not be

Revenue (BIR);[69] and   confiscated, appropriated, or limited without due process of law. 

   

Third, if the input taxes exceed the output taxes, the excess shall be The input tax is not a property or a property right within the constitutional

carried over to the succeeding quarter or quarters.  Should the input taxes purview of the due process clause. A VAT-registered person's entitlement

result from zero-rated or effectively zero-rated transactions, any excess to the creditable input tax is a mere statutory privilege. 

over the output taxes shall instead be refunded to the taxpayer or credited
 
against other internal revenue taxes, at the taxpayer's option. [70]

The distinction between statutory privileges and vested rights must be


 
borne in mind for persons have no vested rights in statutory privileges. The

Section 8 of R.A. No. 9337 however, imposed a 70% limitation on the input state may change or take away rights, which were created by the law of

tax. Thus, a person can credit his input tax only up to the extent of 70% of the state, although it may not take away property, which was vested by

the output tax. In layman's term, the value-added taxes that a virtue of such rights. [72]

person/taxpayer paid and passed on to him by a seller can only be credited


 
up to 70% of the value-added taxes that is due to him on a taxable

transaction. There is no retention of any tax collection because the Under the previous system of single-stage taxation, taxes paid at every

person/taxpayer has already previously paid the input tax to a seller, and level of distribution are not recoverable from the taxes payable, although it

the seller will subsequently remit such input tax to the BIR. The party becomes part of the cost, which is deductible from the gross revenue.

directly liable for the payment of the tax is the seller. [71] What only needs When Pres. Aquino issued E.O. No. 273 imposing a 10% multi-stage tax on

to be done is for the person/taxpayer to apply or credit these input taxes, all sales, it was then that the crediting of the input tax paid on purchase or

as evidenced by receipts, against his output taxes.  importation of goods and services by VAT-registered persons against the

output tax was introduced. [73]This was adopted by the Expanded VAT


 
Law (R.A. No. 7716), [74] and The Tax Reform Act of 1997 (R.A. No.
8424). [75] The right to credit input tax as against the output tax is clearly acquisition cost of P1 Million pesos, exclusive of the VAT component. Such

a privilege created by law, a privilege that also the law can remove, or in spread out only poses a delay in the crediting of the input tax. Petitioners'

this case, limit.  argument is without basis because the taxpayer is not permanently

deprived of his privilege to credit the input tax. 


 

 
Petitioners also contest as arbitrary, oppressive, excessive and

confiscatory, Section 8 of R.A. No. 9337, amending Section 110(A) of the It is worth mentioning that Congress admitted that the spread-out of the

NIRC, which provides: creditable input tax in this case amounts to a 4-year interest-free loan to

the government. [76] In the same breath, Congress also justified its move
 
by saying that the provision was designed to raise an annual revenue of
SEC. 110. Tax Credits. ' 
  22.6 billion. [77]The legislature also dispelled the fear that the provision
(A) Creditable Input Tax. '
will fend off foreign investments, saying that foreign investors have other
 
Provided, That the input tax on goods purchased or tax incentives provided by law, and citing the case of China, where despite
imported in a calendar month for use in trade or business
for which deduction for depreciation is allowed under this a 17.5% non-creditable VAT, foreign investments were not
Code, shall be spread evenly over the month of
acquisition and the fifty-nine (59) succeeding months if deterred. [78] Again, for whatever is the purpose of the 60-month
the aggregate acquisition cost for such goods, excluding
the VAT component thereof, exceeds One million pesos amortization, this involves executive economic policy and legislative
(P1,000,000.00): Provided, however, That if the
estimated useful life of the capital goods is less than five wisdom in which the Court cannot intervene. 
(5) years, as used for depreciation purposes, then the
input VAT shall be spread over such a shorter  
period: Provided, finally, That in the case of purchase of
services, lease or use of properties, the input tax shall be
creditable to the purchaser, lessee or license upon
With regard to the 5% creditable withholding tax imposed on payments
payment of the compensation, rental, royalty or fee.
made by the government for taxable transactions, Section 12 of R.A. No.
 
9337, which amended Section 114 of the NIRC, reads:

 
 
SEC. 114. Return and Payment of Value-added
Tax. ' 
The foregoing section imposes a 60-month period within which to amortize
 
the creditable input tax on purchase or importation of capital goods with
(C) Withholding of Value-added Tax. ' The Government or
any of its political subdivisions, instrumentalities or present Section 114(C), these different rates, except for the 10% on lease
agencies, including government-owned or controlled
corporations (GOCCs) shall, before making payment on or property rights payment to nonresidents, were deleted, and a uniform
account of each purchase of goods and services which
rate of 5% is applied. 
are subject to the value-added tax imposed in Sections
106 and 108 of this Code, deduct and withhold a final
value-added tax at the rate of five percent (5%) of the  
gross payment thereof: Provided, That the payment for
lease or use of properties or property rights to
nonresident owners shall be subject to ten percent (10%) The Court observes, however, that the law the used the word final. In tax
withholding tax at the time of payment. For purposes of
this Section, the payor or person in control of the usage, final, as opposed to creditable, means full. Thus, it is provided in
payment shall be considered as the withholding agent.
  Section 114(C): final value-added tax at the rate of five percent (5%). 
The value-added tax withheld under this Section shall be
remitted within ten (10) days following the end of the  
month the withholding was made.

  In Revenue Regulations No. 02-98, implementing R.A. No. 8424 (The Tax

  Reform Act of 1997), the concept of final withholding tax on income was

explained, to wit:

Section 114(C) merely provides a method of collection, or as stated by


 
respondents, a more simplified VAT withholding system. The government in
SECTION 2.57. Withholding of Tax at Source
this case is constituted as a withholding agent with respect to their  
(A) Final Withholding Tax. ' Under the final withholding
payments for goods and services. 
tax system the amount of income tax withheld by the
withholding agent is constituted as full and final
  payment of the income tax due from the payee on the
said income. The liability for payment of the tax rests
primarily on the payor as a withholding agent. Thus, in
Prior to its amendment, Section 114(C) provided for different rates of case of his failure to withhold the tax or in case of
underwithholding, the deficiency tax shall be collected
value-added taxes to be withheld -- 3% on gross payments for purchases from the payor/withholding agent. '
 
of goods; 6% on gross payments for services supplied by contractors other (B) Creditable Withholding Tax. ' Under the creditable
withholding tax system, taxes withheld on certain income
than by public works contractors; 8.5% on gross payments for services payments are intended to equal or at least approximate
the tax due of the payee on said income. ' Taxes withheld
supplied by public work contractors; or 10% on payment for the lease or
on income payments covered by the expanded
withholding tax (referred to in Sec. 2.57.2 of these
use of properties or property rights to nonresident owners. Under the
regulations) and compensation income (referred to in
Sec. 2.78 also of these regulations) are creditable in shall be creditable against the value-added tax
nature. liability of the seller or contractor : Provided,
  however, That in the case of government public works
  contractors, the withholding rate shall be eight and one-
half percent (8.5%): Provided, further, That the payment
for lease or use of properties or property rights to
As applied to value-added tax, this means that taxable transactions with nonresident owners shall be subject to ten percent (10%)
withholding tax at the time of payment. For this purpose,
the government are subject to a 5% rate, which constitutes as full the payor or person in control of the payment shall be
considered as the withholding agent.
payment of the tax payable on the transaction. This represents the net VAT  

payable of the seller. The other 5% effectively accounts for the standard
The valued-added tax withheld under this Section shall be remitted within
input VAT (deemed input VAT), in lieu of the actual input VAT directly or ten (10) days following the end of the month the withholding was
made. (Emphasis supplied)
attributable to the taxable transaction. [79]
 
 
 

The Court need not explore the rationale behind the provision. It is clear
As amended, the use of the word final and the deletion of the
that Congress intended to treat differently taxable transactions with the
word creditable exhibits Congress's intention to treat transactions with the
government.[80] This is supported by the fact that under the old provision,
government differently.  Since it has not been shown that the class subject
the 5% tax withheld by the government remains creditable against the tax
to the 5% final withholding tax has been unreasonably narrowed, there is
liability of the seller or contractor, to wit:
no reason to invalidate the provision. Petitioners, as petroleum dealers, are
 
not the only ones subjected to the 5% final withholding tax. It applies to all

SEC. 114. Return and Payment of Value-added Tax. '  those who deal with the government. 
 
(C) Withholding of Creditable Value-added Tax. ' The
Government or any of its political subdivisions,  
instrumentalities or agencies, including government-
owned or controlled corporations (GOCCs) shall, before
making payment on account of each purchase of goods Moreover, the actual input tax is not totally lost or uncreditable, as
from sellers and services rendered by contractors which
are subject to the value-added tax imposed in Sections petitioners believe. Revenue Regulations No. 14-2005 or the Consolidated
106 and 108 of this Code, deduct and withhold the value-
Value-Added Tax Regulations 2005 issued by the BIR, provides that should
added tax due at the rate of three percent (3%) of the
gross payment for the purchase of goods and six percent
the actual input tax exceed 5% of gross payments, the excess may form
(6%) on gross receipts for services rendered by
contractors on every sale or installment payment which
part of the cost. Equally, should the actual input tax be less than 5%, the is enjoyed by other persons or other classes in the same place and in like

difference is treated as income.[81] circumstances. [83]

   

Petitioners also argue that by imposing a limitation on the creditable input The power of the State to make reasonable and natural classifications for

tax, the government gets to tax a profit or value-added even if there is no the purposes of taxation has long been established. Whether it relates to

profit or value-added. the subject of taxation, the kind of property, the rates to be levied, or the

amounts to be raised, the methods of assessment, valuation and collection,


 
the State's power is entitled to presumption of validity. As a rule, the

Petitioners' stance is purely hypothetical, argumentative, and again, one- judiciary will not interfere with such power absent a clear showing of

sided. The Court will not engage in a legal joust where premises are what unreasonableness, discrimination, or arbitrariness. [84]

ifs, arguments, theoretical and facts, uncertain. Any disquisition by the  


Court on this point will only be, as Shakespeare describes life

in Macbeth, [82] full of sound and fury, signifying nothing.  Petitioners point out that the limitation on the creditable input tax if the

entity has a high ratio of input tax, or invests in capital equipment, or has
 
several transactions with the government, is not based on real and

What's more, petitioners' contention assumes the proposition that there is substantial differences to meet a valid classification.

no profit or value-added. It need not take an astute businessman to know  


that it is a matter of exception that a business will sell goods or services

without profit or value-added. It cannot be overstressed that a business is The argument is pedantic, if not outright baseless. The law does not make

created precisely for profit.  any classification in the subject of taxation, the kind of property, the rates

to be levied or the amounts to be raised, the methods of assessment,


 
valuation and collection. Petitioners' alleged distinctions are based on

The equal protection clause under the Constitution means that 'no person variables that bear different consequences. While the implementation of

or class of persons shall be deprived of the same protection of laws which the law may yield varying end results depending on one's profit margin and
The rule of taxation shall be uniform and equitable. The
value-added, the Court cannot go beyond what the legislature has laid Congress shall evolve a progressive system of taxation.

down and interfere with the affairs of business. 


 

The equal protection clause does not require the universal application of  

the laws on all persons or things without distinction. This might in fact
Uniformity in taxation means that all taxable articles or kinds of property of
sometimes result in unequal protection. What the clause requires is
the same class shall be taxed at the same rate. Different articles may be
equality among equals as determined according to a valid classification. By
taxed at different amounts provided that the rate is uniform on the same
classification is meant the grouping of persons or things similar to each
class everywhere with all people at all times. [86]
other in certain particulars and different from all others in these same

particulars. [85]  

 
In this case, the tax law is uniform as it provides a standard rate of 0% or
Petitioners brought to the Court's attention the introduction of Senate Bill 10% (or 12%) on all goods and services. Sections 4, 5 and 6 of R.A. No.
No. 2038 by Sens. S.R. Osmea III and Ma. Ana Consuelo A.S. ' Madrigal on
June 6, 2005, and House Bill No. 4493 by Rep. Eric D. Singson. The 9337, amending Sections 106, 107 and 108, respectively, of the NIRC,
proposed legislation seeks to amend the 70% limitation by increasing the
same to 90%. This, according to petitioners, supports their stance that the provide for a rate of 10% (or 12%) on sale of goods and properties,
70% limitation is arbitrary and confiscatory. On this score, suffice it to say
that these are still proposed legislations. Until Congress amends the law, importation of goods, and sale of services and use or lease of properties.
and absent any unequivocal basis for its unconstitutionality, the 70%
limitation stays.  These same sections also provide for a 0% rate on certain sales and

transaction. 
 
 
B. Uniformity and Equitability of
Taxation
Neither does the law make any distinction as to the type of industry or
 
trade that will bear the 70% limitation on the creditable input tax, 5-year
  amortization of input tax paid on purchase of capital goods or the 5% final

withholding tax by the government. It must be stressed that the rule of


Article VI, Section 28(1) of the Constitution reads:
uniform taxation does not deprive Congress of the power to classify

 
subjects of taxation, and only demands uniformity within the particular was not oblivious to this. Thus, to equalize the weighty burden the law

class. [87] entails, the law, under Section 116, imposed a 3% percentage tax on VAT-

exempt persons under Section 109(v), i.e., transactions with gross annual


 
sales and/or receipts not exceeding P1.5 Million. This acts as a equalizer

R.A. No. 9337 is also equitable. The law is equipped with a threshold because in effect, bigger businesses that qualify for VAT coverage and VAT-

margin. The VAT rate of 0% or 10% (or 12%) does not apply to sales of exempt taxpayers stand on equal-footing.

goods or services with gross annual sales or receipts not  


exceeding P1,500,000.00. [88]Also, basic marine and agricultural food

products in their original state are still not subject to the tax, [89] thus Moreover, Congress provided mitigating measures to cushion the impact of

ensuring that prices at the grassroots level will remain accessible. As was the imposition of the tax on those previously exempt. Excise taxes on

stated in Kapatiran ng mga Naglilingkod sa Pamahalaan ng Pilipinas, Inc. petroleum products [91] and natural gas [92] were reduced. Percentage

vs. Tan: [90] tax on domestic carriers was removed. [93] Power producers are now

exempt from paying franchise tax. [94]


 

 
The disputed sales tax is also equitable. It is imposed
only on sales of goods or services by persons engaged in
business with an aggregate gross annual sales
exceeding P200,000.00. Small corner sari-sari stores are Aside from these, Congress also increased the income tax rates of
consequently exempt from its application. Likewise
corporations, in order to distribute the burden of taxation. Domestic,
exempt from the tax are sales of farm and marine
products, so that the costs of basic food and other foreign, and non-resident corporations are now subject to a 35% income
necessities, spared as they are from the incidence of the
VAT, are expected to be relatively lower and within the tax rate, from a previous 32%. [95] Intercorporate dividends of non-
reach of the general public.
resident foreign corporations are still subject to 15% final withholding tax
 
but the tax credit allowed on the corporation's domicile was increased to

  20%. [96] The Philippine Amusement and Gaming Corporation (PAGCOR)

is not exempt from income taxes anymore.[97] Even the sale by an artist


It is admitted that R.A. No. 9337 puts a premium on businesses with low of his works or services performed for the production of such works was
profit margins, and unduly favors those with high profit margins. Congress not spared.
 
Taxation is progressive when its rate goes up depending on the resources

of the person affected. [98]


All these were designed to ease, as well as spread out, the burden of

taxation, which would otherwise rest largely on the consumers. It cannot  

therefore be gainsaid that R.A. No. 9337 is equitable.


The VAT is an antithesis of progressive taxation. By its very nature, it is
 
regressive. The principle of progressive taxation has no relation with the
C.                           Progressivity of Taxation VAT system inasmuch as the VAT paid by the consumer or business for

  every goods bought or services enjoyed is the same regardless of income.

In 
 

other words, the VAT paid eats the same portion of an income, whether big
Lastly, petitioners contend that the limitation on the creditable input tax is
or small. The disparity lies in the income earned by a person or profit
anything but regressive. It is the smaller business with higher input tax-
margin marked by a business, such that the higher the income or profit
output tax ratio that will suffer the consequences.
margin, the smaller the portion of the income or profit that is eaten by
  VAT. A converso, the lower the income or profit margin, the bigger the part

that the VAT eats away. At the end of the day, it is really the lower income
Progressive taxation is built on the principle of the taxpayer's ability to pay.
group or businesses with low-profit margins that is always hardest hit. 
This principle was also lifted from Adam Smith's Canons of Taxation, and it
 
states:

  Nevertheless, the Constitution does not really prohibit the imposition of

I. The subjects of every state ought to contribute towards indirect taxes, like the VAT. What it simply provides is that Congress shall
the support of the government, as nearly as
"evolve a progressive system of taxation." The Court stated in
possible, in proportion to their respective
abilities; that is, in proportion to the revenue
the Tolentino case, thus:
which they respectively enjoy under the
protection of the state.
 
The Constitution does not really prohibit the imposition of
indirect taxes which, like the VAT, are regressive. What it law seeks to remedy. As in other cases, the Court cannot strike down a law
simply provides is that Congress shall 'evolve a
progressive system of taxation. The constitutional as unconstitutional simply because of its yokes. 
provision has been interpreted to mean simply that
'direct taxes are . . . to be preferred [and] as much as  
possible, indirect taxes should be minimized. (E.
FERNANDO, THE CONSTITUTION OF THE PHILIPPINES
Let us not be overly influenced by the plea that for every
221 (Second ed. 1977)) Indeed, the mandate to
wrong there is a remedy, and that the judiciary should
Congress is not to prescribe, but to evolve, a progressive
stand ready to afford relief. There are undoubtedly many
tax system. Otherwise, sales taxes, which perhaps are
wrongs the judicature may not correct, for instance,
the oldest form of indirect taxes, would have been
those involving political questions. . . .
prohibited with the proclamation of Art. VIII, '17 (1) of
 
the 1973 Constitution from which the present Art. VI, '28
Let us likewise disabuse our minds from the notion that
(1) was taken. Sales taxes are also regressive.
the judiciary is the repository of remedies for all political
 
or social ills; We should not forget that the Constitution
Resort to indirect taxes should be minimized but not
has judiciously allocated the powers of government to
avoided entirely because it is difficult, if not impossible,
three distinct and separate compartments; and that
to avoid them by imposing such taxes according to the
judicial interpretation has tended to the preservation of
taxpayers' ability to pay. In the case of the VAT, the law
the independence of the three, and a zealous regard of
minimizes the regressive effects of this imposition by
the prerogatives of each, knowing full well that one is not
providing for zero rating of certain transactions (R.A. No.
the guardian of the others and that, for official wrong-
7716, '3, amending 102 (b) of the NIRC), while granting
doing, each may be brought to account, either by
exemptions to other transactions. (R.A. No. 7716, '4
impeachment, trial or by the ballot box. [100]
amending '103 of the NIRC) [99]

 
 

 
 

The words of the Court in Vera vs. Avelino [101] holds true then, as it still
CONCLUSION
holds true now. All things considered, there is no raison d'tre for the
 
unconstitutionality of R.A. No. 9337. 

It has been said that taxes are the lifeblood of the government. In this  

case, it is just an enema, a first-aid measure to resuscitate an economy in


WHEREFORE, Republic Act No. 9337 not being unconstitutional, the
distress. The Court is neither blind nor is it turning a deaf ear on the plight
petitions in G.R. Nos. 168056, 168207, 168461, 168463, and 168730, are
of the masses. But it does not have the panacea for the malady that the
hereby DISMISSED.
 

There being no constitutional impediment to the full enforcement and

implementation of R.A. No. 9337, the temporary restraining order issued

by the Court on July 1, 2005 is LIFTED upon finality of herein decision.

SO ORDERED.

You might also like